Gate TB

You might also like

Download as pdf or txt
Download as pdf or txt
You are on page 1of 693

About Pearson

Pearson is the world’s learning company, with presence across 70 countries


worldwide. Our unique insights and world-class expertise comes from a long
history of working closely with renowned teachers, authors and thought
leaders, as a result of which, we have emerged as the preferred choice for
millions of teachers and learners across the world.
We believe learning opens up opportunities, creates fulfilling careers and
hence better lives. We hence collaborate with the best of minds to deliver you
class-leading products, spread across the Higher Education and K12 spectrum.
Superior learning experience and improved outcomes are at the heart of
everything we do. This product is the result of one such effort.
Your feedback plays a critical role in the evolution of our products and you
can contact us - reachus@pearson.com. We look forward to it.
Also Helpful for GAIL, BARC, HPCL, BHEL, ONGC, SAIL, DRDO & Other PSU’s

G R AD UAT E A PT I T UD E T E S T I N E NG I N EE R I N G

27
TOPIC-WISE
PREVIOUS
YEARS’ SOLVED YEARS
PAPERS

ELECTRONICS AND
COMMUNICATION
ENGINEERING
2020
Sr. Editor—Acquisitions: Sharel Simon
Editor—Development: Aishwarya Gupta
Sr. Editor—Production: Vipin Kumar

The aim of this publication is to supply information taken from sources believed to be valid and reliable. This is not an
attempt to render any type of professional advice or analysis, nor is it to be treated as such. While much care has been
taken to ensure the veracity and currency of the information presented within, neither the publisher nor its authors bear any
responsibility for any damage arising from inadvertent omissions, negligence or inaccuracies (typographical or factual)
that may have found their way into this book.

Copyright © 2019 Pearson India Education Services Pvt. Ltd

This book is sold subject to the condition that it shall not, by way of trade or otherwise, be lent, resold, hired out, or other-
wise circulated without the publisher’s prior written consent in any form of binding or cover other than that in which it is
published and without a similar condition including this condition being imposed on the subsequent purchaser and without
limiting the rights under copyright reserved above, no part of this publication may be reproduced, stored in or introduced
into a retrieval system, or transmitted in any form or by any means (electronic, mechanical, photocopying, recording or
otherwise), without the prior written permission of both the copyright owner and the publisher of this book.

ISBN 978-93-534-3779-4

First Impression

Published by Pearson India Education Services Pvt. Ltd, CIN: U72200TN2005PTC057128.

Head Office: A-8(A), 7th Floor, Knowledge Boulevard, Sector 62, Noida 201 309, Uttar Pradesh, India.
Registered Office: The HIVE, 3rd Floor, Metro Zone, No.44, Pillayar Koil Street, Jawaharlal Nehru Road, Anna Nagar,
Chennai 600 040, Tamil Nadu, India.
Phone: 044-66540100
Website: in.pearson.com, Email: companysecretary.india@pearson.com

Compositor: D Book Media


Printed in India
Dedication
This book is dedicated to almighty GOD, my parents, wife Hemani, daughter Aarchishya, and son Ruchirangad. It was not
possible for me to take up this task without their support.
Contents
Preface     ix

Reviewer     x

About the Author     xi

Syllabus: Electronics and Communication Engineering     xii

Important Tips for GATE Preparation     xiv

Unit 1: Network Theory 1.1


Chapter 1:  Basics of Network Analysis 1.3
Chapter 2:  Sinusoidal Steady State 1.24
Chapter 3:  Network Theorems 1.39
Chapter 4:  Transient Analysis  1.49
Chapter 5:  Two Port Networks 1.64
Chapter 6:  Graph Theory and State Equations 1.80
Chapter 7:  Network Functions  1.84

Unit 2:  Signals and Systems 2.1


Chapter 1:  Basics of Signals and Systems 2.3
Chapter 2: LTI Systems Continues and Discrete 2.11
Chapter 3: Fourier Series 2.20
Chapter 4: Fourier Transforms 2.25
Chapter 5: Laplace Transform 2.41
Chapter 6:  Z-Transform 2.52
Chapter 7: DTFT and DFT 2.65
Chapter 8: Sampling 2.71

Unit 3: Control Systems 3.1


Chapter 1:  Basics 3.3
Chapter 2:  Block Diagram and SFG 3.9
Chapter 3: Compensators and Controllers 3.18
Chapter 4: Time Response Analysis 3.25
Chapter 5: Stability Analysis 3.44
Chapter 6: Root Locus 3.55
Chapter 7: Frequency Analysis 3.62
Chapter 8: State Space Analysis 3.77

Unit 4: Electronic Devices and Circuits 4.1


Chapter 1:  Basic 4.3
Chapter 2:  PN Junction 4.22
Chapter 3:  Bipolar Junction Transistor 4.35
viii | Contents

Unit 5: Analog Electronics 5.1


Chapter 1: Operational Amplifiers 5.3
Chapter 2: Diodes Applications 5.34
Chapter 3:  BJT Analysis 5.48
Chapter 4: FET and Mosfet Analysis 5.79
Chapter 5: Frequency Response of Amplifier 5.100
Chapter 6: Feedback Amplifiers 5.104
Chapter 7: Oscillator Circuits 5.109
Chapter 8:  Power Amplifiers 5.113
Chapter 9: Multi-vibrators and 555 Timers 5.115

Unit 6: Digital Circuits 6.1


Chapter 1:  Number System 6.3
Chapter 2:  Boolean Algebr 6.6
Chapter 3: Logic Gates 6.13
Chapter 4: Combinational Circuits 6.26
Chapter 5: Sequential Circuits 6.42
Chapter 6: Logic Families 6.64
Chapter 7: Memories 6.73
Chapter 8: ADC and DAC 6.78

Unit 7: Microprocessors 7.1


Chapter 1:  Basics of 8085 7.3
Chapter 2: Instructions of 8085 Microprocessor 7.5
Chapter 3: Memory Interfacing 7.9
Chapter 4: Microprocessor 8085 Interfacing 7.12
Chapter 5: Microprocessor 8085 Interrupts 7.14
Chapter 6: Microprocessor 8085 Programming 7.15

Unit 8: Communication 8.1


Chapter 1: Analog Communication Systems 8.3
Chapter 2: Random Signals and Noise 8.25
Chapter 3: Digital Communication Systems 8.46
Chapter 4: Information Theory 8.67

Unit 9: Electromagnetic Theory 9.1


Chapter 1:  Basics 9.3
Chapter 2: Uniform Plane Waves 9.20
Chapter 3: Transmission Lines 9.39
Chapter 4: Waveguide 9.56
Chapter 5: Antenna 9.67
Preface
Graduate Aptitude Test in Engineering (GATE) is an all India examination that primarily tests the comprehensive under-
standing of the candidate in various undergraduate subjects in Engineering/Technology/Architecture and postgraduate
level subjects in Science. Owing to multifaceted opportunities open to any good performer, the number of aspirants appear-
ing for the GATE examination is increasing significantly every year. Apart from giving the aspirant a chance to pursue an
M.Tech. from institutions such as the IITs /NITs, a good GATE score can be highly instrumental in landing the candidate
a plush public sector job, as many PSUs are recruiting graduate engineers on the basis of their performance in GATE. The
GATE examination pattern has undergone several changes over the years—sometimes apparent and sometimes subtle. It is
bound to continue to do so with changing technological environment.
We are pleased to present before the readers GATE Topic-wise Previous Years’ Solved Question Papers for Electronics
and Communication Engineering which is a one-stop solution for GATE aspirants to crack the GATE exam. The book
includes around 27 years GATE questions segregated topic-wise along with exam analysis which is provided at the begin-
ning of every unit. This in turn will help the aspirants to get an idea about the pattern and weightage of questions asked in
the examination. The salient features of the book are given below.

Salient Features
• Includes around 27 years GATE questions arranged topic-wise.
• Gate 2019 paper with topic-wise Analysis.
• Detailed solutions for better understanding.
• Topic-wise detailed analysis of previous year questions provided for each section.
I acknowledge the help provided in compilation and by giving technical inputs by Pearson Editorial Team and many others.
Despite of our best efforts, some errors may have inadvertently crept into the book. Constructive comments and sugges-
tions to further improve the book are welcome and shall be acknowledged gratefully.

Rajiv Kapoor
Reviewers
We would like to thank the below mentioned reviewers for their continuous feedback and suggestions which has helped in
shaping this book.

Jyotsna Singh Associate Professor, Netaji Subhas Institute of Technology, Dwarka, New Delhi
S. K. Mydhili  ssistant Professor (Senior Grade), SVS College of Engineering, Coimbatore,
A
Tamil Nadu
Arun Khosla Associate Professor, National Institute of Technology, Jalandhar, Punjab
Manav Bhatnagar Associate Professor, Indian Institute of Technology, New Delhi
B.V.R. Reddy Professor and Dean, USET, Guru Gobind Singh Indraprastha University, Dwarka, Delhi
About The Author
Rajiv Kapoor, Ph.D. in Electronics and Communication Engineering from Panjab University (Punjab Engineering College),
Chandigarh and has worked in industries of repute and also worked in Engineering institutions of repute and having a
total experience of 23 years in teaching Engineering students. He is presently working as professor in Electronics and
Communication Engineering Department at Delhi Technological University (Formerly Delhi College of Engineering). He
published more than 100 papers in the various Journals/Conferences of repute and has filed 6 patents also. He has under-
taken 21 R&D projects and is not only a strong believer but a practitioner for ‘MAKE in INDIA’. He has an endeavour to
design new products for Industries and work for society with in-depth knowledge of the subject, he has honed his skills to
foresee the obstacles of students in their quest to prepare them for these examinations (GATE, NET, etc.) and successfully
weeds them out.
Syllabus: Electronics and
Communication Engineering
Networks, Signals and Systems
Network Solution Methods: Nodal and mesh analysis; Network theorems: superposition, Thevenin and Norton’s, maxi-
mum power transfer; Wye-Delta transformation; Steady state sinusoidal analysis using phasors; Time domain analysis of
simple linear circuits; Solution of network equations using Laplace transform; Frequency domain analysis of RLC circuits;
Linear 2-port network parameters: driving point and transfer functions; State equations for networks.
Continuous-time Signals: Fourier series and Fourier transform representations, sampling theorem and applications;
Discrete-time signals: discrete-time Fourier transform (DTFT), DFT, FFT, Z-transform, interpolation of discrete-time sig-
nals; LTI systems: definition and properties, causality, stability, impulse response, convolution, poles and zeros, parallel
and cascade structure, frequency response, group delay, phase delay, digital filter design techniques.

Control Systems
Basic control system components; Feedback principle; Transfer function; Block diagram representation; Signal flow graph;
Transient and steady-state analysis of LTI systems; Frequency response; Routh-Hurwitz and Nyquist stability criteria;
Bode and root-locus plots; Lag, lead and lag-lead compensation; State variable model and solution of state equation of LTI
systems.

Electronic Devices
Energy bands in intrinsic and extrinsic silicon; Carrier transport: diffusion current, drift current, mobility and resistivity;
Generation and recombination of carriers; Poisson and continuity equations; P-N junction, Zener diode, BJT, MOS capaci-
tor, MOSFET, LED, photo diode and solar cell; Integrated circuit fabrication process: oxidation, diffusion, ion implanta-
tion, photolithography and twin-tub CMOS process.

Analog Circuits
Small signal equivalent circuits of diodes, BJTs and MOSFETs; Simple diode circuits: clipping, clamping and rectifiers;
Single-stage BJT and MOSFET amplifiers: biasing, bias stability, mid-frequency small signal analysis and frequency
response; BJT and MOSFET amplifiers: multi-stage, differential, feedback, power and operational; Simple op-amp cir-
cuits; Active filters; Sinusoidal oscillators: criterion for oscillation, single-transistor and opamp configurations; Function
generators, wave-shaping circuits and 555 timers; Voltage reference circuits; Power supplies: ripple removal and regulation.

Digital Circuits
Number systems; Combinatorial circuits: Boolean algebra, minimization of functions using Boolean identities and
Karnaugh map, logic gates and their static CMOS implementations, arithmetic circuits, code converters, multiplexers,
decoders and PLAs; Sequential circuits: latches and flip-flops, counters, shift-registers and finite state machines; Data
converters: sample and hold circuits, ADCs and DACs; Semiconductor memories: ROM, SRAM, DRAM; 8-bit micropro-
cessor (8085): architecture, programming, memory and I/O interfacing.

Communications
Random processes: Autocorrelation and power spectral density, properties of white noise, filtering of random signals
through LTI systems; Analog communications: amplitude modulation and demodulation, angle modulation and demodula-
tion, spectra of AM and FM, superheterodyne receivers, circuits for analog communications; Information theory: entropy,
mutual information and channel capacity theorem; Digital communications: PCM, DPCM, digital modulation schemes,
Syllabus: Electronics and Communication Engineering  |  xiii

amplitude, phase and frequency shift keying (ASK, PSK, FSK), QAM, MAP and ML decoding, matched filter receiver,
calculation of bandwidth, SNR and BER for digital modulation; Fundamentals of error correction, Hamming codes;
Timing and frequency synchronization, inter-symbol interference and its mitigation; Basics of TDMA, FDMA and CDMA.

Electromagnetics
Electrostatics; Maxwell’s equations: differential and integral forms and their interpretation, boundary conditions, wave
equation, Poynting vector; Plane waves and properties: reflection and refraction, polarization, phase and group veloc-
ity, propagation through various media, skin depth; Transmission lines: equations, characteristic impedance, impedance
matching, impedance transformation, S-parameters, Smith chart; Waveguides: modes, boundary conditions, cut-off fre-
quencies, dispersion relations; Antennas: antenna types, radiation pattern, gain and directivity, return loss, antenna arrays;
Basics of radar; Light propagation in optical fibers.
Important Tips for GATE Preparation
The followings are some important tips which would be helpful for students to prepare for GATE examination.

1. Go through the pattern (using previous years’ GATE paper) and syllabus of the exam and start preparing accordingly.
2. Preparation time for GATE depends on many factors, such as, individual’s aptitude, attitude, fundamentals,
concentration level etc., Generally rigorous preparation for four to six months is considered good but it may vary
from student to student.
3. Make a list of books which cover complete syllabus, contains solved previous year questions and mock tests for
practice based on latest GATE pattern. Purchase these books and start your preparation.
4. Make a list of topics which needs to be studied and make priority list for study of every topic based upon the marks
for which that particular topic is asked in GATE examination. Find out the topics which fetch more marks and give
more importance to those topics. Make a timetable for study of topics and follow the timetable strictly.
5. An effective way to brush up your knowledge about technical topics is group study with your friends. During group
study you can explore new techniques and procedures.
6. While preparing any subject highlight important points (key definitions, equations, derivations, theorems and laws)
which can be revised during last minute preparation.
7. Pay equal attention to both theory and numerical problems. Solve questions (numerical) based on latest exam pattern
as much as possible, keeping weightage of that topic in mind. Whatever topics you decide to study, make sure that
you know everything about it.
8. Try to use short-cut methods to solve problems instead of traditional lengthy and time consuming methods.
9. Go through previous years’ papers (say last ten years), to check your knowledge and note the distribution of different
topics. Also analyze the topics in which you are weak and concentrate more on those topics. Always try to solve
papers in given time, to obtain an idea how many questions you are able to solve in the given time limit.
10. Finish the detail study of topics one and a half month before your exam. During last month revise all the topics once
again and clear leftover doubts.
Unit I
Network Theory

Chapter 1: Basics of Network Analysis 1.3


Chapter 2: Sinusoidal Steady State 1.24
Chapter 3: Network Theorems 1.39
Chapter 4: Transient Analysis 1.49
Chapter 5: Two Port Networks 1.64
Chapter 6: Graph Theory and State Equations 1.80
Chapter 7: Network Functions 1.84
EXAM ANALYSIS
Exam Year 92 93 94 95 96 97 98 99 00 01 02 03 04 05 06 07 08 09 10 11 12 13 14 15 16 17 18 19
Set 1 Set 2 Set 3 Set 1 Set 2 Set 3 Set 1 Set 2
1 Marks Ques. - 2 4 7 3 4 6 2 2 4 2 4 5 5 6 2 2 3 3 3 4 3 2 3 3 1 1 4 1 1 2 2 1
2 Marks Ques. 5 4 1 - 1 2 - 3 2 4 2 7 5 6 - 4 7 4 3 3 4 6 4 3 4 4 3 4 3 2 4 4 3
3 Marks Ques. - - - - 1 1 - - - - - - - - - - - - - - - - - - - - - - - - - - -
5 Marks Ques. - 3 4 2 3 2 1 2 4 3 - - - - - - - - - - - - - - - - - - - - - - -
Total Marks 10 25 26 17 23 21 11 18 26 27 21 18 15 17 6 10 16 11 9 9 12 15 10 9 11 9 7 12 7 5 10 10 7

Basics of Network 1 2 - 2 - 4 2 1 2 3 2 3 5 2 - - - 2 1 1 3 5 - 5 2 3 - -
Analysis
Sinusoidal Steady 1 2 1 4 2 - 1 - - 1 - 2 5 3 - 1 - 1 2 1 - 1 - 2 3 3 1 1
State
Network Theorems - 2 1 1 - - 1 2 1 1 1 1 - 2 - 2 1 1 - 2 2 2 - 4 1 1 - 1
Transient Analysis 1 - 1 - 1 1 - - - - 1 2 2 1 2 1 4 2 1 1 1 - - 3 2 - 1 -
Two Port Networks 1 - 1 - - 1 1 1 - 2 - 1 2 2 2 - 2 - 2 1 2 - - 2 3 - 1 1
Graph Theory and 1 - - - 1 - 1 1 - - 1 1 - - - 1 - - - - - 1 2 1 1 1
State Equations
Network Functions - - 1 - - - - - 1 - - 1 - 1 2 2 1 1 - - - 1 - - 2 1 - -
Chapter 1
Basics of Network Analysis
2. A connection is made consisting of resistance A in
One-mark questions series with a parallel combination of resistances B and
C. Three resistors of value 10 W, 5 W, 2 W are provided.
1. In the circuit shown in the figure, the positive angular Consider all possible permutations of the given resis-
frequency w (in radians per second) at which the mag- tors in to the positions A, B, C, and identify the con-
nitude of the phase difference between the voltages V1 figurations with maximum possible overall resistance,
π and also the ones with minimum possible overall resist-
and V2 equals ⋅radians, is ____________. [2017]
4 ance. The ratio of maximum to minimum values of the
resistances (up to second decimal place) is _________.
V2
 [2017]
1Ω 1H
3. The damping ratio of a series RLC circuit can be
expressed as [2015]
100 cos ω t
∼ 1Ω V1 RC
(A) (B)
2
2 L
2L R 2C
R C 2 L
(C) (D)
2 L R C
Solution:  From the given data
Let Z1 = 1 W = 1 ∠ 0° W R sL

Z2 = 1 + jw = 1 + ω ∠ θ2 Ω
2

−1 + +
Where θ 2 = tan ω V VC 1/sC
I
– –
V1 = Z1 . i(t )
V2 = Z 2 . i(t )

Let i(t) = I m ∠q Amp. V


Solution:  I =
1
V1 = I m ∠ q × 1 R + sL +
sC
V2 = Im ∠q × 1 + ω 2 ∠q2 I
VC =
V2 = I m 1 + ω 2 ∠ q + q2 sC 
⎧ ⎫
π ⎪ V ⎪ 1
Given θ + θ 2 − θ = ∴ VC = ⎨
4 ⎬×
⎪ R + sL + 1 ⎪ sC
π ⎩ sC ⎭ 
∴θ 2 =
4
VC 1
ω  = H (s) = 2
−1
Tan   = θ 2
V s LC + sRC + 1 
1
1 LC
∴ ω = 1 rad/ sec H (s) =
⎛ R⎞
Hence, the correct answer is (0.9 to 1.1). s2 + s ⎜ ⎟ + 1
⎝ L⎠ LC

1.4 | Network Theory

From the above T/F compare to 2nd order system R 2C


= 1 − ω 02 LC
1 L 
ωn = ωo =
LC  R 2
C
R ω o2 LC = 1 −
2ζωn = L 
L
1 R 2C
R R C 1 ωo = . 1−
ζ= × LC = =
LC L 
2L 2 L 2Q 
Hence, the correct option is (C). 1R 2C
fo = . 1−
4. An LC tank circuit consists of an ideal capacitor C con- 2π LC L 
nected in parallel with a coil of inductance L having
Hence, the correct option is (B).
an internal resistance R. The resonant frequency of the
tank circuit is [2015] 5. Consider the configuration shown in the figure which is
a portion of a larger electrical network
1 1 C
(A) (B)  1 − R2
2π LC 2π LC L i5
i2
1 1 1 ⎛ 2 C⎞
(C) 2
(D)  1− ⎜⎝1 − R ⎟⎠
2π LC RC 2π LC L
Solution:  From the given data the equivalent tank R R i3
­circuit is i4 R
a

sL i1 i6
1/sC
R
For R = 1 Ω and currents i1 = 2 A, i4 = -1 A, i5 = -4 A,
which one of the following is TRUE?
b (a) i6 = 5 A
(b) i3 = -4 A
Z(s)
(c) Data is sufficient to conclude that the supposed
1 currents are impossible
Z(s) = {R + sL} || (d) Data is insufficient to identify the currents i2, i3 and
sC 
1 i6 [2014]
( R + sL ) sC Solution: (a)
=
1
R + sL + i5
sC  A
R + sL
Z(s) = 2
s LC + RCs + 1  R R
i3
R + jω L R + jω L
Z ( jω ) = −ω 2 LC + jω RC + 1 = 1 − ω 2 LC + jω RC
i2
( ) i4
B C

( R + jω L ) {(1 − ω 2 LC ) − jω RC }
i1 i6
R
Z ( jω ) =
(1 − ω LC )
2
+ (ω RC )
2 2
At A, i5 = i3 – i2

at resonance At B, −i4 = i1 – i2
Imag{Z(jω)} = 0 At C, i6 = i1 – i3
i6 + (i2 + i5) – i1 = 0
Imag(z} = 0
i6 + i1 + i4 + i5 – i1 = 0
∴ –j ωR2C + j ωL (1 – ω2LC) = 0 i6 + (2 – 1 – 4) – 2 = 0
At resonance ω = ω0 i6 = 5A
R2C = L [1 – ω 02 LC } Hence, the correct option is (a).
Chapter 1  Basics of Network Analysis  |  1.5

6. The circuit shown in the figure represents a the delta connection are scaled by a factor k, k > 0, the
elements of the corresponding star equivalent will be
I1 scaled by a factor of

AI1 R
Ra Rb Ra
Rb Rc RA

(a) voltage controlled voltage source


(b) voltage controlled current source
(c) current controlled current source (a) k2 (b) k
(d) current controlled voltage source [2014] (c) 1/k (d) [2013]
k
Solution: (c)
Solution: (b)
Output current = function (input current) = CCCS
Hence, the correct option is (c).
Ra RC RB
7. The magnitude of current (in mA) through the resistor
R2 in the figure shown is______ [2014] Rb RA

R2

1kΩ
Appling star –delta conversion for the given circuit
10 mA R1 2kΩ R3 4kΩ 2 mA
R1 Rb Rc K 2 ( Rb Rc )
RA′ = = = KRA
Rb + Rc + Ra K [ Rb + Rc + Ra ]
3kΩ
RA = KRA
Solution: 2.8 mA Hence, the correct option is (b).
R 2 = 1k Ω 9. In the circuit shown below, the current through the
R 2 = 1k Ω inductor is

10 mA R 1 = 2k Ω R 3 = 4k Ω 2 mA

10 mA R 1 = 2k Ω R 3 = 4k Ω 2 mA i1 Ω
1 Ω

1 0A
KVL: 1kΩ 1 0V 1 0V
1kΩ
- + + -
2kΩ 4kΩ 1 0A
2kΩ 4kΩ
20 V
8V –1 Ω 1 Ω
20 V
8V

3k
3k
(a) 2 A (b) −1
A
20 = 2I + I + 4I – 8 + 3I 1+ j 1 +j
I = 2.8 mA
(c) 1 (d) 0A [2012]
8. Consider a delta connection of resistors and its equiva- A
lent star connection as shown below. If all elements of 1+ j
1.6 | Network Theory

Solution: (c) (a) 10 A (b) 13 A


(c) 15 A (d) 18 A [2009]
D
Solution: (a)

i1 Ω + -
1 Ω
1A 20 V
+
I 60 V
-
1 0 1 0 12 A
i
- + A + -
A
1A
Since 60 V is absorbing power, it means the current is
–1 Ω 1 Ω
entering positive terminal. Applying KCL at A,
I + i = 12
I = 12 – i value of i < 12
So, out of given options, only (a) can be the answer.
Applying KCL at node A, there will be no current in
voltage sources. Hence, the correct option is (a).
Applying KVL in upper loop, 12. A fully charged mobile phone with a 12 V battery is
good for a 10 minute talk-time. Assume that, during
1 + i + (i + 1) j1 + 1∠0° − 1∠0° = 0
the talk-time the battery delivers a constant current of
−j 2 A and its voltage drops linearly from 12 V to 10 V as
i=
1+ j shown in the figure. How much energy does the battery
deliver during this talk-time?
1− j 1
∴ Current through inductor = = (a) 220 J (b) 12 kJ
j +1 1+ j (c) 13.2 kJ (d) 14.4 J [2009]
Hence, the correct option is (c).
V(t)
1 0. The average power delivered to an impedance
(4 - j3)Ω by a current 5 cos (100 πt + 100) A is 12 V
(a) 44.2 W (b) 50 W 10 V
(c) 62.5 W (d) 125 W [2012]
Solution: (b)
Average power is equivalent to RMS power t
0 10 mn
I 5
=
I rms =
2 2 Solution: (c)
2 25 Given current I = 2A
P= I rms R = × 4 = 50W
2 2 × (12 + 10)
Hence, the correct option is (b). E = VIT = I (VT ) = 2
× 10 × 60

1 1. In the interconnection of ideal sources shown in the fig- E = 13.2 kJ.


ure, it is known that the 60 V source is absorbing power. Hence, the correct option is (c).
20 V 1 3. The equivalent inductance measured between the termi-
+- nals 1 and 2 for the circuit shown in the figure is
M
+
I 60 V
- + +
1
12 A L1 L2

Which of the following can be the value of the current 2


source I?
Chapter 1  Basics of Network Analysis  |  1.7

(a) L1 + L2 + M (b) L1 + L2 - M (a) delivers 80 W (b) absorbs 80 W


(c) L1 + L2 + 2M (d) L1 + L2 - 2M (c) delivers 40 W (d) absorbs 40 W
 [2004]  [2002]
Solution: (d) Solution: (a)
Appling KVL for the given circuit, equation can be 5Ω
given as

dIdI dIdI dIdI dIdI


VV==−−MM dt++L1L1 dt++−−MM dt++LL 2
2 dt +
5Ω
dt dt dt dt V1
dIdI A
= ( L + L − 2 M )
= ( L1 1+ L2 2− 2 M ) dt -
V1 = 20 V 5
dt
So, equivalent inductance = Lequ = L1 + L2 − 2 M .
Hence, the correct option is (d). Applying KCL at node A,
1 4. The minimum number of equations required to analyze V1 − VA V1 VA
+ =
the circuit shown in the figure is 5 5 5
C C ∴V1 = VA
Power delivered by current source (as current is leaving
the positive terminal) = VA × I.
R R = V1 × V1
5
R C R V12
=
5
4
20 × 20
= = 80 W
(a) 3 (b) 4 5
(c) 6 (d) 7 [2003]
Hence, the correct option is (a).
Solution: (a)
16. The voltage e0 in the figure is
C 2 C

4Ω + 2Ω

1 12V 4Ω e0 2Ω
R R 3
-
V R C R
b

4
(a) 2 V (b) V
3
(c) 4 V (d) 8 V [2001]
The node voltages are numbered as 1, 2, 3
Solution: (c)
Number of equation = Number of nodes = 3.
Hence, the correct option is (a).
4Ω + 2Ω
1 5. The dependent current source shown in the figure
12V 4Ω e0 2Ω
5Ω
-

I b
+ 5Ω V1
A
V1 = 20 V 5 Equivalent resistance for the circuit = 4 + 4||4
-
= 4 + 2 = 6Ω.
1.8 | Network Theory

12 V2 OV
=
Total current = 2A.
6 +
- - +
∴ e0 = 2 × 4 × 4 = 2 × 16 = 4 V.
4+4 8 2V 1V
Hence, the correct option is (c).
17. If each branch of a Delta circuit has impedance 3 E=?
Z, then each branch of the equivalent Wye circuit has
impedance
Z +
(a) (b) 3 Z + -
3 -
Z 4V 5V
(c) 3 3Z (d)  [2001] V1 10V
3
Solution: (a)
(a) -16 V (b) 4 V
ZΔ = 3ZY (c) -6 V (d) 16 V [2000]
Z∆ Solution: (a)
3Z ∆ = 3ZY ⇒ ZY =
3 OV
V2
Hence, the correct option is (a).
18. In the circuit of the figure, the voltage V(t) is
2V 1V
1Ω 1 1Ω

+ E
r V(t) 1H tx
e e
-
4V 5V

eat - ebt (b)


(a) eat + ebt
V1 10V
ae - be (d)
(c) at bt
aeat + bebt [2000]
Solution: (d) Applying KVL in the given arrow direction and solving
0 = 1 + E + 5 + 10,
1Ω + 1Ω E = −16 V.
at V(t) 1H tx
Hence, the correct option is (a).
e e
- 20. The nodal method of circuit analysis is based on
(a) KVL and Ohm's law
(b) KCL and Ohm's law
(c) KCL and KVL
The voltage across inductor is given by differentiation (d) KCL, KVL and Ohm's law [1998]
of current across it. So, the voltage across 1H inductor Solution: (b)
is given by
Nodal analysis is based on KCL and ohm’s law.
V (t ) = L d (i1 + i2 ) 21. The voltage across the terminals a and b in the figure is
dt
2Ω a 1Ω
di di
V (t ) = 1 + 2 = ae at + be bt
dt dt +
1V 2Ω 3A
Hence, the correct option is (d). –

19. In the circuit of the figure, the value of the voltage


source E is b
Chapter 1  Basics of Network Analysis  |  1.9

(a) 0.5 V (b) 3.0 V Appling KCL at node B, sum of all currents will be
(c) 3.5 V (d) 4.0 V [1998] zero. So,
i4 + i1 + i0 = 0
Solution: (c)
i4 = −i1− i0 = − 5−7
a
i4 = −12A
2L 1Ω Hence, the correct option is (b).
3
+
1V i 2L 3A
23. The voltage V in the figure is equal to____
-
4V
+ -
b
+ 2Ω +
5V -- - 4V
Applying KVL at first mess, 1 = 2i + 2(i + 3)
  1 = 2i + 2i + 6
 −5 = 4i + V-

−5
i= (a) 3 V (b) –3 V
  4 (c) 5 V (d) None of these
−5 + 6 [1997]
Vab = 2i + 6 = 2 × = −2.5 + 6
4 Solution: (a)
 Vab = 3.5 V.
+ -
Hence, the correct option is (c).
4V
22. The current i4 in the circuit of the figure is equal to____ + 2Ω +
5V -- - 4V

i 1 = 5A i 2 = 3A

B + V-

i 0 = 7A
Applying KVL in outer mesh,
5 = 4 + 4 – V.
So, V = 3V.
Hence, the correct option is (a).
A
i4 = ? i 3 = 4A 2 4. The voltage V in the figure is always equal to____
2A 2Ω
(a) 12 A (b) -12 A
(c) 4 A (d) None of these +
[1997] +
V - 5V
Solution: (b)
-
i 1 = 5A i 2 = 3A

(a) 9 V (b) 5 V
(c) 1 V (d) None of these
[1997]
Solution: (d)
Since the voltage across current source is unknown, we
A cannot apply KVL to find V.
i4 = ? i 3 = 4A Hence, the correct option is (d).
1.10 | Network Theory

25. The voltage V in the figure is [1997] of coupling being 0.1. The total inductance of the com-
bination can be
3Ω
+ (a) 0.4 H (b) 3.2 H
(c) 4.0 H (d) 4.4 H [1995]
+ + Solution: (d)
V 10V 5V
- -
Given inductance of first and second coils as L1 = 2H,
L2 = 2H and coupling coefficient as K = 0.1, let mutual
- inductance between these coils be given by M. Then
(a) 10 V (b) 15 V equivalent inductance is given as
(c) 5 V (d) None of the these Leq = L1 + L2 + 2M = 2 + 2 + 2 K L1 L2
Solution: (a)
where  M = K L1 L2 = 4 + 0.1 2 × 2
3Ω     = 4.4 H.
+
Hence, the correct option is (d).

V + 10 V + 5V
- -
Two-marks Questions
- 1. Consider the network shown below with R1 = 1 W, R2 =
2 W and R2 = 3 W.. The network is connected to a con-
Voltage in parallel connection remains equal. So, stant voltage source of 11 V.
V = 10 V.
R1 R1
Hence, the correct option is (a).
2 6. A DC circuit shown in the figure given below has a
R2
voltage source V, a current source I and several resis-
R1 R3 R1 +
tors. A particular resistor R dissipates a power of 4 W R3 – 11 V
when V alone is active. The same resistor R dissipates
a power of 9 W when I alone is active. The power dis-
R2
sipated by R when both sources are active will be R1 R1

+
Resistive R The magnitude of current (in amperes, accurate to two
- Network decimal places) through the source is _______. [2018]
Solution: 
Resistance R1 = 1W
I Resistance R2 = 2W
(a) 1 W (b) 5 W Resistance R3 = 3W
(c) 5 W (d) 25 W [1995] Current I = ?
Solution: (d) I

Superposition theorem cannot be applied for nonlinear


parameters (power). R1 R1
When V1 is applied, power P1 = 4 W.
When I is applied,     P2 = 9W.
1 2

( )
2 R3 R2
With both sources,     P = P1 + P2 + 11 V
R1 R3 R1 –
   P = (2 + 3)2 = 25 W. R2
3 4
Hence, the correct option is (d).
27. Two 2H inductance coils are connected in series and are
also magnetically coupled to each other, the coefficient R1 R1
Chapter 1  Basics of Network Analysis  |  1.11

We know that above network is symmetrical, so redraw- 1


H( jω) =
ing the given circuit. 1 + j 5 × 200 × 103 × 1× 10 −6 
V1 = V2  and  V3 = V4
1
=
I 1 + j1

1
1Ω 1Ω H = ∠ –45°
2

+ 11 V 5
1Ω 3Ω 3Ω 1Ω – Vc(t) = sin(5t − 45°) V
2 
Vc(t) = 2.5 2 sin (5t – 45°) V
1Ω 1Ω
Hence, the correct option is (C)
3. In the given circuit, each resistor has a value equal to
1 Ω.
Req = {(1 || 1) + {1.5 || 0.5} + 0.5}  What is the equivalent resistance across the terminals a
and b? [2016]
11
=
0.5 + 0.5 + 0.375 = Ω 1 1
8  (A) Ω (B) Ω
6 3
11
I = = 8 Amp 9 8
11 8 (C) Ω (D) Ω
20 15
Hence, the correct answer is 7.9 to 8.1.
Solution:  Apply ∆ ↔ Y conversion to the given net-
2. For the circuit given in the figure, the voltage Vc (in
work
volts) across the capacitor is  [2017]
100 kΩ

4
+ + 3
1Ω 1Ω
5 sin (5t) V 1 μF VC
– –
4 4
3 3
100 kΩ

(A) 1.25 2 sin (5t – 0.25p)
(B) 1.25 2 sin 5t – 0.125p a
(C) 2.5 2 sin (5t – 0.25p) 1Ω
(D) 2.5 2 sin (5t – 0.125p) 4Ω 1Ω

Solution: Consider the figure given below

200 kΩ b

+ +
5 sin 5t VC 1 μF
– – 1Ω

Vc (t) = ?
4 4
Ω Ω
5 5
We know that:
Vc ( S )
H(S) =
Vin ( S )

4
Ω
1 5
H(S) =
1+ jω RC

1.12 | Network Theory

i1 = 0.16Vx
⎛ 4 ⎞ ⎛ 8⎞
∴ Rab = ⎜ ⎟ // ⎜ ⎟ 
⎝ 5 ⎠ ⎝ 5⎠ i2 = 0.2Vx
4 8 5(0.16Vx) + 8(0.2Vx) = 60
×
Rab = 5 5
4 8 Vx [0.8 + 1.6] = 60
+
5 5  Vx = 25 volts
8×4 5
Rab = × 25
25 12  i2 = = 5A
5 
8
= Ω iR2 = 5 A
15  
Hence, the correct option is (D). Hence, the correct Answer is (5 A).
4. In the circuit shown in the figure, the magnitude of the 5. In the figure shown, the current I (in ampere) is ___ .
current (in amperes) through R2 is ____.  [2016]  [2016]

a
1A 5Ω
8V +

1Ω 1Ω
+ 8V

i

b Solution: 
1A 5Ω
R1 R2

5Ω 3Ω
R3 +
+
+ 1A 4A 1Ω – 8V
– 60 V 5Ω vx
0.04 vx
– 1Ω 1Ω
O O
V1
Solution:  Consider the labelled figure given below 4A + 8 V4 A

R1 R2 5A i
4A 1Ω
5Ω 3Ω
+
Vx 1A
60 V +
– i1
0.04 v i2 5Ω

Apply nodal analysis at node V1


Mesh equation
V1 − 0 V1 − 8 V1 − 0 V − 8
−60 + i1 (5Ω) + i2 (3Ω) + i2 (5Ω) = 0  + + + =0
1 1 1 1 
5i1 + 8i2 = 60  4V1 = 16 ⇒ V1 = 4 volts

Vx = i2 (5Ω) (i) ∴ I = –1 Amp

Now we have, Hence, the correct Answer is (-1 amp).
i1 – i2 = –0.04Vx 6. At very high frequencies, the peak output voltage V0
vx (in Volts) is _______. [2015]
i1 = − 0.04Vx
5 
Chapter 1  Basics of Network Analysis  |  1.13

100 μF a

1 kΩ 1 kΩ V0 2R R R 2R
100 μF
+
1.0 sin(ω t) V –

1 kΩ 1 kΩ b
Rab
100 μF
∴ Rab = (2R||2R)||(R||R) = (R||R||R)

Solution:  At high frequencies the given equivalent = R/3


network. But give R = 300 Ω
Shown in below ∴ Rab = 100 Ω 
Zc → short circuit
Hence, the correct Answer is (99.5 to 100.5).
8. In the given circuit, the values of V1 and V2 respectively
V0 are [2015]
1 kΩ 1 kΩ

Vm
I
1 kΩ 1 kΩ + 2I +
V2 5A 4Ω 4Ω V1
– –

Vm × 1k Vm
Vo = = (A) 5 V, 25 V (B) 10 V, 30 V
2k 2 (C) 15 V, 35 V (D) 0 V, 20 V
1 Solution:  Apply nodal analysis to the given network.
∴ The peak o/p voltage = = 0.5 volts
2 V1 V1
–5 + + + 2I = 0
Hence, the correct Answer is (0.49 to 0.51). 4 4 
7. In the network shown in the figure, all resistor are iden- V1
tical with R = 300 Ω. The resistance Rab (in Ω) of the + 2I = 5
network is ______. [2015] 2 
V1
a But, I=
4
R R R R
R R V1 V1
R R R + =5
R = 300 Ω 2 2 
R R ⇒ V1 = 5 volts
Rab R R R R
Apply KVL in 1st loop
b
V2 – 4 × 5 – V1 = 0
Solution:  The given network is in balanced mode, so
V2 = 20 + 5
redraw equivalent network.
a =
25 Volts

R R R R Hence, the correct option is (A).


R R
9. The voltage (VC) across the capacitor (in Volts) in the
network shown is ______. [2015]
R R Vc
R R R R 80 V 40 V

b
Rab

100 V, 50 Hz
1.14 | Network Theory

Solution:  We know Two such blocks are connected in cascade, as shown in


the figure.
V = VR2 + (VL − VC ) VR2 + (VC − VL )
2 2
(or)

100 = (80 ) + (VL − VC )
2 2

 +
(100)2 = 802 + (VL – VC)2 V1 (s) Network N Network N V2 (s)
(VC – VL)2 = 3600
-
VC – VL = 60
VC = 100 V
Hence, the correct Answer is (100).
V3 (s)
10. A Y-network has resistances of 10 Ω each in two The transfer function of the cascaded network is
of its arms, while the third arm has a resistance of V1 (s)
11 Ω. In the equivalent A-network, the lowest value (in s s2
Ω) among the three resistances is_____. [2014] (a) (b)
1+ s 1 + 3s + s 2
2 2
 s   s 
(c)   (d)
 
1+ s  2+s
 [2014]
R1 Solution: (b)
Ra Rb

R3 1/SC 1/SC
R2 R
V1 (s) R V3 (6)
I1 I2

Rc
 1 
Applying KVL,  R + I1 ( s) − RI 2 ( s) = V1 ( s)
RR
Ra = R1 + R2 + 1 2  SC 
R3
 1 
RR  R + SC  I 2 ( s) − RI 2 ( s) = 0
Rb = R1 + R3 + 1 3  
R2
  V3(s) = I2(s)R
R2 R3
Rc = R2 + R3 + V ( s) s 2 R 2C 2
R1 ∴ 3 = 2 2 2
V ( s) s R C + 1 + 3RCs
 Here R1 = 11Ω, R2 = 10Ω, R3 = 10Ω 1
∴Ra = Rb = 32 Ω, Rc = 29.09 Ω → lowest value     R = 10 kΩ, C = 100 μF, RC = 1
11. Consider the building block called ‘Network N’ shown  
V3 ( s) s2
in the figure. = 2
V1 ( s) s + 3s + 1
Let C = 100 µF and R = 10 kΩ
Hence, the correct option is (b).
Network N
1 2. In the circuit shown in the figure, the value of node
voltage V2 is
C
10 ∠0°
+−

V1 4Ω V2

V1 (s) R V2 (s)
4 ∠ 0° A –j 3 Ω 3Ω j 6Ω
Chapter 1  Basics of Network Analysis  |  1.15

(a) 22 + j2 V (b) 2 + j22 V 14. The equivalent resistance in the infinite ladder network
(c) 22 - j2 V (d) 2 - j22 V [2014] shown in the figure is Re
Solution: (d) 2R R R R

+−
Re R R R R
V1 4Ω V2

4 0° j3 Ω 6Ω j6 Ω
The value Re/R is______. [2014]
Solution: 2R R R R
2R R R R
2R R R R
R R R
V1 V2 V2
V1 − V2 = 10∠0°; + + = 4∠0° R R R
−3 j 6 j 6 R R R

20 + j 24
∴V2 = = 22.09∠ − 84.8
( −1 + j ) R R
  2R RR RR R
   V2 = 2 – 22 j R R
R R ...
...

8 8
Hence, the correct option is (d). R RR RR
13. For the Y-network shown in the figure, the value of R1 R R ...

8
(in Ω) in the equivalent ∆-network is____. [2014] Req
Req
R1 Req
R RR
R
5Ω 3Ω R
R RR
... R

8
eq
Req
R
Req
R
Req Req
Req
7.6 Ω

Req
R + R.Req
R 2R
Req = 2R
R + Req
2R
Req = 1.618 R
R R Req
 5 R
Req = 1.618 R
Req = 1 + R Req = 1.618 R
 2  R

R1 Req
Req = 1.618 R
2R
5Ω

3Ω Req = 1.618 R
R
7.5 Ω

R × 1.618 R
Rin = 2 R +
R + 1.618 R

Rin = 2.618 R → Rin = 2.618


R
15. The following arrangement consists of an ideal trans-
5 + 3 + 5× 3 former and an attenuator which attenuates by a factor of
R1 + = 10Ω 0.8. An ac voltage VWX1 = 100 V is applied across WX
7.5
1.16 | Network Theory

to get an open circuit voltage VYZ1 across YZ. Next, an Solution: (c)
ac voltage VYZ2 = 100 V is applied across YZ to get an
C2 C3
open circuit voltage VWX2 across WX. Then, VYZ1/VWX1,
VWX1/VYZ2 are, respectively,
W
1: 1.25
Y

X Z
C1

(a) 125/100 and 80/100 Q1 = C1V1 = 100 μC


(b) 100/100 and 80/100 Q2 = C2V2 = 25 μC
(c) 100/100 and 100/100 Q3 = C3V3 = 4 μC
(d) 80/100 and 80/100 [2013] In series, charge remains same so, the charge on C2 and
Solution: (b) C3 must be 4 μC.
VYZ1 = 100 × 1.25 × 0.8 = 100 V C2C3 10
C23 = = µF
When 100 V is applied at YZ, this will appear across C2 + C3 7
secondary winding.
θ 4 µC 28
Vwx2 = 100/1.25 = 80V V23 = = = = 2.8 V
C23 10 10
µC
VYZ1 100 VWX 2 80 7
= , =
VWX1 100 VYZ2 100 In parallel, voltage will remain the same
V1 = V23 = 2.8 V
W Q1 = C1V1 = 28 μC
1: 1.25
Total charge = Q1 + Q23 = 32 μC
Y
Hence, the correct option is (c).
Common Data For Questions 8 and 9
X Z
Consider the following figure:
A
Hence, the correct option is (b).
1 6. Three capacitors C1, C2 and C3 whose values are 10 µF, i1
5 µF, and 2 µF, respectively, have breakdown voltages
5Ω Is
of 10 V, 5 V and 2 V, respectively. For the interconnec- 2Ω 1Ω
tion shown below, the maximum safe voltage in volts 10V
+
that can be applied across the combination, and the cor- 2A
Vs
responding total charge in µC stored in the effective -
capacitance across the terminals, are, respectively
C2 C3
17. The current Is in Amps in the voltage source, and voltage
Vs in volts across the current source respectively, are
(a) 13, -20 (b) 8, -10
(c) -8, 20 (d) -13, 20 [2013]
Solution: (d)
A

i1

C1 5Ω Is
2Ω l Ω
10V
(a) 2.8 and 36 (b) 7 and 119 +
(c) 2.8 and 32 (d) 7 and 80 Vs 2A
 [2013] -
Chapter 1  Basics of Network Analysis  |  1.17

In parallel, voltage remains same, so the voltage across 20. In the circuit shown below, the current I is equal to
1Ω remains 10 V.
10
=
i1 = 10 A j4 Ω -j4 Ω
1
+ 6Ω
Voltage across 2Ω = 10 V 14 0 A
0

10 -
∴i2 = =5A 6Ω 6Ω
2
Applying KCL at A, −2 + If + i2 + i1 = 0
IS = −13 A (a) 1.4 ∠0°A (b) 2.0 ∠0°A
VA = 10 V = VS = VA + 10 = 20V (c) 2.8 ∠0°A (d) 3.2 ∠0°A [2011]
Hence, the correct option is (d). Solution: (b)
18. The current in the 1 Ω resistor in Amps is Applying delta to star conversion,
(a) 2 (b) 3.33
(c) 10 (d) 12 [2013]
Solution: (c) j4 Ω -j4 Ω

10
Current in 1 Ω resistor is = =
I1 = 10 Amp 14 0 A
0
2Ω 2Ω
1
Hence, the correct option is (c).
19. If VA - VB = 6 V, then Vc - VD is 2Ω

R VA 2 Ω VB R
Zeq = (2 + 4j) || (2 – 4j) + 2
R R -
R R 1Ω R +
10V Zeq = 7Ω
R 2 Ω VB RR
VA V 14∠00
+ - Vc V0 I= = = 2∠0°A
5V
R
2A
-
Zeq 7
R 10V
R R 1Ω R +
(a) -5 V (b) R2 V Hence, the correct option is (b).
(c) 3 V + - Vc V(d)
0
6 V [2012] 1 1. In the circuit shown, the power supplied by the voltage
5V 2A
Solution: (a) source is
1Ω
R VA 2 Ω VB R

R R - 1Ω 1Ω
R R 1Ω R 10V
+
R + 10V
-
+ - Vc V0
5V 2A 1A
−1Ω 2A
1Ω
Since VA – VB is known, we can replace the left and right
half as shown.
(a) 0 W (b) 5 W
2Ω (c) 10 W (d) 100 W [2010]
A B
Solution:  (a)
N1 1Ω N2 3+i i

3A
C 2A D 3A
+
Total current = A5A 1Ω 2Ω
through - 10V
B in anti-clock wise 2+i
1A
direction 2A
∴VCD = −5 N1 × 1 = −5V 1 Ω N2
Hence, the correct option is (a).
1Ω
3A
C 2A D
1.18 | Network Theory

Applying KVL in outer loop, Solution: (c)


(3 + i) × + (2 + i) × 2 = 10 For given circuit given parameters are
i=0 R1 = R2 = R4 = R and R3 = 1.1R
∴Power supplied by voltage source = VI = 0
Hence, the correct option is (a). R1 R4
2 2. Impedance Z as shown in the given figure is +
10V a b
j5 Ω j2 Ω + -
-
R2 R3

j10 Ω j2 Ω

j10 Ω
R2
Va = × 10 = 5 V
1 + R2
Z R

(a) j29 Ω (b) j9 Ω R5 1.1


Vb = × 10 = × 10 = 5.238 V
(c) j19 Ω (d) j39 Ω [2005] R3 + R5 2.1
Solution: (b) So, voltage across terminal ab is
j5 Ω j2 Ω Vab = Va – Vb = −0.238 V.
Hence, the correct option is (c).
24. For the circuit shown in the figure, the initial conditions
j10 Ω are zero. Its transfer function
j2 Ω

j10 Ω Vc ( s)
H ( s) = is
Vi ( s)

Z
10k Ω 10mH
Impedance of the circuit will be given by
V0(t) 100 µ F V0(t)
Z = X L1 + X L2 + X L3 + 2 M 23 − 2 M13
Z = j5 + j2 + j2 + 2 × 10j – 2 × 10j
Z = 9 j Ω
1 6
Hence, the correct option is (b). (a) 2 6 6
(b) 10
2 3. If R1 = R2 = R4 = R and R3 = 1.1 R in the bridge circuit s + 10 s + 10 s + 10 s + 106
2 3

shown in the figure, then the reading in the ideal volt-


meter connected between a and b is 103 6
(c) 2 3 6
(d) 10
s + 10 s + 10 s 2 + 106 s + 106
R1 R4  [2004]
+ Solution: (d)
10V a b
+ - Given circuit
-
R2 R3
10k Ω 10mH

V1(t) 100 µ F V0(t)

(a) 0.238 V (b) 0.138 V


(c) -0.238 V (d) 1 V [2005]

R δL

V1(s) 1 V0(s)
SC
10k Ω 10mH

V1(t) 100 µ F V0(t)

Chapter 1  Basics of Network Analysis  |  1.19

Writing in s-admin, 5
(a)  W (b) 1 Ω
6
R δL
6 3
V0(s) (c)  W (d) 
W [2003]
V1(s) 1 5 2
SC
Solution:(a)
i
i 3
Appling KVL in given circuit a i
3
1 i
Vi ( s) = I ( s) R + sL + i
sc 3
6
I ( s) 2
Vi ( s) = ( s LC + sCR + 1) i
sc b
i
I ( s) 3
V0 ( s) =
SC Equivalent resistant can be calculated as ratio of volt-
1
V0 ( s) 1 age to current across terminal
∴ = = LC
Vi ( s) s 2 LC + sCR + 1 L 1 i i i
s2 + s + Vab = × 1 + × 1 + × 1
R LC 3 6 3
V0 ( s) 106 Vab 5
= 2 Req = = Ω
V ( s) s + 106 s + 106 i 6
i
Hence, the correct option is (d). Hence, the correct option is (a).
2 7. The current flowing through the resistance R in the cir-
25. The transfer function H ( s) = V0 ( s) of an R-L-C
cuit in the figure has the form P cos 4t, where P is
Vi ( s)
M=0.75 H
106 1/10.24 F
circuit is given by H ( s) = 2 6
.
s + 20 s + 10
The Quality factor (Q-factor) of this circuit is R=3.92 Ω
3Ω
(a) 25 (b) 50
(c) 100 (d) 5000 [2004]
Solution: (b) V=2cos4t +
20 V
Transfer function is given as,
106 (a) (0.18 + j 0.72)
H ( s) =
s 2 + 20 s + 106 (b) (0.46 + j 1.90)
Comparing parameters with standard equation (c) -(0 18 + j 1.90)
(d) (0.23 - 0.35 j) cos 4t [2003]
W02
f Solution:  (d)
s 2 + 0 s + w02
Q Here the value of inductance is not g­ iven we can ignore
it
f0 f
= 20 ⇒ Q = 0 2 cos 4t
Q 20 I1 = = 0.67 cos 4t i.e.ω = 4
3
3
So f 0 = 103 ⇒ Q = 10 = 50 − jω M − j 4 × 0.75 I1
I2 = =
20 R − [ j / ωC ] 3.92 − 2.56 j
Hence, the correct option is (b). − j 3 × 0.67Cos 4t 3.92 + 2.56 j
= ×
3.92 − 2.56 j 3.92 + 2.5 j
26. Twelve 1 Ω resistances are used as edges to form a = (0.23 − 0.35 j )Cos 4t
cube. The resistance between two diagonally opposite
corners of the cube is Hence, the correct option is (d).
1.20 | Network Theory

28. If the three-phase balanced source in the figure delivers Appling KCL to calculate e0
1500 W at a leading power factor 0.844. then the value e0 − 80 e0 e0 − 16
of ZL (in ohm) is approximately + + =0
12 12 6
ZL ZL e0 = 28 V
3 PHASE
400 V
Hence, the correct option is (d).
BALANCED ZL
SOURCE 3 0. For the circuit in the figure, the voltage V0 is

2Ω 2Ω
(a) 90 ∠32.44° (b) 80 ∠32.44° +
-
-
4V 2V
(C) 80 ∠-32.44° (d) 90∠-32.44° - +
V0
[2002] 2Ω
+
Solution: (d)
Power delivered = 3VP I P COSq (a) 2 V (b) 1 V
3VP I P cos θ = 1500 (c) -1 V (d) None of these
[2000]
 V  V 
3 L   L  cos θ = 1500 Solution: (d)
3   3Z L 
  V

VL2 cos θ 400 2 × 0.844 2Ω 2Ω


ZL = =
1500 1500 + -
-
4V 2V
ZL = 90Ω - +
V0
q = cos−1 (0.84) = 32.44 2Ω
+
As power factor is leading, it implies load is capacitive
= q = - 32.44º
Since diode is Forward Bias, so it will be replaced by
Hence, the correct option is (d). short circuit wire, and now applying KCL for voltage V.
29. The voltage e0 in the figure is V −4 V V +2
+ + =0
2 2 2
2Ω
16 V + 2
BA 10 Ω 12 Ω e0
V=
  3
6Ω -
−2
V0 = V =
  3
(a) 48 V (b) 24 V
(c) 36 V (d) 28 V [2001] Hence, the correct option is (d).
Solution: (d) 3 1. ADelta-connected network with its Wye-equivalent is
2Ω
16 V + shown in the figure. The resistances R1, R2 and R3 (in
BA 10 Ω
2Ω 12 Ω e0 ohms) are respectively
616
ΩV -+
BA 10 Ω 12 Ω e0 a
6Ω -
a
R1
5Ω 30 Ω
Converting current source of 8A to equivalent voltage
R2
source, R3
b c
e0 15 Ω b c

10 Ω 2Ω e0 16 V (a) 1.5, 3 and 9 (b) 3.9 and 1 5


2Ω 12 Ω (c) 9, 3 and 1.5 (d) 3, 1.5 and 9
10 Ω
616
ΩV
12 Ω [1999]
6Ω
Chapter 1  Basics of Network Analysis  |  1.21

Solution: (d) From the figure,


For delta to star conversion, the equivalent resistant in 5−2
star network is given by i= = 1A
3
Ra Rb 150 Hence, the correct option is (c).
R1 = = = 3Ω
Ra + Rb + Rc 20iD+ 30 33. A network contains linear resistors and ideal voltage
4Ω sources. If values of all the resistors are doubled, then
Rb Rc −75i
R2 = = iD = 1.5Ω the voltage across each resistor is
Ra + Rb +4 RΩc 50 D (a) halved (b) doubled
+
4Ω
10 V 4Ω 1Ω 2A (c) increased by four times (d) not changed
– 3
i
R +R 15 ×D 30 [1993] t
R3 = 10 V a+ c 4 Ω = 4Ω =119Ω
ΩΩ
2A
10 V – 4Ω 2A
Ra + R–b + Rc 50 Solution: (d)
1
When the value of all resistors is doubled then the cur-
Hence,10the +
V correct option
4 Ω is (d).
1Ω 2A rent in corresponding resistors is halved, hence voltage

32. In the circuit shown in the figure the current iD through remains unchanged.
the ideal diode (zero cut in voltage and zero forward Hence, the correct option is (d).
resistance) equals 34. The two electrical sub networks N1 and N2 are con-
iD
nected through three resistors as shown in the figure.
4Ω
The voltages across 5 ohm resistor and 1 ohm resistor
are given to be 10 V and 5 V, respectively. Then voltage
across 15 ohm resistor is
+ 4Ω 1Ω 2A + 10 V -
10 V
– 5Ω

15 Ω
N1 N2
(a) 0 A (b) 4 A
1Ω
(c) 1 A (d) None of these
[1997] + +5V-
10 V 4Ω 1Ω 2A
Solution: –(c) (a) -105 V (b) + 105 V
+ 4Ω 1Ω 2A (c) -15 V (d) + 15V [1993]
10 V + 4Ω 1Ω 2A
10 V –

Solution: (a)
+ U + 10 V - V
10 V 4Ω 1Ω 2A
– 5Ω

15 Ω
N1 N2
2.5 V 4Ω 4Ω 1Ω 2A +5V-

2.5 V 4Ω 4Ω 1Ω 2A 1Ω
2.5 V 4Ω 4Ω 1Ω 2A

When two networks are connected then net incoming


2.5 V + 4Ω 4Ω 1Ω 2A current is equal to net outgoing current
10 V 4Ω 1Ω 2A
– 10
2.5 V 2Ω 1Ω 2A Current through 5Ω, =
i5 = 2A
5
2.5 V 2Ω 1Ω 2A
2.5 V 2Ω 1Ω 2A
Current through 1Ω, i1= 5= 5A
1
2.5 V 2Ω 1Ω 2A So, current through 15Ω resistor,
  i15 = −(i1 + i5) = − 7A
2Ω 1Ω
2.5 V 4Ω i 4Ω 1Ω 2A ∴V15 = 15(−7) = −105 V
10 V 2A
2Ω 1Ω Hence, the correct option is (a).
2Ω i 1Ω
i
10 V 2A 35. Of the networks N1, N2, N3 and N4 of figure, the net-
10 V 2A
works having identical driving point function are
2Ω 1Ω
i
10 V 2A
2.5 V 2Ω 1Ω 2A
2H 1Ω
1.22 | Network Theory 2H 1Ω

2H For network 1 2H 1Ω
2Ω 1F
1Ω
1F
12 Ω 1
y1 (b) = s + +
2 s +2 Ω
1 1 1F
2Ω 1F 1F N1 +2
s 1F N1
2
2s + 2s + 1
1F N1 y1 (b) = 1F N1
2s + 1
2H 1Ω
2H 1Ω
2H 1Ω
1H
2H 2H 1Ω
1Ω 1H
2Ω 1F
1H 1H
N2
2Ω 1F N
2Ω 1F 1F N 12

N2 N2
2Ω 2Ω 1F
1F
For network 2
1 1 1+ s
y2 ( b ) = + 2H = 1Ω
1Ω 1Ω 2s + 1 1 2s + 1
2+ 1Ω
1Ω s
1H

1Ω 1Ω 1Ω
1Ω

N2
2Ω 1F
1F N3
1F N3

N3 1F N3
1F

1Ω 2H
1Ω
1Ω 12H

For network 3 2H
1Ω 2H 1Ω
1 1+ s
y3 (b) = s + = s+
1 s +1+ s
1+
N4 1
1F 1+ N
s 1F
1F N 34
2s2 + 2s + 1 1F N4
1F N4 y3 (b) =
(a) N1 and N2 (b) N2 and N4 2s + 1
(c) N1 and N3 (d) N1 and N4
 [1992] 1Ω 2H
Solution: (c)
To calculate identical driving point function we need to
calculate driving point admittance for each network (for
parallel branches admittance can be easily calculated)
2H 1Ω 1F N4

2Ω 1F
For network 4
1 2s2 + s + 1
1F N1 y4 ( b ) = s + =
2s + 1 2s + 1
Hence, the correct option is (c).

2H 1Ω

1H
Chapter 1  Basics of Network Analysis  |  1.23

36. A square waveform as shown in figure is applied across


1 mH ideal inductor. The current through the inductor
is a.........wave of..........peak amplitude. [1987]
V

0 0.5 1 1 (m sec)

-1 Solution:

Solution: triangular and 2


Given L = 1 mH
di
Voltage across inductor is given as, V = L
dt
di V
=
dt L
t t
1 1
i=
L ∫ Vdt =
L
Vdt∫ RAB =
VAB
−∞ 0
I
V Connect current source IA between A and G. I enters
–1 terminal A, goes through the infinite grid and comes
back to through ground.
+ I   I  I
0 0.5 1 V1 AB = VAC + VCB =  ×1 +  ×1 =
 4   12  3

–1
Now,
Connect a current source, I between B and G. I enters
The current through inductor is integration applied the infinite grid through G and leaves B.
voltage across the inductor. As integration of step volt-
age is triangular wave, so current through inductor is
a triangular wave and slope of triangular wave = step
change in square wave
1− 0
slope = =2
0.5 − 0

i
2 × 0.5 +2 -2 +2 -2
=1
t  I  I  I
0 0.5 1.5 V2 AB = VAC + VCB =  × 1 +  × 1 =
 12   4  3

I p = 1A
I I 2I
∴VAB = V1 AB + V2 AB = + =
3 3 3
VAB 2
∴ RAB = = Ω
Five-marks Questions I 3
1. An infinite grid is built up by connecting in the manner
indicated in figure, where each branch represents one
ohm resistor. Calculate the effective resistance between
the nodes A and B.
Chapter 2
Sinusoidal Steady State
1
One-mark Questions ω= , we get the value from which the ratio
( RLC )
1. In the circuit shown, V is a sinusoidal voltage source. of IL/IR can be evaluated.
The current I is in phase with voltage V. The ratio Current across the inductor is
Vc 100Vc
amplitude of voltage across the capacitor is_______. IL = −3
⇒ IL =
jω10 × 10 ω
 amplitude of voltaage across the resistor [2017]
Current across resistance will be
5Ω 5H I V V
I R = 10c ⇒ I R = c
10
At resonance,
V ∼ 5F
1 1
ω= = = 107
−9 −7
10 × 10 × 10 10
2. The figure shows an RLC circuit with a sinusoidal The required ratio of current across inductor and resist-
­current source. ance is
IL 100V 107 1000 1
IR IL IC = = = = 0.316
IR V 10 10 7 10
Im sin ω t R L C
Hence, the correct Answer is (0.316).
10 Ω 10 mH 10 μF
3. A series RC circuit is connected to a DC voltage source
at time t = 0. The relation between the source voltage
At resonance, the ratio |IL|/|IR| i.e., the ratio of the mag- Vs, the resistance R, the capacitance C, and the current
nitudes of the inductor current phasor and the resistor i(t) is given below:
current phasor, is _____. [2016] t
1
Solution:  Resistance R = 10 ohms
Inductance L = 10 mH
Vs = Ri (t ) +
C ∫ i (t )dt
0
Capacitance C = 10 microfarad (µF)
We know that voltage across all the elements remain Which one of the following represents the current i(t)?
constant (a) I(t)

IL +

10 mH 10 μF t
Im sin ω t 10 Ω (b)
Vc I(t)
IR

Using the resonance condition when R, L, C are con- t


nected in parallel
Chapter 2  Sinusoidal Steady State  |  1.25

(c)
I(t) 1
R.
 1  SC
V1 ( s) =  R + I ( s) + I ( s)
 SC  1
R+
t SC

(d)
I(t)  1 + SCR  R
V1 ( s) =   I ( s) + I ( s)
 SC  1 + SCR

 t  1
t Vi = V p cos   ,W =
 RC  RC
[2014]
Solution: (a)  1 + jωCR   R 
Vi ( s) =   I ( s) +   I ( s)
I ( s)  jω C   1 + jω CR 
V ( s) = RI ( s) +
SC
 (1 + j ) R R 
V ( s) Vi ( s) =  +  I ( s)
I ( s) = ; V(s) = 1/5  j 1+ j 
1
R+
SC V ( s)
I ( s) = (1 + j )
1 3R
I ( s) =
 1   1 
R s + V0 ( s) =  R ||  I ( s)
 RC   SC 

1 −t / RC R
i (t ) = e
R
V0 ( s) = SC I ( s)
Hence, the correct option is (a). 1
R+
SC
4. The circuit shown below is driven by a sinusoidal input
vi = Vp cos(t/RC). The steady output v0 is V1 ( s)
V0 ( s) =
3
Vi (t ) V p  t 
V0 (t ) = = cos  
3 3  RC 

W = 1/RC
Hence, the correct option is (a).
(a) (Vp/3)cos(t/RC) (b) (Vp/3)sin(t/RC) 5. For a parallel RLC circuit which one of the following
(c) (Vp/2)cos(t/RC) (d) ( V p / 2 ) s i n ( t / R C ) statements is NOT correct?
 [2011] (a) The bandwidth of the circuit decreases if R is
Solution:  (a) increased
(b) The bandwidth of the circuit remains same if L is
increased
(c) At resonance, input impedance is a real quantity.
(d) At resonance, the magnitude of input impedance
attains its minimum value [2010]
Solution: (d)
1
B.W. = for parallel R-L-C
RC
R+ 1
SC At resonance, impedance is maximum in parallel
1
V1(s) ~ (R SC ) R-L-C.
Hence, the correct option is (d).
6. The condition on R, L and C such that the step response
y(t) in the figure has no oscillations is
1.26 | Network Theory

Solution: (a)
i(t)

V(t) R L C

1 L L
R≥
(a) R≥
(b) i(t) = V(t) . Y
2 C C
1 1 
L 1 i (t ) = V (t )  + + jωC 
R≥2
(c) R=
(d)  R jω L 
C LC
 [2005]  4 
i(t ) = sin 2t 3 + + j × 2 × 3
Solution: (c)  2 

Z=
R C
≥1 i(t ) = sin 2t [3 − 2i + 6 j ]
2 L
i(t ) = sin 2t [3 + 4 j ]
L 4
∴R > 2 i(t ) = 5 sin 2t ∠ tan −1 = 5 sin( 2t + 53.1°)
C 3
Hence, the correct option is (c). Hence, the correct option is (a).
7. In a series, RLC circuit, R = 2 kΩ, L = 1 H, and 9. For the circuit shown in the figure, the time constant
1 RC = 1 ms. The input voltage is vi(t) = 2 sin103t. The
C= µF
400 output voltage v0(t) is equal to
R
The resonant frequency is
1
(a) 2 × 104 Hz (b) × 10 4 Hz
π Vi(t) C V0(t)
(c) 10 Hz
4
(d) 2π × 10 Hz
4

 [2005]
Solution: (b)
(a) sin(103t - 45°) (b) sin(103t + 45°)
4
1 10 (c) sin(10 t - 53°)
3
(d) sin(103t + 53°)
f = = Hz
2π LC TC [2004]
Hence, the correct option is (b). Solution: (a)
8. The circuit shown in the figure, with
1 1
R = Ω, L = H, C = 3 F has input voltage V(t) = sin
3 4
2t. The resulting current i(t) is
i(t) 1
jωC
V0 (t ) = − Vi (t )
1
V(t) R L C R+
jωC
1
V0 (t ) = 2 sin 103 t
1 + jωCR
(a) 5 sin (2t + 53.1°)
(b) 5 sin (2t – 53.1°) V0(t) = sin(103 t – 450)
(c) 25 sin (2t + 53.1°) Hence, the correct option is (a).
(d) 25 sin (2t – 53.1°) [2004]
Chapter 2  Sinusoidal Steady State  |  1.27

10. A series RLC circuit has a resonance frequency of 1 (a) 1 A


kHz and a quality factor Q = 100. If each of R, L and (b) 5 A
C is doubled from its original value, the new Q of the (c) 7 A
circuit is (d) None of the above [1996]
(a) 25 (b) 50 Solution: (b)
(c) 100 (d) 200 [2003]
Solution: (b)
f0 = 1 kHz, ϑ = 100

θ = 1 L for series RLC


R C
1 2L 1 L θ1
θ2 = = =
2 R 2C 2 R C 2 A12 − A22 + A32

q2 = 50 A12 = 32 + 4 2 = 25 ⇒ A1 = 5A
Hence, the correct option is (b). Hence, the correct option is (b).
11. The parallel RLC circuit shown in figure is in reso- 13. In the circuit of the figure, assume that the diodes are
nance, In this circuit ideal and the metre is an average indicating ammeter.
IC The ammeter will read
IR IL

1 mA RMS
C
R L

(a) |lP| < 1 mA (b) |lR + lL| >1 mA


(c) |lR + lC| >1 mA (d) |IL + IC| > 1 mA
 [1998] (a) 0.4 2 mA (b) 0.4 mA
Solution: (b)
0.8 0.4
(c) mA (d) mA  [1996]
IR IL p p
1 mA R L Solution: (d)
C
When Vin is ( + ) ve:- D1 will conduct for ( + ) ve half
cycle.
V 1 4 1
I avg = m × = ×
I – IR = 1 mA TC R π 10 × 103
2 2 0.4
I R − J L = J R + I L > 1 mA I avg =
π
mA

|IR + IL| > 1 mA
Hence, the correct option is (d).
Hence, the correct option is (b).
14. A DC voltage source is connected across a series R-L-C
12. In the below given figure, A1, A2 and A3 are ideal amme- circuit. Under steady-state conditions, the applied DC
ters. If A2 and A3 read 3 A and 4 A, respectively, then A1 voltage drops entirely across the
should read (a) R only
L (b) L only
A2 (c) C only
A1 (d) R and L combination [1995]
R
A3 Solution: (c)
For DC supply, inductor behaves as short circuit and
~ capacitor behaves as an open circuit. So, under steady-
state conditions, the applied DC voltage drops entirely
Sinusoldal across capacitor only.
voltage source
Hence, the correct option is (c).
1.28 | Network Theory

15. Consider a DC voltage source connected to a series VC = I( − jXC) = −j400


R-C circuit. When the steady-state reaches, the ratio of VC = 400∠ − 90°
the energy stored in the capacitor to the total energy
Hence, the correct option is (d).
supplied by the voltage source is equal to
(a) 0.362 (b) 0.500 1 9. In the series circuit shown in figure, for series reso-
(c) 0.632 (d) 1.000 [1995] nance, the value of the coupling coefficient k will be
Solution: (b)
1
WS = CVS2 and WC = CVS2
2
WC
= 0.5 (a) 0.25 (b) 0 5
WS (c) 0.999 (d) 10 [1993]

Hence, the correct option is (b). Solution: (a)
1 6. The current, i(t), through a 10-Ω resistor in series with At resonance net capacitive impedance = net inductive
an inductance, is given by impedance.
i(t) = 3 + 4sin(100t + 45°) + 4 sin (300t + 60°) amperes. So, = XL =
X C J 12
The RMS value of the current and the power dissipated XL = J12
in the circuit are:
Let k denote the coupling coefficients then
(a) 4 A, 410 W, respectively
X L = X L1 + X L 2 + 2k X L1 X L 2
(b) 35 A, 350 W, respectively
(c) 5 A, 250 W, respectively J 2 + J 8 + 2k J 2 J 8 = J 12
(d) 11 A, 1210 W, respectively [1995] = 2kj 4 = j 2
Solution: (c)
     k = 0.25
i(t) = 3 + 4 sin (100t + 45°) + 4sin(300t + 60°)
Hence, the correct option is (a).
2 x
 4   4 
I rms = 32 +   +  = 5A
 2  2 Two-marks Questions
2
P = I rms R = (5) 2 × 10 = 250 W 1. The Figure shows an RLC circuit exited by the sinu-
soidal voltage 100 cos (3t) Volts, where t is in seconds.
Hence, the correct option is (c). amplitude of V2
The ratio is ________. [2017]
1 7. A series R-L-C circuit has a Q of 100 and an impedance amplitude of V1
of (100 + j0) Ω at its resonant angular frequency of 107
radians/sec. The values of R and L are: V1
R = ________ohms. L = ______henries. [1995]
4Ω 1H
Solution:
100 cos 3t 5Ω
Z = R + j(XL – XC) = 100 + j0 ∼ V2
1 F
∴R = 100Ω, Q = wL and L = QR = 100 × 100 36
R ω 107
L = 1 mH
Solution:  From the given data, w = 3 rad/sec
18. A series LCR circuit consisting of R = 10 D, IXLI = 20
Ω and IXCI = 20 Ω is connected across an A.C. supply  Vin = 100 ∠0° V
of 200 V rms. The rms voltage across the capacitor is      Z1 = 4 + j × 3 × 1 = ( 4 + j 3) Ω = 5 ∠36.86°
(a) 200 ∠-90° V (b) 200 ∠ + 90° V 36
(c) 400 Z + 90° V (d) 400 Z - 90° V   Z2 = 5 − j × = ( 5 − j12 ) Ω = 13 ∠ −67.38° Ω
3
[1994]
Z2
Solution: (d) V2 = × Vin
Z1 + Z 2
∴XL = XC ⇒ circuit is at resonance
Z1
V 200 V1 = × Vin
=
I = = 20A    Z1 + Z 2
R 10
Chapter 2  Sinusoidal Steady State  |  1.29

V2 Z2 leading power factor and the second one draws 10 kVA


= at 0 8 lagging power factor. The complex power deliv-
V1 Z1
ered by the source is
13 (a) (18 + j 1.5) kVA (b) (18 - j 1.5) kVA
=
5 (c) (20 + j 1.5) kVA (d) (20 - j 1 5) kVA
[2014]
  = 2.6
Solution: (b)
Hence, the correct answer is (2.55 to 2.65).
Vrms = 230 V, Z ⇒ 10 kw at 0.8 leading
2. In the circuit shown, at resonance, the amplitude of
the sinusoidal voltage (in Volts) across the capacitor is Z 2 ⇒ 10 KVA at 0.8 lagging power factor

P1 = V1 I1 cos φ
______. [2015]
4Ω 0.1 mH
P1
I1 = = 54.34∠36.86
+ V1 cos φ

10 cos ω t 1μF
(Volts) – P = VI2
I 2 = 43.47∠ − 36.86°

  
Solution: 
We know for a RLC series circuit I = I1 + I 2 = 78.25 + j 6.25

1 L P = VI* = 230 (78.25 – j6.25)
Q=
R C 
P = (18 – j1.5) KVA
1 10 −4 10 Hence, the correct option is (b).
Q = = = 2.5
4 10 −6 4 
5. A periodic variable x is shown in the figure as a func-
VC = QV∠–90° tion of time. The root-mean-square (rms) value of x
is________. [2014]
|VC| = 2.5 × 10 = 25 V
Hence, the correct Answer is (24 to 26).
3. In the circuit shown, the average value of the voltage Vab
(in Volts) in steady state condition is _______. [2015]
1 μF
1 kΩ b a 1 mH 2 kΩ
– +
5π sin(5000t) Vab
5V

Solution: 
0.408

Solution:  At steady state redraw the given circuit


C – Open circuit
L – Shot circuit
1 kΩ – + 2 kΩ
Vab
b a
+ + 5V
0V
– –

T
1
0 + Vab + 0 – 5 V = 0 RMS Value =
T ∫
η 2 (t )dt
Vab = 5 V 0

Hence, the correct Answer is (4.9 to 5.1). T /2 2


1  2t 
4. A 230 V rms source supplies power to two loads con- =
T ∫
0
 T  dt
 
nected in parallel. The first load draws 10 kW at 0.8
1.30 | Network Theory

T /2 series, their effective Q factor at the same operating fre-


1 4
∫T quency is
2
= 2
t dt = 0.408
T
0
(a) q1 + q2
(b) (1/q1) + (1/q2)
6. The steady-state output of the circuit shown in the fig- (c) (q1R1 + q2R2)/(R1 + R2)
ure is given by (d) (q1R2 + q2R1)/(R1 + R2) [2013]
y(t) = A(ω)sin(ωt + φ(ω)). If the amplitude Solution: (c)
A(ω)| = 0.25, then the frequency ω is ω L1 ω L2
θ1 = , θ2 =
R1 R2

ω L1 = q1 R1 and ω L2 = q2 R2
∴ Q.R = ωL1 + ωL2 = q1R1 + q2R2
q1 R1 + q2 R2
q=
R1 + R2

2 Hence, the correct option is (c).
(a) 1 (b)
3RC 3RC 8. The current I in the circuit shown is
1 2
(c) (d)  [2014]
RC RC
Solution: (b)

(a) -j1 A (b) -j1 A


(c) 0 A (d) 20 A [2010]
Solution: (a)
VA
Applying KCL at A,
20mH
VA − sin wt V V
+ A + A =0 200
R 1 2 w=103 ~ 1 50F

jωC jωC I

2
VA =
2 + 3RCjw VA − 20∠0° VA V
+ + A =0
jω L 1 1
VA 1
Y = = jω C
2 2 + 3RCjω

− j j 
1 VA  +1+  = − j
A(ω ) =  20 20 
4
1 1 V = − j = 1∠ − 90°
A
=
4 4 + ( qw 2 R 2C 2 ) VA
I= = 1∠ − 90° A
1
2
w= Hence, the correct option is (b).
3 RC
9. An AC source of RMS voltage 20 V with internal
Hence, the correct option is (b). impedance Zs = (1 + 2j)Ω feeds a load of impedance
7. Two magnetically uncoupled inductive coils have Q fac- ZL = (7 + 4j)Ω in the figure below. The reactive power
tors q1 and q2 at the chosen operating frequency. Their consumed by the load is
respective resistance is R1 and R2. When connected in (a) 8 VAR (b) 16 VAR
(c) 28 VAR (d) 32 VAR [2009]
Chapter 2  Sinusoidal Steady State  |  1.31

11. For the circuit shown in the figure, the instantaneous


current i1(t) is

Solution: (b) 10 3
(a) ∠90o A
2
(b) 10 3
∠ − 90o A
2
(c) 5∠60° A
(d) 5∠-60° A [2005]
Solution: (a)

5∠0° + i1 = 10∠60°
20∠0° 10
I= = = 2∠ − 368i
(1 + 2 j ) + (7 + 4 j ) 4 + 3 j i1 = 10∠60° − 5∠0°

Reactive power = I2XL = 4 × 4 = 16 VAR 10 3
i1 = < 90° A
2
Hence, the correct option is (b).
10. In the AC network shown in the figure, the phasor volt- Hence, the correct option is (a).
age VAB (in volts) is 1 2. Consider the following statements S1 and S2
S1: At the resonant frequency the impedance of a series
R-L-C circuit is zero.
S2: In a parallel G-L-C circuit, increasing the conduct-
ance G results in increase in its Q factor.
Which one of the following is correct?
(a) S1 is FALSE and S2 is TRUE
(b) Both S1 and S2 are TRUE
(c) S1 is TRUE and S2 is FALSE
(d) Both S1 and S2 are FALSE [2004]
Solution: (d)
(a) 0 (b) 5∠30° Both the statements are false.
(c) 12.5∠30° (d) 17∠30° [2007] Correct S1: At resonant frequency, impedance is mini-
Solution: (d) mum for series R-L-C
Correct S2: In parallel G-L-C, increasing G decreases ϑ
factor.
Hence, the correct option is (d).
13. An input voltage v(t ) = 10 5 cos (t + 10º) + 10 5
(t + 10°) V is applied to a series combination of resist-
ance R = 1 Ω and an inductance L = 1 H. The resulting
steady-state current i(t) in ampere is
(a) 10cos(t + 55°) + 10cos(2t + 10° + tan- 12)

(b) 10cos (t + 55°) + 10 J – 10 3 cos (2t + 55)


2
(c) 10cos(t - 35°) + 10cos(2t + 10° - tan- 12)
(5 + 3 j )
VAB = 5∠30°× = 17∠30° (d) 10cos (t - 35) + 10 3 cos (2t - 35°)

(5 + 3 j + 5 − 3 j )
2
Hence, the correct option is (d). [2003]
1.32 | Network Theory

Solution: (c) Em cos wt


i2 (t ) =
v(t ) = 10 5 cos(t + 10°) + 10 5 cos(2t + 10°)
1
R2 +
jωC
R = 1Ω, L = 1H
V (t ) V (t ) V2 (t ) jωC
i (t ) = = 1 + i2 (t ) = Em ∠0° −
R + jω L R + jω L R + jω2 L 1 + jωCR2

10 5 cos(t + 10°) 10 5 cos(2t + 10°) Em ∠0°× ∠90°× ωC


i (t ) = + i2 (t ) =
2∠45° 5∠ tan −1 2 1 + W 2C 2 R 2 ∠ tan −1 ωCR2

Hence, the correct option is (c). E mωC
i2 (t ) = ∠90° − tan −1 ωCR2
1 4. When the angular frequency ω in the figure is varied 1+W C + R 2 2 2

from 0 to ∞, the locus of the current phasor is given by
When w = 0 i2(t) = 0, ∠90°
Em
When w = ∞ ⇒ i2 (t ) = ∠0°
R2

(a)

(b)
=0 E = Em0
Hence, the correct option is (a).
Em
2R2 1 5. In the given figure, A1, A2 and A3 are ideal ammeters. If
A1 reads 5A, A2 reads 12 A, then A3 should read
Em
2R2 I2 R
A1
= A3
C
(c) I2 A2
=0 =
~
Em Em Em 100 sin w
E = Em0
2R2 2R2 2R2
(a) 7 A (b) 12 A
(d)
=0
(c) 13 A (d) 17 A [1993]
Em E = Em0
2R2 Solution: (c)
Em Given I A= I= 5A
1 R
2R2
I A= I= 12A
Em I2 2 C

2R2 Resultant of current through resistor and capacitor will


[2001] be
=
Solution: (a) I = I R2 + I C2 = 13A
A3
Hence, the correct option is (c).
16. For the series R-L-C circuit in the below given figure
1, the partial fissure diagram at a certain frequency is a
shown in figure 2. The operating frequency of the cir-
cuit is
Chapter 2  Sinusoidal Steady State  |  1.33

VI
18. The resonant frequency of the series circuit shown in
VA
+ o + o figure is
+ M = 1H
+
V
o VC
o

2H 2H 2F
VR
1 1
V (a) Hz (b) Hz
4p 3 4p
1 1
(c) Hz (d) Hz
2p 10 4p 2
VC  [1990]
(a) equal to the resonance frequency Solution: (b)
(b) less than the resonance frequency M = 1H
(c) greater than the resonance frequency
(d) not zero [1992]
Solution: (b)
Since the current is leading voltage, it behaves as a
2H 2H 2F
capacitive circuit.
IR I
I= Equivalent inductance will be given as
=
V R
Leq = L1 + L2 – 2M
V
Leq = 2 + 2 – 2 = 2H
1 1 1
j= = = Hz
2π LC 2π 2 × 2 4π

VC
Hence, the correct option is (b).
So, voltage across capacitor will be greater than volt- 1 9. The half-power bandwidth of the resonant circuit of
age across inductor figure can be increased by:
VC > VL
IXC > IXL
R1
XC > XL
1 C R2
> ωL
ωC L
1
ω2 < ⇒ ω < wr
LC
(a) increasing R1 (b) decreasing R1
Hence, the correct option is (b).
(c) increasing R2 (d) decreasing R2
17. In a series RLC high Q circuit, the current peaks at a
[1989]
frequency
(a) equal to the resonant frequency Solution: (a) and (d)
(b) greater than the resonant frequency Selectivity ∝ Q

(c) less than the resonant frequency
(d) none of the above is true [1991] f
Q=
B.W
Solution: (a)
In series RLC circuit at resonance frequency, imped- f 1
Q = B.W or B.W ∝
ance is minimum. So, the current is maximum. Q
Hence, the correct option is (a).
1.34 | Network Theory

1
B.W ∝
selectivity

If R1 → 0 and R1 → ∞
then circuit will have only L and C element and has
selectivity.
HPBW can be reduced by decreasing selectivity and
vice versa.
[2002]
So, by increasing series resistance R1 and decreas-
Solution:
ing parallel resistance R2 the half power BW can be
(a) Let L5 be the equivalent inductance for the combi-
increased.
nation of L3 and L4 as shown in below figure.
Hence, the correct option is (a) and (b).

20. The value of current through the 1 Farad capacitor of


figure is

0.5 F
1 1 2
1F
~ 2 sin 100t
The coupling is series aiding
1 1 1H
L5 = L3 + L4 + 2M34
0.5 F
  = 1 + 4+ 4 = 9H
(a) zero (b) one (b) The circuit for calculating the equivalent induct-
(c) two (d) three [1987] ance Le across the points A and B is shown in below
figure.
Solution:  (a)

2. For the circuit shown in figure determine the


phasorsE2, E0, land l1.
 2  2
1 1 +  = 1 1 + 
 5  5
Circuit given is the same as a bridge circuit, as prod-
uct of opposite arms impedance is equal. So, current
through the diagonal element (1F) = 0.
Hence, the correct option is (a).
Solution:

Five-marks Questions
1. For network in figure, R = 1 kΩ, Ll = 2H, L2 = 5H1 L3
= 1H, L4 = 4H and C = 0.2µF. The mutual inductances
are M12 = 3 H and M34 = 2H. Determine
(a) the equivalent inductance for the combination of L3
and L4. For loop 2,
(b) the equivalent inductance across the points A and B E0
in the network. (0.5 + j 2) + E0 = I1 (1 − j )
(c) the resonant frequency of the network. 0.5
Chapter 2  Sinusoidal Steady State  |  1.35

⇒E0(2 + j4) = I1(1 – j)  (1) 4. A coil with a quality factor (Q) of 10 is put in series
KCL at node E2 with a capacitor C1 of 10 µF, and the combination is
found to draw maximum current When sinusoidal volt-
E0
⇒ I = I1 + (2) age of frequency 50 Hz is applied. A second capaci-
0.5 tor C2 is now connected in parallel with the capacitor.
E2 = I1(1 – j)  (3) What should be the capacitance of C2 for the combined
For loop 1, circuit to act purely as a resistance for a sinusoidal
E1 = jI + E2 = 10 20° (4) excitation at a frequency of 100 Hz? Calculate the rms
current drawn by the combined circuit at 100 Hz if the
⇒ JI + I1(1 – J) = 10ej20°(5) applied voltage is 100 V(rms). [1999]
From equation (2) and (5), Solution:
jE0
(1 − j ) I1 + jI1 + = 10e j 20° (6)
0.5
⇒ I1 + 2JE0 = 10eJ20°
⇒ I1 = 10ej20° – 2jE0(7)
From equation (1) and (7),
⇒ E0(2 + J4)=(10ej20° – 2JE0
⇒ E0(4 + J6)=(1 – J)10ej20° 1
5(1 − j ) j 20° f r = 50 Hz =
⇒ E0 = e 2π LC1
2 + j3  

1 1
=1.96e – j81.3°V(8) ⇒L= =
2
4π C1 (50) 2
4π × 10 × 10 −6 × 2500
2
From equation (1),
E0(2 + j4) = (1 – j1)I1 = 1.013 H.
2 + j4 ω0 L
⇒ I1 = E0 = 6.2e j 27.13° A (9) Q= = 10
1 − j1   R

From equation (3) and (9), ω0 L 2π × 50 × 1.013


⇒R= = = 31.8Ω
E2 = (1 – j)I1 10 10
= (1 – j)6.2ej27.13° Now,
= 8.77e – j17.9°

From equation (2) and (8)


E0
⇒ I = I1 + = I1 + 2 E0
0.5
   = 6.2e – j9.75°A
3. For the circuit in figure which is in steady state,
1
fr =
2π LCeq
  
1 1
⇒ Ceq = =
4π 2 Lf r2 4π 2 (1.013)(100) 2

⇒ Ceq = 2.5 μF = C1 + C2
(a) Find the frequency ω0 at which the magnitude of It is not possible
the impedance across terminals a, b reaches maxi- It means C2 is in series with C1.
mum.
(b) Find the impedance across a, b at the frequency ω0. C1C2
Ceq = = 2.5
(c) If Vi(t) = Vsin((ω0t), find iL(t), ic(t), iR(t). C1 + C2

 [2000]
1.36 | Network Theory

C2 × 10 1
⇒ = 2.5 ⇒ ω0 =
C2 + 10 LC

⇒ 10C2 = 2.5C2 + 25 1
⇒ f0 =
⇒ 7.5 C2 = 25 2π LC

25
⇒ C2 = = 3.33 µ F R ω02 RC 2
7.5 y= +
R 2 + ω02 L2 1 + ω02 R 2C 2
V 100   
∴ I rms = rms = = 3.14 A Ans
R 31.8 2 1
5. Determine the frequency of resonance and the resonant Put ω0 =
LC
impedance of the parallel circuit shown in figure. What RC
happens when L = CR2? R L
∴y = +
L = CR2? 2 L R 2C
R + 1+
  C L
RC RC 2 RC
 ⇒y= + =
R 2C + L R 2C + L R 2C + L
1 R 2C + L
⇒ Z = =
y 2 RC

  If, L = R2C
R 2C + R 2C
[1998] ∴Z = =R
   2 RC
Solution:
So, at L = R2C, the circuit resonant at infinite number of
1 R − jω L frequencies.
yL = =
R + jω L R 2 + ω 2 L2 6. In the circuit of figure all currents and voltages are

R ωL sinusoids of frequency ω rad/sec.
= 2 2 2
−j (a) Find the impedance to the right of (A, B) at ω = ∞
  R +ω + L R + ω 2 L2
2
rad/sec and ω = ∞ rad/sec.
1 jωC (b) If ω = ω0 rad/sec and i1(t) = Isin(ω0t)A1 where I is
yC = = positive, ω0 ≠ 0, ω0 ≠ ∞, then find I, ω0 and i2(t).
1 1 + jω RC
R+
jωC
jωC (1 − jω RC )
=
  1 + ω 2 R 2C 2

ω 2 RC 2 ωC
2 2 2
+j 2 2 2
  1 + ω R C 1 + ω R C
[1997]
⇒ y = yL + yC
Solution:
For resonance, Im(y) = 0
(a) At w = 0
−ω L ωC XL = ωL = 0 short circuit
⇒ + =0
R 2 + ω 2 L2 1 + ω 2 R 2C 2
1
XC = = ∞ ⇒ open circuit
2 2 2 2 2 2
ωc
⇒ L(1 + ω0 R C ) = C ( R + ω0 L )
⇒ RAB = 1||1 = 0.5Ω
2 2 2 2 2 2
⇒ L + ω0 R C L = R C + ω0 L C At ω → ∞
ω20 LC (R2C – L) = R2 C – L XL = ωL → ∞, ⇒ open circuit
1 1
⇒ ω02 = XC = = 0 ⇒ short circuit
LC ωC
Chapter 2  Sinusoidal Steady State  |  1.37

⇒ RAB = 1||1 = 0.5 Ω RCs + 1


(b) ω = ω0 = resonant frequency 2R
⇒ Cs =
1 1 R R
y AB = + +1
1 + 4s 4 RCs + 1
1+
s ⇒(RCs + 1)2 = 2RCs
8s 2 + 19 s + 8 ⇒1 + R2 C2 s2 = 0
y AB =
4 s 2 + 17 s + 4 −1
⇒ s2 =
R 2C 2
−8ω02 + 8 + j19ω0
=
−4ω02 + 4 + j17ω0 −1
⇒ ( jω ) 2 =
R 2C 2
yAB is purely real if ω0 = 1
−1
i.e., y AB =
19 17
⇒ Z AB = Ω ⇒ −ω 2 =
17 19 R 2C 2

V1 (t ) 190 1
i1 (t ) = = sin ωt = I sin ωt ⇒ω =
Z AB 17 RC

190 1
or f =
⇒I = A 2π RC
17
8. Write down the mesh equations of the following net-
V (t ) V (t ). jω0 10 sin ωt .i work in terms of i1(t) and i2(t). Derive the differential
i1 (t ) = = =
4 4 + jω0 4+i equation for i1(t) from these and Sove it.
1+
jω0
10 10

10 t=0
= sin(ωt + 76°) A +
17 10e-t i1(t) 1H i2(t) 5
-

7. Calculate the frequency at which zero – transmission is


obtained from the Wien – bridge shown in figure.
[1994]
Solution: For loop 1,
di1 (t ) di2 (t )
10i1 (t ) + − = 10e −t (1)
dt dt
For loop 2,
di2 (t ) di1 (t )
15i2 (t ) + − = 0 (2)
dt dt
Adding both equations,
10i1 (t) + 15i2(t) = 10e–t
10e −t 10
[1994] ⇒ i2 (t ) = − i1 (t ) (3)
15 15
1
Solution: Z1 = R + , Z = 2R Substituting i2(t) in equation (1), we get
Cs 2
1 di1 (t ) 10 de −t 10 di1 (t )
R× 10ii (t ) + − + = 10e −t
Cs R dt 15 dt 15 dt
Z = = ,
3 1 RCs + 1 Z4 = R
R+ 5 di1 (t ) 10 de − t
Cs ⇒ + 10i1 (t ) = + 10e − t
For balance condition, 3 dt 15 dt
Z1 Z 2 di1 (t ) 1  dVi (t ) 
= ⇒ + 6i1 (t ) =  + 15V1 (t ) 
Z3 Z 4 dt 25  dt 

1.38 | Network Theory

And, Vi(t) = 10e−t.u(t) 2( s + 15)


Take the laplace transform, ⇒ I1 ( s ) =

(5)( s + 6)( s + 1)
1 10
⇒ ( s + 6) I1 ( s) = ( s + 15).  2.8 1.8 
25 s +1 = − 0.4
 s + 1 s + 6 

= (1.12 e−t – 0.72 e−6t) v(t)
Chapter 3
Network Theorems
2. In the given circuit, the maximum power (in Watts) that
One-mark Questions can be transferred to the load RL is _______. [2015]

1. In the circuit shown below, Vs is a constant voltage
source and IL is a constant current load.
j2 Ω RL
R 4 ∠0 Vms
+
Vs IL

Solution:  Find the Thevenin equivalent circuit at the


The value of IL that maximizes the power absorbed by terminal of RL
the constant current load is [2016] Zth
Vref Vs
(A) (B)
2R 2R +
V Vth I RL

(C) s (D) –
R
Solution: 

R
+ + Zth = (2 || j2) = (1 + j1) Ω = 2 ∠45° Ω 
Vs IL VL
– – RL = |Zth| = 2 Ω

4 × j2
Load voltage can be expressed as Vthrms = = 2 + 2i = 2 2 ∠45° V
2 + j2
VL = VS – ILR
Vth 2 2 ∠45° 2 2 ∠45°
Load power is PL = VL ⋅ IL I rms = = =
Zth + Rth 1 + j1 + 1.414 2.414 + j1

PL = (VS – ILR) IL
2 2∠45°
To get IL that maximize the power observed by the load I rms = = 1.082 ∠22.5° Amp
2.613∠22.5
dPL
=0 1

dI L Pmax = |I|2.RL = |Irms|2 .RL
2
VS – 2ILR = 0
= 1.655 watts
2ILR = VS
Hence, the correct Answer is (1.6 to 1.7).
Vs
IL = 3. In the circuit shown, the Norton equivalent resistance
2R 
(in Ω) across terminals a – b is _____.
Hence, the correct option is (B).
1.40 | Network Theory

2Ω 5Ω 5Ω
a
I
+
4I 2Ω 4Ω + 10 Ω
– 5V 25 1A

I
b

Solution:  The given network consists only dependent [2014]


source, connect one test source and find the equivalent Solution: 0.5 A
resistance across the test source.
5Ω 5Ω
2Ω a It J
I
(I + 1)
+ 10 Ω
+ 5V − 1A 25
4I 2Ω 4Ω +
– – Vt
I

b
Req Applying KVL,
5 = 5I + 5(I + 1) + 10(I + 1)
Vt
Req = RN = 5 = 20I + 5 + 10
It
 |I| = 0.5A
Applying Nodal analysis 6. A source vs (t) = wcos 100 πt has an internal impedance
Vt − 4 I Vt of (4 + j3)W. If a purely resistive load connected to this
+ + I = It source has to extract the maximum power out of the
2 2  source, its value in W should be
Vt – 4I + Vt + 2I = 2It  (a) 3 (b) 4
(c) 5 (d) 7 [2013]
⇒ 2Vt – 2I = 2It Solution: (c)
Vt R = RS2 + X S2 = 5Ω
but I= L
4 
Vt Hence, the correct option is (c)
⇒ 2Vt – = 2It
2 7. The impedance looking into nodes 1 and 2 in the given
3Vt = 4It circuit is
ib
Vt 4
=
It 3

1k Ω 99i
RN = 1.33 Ω
b

9kΩ
Hence, the correct Answer is (1.3 to 1.35).
1
4. Norton’s theorem states that a complex network con- 100 Ω
nected to a load can be replaced with an equivalent 2
impedance
(a) in series with a current source
(b) in parallel with a voltage source (a) 50 W (b) 100 W
(c) in series with a voltage source (c) 5 kW (d) 10.1 kW
(d) in parallel with a current source [2014]  [2012]
Solution: (d) Solution: (a)
In parallel with current source 1k Ω A iAB
B
Hence, the correct option is (d) i i
Ith
b A

9k Ω 100k Ω
5. In the figure shown, the value of the current I (in 99i b 1V
Amperes) is_________.
Chapter 3  Network Theorems  |  1.41

Applying KCL, iAB + 99ib = Ith (a) 5 W (b) 10 W


ib = iA + iAB (c) 15 W (d) 20 W [2011]
∴ 100ib – iA = Ith Solution: (c)
Applying KVL in outer loop, For MPT, RL = Rth
10 × 103ib = 1 10 Ω 10 Ω
ib = 10−4 A
iA = −100ib 10 Ω
∴ 100ib + 100ib = Ith Rth

Ith = 200ib = 0.02


1
Z in = = 50 Ω Rth = 10 + 10 || 10 = 15Ω
I th
Hence, the correct option is (c)
Hence, the correct option is (a) 1 0. An independent voltage source in series with an imped-
8. In the circuit shown below, the Norton equivalent cur- ance Zs = Rs + jXs delivers a maximum average power to
rent in amperes with respect to the terminals P and Q is a load impedance ZL, when
(a) ZL = Rs + jXs
j30 Ω
(b) ZL = Rs
P (c) ZL = jXs
(d) ZL = R – jXs [2007]
0
16 0 A 25 Ω −j50 Ω
Solution: (d)
For MPT, Z L = Z S* = RS − jX S
Q
15 Ω Hence, the correct option is (d)
1 1. The maximum power that can be transferred to the load
25
I sc = 16 ∠0o = 8∠ 36.86 resistor RL from the voltage source in the figure is
15 + 30 j + 25
100
(a) 6.4 – j4.8 (b) 6.56 – j7.87
(c) 10 + j0 (d) 16 + j0 [2011]
Solution: (a)
In order to find the Norton equivalent, we have to find
ISC.
j 10V RL

j30 Ω
Isc
16 0
0
25 Ω −j50

15 Ω (a) 1 W (b) 10 W
Hence, the correct option is (a) (c) 0.25 W (d) 0.5 W [2005]
9. In the circuit shown below, the value of RL such that the Solution: (c)
power transferred to RL is maximum is Vth V2 100
= = th = = 0.25 W
10 Ω 10 Ω 4 RL 4 RS 4 × 100
Hence, the correct option is (c)
10 Ω RL 1 2. A source of angular frequency 1 rad/sec has a source
impedance consisting of 1 W. resistance in series with
1 H inductance. The load that will obtain the maximum
+ + power transfer is
− 5 − 2V 1A
(a) 1 W resistance
(b) 1 W resistance in parallel with 1 H inductance
1.42 | Network Theory

(c) 1 W resistance in series with 1 F capacitor Ideal


(d) 1 W resistance in parallel with 1 F capacitor 8Ω transformer
 [2003]
Solution: (c) 40
V0 2Ω
turns
Z1 =Z S* = 1 − j (1Ω || 1F )
Hence, the correct option is (c)
(a) 20 (b) 40
13. Superposition theorem is NOT applicable to networks (c) 80 (d) 160 [1993]
containing
Solution: (c)
(a) non–linear elements
2
(b) dependent voltage sources  n2  ZL
(c) dependent current sources   =
(d) transformers [1998]  n1  ZS
2
Solution: (a)  n2  2 1
  = +
Super position works on principle of linearity, so is  n1  8 4
applicable to linear network only.
n2 1
Hence, the correct option is (a) =
n1 2
14. The value of the resistance R, connected across the ter-
n1 = 2n2 = 2 × 40 = 80.
minals, A and B, (see the figure), which will absorb the
maximum power, is Hence, the correct option is (c)

3 kΩ 4 kΩ Two-marks Questions
A B 1. In the circuit shown in the figure, the maximum power
(in watt) delivered to the resistor R is _____. [2016]
6 kΩ 10 kΩ
4 kΩ
3 kΩ
(a) 4.00 kW (b) 4 11 kW
(c) 8.00 kW (d) 9 00 kW [1995] +
+ 100 V
+ 5V 2 kΩ V0 40 kΩ R
Solution: (a) – –

3 kΩ 4 kΩ

A B
Solution: 
R 10 kΩ
6 kΩ 4 kΩ
+
200 V 40 kΩ R

RAB = 3||6 + 4||4 = 4 kΩ
Hence, the correct option is (a)
15. A generator of internal impedance, ZG, delivers maxi-
Maximum power transfer theorem states that a network
mum power to a load impedance, ZL, only if ZL = .........
will transfer maximum power to load if load impedance
 [1994]
will be equal to its internal impedance, i.e., Rth. To find
Solution: RG – jXG Rth disconnect the load and deactivate all independent
Z L = ZG* sources and find the equivalent impedance across the
open circuit.
ZG = RG + JX G
VOC
so Z L = RG − JX G Rth =
I SC
16. If the secondary winding of the ideal transformer 
40
shown in the circuit of figure has 40 turns, the number VOC = 200 x
of turns in the primary winding for maximum power 50 
transfer to the 2 W resistor will by =
160 V
Chapter 3  Network Theorems  |  1.43

ISC = 20 mA 3. In the circuit shown, the voltage Vx (in Volts) is ________.
 [2015]
160
Rth = = 8k Ω 5 Vx
20 
8 kΩ

+
160 V R 10 Ω

+
+
5A Vx 20 Ω 8Ω 0.25 Vx
2 –
⎛ 160 ⎞ –
p=⎜ xR
⎝ 8 + R ⎟⎠
Solution: 
For maxima or minima
0.5 Vx
dP
=0
dR
⎡ (8 + R )2 − 2 R (8 + R ) ⎤
⎢ ⎥=0

⎢⎣ (8 + R)2 ⎥⎦

10 Ω

R=8Ω +
+
Vx 20 Ω 8Ω 0.25 Vx
Hence, at load impedance of 8 ohm network will trans- 5A

fer maximum power –

pmax =
(160)2 =
160 × 160 800
= = 0.8 W
4 x8 4×8 1000 Redraw the given network
0.5 Vx
Hence, the correct Answer is (0.8).
Vx 10 Ω Vx/4
– +
2. For the circuit shown in the figure, the Thevenin equiva-
lent voltage (in Volts) across terminals a–b is _______. + Vx
 [2015] 5A 20 Ω 8Ω
– 4

3Ω a

12 V 1A 6Ω
Applying Nodal analysis.
b
Vx Vx + 5Vx − 0.25Vx
–5 + + =0
Solution:  20 10 

a
100 = Vx + 2{5.75 Vx}

12 V 1A 6 Ω Vth Vx = 8 V
Hence, the correct Answer is (7.95 to 8.05).
b
4. In the circuit shown in the figure, the angular frequency
Apply Nodal analysis:
w (in rad/s), at which the Norton equivalent imped-
Vth − 12 V ance as seen from terminals b-b' is purely resistive,
− 1 + th = 0 is______.
3 6
1Ω 1F
2(Vth – 12) – 6 + Vth = 0 b
+
3Vth – 24 – 6 = 0 0.5H
0 cosωt (Volts)

Vth = 10 volts -
b

Hence, the correct Answer is (10).


[2014]
1.44 | Network Theory

Solution: Solution: (a)


Zth = ? 7
Current through       R=
Z = 1× j 0.05ω 1 2+ R
th +
1 + j 0.5ω jω 3
Current through capacitor  =
j
2 − ω 2 + jω
Zth =
2 jω − ω 2  7  −3
Current through 3V source =  
 2+ R  j
 2ω 2 + ω 4 + 2ω 2   ω 3 − 4ω 
Zth = −   + j 4 
 ω 2 + 2ω 2  ω + 2ω 2 7
    = +3j
Put Jm (Zth) = 0   ( 2 + R)
∴ ω = 2 rad/s Power delivered from circuit B to A
2
5. In the circuit shown below, if the source voltage Vs =  7   7 
P =  R+ − 3 j 3
100∠53.13°V then the Thevenin’s equivalent voltage in    2+ R   2+ R 
volts as seen by the load resistance RL is
∂P
3 j4 j6 3 = 0 ⇒ R = 0.8Ω
+ _ ∂R
VLI Hence, the correct option is (a)
V1 ~ I1 j40I2 +
_ +
_ 10VL1 I R = 10 
2 L
7. In the circuit shown, what value of RL maximizes the
power delivered to RL?
(a) 100∠90° (b) 800∠0° Vx
4Ω
(c) 800∠90° (d) 100∠90° [2013] _ +

Solution: (c) 4Ω 4Ω
For finding, Vth, I2 = 0 _
Vx +
+
V1 _ 100 V RL

8
(a) 2.4 W (b) W
3
j4
VL1 = × Vs = 80∠90° (c) 4 W (d) 6 W [2009]
j4 + 3
Solution: (c)
Vth = 10VL1 + (3 + 6 j ) I 2 = 800∠90° For maximum power transfer, RL = Req

Hence, the correct option is (c) Applying a source of 1V, we have
6. Assuming both the voltage sources are in phase, the Vx
4Ω
_ + I1
value of R for which maximum power is transferred
from circuit A to circuit B is
4Ω 4Ω I
2Ω R I − I1 +
1Y
+ + _
10V −j1Ω 3V
_ _
1 = 4I1 + Vx
Circuit A Circuit B Vx = 4(I – I1)
1
∴ Req = = 4−2
(a) 0.8 W (b) 1.4 W I
(c) 2 W (d) 2.8 W [2012] Hence, the correct option is (c)
For RTH :- V
X
1Ω
L
+ 1Ω 2Ω 1A
2i _

Y
Chapter 3  Network Theorems  |  1.45

8. The Thevenin equivalent impedance Zrh between the For Vth:–


nodes P and Q in the following circuit is V+1
X
1H IF 1Ω
i
+ 1Ω 2A 2Ω
2i _
1Ω P
1Ω
Y
Q
+
10V - 1A
Vth Vth Vth − 2i
2= + + , i = Vth
2 1 1
1
(a) 1 (b) 1+ s + Hence, the correct option is (d)
s
10. For the circuit shown in the figure, Thevenin’s voltage
1 s2 + s + 1 and Thevenin’s equivalent resistance at terminals a–b
2 + s + (d)
(c)
s s2 + 2s + 1 are
 [2008] 5Ω

Solution: (a) I1
+ a +
 1 0.5l1 5Ω _10V
Zeq = ( s + 1) || 1 +  = 1 b
 s _

Hence, the correct option is (a)
9. For the circuit shown in the figure, the Thevenin volt- (a) 5 V and 2 W (b) 7.5 V and 2.5 W
age and resistance looking into X–Y are (c) 4 V and W (d) 3 V and 2.5 W[2005]
1Ω Solution: (b)
X 5Ω
I
+ I1
2i _ 1Ω 2A 2Ω + +
a
0.5l1 5Ω _10V
b
_
Y

(a) 4/3 V, 2 W (b) 4 V, 2/3 W


(c) 4/3 V, 2/3 W (d) 4 V, 2 W [2007] Vab Vab − 10 5Ω
+ =1 5Ω
Solution: (d) 5 5

For RTH :- Vab = 7.5 V I1


V
X 10+Ω 5Ω a +
1Ω 0.5 J0.5l
1 1 5 5Ω _10V
L =_ ⇒ R = 10Ω b
+ J1 R
2i _ 1Ω 2Ω 1A
Rth:–
Y 5Ω

1 + 2i − V = V + V
10 Ω 5Ω
2 V+1
X
1 + 2i = 2.5 V 1 Ω
i
i = V ⇒ 1++ 20 = 2.5
1Ω
V 2A 2Ω
2i _
0.5V = 1 Rth = 5||5 = 2.5Ω
V = 2 Hence, the correct option is (b)
Y
V 11. In the network of the figure, the maximum power is
Rth = = 2Ω delivered to RL if its value is
1
1.46 | Network Theory

I1 13. Use the data of the figure (a). The current I in the circuit
of figure (b) will be ____.
40 Ω
R2 R2
0.5 l1 20 Ω RL +
R1 R3 R1 R3
50V
− + +
R4 20V
10V 2A i =? R4

40 (a) (b)
(a) 16 W (b) Ω
3 (a) –2 A (b) 2 A
(c) –4 A (d) + 4 A [2000]
(c) 60 W (d) 20 W [2002]
Solution: (c)
Solution: (a)
Since the N/W is reciprocal and linear, doubling voltage
Short–circuit the independent voltage source and apply
will double current
1A sauce in place of RL
i = −4A.
I1 14. The Thevenin equivalent voltage VTH appearing
40 Ω
between the terminals A and B of the network shown in
0.5l1 20 Ω IA the figure is given by
3Ω A

100 0 V
V 0
0.5 J1 + 1 = I1 + j2 −j6 j4 VTH
20
V
0.5 J1 + =1 B
20
V V (a) j16(3 – j4) (b) j16(3 + j4)
0.5 + =1 (c) 16(3 + j4) (d) 16(3 – j4) [1999]
40 20
V + 4V = 80 Solution: (a)
3Ω
V = 16 V A

V
Rth = = 16Ω
1
0
100 0

j2 −j6 j4 VTH
Hence, the correct option is (a)
12. In the figure, the value of the load resistor R which
B
maximizes the power delivered to it is
V = 100∠0°
10 Ω 1H
4j
Vth = 100∠0°×
3+ 4 j
Emcos10t

RL Vth = 16j (3 – 4j)


Hence, the correct option is (a)
1 5. The value of R (in ohms) required for maximum power
transfer in the network shown in the figure is
(a) 14.14 W (b) 10 W
5Ω 4Ω
(c) 200 W (d) 28.28 W [2001]
Solution: (a)
RL = R 2 + X L2 for MPT +
25V 20 Ω 3A R

RL = 100 + 100 = 14.14Ω

Hence, the correct option is (a)


Chapter 3  Network Theorems  |  1.47

(a) 2 (b) 4 (a) 1 W (b) 3 W


(c) 8 (d) 16 [1999] (c) 3 W (d) 5 W [1989]
Solution: (c) Solution: (a)
Rth = 4 + (5 || 20) = 8Ω Given
P1 = 1 W
5Ω 4Ω
P2 = 1 W
Since the polarity of both the sources is different
2
R P = ( P1 − P2 )
20 Ω 3A
2 2
P = ( 1 − 4 ) = (1 − 2)
P =1W

Hence, the correct option is (c) Hence, the correct option is (a)
1 6. In the circuit of the figure given below, when switch 18. A load, ZL = RL + jXL, is to be matched, using an ideal
S1 is closed, the ideal ammeter M1 reads 5A. What will transformer, to a generator of internal impedance, Zs =
the ideal voltmeter M2 read when S1 is kept open ? (The Rs + jXs. The turns ratio of the transformer required is
value of E is not specified.) [1993] |Z L /RS |
(a) |Z L /Z S | (b)
M2
(c) |Z L /RS | (d)
|Z L /RS |  [1989]
V
Solution: (a)
4Ω 3Ω 2
Z L  n2  n ZL
S1 =  ⇒ 2 =
6Ω Z S  n1  n1 ZS
A M1
− + 10 Ω Hence, the correct option is (a)
E 8Ω 5Ω 19. If an impedance ZL is connected across a voltage source
V with source impedance Zs, then for maximum power
transfer the load impedance must be equal to
2Ω 3Ω
(a) source impedance Zs
Solution: 50 V (b) complex conjugate of Zs
(c) real part of Zs
Across switch S1, Isc = 5 amp (d) imaginary part of Zs [1988]
We have to find Vth Solution: (b)
Vth = ISC Rth For maximum power transfer ZL = ZS*
ISC = 5A Hence, the correct option is (b)
Rth = [(4 || 6 + 2 || 8) + 3 + 3] || 10 + 5
Rth = 10Ω Five-marks Questions
Vth = 10 × 5 = 50 V.
1 7. In the circuit of figure, the power dissipated in the resis- 1. For the circuit in figure.
tor R is ‘1’ W when only source '1' is present and ‘2’ is
replaced by a short. The power dissipated in the same
resistor R is 4 W when only source '2' is present and ‘1’
is replaced by a short. When both the sources ‘1’ and
‘2’ are present, the power dissipated in R will be :.
1Ω 2Ω
V1 e jul volts

V2 e jul volts


+
SOURCE R =1 Ω (a) Find the Thevenin equivalent of the sub circuit faced
+ 1 by the capacitor across the terminals A, B.
SOURCE −
2 (b) Find Vc(t), t > 0, given vc(0) = 0.
(c) Find i(t), t > 0. [2000]
1.48 | Network Theory

Solution: (a) Vc(t) = V∞ + (1i – V∞) e–t/∞


2 = 2 + (0 – 2)e–t/b
VAB = Vth = × 4 = 2V i.e. Thevenin voltage
2+2 = 2(1 – e–t/6)V Ans.
2 || 2 1 4
And, Rth = 2||2 + 2 = 1 + 2 = 3Ω i.e. Thevenin Resistance (c) V+ = ×4 = ×4 = V
(b)
0 2 || 2 + 2 1+ 2 3
V 2
0+
i = = A
0+ 2 3

4
i∞ = = 1A
4
τ = RC = 6 sec
i(t) = i∞ + (i0+ – i∞)e–t/τ
VC(σ) = VC(0+) = OV
2 
Vc(∞) = 2V = 1 +  − 1 e −t / b
3 
τ = RthC = 3 × 2 = 6 sec. (Time constant)
1
= 1 − e −t / 6 Aa
3
Chapter 4
Transient Analysis
Vc(t) = Vf + (Vi – Vf) e–t/T
One-mark Questions
= 10 + ( 4 − 10)e − t /T 

1. The switch has been in position 1 for a long time and Now consider the figure given below
abruptly changes to position 2 at t = 0.
4Ω 2Ω
3Ω 1 2 4Ω 2Ω

t=0 2Ω
+ +
10 V 2Ω Vc 2Ω 5A

0.1 F –
Req
Req = 4 + 2 = 6 Ω
If time t is in seconds, the capacitor voltage Vc (in volts) Time constant
for t > 0 is given by [2016]
(A) 4(1 – exp (–t/0.5)) T = RC = 6 × 0.1 = 0.6 sec
(B) 10 – 6 exp (–t/0.5) The voltage across capacitor can be expressed as
(C) 4(1 – exp (–t/0.6) Vc(t) = 10–6 ⋅ e(–t/0.6)
(D) 10 – 6 exp (–t/0.6) Hence, the correct option is (D).
Solution:  As we know that capacitor gets S.C. when 2. In the circuit shown, the current I flowing through the
AC supply is used and remain O.C when the supply is 50 Ω resistor will be zero if the value of capacitor C (in
in form of DC. Here the voltage appearing across C will µF) is ______. [2015]
not die out immediately after switching, instead it will
decay exponentially as given in the expression below. 50 Ω 1 mH 1 mH

3Ω I
+ 1 mH
5π sin(500t) C

10 V 2Ω VC
Solution: 
Given network is
– 50 Ω 1 mH 1 mH
I
For time t < 0:
VC = 10 × 2/5 = 4V
V(t) 1 mH C
Vc(0–) = 4 V
For t > 0 :
at t = 0 + :
Zeq
Vc (0 + ) = 4 V

For time t → ∞ Find the equivalent impedance in S-domain

Vc = 5 × 2 = 10 V Z(s) = (R + SL) + {SL || (SL + 1 )}


SC
1.50 | Network Theory

Let Z(s) = Z1 + Z2 ⇒ Vc(t) = Vc(∞) + {Vc(0–) – VC (∞)}. e − t τ 

Z2 =
{
SL SL + 1 }2
SC = SL S LC + 1{ } τ = Req.C
Req = (3||2) = 1.2 Ω
SL + SL + 1 2S 2 LC + 1
SC  5
But, given I = 0 C= F
6 
So Z2(s) = ∞ or open circuit
5 6
∴ 2S2 LC + 1 = 0 RC = ×
6 5
Sub S = jω and L value
τ = 1 sec
ω = 5000
Vc(t) = 4 + {0 – 4}.e–t/1
–2 ω2LC + 1 = 0
Vc(t) = 4 {1 – e–t} volts
1
LC = at t = 1 sec
2ω 2 
Vc(1) = 4 {1 – e–1} = 2.528 V
1
C=
2 Lω 2  Hence, the correct Answer is (2.48 to 2.58).
1
C= −3 4. For maximum power transfer between two cascaded
2 × 1 × 10 × 25 × 106 
sections of an electrical network, the relationship
C = 0.02 × 10–3 F between the output impedance Z1 of the first section to
the input impedance Z2 of the second section is
C = 20 µF
(a) Z1 = Z1 (b) Z2 = -Z1
Hence, the correct Answer is (20). (c) Z2 = Z1* (d) Z2 = -Z1*
3. In the circuit shown, switch SW is closed at t = 0.  [2014]
Assuming zero initial conditions, the value of Vc(t) (in Solution: (c)
Volts) at t = 1 sec is _____ [2015] Z 2 = Z1* for MPT
t=0
3Ω Hence, the correct option is (c).
+
SW
5. In the figure shown, the ideal switch has been open for
10 V 2Ω 5 vc(t)
6
F a long time. If it is closed at t = 0, then the magnitude
– of the current (in mA) through the 4 kW resistor at t =
0+ is_________. [2014]
Solution:  Given all initial conditions are zero
5kΩ 4kΩ 1kΩ
For t < 0, switch was opened
So VC (0–) = 0 V
10V + 10 µF t=0 1mH
-
For t > 0; switch was closed
at t = 0+
Solution: 1.25 mA
VC (0+) = VC(0–) = 0 V
At t = 0−
If t → ∞
5kΩ 4kΩ 1kΩ
circuit is in steady state
3Ω i
+ 10V

10 V 2Ω VC(∞ )

– 10 V
i= = 1 mA
10 kΩ
2 × 10
VC (∞) = = 4V At t = 0 switch gets
2+3 5kΩclosed4kΩ 1kΩ
 V

10V 5V
5kΩ 4kΩ 1kΩ

i
10V

Chapter 4  Transient Analysis  |  1.51

5kΩ 4kΩ 1kΩ 1K


V

+ +
10V 5V

4 µF
VI(t) 4K 1 µF VI(t)

5 – –
I= = 1.25 mA
4
(a) 8e-1/0.004 volts
6. In the following figure, C1 and C2 are ideal capacitors.
(b) 8(1 - e-t/0.004) volts
C1 had been charged to 12V before the ideal switch S is
closed at t = 0. The current i(t) for all t is (c) 8 u(t) volts
(d) 8 volts [2006]
S t=
0
Solution: (c)
As R1C1 = R2C2
Z2
C1 C2
V0 = V1 (t ) = 0.8 ×10 u(t )
i(t) Z1 + Z 2
V0 = δ u( −1)
Hence, the correct option is (c).
(a) zero 9. A 2 mH inductor with some initial current can be
(b) a step function represented as shown below where s is the Laplace
(c) an exponentially decaying function Transform variable. The value of initial current is
(d) an impulse function [2012]
Solution: (d)
I(s)
R = 0 ⇒ T = 0 ⇒ capacitor allows sudden change of
voltage through it when impulse response is found i.e.
excitation = δ(t) 0.002s

Hence, the correct option is (d).

7. In the following circuit, the switch S is closed at



di +
t = 0. The rate of change of current (0 ) is given by
dt ImV
+
Rs I s
(a) 0 (b) L

( R + Rs ) I s (a) 0.5 A (b) 2.0 A


(c) ∞
(d) [2008]
L (c) 1.0 A (d) 0.0 A [2006]

Solution:(b) Solution: (a)


di + di
VL = L (0 ) V =L
dt dt
di + V I R V(s) = sLI(s) – Li(0 + )
(0 ) = L = S S
dt I L −Li(0 + ) = −1 mV ⇒ i(0 + ) = 0.5 A
Hence, the correct option is (b). Hence, the correct option is (a).
8. In the figure shown below, assume that all the capaci-
10. For the R-L circuit shown in the figure, the input volt-
tors are initially uncharged. If v1(t) = 10 u(t) volts, v0(t)
age v1(t) = u(t). The current i(t) is
is given by
- +
Vx

1.52 | Network Theory

1H 1 1 1  1
I ( s) = = − ×
s( 2 + s)  s s + 2  2

V(t) i(t) 2Ω 1
i (t ) = 1 − e −2t
2
At t = 0, i(t) = .5
Hence, the correct option is (c).
(a) 1 1. In the figure, the switch was closed for a long time
before opening at t = 0 The voltage Vx at t = 0 + is
t=0

2.5A

20 Ω

5H

(b)
20 Ω
– +
Vx

(a) 25 V (b) 50 V
(c) -50 V (d) 0 V [2002]
Solution: (c)
Steady state, iL(0−) = 2.5A
V
(c)
2.5A

20 Ω

2.5A

20 Ω

– V +

∴V = −2.5 × 20 = −50V
(d)
The circuit is given. Assume that the switch S is in posi-
tion 1 for a long time and thrown to position 2 at t = 0.
Hence, the correct option is (c).
12. A ramp voltage, v(t) = 100 t volts, is applied to a RC
differentiating circuit with R = 5 kΩ and C = 4µF. The
maximum output voltage is
(a) 0.2 volt (b) 2.0 volts
(c) 10.0 volts (d) 50.0 volts
 [1994]
 [2004] Solution: (b)
Solution: (c) dv
= = 2V
V RC
1 dt
V1 (t ) = U (t ) → V1 ( s) =
s Hence, the correct option is (b).
Chapter 4  Transient Analysis  |  1.53

The current i(t) (in ampere) at t = 0.5 seconds is


Two-marks Questions __________.[2017]
1. For the circuit given in the figure, the magnitude of 3. The switch S in the circuit shown has been closed for a
the loop current (in amperes, correct to three decimal long time. It is opened at time t = 0 and remains open
places) 0.5 second after closing the switch is ________. after that. Assume that the diode has zero reverse cur-
[2018] rent and zero forward voltage drop.
1V 1 kΩ
– + t=0

S –
1Ω 1H
10 V +
– 1 mH 10 μF VC
+

Solution:  Consider the figure given below The steady state magnitude of the capacitor voltage Vc
200 kΩ
(in volts) is _____.[2016]
Solution:  Inductance L = 1 mH,
Voltage Vc = 10 × 10–6 F,
+ Resistance R = 1 ohm
1V 1H
– Energy from inductor transfers to capacitor
1 2 1
LI = CV 2
2 2 
As per circuit diagram given above we have 1 1
× 10 −3 × 100 = × 10 × 10 −6 × Vc2
IL (o–) = IL (o+) = 0 2 2 
For time t > 0: Vc = 10 4 = 100 volts 

As t → ∞
1 Hence, the correct Answer is (100 volts).
IL (∞) =
A 4. In the circuit shown, the switch SW is thrown from
2
position A to position B at time t = 0. The energy (in µJ)
L 1
t = = = 0.5 sec taken from the 3 V source to charge the 0.1 µF capaci-
R 2  tor from 0 V to 3 V is [2015]
We know that: SW
3V 120 Ω B A
i(t) = IO (1– e–t /t  )Amp
t=0
=
0.5 (1 –e–t/0.5) 0.1 μF
At t = 0.5 sec
IL (0.5) = 0.5 (1– e–1) (A) 0.3 (B) 0.45
= 0.316 (C) 0.9 (D) 3
Hence, the correct answer is 0.284 to 0.348. Solution:  For t < 0:
2. The switch in the circuit, shown in the figure, was open at t = 0 , the switch connected to position A

for long time and is closed at t = 0 So, VC (0–) = 0 V.

For t > 0:

i(t) at t = 0+. The switch was connected to position B.
VC(0+) = VC(0–) = 0 V
t=0

↑ 10 A If t → ∞,.VC(∞) = 3 V
2.5 H τ = Rc = 120 × 0.1 × 10–6 = 12 µ sec

{ } { }
VC(t) = Vo 1 − e − t τ = 3. 1 − e − t τ volts
1.54 | Network Theory

dVc (t )
iC(t) = C .
dt
⎧d⎫
{ }
= 3 × 0.1 × 10 −6 ⎨ ⎬ 1 − e − t τ
⎩ dt ⎭ 
0.3 × 10 −6 − t τ

ic (t ) =
τ
e{ } 
0.3 − t τ
ic(t) = .e 5 2
12  (a)
I (t ) = (1 − e −t /τ ), τ = m sec
3 3
We know Instantaneous power of source = V(t) ⋅ i(t)
5 2
⎧ 0.3 ⎫ I (t ) =
(b) (1 − e −t /τ ), τ = m sec
P(t) = 3. ⎨ ⋅ e − t τ ⎬ 2 3
⎩ 12 ⎭
5
0.3 − t τ I (t ) = (1 − e −t /τ ), τ = 3 m sec
(c)
= ⋅e 3
4 
5
∞ ∞
0.3 − t τ I (t ) =
(d) (1 − e −t /τ ), τ = 3 m sec
E = ∫ p.dt = ∫ ⋅e 2
4
0 0   [2014]
0.3 0.3

= {τ } = × 12 μ J  Solution: (a)
4 4

E = 0.9 µJ

Hint: Total energy of the source = CV2


Hence, the correct option is (C).

5. In the circuit shown in the figure, the value of capacitor


C (in mF) needed to have critically damped response
i(t) is_____.

5
V ( s) −
Applying KCL, 6 + V ( s) + V ( s) = 0
R1 R2 SC

R1 = 1 kΩ, R2 = 2 kΩ, C = 1 μF
 [2014]
 1  5
Solution: 10 mF V ( s) 1 + + s  =
 2  6
For series R-L-C, θ = 1 L V ( s) =
5
R C  3
s s+ 
R C  2

ξ=
2 L 10 10
V ( s) = +
3s  3
2 C
2 3 3 + 
ξ =1⇒   =  2
R L
10  − t
2

4 V (t ) = 1 − e 3 
=C = L 10 mF 3 
 
R2

V (t ) 5  − t
3
6. In the figure shown, the capacitor is initially uncharged. i (t ) = = 1 − e 2  mA
Which one of the following expressions describes the R2 3 

current I(t) (in mA) for t > 0?
Hence, the correct option is (a).
Chapter 4  Transient Analysis  |  1.55

7. In the circuit shown in the figure, the value of v0(t) (in +


volts) for t → ∞ is___. V (0 ) = −50 V
V (t ) = V (∞) + V (0) − V (∞)  e −t / T

T = RC = 5 × 10−4
V(t) = 100 – 150e−t/T

dv +150
i (t ) = C = × C × e −t /τ
dt t
−2×10 t 3
i(t ) = 15e A
[2014]
Hence, the correct option is (a).
Solution: 31.25 V
9. In the circuit shown, the switch S is open for a long time
At t → ∞ L → S.C C→ O.C. and is closed at t = 0 The current i(t) for t ≥ 0+ is
Vx = 5ix
Vx − Zix
Also,10 = + ix
5
5ix − 2ix
10 = + ix
5
3ix (a) i(t) = 0.5 - 0 125e-1000t A
10 = + ix (b) i(t) = 1.5 - 0.125 e-1000t A
5
(c) i(t) = 0.5 - 0.5e-1000t A
10 = 1.6ix (d) i(t) = 0.375 e-1000t A [2010]
100 Solution: (a)
ix =
16 When circuit is in steady state then

V= 5=
ix
500
= 31.25V i = 0.75A (t = 0) = 0.375 A
x L
16 2
At t = ∞ iL = 0.5A (t = ∞)
8. In the circuit shown below, the initial charge on the
− tR
capacitor is 2.5 mC, with the voltage polarity as indi-
iL (t ) = i(∞) + i(0) − i(∞)  e
L
cated. The switch is closed at time t = 0. The current i(t)
tR
at a time t after the switch is closed is −
iL (t ) = 0.5 − 0.125e
L

∴ i(t) = 0.5 – 0.125 e−1000t


(a) i(t) = 15exp(- 2 × 103t) A Hence, the correct option is (a).
(b) i(t) = 5exp(- 2 × 103t) A 1 0. The switch in the circuit shown was on position a for a
(c) i(t) = 10exp(- 2 × 103t) A long time, and is moved to position b at time t = 0. The
(d) i(t) = -5 exp(- 2 × 103t) A [2011] current i(t) for t > 0 is given by
Solution: (a)
Q0 = 2.5 mC, C = 50 μF
Q
V= = 50V
C
V (∞) = 100 V

1.56 | Network Theory

(a) 0.2e -125t u (t) mA (b) 20e -1250t u(t) mA Solution: (a)


(c) 0 2e -1250t , u(t) mA (d) 20e -1000t u(t) mA Hence, the correct option is (a)
[2009]
12. The circuit shown in the figure is used to charge the
Solution: (b)
capacitor C alternately from two current sources as
In position a:- indicated. The switches S1 and S2 are mechanically cou-
pled and connected as follows:
For 2nT ≤ t < (2n + 1)T, (n = 0, 1, 2...) S1 to P1 and S2 to
P2.
For (2n + nT ≤ t < (2n + 2)T, (n = 0,1,2....) S1 to Q1 and
S2 to Q2

Ceq = (0.5 + 0.3) || 0.2 = 0.16 μF


t>0

Assume that the capacitor has zero initial charge. Given


that u(t) is a unit step function, the voltage Vc(t) across
the capacitor is given by


n
(a) ( −1) tu(t − nT )
n=0
100 1 ∝
I ( s) = ×
s  1
; C = 16 μF
 eq
u (t ) + 2
(b) ∑ (−1) u(t − nT )
n

 + 5 n =1
 Sceq  ∝
 

i (t ) =
100 −t ×1250
×e
tu(t ) + 2
(c) ∑ (−1) (t − nT )u(t − nT )
n =1
n

5

i(t ) = 20e −1250t

(d) 0.5 − e
− ( t − 2 nT )
+ 0.5e −( t − 2 nT −T ) 

11. The time domain behaviour of an RL circuit is repre-  n=0 [2008]
sented by Solution: (c)
di The waveform of voltage VC(t) is as shown:
L + Ri = V0 (1 + Be− Rt/L sin t )u(t )
dt
V0
For an initial current of i(0) = the steady-state value
R
of the current is given by
V
(a) i(t ) → 0
R
2V0 VC(t) = tu(t) – 2(t – T) u(t – T) + 2(t – 2T) u(t – 2T)
i(t ) →
(b) ∞
R
V
VC (t ) = tu(t ) + 2 ∑ (−1) (t − nT )u(t − nT )
n

i(t ) → 0 (1 − B )
(c) n =1
R Hence, the correct option is (c)
V0 Common Data for Questions 9 and 10
i(t ) →
(d) (1 + B)
R The following series RLC circuit with zero initial con-
 [2009] ditions is excited by a unit impulse function δ(t).
Chapter 4  Transient Analysis  |  1.57

Solution: (b)
1 s
VR ( s) = =
 1 2
s + s +1
 s +1+ s 
 
1 1
s+ −
VR ( s) = 2 2
2 2
13. For t > 0, the output voltage Vc(t) is  1  3
 s + +  
 1 3   2   2 
(a) 2  − 2t − t

e −e 2
3   1 1
  s+
1
VR ( s) = 2 − 2
(b) 2 - 2t 2 2
te  1  3 
2
1  3
3 s + +    s + +  

 2   2   2   2 
1
 3 
2 − t
(c) e 2 cos  t
  3t  1 −t / 2  3t 
3  2  VR (t ) = e −t / 2 cos   = sin 
 2 
e
 2  3
1    
2 − 2t  3 
(d) e sin 
 2 
t Hence, the correct option is (b).
3  
 [2008] 15. In the circuit shown, Vc is 0 volts at t = 0 sec. For t > 0,
Solution (d) the capacitor current ic(t), where t is in seconds, is given
1 1 by
VC ( s) = ×
s 1
+ s +1
s
1/s 1
1× 2
= 2
s + s +1 ( s + s + 1)
s
1
VC ( s) = 2 (a) 0.50 exp (-25 t) mA
2
 1  3
 s + 2  +  2  (b) 0.25 exp (-25 t) mA
    (c) 0.50 exp (-12.5 t)mA

 3t  (d) 0.25 exp (-6.25 t) mA [2007]
2 −1 / 2
VC (t ) = sin 
 2 
e
3 Solution: (a)
 
We know that i(t ) = i(∞) + [i(0) − i(∞)]e −t / PC
Hence, the correct option is (d).
At infinity capacitor will behave as output and i(∞) = 0
14. For t > 0, the voltage across the resistor is
For finding i(0), S.C. the capacitor.
 − 3 − t 
1
1
(a)  e 2 − e 2  ∴ iC (0) =
10
= 0.5 mA
3 
20
 
1
− t   3t  1  3t   T = ReqC ( 20 || 20) kΩ × 4 …F = 40 ms

(b) e 2 cos  −
  2  
sin 
  2  3 −25t
  ∴ iC (t ) = 0.5e mA
1
2 − t  3t  Hence, the correct option is (a).
(c) e 2 sin 
3  2 
  16. A square pulse of 3 volts amplitude is applied to C-R
1 circuit shown in the figure. The capacitor is initially
2 − t  3t 
cos  uncharged. The output voltage V2 at time t = 2 sec is
 2 
(d) e 2
3   (a) 3 V (b) -3 V
(c) 4 V (d) -4V
 [2008]
1.58 | Network Theory

Solution: (b) 18. At t = 0+ , the current i1 is

-V (b)
(a) -V
2R R
-V
(c) (d) zero [2003]
4R

Solution: (a)
In steady state,

[2005]
As RC = 100 μs ⇒ steady state will be reached in
i1 (t) = i2(t) = 0
approx. 500 μs and at 2s, it will be in state.
VC (0−) = V
∴VC = 3V, V2 = −VC = −3V
At t = 0+
Hence, the correct option is (b).
17. The circuit shown in the figure has initial current 1L (0)
= 1 A through the inductor and an initial voltage vc(0-)
= -1 V across the capacitor. For input v(t) = u(t), the
Laplace transform of the current i(t) for t > 0 is

−V
∴4 =
2R
Hence, the correct option is (a).

19. I1 (s) and I2 (s) are the Laplace transforms of i1 (t) and
i2 (t), respectively. The equations for the loop currents
s s+2 I1 (s) and l2 (s) for the circuit shown in the figure, after
(a) 2 (b) 2 the switch is brought from position 1 to position 2 at t
s + s +1 s + s +1
= 0, are
s−2 s−2  1 
(c) 2 (d) 2  [2004]  R + Ls + Cs − Ls 
 I ( s) 
V 
s + s +1 s + s +1 (a)   1  =  s 
 1 I ( s)  
Solution: (b)  − Ls R +   2   0 
Cs 

di 1
V (t ) = Ri(t ) + L
dt
(t ) +
C∫i(t )dt  1
 R + Ls + Cs − Ls 

 I ( s) 
 V


  0 (b)
  1  =  s 
 1   I 2 ( s)   
I (1) VC (0 + )  − Ls R+   0 
V ( s) = RI ( s) + sLI ( s) − LI (0 −1 ) + + Cs 
SC s


 1 
s+2  R + Ls + Cs − Ls   I1 ( s)   − V 
  ∴ I ( s) =
     s
s2 + s + 1
(c) =
 1   I 2 ( s)   
Hence, the correct option is (a).  − Ls R + Ls +   0 
Cs 
Chapter 4  Transient Analysis  |  1.59


 1 
 R + Ls + Cs

− Ls   I1 ( s)   V 
  

E = i 2 Rdt = 16J
(d) = s 0
 1   I 2 ( s)   
 − Ls R + Ls +   0  Hence, the correct option is (b).
Cs 
2 1. The voltage VC1, VC2 and VC3 across the capacitors in the
 [2003]
circuit in figure, under steady state, are respectively.
Solution: (c)

(a) 80 V, 32 V, 48 V (b) 80 V, 48 V, 32 V
(c) 20 V, 8 V, 12 V (d) 20 V, 12 V, 8
 [1996]
KVL in loop 1:-
Solution: (b)
I (b) R + V + I (b). 1 +  J ( s) − I ( s)  OL = 0 At steady state, L → S.C. and C → O.C.
SC  
1 1 1 2
s
KVL in loop 2:-
 1 
 I 2 ( s) − I1 ( s) sL + I 2 ( s) R + I 2 ( s). = 0  (2)
 SC 
∴From (1) and (2)

 1 
 R + sL + SC − sL   I1 ( s)   −V 
VC1 = 100 ×
40
= 80 V
   
= s  50
 1   I 2 ( s)  
 − sL R + sL +  0  3
 SC  V
C2 = 80 × = 48V
2+3
Hence, the correct option is (c).
VC3 = 80 − 48 = 32V
20. In the circuit of the figure the energy absorbed by the 4
W resistor in the time interval Hence, the correct option is (b).
2 2. For the compensated attenuator of figure, the impulse
response under the condition
R1C1 = R2C2 is

(a) 36 Joules
(b) 16 Joules
(c) 256 Joules
(d) None of the above [1997]
1
Solution: (b) (a) R2 [1 − e R1C1 ]u(t )
R1 + R2
VC (∞) = 10V R2
V2 (t ) =
(b) δ (t )
VC (0) = σ V R1 + R2
VR = 10 – 6 = 4V R
(c) 2 u(t )
IR = 1A R1 +R2
i(t ) = i(∞)t + [i(0) − i(∞)]e −t / PC R
1
(d) 2 e R1C1u (t )
t /δ R1 + R2
i (t ) ∞ = e
 [1992]
1.60 | Network Theory

Solution: (b)
V2 (b) Z 2 ( s)
Five-marks Questions
=
V1 (b) Z 2 ( s) + Z1 ( s)
1. The switch in figure, has been in position 1 for a long
R2 time and is then moved to position 2at = 0.
Z 2 ( s) =
SC2 R2 + 1 (a) Determine Vc(0+) and IL(0+)
dv (t )
R1 (b) Determine c at t = 0+
Z1 (b) = dt
SC1 R1 + 1 (c) Determine Vc(t) for t > 0.
R1C1 = R2C2
V2 ( s) R2

∴ =
V1 ( s) R1 + R2
R2
V2 (t ) = δ (t )
R1 + R2
Hence, the correct option is (b).
[2002]
23. A 10 Ω resistor, a 1 H inductor and 1 µF capacitor are
connected in parallel. The combination is driven by a 2. The circuit shown in figure is operating steady–state
unit step current. Under the steady-state condition, the with switch S1 closed. The switch S1 is opened at t = 0.
source current flows through: (a) Find iL (0+)
(a) the resistor (b) Find e1 (O+).
(b) the inductor (c) Using nodal equations and Laplace transform ap-
(c) the capacitor only proach, find an expression for the voltage across
(d) all the three elements [1989] the capacitor for all t > 0.
Solution: (b)
In steady state, current flows through the inductor
Hence, the correct option is (b).
24. If the Laplace transform of the voltage across a capaci-
1
tor of value of F is
2 [2001]
s +1 Solution: at t = 0−
VC ( s) =
s + s2 + s + 1
3

the value of the current through the capacitor at


t = 0 + is
(a) 0 A (b) 2 A
(c) (1/2) A (d) 1 A [1989]
Solution: (c)
Cdv for 16V source →
I= ⇒ I ( s) = SCV ( s) L′L(0–) = 16/4 = 4Amp
dt
lim i(t ) = lim SI ( s) 10
t →0+
i =i = = 2A
0− 0+
s →0
5
s ( s + 1)
= lim × 3 2 For 10V source →
s →∞ 2 ( s + s + s + 1)
iL′′ (0–)10/5 = 2Amp
s s( s + 1) s VC′′ (0–) = 0V
= lim =
s →∞ 2 2( s 2 + 1)( s + 1) 2( s 2 + 1) So
1 iL(0–) = 1′L(0–) = 1′′L(0–)
= A
2 =4+2
Hence, the correct option is (c). iL(0−) = 6 Amp.
Chapter 4  Transient Analysis  |  1.61

VC(0–) = VC′ (60–) + VC′′ (0–) 8 4×4


= −
=8+0 S ( S + 4) 2 + 4 2

= 8V
taking inverse laplace transform
VC(0−) = 8V
e1(t) = 84(t) – 4e−4t sin u(t) u(t)
(a) iL(0−) = iL(0−) = 6 Amp
e1(t) = y(2 − e−4t sin u(t))u(t).V
(b) e1(0−) = e1(0−) = 8V
3. The network N in figure consists only of two elements:
(c) at t = 0+ →
a resistor of 1 Q and an inductor of L Henry. A 5 V
source is connected at the input t = 0. The inductor cur-
rent is zero at t = 0. The output voltage is found to be
5e-3t V, for t > 0.
(a) Find the voltage transfer function of the network.
(b) Find L, and draw the configuration of the network.
(c) Find the impulse response of the network.

Apply Nodal Analysis at node e1(t)


e1 (t ) − 16u(t ) 1 d e (t ) − e2 (t )
+ e1 (t ) + 1 =0
2 8 dt 2 (1) [2000]
Apply nodal analysis at node e2(t) Solution:
e2 (t ) − c1 (t ) 1 5

2
+
0.5 ∫
e2 (t )dt = 0  (2) (a) V0 ( s) =
s+3
From equation (i) 5
Vi ( s) =
de1 (t ) 3
+ 8e1 (t ) − 4e2 (t ) − 64u(t ) = 0  (3)
dt V0 ( s) s
H ( s) = =
taking the laplace transform on both side Vi ( s ) s + 3
ey(0−) = 8V (b) For series R – L n/w
64 LS LS S
−8 + SE1 ( S ) + 8 E1 ( S ) − 4 E2 ( S ) − =0 H (S ) = = =
S R + LS 1 + LS S + 1/L
64 1
( S + 8) E1 ( S ) − 4 E2 ( S ) = +8 =3
S L
8S + 64  1
= (1) L= H
S 3
taking laplace transform of equation (2)  s   3 
h(t ) = L−1 
(c)  = L−1  1 − 
iL(0−) = 6 Amp  s+3  s+3
E2 ( S ) 6 h(t) = δ(t) – 3e–3t
0.5 S E2 ( S ) − 0.5 E1 ( S ) + 2 + =0
S S 4. In the circuit of figure, the switch 'S' has remained open
0.5 S E1(S) + (0.5 S + 2) E2(S) = −6  (4) for a long time. The switch closes instantaneously at t = 0.
by Solving equation (4) and (5)
8S 2 + 48S + 256
E1 ( S ) =
S ( S 2 + 8S + 32)

8( S 2 + 8S + 32) 16 S
= −
S ( S 2 + 8S + 32) S ( S 2 + 8S + 32) (a) Find V0 for t ≤ 0 and as t → ∞.
(b) Write an expression for V0 as a function of time for
8 16 0 < t < ∞.
= − 2
5 S + 8S + 32 (c) Evaluate V0 at t = 25 µsee. [1999]
1.62 | Network Theory

Solution:
(a) At steady state capacitor behave as open ­circuit
V0– = 2 × 5 = 10V
(b) V0t = V0–=10V
 10  20
V∞1 =  2 × ×5 = 3 V
 10 + 5 

V∞ due to 25V
5 125 25 (1) Current increases monotonically with time.
V∞11 = 25 × = = (2) Current decreases monotonically with time.
5 + 10 15 3
(3) Current remains constant at V/R
  V∞ = V1∞ + V11∞
(4) Current first increases, then decreases.
   = 20 + 25 = 15V (5) No current can ever flow. [1996]
3 3 6. The circuit shown in figure below, is initially in its
VC(t) = V∞(V0 – V∞)e–t/τ steady–state. The switch is opened at t = 0.
   = 15 + (10 – 15)e−t/t (i) Determine the initial voltage, Vc(0), across the ca-
pacitor, and the initial current, iL(0–), through the
   = 15 – 5e−t/t inductor.
  τ = Req C (ii) Calculate the voltage, vL(t), across the inductors for
10 t > 0.
  τ = × 2.5 × 10 −6
3
25
= × 10 −6 = 8.33 × 10 −6
3
VC(t) = 15 – 5e–t/833 × 10–6
 =15 – 5e–1.2 × 105tV
5 –6
(c) V0(25µsec) = 15–5e–1.2 × 10 × 25 × 10
V0 = 15 – 5 (0.05)
 = 14.75 V
[1994]
5. Match each of the items A, B and C on the left, with
an appropriate item on the right. In toe circuit shown Solution: (i) At steady state
in Figure (a) to (c), assuming initial voltages across
capacitors and currents through the inductors to be
zero at the time of switching (t = 0), then at any time
t > 0,

12
iL (0 − ) == 4A
1+ 2
VC(0) = 2 × iL(0−) = BA
(ii) for t > 0, drawing the equivalent circuit in s–do-
main
2 Vc(s)
104 2
s
12 10s
s
VL(s)
+ +
8/s 40
- -
Chapter 4  Transient Analysis  |  1.63

Using KCL, τ = 0.5Rϑ


VC ( S ) − 12 /S VC ( S ) − 8/S VC + 40 i(t) = i∞ + (i1 – i∞)e –t/τ
+ + =0
2 10 4 /S 10 S + 2 = 0 + (1.67 – 0)e − t/0.5Rϑ
On Solving, = 1.67 e −2t/Rϑ
V0– = V0 + = OV
80 s 2 + 20 × 10 4 s + 12 × 10 4
VC ( S ) = V∞ = 5V
10 s3 + 5 × 10 4 s 2 + 2 × 10 4 s  (1)
T = RC
 V ( S ) + 40  Vi(t) = V∞ + (V0+ – V∞)e–t/τ
∴VL ( S ) =  C 10 S − 40
 10 S + 2  = (5 + 10 – 5 )e −t/RC

On putting the value of VC(S) from equation (i) = 5(1 − e−t/RC)
We get,
4 4
VL ( S ) ≅ −
s + 5000 s + 0.4
Taking laplace transform
VL(t) = (ye – 500t – 4e – 0.4t)V
7. In the following circuit the capacitance varies as C =
KQ, where K is a constant equal to 0.5 Farads/Coulomb
and Q, the charge on the capacitor in Coulombs.
Determine the current through the circuit and sketch
the voltage waveform across the capacitor (Vc) for a 8. The network shown in figure is initially under steady–
step input Vi as shown in figure. state condition with the switch in position 1. The switch
is moved from position 1 to position 2 at t = 0. Calculate
the current (i) through R1 after switching.

[1991]
Solution:
10
i0− = i0+ = 5 = 2 A

i∞ = 0
Req = R1 + R2 = 5 + 5 = 10Ω
[1993]
L = 2H
Solution:
L 2 1
5 τ= = =
i0+ = = 1.67 mA R 10 5
3k
i(t) = i∞ + (i0+ – i∞)e–t/τ
i∞ = 0 = 0 + (2 – 0) e−t/15
τ = RC = 2e−5t; + ≥ 0
C = 0.5ϑ or i(t) = 2.e−5tu(t)
Chapter 5
Two Port Networks
According to reciprocity theorem
One-mark Questions Response
= Constant
1. Consider the two-port resistive network shown in the fig- Excitation
ure. When an excitation of 5 V is applied across Port 1, V
∴ = contant
and Port 2 is shorted, the current through the short circuit I
at Port 2 is measured to be 1 A (see(a) in the figure). V V
∴ 1= 2
  Now, if an excitation of 5 V is applied across port 2, I1 I 2
and Port 1 is shorted (see (b) in the figure), what is the 5 5
current through the short circuit at Port 1? [2019] =
I2 1
1Ω 2Ω
∴ I 2 = 1 Amp

Port 1 RΩ Port 2 Hence, the correct option is (C)
2. Consider the sequence x[n] = anu[n] + bn u[n], where
u[n] denote the unit step sequence and 0 < |a| < |b| < 1.
1Ω 2Ω The region of convergence (ROC) of the z transform of
x[n] is [2016]
1A (A) |z| > |a| (B) |z| > |b|
5V +
– RΩ
(C) |z| < |a| (D) |a| < |z| < |b|

(a) Given x [ n] = a n ⋅ u ( n) + b n ⋅ u ( n)
Solution: 
1Ω 2Ω
It is a right-sided signal.

? So ROC is z > a  and  z > b 


RΩ + 5V

But given 0 < a < b < 1
(b)
(A) 0.5 A (B) 2.5 A
(C) 1 A (D) 2 A Unit circle
Solution:
1Ω 2Ω
–a b
1A
5V +
– RΩ
ROC

1Ω 2Ω

1A RΩ + 5V ∴ ROC z > b
– 
Hence, the correct option is (B).
Chapter 5  Two Port Networks  |  1.65

3. The z-parameter matrix for the two-port network shown is ⎡V1 ⎤ ⎡ A B ⎤ ⎡ V2 ⎤


⎢ I ⎥ = ⎢C D ⎥ ⎢ − I ⎥
⎡ 2 jω jω ⎤ ⎣ 1⎦ ⎣ ⎦⎣ 2⎦ 
⎢ jω 3 + 2 jω ⎥
⎣ ⎦ V1 = AV2 – BI2
Where the entries are in Ω. Suppose zb ( jω) = Rb + jω.
I1 = CV2 – DI2
Za Zb
1 2
Zc
Let I2 = 0
1′ 2′ I1
C= [∴ I2 = 0]
V2
Then the value of Rb (in Ω) equals _____. [2016]
1
∴ C=
⎡V1 ⎤ ⎡ za + zc z c ⎤ ⎡ I1 ⎤ Z 21
⎢ ⎥=⎢
Solution:  
⎣V2 ⎦ ⎣ zc zb + zc ⎥⎦ ⎢⎣ I 2 ⎥⎦ We know for a T-network
Zc = jω Z21 = ZC = 2 Ω
Zb + Zc = 3 + 2jω
1
Rb + jω + jω = 3 + 2jω C = = 0.5Ω
2
Rb = 3 Ω
From the given options Choice B is correct
As we know that the values of z parameters for t model
Hence, the correct option is (B).
can be given as
5. If the scattering matrix [SI] of a two port network is
Z11 = Z a + Zb
 0.2∠0 0.9∠90 
Z12 = Z 21 = Zb S= 
 
Z 22 = Zc + Zb 
 0. 9∠90 0 . 1∠90 
Hence, the correct Answer is (3 Ω). then the network is
4. The ABCD parameters of the following 2-port network (a) lossless and reciprocal
are [2015] (b) lossless but not reciprocal
(c) not lossless but reciprocal
(5 + j4)Ω (5 – j4)Ω
(d) neither lossless nor reciprocal [2010]
Solution: (c)
(2 + j0)Ω For reciprocal network
S12 = S21
but |S11|2 + |S12|2 ≠ 1 ⇒ not loss less
⎡3.5 + j 2 20.5 ⎤ Hence, the correct option is (c)
(A) ⎢
⎣ 20.5 3.5 − j 2⎥⎦ 6. For the two port network shown below, the short-circuit
admittance parameter matrix is
⎡3.5 + j 2 30.5 ⎤
(B) ⎢
⎣ 0.5 3.5 − j 2⎥⎦ 0.5 Ω
1 2
⎡ 10 2 + j 0⎤
(C) ⎢2 + j0
⎣ 10 ⎥⎦ 0.5 Ω 0.5 Ω

⎡7 + j 4 0.5 ⎤
(D) ⎢ 30.5 7 − j 4 ⎥ 1‘ 2‘
⎣ ⎦
Solution:  4 −2   1 −0.5
(a)
  S (b)
 S
Za Zb  −2 4   −0. 5 1 
+ +
 1 0.5 4 2
(c)
  S (d)
 S
V1 Zc V2
0.5 1  2 4

– –  [2010]
1.66 | Network Theory

Solution: (a) (a) re and br0 (b) 0 and - br0


0.5 (c) 0 and br0 (d) re and - br0
 [2006]
Solution: (b)
0.5 0.5
When I1 = 0 ⇒ V1 = 0
V1
\ = 0= Z12
I2
Y parameters will be written as:
When I2 = 0 ⇒V2 = −βI1r0
1
y=
a y=b y=
c = 2 V2
.5 = − β x0 = Z 21
y11 = y22 = 2 + 2 = 4 I1
y21 = y21 = −2 Hence, the correct option is (b)
Hence, the correct option is (a) 9. The ABCD parameters of an ideal n:1 transformer
7. A two-port network is represented by ABCD param- shown in the figure are  n 0 
eters given by  
0 X 
V1   A B   V2  The value of X will be
 =  
 I t  C D   − I 2  I1 I2

If port-2 is terminated by RL, the input impedance seen


at port-1 is given by
A + BRL ARL + C V1 V2
(a) (b)
C + DRL BRL + D

DR + A B + ARL n:1
(c) L (d)
BRL + C D + CRL
 [2006] 1
(a)
n (b)
n
Solution: (d)

I1 I2 (c) 1 
n2 (d) [2005]
n2
V1 V2
Solution: (b)
I 2 V1 n
For transformer, = =
J1 V2 1
ABCD parameters will be written by
V1 = AV2 – BI2 V1 = AV2 – BI2
I1 = CV2 – DI2 and V2 = −I2RL I1 = CV2 – DI2
V1 AV2 − BI 2 − AI 2 RL − BI 2 V1 I1 V2 1
= = = A = =
n, D = =
I1 CV2 − DI 2 −CI 2 RL − DI 2 V2 I =0 I 2 V =0 V1 n
2 2

ARL + B Hence, the correct option is (b)


J/P imp =
CRL + D 10. The admittance parameter Y12 in the two-port network
Hence, the correct option is (d) in figure is
8. In the two-port network shown in the figure below, Z12, I1 20 Ω I2
and Z21 are, respectively
l I2

E1 5Ω 10 Ω E2

r0 β I1 r0
Chapter 5  Two Port Networks  |  1.67

(a) -0.2 mho (b) 0.1 mho 12. The short-circuit admittance matrix of a two-port net-
(c) -0.05 mho (d) 0.05 mho [2001] work is
Solution: (c)  0 −1 / 2 
 
V1 20 Ω V2 1 / 2 0 

I1 I2 The two-port network is


(a) non-reciprocal and passive
(b) non-reciprocal and active
5Ω 10 Ω
(c) reciprocal and passive
(d) reciprocal and active [1998]
Solution:(b)
y12 ≠ y21 ⇒ N/W is non-reciprocal and active
V1 (V1 − V2 ) Hence, the correct option is (b)
I1 = +
5 20
13. The condition that a two-port network is reciprocal
 1 1  −V2 can be expressed in terms of its ABCD parameters
I1 = V1  + 
 5 20  20 as______ [1994]
 1  −V Solution: AD – BC = 1
I1 = V1   2 The condition is AD – BC = 1
 4  20
Hence, the correct option is (c)
Two-marks Questions
I1 = Y11V1 + Y12V2 ⇒ Y12 = −1 = −0.05 mho 1. The state transition diagram for the circuit shown is
20
 [2019]
11. A two-port network is shown in the figure. The param-
eter h21 for this network can be given by
I1 I2
+ +
R R
D Q 1
V1 R V2

Q 0
− −

CLK
(a) -1/2 (b) +1/2 A
(c) -3/2 (d) +3/2 [1999]
Solution: (a) (A) A = 1 A=0
A=0
I1 I2
Q=0 Q=1
+ +
R R A=1
(B) A=0
V1 R V2 A=0
A=1
− −
Q=0 Q=1

V1 = 2RI1 + RI2 A=1

V2 = RI1 + 2RI2 (C) A=0 A=0


A=1
I2 Q=0 Q=1
h21 =
I1
V2 = 0 A=1

Put V2 = 0 (D) A=0
A=0

\ 0 = RI1 + 2RJ2 ⇒ I 2 = −1 A=1


I1 2 Q=0 Q=1
Hence, the correct option is (a) A=1
1.68 | Network Theory

Solution: Solution: 
I(t)

C
v(t) +
– R R

D Q 1
C
B C
ClK Q 0
R
ClK V(t) = 2 sin (1000t)
A
R=1K
C = 1mF
B = AQ + AQ Using Y to D conversion for the viner circuit.
+
D = B⋅Q
C/3
= AQ + AQ ⋅ Q C/3

= A⊕ Q + Q R
R C/3

Present state i/P Next state –


Q Q+
R
0 0 1
0 1 1 1

1 0 1 jWC / 3 R
Z= =
1 1 0 1 C
R+ 1 + jWR
jWC / 3 3
A=1 = 900 − 300i
A=0
A=0 +
Q=0 Q=1
A=1
Z1
V(t) Z2
Hence, the correct option is (B).

2. In the circuit shown, if n(t) = 2 sin(1000 t) volts, R = 1


kW and C = 1 mF, then the steady-state current i(t), in –
milliamperes (mA), is [2019] Z3

2 2
i(t) ZT = Z1 = [900 − 300i ]
3 3
2 × 103
+ C = = 600 − 200i
v(t) 3+ j
R R

V (t ) 2 sin (1000t )
C C l (t ) = =
Zt 600 − 200i
= (3 + j ) sin (100t ) 
R
i (t ) = 3 sin 100t + j sin 1000t
(A) 3 sin(1000 t) + cos(1000 t)
= 3 sin 1000t + 1 < 90 sin 1000t
(B) sin(1000 t) + cos(1000 t)
(C) sin(1000 t) + 3 cos(1000 t) = 3 sin 1000t + siin 1000t + 90

( )
(D) 2 sin(1000 t) + 2 cos(1000 t) i t = 3 sin 1000t + cos 1000t
Hence, the correct optiojn is (A).
Chapter 5  Two Port Networks  |  1.69

3. The RC circuit shown below has a variable resistance


l = C (3)
2
R(t) given by the following expression:
1
⎛ t⎞ C=
R(t) = R0 ⎜ 1 – ⎟ for 0 ≤ t ≤ T 9
⎝ T⎠
1
i (t ) = (3 − t ) A
2

1000 Ω 9
I(t)
T
for t =
C 2
Vs +

1
t=0
i (1.5) =
9
[ 3 − 1.5]
2

2
Where R0 = 1 W and C = 1 F. We are also given that T = 1  3 1
= × = = 0.25
3 R0C and the source voltage is VS = 1 V. If the current 9  2  4
at time t = 0 is 1 A, then the current I(t), in amperes, at
Hence, the correct answer is 0.25.
time t = T/2 is _____ (rounded off to 2 decimal places).
 [2019] 4. The ABCD matrix for a two-port network is defined by:
Solution: ⎡ v1 ⎤ ⎡ A B ⎤ ⎡ V2 ⎤
i(t) R(t) ⎢ I ⎥ = ⎢C D ⎥ ⎢ − I ⎥
⎣ 1⎦ ⎣ ⎦⎣ 2⎦
+ I1 2Ω 2Ω I2
Vs +
– Vc
– + +
t=0
V1 5Ω V2

 t
R (t ) = R0 1 −  0 ≤ t ≤ T – –
 T
R0 = 1Ω The parameter B for the given two – port network (in
C = 1F ohms, correct to two decimal places) is ______.[2018]
T = 3R0C Solution:  Consider the figure given below
VS = 1V I1 2Ω 2Ω I2
+ +
 t
R (t ) = 1⋅  1 − 
 3 V1 5Ω V2
1
R (t ) ⋅ i (t ) = ∫ i (t ) dt = VS – –
C TA TB
 t
1 −  ⋅ P (t ) + ∫ i (t ) dt = 1.
From the above figure
 3  ⎡ ZA ⎤
On differentiation w.r. to t ⎢1 + Z ZA ⎥
TA = ⎢ B ⎥
 t  di (t ) i (t ) ⎢ 1 ⎥
1 − 3  dt − 3 = −i (t ) ⎢ Z 1 ⎥
⎣ B ⎦
di (t ) ⎡1.4 2 ⎤
(3 − t ) + 2i = 0 TA = ⎢ ⎥
dt ⎣0.2 1 ⎦
di −2
= dtt ⎡2 2⎤
i (3 − t ) TB = ⎢

⎣0 1 ⎦
Integrating path sides we get
ln i = 2 ln (3 − t ) + ( nCC ) . T = [TA ][TB ]

i (t ) = C (3 − t )
2
⎡ 2.8 4.8⎤
T = ⎢ ⎥
i ( o) = 1 A ⎣0.4 1.4 ⎦

1.70 | Network Theory

From the above matrix T Let I1 = 0 


B = 4.8 I V2
Z12 = 2 and Z 21 =
Hence, the correct answer is 4.3 to 5.3. V1 I1


⎡z z12 ⎤
5. The z–parameter matrix ⎢ 11 for the two port
z22 ⎥⎦
+
I2
⎣ z21
network shown is [2016] +
V1 – V2

3Ω –

Input port Output port
Z22 = (3 || 6)

= 2Ω
6Ω V1 V
Z12 = =− 2
I2 I2
⎡ 2 −2⎤ ⎡ 2 2⎤ 
⎢ −2 2 ⎥ (B)
(A) ⎢ 2 2⎥
⎣ ⎦ ⎣ ⎦ = −2 Ω 
⎡9 −3⎤ ⎡ 9 3⎤ Hence, the correct option is (A).
⎢6 9 ⎥ (D)
(C) ⎢6 9 ⎥
⎣ ⎦ ⎣ ⎦ 6. The 2-port admittance matrix of the circuit shown is
given by [2015]
Solution:  5Ω

10 Ω 10 Ω
Input port Output port

6Ω ⎡ 0.3 0.2⎤ ⎡15 5 ⎤


(A) ⎢0.2 0.3⎥ (B) ⎢ 5 15⎥
As we know that the equations for Z parameters are ⎣ ⎦ ⎣ ⎦
⎡3.33 5 ⎤ ⎡ 0.3 0.4 ⎤
V1 = Z11 I1 + Z12 I2 (C) ⎢ 5 (D)
 ⎣ 3.33⎥⎦ ⎢0.4 0.3⎥
⎣ ⎦
V2 = Z21 I1 + Z22 I2 Solution: 

0.2 Ω
Let, I2 = 0
Yc
P
Ya 0.1 Ω Yb 0.1 Ω

V1 3Ω 6Ω

We know for π -network



⎡Y + Y Yc ⎤
V V [Y] = ⎢ a c
Z11 = 1 and Z 21 = 2 ⎣ Yc Yb + Yc ⎥⎦
Ι1 Ι1 
V1 ⎡ 0.3 0.2⎤
=2Ω =⎢ ⎥ Ω
I1 ⎣0.2 0.3⎦

V2 V Hence, the correct option is (A).
Z 21 = = − 1 = −2 Ω
I1 I1
7. In the h-parameter model of the two-port network given
V2  in the figure shown, the value of h22 (in S) is_____
= Z 21 = −2 Ω 
I1  [2014]
Chapter 5  Two Port Networks  |  1.71

3Ω  6 24 
(a)
 9 8 
 (b)
 42 9   
3Ω 3Ω 8 24 
1 2
(c)  42 6 
(d)
9 6   

6 24
  6 60 
2Ω
2
 
1
2Ω 2Ω  [2014]

Solution: (c)
Applying Y to Z conversion
Solution:
3Ω 10 Ω
For parallel combination, equivalent admittance is
given by [Y] = [Y1] + [Y2]

1 1 −1  6Ω
3 + 3 3 
[ y1 ] =  
 −1 1 1
+
 3 3 3 
9Ω 6Ω 
1 1 −1  [Z ] =  
 + 2  6Ω 24Ω  .
[ y2 ] =  2 2 
 −1 1 1
+ 
Hence, the correct option is (c)
 2 2 2 Common Data for Questions 3 and 4
 5 −5  With 10 V dc connected at port A in the linear nonre-
 3 ciprocal two-port network shown below, the following
6 
∴ [ y] =   were observed.
 −5 5  (i) 1 W connected at port B draws a current of 3 A.
 6 3 
(ii) 2.5 W connected at port B draws a current of 2 A.
5 5 +
I1 = V1 − V2
3 6
A B
−5 5
I 2 = V1 + V2 −
6 3

I1 9. For the same network, with 6 V dc connected at port A,


h22 = 1 W connected at port B draws 7/3 A. If 8 V dc is con-
V2
I1 = 0
nected to port A, the open circuit voltage at port B is
V2 (a) 6 V (b) 7 V
When I1 = 0 ⇒ V1 = (c) 8 V (d) 9 V [2012]
2
Solution: (b)
I 2 15
h=
22 = = 1.2 s Given
V2 12
V1 = 10V, V2 = 3V, I2 = −3A
8. For the two-port network shown in the figure, the V1 = AV2 – BI2
impedance (Z) matrix (in W) is 10 = 3A + 3B (1)
V2 = 5V, I2 = −2A
30 Ω
10 = 5A + 2B(2)
1 2
+ 10 20
∴A= ,B=  from (1) and (2)
9 9
10 Ω 60 Ω
Given:- V1 = δV, (V2)oc = ?

1‘ 2‘ I2 = 0
1.72 | Network Theory

∴V1 = AV2 (i) S-Open, S2-closed A1 = 0 A, V1 = 4.5 V, V2 = 1.5V,


V1 80 A2 = 1 A
=
(V2 )oc = = 7.2V (ii) S1-Closed. S2-Open A1 = 4 A, V1 = 6 V, V2 =
A 10 / 9
6 V, A2 = 0 A
Hence, the correct option is (b)
S1 S2
1 0. With 10 V dc connected at port A, the current drawn by + - 1 2 − +
A1 A2
7 W connected at port B is
+
(a) 3/7 A (b) 5/7 A + Two Port + +
6V V1 V2 1.5V
(c) 1 A (d) 9/7 A [2012] Network

− − −
Solution: (c) 1 2
Using ABCD parameters for given circuits we can
write 12. The Z-parameter matrix for this network is
V1 = AV2 – BI2 and V2 = −7I2
1.5 1.5
\ 10 = −7I2A – BI2 (a)
 1.5 4.5
 (b)
−70 −20  4.5 1.5  
10 = I2 I2 1.5 4.5
9 9 1.5 4.5
(c)
  4.5 1.5 
 (d)

1.5 1.5   
I2 = −1A 1.5 4.5
 [2008]
Hence, the correct option is (c)
Solution: (c)
11. In the circuit shown below, the network N is described
by the following Y matrix: When I = 0
V V1 = 4.5 V
 0.1S −0.01S  . The voltage gain 2 is
Y =  V1 V2 = 1.5 V
0.01S 0.1S 
J2 = 1 A
25 Ω I1 I2 V1
= Z12 = 4.5
+ + I 2 I =0
+ 1
100 V - V1 N V2 100 Ω
V2
- - = Z 22 = 1.5
I 2 I =0
1

When I2 = 0 then I1 = 4A
(a) 1/90 (b) -1/90 V1 = 6V1 V2 = 6V
(c) -1/99 (d) -1/11 [2011] V1 6
Solution: (d) =
Z11 = = 1.5
I1 I =0 4
Y parameters can be written as 2

I2 = Y21V1 + Y22V2 V2 6
= Z 21 = = 1.5
−V2 I1 I =0 4
V2 = −I2RL = −100I2 ⇒ I 2 = 2
100
−V2 1.s 4.s 
\ = 0.01V1 + 0.1V2 [Z ] =  
100 1.s 1.s 

V1/V2 = –1/11 Hence, the correct option is (c)


Hence, the correct option is (d) 1 3. The h-parameter matrix for this network is
 −3 3   −3 −1 
Linked Answer Questions 6 to 7. 
(a) 
 (b) 
 −1 0.67   3 0.67 
A two-port network shown below is excited by exter-
nal dc sources. The voltages and currents are measured
3 3  3 1 
with voltmeters V1 V2 and ammeters A1 A2 (all assumed (c)
  (d)
 
to be ideal) as indicated. Under following switch condi- 1 0 . 67   −3 −0.67 
tions, the readings obtained are:
 [2008]
Chapter 5  Two Port Networks  |  1.73

Solution: (a) 15. For the lattice circuit shown in the figure, Za = J2W
Z Z and Zb = 2Q. The values of the open circuit impedance
h11 = Z11 − 12 21 = −3 parameters
Z 22

− Z 21 Z  Z11 Z12  are
h21 = = −1  
Z 22  Z 21 Z 22 

V1
=
h12 = 3 Zb
V2 I =0
1 1 3

I2
h22 = 0.67 Za
V2 I1 = 0

 −3 3  Za
[h] =  
 −1 0.67 
2 4
Hence, the correct option is (a)
Zb
14. The h parameters of the circuit shown in the figure are
I1 10 Ω I2
1 − j 1 + j
(a)
 
+ + 1 + j 1 + j

V1 20 Ω V2  1− j 1+ j
(b)
 
 −1 + j 1 − j 
− −
(c)
1 + j 1 + j 
 
 0.1 0.1 10 −1  1 − j 1 − j 
(a)
  (b)
 
 −0 . 1 0 . 3   −1 0.05  1 + j −1 + j  
(d) [2004]
 
30 20  10 1   −1 − j 1 + j 
(c)
  (d)
  Solution: (d)
 20 20   −1 0.05
 Za + Zb Za − Zb 
 [2005]  2 2 
Solution: (d) For lattice N/W [ Z ] =  
 Za − Zb Za + Zb 
I1 I2  2 2 
10 Ω
Za = 2j, Zb = 2Ω
+ +  1 + j −1 + j 
[Z ] =  
V1 20 Ω V2  −1 + j 1 + j 
Hence, the correct option is (d)
- -
16. The impedance parameters Z11 and Z12 of the two-port
V1 = 30I1 + 20I2 network in the figure are
V2 = 20I1 + 20I2 1 2
2Ω 2Ω 2Ω
I2 = −20I1 + V2 and I2 = h21I1 + h22V2
1Ω 1Ω
1
∴ h21 = −20 = −1, h22 =
20 20
1 2
20(V2 − 20 I1 )
V1 = 30 I1 + Z11 = 2.75 W and Z12 = 0.25W
(a)
20
Z11 = 3W and Z12 = 0.5 W
(b)
V1 = 10I1 + V2
Z11 = 3Q and Z12 = 0.25 W
(c)
∴ h11 = 10, h12 = 1 Z11 = 2.25 W and Z12 = 0.5 W
(d) [2003]
Hence, the correct option is (d)
1.74 | Network Theory
V 2Ω
1 2
Solution: 2Ω
(a) 1Ω 3Ω
2Ω E1 = 6I1 + 4I2 – 10E1 ⇒ 11E1 = 6I1 + 4I2

Using Δ-Y conversion
1 6 4 6
2Ω
E1 = J1 + I 2 ⇒ Z11 = Ω
2Ω 11 11 11
V
11 2 2
2Ω 1Ω 3Ω
2Ω E2 = 4J1 + 4I2 – 10E1
1Ω 6 4
2Ω E2 = 4 I1 + 4 I 2 − 10 J1 + J2
11 11
1 2
60 40
E2 = 4 I1 + 4 I 2 − J1 − J2
11 11
2Ω 0.5 Ω 0.5 Ω 2Ω
−16 4I −16
E2 = J1 + 2 ⇒ Z 21 = Ω
11 11 11
0.25 Ω
Hence, the correct option is (c)
2Ω 0.5 Ω 0.5 Ω 2Ω
1 8. In the circuit of the figure given below, the equivalent
impedance seen across terminals A, B is
0.25 Ω
 Z + Z3 Z3 
∴ [Z ] =  1  A 2Ω 4Ω
 Z3 Z 2 + Z3  j3 Ω

Zeq
 2.75 0.25 
[Z ] =  
 0.25 2.75 2Ω
−j4 Ω
B 4Ω
Hence, the correct option is (a)
17. The Z parameters Z11 and Z21 for the two-port network
(a) (16/3) W
in the figure are
(b) (8/3) W
(c) (8/3 + 12 j) W
I1 2Ω I2 (d) None of the above [1997]
4Ω Solution: (b)
E1 E2
Z1 Z3
= ⇒ Bridge is balanced
10E1 Z2 Z4

8
∴ Zeq = ( 2 || 4) + ( 2 || 4) = Ω
3
−6 16
Z11 =
(a) Ω; Z 21 = Ω Hence, the correct option is (b)
11 11
19. For a two-port network to be reciprocal,
6 4
(b)
Z11 = Ω; Z 21 = Ω (a) z11 = z22 (b) y21 = y12
11 11 (c) h21 = -h12 (d) AD - BC = 0
(c) 6 −16  [1992]
Z11 = Ω; Z 21 = Ω
11 11 Solution: (b) and (c)
4 4 y11 = y12 
Z11 = Ω; Z 21 = Ω 
(d) [2001]
11 11 h = −h  for reciprocal N/W
21 12 
Solution: (c)
Hence, the correct option is (b) and (c)
2 0. Two two-port networks are connected in cascade. The
I1 2Ω I2
combination is to be represented as a single two-port
4Ω
network. The parameters of the network are obtained
E1 E2 by multiplying the individual
10E1 (a) z-parameter matrices
(b) h-parameter matrices
(c) y-parameter matrices
(d) ABCD parameter matrices [1991]
Chapter 5  Two Port Networks  |  1.75

Solution: (d) Also, by substituting V2 in equation (1),


 A B   A1 B1   A2 B2  V1 = −2I1 + I2
 =   Z11 = −2 Ω, Z12 = 1 Ω
C D  C1 D1  C2 D2 
Hence, the correct option is (a).
Hence, the correct option is (d)
2 2. For the transfer function of a physical two-port network:
2 1. The open circuit impedance matrix of the two port net- (a) all the zeros must lie only in the left hall of the s-
work shown in figure is plane
I1 2Ω I2 (b) the poles may lie anywhere in the s-plane
+ + (c) the poles lying on the imaginary axis must be sim-
ple
(d) a pole may lie at origin [1989]
1Ω 3I1 Solution: (c) and (d)
V1 V2
The poles lying on imaginary axis must be simple and
− − a pole may lie at origin.
Hence, the correct option is (c) and (d).
 −2 1  −2 −8 23. The condition AD - BC - 1 for a two-port network
(a)
  (b)
 
 −8 3   −8 3  implies that the network is a:
(a) reciprocal network
0 1   2 −1 (b) lumped element network
(c)
  (d)
 
1 0   −1 3  (c) lossless network
(d) unilateral element network [1989]
 [1990]
Solution: (a)
For reciprocal N/W, AD - BC = 1
Solution: (a) Hence, the correct option is (a).
I1
2Ω
I2 24. Two two-port networks are connected in parallel. The
V1 V2
combination is to be represented as a single two-port
network. The parameters of this network are obtained
by addition of the individual
V1 1Ω 3I1 V2 (a) z parameters
(b) h parameters
(c) y parameters
(d) ABCD parameters [1988]
Solution: (c)
V1 V1 − V2 3V1 V2
I1 = + = − For parallel connection, [Y] = [Y]A + [Y]B
1 2 2 2
Hence, the correct option is (c).
V2 − V1
I 2 = 3I1 +
2
Five-marks Questions
V V
I 2 − 3I1 + 1 = 2 1. Find the Y–parameters (short circuit admittance param-
2 2 eters) for the network shown in figure.
V2 = 2 I 2 − 6 I1 + V1  (1)
 V  2
V2 = 2 I 2 − 6 J1 +  J1 + 2 × 3
 2 
2 V
V2 = 2 I 2 − CI1 + I1 + 2
3 3
2V2 16
= 2 I 2 − I1
3 3
V2 = 3I2 – 8I1
Z22 = 3Ω, Z21 = −8Ω
[1993]
1.76 | Network Theory

Solution: Using star–delta conversion 3. Find the current–transfer–ratio, I2/I2 for the network
shown in below figure. Also mark all branch currents.

[1995]
Solution:

Apply mesh analysis in both loop(11)


(I3 + I2 + 1.5I1) × 1 + (I3) × 1 + 2I3 = 0
1.5I1 + I2 + 4I3 = 0
Apply mesh analysis
 I1 
V2 = 0 → ( I 2 ) × 1 +  I 2 +  × 2 − ( I 3 × 1) = 0
 2
I1 1 5
y11 = = = + s (y11 ≠ y1 + y2)
I1 + 3I2 = I3
V1 2i 11
Substitute I3 in equation (i)
I2 3 
y21 = = −V2 = −  + s  1.5I1 + 2I2 + 4(I1 + 3I2) = 0
V1  11 
1.5I1 + I2 + 4I1 + 12I2 = 0
V1 = 0 → I 2 −11
=
3  I1 26
y12 = − y2 = −  + s 
 11 
4. For the 2–port network shown in figure determine the
4 h–parameters. Using these parameters, calculate the
y22 = y2 + y3 = + s
11 output (port '2') voltage v2, when the output port is ter-
minated in a 3Ω resistance and a 1 V(DC) is applied at
2. Assuming that the amplifier shown in figure below, is a
the input port (v1 = 1V).
voltage–controlled voltage source, show that the volt-
age transfer function of the network is given by
V2 (s) K
T(s) = =
V1 (s) s 2 + (3- K)s +1

[1995]
Solution: ly = lx
lY = lX
V2 = 0→

[1994]
Chapter 5  Two Port Networks  |  1.77

5. Find the input resistance Rin of the infinite section resis-


tive network shown in figure.

V1 1× i1 + 3i1′ + 0 [1996]
h11 = = =y
I1 il Solution:
I 2 I1
h=
21 = = 1
I1 I 2
i′1=0
lx = 0
2ix = 0
1× Rin 1 + Rin + Rin
Rin = 1 + =
1 + Rin 1 + Rin
R2in + Rin = 1 + 2Rin
R2 – Rin – 1 = 0
in

+1 ± 1 + 4 1 ± 5
Rin = =
2 2
V1 = 4I1 + V2
I2 = I1 + 05V2 Rin cannot be negative
1+ 5
Rin = Ω
2
6. The open circuit impedance matrix Zoc of a three–ter-
minal two–port network with A as the input terminal, B
as the output terminal, and C as the common terminal,
2 5
is given as [ Zoc ] =  
3 7
Write down the short circuit admittance matrix YSC of
V1 = 1V the network viewed as a two–port network, but now
V2 = − 3L2 taking B as the input terminal, C as the output terminal
l1 = l1 – 1.5 I2 and A as the common terminal. [1996]
2.5 l2=l1 Solution: V1 = 2I1 + 5I2
V2 = 3I1 + 7I2
 V  −2.5V2
I1 = 2.5  − 2  =
 3  3
V1 = 4I1 + V2

 −25V2  −10V2
1 = 4  + V2 = + V2
 3  3

−10V2 + 3V2 −7V2


1= =
3 3
−3
V2 = V
7
1.78 | Network Theory

Ii = y11Vi + V12Vb Solution: When V2 = 0


I0 = y21V1 + y22V0
V1 = −Vb
V2 = −Vi − Vb
I1 = −(Ii + I0)
I2 = Ii
−V0 = −2(I1 + I0) + 5Ii
−V0 = 3Ii + 2I0
Vi – V0 = −3(Ii + I0) + 7Ii
from the fig,
Vi = V0 + 4Ii – 3I0
2Ix = I1 + I2
Vi = Ii – I0 10
V1 = (5||10)I1 = I1
3
 1 −1
[2] =  
 −3 2  ⇒ y11 =
I1
=
3
= 0.3
V1 10
 −2 −1
[ y ] = [2]−1 =   Also, 5Ix = 10(I1 – Ix) = 10I1 – I0Ix
 −3 −1
⇒ 3Ix = 2I1
7. The admittance parameters of a 2–port network shown
in figure are given by Y11 = 2 mho, Y12 = −0.5 mho, Y21 2
⇒ Ix = I1
= 4.8 mho, Y22 = 1 mho. The output port is terminated 3
with a load admittance YL = 0.2 mho. Find E2 for each 4
of the following conditions: I −I
I 2 2 I x − I1 3 1 1 1
(a) E1 = 10∠0°V ∴ y21 = = = = = 0.1
V1 10 10 10
(b) I1 = 10∠0°A I I
3 1 3 1
(c) A source 10∠0° V in series a 0.25Ω resistor is con-
When V1 = 0,
nected to the input port.

[2001]
8. Consider the network in figure 1.
(a) Find its short–circuit admittance parameters. −10
(b) Find the open–circuit impedance Z22.    V2 = I1
3
I1
  ⇒ y12 = = −0.3
V2
2
   I x = I1
3
V2
⇒ I1 + I 2 = + 2I x
10
V2 4
     = + I1
10 3
[2002]
Chapter 5  Two Port Networks  |  1.79

V2 1 I2
   ⇒ I 2 = + I1   ⇒ y22 = =0
10 3 V2
V2 1  −3  0.3 −0.3
  = + V2    ∴ [ y ] =   Ans.
10 3  10   0.1 0 
   ⇒ I2 = 0
y11 0.3
Z 22 =
(b) = = 10Ω Ans.
| y | 0.03
Chapter 6
Graph Theory and
State Equations
One-mark Questions [3
⎡3S + 13⎤
−2] ⎢ ⎥
H(s) = ⎣ − S + 2 ⎦ = 9S + 39 + 2S − 4
1. The network is described by the model is ( S − 2) ( S + 4 ) ( S − 2) ( S + 4 )

x1 = 2x1 – x2 + 3u 11S + 35
∴ H (s) =
x2 = – 4x2 – u ( S − 2) ( S + 4 ) 
y = 3x1 – 2x2 Hence, the correct option is (A).
2. In the following graph, the number of trees (P) and the
⎛ Y (s) ⎞ number of cut–sets (Q) are
The transfer function H(s) ⎜ = ⎟ is [2015]
⎝ U (s)⎠ (1)

11s + 35 11s − 35
(A) (B)
( s − 2) ( s + 4 ) ( s − 2) ( s + 4 )
11s + 38 11s − 38 (2) (3)
(C) (D)
( )( )
s − 2 s + 4 ( − 2) ( s + 4 )
s

⎡ 2 −1⎤ (4)
Solution:  A = ⎢ ⎥
⎣0 −4 ⎦ (a) P = 2, Q = 2 (b) P = 2, Q = 6
⎡3⎤ (c) P = 4, Q = 6 (d) P = 4, Q = 10
B = ⎢ ⎥ ; C = [3 −2]
⎣ −1⎦  [2008]

D = 0, we know Solution: (c)
H(s) = C. (SI – A)–1. B + D Hence, the correct option is (c)
⎡S − 2 1 ⎤ For given graph, number of tree can be calculated as
(SI – A) = ⎢
⎣ 0 S + 4 ⎥⎦ (1)

1
( SI − A) = .adj ( SI − A)
−1

SI − A

|SI – A| = (S – 2) (S + 4)
(2) (3)
1 ⎡ S + 4 −1 ⎤
(SI – A)-1 =
SI − A ⎢⎣ 0 S − 2⎥⎦


1 ⎡ S + 4 −1 ⎤ ⎡ 3 ⎤
∴ H (s) = . [3 −2] ⎢ (4)
( S − 2) ( S + 4 ) ⎣ 0 S − 2⎥⎦ ⎢⎣ −1⎥⎦
Chapter 6  Graph Theory and State Equations  |  1.81

Number of trees = 4

[2004]

(a)

(b)

(c)

And cut set will be drown as


(1)

(4)
(d)

(2) Solution: (b)


(3)
The tree doesn’t form closed loop.
Hence, the correct option is (b)
(5) (6)
4. The differential equation for the current i(t) in the cir-
cuit of the figure is
Total cut–sets = 6
3. Consider the network graph shown in the figure. Which
one of the following is NOT a ‘tree’ of this graph?
1.82 | Network Theory

d 2i di Solution: (c)
2 2 + 2 + i(t ) = sin t
(a) Number of independent loops or mess is given by
dt dt
e = b – (n – 1)
d 2i di
(b) 2 + 2 + 2(t ) = cos t b = e + (n – 1) = 5 + (7 – 1) = 5 + 6
dt dt
b = 11
(c) d 2i di Hence, the correct option is (c)
2 2 + 2 + i(t ) = cos t
dt dt
7. The number of independent loops for a network with n
d 2i di nodes and b branches is
(d) + 2 + 2(t ) = sin t  [2003]
dt 2 dt (a) n–1
(b) b–n
Solution: (c) (c) b–n+1
Appling KVL for given loop (d) independent of the number of nodes [1996]
Solution: (c)
sin t = 2i + 2di + idt
dt ∫ The number of independent loops for a network with n
nodes and b branches is b – (n – 1) = b – n + 1.
Differentiation:–
Hence, the correct option is (c)
2di(t ) 2d 2 i(t )
cos t = + + i (t )
dt dt 2 Two-marks Questions
Hence, the correct option is (c)
1. Relative to a given fixed tree of a network,
5. Identify which of the following is NOT a tree of the
(a) Link currents form an independent set
graph shown in the figure
(b) Branch currents form an independent set
a
(c) Link voltages form an independent set
(d) Branch voltages form an independent set [1992]
1 2 3 Solution:(a)
b c
Link current forms an independent set.
f Hence, the correct option is (a)
d e g

h Five-marks Questions
4 5
1. Refer to the circuit shown in figure
(a) begh (b) defg
(c) adhg (d) aegh [1999]
Solution: (c)
a

1 2 3

g
Choosing the voltage vc(t) across capacitor, and the
d
current iL(t) through the inductor as state variables, i.e.
Vc (t ) 
[ X (t )] =  
h
5  iL (t ) 
4
(c) does not form closed loop. So adhg is not a tree. Write the state equation in the from
Hence, the correct option is (c) d
dt [x(t)] = [A ][ × ( t ) + [B][ u(t)] and find [A], [B] and
6. A network has seven nodes and five independent loops.
The number of branches in the network is [u(t)]. [1996]
(a) 12 (b) 12
(c) 11 (d) 10 [1998]
Chapter 6  Graph Theory and State Equations  |  1.83

Solution: Apply KCL, (b) If e(t) = 0, t ≥ 0, iL1(0) = 0, vo2(0) = 0,2(0) = 0,


CdVC (t ) + VC + i (t ) = I (t ) iLa(0) = 1A, then what would the total energy dissipated
dt R
L in the resistor in the interval (0, ∞) be? [1997]
dV (t ) 1 i (t ) I (t ) 3. For the circuit in figure, write the state equation using
⇒ C + VC + L = vc and iL as state variable.
dt R1C C C
dVC (t ) −1 1 1
⇒ = VC − iL (t ) + I (t ) (1)
dt R1C C C
Apply KVL in right side loop,
diL
L + iL R2 + V (t ) − VL = 0
dt
diL 1 R 1
⇒ = VL − 2 iL − V (t ) (2)
dt L L L
From equation (1) and (2)
 −1 −1  1 
0
VC   R1C C  VC   C  I (t ) 
 =  +  
iL   1 − R2  iL   −1  V (t ) 
  0
 L C   L 
 −1 −1  1 
0
d VC (t )   R1C C  VC (t )   C  I (t ) 
⇒  =  +   Solution:
dt iL (t )   1 − R2  iL (t )   −1  V (t ) 

 L L 
  0 L  Apply KCL at node VC

 −1 −1  dVC  di 
+ VC + VC − L  = 0
RC C  dt  dt 
⇒ A=  1 
 1 − R2  dVC 2diL
  ⇒ = −2VC +  (1)
 L L  dt dt
1  Apply KCL at node B
C 0
B=  2diL diL E
−1  VC − = − + iL
0 dt dt 2
 L 
3diL E
 I (t )  ⇒ = VC − iL +
u (t ) =  dt 2

V (t )  diL 1 1 1
⇒ = VC − iL + E  (2)
2. For the circuit shown in figure choose state variables dt 3 3 6
X1, X2, X3 to be iL1(t), vC2(t), iL3(t) From equation (1) and (2),
dVC 1 1 E
= −2VC + 2  VC − iL + 
dt 3 3 6

= −4 VC − 2 iL + 1 E  (3)
3 3 3
From eq (2) and (3), we get
(a) Write the state equations.  4 2
− − 
VC   3 3 VC  1/ 3 
X   X1    =     +   [E E]
1
    iL   1 1  iL  1/ 6 
− 
 X 2  = A  X 2  + B[e(t )]  3 3
 X 3   X 3 
Chapter 7
Network Functions
One-mark Questions R
+ +

V ( s)
1. The transfer function 2 of the circuit shown below V1
is V1 ( s) C
R1 V0

− −

The value of the load resistance RL is


(a) R/4 (b) R/2
(c) R (d) 2R [2009]
0.5s + 1 3s + 6 Solution: (c)
(a) (b)
s +1 s+2 From the given circuit, transfer function can be given as
s+2 s +1 V0 ( a) RL || 1/sC
(c) (d)  [2013] =
s +1 s+2 Vi ( s) 1
R + RL ||
Solution: (d) sC
From a given circuit, transfer function can be given as 1

V2 ( s) R + 1/SC R
= 1+
1
V1 ( s) R + 2 /SC RL ||
sC
RSC + 1
⇒ 1
RSC + 2 ⇒
R( SCR + 1)
1+
10 × 103 × 100 RL
RC = = 1sec 1
106 Comparing with
2 + sCL
V2 ( s) s + 1 R
= We get =1
V1 ( s) s + 2 RL

RL = R
Hence, the correct option is (d)
2. If the transfer function of the following network is Hence, the correct option is (c)
V0 ( s) 1
=
V1 ( s) 2 + sCR .
Chapter 7  Network Functions  |  1.85

3. The RC circuit shown in the figure is (a) |Rneg| ≤ ReZ1(jw), ∀w


(b) |Rneg| ≤ |Z1(jw)|, ∀w
(c) |Rneg| ≤ |ImZ1(jw)|, ∀w
(d) |Rneg| ≤ |≤∠Z1(jw), ∀w [2006]
Solution:(a)
For Z2(s) to be positive real, Re|Z(s)| ≥ |Rneg|
⇒ |Rneg| ≤ Re |Z1(jw)| for all ω.
Hence, the correct option is (a)
5. The first and the last critical frequencies (singularities)
of a driving point impedance function of a passive net-
(a) a low-pass filter work having two kinds of elements are a pole and a
(b) a high-pass filter zero, respectively. The above property will be satisfied
(c) a band-pass filter by
(d) a band reject filter [2007] (a) RL network only
Solution: (c) (b) RC network only
When w → ∞, capacitor → short circuited. (c) LC network only
Circuit looks like (d) RC as well as RL networks [2006]
Solution: (b)
RC impedance junction has
(i) 1st critical frequency due to pole
(ii) Last critical frequency due to zero
Hence, the correct option is (b)
6. The first and the last critical frequency of an RC-driving
point impedance function must respectively be
(a) a zero and a pole
(b) a zero and a zero
At w → ∞, Capacitor → open circuited. (c) a pole and a pole
(d) a pole and a zero [2005]
Solution: (d)
For stability, poles and zero interface on real axis. Since
its RC, first pole should come and zero at last.
Hence, the correct option is (d)
7. The circuit of the figure represents a

Hence, the correct option is (c)


4. A negative resistance Rneq is connected to a passive
network N having driving point impedance as shown
below. For Z2(s) to be positive real

(a) low-pass filter


(b) high-pass filter
(c) band-pass filter
(d) band-reject filter [2000]
Solution: (d)
Analyzing the circuit for ω = 0
∆ω = ∞;
At ω = 0
1.86 | Network Theory

ωL = 0 → inductor (SC) (a) L = 5H, R = 0.5 W, C = 0.1 F


(b) L = 0.1 H, R = 0.5 W, C = 5 F
(c) L = 5H, R = 2 W, C = 0.1 F
(d) L = 0.1 H, R = 2 W, C = 5 F [2008]
Solution: (d)
Equivalent admittance for given RLC series circuit can
be given as
1
Capacitor = ωC = ∞ (OC) y( s) = 1 + 1 + SC (1)
R sL
V0 RL
= (finite value) 1 s 2 LC
VS RL + RS = +
R sL
At w = ∞
1 s 2 + 0.1s + 2
y ( s) = =
2( s) 0.2 s

s2 0.1s 2
= + +
0.2 s 0.2 s 0.2 s
1 10
V0 = RL (finite value) = 5s + + (2)
2 s
VS RL + RS
by comparing (1) and (2)
R = 2Ω
C = 5F
L = 0.1H
Hence, the correct option is (d)
2. Two series resonant filters are as shown in the figure.
Let the 3-dB bandwidth of Filter 1 be B1 and that of
Filter 2 be B2. The value of B1B2 is

At ω = 1
LC

Hence, the correct option is (d)

Two-marks Questions
1. The driving point impedance of the following network

0.2 s (a) 4 (b) 1


is given by Z ( s) = . The component val-
s 2 + 0.1s + 2 1 1
(c) (d)  [2007]
ues are 2 4
Chapter 7  Network Functions  |  1.87

Solution: (d) 2( S + 3)
R ∴ Z (S ) = 2
Bandwidth of series RLC circuit = S + 2S + 2
L
R R Hence, the correct option is (b)
B1 = ; B2 =
L1 L2 4. Indicate True/False and give reason for the following
question.
4R
B2 =
L1 5
Z ( s) = represents the input impedance of a net-
2
s +4
B1 1 work. [1994]
=
B2 4 Solution: FALSE.

Hence, the correct option is (d) For Z(s) to represent the input impedance of a pas-
3. The driving-point impedance Z(s) of a network has the sive network, the numerator and denominator degrees
pole-zero locations as shown in the figure. If Z(0) = 3, should not differ by more than 1.
then Z(s) is 5. The necessary and sufficient condition for a rational
function of s, T(s), to be a driving point impedance of
an RC network is that all poles and zeros should be
(a) simple and lie on the negative real axis of the s-
plane
(b) complex and lie in the left half of the s-plane
(c) complex and lie in the right half of the s-plane
(d) Simple and lie on the positive real axis of the s-
plane [1991]
Solution: (a)
Simple and lie on the negative real axis of the s-plane.
3( s + 3) 2( s + 3)
(a) 2 (b) The poles and zeros of the ZRC(s) should be simple and
s + 2s + 3 2
s + 2s + 2 alternate on the negative real axis of the s-plane.
3( s - 3) 2( s - 3) Hence, the correct option is (a)
(c) 2 (d)
2
s - 2s - 2 s - 2s - 3 6. A driving point admittance function has pole and zero
 [2003] locations as shown below. The range of s for which the
Solution: (b) function can be realized using passive elements is
From pole –zero diagram Impedance function can be
written as,
K (S − Z )
Z ( s) =
( S − P1 )( S − P2 )
K ( S + 3)
=
( S + 1 + j )( S + 1 − j )
K ( S + 3)
Z (S ) =
( S + 1) 2 − J 2
(a) σ < - 1 (b.) σ > 1
K ( S + 3)
= (c) σ < 1 (d) σ > - 1 [1988]
( S + 1) 2 + 1 Solution: (b)
Z(0) ω =0 = 0 Function can be realized when

σ −1 > 0
3K
⇒ =3⇒ k = 2 σ>1
2
Hence, the correct option is (b)
Unit II
SIGNALS AND SYSTEMS

Chapter 1: Basics of Signals and Systems 2.3


Chapter 2: LTI Systems Continues and Discrete 2.11
Chapter 3: Fourier Series 2.20
Chapter 4: Fourier Transforms 2.25
Chapter 5: Laplace Transforms 2.41
Chapter 6: Z - Transform 2.52
Chapter 7: DTFT and DFT 2.65
Chapter 8: Sampling 2.71
EXAM ANALYSIS
Exam Year 92 93 94 95 96 97 98 99 00 01 02 03 04 05 06 07 08 09 10 11 12 13 14–1 14–2 14–3 14–4 15 16 17 18 19

Set 1 Set 2 Set 3 Set 1 Set 2 Set 3 Set 1 Set 2

1 Mark Questions - - 5 6 3 2 11 3 3 4 3 4 3 6 3 1 2 3 2 3 2 7 4 3 3 4 5 3 3 4 3 3 3 3 3 4

2 Marks Questions 3 4 - 1 1 2 - 2 3 1 5 3 6 6 3 3 8 5 3 4 3 3 4 4 4 4 1 3 4 3 2 6 3 6 6 3

5 Marks Questions - 2 1 2 4 1 1 - 3 1 - - - - - - - - - - - - - - - - - - - - - - - - - -

Total Marks 6 18 10 18 25 11 16 7 24 11 13 10 15 18 9 7 18 13 8 11 8 13 12 9 11 12 7 9 11 10 7 15 9 15 15 10

Chapter wise marks


distribution
Basics of Signals and - - - - - - - - 1 2 - 2 2 2 3 - 3 - - - 2 3 1 3 4 6 2 4 1 1 1
Systems
Divide and Conquer - - - 1 - - 1 - 2 2 1 - 3 1 - - 3 - - 1 2 2 3 2 1 1 4 2 7 - -

Greedy Method 2 2 1 1 1 - 2 2 2 - 1 1 - 1 - - - 1 - 1 - - - - - - 2 1 2 1 -

Dynamic Programming 2 2 1 2 2 3 4 1 1 - 2 5 5 6 1 2 6 3 - - 2 1 - 3 - 2 - 4 1 - -

P and NP Concepts - 4 2 3 2 3 1 1 3 1 2 1 - - 2 1 - 2 2 5 1 4 2 - - - 2 1 - - 1

Optimal Binary Search 2 - - - - - 1 3 - 1 2 1 3 1 1 2 4 3 5 2 1 - 3 3 5 - 4 3 1 - 1


Tree
Miscellaneous Topics - - - - - - - - - - - - - 7 - 2 2 2 1 2 - 2 2 - - 2 1 1 1 1 3

Miscellaneous Topics - - 1 1 - - 2 - - - - - 2 - 2 - - 2 - - - 1 1 - 1 1 2 2 1 1 1
Chapter 1
Basics of Signals and
Systems
(B) f(x)
One-mark Questions
1. Consider the random process –5 0 +5 x
X(t) = U + Vt
Where U is a zero-mean Gaussian random variable and
V is a random variable uniformly distributed between 0 (C) f(x)

and 2. Assume that U and Y are statistically independ-


–5 0 +5 x
ent. The mean value of the random process at t = 2 is
______. [2017]
Solution:  x (t) = U + Vt
At, t = 2s, x(t) = x (2) = U + 2 V (D) f(x)

E [x (t)] = E[U + 2 V]
–5 +5
= E[U] + 2 E[V] 0 x
E [U] = 0

Solution:  F(x) is an implicit function here and from
E[V ] = ∫
−∞
f v (V )dv
the given figure, f(x) is linear so the result will be in
2 quadratic form. Thus option (c) is correct with –ve
1
= ∫ dv = 1 peak.
2
0 Hence, the correct option is (C).
∴ E[ x(t )] = 0 + 2(1) = 2 3. Which one of the following is an Eigen function of
Hence, the correct answer is (1.9 to 2.1). the class of all continuous time, linear, time invariant
­systems (u(t) denotes the unit step function)? [2016]
2. Consider the plot of ƒ(x) versus x as shown below
(A) e jω 0 t u(t ) (B) cos (ω0t)
f(x)
+2
(C) e jω 0 t (D) sin (ω0t)
Solution:  e is an Eigen function and also continu-
jωot
–5 0 +5 x
–2
ous, time invariant and linear out of all given option. So
option (C)
Suppose F(x) = f −5x ƒ(y) dy. Which one of the following Hence, the correct option is (C).
sin( 4π t )
is a graph of F(x)? [2016] 4. The energy of the signal x(t) = is _____.
 4π t [2016]
(A) f(x)
Solution:  The given signal is
sin ( 4π t )
–5 0 +5 x x (t ) = is
4π t
2.4 | Signals and Systems

According to Parseval’s identity, Energy content of x(t) is The filter can be used to approximate a [2016]
∞ ∞
1 (A) low-pass filter (B) high-pass filter
∫ × (ω ) dω
2

2
x(t ) dt =
2π (C) band-pass filter (D) band-stop filter
−∞ −∞ 
Solution:  x[n] → unit delay → x[n – 1]
sin at F .T ⎧1. ω <a
For ←⎯⎯ → Pa (ω ) = ⎨ from the figure
πt ⎩0, ⎧ 1ω >a
1 sin 4π T F .T ⎪ ω < 4π y[n] = 5x[n] – 5x[n – 2]
For ←⎯⎯ → Pa (ω ) = ⎨ 4
4 πt ⎪0,
⎩ ω > 4π y(z) = 5x(z) – 5z–2 X(z)
So
Y ( z)
1
+4π
⎛ 1⎞
2
1 1 H(z) = = 5 − 5 z −2
Energy of x(t) =
2π ∫ ⎜⎝ 4 ⎟⎠ dω = 2π × 16 (8π ) X ( z) 
−4π
Putting z = ejω
1
= = 0.25
4 H(ejω) = 5(1 – e–j2ω)
Hence, the correct Answer is (0.25).
As frequency vary from –∞ to +∞, ω varies from –π to +π
sin(t ) sin(t )
5. If the signal x(t) = * with * denoting the At low frequency ω = –π
πt πt
convolution operation, then x(t) is equal to
( )
H e j (−π ) = 5 (1 – 1) = 0
 [2016]
At high frequency 2 = π
sin(t ) sin( 2t )
(A)
πt
(B)
2π t ( )
H e j (π ) = 5 (1 – 1) = 0
2 π
2sin(t ) ⎛ sin(t ) ⎞ At intermediate frequency ω =
(C) (D)
⎜⎝ ⎟ 2
πt πt ⎠
⎛ jπ ⎞ ⎛ π
− j 2× ⎞
Solution:  We know that convolution of two sinc pulses H ⎜ e 2 ⎟ = 5 ⎜1 − e 2 = 5 (1 – ( –1)) = 10

⎝ ⎠ ⎝ ⎠
is again sinc pulse
The given signal is So it is band pass filter
sin(t ) sin(t ) Hence, the correct option is (C).
x(t) = ∗
πt πt  7. Input x(t) and output y(t) of an LTI system are related
sin(t ) by the differential equation y″(t) – y′(t) – 6y(t) = x(t).
Let, x1(t) = If the system is neither causal nor stable, the impulse
πt 
response h(t) of the system is  [2015]
x(t) = x1(t) ∗ x2 (t) 1 1
(A) e3t u ( −t ) + e −2t u ( −t )
5 5
X(ω) = X1(ω). X2 (ω) = X1(ω)
1 3t 1
(B) − e u ( −t ) + e −2t u ( −t )
∴ X(ω) = X1(ω) 5 5
1 1
x(t) = x1(t) =
sin(t ) (C) e3t u ( −t ) − e −2t u (t )
πt  5 5
1 1
Hence, the correct option is (A). (D) − e3t u ( −t ) − e − u (t )
5 5
6. The direct form structure of an FIR (finite impulse
response) filter is shown in the figure Solution:  y″(t) – y(t) – 6y(t) = x(t)
By taking laplace transforms on both the sides
Unit Unit
x[n] S2Y(S) – Y(S) – 6Y(S) = X(S)
delay delay

5 5 Y(S) [S2 – S – 6] = X(S)



y[n] Y (S ) 1 1
+ H(S) = = 2 =
X (S ) S − S − 6 (S + 2) (S − 3) 
Chapter 1  Basics of Signals and Systems  |  2.5
2
1 1 The power of the7 th harmonic  a7 
H(S) = − Now = 
5 ( S − 3) 5 ( S + 2)  power of the 5 th harmonic  a5 
∵ System is not causal so 1 1
a7 = 2 [1 − 2 j − 2] = 2 7 [ −1 − 2 j ]
1 1 n j7 n j
H(t) = – e +3t u ( −t ) + e −2t u ( −t )
5 5 = ( 2p a7 ) 2 = 5
Hence, the correct option is (B). a5 = ( 2pa5 ) 2 = 7
8. A discrete time signal x[n] = sin (p2n), n being an inte- 2 2
a   5 25
ger, is [2014] ∴ 7  =   = = 0.51
(a) periodic with period p  a5   7 49
(b) periodic with period p2
10. For a periodic signal
(c) periodic with period p/2
v (t) = 30 sin 100t + 10 cos 300t + 6 sin (500t + p/4), the
(d) not periodic
fundamental frequency in rad/s is [2013]
Solution: (d) (a) 100 (b) 300
x(n) = sin (x2n) (c) 500 (d) 1500
2p
N = ×k Solution: (a)
w For 30 sin 100 t → w1 = 100.
2p 2k For 10 cos 300 t → w2 = 300.
∴ N = 2 ×k =
p p For 6 sin 500 t → w3 = 500
where k is min integer so that N is a natural number. Fundamental frequency = HCF of (100, 300, 500)
No integer is possible. = 100
Hence, the correct option is (d). Hence, the correct option is (a).
9. Consider the periodic square wave in the figure shown 11. The input and output of a continuous time system are,
x respectively, denoted by x(t) and y (t). which of the fol-
1 lowing descriptions corresponds to a causal system?
 [2008]
t (a) y (t) = x (t - 2) + x (t + 4)
0 1 2 3 4
(b) y (t) = (t - 4) × (t + 1)
(c) y (t) = (t + 4) × (t - 1)
–1
(d) y (t) = (t + 5) × (t + 5)
The ratio of the power in the seventh harmonic to the
Solution: (c)
power in the fifth harmonic for this waveform is closest
For a causal system, o/p should depend only on present
in value to ____ [2014]
value or past values of I/P.
Solution: 0.51 Hence, the correct option is (c).
For wave form x(t)

12. The Dirac delta function d (t ) is defined as [2006]
x(w ) = ∑ 2pa kd (w − kw )
k = −∞ (a)
1
d (t ) = 
t=0
2 0 otherwise
1
2p ∫0
⇒ ak = x( t ) e − jkw0 t dt ∞ t=0
d (t ) = 
(b)
0 otherwise
1  
1 2

∫ 1 ⋅ e dt + ∫ −1 ⋅ e − jw0 kt dt 

⇒ ak = − jk w0 t
1 t=0
2p  0  d (t ) = 
(c) and ∫ d (t ) dt = 1
0 otherwise
1
−∞
1 2
1  e − jk w0 t  1  e − jk w0 t  ∞ t=0 ∞
=  k w0  +   d (t ) = 
(d) and ∫ d (t ) dt = 1
2p  − j 0 2p  j k w0 1 0 otherwise −∞

1 Solution: (d)
= [ 1 − 2e − jkw0 + 2e − 2 jkw0 ]
2p j kw0 d (t ) is an impulse fxn with infinite amplitude at t = 0
and zero amplitude for other values and also the area
1 1 − 2e − j k n / 2 + 2 e − 2 j k p / 2  ∴ w 0 = p under an impulse function is equal to 1.
kw0  
2p j 2 Hence, the correct option is (d).
2.6 | Signals and Systems

13. The function x(t) is shown in the figure. Even and odd 1
parts of a unit step function u(t) are respectively.[2005] ⇒ x (t )
2
x(t) 1
∴ xo ( t ) = x( t )
2
1
Hence, the correct option is (a).
t 1 4. The power in the signal
0
 p
−1 s( t ) = 8 ( cos )  20 p t −  + 4 sin(15p t ) is [2005]
 2

(a) 40 (b) 41
1 1 −1 1 (c) 42 (d) 82
(a) , x( t ) (b) , x (t )
2 2 2 2 Solution: (a)
For a sinusoidal signal Am cos wmt or Am sin wmt
1 −1 −1 −1
(c) , x(t ) (d) , x(t ),
2 2 2 2 Am2
Power =
2
Solution: (a)
82 4 2
x(t )
∴ Power = + = 32 + 8 = 40
2 2
Hence, the correct option is (a).
1
15. Let d (t) denote the delta function. The value of the

t  3t 
integral ∫ d (t ) cos  2  dt is
−∞
[2001]

(a) 1 (b) -1
Unit step function
p
For even function, even part is given as, (c) 0 (d)
2
x (t ) + x ( −t ) Solution: (a)
Now, xe (t ) = for a function u(t)
2 As for function f (t)
u (t ) + u ( − t ) 1 ∞
xe (t ) = =
2 2 ∫
−∞
f (t ) d (t ) dt = f (o)

u (−t ) 1/2 ∞
 3t  3× 0
So, ∫ d (t ) cos  2  dt = cos2
=1
1 ⇒ −∞
Hence, the correct option is (a).
16. If a signal f (t) has energy E, the energy of the ­signal f
(2t) is equal to [2001]
1 E
∴ x e (t ) = (a) E (b)
2 2
(c) 2 E (d) 4 E
For odd function,
Solution: (b)
x0 (t ) − x0 ( −t ) u(t ) − u( −t ) 1 ∞
now x0 (t ) = = = x (t ) E= ∫f
2
( t ) dt
2 2 2 −∞

E 1 = ∫ f 2 ( 2t ) dt
1/2 ∞

du E
= ∫f =
2
0
(u)
−∞
2 2
−1/2 Let 2t = u
Then 2 dt = du
∴ E1 = E/ 2
Hence, the correct option is (b).
Chapter 1  Basics of Signals and Systems  |  2.7
⎛ t − 1⎞
17. A system with an input x(t) and output y(t) is described A signal g(t) is defined by g(t) = x ⎜ .The average
⎝ 2 ⎟⎠
by the relation: y(t) = T × (t). This s­ ystem is________. power of g(t) is _____
 [2000] Solution:  g(t) =
(a) linear and time invariant
g(t)
(b) linear and time varying
(c) non-linear and time invariant
(d) non-linear and time varying
Solution: (b) –2
y(t) = tx(t) –6 –4 0 2 4 6 8 10 t
Let x (t) = x1 (t); then y (t) = t x1 (t).
Similarly for x (t) = x2 (t) ∴ y2 (t) = tx2 (t) –3
∴ y1 (t) + y2 (t) = t [x1 (t) + x2 (t)](1)
Now let x (t) = x1 (t) + x2 (t)
+∞
then y (t) = t [x1 (t) + x2 (t)](2) 1
∫ g (t ) dt
2
Now average power = Lim
As equations (1) and (2) are equal, so it follows super- T →∞ T
−∞
position principle.
Now let x′(t) → ax (t) ⎧ 3
⎪ − x + 3, 0 ≤ t ≤ 4
Then y′(t) = at x (t) Now g(t)T = ⎨ 2
Also a y (t) = at x (t) ⎪⎩ 0 4≤ t ≤ 6
Hence, homogeneity is also satisfied, so, linear system 
Period T = 6
Also y (t - to) = (t - to) x(t - to)
4
So, the system is a time varying system. 1 ⎛ 3 ⎞
Hence, the correct option is (b).
So Pav = ∫
60
⎜⎝ − x + 3⎟⎠ dx
2

4
1 ⎡9 2 ⎤
Two-mark Questions = ∫ ⎢ x + 9 − 9x⎥
6 0 ⎣4 ⎦
⎛ 2π ⎞
1. Consider the signal f(t) = 1 + 2 cos(p t) + 3sin ⎜ t +
⎝ 3 ⎟⎠ 1 ⎡ 9 x3 9x2 ⎤
4

⎛π π⎞ = ⎢ . + 9x − ⎥
4 cos ⎜ t + ⎟ , where t is in seconds. Its fundamen- 6 ⎣4 3 2 ⎦0
⎝2 4⎠ 
tal time period, in seconds, is _____. [2019] 1 ⎡9 4×4×4 9 ⎤
= ⎢ . + 9 × 4 − × 4 × 4⎥
Solution: 6 ⎣4 3 2 ⎦
 2Πt   Πt Π  1
f (t ) = 1 + 2 cos Πt + 3 sin  + 4 cos  +  = [48 + 36 − 24 × 3]
  3   2 4 6
T1    
T2 T3
= 14 – 12 = 2
2Π 2Π 2Π Hence, the correct Answer is (2).
T1 = = 2; T2 = = 3; T3 = =4
Π 2Π Π
3 2 3. Let h(t) denote the impulse response of a causal system
f(t) 1
with transfer function .
s +1
T = LCM (T1 , T2 , T3 )
Consider the following three statements
= LCM ( 2, 3, 4) S1: The system is stable
= 12 S2:
n(t +1)
is independent of t for t > 0
Hence, the correct answer is 12. n(t )
2. The waveform of a periodic signal x(t) is shown in the S3: A non-causal system with the same transfer function
figure. [2015] is stable.
x(t)
For the above system [2014]
(a) only S1 and S2 are true.
3
(b) only S2 and S3 are true.
–2 1 4 (c) only S1 and S3 are true
t
–4 –3 –1 2 3 (d) S1, S2 and S3 are true.
–3
2.8 | Signals and Systems

Solution: (a) For f (t ) ↔ F ( s)

1 − d F ( s)
T ( s) = t . f (t ) ↔
s +1 ds
h(t ) = L−1 [T ( s)] = e −1 d 1 −[ −( 2 s + 1)]
− = 2
That is, a decaying exponential, so the system is stable. ds ( s + s + 1) ( s + s + 1) 2
2

h(t + 1) e − ( t +1) 2s + 1
= − t = e −1 =
h(t ) e ( s + s + 1) 2 2

e-1 is independent of Z. Hence, the correct option is (d).


A non-causal system for the same transfer function will 6. A stable linear time-invariant (L T I) system has a trans-
not include jw as s will be less than 1. So, system will 1
not be stable. fer function H ( s) = 2 .To make this system
s + s−6
Hence, the correct option is (a). causal it needs to be cascaded with another LTI system
4. Let x(t) be a wide sense stationary (wss) random with having a transfer function H1(s). A ­correct choice for
power spectral density sx( f ). If y (t) is the process H1(s) among the following option is. [2014]
defined as y(t) = x (2t – 1), the power spectral density (a) s = 2 pole should be added
Sy( f ) is____ [2014] (b) s = 2 zero should be added
(c) s = 3 pole should be added
1  f  − jp f
sy ( f ) =
(a) sx e (d) s = 3 zero should be added
2  2 
Solution: (b)
1  f  − jp f / 2
sy ( f ) =
(b) sx e
2  2  H ( s) =
1
=
1
s + s − 6 ( s + 3)( s − 2)
2

1  f
sy ( f ) =
(c) sx    for the system to be causal. All poles must lie on
2  2 the left half plane. So, for making the system ­causal s =
1  f  j 2p f 2, zero should be added.
sy ( f ) =
(d) sx e
2  2  Hence, the correct option is (b).
7. A causal LTI system has zero initial conditions and
Solution: (c) impulse response h(t). Its input y(t) and output x(t) are
In case of PSD, no effect of shifting takes place. related through the linear constant coefficient differen-
for x(t ) PSD
 → sx ( f ) tial equation.

1 d 2 y(t ) a dy (t )
x( 2t )  → s x ( f / 2)
PSD + + a 2 y (t ) = x (t )
2 dt 2 dt
Let another signal g(t) be defined as
1
x( 2t − 1) → sx ( f / 2) t
2 dh (t )
g (t ) = a 2 ∫ h(t ) dt + + ah(t )
1 dt
s y ( t ) → s x ( f / 2) 0
2
If G(s) is the Laplace transform of g(t), then the num-
Hence, the correct option is (c). ber of poles of G(S) is______. [2014]
1 Solution: 1
5. The unilateral Laplace transform of f(t) is .
s2 + s + 1
Which one of the following is the unilateral Laplace d 2 y(t ) a dy (t )
+ + a 2 y (t ) = x (t )
transform of g (t) = t . f (t)? [2014] dt 2 dt
−s −( 2 s + 1) ( s 2 + a s + a 2 ) y ( s) = x ( s)
(a) 2 (b)
( s + s + 1) 2 ( s 2 + s + 1) 2 y ( s) 1
H ( s) = = 2
s 2s + 1 x ( s) s + a s + a 2
(c) 2 (d)
( s + s + 1) 2 ( s 2 + s + 1) 2 t
dn (t )
Solution: (d) Now g(t) = a 2 ∫ h (t ) dt + + h(t )
0
dt
1
f (3) =
s2 + s + 1
Chapter 1  Basics of Signals and Systems  |  2.9

a 2  Properties
G ( s) =  + s + a  H ( s) P1 = linear but not time invariant
 s 
P2 = Time invariant but not linear
a 2 + s2 +a s 1 1 P3 = linear and time invariant
= × 2 = sw Relations
s s + as + a 2
s
  R1 = y (t) = t2x(t)
Number of poles at G(s) = 1.
R2 = y (t ) = t x(t )
8. The impulse response of a continuous time system is
given by h(t ) = d (t − 1) + d (t − 3). The value of the step R3 = y(t ) = x(t )
response at t = 2 is [2013] R4 = y (t) = x (t - s)
(a) 0 (b) 1
(c) 2 (d) 3 (a) (P1, R1), (P2, R3), (P3, R4)
(b) (P1, R2), (P2, R3), (P3, R4)
Solution: (b) (c) (P1, R3), (P2, R1), (P3, R2)
step response = ∫ Impulse response, dt (d) (P1, R1), (P2, R2), (P3, R3)
∴ step response = u (t-1) + u (t-3) Solution: (a)
At t = 2 step response = u (1) + u(- 1) R1 = y(t) = t2x(t)
As u(t) = 0 for t < 0 ∴u (-1) = 0 a1y1 (t) + a2y2 (t) = t2 [a1x1 (t) + a2x2 (t)]
Hence step response = 1. ∴ Linear system
Hence, the correct option is (b). y(t - t0) = (t - t0)2x(t - t0)
Not time invariant
9. The input x(t) and output y(t) of a system are related as
t R2 : y(t ) = t | x(t ) |
y(t ) = ∫ x(t ) cos(t )dt . The system is [2012] This system is not linear and also time variant
(a) time
−∞ invariant and stable
R3 : y(t) = |x(t)|
(b) stable and not time invariant This system is not linear as if a -ve then ay(t) can be
(c) time invariant and not stable -ve but |ax (t)| = +ve
(d) not time invariant and not stable y (t - t0) = |x (t - t0)
Solution: (d) ∴ No delay, invariant system
t
R4 = y(t) = x(t - 5)
a1y1(t) + a2y2(t) = a1x1(t- 5) + a2x2(t- 5)
y 1 ( t )x =( t − t0 ) = ∫ x( t − t
−∞
0 ) cos( 3 ( t − t0 )) dt
So, linear system
t Also y(t - t0) = x(t - t0 - 5)
y 1 ( t ) = ∫ x( t − t0 )cos( 3 ( t − t0 )) dt So, time-invariant system.
Hence, the correct option is (a).
t − t0
11. A system with input x[n] and output y[n] is given as
y ( t − t0 ) = ∫ x( t )cos( 3t ) dt
 5 
−∞ y[ n ] =  sin p n × ( n ) x[n]. The system is [2006]
∴ System is time variant  6 
Also for an I/p like as shown (a) linear, stable and invertible
(b) non-linear, stable and non-invertible
x(t ) (c) linear, stable and non-invertible
(d) linear, unstable and invertible
Solution: (c)
For linearity
 5 
x( n) = x1 ( n) ⇒ y1 ( n) =  sin pn x1 ( n)
 6 
x(t) cosw0 t will be +ve and hence y(t) will be infinite,
and so an unstable system.  5 
For x( n) = x2 ( n) ⇒ y2 ( n) =  sin pn x2 ( n)
Hence, the correct option is (d).  6 
10. Let x(t) be the input and y(t) be the output of a con- If
tinuous time system. Match the system properties P1, P2,  5 
and P3 with system relations R1, R2, R3, R4. [2008] x( n ) = x1 ( n ) + x2 ( n ) ⇒ y( n ) =  sin pn
 6 
( x1 ( n ) + x2 ( n ))
2.10 | Signals and Systems

Solution: (a)
∴ y( n) = y1 ( n) + y2 ( n)
I
Also ay (n) = F[ a x (n)] As the signal is varying at different time i­nstances,
∴ Linear system P
so it is a time variant system.
Also for a bounded I/P we get bounded o/p Also y(n) = x(n + 1), so it depends on future values,
So, stable system I
Now if n = 0, y(n) = 0 hence a non-casual system. For a bounded this sys-
P
n = 2, y(n) = 0 tem provides bound of P. So, it is stable
I
If for different values of , y (n) is same then the sys- Also, this satisfies homogeneity and super position the-
tem is non-invertible. P orem and so it is linear.
Hence, the correct option is (c). Hence, the correct option is (a).
12. Consider the sequence x[n] = [–4 – 5j1 + 2j 4]. 14. An excitation is applied to a system at t = T and its
The conjugate anti-symmetric part of the sequence is response is zero for –∞ < t < T. Such a system is a
 [2004]  [1991]
⇒ x (a)
CAS ( n ) = [ −4 − 2 ⋅ 5 j 2 j 4 − 2 ⋅ 5 j ] (a) non-causal system
(b) [ -j + 1 j2 -5] (b) unstable system
(c) [-j5 j2 0] (c) causal system
(d) [-4 1 4] (d) unstable system
Solution: (a) Solution: (c)
For a conjugate anti-symmetric part For a system whose o/p is zero at the instant as t < T
I
x ( n ) − x *( − n ) when the is applied on T is termed as causal ­system.
x CAS ( n ) = P
2 Hence, the correct option is (c).
⇒ x( n) = [ −4 − 5 j 1 + j 2 4]
⇒ x *( − n) = [4 1 − j2 − 4 + 5 j]
Five-marks Question
1. A system having a unit impulse response h(n) = u(n) is
x( n) − x *( − n)  5 5  excited by a signal x(n) = a n u(n). Determine the output
⇒ =  −4 − j 2 j 4 − j
2  2 2  y(n) [1996]
⇒x CAS ( n ) = [ −4 − 2 ⋅ 5 j 2 j 4 − 2 ⋅ 5 j ] Solution:

Hence, the correct option is (a). y ( n) = x ( n) * h( n) = ∑ h( k ) . x ( n − k )
1 3. Let P be linearity, Q be time invariance, R be c­ ausality k = −∞

and S be stability. A discrete time system has the input– given, x(n) = a nu(n)
output relationship.   h(n) = u(n)
n
 x( n) n ≥1 y (n) = ∑ a n − k for n ≥ 0,
 So,
y ( n) = o n=o k =0

 x( n + 1) n ≤ −1
 n
a n +1 − 1
= a n ∑ a −k = n≥0
where x(n) is the input and y(n) is the output. The above k =0 a −1
system has the properties [2003]      = 0 n < 0
(a) P, S but not Q, R
(b) P, Q, S, but not R
(c) P, Q, R, S
(d) Q, R, S but not P
Chapter 2
LTI Systems Continues
and Discrete
Which one of the following statements is true about the
One-mark Questions system?
(A) It is causal and stable
1. Let the input be u and the output be y of a system, and (B) It is casual but not stable
the other parameters are real constants. Identify which (C) It is not casual but stable
among the following systems is not a linear system: (D) It is neither casual nor stable
 [2018]
Solution: 
d3 y d2 y dy du d 2u
(A) 3 + 2 + a2 + a3 y = b3u + b2 + b1 2 y {n} = 
dt dt dt dt dt nx{n}; for 0 ≤ n ≤ 10

(with initial rest conditions)  x{n} − x{n − 1}; otherwise
t
y (t) =
(B) ∫ e a(t − r ) β u(τ )dτ The output in both the case is depending on either the
0 present or past values of input, so it is causal and in
(C) y = au + b, b ≠ 0 both the cases, bounded input will lead to bounded out-
(D) y = au put, Hence, stable.
Solution:  4. Consider the following statements about the linear
Hence, the correct option is (C) dependence of the real valued functions y1 = 1, y2 = x
and y3 = x2 over the field of real numbers.
2. A discrete-time all-pass system has two of its poles at
0.25∠0° and 2∠30°. Which one of the following state- I.  y1, y2, and y3 are linearly independent on -1 ≤ x ≤ 0
ments about the system is TRUE? [2018] II. y1, y2, and y3 are linearly dependent on 0 ≤ x ≤ 1
(A) It has two more poles at 0.5∠30° and 4∠0°. y1, y2, and y3 are linearly independent on 0 ≤ x ≤ 1
III. 
(B) It is stable only when the impulse response is two- IV. y1, y2, and y3 are linearly dependent on -1 ≤ x ≤ 0
sided.
(C) It has constant phase response over all f­ requencies. Which one among the following is correct? [2017]
(D)  It has constant phase response over the entire (A) Both I and II are true
­z-plane. (B) Both I and III are true
(C) Both II and IV are true
Solution: 
(D) Both III and IV are true
Hence, the correct option is (B)
Solution:  Given y1 = 1, y2 = x, and y3 = x2
3. Consider a single input and single output discrete-time
For -1 ≤ x ≤ 0 or 0 ≤ x ≤ 1, the linear combination of y1,
system with x[n] as input and y[n] as output, where the
y2, and y3, ay1 + by2 + cy3 = 0 only when a = b = c = 0
two are related as: [2017]
\ y1, y2, and y3 are linearly independent on -1 ≤ x ≤ 0
 n x [ n] , as well as on 0 ≤ x ≤
y [ n] =  for 0 ≤ n ≤ 10
 x [ n] − x [ n − 1] , \ Both I and III are true Choice (B)
Hence, the correct option is (B).
2.12 | Signals and Systems

5. Consider the following statements for continuous-time α α


3 = =
linear time invariant (LTI) systems.  [2017] 1 − 0.5 0.5 
I. There is no bounded input bounded output (BIBO) α = 3 × 0.5 = 1.5
stable system with a pole in the right half of the
complex plane. Hence, the correct Answer is (1.49 to 1.51).
II. There is no causal and BIBO stable system with a 9. A continuous, linear time-invariant filter has an impulse
pole in the right half of the complex plane. response h(t) described by
Which one among the following is correct? 3 for 0 ≤ t ≤ 3
(A) Both I and II are true h (t ) = 
(B) Both I and II are not true 0 otherwise
(C) Only I is true
When a constant input of value 5 is applied to this filter,
(D) Only II is true
the steady-state output is ____________. [2014]
Solution:  Only I is true a causal system may be stable
Solution: 45
may not be stable [ e.g., et u(t) is causal but unstable due
to RHP.] 3 for 0 ≤ t ≤ 3
h (t ) = 
Hence, the correct option is (D). 0 otherwise
6. An LTI system with unit sample response h[n] = 5δ [n] − 7δ [n − 1] + 7δ [n − 3x(t) δ [5n − 4]
] − 5=
h[n] = 5δ [n] − 7δ [n − 1] + 7δ [n − 3] − 5δ [n − 4] is a  [2017]
∞ 3
3e − st
3
(A) Low-pass filter (B) High-pass filter H (s ) = ∫ h (t )e dt = ∫ 3e dt =
− st − st

(C) Band-pass filter (D) Band-stop filter −∞ 0


−s 0

7. The input x(t) and the output y(t) of a continuous-time 1 − e −3s 


= 3 
system are related as y(t ) = ∫
t
x(u )du. The system is  s 
t −T
5
 [2017] x (s ) =
s
(A) Linear and time-variant  1 e −3s  5
(B) Linear and time-invariant y (s ) = x (s ) H (s ) = 3  − .
s s  s
(C) Non-linear and time-variant
(D) Non-linear and time-invariant
For steady-state output
8. Two sequences x1[n] and x2[n] have the same energy.
1 − e −3s  5
Suppose x1[n] = α0.5n u[n], where α is a positive real Lt y (t ) = Lt Sy (s ) = Lt 3 s  .
number and u[n] is the unit step sequence. Assume t →∞ s →0 s →0
 s  s
 [2015]
5×3
⇒ Lt [1 − e −3s ] = Lt [1 + e −3s ]3 × 3 × 5 = 45
⎧⎪ 1.5 for n = 0,.1 s →0 s s →0
x2[n] = ⎨
⎩⎪ 0 otherwise
10. The input −3e−2tu(t), where u(t) is the unit step func-
Then the value of α is _______ s−2
tion, is applied to a system with transfer function .
s +3
Solution:  Energy of x2[n] = ∑ ⎡⎣ x [ n]⎤⎦
2
If the initial value of the output is −2, then the value of

1
the output of steady state is _________ [2014]
= ∑ 1.5 = 1.5 + 1.5 Solution: 0
n = 0 x (t ) = −3e −2t u (t )
= 3
Now  Energy of x1[n] = Energy of x2[n] s−2
H (s ) =
+∞ s +3
∑ α (0.5) u [n]
n
So 3 = −3
n = −∞  x (s ) =
∞ s+2
= ∑ α (0.5)
n
 s − 2   −3 
n= 0  Lt y (t ) = Lt Sy (s ) = Lt S   =0
x →∞ y →0 y →0
 s + 3  s + 2 
Chapter 2  LTI Systems Continues and Discrete  |  2.13

11. The sequence x[n] = 0.5nu[n], where u[n] is the unit Solution: (c)
For a causal and stable system, all poles must lie within
step sequence, is convolved with itself to obtain y[n.]
+∞ |s| = 1.
Then ∑ y [n ] is ____________
n = −∞
[2014] Hence, the correct option is (c).

Solution: 4 14. A system is defined by its impulse response h(n) =


Given x(n) = 0.5nu[n] 2nu(n - 2), the system is [2011]
(a) stable and causal
y ( n ) = x ( n ) ⊗ h( n ) (b) causal but not stable
∞ (c) stable but not causal
y( n) = ∑ x(k )h(n − k )
k =−∞
(d) unstable and non-causal
∞ Solution: (b)
y( n) = ∑ 0.5 µ(k ).0.5
k =−∞
k n− k
u( n − k ) h(n) = 2nu(n - 2)
This system depends only on present or past ­inputs, so,
y(n) = (0.5) (n + 1) μ(n)
n
causal system.

Let A = ∑ a n ( n + 1) also at z = 0.5 For 2nu(n - 2)
n=0 1

∴ A = ∑ a n ( n + 1) 2
n=0

∴ Solving above equation 1 • • • •


1
A (1 − z ) =
1− z
1
A=
(1 − z ) 2 So, it is an increasing function and hence an unstable
.1 1 system.
A= = =4 Hence, the correct option is (b).
(1 − 0.5) 2 (0.5) 2
= x(0) h(4) + x(1) h(3) + x(2) h(2) 15. The impulse response h(t) of a linear time-invariant
+ x(3) h(1) + x(4) h(0) continuous time system is described by h(t) = exp (dt)
=0+3+4+3+0 ut + exp (bt) u(−t), where u(t) denotes the unit step
= 10 function and a and b are real constants. This system is
stable if [2008]
12. Two systems with impulse response h1(t) and h2(t)
are connected in cascade. Then the overall impulse (a) a is positive and b is positive
response of the cascaded system is given by [2013] (b) a is negative and b is negative
(a) product of h1(t) and h2(t) (c) a is positive and b is negative
(b) sum of h1(t) and h2(t) (d) a is negative and b is positive
(c) convolution of h1(t) and h2(t) Solution: (d)
(d) subtraction of h1(t) from h1(t)
Solution: (c) h (t )e at m (t ) + e bt m ( −t )
For the two systems connected in cascade, output is Here for a < 1
given as convolution of transfer fxn of two ­systems.
eαt eαt
Hence, the correct option is (c).
13. Which one of the following statements is NOT TRUE
for a continuous time causal and stable LTI system? t
 [2013]    t
(a) All the poles of the system must lie on the left side ∴a should be -ve
of the jw axis for B > 1
for B < 1
(b) Zeros of the system can lie anywhere in the s-plane
(c) All the poles must lie with |s| = 1
(d) All the roots of the characteristics equation must be
located on the left side of the jw axis.

  
2.14 | Signals and Systems

∴ b should be greater than 0, i.e. +ve 18. Convolution of x(t + 5) with impulse function
Hence, the correct option is (d). d(t - 7) is equal to [2002]
16. Which of the following can be impulse response of a (a) x(t - 12) (b) x(t + 12)
causal system? (c) x(t - 2) (d) x(t + 2)
(a) h(t) Solution: (c)
x (t + 5) ⊗ d (t − 7) = x (t − 2)
[f (t ) ⊗ d (t − t 0 ) = f (t − t 0 )]
t Hence, the correct option is (c).
(b) h(t)
1 9. The unit impulse response of a linear time-invariant
system is the unit step function u(t). For t > 0, the
response of the system to an excitation e−atu(t), a > 0
will be [1998]
t
1
(c) h(t)
(a) ae−at (b)  a  1− e
 
− at
( )
(c) a(1 − e−at) (d) 1 − e−at
Solution: (b)
t y (t ) = u (t ) ⊗ e − at u (t )
(d) h(t)  [2005] ∞

∫ u(t )e
− at
= u (t −)dt
−∞
t1
e − at
t
1
= ∫ e dt =
− at
= 1 − e − at 
a
t
0
−a 0

Solution: (b)
For a causal system h(t) = 0 for t < 0. Hence, the correct option is (b).
Hence, the correct option is (b). 2 0. Let h(t) be the impulse response of a linear time in
17. The impulse response h[n] of a linear time-invariant sys- variant system. Then the response of the system for any
tem is given by input u(t) is [1995]
h[n] = u[n + 3] + u[n - 2] - 2u[n - 7], t

where u[n] is the unit step sequence. The above system ∫ h(t )u(t − t )dt
(a)
0
is [2004]
(a) stable but not causal d
t

(b) stable and causal (b) ∫ h (t )u (t − t )dt


dt 0
(c) causal but unstable
(d) unstable and not causal t
t 
∫0 ∫0 h(t )u(t − t )dt  dt
(c) 
Solution: (a)  
t

∫ h (t )u(t − t )dt
2
2 (d)
0

1
Solution: (a)

7
y (t ) = x (t ) ⊗ h (t )
h(n)
−3 2 = u (t ) ⊗ h (t )

= ∫ h(t )u(t − t )dt


−∞
−2
As u(t) = 0 for t < 0
∴ As the output amplitude is finite for finite input, sys- t

tem is stable. But as u(n + 3) term is present so not = ∫ h (t )u (t − t )dt


0
causal.
Hence, the correct option is (a). Hence, the correct option is (a).
Chapter 2  LTI Systems Continues and Discrete  |  2.15

1 1 − jω 1 − j 2ω
Two-marks Questions H (ω ) = + e + e
3 3 3
1. Two discrete-time signals x [n] and h [n] are both non- 2π
zero only for n = 0, 1, 2 and are zero otherwise. It is if ω = ; H (ω ) = 0
3
given that x [0] = 1, x[1] = 2, x[2] = 1, h[0] = 1.
Let y [n] be the linear convolution of x [n] and h [n]. So, w = 2.0943 rad/sec
Given that y [1] = 3 and y [2] = 4, the value of the Hence, the correct answer is (2.05 to 2.15).
expression (10y [3] + y[4]) is ________.  [2017]
3. A continuous time signal x(t) = 4 cos (200 p t) + 8 cos
Given x( n) = {1, 2,1}
Solution:  (400 p t), where t is in seconds, is the input to a linear
time invariant (LTI) filter with the impulse response


h( n) = {1, b, c}  2 sin ( 300π t )
h(t ) =  , t≠0
↑ πt

 600, t = 0
using cross table method
1 b c Let y (t) be the output of this filter. The maximum value
of y(t ) is ____________. [2017]
Solution:  The Frequency response of the filter can be
1 1 c
b given as
2
H (ω )
2

−300π 0 300π ω
y(0) = 1; given y (1) = 3
In the input x(t) = 4 cos (200pt) + 8 cos (400pt) only the
⇒2+b=3 ⇒b=1 first component is passed through the system.
given y (2) = 4 Output of the system is
⇒2b+c+1=4 y(t) = 8cos (200pt)
⇒c=4–2–1 y (t ) = 8

⇒c=1
Hence, the correct answer is (7.9 to 8.1).
y (3) = b + 2c = 3
4. Consider an LTI system with magnitude response
y (4) = c = 1
H( f ) = 
10y (3) + y (4) = 10 × 3 + 1 = 31.  f
 1− f ≤ 20 and phase response arg
Hence, the correct answer is (31 to 31). 20

2. Let h [n] be the impulse response of discrete-time lin-  0, f > 20
ear time invariant (LTI) filter. The impulse response is {H ( f )} = −2 f
1 1 1
given by=h(0) = ; h[1=
] ; h[2] = ; and h[n] 0 for n
3 3 3 If the input to the system is
< 0 and n >2.  π  π  π 
x(t ) = 8 cos  20π t +  + 16 sin  40π t +  + 24 cos  80π t + 
Let H (w) be the discrete-time fourier transform  4  8  16 
(DTFT) of h[n], where w is normalized angular fre-
 π  π  π 
quency in radians. Given that H(w0) x=(t0) = 8 cos
and w0π<t +  + 16 sin  40π t +  + 24 cos  80π t +  then the aver-
0 < 20

p, the value of w0 (in radians) is equal to _________. 4  8  16 
 [2017] age power of the output signal y(t) is ___________.
 [2017]
Solution:  Given
1 1 1 5. The transfer function of a causal LTI system is H(s) =
h[0] = ; h[1] = ; h[2] = 1/s. If the input to the system is x(t ) = [sin(t ) /π t ]u(t ),
3 3 3

H (ω ) = ∑ h ( n ) e − jω n where u(t) is a unit step function, the system output y(t)
n −∞ as t → ∞ is ___________. [2017]
2.16 | Signals and Systems

6. Consider the parallel combination of two LTI systems


y(t) = −e j 2ω 0 t − −e − j 2ω 0 t + − e j 3ω 0 t − − e − j 3ω 0 t 
shown in the figure
for k = 3, C3 = 1
h1(t)
Hence, the correct option is (B).
x(t) + y(t) 8. A second order linear time invariant system is described
by the following state equations
h2(t)
d
x (t) + 2x1(t) = 3u(t)
dt 1
The impulse responses of the systems are
d
h1 (t ) = 2δ (t + 2) − 3δ (t + 1) x (t) + x2(t) = u(t)
dt 2
h2 (t ) = δ (t − 2)
Where x1(t) and x2(t) are the two state variables and u(t)
If the input x(t) is a unit step signal, then the energy of denotes the input. If the output c(t) = x1(t), then the sys-
y (t) is _______. [2017] tem is [2016]
(A) controllable but not observable
⎛ 2π ⎞ (B) observable but not controllable
7. A signal 2cos ⎜ t –cos(pt) is the input to an LTI
⎝ 3 ⎟⎠ (C) both controllable and observable
system with the transfer function H(s) = es + e–s. (D) neither controllable nor observable
If Ck denotes the kth coefficient in the exponential d
Fourier series of the output signal, then C3 is equal to Solution:  x1 (t ) + 2 x1 (t ) = 3u(t )
dt
 [2016]
(A) 0 (B) 1 d
(C) 2 (D) 3 x2 ( t ) + 2 x2 ( t ) = u ( t )
dt 
Solution:  By writing in Fourier form (s = jω) Output c(t) = x1(t)
H(ω) = ejω + e–jω = 2 cosω ⎡ x1 (t ) ⎤ ⎡ −2 0 ⎤ ⎡3⎤
⎢ ⎥=⎢ ⎥ x (t ) + ⎢ ⎥ u (t )
⎛ 2π ⎞ ⎣ x2 ( t ) ⎦ ⎣ 0 1 ⎦ ⎣1 ⎦
Input x(t) = 2cos ⎜ t − cos (π t )
⎝ 3 ⎟⎠ c(t) = [1  0]x(t)

When input is componential input x(t) = Aejωt test for controllability:
Output y(t) = H(ω)Aejωt Tc = [B AB]
2π 2π ⎡ −2 0 ⎤ ⎡3⎤ ⎡ −6 ⎤
x(t) = e +e
j
3
1
t −j
3
t
− ⎡⎣e jπ t + e − jπ t ⎤⎦
AB = ⎢ ⎥⎢ ⎥ = ⎢ ⎥
2 ⎣ 0 1 ⎦ ⎣1 ⎦ ⎣1 ⎦ 

π ⎡3 −6 ⎤
Fundamental frequency (ω0) = Tc = ⎢
3 ⎥
⎣1 1 ⎦ 
1 1 ITcI ≠ 0
x(t ) = e j 2ω 0 t + e − j 2ω 0 t − e j 3ω 0 t − e − j 3ω 0 t y(t)
2 2
∴ It is controllable
=
H(ω) ⋅ ejωt
Test for observability:
H(2ω0) e j 2ω 0 t + H(–2ω0) e − j 2ω 0 
= T0 = [CT ATCT]

1 1 ⎡ −2 0 ⎤ ⎡1 ⎤ ⎡ −2⎤
− H (3ω 0 )e j 3ω 0 t − H ( −3ω 0 )e − j 3ω 0 t
2 2  AT C T = ⎢ ⎥⎢ ⎥ = ⎢ ⎥
⎣ 0 1 ⎦ ⎣0 ⎦ ⎣0 ⎦ 
⎛ 2π ⎞
H(2ω0) = 2cos(2ω0) = 2cos ⎜ ⎟ = −1 ⎡1 −2⎤
⎝ 3⎠ T0 = ⎢
 ⎥
H(–2ω0) = 1 ⎣0 0 ⎦ 

π⎞ IT0I = 0

H(3ω0) = 2cos(3ω0) = 2cos ⎜ 3 × ⎟ = −2
⎝ 3⎠ ∴ so it is not observable

H(–3ω0) = 2 Hence, the correct option is (A).
Chapter 2  LTI Systems Continues and Discrete  |  2.17

9. The impulse response of an LTI system can be obtained So h[n] is real for all n
by [2015] Hence, the correct option is (A).
(A) Differentiating the unit ramp response. ∞
(B) Differentiating the unit step response. 12. For a function g(t), it is given that ∫ g( t ) = e jwt dt = we −2 w
2

(C) Integrating the unit ramp response. −∞



(D) Integrating the unit step response. t

∫ g( t ) = e jwt dt = we −2 w for any real value w. If y( t ) = ∫ g( t )dt , then


2

Solution:  The impulse response of an LTI system −∞


can −∞
be obtained by differentiating the unit step response. ∞

Hence, the correct option is (B). ∫ y (t )dt is _________


−∞
[2014]

10. The result of the convolution x(–t) * δ(–t – t 0) is (a) 0 (b) -j


 [2015]
(A) x(t + t0) (B) x(t – t0) -j j
(c) (d)
(C) x(–t + t0) (D) x(–t – t0) 2 2
Solution:  The result of convolution x (–t) * δ (–t – t0) is Solution: (b)
Convolution with delta function gives the same func- ∞
tion with shifting
∫−∞ g (t )e dt = we
− jwt −2w 2

So x(–t) * δ(–t – t0) = x(–t – t0)


2
Hence, the correct option is (D). G (w ) = we −2w
11. The pole-zero diagram of a causal and stable discrete- t

time system is shown in the figure. The zero at the y( t ) = ∫ g( t )dt ,


origin has multiplicity 4. The impulse response of the −∞

system is h[n]. If h[0] = 1, we can conclude [2015] we −2w 2


i.e., f  ∫ g (t )  =
Im(z)   jw
2
we −2w
x 0.5 x
y (t ) ←
F

jw
4 ∞
we −2w
2

Re(z)
∫ y (t )e
− jwt
dt =
–0.5 0.5 −∞
jw
x –0.5 x
for w = 0
1
∫ y (t )dt =

j
= −j

(A) h[n] is real for all n Hence, the correct option is (b).
(B) h[n] is purely imaginary for all n
(C) h[n] is real for only even n 13. Consider a discrete-time signal
(D) h[n] is purely imaginary for only odd n n for 0 ≤ n ≤ 10
x [n ] = 
Solution:  According to pole zero pattern 0 otherwise
Z4 If y[n] is the convolution of x[n] with itself, the value of
H(z) = y[n] is ___________ [2014]
⎡( Z + 0.5)2 + . (0.5)2 ⎤ ⎡( Z − 0.5)2 + (0.5)2 ⎤
⎣ ⎦⎣ ⎦
Solution: 10
4
Z4 Z
= 2 = 4 n for 0 ≤ n ≤ 10

( )( 2
Z + 0.5 − Z Z + 0.5 + Z ) Z + 0.25  x [n ] = 
0 otherwise
h(n) = x(n)
H(z) = 1 – 0.25 Z–4 ….
∞ n

h[n] = [1, 0, 0, 0, –0.25….]


y (n) = ∑ x ( k )h( n − k ) ⇒ y ( n ) = ∑ x ( k )h( n − k )
k = −∞ k =0

Now 14. Let y[n] denote the convolution of h[n] and g[n], where
h[0] = 1 h[n] = (1/2)nu[n] and g[n] is a causal sequence. If y[0]
= 1 and y[1] = ½, then g[1] equals [2012]
2.18 | Signals and Systems

(a) 0 (b) ½ 
If the input to the above system is the sequence
(c) 1 (d) 3/2 e jpn/4, then the output is [2004]
Solution: (a) (a) 4 2e jpn / 4 (b) 4 2e - jpn / 4
n
(c) 4ejpn/4 (d) −4ejpn/4
1
h ( x ) =   u ( n ) g ( n ) = x ( n )u ( n ) Solution: (d)
2
y[0] = 1 y[1] 1/2 g[1] = ?
( )
y ( n ) = H e jw .e jpn / 4 [As per the concept of eigen vec-
tors]
∞ n−k
1
y [n ] = g ( n ) ⊗ h( n ) = ∑ x (k )u(k )  2 
k =−∞
u(n − k )
H e jw =( ) ∑ h(n)e

− jwn

n = −∞
n n−k

y ( n ) = ∑ x ( k )(1/ 2) = −2 2e jw − 2 2e jw + 4 2e −2 jw + 4 2e 2 jw
k =0
= −2 2 × 2 cos w + 8 2 cos 2w
1
y ( 0) = x ( 0)   = x ( 0) = 1 = −4 2 cos w + 8 2 cos 2w
2
p
1
1
1− k
1 1 ( )
w = = H e jw = −4 2 .
1
= −4
y (1) = ∑ x ( k )   = x (0)   + x (1)   4 2
k =0  2 2 2
∴ y( n ) = −4 e jpn / 4
1 1
= + x (1) ∴ x( 1 ) = 0
2 2 Hence, the correct option is (d).
x=
( 1 ) g=
(1) 0 1 7. The impulse response functions of four linear sys-
tems S1, S2, S3, S4 are given, respectively, by h1(t) = 1
Hence, the correct option is (a).
h2(t) = u(t)
1 5. A discrete time linear shift-invariant system has an
impulse response h[n] with h[0] = 1, h[1] = −1, h[2] = 2, u (t )
and zero otherwise x[n] with x[0] = x[2] = 1 and zero oth- h3 (t ) = h4 (t ) = e −3t u (t )
t +1
erwise. The number of non-zero samples in the output
sequence y[n] and the value of y[2] are, respectively, where u(t) is the unit step function. Which of these sys-
 [2008] tems is time invariant, causal and stable? [2001]
(a) 5, 2 (b) 6, 2 (a) S1 (b) S2
(c) 6, 1 (d) 5, 3 (c) S3 (d) S4
Solution: (d) Solution: (d)
1, −1, 2 h1(t) → non-causal
h( n ) =   h2(t) → time-variant
↑  h3(t) → time-variant
1, 0,1 h4(t) → all conditions satisfied
x (n) =   Hence, the correct option is (d).
↑ 
18. Let u(t) be the step function. which of the waveforms in
y ( n ) = x ( n ) ⊗ h( n ) the figure corresponds to the convolution of u(t) - u(t
- 1) with u(t) - u(t - 2)? [2000]
1, −1, 3, −1, 2
y (n) =  
↑  (a)
∴ Number of non-zero samples = 5
y[2] = 3.
Hence, the correct option is (d). 0 1 2
t
16. The impulse response h[n] of a linear time-invariant
system is given as
(b) 1
−2 2 n = 1, −1

h[ n ] =  4 2 n = 2−2
0, otherwise
 0 1 2 3
t
Chapter 2  LTI Systems Continues and Discrete  |  2.19

(c) 1 1.5 x(t)

1/2
0 1.5 2 3

(d)
1 0 2 6 t (sec) [1990]

1
(a) 0 (b)
2
0 1 3 3
(c) (d) 1
2
Solution: (b)
Solution: (b)
u(t)−u(t −1) u(t)−u(t −2)
y (t ) = h (t ) ⊗ x (t )

1 1 dy (t ) dx (t )
= h (t ) ⊗
dt dt
t t
0 1 0 2 dx(t)/dt

When two rectangular waveforms of different widths
1
are convolved, then the result is a trapezoidal waveform
with the slope equal to the product of amplitudes of 6
both signals given and the width of the constant ampli- 0
tude starts from upper hand limit of width signal.
−1

1
⇒ d (t ) − d (t − 6)

t dy
0 1 2 3 ∴ (t ) = x (t ) − x (t − 6)
dt
Hence, the correct option is (b).
dy(t)/dt
1 9. The impulse response and the excitation function of
a linear time-invariant causal system are shown in
Figures (a) and (b), respectively. The output of the sys- 1/2
tem at t = 2 sec is equal to
6
h (t) t
0 2

1 1
2
dy (t ) 1 1
y (t ) = ∫ = dt = ∫ dt =
dt 0
4 2
0 6 t (sec)
Hence, the correct option is (b).
Chapter 3
Fourier Series
So that, signal is odd and half wave symmetric. In the
One-mark Questions resulting fourier series expansion, a0 and an will exist
only for odd harmonics (due to half wave symmetry)
1. Let x (t) be a periodic function with period T = 10. The
Hence, the correct option is (A).
Fourier series coefficients for this series are denoted by
ak, that is ⎛ π n⎞
3. Let x [ n] = 1 + cos ⎜ ⎟ be a periodic signal with


jk t ⎝ 8⎠
x(t) = ∑ ∞K =−∞ ak e T
period 16. Its DFS coefficients are defined by ak =
The same function x(t) can also be considered as a 1 15 ≈ ⎛ π ⎞
periodic function with period T 1 = 40. Let bk be the ∑ x [ n] exp ⎜⎝ − j 8 kn⎟⎠ for all k. the value of the
16 n = 0
Fourier series coefficients when period is taken as T1.
coefficient a31 is _________. [2015]
If ∑ ∞K =−∞ ak = 16, then ∑ ∞K =−∞ a \ bk is equal to
 [2018] ⎛ π n⎞
(A) 256 (B) 64 X[n] = 1 + cos ⎜ ⎟ 
Solution:  ∵ N = 16
⎝ 8⎠
(C) 16 (D) 4
2π 2π n
j −j
Solution:  We know that change in time period does e 16 +e 16
not change the fourier series coefficients =1+
2
∞ ∞
∑ ∑
2π n 2π n
∴ bk = ak = 16  1 j 16 1 − j 16
k = −∞ k = −∞
=1+ e + e
2 2 
Hence, the correct option is (C) 1 1
2. A periodic signal x(t) has a trigonometric Fourier series a0 = 1, a1 = , a–1 =
2 2
expansion
So a0+16 = a16 = 1

x(t ) = ao + ∑ ( an cos nω0 t + bn sin nω0 t ) a–1+16 = a15 =
1
n −1 2
If x (t) = -x (-t) = -x (t - p/w0), we can conclude that 1
a31 = a16+15 = a15 =
 [2017] 2
(A) an are zero for all n and bn are zero for n even Hence, the correct Answer is (0.48 to 0.52).
(B) an are zero for all n and bn are zero for n odd
(C) an are zero for n even and bn are zero for n odd 4. The trigonometric Fourier series of an even ­function
(D)  an are zero for all n odd and bn are zero for n even does not have the [2011]
(a) dc term
Solution:  Given x(t) = - x (-t) means the signal is odd (b) cosine term
 π  (c) sine terms
x(t ) = − x  t −  , which says that the signal is half
 ω0  (d) odd harmonic terms
wave symmetric. Solution: (c)
For an even function bn = 0 ∴ no sine terms are present.
 T
Condition for half wave symmetry is x(t ) = − x  t ±  Hence, the correct option is (c).
 2
5. The Fourier series of a real periodic function has only
P. Cosine terms if it is even
Chapter 3  Fourier Series  |  2.21

Q. Sine terms if it is even (c) x(t) = cost + 0.5


R. Cosine terms if it is odd (d) x(t) = 2 cos 1.5p t + sin 3.5p t
S. Sine terms if it is odd Solution: (b)
 Which of the above statements are correct? For Fourier series expression, x(t), should be periodic
 [2009] for x(t) = 2 cost + 3 cos 3t
(a) P and S
T
(b) P and R T1 = 2p T 2 = 2p ∴ 1 = 3 = rational
(c) Q and S 3 T 2

(d) Q and R ∴ Periodic for x(t) = 2 cosp t + 7 cost


Solution: (a)
2p T 1
For an even fxn, bn = 0, so sine terms are omitted and =
T1 = 2; T2 = 2p 1 = ⇒ Not periodic
only cosine terms are present. p T2 p
For an odd  fxn, a0 = 0, an = 0; so only sine terms are As not x(t) is not periodic. So, Fourier series ­expression
present. is not possible.
Hence, the correct option is (a). Hence, the correct option is (b).
6. Choose the function f(t), −∞ < t < ∞, for which a 9. The trigonometric Fourier series of a periodic time
Fourier series cannot be defined, [2005] function can have only [1998]
(a) 3 sin (25t) (a) cosine terms
(b) 4 cos (20t + 3) + 2 sin (710t) (b) sine terms
(c) exp (-|t|) sin 25t (c) cosine and sine terms
(d) 1 (d) dc and cosine terms
Solution: (c) Solution: (c)
Fourier series expression of a signal x(t) must be peri- Trigonometric Fourier series consists of cosine and sine
odic terms.
• f(t) = 3 sin (25t) Hence, the correct option is (c).
It is periodic signal. 10. A periodic signal x(t) of period T0 is given by
• f(t) = 4 cos (20t + 3) + 2sin (710t) 1 t < T1

2p 2p . T1 71 x (t ) =  T 0 . The component of x(t) is
T1 = ; T2 = = ⇒ rational 0 T1 < t <
20 710 T2 2  2
∴ Periodic  [1998]
• f(t) = e-|t| sin (25t) T1 T1
This signal is not periodic due to modulus fxn. So, Fou- (a) (b)
T0 ( 2 T0 )
rier series representation is not possible.
Hence, the correct option is (c). 2T T0
(c) 1 (d)
7. The Fourier series expression of a real periodic signal T0 T1
with fundamental frequency f0 is given by Solution: (c)
∞ For dc component
 gp (t ) = ∑C e n
j 2pnf 0t
. It is given that C3 = 3 + j5. Then
T
n = −∞ 1
T ∫0
C-3 is [2003] a0 = x (t ) dt
(a) 5 + j3
1  1  2T
T
(b) -3 – j5 =  ∫ 1.dt  = 1
(c) -5 + j3 10  −T1  T 0
(d) 3 – j5
2T1
Solution: (d) a0 =
For a Fourier series T0
Cn = C-n* Hence, the correct option is (c).
C-3 = 3 – 5j 1 1. The trigonometric Fourier series of an even function of
Hence, the correct option is (d). time does not have [1996]
8. Which of the following cannot be the Fourier series (a) the dc term
expression of a periodic signal? [2002] (b) cosine terms
(a) x(t) = 2 cost + 3 cost (c) sine terms
(b) x(t) = 2 cosp t + 7 cost (d) odd harmonic terms
2.22 | Signals and Systems

Solution: (c) For the three statements above which one of the follow-
For even periodic fxnbn = 0 ing is correct? [2017]
∴ Sine terms are not present. (A) Only II and III are true
Hence, the correct option is (c). (B) Only I and III are true
12. The RMS value of a rectangular wave of period T, hav- (C) Only III is true
ing a value of +V for a duration T1(< 7) and –V for the (D) Only I is true
duration T – T1 = T2 equals: [1995] Solution:  Fundamental period T = 1 s. w0 = 2p time
T1 - T 2 scaling property of complex fourier series says that
(a)
V (b) V x(t ) ↔ ak
T
V T1 x( at ) ↔ ak
(c) (d) V
2 T T
and ω0 → α (ω0 )
2
But T →
Solution: (a) α

V x(t ) ←
→ ak
x(3t ) ←
→ ak
T1
ω0 ←
→ 3( 2π ) = 6π
−V
I and III are true.

Hence, the correct option is (B).


1 2 
T T T
1
V rms = ∫ =  ∫ + ∫ V 2dt  2. The magnitude and phase of the complex Fourier series
2
V dt V dt
T 0 T  0 T1  coefficient ak of a periodic signal x(t) are shown in
the figure. Choose the correct statement from the four
1
= V 2T1 + V 2 [T − T1 ] = V
 choices given. Notation C is the set of complex num-
T bers, R is the set of purely real numbers, and P is the set
of purely imaginary numbers. [2015]
Hence, the correct option is (a).
ak
13. The Fourier series of an odd periodic function ­contains 3 3
only[1994] 2 2
(a) odd harmonics (b) even harmonics 1
(c) cosine terms (d) sine terms k
–5 –4 –3 –2 –1 0 1 2 3 4
Solution: (b)
For odd harmonic fxn ∠ak
–5 –4 –3 –2 –1 1 2 3 4
a0 = an = 0 k
0
∴ Only sine terms are present.
–π
Hence, the correct option is (b).
(A) x(t) ∈ R
Two-marks Questions (B) x(t) ∈ P
(C) x(t) ∈ (C – R)
1. Let x(t) be a continuous time periodic signal with fun- (D) the information given is not sufficient to draw any
damental period T = 1 seconds. Let {aK} be the com- conclusion about x(t)
plex fourier series coefficients of x(t), where k is integer
Solution:  Magnitude spectrum of ak is even so corre-
valued. Consider the following statements about x (3t):
sponding signal x(t) in time domain is real and even.
I. The complex fourier series coefficients of x(3t) are x(t) ∈ R
{aK} where k is integer valued
II. The complex fourier series coefficients of x(3t) are Hence, the correct option is (A).
{3aK} where k is Integer valued 3. One period (0, 7) each of two periodic waveforms w1
III. 
The fundamental angular frequency of x(3t) is and w2 are shown in the figure. The magnitudes of the
6p rad/s nth Fourier series coefficients of w1 and w2, for n ≥ 1,
n odd, are respectively proportional to:
Chapter 3  Fourier Series  |  2.23

1 1 5. Fourier series of the periodic function (period 2p)


W1 W2
defined by
T/2 T 0, −p < x < 0
0
T
0
f (x ) =  is
T/2
x 0 < x < n
p ∞  1 1 
−1 −1 + ∑  2 [cos( np ) − 1]cos( nx ) − cos( np ) sin( nx ) .
 [2000] 4 1 pn n 
By putting x = p in the above equation, one can deduce
(a) |n|-3 and |n|-2 that the sum of the series
(b) |n|-2 and |n|-3
(c) |n|-1 and |n|-2 1 1 1
1+ + + + is [1993]
(d) |n|-4 and |n|-2 32 52 72
Solution: (c) p2
Solution:
In Fourier series representation 8
At x = p
1
C n ∝ 1 for fxn consisting of unit steps p 1
n f (p ) = + ∑ 2 [cos np − 1] cos np
4 1 pn
1
Cn∝ for fxn consisting of ramps. 1
n2 − cos( np ) sin( np )
Hence, the correct option is (c). n
p 1
= + ∑ 2 ( −1 )2 n − ( −1 )n  − 0
4. The Fourier series representation of an impulse train 4 1 pn

denoted by s (t ) = ∑ d (t − nT 0 ) is given by [1999] p 1
= + ∑ . 2 1 − ( −1 )n 
1 ∞
− j 2p nt
n = −∞ 4 1 pn
(a) ∑ exp p 2 2 2
T 0 n = −∞ T0 = + + + +
4 p 9p 25p
1 ∞ − jp nt
(b) ∑ exp p 2 1 1 
T 0 n = −∞ T0 = + 1 + 2 + 2 + 
4 p 3 5 
1 ∞ jp nt
(c) ∑ exp As the f(x) has a period of 2p, so, the f(x) converges to
T 0 n = −∞ T0 2p
middle value at where p is the point of discontinu-
1 ∞ − j 2p nt 2
(d) ∑ exp ity at which value changes from 0 to 1
T 0 n = −∞ T0
p p 2 1 1 
Solution: (d) ∴ = + 1 + 2 + 2 + 
2 4 p 3 5 

s( t ) = ∑ d (t − nT ) p p 
2−4
0
n =−∞ 1 1
  = 1 + 2 + 2 +
for Fourier series  2  3 5
∞  p 
f (t ) = ∑C e k
jkw 0t

n = −∞ p2 1 1
= 1 + 2 + 2 +
1 8 3 5
for an impulse function C k =
T0
1 1 p2
∞ ∴ 1+ + +  =
1 32 5 2 8
∴ f (t ) =
T0
∑e
n = −∞
jkw 0t

6. Which of the following signals is/are periodic? [1992]


2p (a) s(t) = cos 2t + cos 3t + cos 5t
w0 = and k → n
T0 (b) s(f ) = exp ( j8pt)
1 ∞ n 2p (c) s(f ) = exp (-7t) sin 10 pt
∑e
h t
f (t ) = 10
(d) s(t) = cos 2t cos 4t
T0 n = −∞

Hence, the correct option is (d).


2.24 | Signals and Systems

Solution: (a), (b) and (d) 2p 2p


For a periodic signal, ratio of time period must be a T1 = T2 =
6 2
rational number
T1 1
s(t) = cos 2t + cos 3t + cos 5t = ⇒ Rational No, so periodic
T2 3
2p 2p 2p
=T1 = p; T 2 = T3 = Hence, the correct options are (a), (b) and (d).
2 3 5
7. A half-wave rectified sinusoidal waveform has a peak
T1 p 3
= × 3 = → rational number voltage of 10 V. Its average value and the peak value of
T 2 2p 2 the fundamental component are, respectively, given by
T 2 2p 5 5  [1987]
= × = → rational number
T3 3 2p 3 20 10 10 10
(a) V, V (b) V, V
T1 5p 5 p 2 p 2
= = → rational number
T3 2p 3 10 20
(c) V, 5 V (d) V, 5 V
p p
• s (t ) = e j 8pt so, it is periodic
Solution: (c)
e jw 0t is always a period signal For a half wave rectified sinusoid
−7t
• s (t ) = e sin 10pt V m 10
Vav = = V
2p 2p p p
 Here T1 = T2 =
7 10p V 10
and fundamental component, m = =5V
T1 10p 2 2
= ≠ Rational Number
T2 7 Hence, the correct option is (c).
 So, not periodic.
1
• s (t ) = cos 2t cos 4t = [cos 6t + cos 2t ]
2
Chapter 4
Fourier Transforms
Discrete Fourier transform is given as
One-mark Questions n
 2  − J wn
1. A real valued signal x(t) limited to the frequency band
x (e J ′w ) = ∑
n =−
[n ]e − Jwn = ∑ 3 e
n =−3  
W Put n + 3 = m
f ″ passed through a linear time-invariant system
2 m −3
whose frequency response is
( ) ∑  23 
So x e J ′w = e − J w ( m − 3)
 − J 4pf W m =0
e f ≤ 3 m
2 3 2  Jw 3
H (f ) = 
W
= 
2
∑  3 e − Jw
 e

 0 f > m =0
 2 Applying ∞ G.P summation
The output of system is [2014]
(a) x(t + 4) (b) x(t - 4) 3.375e J ′ 2pf 3
=
(c) x(t + 2) (d) x(t - 2) 2
1 − e − J 2pf
Solution: (d) 3
So, A = 3.3375.
 − J 4pf W
e f ≤ 3. An FIR system is described by the system function
Given H (f ) ≠  2
7 3
 0 W H ( z ) = 1 + z −1 + z −2. The system is [2014]
f > 2 2
 2
H (f ) = e − J 4pf ⇒ H (w ) = e − J 2w (a) maximum phase
(b) minimum phase
⇒ h (t ) = d (t − 2) (c) mixed phase
As we know that (d) zero phase
y (t ) = x (t ) * h (t ) = x (t ) * d (t − 2) = x (t − 2) Solution: (c)
Poles of system H(z) are lying inside and outside of the
so y (t) = x(t − 2) unit circle. So, system will be the mixed phase system.
Hence, the correct option is (d). Hence, the correct option is (c).
2. A Fourier transform pair is given by 2
4. Let g(t) = e -pt and h(t) is filter matched to g(t). If g(t)
n is applied as input to h(t), then the Fourier transform of
2 F ⋅T A ⋅ e − J 6pf
3 ( n + 3) ← → the output is [2013]
  2
1 −   e − J ′ 2pf (a) e -pf 2

3 pf 2
-
e 2
(b)
where u[n] denotes the unit step sequence, and the val-
ue of A is _____ [2014] e -p |f |
(c)
2

Solution: 3.3375 (d) e -2pf


n
2 Solution: (d)
Let x [n ] =   u ( n + 3) g(t) is a Gaussian pulse whose Fourier transform is also
3
Gaussian pulse.
2.26 | Signals and Systems

2 2 ⇒ So, H( f ) = 4e−j4p f
g (t ) = e − pt ←
F ⋅T
→ e − pf = G (w )
Hence, the correct option is (c).
2 2 2
o/p Y (w ) = G (w ) ⋅ G (w ) = e − pf ⋅ e − pf = e −2pf t
Hence, the correct option is (d). 8. The Fourier transform of F{e−tu(t)} is equal to
 1  1 + j 2pf
5. A function is given by f(t) = sin2t + cos 2t. Which of the . Therefore, F   is [2002]
following is true? [2009]  t + j 2pt 
1 (a) efu( f ) (b) e−fu( f )
(a) f  has frequency component of 0 and H3 (c) e u(−f ) (d)
f
e−fu(−f )
2p
1 Solution: (d)
(b) f  has frequency component of 0 and H 3 t
p Given f (t ) = e −t u (t ) ←
F ⋅T

1 + 2pf ⋅ j
1 1
(c) f  has frequency component of and H 3 From duality property,
2p p
t F ⋅T
1 1 ← → f (t ) = e − f u ( −f )
(d) f  has frequency component of 0, and H 3 1 + 2pj
2p p
Hence, the correct option is (d).
Solution: (b)
9. A linear phase channel with phase delay Tp and group
1 delay Tg must have [2002]
Given f (t ) = (1 − cos 2t ) + cos 2t
2 (a) Tp = Tg = constant
Frequency components are f1 = 0 (b) Tp ∝ f and Tg∝ f
w2 2 1 (c) Tp = constant and Tg∝ f
=
f2 = = H3
2p 2p p (d) Tp ∝ f and Tp = constant, ( f denotes frequency)
Hence, the correct option is (b). Solution: (a)
6. Let x(t) ↔ x(  jw) be Fourier transform pair. The Fourier For a distortionless system, phase is given by q(w) =
transform of signal x(5t−3) in terms of x(  jw) is given −wt0
as [2006] −q(w )
then phase delay T p = = t0
w
1 j 3w Jw  1 − j 35w  jw  dq(w )
(a) e 5 X   (b) e X  and group delay =Tg = t0
5  5  5  5  dw
So, Tp = Tg = constant.
1 1 j 3w  J w 
 J w  (d) Hence, the correct option is (a).
(c) e − j 3w X  5  e X 
5   5  5  2
10. The Fourier transform of signal x(t) = e - Bt is of the
Solution: (a) following form where A and B are constants: [2000]
Using scaling and shifting property (a) A e - B f (b) A ⋅ e − Bf 2
x(t) ↔x(  jw) (c) A + B | f  2| (d) A ⋅ e − Bf
  3   1  J w  j 35w Solution: (b)
X 5  t −   = ×  e
2
Given f (t ) = e − Bt (1)
  5  5  5  F ⋅T
Let f (t ) ← → F (w )
Hence, the correct option is (a). Differentiating equation (1) both side.
7. Let x(t) be the input to a linear time-invariant system. df (t ) 2

The required o/p is 4 x(t−2). The transfer function of = −2at e − at ← F ⋅T


→(J É )F (w )
dt
the system should be. [2003] 2  −J w 
(a) 4ej4p f (b) 2e−j8p f ⇒ t ⋅ e − at ← FT
→  F (w ) (2)
(c) 4e−j4p f (d) 2ej8p f  2a 
Now multiplication by t in time-domain, ↔ different into
Solution: (c)
domain.
y (t ) = 4 x (t − 2) So,
2 dF (w )
Taking Laplace transform: y (s ) = 4e −2d x (s ) t ⋅ e − at ← →J ⋅ (3)
dw
y (s ) From equations (2) and (3)
⇒ = 4 ⋅ e −2d
x (s )
dF (w )  − J w 
but s = j2p f ⇒J⋅ =  F (w )
dw  2a 
dF (w )  −1 
⇒ = w dw ⇒
F (w )  2a 
dF (w )  − J w  Chapter 4  Fourier Transforms  |  2.27
⇒J⋅ =  F (w )
dw  2a 
dF (w )  −1  x (t ) ←
Fourier
→ x (f )
⇒ = w dw ⇒
F (w )  2a 
Transform

Hence, the correct option is (b).


Integrating on both sides 14. The amplitude spectrum of a Gaussian pulse is [1998]
−1 w w 2 2 (a) uniform
log F (w ) = ⋅ = (b) a Sine function
2a 2 4a
(c) Gaussian
 Or F( w ) = e (d) an impulse function
put a = 3, and w = 2pf Solution: (c) 2

⇒ F (f ) = e
− pf 2
3
= Gaussion pulse Normalized Gaussian pulse is defined as x (t ) = e −pt
Fourier transfer will be
Hence, the correct option is (b). 2 2
x (t ) = e − pt ←
F ⋅T
→ e − pf = X (t ) ⇒ Gaussian pulse
11. A signal x(t) has a Fourier transform x(w). If x(t) is a Hence, the correct option is (c).
real and odd function of t, then x(w) is [1999] 15. The ACF of a rectangular pulse of duration T is:[1998]
(a) a real and even function of w (a) a rectangular pulse of duration T
(b) an imaginary and odd function of w (b) a rectangular pulse of duration 2T
(c) an imaginary and even function of w (c) a triangular pulse of duration T
(d) a real and odd function of w (d) a triangular pulse of duration 2T
Solution: (b) Solution: (d)
Example of real and odd function of t is ACF for signal x(t) is given by.
x(t) = A sin w0t
R(t) = x(d ) * x (−d )
then Fourier transform of x(t) R (t)
x(d ) x(−d )
x (w ) = A jp [d (w + w 0 ) − d (w − w 0 )] ⇒ Imaginary and 1
odd function w 1 1
= * =
Hence, the correct option is (b).
0 T −T −T 0 T
12. The Fourier transform of voltage signal x(t) is x( f ).
The unit of |x( f )| is [1998] 2T
(a) Volt (b) Volt-sec
(c) Volt/sec (d) Volt2- = Triangular pulse of
duration 2T
Solution: (b)
Fourier transform of signal x(t) is given as Hence, the correct option is (d).
1 6. The function f(t) has Fourier transform g(w). The
x (f ) ∫ x (t ) ⋅ e − j 2pft dt Fourier transform g(t) [1997]

∫ g (t )e
− J wt
g (t ) = dt , is
if x(t)→ volt, −
V olt V olt
then x( f )→ Volt-sec → → 1 1
( 1S ec ) Hertz f ( -w )
(a) f (É ) (b)
Hence, the correct option is (b). 2p 2p
2pf ( -w )
(c) (d) name of these
13. The Fourier transform of function x(t) is X(t). The Solution: (c)
dx (t )
Fourier transform of will be [1998] Using duality property of Fourier transform
dt
F ⋅T
f (t ) ← → g (w ) .
(a) dx (f ) (b)
j2p f x( f )
dt Then Fourier transform of g(t)
x (f ) F ⋅T
g (t ) ← → 2p f ( −w )
(c)
x( f ) (d)
jf Hence, the correct option is (c).
Solution: (b)
1 7. The Fourier transform of a real valued time signal has
dx (t )  [1996]

→( j 2pf ) x (f ). (a) odd symmetry
dt
(b) even symmetry
2.28 | Signals and Systems

(c) conjugate symmetry e (s ) wn 2


(d) No symmetry. = 2 , is given by
R (s ) d + 2xwn + wn 2
Solution: (d)
Fourier transform of signal x(t) is given by wn 1 - 2x 2
(a)

wn (1 − x 2 ) + x 4 − x 2 + 1
(b)
X (iw ) =

∫ x (t ) ⋅ e − Jwt dt .
wn (1 − 2x 2 ) + 4x 4 − 4x 2 + 2
(c)
Taking conjugate on both sides,
 
*
wn (1 − 2x 2 ) − 4x 4 − 4x 2 + 2
(d)
⇒ X * ( jw ) =  ∫ x (t ) ⋅ e − Jwt dt  = ∫ x * (t ) ⋅ e + Jwt = X ( −J w )
−  −
Solution: (d)
Given transfer function

⇒ So x * (jw) = x(−jw) ⇒ Conjugate symmetric. c (s ) wn 2


H (s ) = = 2 (1)
∴ [x*(t) = x(t) for real valued signal. R (s ) s + 2xwns + wn 2
Hence, the correct option is (d). Put s = J′w, and divide numerator and denominator by
18. A rectangular pulse of duration T is applied to a filter wn2
method to this input. The output of filter is a[1996] t
(a) rectangular pulse of duration T. H (J w ) = (2)
 w  2
 2xwn 
(b) rectangular pulse of duration 2T. 1 − 2  + J ′  
(c) triangular pulse  wn   wn 
(d) sine function É
Let x = from xgn (2)
Én
Solution: (c)
A rectangular pulse can be expressed as t
H ( jw ) = (3)
x(t) (1 − x 2 ) + j ( 2xx )
at 3 dB frequency |H (  jwc)| = 1
2 , from xgn (3)
1
1 1
⇒ = (4)
(1 − x ) + ( 2xx )
2 2
2
t
T Solving equation (4)
For matched filter o/p for i/p x(t) is
h(t) = x(T − t) x = (1 − 2x 2 ) ± ( 4x 2 − 4x 2 + 2) (5)
h(t) can be represented as
wc
h(t) as x =
wn
So from equation (5)
1
wc = wn (1 − 2x 2 ) ± ( 4x 2 − 4x 2 + 2)
t As wC can’t be negative,
T
then time duration output will be y(t) = x(t) * h(t) So w c = w n (1 − 2x 2 ) + 4x 4 − 4x 2 + 2
y (t )
Hence, the correct option is (d).
x(t) h(t )
1
1 Two-marks Questions
y(t) = * =
t t 1. Consider the signal x[n] = 6δ[n + 2] + 3δ[n + 1] + 8δ[n]
0 T 0 T 0 T 2T + 7δ[n – 1] + 4δ[n – 2].
= Triangular pulse of If x(ejω)is the discrete-time Fourier transform of x[n],
duration 2T π

Hence, the correct option is (c).


then
1
π −∫π
( )
x e jω sin2(2ω) dω is equal to ____.[2016]

19. The 3-dB bandwidth of a typical second-order s­ ystem Solution:  x[n] = 6 δ [n + 2] + 3 δ [n + 1] + 8 δ [n] + 7
with the Transfer function[1994] δ [n – 1] + 4 δ [n – 2]
x[n] = {6, 3, 8, 7, 4} its DTFT π is X(ejω)
Chapter 4  Fourier Transforms  |  2.29

Now (a) 1/4


π (b) 1/2
1
X (e jω ) sin 2 ( 2ω ) dω
π −∫π
(c) 1
(d) 2
π
1 ⎛ 1 − cos 4ω ⎞ Solution: (a)
π −∫π
= X (e jω ) ⎜ ⎟⎠ dω
⎝ 2  2 sin 2w 
H (J w ) = 2 cos w   = H 1 (J w ) ⋅ H 2 (J ′w )
π π Given  2w 
∫ X (e ) dω − 2π ∫ X (e ) cos 4ω dω
1 jω jω 1
= where
2π −π −π
⇒H1 (  jw) = 2cosw =
π
⎛ e j 4ω + e − j 4ω ⎞
∫ ( )
1
= x [0 ] − X e jω × ⎜ ⎟ dω 2 sin 2w
2π ⎝ 2 ⎠ =
H 2 ( jw ) = 2da( 2w )
−π 2w
1 ⎡
π π

= 8− ⎢ ∫ X e dω + ∫ X e e
4π ⎢⎣ −π

( )jω − j 4ω
dω ⎥
⎥⎦
( )
−π
1/2
1 1
= 8 − ⎡⎣ x [ 4] + x ( −4 )⎤⎦ = 8 − [0 + 0] = 8
π 2
Hence, the correct Answer is (8). t
−2 +2

∫ sin c
2
2. The value of the integral (5t ) is [2014] h2(t )

Solution: 
0.2

=
Let =
f (t ) sin c (5t ) Sa(5pt ) 2sa (2ω)
(∵ sin c (t ) = Sa(pt )) t
−2 0 +2
Fourier transform of Sa (t) is given as
 T  F ⋅T ⇒ H 1 (J w ) = 2 cos w = e Jw 1 + e Jw ( −1)
TSa  t ⋅  ←→ 2pGT (w )
 2
So, H (J w ) = e Jw H 2 (J w ) + e − Jw H 2 (J w )
T
= 5p ⇒ T = 10p Or h (t ) = h2 (t + 1) + h2 (t − 1)
2

⇒ 10pSa(5pt ) ←→ G10p (w )
5
2 1
⇒ ESDf = F (w )
h (t) = sa h (o) = 1
1 1/2
Ef = ∫

f 2 (t )dt =
2À ∫

ESDf dw

F(ω ) = G10π(ω ) 1
−3 −1 1 3
= F(ω )
1/5
Hence, the correct option is (a).
4. Consider a system whose i/p and o/p y are related by
ω the equation
−5π 5π

1
5p
1
2
1 1 1
y (t ) =
 ∫ x (t − t )h(2t )dt , and h(t) is shown in graph
⇒ ∫ sin c (5t )dt =
2À −∫5p  5 
dw = × × 10p = = 0.2
2


2p 25 5
h(t )

3. The Fourier transform of a signal h(t) is [2012] t


0

H (J w ) = ( 2 cos w )(sin 2w ) / w the value of h(o) is
2.30 | Signals and Systems

Which of the following your properties are possessed Solution: (a)


by the system? BIBO: Bounded i/p fixes o/p Given h (t ) = e −2t u (t )
Causal: the system is causal
LP: The system is low pass ∞
−1  −( 2tJ w )t 
LTI: the system is linear and time-invariant [2009] H (w ) = ∫ h(t )e
− J wt
dt = ∫ e − ( 2tJ w )t
dt =
( 2tJ w ) 
e
(a) Causal, LP − 0   0
(b) BIBO, LTI 1
(c) BIBO, Causal, LTI =
1 + 2Jw
(d) LP, LTI
Hence, the correct option is (a).
Solution: (b)
7. The signal x(t) is described by
y (t ) = ∫ x (t − t ) h(2t )dt
− 1 for − 1 ≤ t ≤ 1
given  x (t )  Two of the angular frequencies
0 otherwise
h(t) where Fourier transform became zero are [2008]

(a) p, 2p
(b) 0.5p, 1.5p
t (c) 0, p,
(d) 2p, 2.5p
Solution:  (a) and (c)
The system is LTI, BIBO, but not a low pass filter.
Fourier transform
Hence, the correct option is (b). 1
1 Jw
5. The output of the system to sinusoid input x(t) = 2cos x (w ) ∫ x (t )e − Jwt dt = ∫ e − Jwt dt =

(
e − e − Jw )
(2t) for all time t is[2008] − −1

(a) 0 ⇒ x (w) = 0
(b) 2−0.2cos (2t − 0.125p) So e Jw − e−Jw = 0
(c) 2−0.25cos (2t − 0.125p) ⇒ e2Jw = f ⇒ w = p, 2p or 0.p
(d) 2−0.5cos (2t − 0.25p) Hence, the correct options are (a) and (c).
Solution: (d) 8. The 3-dB bandwidth of the LOW-pass signal, e−t u (t),
Given input x(t) = 2cos2t where u(t) is the unit step function, is given by [2007]
1
(a) H z
then x (w ) = 2p [d (w − 2) + d (w + 2)] 2p
1 1
⇒ H (w ) = (b) 2 - 1 Hz
1 + Jw 2p
o/p = (c) ∞
(d) 1 Hz
1
y (w ) = H (w ) ⋅ x (w ) =
2 + jw
[( 2p )[d (w − 2) + d (w + 2)]] Solution: (a)
1
p p Laplace transform of e −t u (t ) ←
L ⋅T

= [d (w − 2) + d (w + 2)] − j [d (w − 2) − d (w + 2)] (s + 1)
2 2
t
cos 2t sin 2t 2 1 1  ∴ Magnitude at 3dB frequency =
= + = cos 2t + sin 2t 
2  2
2
2 2 2 
1 1 1 1
= 2−0.5 ⋅ cos( 2t − 0.25p ) ⇒ = = ⇒ w = 1 real or f = Hz
2 s + 1 1+ w 2 2p
Hence, the correct option is (d). Hence, the correct option is (a).
6. The frequency response H(w) of this system in terms of 9. The output y(t) of a linear time-invariant system is
angular frequency w, is given by H(w) = [2008] related to its input x(t) by the equation y(t) = 0.5 x (t − td
+ T ) + x(t − td) + 0.5x (t − td + T ). The filter transfer
1 sin(w ) function H(w) of such system is given by [2005]
(a) (b)
1 + 2Jw w (a) (1 + coswT ) e−Jwtd
(b) (1 + 0.5 coswT ) e−Jwtd
1 jw
(c) (d) (c) (1 − coswT ) e−Jwtd
1+ jw 2 + jw (d) (1 − 0.5 coswT ) e−Jwtd
Chapter 4  Fourier Transforms  |  2.31

Solution: (a) 12. The Fourier transform of a conjugate symmetric func-


Given y(t) = 0.5 x (t − td + T ) + 0.5x (t − td + T ) tion is always: [2004]
+ x(t − td) (a) Imaginary
taking Fourier transform (b) Conjugate anti-symmetric
(c) real
y (w ) = 0.5 e jw ( −td +T ) + 0.5 e jw ( −td +T ) + e − jwtd  x (w ) (d) Conjugate symmetric
y (w ) Solution: (c)
⇒ H (w ) = = e − jwtd 0.5e jwt + 0.5e − jwt + 1 Fourier transform of a symmetric function is a purely
x (w )
real fn.
H (w ) = e − jwtd [cos wT + 1] Hence, the correct option is (c).
Hence, the correct option is (a). 13. A rectangular pulse train s(t) as shown in the figure is
1 0. Match the following and choose the correct convolved with the signal cos2 (4p × 103t).
­combination. [2005] Convolved signal will be a
S(t )
Group-1
E. Continuous and aperiodic signal
F. Continuous and periodic signal
G. Discrete and aperiodic signal
H. Discrete and periodic signal • • • • • • • •

Group-2
1. Fourier representation is continuous and periodic 0
2. Fourier representation is discrete and periodic 0.1 m sec
3. Fourier representation is continuous and periodic  [2004]
4. Fourier representation is discrete and periodic (a) D.C. (b) 12 kH3 sinusoid
(c) 9 kH3 sinusoid (d) 14 kH3 sinusoid
(a) E − 3, F − 2, G − 4, H − 1
(b) E − 1, F − 3, G − 2, H − 4 Solution: (a)
(c) E − 1, F − 2, G − 3, H − 4 Time period for pulse train To = 0.1 msec = 10-4 sec
(d) E − 2, F − 1, G − 4, H − 3 1
f0 = = 10 kH 3
Solution: (c) T0
Continuous and aperiodic signal has continuous and Fundamental fg in cos2 (4p × 103t) is = 4kH3
periodic representation.
Continuous and periodic signal has discrete and peri-
odic representation.
Hence, the correct option is (c).
11. For a signal x(t), the Fourier transform is x( f ), then
the inverse Fourier Transform of x(3f + 2) is given by −4 kHz 0 4 kHz 0 10 kHz 30 kHz
 [2005] So only at ‘0’ we get output after convolution. (only odd
1  t  − j 4p 13 harmonics are present)
(a) ×   e j 3pt (b)
1 t
× e
2 2 3  3  ↑ Constant in time domain.
Hence, the correct option is (a).
3 × (3t)e − j4pt (d)
(c) x(3t + 2) 14. Let x(t) and y(t) (with Fourier transforms x( f ) and y( f )
respectively) be related as shown in the figure [2004]
Solution: (b)
x(t) y(t)
Scaling and translation property for Fourier transform
pair are given as. 1

F ⋅T 1 f 
⇒ x (at ) ← → ×
| a |  a  t
−2 −1 0
t
F ⋅T − J 2pft 0 −2 0 2
⇒ x (t − t 0 ) ←
→ x (f )e
  2   1  t  − J 4p 3t −1
× 3  f +   = ×   ⋅ e
  3  3  3 
Hence, the correct option is (b).
2.32 | Signals and Systems

−1  f  −1  f  j 2pf Solution: (c)
(a) x   e − J 2pf (b) x e
2  2 
1
2 2 Given, H (w ) =
1 + wRC
f  f 
x   e j 2pf (d)
(c) − x   ⋅ e − j 2pf and qw = − tan −1 ( RCw )
2 2
from equation (1) 4.19
Solution: (b)
put a value of R, C and w to calculate tg
From the plot, y(t) can be represented as
y (t ) = − x ( 2(t ) + 1) dq(w ) RC
tg = = ,
x (t x−(t 0−) t← x→(fx)(ef−)Je2p−ftJ02pft 0 : t 0 :=t 0−=1 −1 dw 1 + R 2C 2w 2
Using property: 0 )
→

put R = 103, C = 1 × 10−6f = 100H3
1 f  So tg = 0.717 ms.
x (at ) ←
→ ×  a = −2
a a Hence, the correct option is (c).
−1  f  J 2pf . 17. If Fourier transform of deterministic signal g(t) is G( f ),
So, y (f ) = × e
2  2 
then
(1) the Fourier transform of g(t − 2) is
Hence, the correct option is (b). (2) the Fourier transform of g(t/2) is
15. Let H( f ) denote the frequency response of the (a) G ( f )·e−J4pf (b) G (2f )
RC-LPF. Let f1 be the highest frequency such that (c) 2G (2f ) (d) G ( f−2)
| H (f 1 ) | Match each of the items (1) and (2) on the left with the
o ≤ | f | ≤ f1: ≥ 0.95 then f1 (in H3) is [2003]
H (o ) most appropriate item a, b, c, or d on the right. [1997]
(a) 327.8 (b) 163.9 Solution: (1 − a, 2 − c)
(c) 52.2 (d) 104.4 F ⋅T
Given g (t ) ← → G (f )
Solution: (c) F ⋅T
then g (t − 2) ← → e − j 2p 2f G (f ) = G (f ) ⋅ e − j 4pf
Rc-LPF can be given as
R  t  F ⋅T 1 f 
and g   ← → G   = 2G ( 2f )
2 ( 12)  12 
Vi C Vo
18. Match each of items A, B and C with an appropriate
item from 1, 2, 3, 4 and 5. [1995]
List-I
(A) Fourier transform of a Gaussian function
Transfer function will be given by.
(B) Convolution of rectangular pulse with itself
1 (C) Current through an inductor for a step input volt-
vo j 2pf ⋅ c t age.
= = (1)
vi t 1 + j 2pf ⋅ RC List-II
R+
J ⋅ 2pf ⋅ c (1) Gaussian function
(2) Rectangular pulse
1
H (f ) = , H (o ) = 1 (3) Triangular pulse
1 + j 2pfRC (4) Ramp function
⇒ cutoff filter is cancelled at point where magnitude is (5) Zero
5% of maximum value. Solution: (A − 1, B − 3, C − 4)
| H (f 1 ) | 1 (A) Gaussian pulse is given as f(t) = e−p t 2
So, = ≥ 0.95.(2)
| H (o ) | 1 + 4p f 2 R 2C 2
2
So, e − pt ←
F ⋅T 2
→ e − pf
2

Put a value of R and C, and find f1 ⇒f1max = 52.2Hz. (B) Conversion of a rectangular pulse with a rectangu-
Hence, the correct option is (c). lar pulse is a triangular pulse.
16. Let tg ( f ) be the group delay function of the given (C) Current through an inductor is given by
RC-LPF and f2 = 100H3. Then tg( f2)(ms) [2003] 1
L∫
(a) 0.717 (b) 7.17 iL = V dt
(c) 71.7 (d) 4.505
Chapter 4  Fourier Transforms  |  2.33

For step input V = u(t) y(t )

1 1
So, i L =
L ∫ u (t )dt = r (t )
L 2m2T
3
19. Sketch the waveform (with properly marked axis) at the
output of a match filter matched for a signal S(t), of
duration T, given by
0 T T ST
 2T
A for 0 ≤ t ≤
3
S (t ) =  20. If G(  f  ) represents the Fourier transform of a Signal
0 2T
for ≤ t ≤T g(t), which is real and odd symmetric in time, then
 3  [1993]
 [1992]
Solution: (a) G ( f ) is complex
y(t) (b) G ( f ) is imaginary
(c) G ( f ) is real
2m2T
(d) G ( f ) is real and non-negative
3 Solution: (b)
We take a real and odd symmetric signal as exempla,
and verify its properties.
t
0 T T ST Let f(t) = −eat u(−t)+ e−at u(t)
e−a(t)

Signal S(t) is given by


S(t) e−atu (t )
e atu (−t)

t
µ
real and odd symmetric.
taking Fourier transform
t
0 T 2T T 0

3 3 F (w ) = ∫ f (t ) ⋅ e − Jwt dt = − ∫ e at ⋅ e − Jwt dt + ∫ e − at ⋅ e − Jwt dt


Impulse-response of a matched filter is given by h(t) = − − 0

S(T-t) 1 1 1 2J w
= =− + = 2
h(t) 1 + aJ a − Jw a + Jw a + w 2
2Jw
So, F(w) = ⇒ poorly imaginary and odd.
a2 + w 2
µ
Hence, the correct option is (d).
2 1. The magnitude and phase function for a distortionless
t filter should, respectively, be[1990]
0 T 2T T
(Magnitude) (Phase)
3 3
(a) Linear Constant
The output of the matched filter y(t)
(b) Constant Constant
y(t) = S(t)*h(t)
(c) Constant Linear
=> dy (t ) = s (t ) * dh(t ) = u(t ) − u  t − 2T   * S  t − T  − d (t − T ) (d) Linear Linear
3     3 
dt dt    Solution: (c)
 T  5T  For distortionless filter
= u  t −  − u (t − T ) − u (t − T ) + u  t −
 3  3 
 T  5T  f (t ) → h(t ) or h (w ) → A ⋅ f (t − t 0 )
= u  t −  − 2u (t − T ) − u  t −
 3  3  If Fourier transform of function f(t) →F(w),
t then Fourier transform of A·f(t − t0) → A F(w)·e−Jwt0
u T   5T 
=> y (t ) = ∫  t − 3  − 2u(t − T ) + u  t − 3  dt
 y (w )

So , H (w ) = = A ⋅ e − J wt 0
 T  5T  x (w )
= r  t −  − 2r (t − T ) + r  t −
 3  3 
2.34 | Signals and Systems

Taking Inverse Fourier transform of H(w) 2. The Fourier transform G(w) of the signal g(t) in figure
h (t) = A d (t − t0). 1 is given as:
Taking magnitude and phase of H(w)
1 jw
|H(w) = A| and ∠ H(w) = −w t0 G (w ) =
 (e − jw e jw − 1). Using the information,
as H(w) = A ⇒ constant (Magnitude Response) w2
and the time-shifting and time-scaling properties, de-
and ∠H(w) = −w t0 ⇒ Linear (Phase Response)
termine the Fourier transform of signals in figures (2),
Hence, the correct option is (c).
(3) and (4).[2000]
g (t)
Five-marks Questions
1. A periodic signal g(t) is shown in figure. Determine the 1
PSD of g(t).[2001]
g(t)
t
−1

1 g 1(t)

t 1
−2π −π / 2 π /2 2π 4π (sec)
Solution:
g(t) is a periodic signal with, T0 = 2p t
1
1 1 g 2(t)
f0 = =
T0 2p
1 1
w 0 = 2p f 0 = × 2p = 1 rad/sec
2p
Express g(t) as a Fourier series, t
−1 0

g (t ) = ∑e
n = −∞
m e j 2p nf0 t g 3(t)

1
T0 1
∫e
− j 2 np f0 t
Cn = dt
T0 0

Cn can be calculated as, t


−1/ 2 0 1/2
 np 
sin  
1  2  1  np  Solution:
Cn = = sa  
2  np  2  2
  If g (t ) ←
F .T
→ G (w )
2
1 w
then, g (at ) ← →
F .T
1 G 
C0 =
      |a|  a
2
Cn = 0 n = ±2, ±4, ±6……….
      g ( −t ) F
.T .
→ G ( −w )
1 g (t − t0 ) F
.T .
→ e − jwt0 G (w )
Cn = n = ±1, ±5, ±9
pn
Using above properties, G1(w) = G(−w)
−1
Cn = n = ±3, ±7, ±11……. . 1 − jw
np G1 (w ) =e + jw e− jw − 1
w2 
Let, PSD of g(t)
2
and g2(t) = g(−t −1) = g(−(t + 1))
∞ ∞
  np  
Sg ( f ) = ∑ |Cn |2 = ∑  0.5sa  2   G2 (w ) = e jw G1 (w ) =
1
[1 + jw − e jw ]
n = −∞ n = −∞
w2
For n = ±1, ±3, ±5……….
Chapter 4  Fourier Transforms  |  2.35

g 1(t + 1/2)
t0
1
1 Rxx (t ) =
T0 ∫ x(t ) . d (t − t )dt
0
1

= ∫ cos( 2p t) cos 2p (t − t )dt


0
t
−1/ 2 1/2
Using, 2 cosA cosB = cos(A − B) + cos(A + B)
g (t − 1/2)
1 1
1 1 1 1
Rxx (t ) = cos( 2pt ) = cos(2pt )∫ dt + ∫ cos( 4 pt − 2 pt ) dt
2 2 0
20
1 1
= cos( 2pt ) + [sin(4p − 2pt ) − sin( −2pt )]
2 4p
t
−1/ 2 1/ 2
1 1
g 3(t)
= cos( 2pt ) + [sin(4p ) cos(2pt )
2 4p
− cos( 4p ) sin(2pt ) + sin(2pt )]
1
Solving
1
t = cos2pt
−1/ 2 1/2 2
1
 1  1 Rxx (t ) ←
F .T
→ S xx ( f ) = [ f ( f − 1) + f ( f + 1)]
g3 (t ) = g1  t +  + g  t −    4
 2  2
1 − jw 1 d 1 d
G3 (w ) = e jw / 2 . e + jwe − jw − 1 (b) y (t ) = x( f ) = cos 2p t = − sin 2p t
w2  2p dt 2p dt
1   1   1
+ e − jw / 2 2 [e jw − jw tw − 1] y (t ) = cos  2p  t +   = xt + 
w   4   4

Solving
As time shift has no effect on ACF and PSD
1 0.5 jw
= [e − e −0.5 jw ] So,
jw
1
Ryy (t ) = Rxx (t ) = cos( 2p t)
2 w w 2
= sin   = sa  
w  2  2 1
S yy ( f ) = S xx ( f ) = [ f ( f − 1) + f ( f + 1)]
4
3. A deterministic signal x(t) = cos 2pt is passed through
a differentiator as shown in figure. T0
1
(a) Determine the autocorrelation Rxx(p) and the power (c) Rxy (t ) =
T0 ∫ x(t ) . y(t − t )dt
spectral density Sxx(  f   ). 0

(b) Find the output power spectral density syy(  f   ). 1


T0

(c) Evaluate Rxy(0) and Rxy  1  .[2000]


=
T0 ∫ cos(2p t).[− sin{2p (t − t )]dt
  0
4
−1  0 
T T0

x(t ) y (t)
= ∫ sin(upt − 2p t) dt − ∫ sin 2p , dt 
1 d T0  0 0 
2π dt
1 0 T

Rxy (t ) = sin(2pt ) ∫ dt
Solution: 2 0
Given signal is periodic with T0 = 1 sec
1
(a) and f0 = 1 Hz ACF = sin(2pt)
2
2.36 | Signals and Systems

 Rxy(0) = 0 y (t )

 1 1 p 1
Rxy   = sin   = A2T
 4 2  2 2

t
4. Consider a rectangular pulse g(t) existing between −T +T
T T
t=− and . Find and sketch the pulse obtained
2 2 g (t ) F
.T
→ G ( f ) = AT sin c ( fT )
by convolving g(t) with itself. The Fourier transform
of g(t) is a sine function. Write down the Fourier trans- y(y) = G(  f  ) . G(  f  ) = A2T 2 sin c2 (  f T )
form of the pulse obtained by the above convolution. x(t) = cos 2p
 [1998]
Solution: 5. A signal 3 sin(p f0t) + cos(3p f0t) is applied to an RC low
T pass filter of 3 dB cut-off frequency f0. Determine and
Given that, g(t) = A t≤
2 plot the output power spectrum and also calculate the
 = 0      otherwise. total input and output normalized power. [1996]
g(t) Solution:

x(t ) RC / Oω
A y (t)
Pass filter

Given, x(t) = 3 sin(pf0t) + cos(3pf0t)


t
−1/ 2 1/2  f   3f 
x( f ) = 3 sin  2p 0 t  + cos  2p 0 t 
d g (t)  2   2 
dt
the frequency response of TC low pass filter.
A
1
H( f ) =
1/2 1 + j 2pRC
t
−1/ 2
1
3 dB cut off frequency, f 0 =
−A 2pRC

1  f 
d ∴ H( f ) = , LH ( f ) = − tan −1  
Let, y1 (t ) = g (t ) * g (t ) then, y (t ) = ∫ y1 (t )dt  f   f0 
dt t 1+ J  
 f0 
 T  T
y1 (t ) = g (t ) * Af  t +  − g ( f ) * Af  t − 
 2 2  1
q1 = − tan −1   at f = f 0
 2 2
 T  T
⇒ y1 (t ) = Ag  t +  − Ag  t − 
 2   2  3
q 2 = − tan −1   at f = 3 f 0
 2 2
x(t)
1
H( f ) = ,
2
A2  f 
1+  
 fb 
T
t 1 2 f0
−T H ( f1 ) = = at f =
2 5 2
 1
1+  
−A2  2
y (t ) = ∫ y1 (t )dt
Chapter 4  Fourier Transforms  |  2.37

3 f0 So(t )
1 2
H ( f2 ) = = at f 0 =
2
13 2
 3
1+   50/26 50/26
 2
18/10 18/10
So output y(  f  )

2 2
y( f ) = 3 × sin(p f 0 f + q1 ) + 5 cos (3p f 0 tPq 2)
5 13 −3f0 /2 −f0 /2 0 f0 /2 3f0 /2


the input periodic signal consists of 1st harmonic 6. An input signal A exp(−at) with a > 0 is applied to a
 3 
2 causal filter, the impulse response of which is A exp(−
of  f0/2 having power =  = 4.5 W at). Determine the filter output sketch it as a function of
 2 
time and label the important points. [1996]
2
 5  Solution:
Power of 3rd harmonic =  = 12.5
 2  A
Input x(t) = Ae −t u (t ) F
.T .
→ x( f ) =
the input power spectral is given by, a + j 2p f
Impulse response
9  f   f 
f i (t ) =  f  f − 0  + f  f + 0   1
2  2  2  h( f ) = Ae −a tu (t ) F
.T .
→H( f ) =
a + j 2p f
25  3 f  25  3f 
+ f  f − 0 + f  f + 0
4  2  4  2 
x(t ) h (t) y (t)

So, total input power = 4.5 + 12.5 = 17 W


 6  18
2
f0 y(t) = x(t) * h(t) → x(s) . H(s)
Output power = = W at and
 5 2  5 2
A A A2
2 y( f ) = × =
 10  50 3f (a + j 2p f ) (a + j 2p f ) (a + j 2p f ) 2
= = W at 0
 3 2  13 2
1
18 50 484 y (t ) = A2 te −a t u (t ) f .e −a t u (t ) →
Total output power = + = W (a + j 2p f ) 2
5 13 65
The output power spectrum is given by, y(t )

18   f   f 
S0 (t ) = f f − 0 + f  f + 0
10   2  2 ym ym = A2/αe
50   3 f0   3 f0    tm = 1/α
+  f  f − 2  + f  f − 2   
26  
tm
Si (t )

d
25/4 25/4 y (t ) = 0 at t = tm
dt
9/4 9/4
1
Solving, tm =
a

−3f0 /2 −f0 /2 0 f0 /2 3f0 /2 A2 e −1 A2


ym = y (tm) = A2 tm e −atm = =
a ae
2.38 | Signals and Systems

7. A signal v(t) = [1 + m(t) cos(wct) is detected using So the spectrum of V0(t) for fm < f < fm is the addition of
a square law detector, having the characteristic three spectrum
v0 = v2. If the Fourier transform of m(t) is constant, 0.5 m(f ) + m(f )
M0, extending from −fm to + fm, sketch the Fourier
transform of v0(t) in the frequency range −fm <  f <  fm. m1 (f ) = m0(1+ m0fm)

 [1995]
mg = m0(1+ 0.5m0fm)
Solution: 0.5

V0(t) = V(t)2 = (1 + m(t))2 cos2(wct)


f
−fm +fm
1 + 2 cos 2 w c t 
= [1 + m 2 (t ) + 2m(t )]  
 2 
8. A signal, f (t) = e−at u(t), where u(t) is the unit step func-
   = [0.5 + m(t) + 0.5 m2(t)] + [0.5 + m(t) tion, is applied to the input of a low-pass filter having
+ (0.5)m2(t)] cos 2p fc b
H (w ) = .
Let, V1(t) = 0.5 + m(t) + 0.5 m2(t) w + b2
2

V1(  f  ) = 0.5f(  f  ) + m(  f  ) + 0.5 ] m(  f  ) * m(  f  )


a
0.5d(f ) Calculate the value of the ratio, for which 50% of
b ­output.
the input signal energy is transferred to the
[1994]
0.5
Solution:

1
e − at u (t ) F
.T
→ F (w ) =
f a + jw
m(t )
1
Input energy spectrum density, ( ESD) x = | F (w ) |2 =

a +w2
2

mo 1
( ESD) x = | F (w ) |2 =
a2 + w 2

1 1
Input signal energy Ex = ∫ | F (w ) dw =
2

f 2p −∞
2a
−fm +fm
Output spectrum density
b2 1
0.5 m(f ) + m(f )
| y (w ) |2 = | H (w ) |2 | F (w ) |2 = × 2
(w + b ) (a + w 2 )
2 2

Multiplication in frequency domain is equivalent to


convolution in time domain.
f
−2fm +2fm
b − b|t | 1 − a|t |
So, Ry (t ) = e e
2 2a
V2(t) = V1(t) cos 2wct
at t = 0,
1
V2 ( f ) = [V1 ( f − 2 f c ) + n1 ( f + 2 f c )]
b  bt at 
0 ∞
2
 ∫ e e dt + ∫ e . e . dt 
− bt − at
R y ( 0) = E y =
4a  −∞ 
Because of frequency shift by 2fc, the spectrum for y2(t) 0

≠ 0 in the range of −2fc − 2fm to −2fc + 2fm and 2fc − 2fm b 1


= = Ex.
to 2fc + 2fm. 2a ( a + b) 2
and y2(t) = 0 for −fn < f < + fm provided fc ≥ 1.5 c
b 1
V0(  f  ) = V1(  f  ) + V2(  f  ) (or) =
2a ( a + b) 4a
V0(  f  ) = V1(  f  ) + V2(  f  )
(or) b = a, so a/b = 1
V0(  f  ) = V1(  f  ), for − fm < y < + fm.
Chapter 4  Fourier Transforms  |  2.39

9. Consider the following interconnection of the three LTI  f  1  f 


1
systems (Figure), h1(t), h2(t) and h3(t) are the impulse g1 (t ) F
.T
→ G1 ( f ) = H1 ( f )  f f − 0  + f  f + 0 
responses of these three LTI systems with H1(w), H2(w) 2  2 2  2 
and H3(w) as their respective Fourier transforms. Given
jp f 0  f  jp f 0  f 
that G1 (t ) = f  f − 0 − f  f + 0
4  2  4  2
d  Sin(w 0 t )   − j 2pw 
h1 (t ) =   , h2 (w ) = exp   jp f 0   f  f 
dt  2p t  
 w0  ⇒ =  f  f − 0  − f  f + 0
4   2  2 
t
h3(t) = u(t) and x(t) = sin2w0t + cos w0
2
Find the out-
put y(t).[1993]
( ) V1(t) = 10 cos 2000 pt + 4 sin 200 pt
V1(t)2 = 100 cos2 2000 pt + 16 sin2 200 pt
+ 80 cos (2000 pt) . sin(200 pt)
+
x(t) h1(t ) + h3(t ) y (t )  cos 4000p t + 1  1 − cos 400p t 
- = 100   + 16  
 2   2 
h2(t )  sin 2200p t − sin 1800pt 
+ 80  
 2 
Solution:
g1(t ) + y (t )
So V2(t) has frequencies (in Hz)
x(t) h1(t ) + h3(t ) y (t ) = 0, 200, 900, 1100 and 2000
- V1(t) has = 100, 1000 Hz,
g2(t )
h2(t )
B.W. of B.P.F. is from 800 to 1200 Hz,
So only term 900 Hz, 1000 Hz and 1100 Hz will pass to
Given that, the output V3(t)
V3(t) = 10 cos (2000 pt) + 4 sin (2200 pt)
d  sin(w 0 t )  d  f 0 sin 2p f 0 t  − 4 sin (1800 pt)
h1 (t ) =  =  
dt  2p t  dt  2p f 0 t  V3(t) = 10 cos (2000 pt) + 8 cos (2000 pt)
sin (200 pt)
d    =2 cos 2000 pt [5 + 4 sin (200 pt)]
= [ f 0 sin e(2p f 0 t)]
dt
 
G2( f ) = G1( f ) . H2( f ), H1 (t ) = exp  − j 2À f 
f 0 sin c(2p f 0 t ) ←
F .T
→  fh 

jp f 0   f   f  
Gg ( f ) = f f − 0  e − jp − f  f + 0  e jp 
0.5 4   2  2 

jp f0   f   f 
f =  − f  f − 0 + f  f + 0
−fo +fo 4   2   2 

jp f 0   f0   f0  
d y1 ( f ) = G1 (t ) − G2 (t ) =  f  f −  − f  f + 
h1 (t ) = [( f 0 sin c(2p f 0 t )] F
.T
→ 2   2  2  
dt
e jp f0  +2p j 20 t − j 2p 20 t 
f f
H1(t) = j2pf × 0.5 = jp f | f | ≥ f0 y1 (t ) = e −e 
= 0 | f | > f0 2  
In the input, x(t) there are two terms with frequencies w0 w t
2f0 and f0/2, the term with frequency 2f0 will not pass =− sin  0 
2  2 
through the block with impulse response. H1(t) or trans-
fer function H(t) as its cut-off frequency is f0. given from block 3
t
w t 1  f   f 
cos  0  →  f  f − 0  + f  f + 0  
 2  2  2   2 
h3 (t ) = u (t ) = ∫
−∞
f (t )dt
2.40 | Signals and Systems

w0 w  Assume that v2(t) = v1(t) + 0.1v21(t) and that the BPF is


y ( f ) = ∫ y1 (t )dt = − ∫ sin  0 f  dt an idea! unity gain filter with pass band from 800 Hz to
2  2 
1200 Hz.[1993]
w 
= cos  0  f
 2 Non-linear BPF
system
V1(t) V2(t ) V3(t)
10. Obtain an expression for the signal v3(t) in figure for
v1(t) = 100cos(2000pt) + 4 sin(200pt).
Chapter 5
Laplace Transforms
3. Let x(t) = α s(t) + s(–t) with s(t) = βe–4t u(t), where u(t)
One-mark Questions is unit step function. If the bilateral Laplace transform
of x(t) is [2015]
1. Let Y(s) be the unit-step response of a causal system
16
having a transfer function G ( s ) =
3−s
that X(s) = − 4 < Re {s} < 4;
( s + 1) ( s + 3) s 2 − 16
G (s)
is Y(s) = . The forced response of the system is then the value of β is ______.
 s [2019] Solution:  x(t) = αs(t) + s(–t)
(A) u(t)
s(t) = βe–4t u(t)
(B) 2u(t)
(C) u(t) – 2e–t + e–3tu(t) x(t) = αβe–4t u(t) + βe+4t u(–t)
(D) 2u(t) – 2e–t u(t) + e–3tu(t) αβ −β
3− S X(s) = +
Solution: Y ( S ) = s+4 s−4 
S ( S + 1) ( S + 3)
αβ β
Applying partial fraction method = −
s + 4 s − 4
A
Y (S ) = +
B
+
C αβ ( s − 4 ) − β ( s + 4 )
S S + 1 S +3 =
s 2 − 16

A = 1; B = −2; C = 1. αβ s − αβ.4 − β s − β.4 16
1 −2 1 X(s) = = 2
Y (S ) = +
2
= s − 16 s − 16 
S S +1 S + 3 It is given
On taking inverse laplace transform
So αβs – βs = 0
Y (t ) = u (t ) − 2e − t u (t ) + e −3t u (t ) βs(α – 1) = 0

Forced response : u(t) ∴ α = 1
Hence, the correct option is (A). and –α β.4 – β.4 = 16
2. Which one of the following is a property of the solu-
–β.4 – β.4 = 16
tions to the Laplace equation: ∇2ƒ = 0? [2016]
(A) The solutions have neither maxima nor minima –8β = 16
anywhere except at the boundaries.
β = –2
(B) The solutions are not separable in the ­coordinates.
(C) The solutions are not continuous. Hence, the correct Answer is (–2).
(D) The solutions are not dependent on the boundary 4. The impulse response of a system is h(t) = tu(t). For an
conditions. input u (t − 1), the output is [2013]
Solution:  It is known that laplace equation has ­maxima t2 t (t − 1)
and minima at boundaries only and at all remaining (a) u (t ) (b) u (t − 1)
2 2
points there are no such conditions.
Hence, the correct option is (A). (t − 1) 2 t 2 −1
(c) u (t − 1) (d) u (t − 1)
2 2
2.42 | Signals and Systems

Solution: (c) 1
Given h (t) = t u (t) 6. The unilateral Laplace transform of f(t) is .
S + S +1
2
Taking LT on both sides The unilateral Laplace transform of tf(t) is [2012]
e −S S −( 2S + 1)
X (S ) = (1) (a) 2 (b)
S (S + S + 1) 2 (S 2 + S + 1) 2

Input x(t) = u (t − 1) −S ( 2S + 1)
(c) 2 (d)
Taking LT on both sides (S + S + 1) 2 (S 2 + S + 1) 2
Solution: (d)
e −S
X (S ) = (2) 1
S L[ f (t )] = = f (S )
S2 + S +1
Y (S ) d
∴ H (S ) = ∴ L [t f (t )] = ( −1)1f (S )
X (S ) dS
∴Y (S ) = H (S ) ⋅ X (S ) d  1 
= −1  2
dS  S + S + 1 
Equation value of H(S) & X(S) from (1) and (2)
taking inverse Laplace transform  −1  ( 2S + 1)
= − 2 2 
( 2S + 1) = 2
 (S + S + 1)  (S + S + 1) 2
(t − 1) 2
y (t ) = u (t − 1) Hence, the correct option is (d).
2
7. If the unit step response of a network is (1− e at ), then
Hence, the correct option is (c). its unit impulse response is [2011]
5. Assuming zero initial condition, the response (a) − at
ae (b) −1 − at
a e
y (t) of the system given below to a unit step input u (t)
is [2013] (c) (1 − a −1 )e −at (d) (1− a )e −at
Solution: (a)
1 Given unit step response S(t) (1− e at )
u(S) Y(S )
S For a given response, unit impulse response is
ds(t )
(a)
u (t) (b)
t u (t) h(t ) =
dt
t2 d (1 − eat )
e−tu (t)
(c) u (t) (d) = = ae −at
2 dt
Solution: (b) Hence, the correct option is (a).
8. If the Laplace transform of a signal y (t) is
Y (S ) 1
H (S ) = = 1
u( S ) S Y(S ) = , then final value is____. [2007]
S ( S − 1)
x (t ) = u (t ) {unit step input} (a) −1 (b) 0
(c) 1 (d) unbounded
Taking Laplace transform on both sides Solution: (d)
Here final value theorem will not be applicable as all of
1
X (S ) = system are not lining left half of s-plane,
S ∴ S = 1 is right S-plane pole.
Y (S ) = H (S ) ⋅ X (S ) ∴ unbounded.
Hence, the correct option is (d).
1 1 1
= × = 2 9. The Laplace transform of i (t) is given by
S S S
Taking inverse Laplace transform 2
I (S ) = , As t → ∞, the value of i (t) tends to
y (t ) = t u (t ) S (S + 1)

 [2003]
Hence, the correct option is (b).
Chapter 5  Laplace Transforms  |  2.43

(a) 0 (b) 1 Solution: (b)


(c) 2 (d) ∞ ⋅T
f (t ) L → F ( s)
Let,
Solution: (c) ⋅T
f (t − T ) L → F ( s)e − ST
Using final value theorem
Time shifting properly of Laplace transforms.
lim i(t ) = lim S ⋅ I ( S ) Hence, the correct option is (b).
t →∞ S →0

2 p
= lim S ⋅ = 2. 13. If L( f (t )) = , then the value of lim f (t )
S →0 S ( S + 1) s2 + w 2 t →∞
 [1998]
Hence, the correct option is (c). (a) cannot be determined
10. The transfer function of a system is given by (b) is zero
1 (c) is unity
H(s) = 2 . The Impulse response of the system (d) is infinite.
S (S − 2)
is [2001] Solution: (a)
(a) (t2 * e−2t) u(t) (b) (t * e2t) u(t) w I ⋅ L ⋅T
(c) (t − e−2t) u(t) (d) (t − e−2t) u(t) F (s ) = ← → f (t ) = sin wt
S2 +w2
Solution: (b)
Taking inverse Laplace transfer to calculate impulse re- ⇒ lim f (t ) = sin wt = lie between −1 and +1 cant de-
sponse t →∞
termined.
  Hence, the correct option is (a).
1 −1  1 1 
h (t ) = L−1 [H (S )] = L−1  2 =L  2×  14. The Laplace transform of eat ⋅ cos(at ) is equal to
 S (S − 2)  S S − 2 
 [1997]
= (t ∗ e 2t )u (t )
(s − a ) (s + a )
Hence, the correct option is (b). (a) (b)
(s − a ) 2 + a 2 (s + a ) 2 + a 2
1 1. Given that
1
(c) (d) None of these
S +2 S 2 +1 (s − a ) 2
L (f (t )) = 2 , L [ g (t )] =
S +1 (S + 3)(S + 2)
Solution: (a)
t Laplace transform of cosatu(t) is given as
h (t ) = ∫ f (t )g (t − T )dT . The L [h(t)] is [2000]
0 L ⋅T S
cos(at )u (t ) ← →
S +a2
2
S 2 +1
(a)
S +3 Multiplying by eat will result shifting in frequency do-
main
1
(b)
S +3 (s − a )
e at cos(at ) u (t ) ←
L ⋅T

S +1 2
(S + 2) (s − a ) 2 + a 2
(c) + 2
(S + 3)(S + 2) S + 1 Hence, the correct option is (a).
(d) None of these
Solution: (b) 2( s + 1)
15. If L( f (t )) = then f(0t) and f(∞) are given by
h (t) shows convolution of f (t) and g (t) and convolu- s 2 + 2s + 5
tion in time domain equivalent to multiplication in s-  [1995]
domain (a) 0, 2 respectively
(b) 2, 0 respectively
1 (c) 0, 1 respectively
∴ L[h(t )] = L[ f (t )] × L[ g (t )] =
S +3 (d) 2/5, 0 respectively.
Hence, the correct option is (b). Solution: (b)
1 2. If L( f (t )) = F ( s), then L( f (t − T )) is equal to [1999] S 2( s + 1)
(a) eST F(S) (b) e−ST F(S) f (ot ) ≠ lim
s →∞ s 2 + 2s + 1
F (s ) F (s ) =2
(c) ST (d)
1+ e 1− e −ST
2.44 | Signals and Systems

S 2( s + 1) zeros of G(s) lie in the right half of s plane, hence given


f (∞) = lim SF ( s) = lim statement is not True.
s→0 s→0 S 2 + 2s + 5
t(∞) = 0. 1
So F (s ) is not always a stable function.
Hence, the correct option is (b). G (s )
1 6. The transfer function of a linear system is the [1995]
(a) ratio of the output v0(t), and input vi(t)
(b) ratio of the derivatives of the output and i/p Two-marks Questions
(c) ratio of the Laplace transform of the output and that 1. A sequence x[n] is specified as
of the i/p with odd initial conditions zero
n
(d) None of these ⎡ x[n] ⎤ ⎡1 1 ⎤ ⎡1 ⎤
Solution: (c) ⎢ x[n − 1]⎥ = ⎢1 0 ⎥ ⎢0 ⎥ for n ≥ 2
⎣ ⎦ ⎣ ⎦ ⎣ ⎦
y (s ) The initial conditions are x[0] = 1, x[1] = 1, and x[n] =
Transfer function given H (s ) = = ratio of Laplace
x (s ) 0 for n < 0. The value of x[12] is _____.  [2016]
transform of output to the input.
Hence, the correct option is (c). ⎡ x [ n] ⎤ n
⎡1 1 ⎤ ⎡1 ⎤
Solution:  Given ⎢ ⎥= ⎢1 0 ⎥ ⎢0 ⎥ , for n ≥ 2
17. The final values theorem is used to find the [1995] ⎢
⎣ x [ n − 1]⎥⎦ ⎣ ⎦ ⎣ ⎦
(a) Steady state value of the system output where x [0] = 1, x [1] = 1.
(b) Initial value of the system output For n = 2, we have
(c) transient behaviour of the system output 2
⎡ x[2]⎤ ⎡1 1 ⎤ ⎡1 ⎤
(d) none of these ⎢ x[1] ⎥ = ⎢1 0 ⎥ ⎢0 ⎥
⎣ ⎦ ⎣ ⎦ ⎣ ⎦
Solution: (a)
Final value theorem used to find the final value or ⎡ 2 1⎤ ⎡1 ⎤
= ⎢ ⎥⎢ ⎥
steady state value of system o/p. F (∞) = lim SF ( s) ⎣1 1⎦ ⎣0 ⎦
s →∞
Hence, the correct option is (a).
⎡ x[2]⎤ ⎡ 2⎤
18. The Laplace transform of unit ramp function ­starting at ∴ ⎢ x[1] ⎥ = ⎢1 ⎥
t = a is [1994] ⎣ ⎦ ⎣ ⎦
1 e − as For n = 3, we have
(a) (b) 3
( s + a) 2 (s + a) 2 ⎡ x[3] ⎤ ⎡1 1 ⎤ ⎡1 ⎤
⎢ x[2]⎥ = ⎢1 0 ⎥ ⎢0 ⎥
e − as 4 ⎣ ⎦ ⎣ ⎦ ⎣ ⎦
(c) 2 (d)
s s2 ⎡3 2⎤ ⎡1 ⎤
= ⎢ 2 1 ⎥ ⎢0⎥
Solution: (c) ⎣ ⎦ ⎣ ⎦
Let ramp function be r(t)
⎡ x[3] ⎤ ⎡3 ⎤
1 ∴ ⎢ x[2]⎥ = ⎢ 2⎥
L ⋅T
r (t ) ← → ⎣ ⎦ ⎣ ⎦
s2
1 e − as ⎡ x[4]⎤ ⎡5⎤
L ⋅T
r (t − a) ← → e − as ×
= 2 Similarly, for n = 4, we have ⎢ ⎥=⎢ ⎥
s2 s ⎣ x[3] ⎦ ⎣3⎦
Hence, the correct option is (c). ∴ x[2] = 2; x[3] = 3 and x[4] = 5.
19. Indicate whether the following statement is true/False. From these values, we can observe that
Give reason for your answer. If G(s) is a stable trans- x[n] = x[n − 1] + x[n − 2];
1
fer function, the F (s ) is always a stable transfer ∴ x[5] = x[4) + x[3] = 8
G (s )
function. [1994] x[6] = x[5) + x[4] = 13
Solution: FALSE x[7] = x[6) + x[5] = 21
If G(s) is stable, all poles must lie in left half of
S-plane and there is no restriction on its zeros, which 
can lie also on right half of S plane. The Inverse
x[12] = x[11] + x[10] = 233.
1
function F (s ) may or may not be stable. The Hence, the correct Answer is (233).
G (s )
Chapter 5  Laplace Transforms  |  2.45

2. The Laplace transform of the causal periodic square By the property.


wave of period T shown in the figure below is [2016] 2π
G(s) =
f(t) (S + 1) + (2π )2 
2

+∞

∫ g (t ) e dt
− st
1 G(s) =
−∞ 
+∞

0
So G(0) = ∫ g (t ) dt = area under g(t)
T/2 T 3T/2 2T t −∞

2π 6.28
1 = = = 0.155
F(s) =
(A) 1 + 4π 2
40.4384

1 + e − sT 2
Hence, the correct Answer is (0.14 to 0.16).
1
F(s) =
(B) 4. Let the signal f(t) = 0 outside the interval [T1, T2], where
⎛ − sT
⎞ T1 and T2 are finite. Furthermore, | f(t)| < ∞. The region
s ⎜1 + e 2 ⎟
⎝ ⎠ of convergence (RoC) of the signal’s bilateral Laplace
transform F(s) is [2015]
1 (A) a parallel strip containing the j Ω axis.
F(s) =
(C)
(
s 1 − e − sT )

(B) a parallel strip not containing the j Ω axis.
(C) the entire s-plane.
1 (D) a half plane containing the j Ω axis.
F(s) =
(D)
1− e − sT Solution:  For a finite duration time signal, ROC of
Solution: 
Given periodic signal Laplace transform is entire S-plane.
Hence, the correct option is (C).
⎧1 0 < t ≤ T 2
f (t ) = ⎨ 5. A system is described by the following differential
⎩0 T 2 < t ≤ T equation, where u(t) is the input to the system and y(t)
Laplace transform for periodic signal is is the output of the system
T
1 y ′(t ) + 5y (t ) = u (t )
∫ f (t )e
− st
F ( s) = dt
1 − e − sT
0  When y(o) = 1 and u(t) is a unit step function, y(t) is
1
T 2  [2014]
∫ 1⋅ e
− st
= dt (a) 0.2 + 0.8 e−5t
1 − e − ST
0  (b) 0.2 − 0.2 e−5t
T (c) 0.8 − 0.2 e−5t
1 ⎡ e − st ⎤ 2 (d) 0.8 − 0.8 e−5t
= ⎢ ⎥
1 − e − ST ⎣ − s ⎦ 0 Solution:(a)
1 1 Given y ′(t ) + 5y (t ) = u (t )
. . ⎛ e 2 − 1⎞
− sT
=
1− e − ST
S ⎝ ⎠ taking Laplace transform
⎛1 − e − sT 2 ⎞ ⎛1 − e − sT 2 ⎞ 1
⎝ ⎠ ⎝ ⎠ SY (s ) − y (o ) + 5Y (s ) = ( y (0) = 1)
= = S
(
s 1 − e − ST ) s ⎛1 − e

− ST
2⎞ ⎛1 + e − ST 2 ⎞
⎠⎝ ⎠ (S + 5)Y (S ) − 1 =
1
S
1 1
= ⇒ (S + 5)Y (S ) − 1 = + 1

(
s 1 + e − ST / 2 ) S
(1 + S ) 1 (S + 1)
Y (S ) = × =
Hence, the correct option is (B). S (S + 5) S (S + 5)
3. Consider the function g(t) = e–t sin (2πt) u(t) where u(t)
is the unit step function. The area under g(t) is _______. Taking partial fraction
 [2015] A B (S + 1)
+ =
Solution:  g(t) = e sin(2 π t) u(t)
–t
S (S + 5) S (S + 5)
2.46 | Signals and Systems

Solving A (S + 5) + BS = S + 1 7. An input x (t) = exp (−2t) u(t) + d (t−6) is applied to


and taking S = 0 an LTI system with impulse response h (t) = u (t).
5A = 1 ∴ A = 0.2 The output is [2011]
Taking S = −5 (a) [1 − exp (−2t)] u(t) + u (t + 6)
−4 (b) [1 − exp (−2t)] u(t) + u (t − 6)
−5B = −4 ∴ B = = 0.8
−5 (c) 0.5 [1 − exp (−2t)] u(t) + u (t + 6)
0.2 0.8 (d) 0.5 [1 − exp (−2t)] u(t) + u (t − 6)
Now Y ( S ) = + Solution: (d)
5 S +5
∴Taking inverse LT, x (t ) = e −2t u (t ) + S (t − 6)
we get answer y(t) = 0.2 + 0.8 e−5t
Taking Laplace transform
Hence, the correct option is (a).
6. A system is described by the differential equation 1
X (S ) = + e −6S
d 2 y 5dy S +2
+ + 6 y (t ) = x (t ). Let x(t) be a rectangular
dt 2 dt h(t ) = u(t )
pulse given by
1
1 0 < t < 2 ∴ H (S ) =
x( t ) =  S
0 otherwise ∴ output
dy
Assuming that y(o) = 0 and = 0 at t = 0, =X
dt Y ((SS )) =
Y X ((33)) ⋅⋅ H H ((SS ))
 1 1
Laplace transform of y (t) is [2013] =  1 +
Y ((SS )) =
Y + ee −−66SS  1
e −2S  SS + + 22  SS
(a)
S (S + 2)(S + 3) 11 11 −6S
Y ((SS )) =
Y = +
+ ee −6S
SS ((SS + 2 )
+ 2) S S
1 − e −2S
(b) 11 1  11 −−66SS
S (S + 2)(S + 3) = 1  1 −
Y ((SS )) =
Y − 1  + + ee
 + 22  SS
22  SS SS +
e −2S
(c)
(S + 2)(S + 3) Taking Inverse Laplace Transform
−2S
1− e
(d) y (t ) = 0.5(1 − e −2t )u (t ) + 4(t − 6)
(S + 2)(S + 3)
Hence, the correct option is (d).
Solution: (b)
1 0 < t < 2 2( S + 1)
x (t ) =  8. If F ( S ) = L[ f (t )] = then the initial and
0 otherwise S 2 + 4S + 7
∴ x (t ) = u (t ) − u (t − 2) final values of f(t) are respectively [2011]
(a) 0, 2 (b) 2, 0
Taking Laplace transform (c) 0, 2/7 (d) 2/7, 0
1 e −2S 1 − e −2S Solution: (b)
X (S ) = − =
S S S
2(S + 1)
given F (S ) = L [f (t )] =
S + 4S + 7
2
d 2 y 5dy
+ + 6 y (t ) = x (t ) Initial value of f (t)
dt 2 dt
dy S ⋅ 2( S + 1)
Apply Laplace transform= ( y (o ) 0= ; 0 at t = 0) Lt f (t ) = Lt S F ( S ) =
dt t →0 t →0 S 2 + 4S + 7
dy Taking S common
=
( y (o ) 0=
; 0 at t = 0)
dt
⇒ S 2Y ( S ) + 5SY ( S ) + 6( S ) = X ( S )  1
S 2 ⋅ 2 1 + 
−2 S  S
1− e Lt f (t ) = Lt
⇒ Y ( S )  S 2 + 5S + 6  = t →0 S →∞  4 7
S S 2 1 + + 2 
 S S 
1 − e −2 S
⇒ Y (S ) = 2(1 + 0)
S ( S + 2)( S + 3) = =2
Hence, the correct option is (b). (1 + 0 + 0)
Chapter 5  Laplace Transforms  |  2.47

For final value


S F(S) − f(o) (b)
(a) 1
S
F (S )
Lt f (t ) = Lt S ⋅ F (S ) S
t →0 t →0

S ⋅ 2(S + 1)
= =0
≡f (t )dÄ (d)
(c)
0
1
S
[F (S ) − f (o )]
S 2 4S + 7
Hence, the correct option is (b). Solution: (b)
9. A continuous time LTI system is described by L ⋅T
Given f (t ) ← → F ( s ),
d 2 y (t ) 4dy (t ) 2dx (t ) t
F ( S ) f −1 (o+ ) F ( S )
+ + 3y (t ) = + 4 x (t ).
dt 2 dt dt then ∫ f (t )dt = + = .
0
S S S
Assuming zero initial condition, the response y(t) of the
above system for the input f (o + ) shows initial conditions, which are zero.
−1

x (t ) = e −2t u (t ) is given by [2010] Hence, the correct option is (b).

11. Consider the function of f(t) having Laplace transform
(e t − e 3t )u (t )
(a)
w
(e − t − e −3t) u(t )
(b) F (s ) = 2 0 2 Re {S } > 0
S + w0
(e − t + e −3t ) u(t )
(c) The final value of f(t) would be______. [2006]
(e + e )u (t )
(d) t 3t (a) 0 (b) 1
(c) −1 ≤ f(∞) ≤ 1 (d) ∞
Solution: (b)
Given equation Solution: (c)
w0
d 2 y(t ) 4 dy(t ) 2dx(t ) Given F (s ) =
2
+ + 3 y (t ) = + 4 x (t ) S + w 02
2
dt dt dt

Considering zero initial condition, Laplace transform  w 


f (t ) = L−1 ( F | S ) = L−1  2 0 2  = sin w 0t
on both side  S + w 0 
S 2Y (S ) + 4SY (S ) + 3Y (S ) = 2SX (S ) + 4 X (3) So −1 ≤f (t) ≤ 1
Hence, the correct option is (c).
⇒ (S 2 + 4S + 3)Y (S ) = ( 2S + 4)X (S )
Y (S ) 2S + 4 2(S + 2) 12. The Laplace transform of a continuous time signal x(t)
= =
X (s ) S 2 + 4S + 3 (S + 1)(S + 3) 5−S
is X ( S ) = 2 . If Fourier transform of signal
S −S −2
x (t ) = e −2t u (t ) exists, then x (t) is [2002]
Now      (a) e2t u(t) − 2e−t u(t)
1
X (S ) = (b) −e2t u(−t) + 2e−t u(t)
S +2 (c) −e2t u(t) − 2e−t u(t)
Sub-value of X (S) in the equation (d) e2t u(−t) − 2e−t u(t)
2(S + 2) 2 1 1
Y (S ) = = = − Solution: (d)
(S + 1)(S + 3)(S + 2) (S + 1)(s + 3) S + 1 S Given
+3
5−S 5−S −2 1
2(S + 2) 2 1 1 X (S ) = = = +
= = − S 2 − S − 2 (S + 1)(S − 2) (S + 1) (S − 2)
(S + 1)(S + 3)(S + 2) (S + 1)(s + 3) S + 1 S + 3
taking inverse Laplace transform.
Taking inverse Laplace transform
x (t ) = −2e −t u(t ) + e 2t u ( −t )
( −1
y (t ) = e − e −3t
) u(t ) ROC will include s = 0 line
Hence, the correct option is (b). Hence, the correct option is (d).
1 0. Given that F(S) is one-sided Laplace transform of f(t), 1 3. A linear time invariant system has an impulse response
the Laplace transform of e2t, for t > 0, if initial condition is 0 and input is e3t, the
output for t > 0 is [2000]
t
(a) e3t − e2t
≡f (t )dt
0
is  [2009]
(b) e5t
2.48 | Signals and Systems

(c) e3t + e2t −e − ps


(d) None of the above Solution: F (s ) =
(s 2 + 1)(1 − e − ps )
Solution: (a)
Given Curve f(t) can be generated from sine curve or ­cosine
1 curve.
L ⋅T
h (t ) = e 2t ← → = H (s ) Let
S −2
f 1 (t ) = −[sin(t −)u (t − p ) + sin(t − 2p ) ⋅ u (t − 2p )]
1
L ⋅T
X (t ) = e 3t ← → = X (s ) o/p The given signal f(t) can be represented as
S −3
1 −1 1 f (t ) 0 < t ≤ 2p
⇒ Y (S ) = X (S ) ⋅ H (S ) = = + f 1 (t ) 
(S − 2)(S − 3) S − 2 S − 3  0 otherwise
Inverse Laplace transform 1
o/pY (t ) = e 3t − e 2t Also sin t u (t ) 
→ , and using shifting property
s2 +1
Hence, the correct option is (a).
x (t − t 0 ) → e − t 0 s ⋅ x (s )
14. Match each of items 1, 2 on the left with most appropri-
ate item A, B, C, D on right. Then
In case of linear time-invariant system [1997]  e − ps e −2ps  1 + e − ps 
(1) poles in the right half plane implies F1 (s ) = −  2 + 2  = −e − ps  2 
(2) Impulse response zero for t ≤ 0 implies  s + 1 s + 1  s +1 
(a) Exponential delay o/p. F1 (s ) F (s )  1 + e − ps  1
⇒ F (s ) = = 1 −2ps = e − ps  2  −2p
(b) System is causal. 1− e T0 s
1− e  s +1  (1 − e s)
(c) No stored energy in system.
−e − ps
(d) System is unstable. F (s ) =
(s + 1)(1 − e − ps )
2
Solution: (1 – D, 2 – B)
(i) Poles at right half of s-plane ⇒ system unstable k
(ii) h(t) = 0, for t ≤ 0 ⇒ causal system. 17. If F ( s) = L( f (t )) = , then lim f(t) is
( s + 1)( s 2 + 4)
15. The inverse Laplace transform of the function given by [1993]
(s + 5) (a) k/4
is [1996] (b) zero
(s + 1)(s + 3)
(c) ∞
(a) 2e −t − e −3t (b)
2e −t + e −3t (d) undefined
−t −3t
(c) e − 2e (d) e −t + 2e −3t Solution: (d)
Solution: (a) k
Given Given F (s ) = , poles = −1, ± 2J
(s + 1)(s 2 + 4)
s +5 2 1
F (s ) = = − Since poles are not at left half of s-plane so the time
(s + 1)(s + 3) (s + 1) (s + 3) value theorem cannot be applied.
Taking inverse Laplace transform, So lim f(t)→ indeterminate
t→∞
f (t ) = 2e −t − e −3t Hence, the correct option is (d).
Hence, the correct option is (a). 18. The pole-zero pattern of a certain filter is shown in the
1 6. The Laplace transform of the periodic function f(t) figure. The filter must be of following type. [1991]
described by curve below i.e. [1993] Jω

sin t if ( 2n − 1)p ≤ t ≤ 2np ( n = 1, 2, 3…) J2


f (t ) 
 0 otherwise J1

f(t )
−1 +1 s
π 2π 3π 4π −J1
0 t
Chapter 5  Laplace Transforms  |  2.49

(a) low-pass K (s 2 + w 02 )
(b) high-pass H (s ) =  [1988]
S 2 + (w 0 / Q )s + w 02
(c) all pass
(d) band-pass (a) all pass filter (b) low pass filter
(c) band pass filter (d) notch filter
Solution: (c)
The pole zero pattern shown is symmetrical about im- Solution: (d)
aginary axis. Such property is shown by all pass filter. K (s 2 + w 02 )
Given that H (s ) =
Hence, the correct option is (c). w 
S 2 +  0  s + w 02
19. The voltage across an impedance in a network is v(s) Q 
= z(s) I (s), where v(s), z(s) and I (s) are the Laplace put H(∞) = K, H(∞) ≠0
transform of the corresponding time functions v(t), z(t)
and i(t). The voltage v(t) is [1991] K (1 + 0)
H (∞) = = K, H (∞) ≠ 0
1+ 0 + 0
v(t) = z(t)−i(t)
(a)
t
So, the given transfer function is a notch filter.
v (t ) = ∫ i (t ) ⋅ z (t − T )dT
(b) Hence, the correct option is (d).
0 22. The Laplace transform of function f(t) u(t), where f(t) is
t periodic with period T, is A(s) times the Laplace trans-
v (t ) = ∫ i (t ) ⋅ z (t + T )dT
(c) form of its first period, then [1988]
0
1
(d) v (t ) = z (t ) + i (t ) (a) A(s) = S (b) A (s ) =
1 − e −Ts
Solution: (c) 1
Multiplication of two functions in frequency domain is (c) A (s ) = +Ts
(d) A (s ) = e Ts
1+ e
equivalent to convolution in time domain
Solution: (b)
t

So v (t ) = ∫ i (t ) ⋅ z (t − t )dt Given function represents causal period signal


0
f(t) u(t) = 0: t > 0
Hence, the correct option is (c). period of f(t) = T for t > 0
2 0. The response of an initially relaxed linear constant Let f 1 (t ) = f (t )u (t )
parameter to a unit impulse applied at t = 0 is 4e−2tu(t). =0
The response of this network to a unit step function will 0 ≤ t ≤ T otherwise,
be. [1990] f (t)u (t ) f 1(t)
(a) 2[1−e−2t] u(t)
(b) 4[e−t−e−2t] u(t)
(c) sin 2t t t
(d) [1−4e−4t] u(t) 0 T 2T 0 T

Solution: (a)
f (t )u(t ) = ∑ f1 (t − nT )
4 n= 0
Given h (t ) = 4e −2t 1
⋅T
→ H (s ) 2

(s + 2) Let e −p f
⋅T
⋅T 1 f1 (t − nT ) 1 → e − nTs F1 ( s)
Given that x (d ) 
→ u (t ) 1 → x (s ) =
s ∞
F1 ( s)
⋅T
f (t )u(t ) 1 → F ( s) = ∑ e − nTs F1 ( s) =
Y (s ) 1 4 1 − e −Ts
⇒ H (s ) = ⇒ Y (s ) = X (s ) ⋅ H (s ) = ⋅ n= 0
X (s ) s (s + 2)
1
1 1  f (t )u(t ) 
→ × transform of first period of f(t)
Y (s ) = 2  −  1 − e − sT
 s ( s + 2)  u(t)
Taking Inverse Laplace transform on both side, 1
A (S ) =
y (t ) = 2[1 − e −2t
]u (t ). 1 − e Ts
Hence, the correct option is (b).
Hence, the correct option is (a).
2 3. Laplace transform of the function tu(t) and sin tu(t) is,
2 1. Specify the filter type if its voltage transfer function
respectively: [1987]
H(s) is given by
2.50 | Signals and Systems

1 S 1 1 Solution:
(a) 2 , 2 (b) , (a)
S S +1 S S 2 +1
x(t )
1 1 S
(c) 2 , 2 (d)
S, 2
S S +1 S +1
Solution: (c) 1
Laplace transform of function f(t) is given as
t
∞ 0 1 2

− st
F |S| = f (t )e dt
−∞ x( s ) − 4 y1 ( s ) − 3 y ( s )
y1 ( s ) =
y(t) = tu(t) s
y1 ( s )
∞ ∞
y(s) = , ⇒ y1(s) = sy(s)
∫ tu(t ) ⋅ e dt = ∫ t ⋅ e − st dt
− st
F (S ) = 2
−∞ 0
x( s ) − 4sy ( s ) − 3 y ( s )
∞ ∞ ⇒
dt  = [0 − 0] −  e 
 t ⋅ e − st   ∞ e − st  − st s

⇒ F (S ) =   −  ∫ 1⋅
 ( −S ) 0  0 ( −S )   S 2  0 1 1 1 
=  − e.e − t + e3 e −3t  u (t − 1)
−1 1 3 2 6 
⇒ F ( S ) = 2 [0 − 1] = 2
S S 1 1 1 
−  − e 2 e − t + e6 e −3t  u (t − 2)
When f(t) = sin at·u(t) 3 2 6 
1 Transform function,
similarly for sin t u(t) the transform is
S +1
2

Hence, the correct option is (c). Y ( s) 1 1


H (s) = = 2 =
X ( s ) s + 4 s + 3 ( s + 1)( s + 3)

Five-marks Questions (b) x(t) = u(t),


1
1. For the linear, time-invariant system whose block dia- x( s ) = ,
s
gram is shown in figure (a), with input x(t) and output
y(t) Y (s)
H (s) = = a
→ ⇒ y ( s ) = h( s ) . x ( s )
(a) Find the transfer function. X (s)

(b) Find the step response of the system [i.e. find y(t)
when x(t) is a unity step function and the initial 1 1 1 1
⇒ Y ( s) = = − +
conditions are zero]. s ( s + 1)( s + 3) 3s 2( s + 2) 6( s + 3)
(c) Find y(t), if x(t) is as shown in figure (b), and the
initial conditions are zero.[2000] 1 1 1 
y (t ) =  − d − t + e −35  n (t )
y1(t)  3 2 6 
x(t) + ∫ ∫ y(t)

− (c) x(t) = u(t − 1) − u(t − 2)
4 For LTI systems,
u(t) → y(t)
3 u(t − 1) → x(t − 1)
u(t − 2) → y(t − 2)
x(t ) So, x(t) = u(t − 1) − u(t − 2) → y(t − 1) − y(t − 2)

1 1 1 
=  − e − (t −1) + e −3(t −1)  u (t − 1)
 3 2 6 
1
2. A sinusoidal signal, v(t) = A sin(t), is applied to an ideal
t full-wave rectifier. Show that the Laplace transform of
0 1 2
the output can be written in the form,
Chapter 5  Laplace Transforms  |  2.51

A A −p s
V0 (s ) = cot h(as ) V1 (t ) L 
.T .
→ V1 ( s ) = V ( s ) + e
s +12
s +1 2

p
where a is a constant. Determine the value of a. A A −
V1 ( s ) = + e s

[1995] ( s 2 + 1) ( s 2 + 1)
Solution:  p2s −ps

−ps
e + e 
2

V(t) V(t ) = t sin(t) u (t) A e 2


 
= × −ps
( s 2 + 1)  2
ps −ps

A e2 e − e 
2

 
π t
2π 3π A  p s p
V0 ( s ) = cot h   , So, a =
s2 + 1  2 2

3. Find the Laplace transform of the waveform x(t) shown


A in figure.[1991]
V (t ) 
→ V ( s) = 2
L .T .

s +1
V0 (t ) 3
x(t)
2
A
1

π 2π t

0
T 2T 3T 4T 5T 6T t
−1
−2
V1(t)
−3

Solution:
π
x(t) is expressed as,
(t −π )
x(t) = u(t) − 3u(t − T) + 4u(t − 2T) − 4u(t − 4T) + 2
−4 / s
1 3e −Ts 4e −2Ts e 4 e 2e − sTs
So, x( s ) = − + − +
π t s s s s s
0 2π 3π
1
    V1(t) = V(t) + V(t − p) = 1 − 3e −Ts + 4e −2Ts − 4e −4Ts + 2e − sTs 
s
A − ps
V (t − p ) L
.T
→ e
s2 + 1
Chapter 6
Z-Transform
(C) z-plane Imaginary
One-mark Questions axis
2
1. Let H(z) be the z-transform of a real-valued discrete- |z| = 1
⎛ 1⎞
time signal h(n). If p(z) = H(z) H  ⎜ ⎟ has a zero at z = 0.5
⎝ z⎠
1 1
+ j and p(z) has a total of four zeros, which one of
2 2 –2 –0.5 0.5 2 Real axis
the following plots represents all the zeros correctly?
 [2019]
(A) z-plane Imaginary
–2
axis
2
(D) z-plane Imaginary
|z| = 1 axis
2
0.5
1
|z| = 1

–2 –0.5 0.5 2 Real axis 0.5

–0.5
–2 0.5 2 Real axis
–0.5
–2
–1
(B) z-plane Imaginary
–2
axis
2
 1
Solution: P ( z ) = H ( z ) ⋅ H  
|z| = 1  z
0.5 1 1
Zero at z = + j
z 2
–2 0.5 2 Real axis if z is zero, z will also be zero
–0.5
z =  1 + 1  1 1
j = − j
 2 2  2 2
–2
Chapter 6  Z-Transform  |  2.53

1 1 −1
also & will also be zero ∴ Re s[ f ( z )] =
z z z =− 1 125
1 1 Hence, the correct option is (B).
− j
1 1
= × 2 2 = 1− j 3. A continuous time function x(t) is periodic with period
z 1 1 1 1
+ j − j T. The function is sampled uniformly with a sampling
2 2 2 2 period Ts. In which one of the following cases is the
1 sampled signal periodic? [2016]
= 1+ j
z (A) T = 2 Ts (B) T = 1.2 Ts
Hence, the correct option is (D). (C) Always (D) Never
1 Solution:  As we know that in Nyquist theorem,
2. The residues of a function f ( z ) = are
( z − 4)( z + 1)3 Fs ≥ 2Fm
 [2017] or Tm ≥ 2Ts

−1 −1 1 −1 or T ≥ kTs
(A) and (B) and Hence, the correct option is (B).
27 125 125 125
s( z )
−1 1 1 −1 4. For f(z) = 2 the residue of the pole at z = 0 is ____.
(C) and (D) and z
27 5 125 5  [2016]
sin Z
Given f(Z) =
Solution: 
1 Z2
Given f ( z ) =
Solution: 
( z − 4)( z + 1)3 Z = 0 is a pole of order 2 for f(Z);
z = 4 and z = −1 are the singularities of f (z) the residue of f(Z) at Z = a is given by,
z = 4 is a simple pole of f(z) 1 ⎡ d n −1 ⎤
Lt ⎢ n −1 ( Z − a) n f ( Z ) ⎥
∴Re s[ f ( z )] = Lt [( z − 4) f ( z )] ( n − 1)! Z → a ⎣ dZ
z=4 z→4 ⎦
 1  Here, Z = a = 0 and n = 2;
= Lt ( z − 4). 3
z→4
 ( z − 4)( z + 1)  ∴ Res [f(Z)] = Z = 0
 1  1 ⎡ d ⎡ 2 sin Z ⎤ ⎤
= Lt   = Lt ( Z − 0)
z → 4 ( z + 1)3
  ( 2 − 1)! Z →0 ⎢⎣ dZ ⎢⎣ 2 ⎥⎥
Z ⎦⎦

1 ⎡ d ⎤
= =Lt (sin Z ) ⎥
( 4 + 1)3 Z →0 ⎢
⎣ dZ ⎦

1 =
Lt cosZ
∴Re s[ f ( z )] = Z →0
z=4 125
=
1
z = −1 is a pole of order 3 for f (z)
Hence, the correct Answer is (1).
1  Lt  d 3−1 
∴ Re s[ f ( z )] =   3−1 (( z + 1) f ( z ))  
3
5. A discrete-time signal x[n] = δ[n – 3] + 2δ [n – 5] has
z =−1 (3 − 1)!  z ⇒ − 1  dz  z–transform X(z). If Y(z) = X(–z) is the z–transform of
another signal y[n], then [2016]
1  d2  1  
=  Lt  2  z − 4    (A) y[n] = x[n] (B) y[n] = x[–n]
2  z → −1  dz    (C) y[n] = –x[n] (D) y[n] = –x[–n]
1  2 
=  z Lt   Solution:  We know the z-transform a n ⋅ x( n) ↔ X ( Z /a)
2  → −1  ( z − 4)3  
(–1)n ⋅ x(n) ↔ X(–Z)
1 2 
=  3  given x(n) = δ[n– 3] + 2δ[n– 5]
2  ( −1 − 4) 
and
2.54 | Signals and Systems

Y[Z] = X [–Z] n n
⎡1⎤ ⎡2⎤
(C) 5 ⎢ ⎥ u [ n] − 5 ⎢ ⎥ u [ n]
∴ y[n] = (–1)n ⋅ x[n] 3 ⎣ ⎦ 3 ⎣ ⎦
y[n] = (–1)n [δ [n– 3] + 2δ[n– 5]] ⎡2⎤
n
⎡1⎤
n
5 ⎢ ⎥ u [ n] − 5 ⎢ ⎥ u [ n]
(D)
∴ y[n] = – δ[n– 3]– 2δ[n– 5] 3 ⎣ ⎦ 3 ⎣ ⎦
or y[n] = – (δ[n– 3]+2δ[n– 5] Solution: 
W[n]
=
–x[n] x[n] +

Z–1
=
–x[n].
1 –5/3
Hence, the correct option is (C). + + y[n]

6. Suppose x[n] is an absolutely summable discrete-time


signal. Its z-transform is a rational function with two Z–1
poles and two zeros. The poles are at z = ±2j. Which one
of the following statements is TRUE for the signal x[n]? –2/9 5/3
 [2015]
(A) It is a finite duration signal. 2
Let W[n] = x[n] + W [n – 1] – W [n – 2]
(B) It is a causal signal 9
(C) It is a non-causal signal
By taking Z transform
(D) It is a periodic signal.
2
Solution:  x[n] is absolute summable so, ROC must in- W(z) = X(z) + Z–1 W(z) – W(z) ⋅ Z–2
clude unit circle 9
2 ⎛ 2 ⎞
W(z) ⎜1 − Z −1 + Z −2 ⎟ = X(z)
1 ⎝ 9 ⎠
W (z) 1 1
= =
X (z) ⎛ −1 2 −2 ⎞ ⎛ 1 −1 ⎞ ⎛ 2 −1 ⎞
⎜⎝1 − Z + Z ⎟⎠ ⎜⎝1 − z ⎟⎠ ⎜⎝ 1 − z ⎟⎠
9 3 3
1 5 5
Now y[n] = − W [ n − 1] + W [ n − 2]
2 3 3 
⎛ 5 5 ⎞
Y(z) = ⎜ − Z −1 + Z −2 ⎟ W ( z )
and poles are at 2. So ROC is inside the circle so x[n] is ⎝ 3 3 ⎠

non-causal signal.
Hence, the correct option is (C). 5
(
+ Z −1 Z −1 − 1 X ( z ) )
7. A realization of a stable discrete time system is shown Y(z) = 3
⎛ 1 −1 ⎞ ⎛ 2 −1 ⎞
in the figure. If the system is excited by a unit step ⎜⎝1 − Z ⎟⎠ ⎜⎝1 − Z ⎟⎠
sequence input x[n], the response y[n] is [2015] 3 3 
1
x[n] + Now unit step response of X(z) =
z–1
1 − z −1 ( )
5
+
1 –5/3
+ y[n] − Z −1
Y(z) = 3
⎛ 1 −1 ⎞ ⎛ 2 −1 ⎞
z–1 ⎜⎝1 − Z ⎟⎠ ⎜⎝1 − Z ⎟⎠
3 3 
–2/9 5/3
A B
= +
⎡ 1⎤
n
⎡ 2⎤
n ⎛ 1 −1 ⎞ ⎛ 2 −1 ⎞
(A) 4 ⎢ − ⎥ u [ n] − 5 ⎢ − ⎥ u [ n] ⎜⎝ 1 − Z ⎟⎠ ⎜⎝1 − Z ⎟⎠
3 3 
3 ⎣ ⎦ 3 ⎣ ⎦
n n A = 5
⎡ 2⎤ ⎡ 1⎤
(B) 5 ⎢ − ⎥ u [ n] − 3 ⎢ − ⎥ u [ n]
3 ⎣ ⎦ 3 ⎣ ⎦ B = –5
Chapter 6  Z-Transform  |  2.55

5 5 Poles are inside of |z|=3 using Cauchy integral ­formula


So Y[z] =
− 
1 −1 2 −1
1− z 1− z z2 −z +4j
3 3 ∫ = 2pi  lim z 2 − z + 4 j  = −p [2 j + 3]
n n z + 2j  zx → 2 j 
⎛ 1⎞ ⎛ 2⎞
y[n] = 5 ⎜ ⎟ u [ n] − 5 ⎜ ⎟ u [ n]
⎝ 3⎠ ⎝ 3⎠ Hence, the correct option is (c).

Hence, the correct option is (C).
8. For an all-pass system H (z ) =
(
z −1 − b ) 10. Let x[n] = x[-n]. Let X(z) be the z-transform of x[n]. If
, where 0.5 + j0.25 is zero of X(z), which one of the following
(
1 − az −1 ) must also be a zero of X(z). [2014]
H (e − jw ) = 1, for all w. If Re(a) ≠ 0,
1
Im(a) ≠ 0, then b equals [2014] (a) 0.5 − j0.25 (b)
(0.5 + j 0.25)
(a) a
1
(b) a∗ (c) (d) 2 + j4
(c) 1/a∗ (0.5 − j 0.25)
(d) 1/a Solution: (b)
Solution: (b) Given x(n) = x(−n),
( z −1 − b ) Using z-transform property then
For a given system, H ( z ) = x(n) →x[z]
(1 − az −1 )
The poles lie at z = 0 and the zero lie at z = 1/b. a is x(−n) →x[z−1]
complex in nature so Now, let (0.5 + j 0.25) is a zero of x(z) for x(n)
a = |a|·ei< a 1
then x ( − n ) → x ( z −1 ) =
for an all pass system, He Jw = 1 0.5 + j 0.25
⇒ for all w, if the pole lies at d then the zero must be at zero of x(z).
1/a Hence, the correct option is (b).
1 1 1 11. If x[n] = (1/3)|n| - (1/2)n u[n], then the region of conver-
∴ zero   = JLa
− or b = a ∗
b  |a|e a gence (ROC) in its Z-transform in the Z-plane will be
Hence, the correct option is (b).  [2012]
9. C is a closed path in the z-plane given by z = 3. The 1 1 1
(a) < z < 3 (b) < z <
value of the integral 3 3 2
1 1
 z2 −z +4j  (c) < z < 3 (d) < z
∫ c  z + 2j  dz is [2014] 2 3
Solution: (c)
|n| n
4p (3 − j 2)
− 4p (1 + j 2) (b)
(a) 1 1
Given signal x ( n ) =   −   u ( n )
(c) − 4p (3 + j 2) (d) 4p (1 − j 2) 3 3
can be written as
Solution: (c) n −n n
Location of poles can be shown as: 1 1 1
x ( n ) =   4( n ) +   u ( − n − 1) −   u ( n )
3 3 2
y
Using z-transform property:
n
1 1 1
 3  u ( n ) ←

1
Roc | z |>
3
  1 − z −1
x 2
(−3, 0) (3, 0)
n
−2j 1 −1
 3  u ( − n − 1) ←
→ Roc | z |< 3
  1 − 3z −1
(0, −3)
n
1 1 1
 2  u ( n ) ←

1
Roc | z |>
2
  1 − z −1
z2 −z +4j 2
So ∫
c
z + 2j
, poles = z = −2j
2.56 | Signals and Systems

Overall ROC will be intersection of these ROC i.e. n


1
1  3  u ( n ) is right sided signal, so ROC will be |z|>1/3.

< z <3  
2
n
Hence, the correct option is (c). 1
 3  u ( n ) is left sided signal, so ROC will be |z|<1/2
12. Consider the z-transform X(z) = 5z2 + 4z−1 + 3; 0 < |z| °  
. The inverse z-transform x[n] is [2010] ⇒ ROC of the function will be:
(a) 5d[n + 2] + 3d[n] + 4d[n − 1] 1 1
< z <
(b) 5d[n − 2] + 3d[n] + 4d[n + 1] 3 2
(c) 5u[n + 2] + 3u[n] + 4u[n − 1] Hence, the correct option is (a).
(d) 5u[n − 2] + 3u[n] + 4u[n + 1]
15. If the region of convergence of x1[n] + x2[n] is
Solution: (a) 1 2
< z < , then the region of convergence of x1[n] −
d [n + n0 ] ←
z
→ z n0 × ( z ) = 5z 2 + 4 z −1 + 3; 0 < z < ∞ 3 3
x2[n] includes [2006]
∴ x ( n ) = 5d ( n + 2) + 4d ( n − 1) + 3dd ( n )
1 2
(a) < z < 3 (b) < z <3
Hence, the correct option is (a). 3 3
13. Two discrete time systems with impulse response h1[n] 3 1 2
(c) < z < 3 (d) < z <
= d[n − 1] and h2[n] = d[n − 2] are connected in cas- 2 3 3
cade. The overall impulse response of the cascaded sys- Solution: (d)
tem is [2010]
(a) d[n − 1] + d[n − 2] x (n) = x1 (n) + x 2 (n)
(b) d[n − 4] 1 1
(c) d[n − 3] ROC is < z <
2 3
(d) d[n − 1] d[n − 2]
Now x 1 ( n ) = x 1 ( n ) − x 2 ( n )
Solution: (c)
→ROC will remain the same as the intersection will
h1 [n ] = d [n − 1] ←
z
→ n1 ( z ) = z −1 not change due to −x sign
h2 [n ] = d [n − 1] ←
z
→ n 2 ( z ) = z −2 1 1
→ROC is < z <
2 3
Overall impulse response in Z-domain
ROC of x 1 ( n ) + x 2 ( n ) and x 1 ( n ) − x 2 ( n ) is the same.
n ( z ) = n1 ( z )n 2 ( z ) Hence, the correct option is (d).
= z −1z −2 16. The region of convergence (ROC ) of z-transform of
= z −3 the sequence
n n
5 6
Overall impulse response in discrete-time domain  6  u (n ) −  5  u ( − n − 1) must be [2005]
   
h[n ] = d ( n − 3)
5 5
Hence, the correct option is (c). (a) z < (b) z >
6 6
1 4. The ROC of z-transform of the discrete time sequence 5 5 6
 [2009] (c) < z < (d) < z <
6 6 5
n n
1 1 Solution: (c)
x (n ) =   u (n ) −   u( − n − 1) is
3 2 Given sequence is
1 1 1 5
n
6
n

(a) < z < (b) z > x ( n ) =   u ( n ) −   u ( − n − 1)


3 2 2 6
  5
1 x (n) = x1 (n) + x 2 (n)
(c) z < (d) 2< z <3
3
ROC of x(z) = [ROC of x1(z)] ∩ [ROC of x2(z)]
Solution: (a) n
5 1
 6  u ( n ) ←

n n
1 1 ,
x ( n ) =   u ( n ) −   u ( − n − 1)   5
3 2 1 − z −1
6
Chapter 6  Z-Transform  |  2.57

1 19. The region of convergence of the z-transform of a unit


Roc |z| > ∴ L [h(t )] = L [f (t )] × L [g (t )] = step function is [2001]
S +3
(a) z >1
n
6 1 6 (b) z <1
−   u ( − n − 1) ←
→ , Roc |z| <
5 6 5 (c) (Real part of z) > 0
1 −   z −1
5 (d) (Real part of z) < 0
5 6 Solution: (a)
ROC of x(z) will be < z < h( n ) = u( n )
6 5

Hence, the correct option is (c). H ( z ) = ∑1 − z − n
17. The z-transform of a system is [2004] n =0
For RVC
z ∞
H (z ) =
z − 0.2 ∑z
n =0
−n
<∞

If the ROC is z < 0.2, then the impulse response of the 1 + z −1 + z −2 + … < ∞
system is z −1 < 1 → z > 1
(a) (0.2)nu[n] (b) (0.2)nu[−n − 1]
Hence, the correct option is (a).
(c) −(0.2) u[n] (d)
n
−(0.2)nu[−n − 1]
2 0. The z-transform F(z) of the function
Solution: (d)
f(nT) = anT is_____. [1999]
z z z
H (z ) = ,| z |< 0.2 (a) T (b)
z − 0.2 z −a z + aT
1 1 z z
H (z ) = −1
← → ⋅r <1 (c) −T (d)
1 − 0.2z 1− r z −a z + a−T
0.2 Z−1< 1 ⇒ |z| > 0.2 Solution :  (a)
But given ROC is |z| < 0.2 f ( nT ) = anT
Hence, the correct option is (d). ∞ ∞

18. A sequence x(n) with the z-transform X(z) = z4 + z2 − 2z z [f ( nT )] = ∑ f ( nT )z − n = ∑ anT z − n


n =0 n =0
+ 2 − 3z−4 is applied as an input to a linear, time-invar- n

iant system with the impulse response h(n) = 2d(n−3) 1
= ∑ (aT z −1 ) =
where n =0 1 − aT z −1
1, n = 0 z
d(n) =   [2003] =
z − aT
0, otherwise
Hence, the correct option is (a).
The output at n = 4 is
(a) −6 (b) Zero 21. The z-transform of the time function [1998]
(c) 2 (d) −4 ƒ

Solution: (b) ∑ d (n − k ) is
k =0

x ( z ) = z 4 + z +2 − 2z + 2 − 3z −4 z z
(a) (b)
h( n ) = 2d ( n − 3) z - aT z + aT
h( z ) = 2z −3 z z
(c) (d)
y ( z ) = h( z ) ⋅ x ( z ) z -a -T
z + a−T
y ( z ) = 2z −3 ( z 4 + z 2 − 2z + 2 − 3z −4 ) Solution:  (*)
Given Signal
y ( z ) = 2z + 2z −1 − 4 z −2 + 4 z −3 − 6 z −7

y ( n ) = 2[d ( n + 1) + d ( n − 1) − 2d ( n − 2) + 2d ( n − 3) − 3d ( n − 7)]
x ( n ) = ∑ S ( n − k ) = S ( n ) + S ( n − 1) + S ( n − 2) + …
k =0
At n = 4 y(4) = 0
x ( z ) = 1 + z −1 + z −2 + …
h ( n ) = −(0.2) n u ( − n − 1) 1 z
= −1
=
Hence, the correct option is (b). 1− z z −1
2.58 | Signals and Systems

Two-mark Questions 2. For the discrete-time system shown in the figure, the
poles of the system transfer function are located at
1. A continuous time filter with transfer function H(s) =  [2015]
2s + 6
2
is converted to a discrete time filter with X[n] + + Y[n]
s + 6s + 8
2 z 2 − 0.5032 z
transfer function G(z) = 2 so that the
z − 0.5032 z + k –1 5
impulse response of the continuous time filter, sam- 6 6
pled at 2 Hz, is identical at the sampling instants to the
impulse response of the discrete time filter. The value Z –1 Z –1
of k is _____. [2016]
1
Solution:  To find the Inverse Laplace of transfer (A) 2, 3 ,3 (B)
2
­function H(s) we need to calculate the values using par-
1 1 1
tial fraction formula as given below: (C) , (D)
2,
2 3 2
2 s +6
H(s) = 2
s + 6s + 8
 5 1
Y[n] =
Solution:  y [n – 1] − y [ n − 2] + x [ n]
A B 6 6
H(s) = +
( s + 2) s + 4  By taking z transform on both sides
A = H(s) ⋅ (s + 2) at s = −2 ⎡ 5 1 ⎤
Y ( z ) ⎢1 − Z −1 + Z −2 ⎥ = X (z)
2 ⎣ 6 6 ⎦
A = = 1
2
Y (z) 1 1
B = H(s) (s + 4) at s = −4 = ⇒=
X ( z ) ⎛ 5 −1 1 −2 ⎞ ⎛ 1 −1 ⎞ ⎛ 1 −1 ⎞
−2 ⎜⎝1 − z + z ⎟⎠ ⎜⎝1 − z ⎟⎠ ⎜⎝1 − z ⎟⎠
B = = 1 6 6 2 3
−2
1 1
Now we have So poles are at Z = ,
1 1 2 3
H(s) = + Hence, the correct option is (C).
s+2 s+4
3. Two causal discrete-time signals x[n] and y[n] are
h(t) = ILT {H(S)} = (e−2t + e−4t)u(t)
related as y[n] = ∑ m = 0 x [ m] . If the z-transform of y[n]
n

1 1 2
We know that, TS = = is , the value of x[2] is _____. [2015]
fs 2 z ( z − 1)
2

Substituting t = n ⋅ TS we get Solution: 


As we know the accumulation property
n
−2 nTs −4 nTs
h(nTS) = (e + e ) ⋅ u(nTS) If y[n] = ∑ x [ m]
=
e−n ⋅ u(n) + e−2n ⋅ u(n)
m=0 
X (z)
H(Z) = ZT{h(nTS)} then Y(z) = 
Z Z
(1 − z ) −1

H(Z) = +
Z −e −1
Z − e −2 2 X (z) Z X (z)
 ⇒ = =
=
Z
+
Z
Z ( Z − 1)
2
(1 − Z )
−1
( Z − 1)

Z − 0.367 Z − 0.135  2 Z −3
So X(z) =

2
Z − 0.135Z + Z − 0.367 Z
=
2 (1 − Z ) −1

2
Z − 0.5032 Z + 0.049  x[n] = 2 u[n – 3]

2 Z − 0.5032 Z
2
= So at n = 2
Z 2 − 0.5032 Z + 0.049
 x[2] = 0
∴ K = 0.049
Hence, the correct Answer is (0).
Hence, the correct Answer is (0.049).
Chapter 6  Z-Transform  |  2.59

n n 6. Let H1(z) = (1− pz−1)−1, H2(z) = (1− qz−1)−1, H(z) =


4. Let X [n] =  -
1  1
 u(n) -  -  u(- n -1) . The Region H1(z) + rH2(z). The quantities p, q, r are real numbers.
 9  3 1 1
of Convergence (ROC) of the z-transform of x[n] Consider p = , q = , r < 1. If the zero of H(z) lies
 [2014] 2 4
on the unit circle, then r = ______. [2014]
1 1
(a) is z > (b) is z > Solution: 0.5
9 3
1
1 1 Given H 1 ( z ) =
(c) is > z > (d) does not exit 1 − pz −1
3 9
1
Solution: (c) and H 2 ( z ) =
1 − gz −1
Given also given p = ½ and g = −1/4
n n
 −1   −1  1 1
 9  u ( n )  3  u ( − n − 1) So H ( z ) = H 1 ( z ) + rH 2 ( z ) = +
x (n) =   −  1 − pz −1 1 − gz −1
x1 (n) x 2 (n) Solving
Here x1 (n) is right sided signal, so  1 −1   1 −1 
1 1 1 1 + 4 z  + r 1 − 2 z 
x1 (n) = Roc | z |≥ or | z |> H (z ) =    
 −1  −1 9 9  1 −1   1 −1 
1−   z
 9  1 − 2 z  1 + 4 z 
  
1 1 Zero of
and x 2 ( n ) = Roc | z | ≤
 −1  −1 3 H ( z ) ⇒ (1 + r ) − ( g + pr )z −1 = 0
1−   z
 3  or,
1 1 g + pr
Roc of overall system will be <| z | < z =
9 3 1+ r
Hence, the correct option is (c).
5. The input-output relationship of a casual stable LTI Since the zero of H(z) lies on the unit circle; therefore
system is given as y[n] = ay[n − 1] + bx[n] |z| = 1 or z = ± 1, taking z = 1 we get
If the impulse response h[n] of this system satisfies the g + pr
∞ z = = 1.
condition ∑ h[n] = 2, the relationship between a and 1+ r
n =0
Solving the equation with p = 1/2 and g = −1/4 we will
b is [2014]
get r = 2.5,
(a) a = 1− b/2 (b) a = 1 + b/2
For |r| < 1 or −1 < r < 1
(c) a = 2b (d) a = −2b
Taking z = −1, in equation (iii) we will get r = 0.5.
Solution: (a)
7. The z-transform of the sequence x[n] is given by
Given relationship is
x ( n ) = az ( n − 1) + bx ( n ) 1
X(z) = , with the region of convergence z
Taking z-transform will give (1 − 2z )
−1 2

2. Then, x[2] is _________. [2014]


y ( z ) = az −1 y ( z ) + bx ( z )
Solution: 12
y (z ) b
Or, = H (z ) = 1
x (z ) 1 − az −1 Given x ( z ) = , | z |> 2 (1)
(1 − 2z −1 ) 2
Taking inverse z-transform
⇒ x ( z ) = (1 − 2z −1 ) −2
n( n ) = b ⋅ a u( n )
n
Expanding equation (1) using binomial expression
∞ ∞
also, given ∑ h( n ) = 2 ⇒ ∑ b ⋅ a
n =0 n =0
n
u( n ) = 2 x ( z ) = 1 + 4 z −1 + 12z −2 + …
Taking inverse z-transform
b b
so = 2 ⇒ b = 2 − 2a OR a = 1 − x ( n ) = {1, 4,12,…
1− a 2
Hence, the correct option is (a). or x [2]=12
2.60 | Signals and Systems

8. Two systems H1(z) and H2(z) are connected in c­ ascade 2. A discrete-time LTI system is stable if the ROC of
as shown below. The overall output y(n) is the same as its system function includes the unit circle, |z| = 1
the input x(n) with a one unit delay. The transfer func-
tion of the second system H2(z) is [2011]  1 −1   1 −1 
1 − 4 z  + 1 − 2 z 
(1 − 0.4 z −1 ) H (z )    
x (n) → H1 (z ) = → H 2 (z ) → y (n)  1 −1   1 −1 
(1 − 0.6 z −1 ) 1 − 4 z  + 1 − 2 z 
   
(a)
(
1 − 0.6z −1
(b) −1
)
z −1 (1 − 0.6z −1 ) 1 1
⇒ H (z ) = +
( )
z −1 1- 0.4z −1 (1 − 0.4z ) 1 1
1 − z −1 1 − z −1
4 2
z (1 − 0.4z )
−1
(1 − 0.4z )
−1 −1

(c) (d) 1
(1 − 0.6z ) z (1 − 0.6z )
−1 −1 −1 For ROC: |z| > , the system is stable and causal
2
Solution: (b) 1
For ROC: |z|  < , ROC does not include unit ­circle.
y [n ] = x [n −1] 4
So system is not stable.
Taking Z-transform both sides
1
y ( z ) = z −1 × ( z ) For ROC: , ROC does not include unit circle. So sys-
4
y (z ) tem in not stable
= z −1
x (z ) 1
Also ROC is not the exterior of z = . So it is not
For cascaded system causal. 2
n ( z ) − n1 ( z ) ⋅ n 2 ( z ) Hence, the correct option is (c).
−1 −1
(1 − 0.4 z ) (1 − 0.6zz ) 1 0. A system with transfer function H(z) has impulse h(n)
z −1 = n2 (z ) ∴ n2 (z ) = defined as h(2) = 1, h(3) = −1 and h(k) = 0 ­otherwise.
(1 − 0.6 z −1 ) (1 − 0.4 z −1 )
Consider the following statements [2009]
Hence, the correct option is (b). S1: H(z) is a low-pass filter
9. The transfer function of a discrete time LTI system is S2: H(z) is an FIR filter.
given by [2010] Which of the following is correct?
3 (a) Only S2 is true
2 − z −1
4 (b) Both S1 and S2 are false
H (z ) =
3 −1 1 −2 (c) Both S1 and S2 are true, and S2 is a reason
1− z + z
4 8 for S1
(d) Both S1 and S2 are true, but S2 is not a reason for S1
Consider the following statements:
S1: The system is stable and causal for Solution: (a)
1 h(2) = 1
ROC: z > h(3) = −1
2
S2: The system is stable but not causal for h(k) = otherwise
1 h(k)
ROC: z <
4
S3: The system is neither stable nor causal 1
1 1 3
for ROC: < z < k
4 2 2
Which one of the following statements is valid? −1
(a) Both S1 and S2 are true
(b) Both S2 and S3 true
(c) Both S1 and S3 are true ⇒It is an FIR filter and not a low pass filter.
(d) S1, S2 and S3 are all true Hence, the correct option is (a).
Solution: (c) Statement for Linked Answer Questions 8 and 9.
1. A discrete time LII system is causal if the ROC of In the following network, the switch is closed at
its system function is the exterior of a circle, in- t = 0- and the sampling starts from t = 0. The s­ ampling
cluding infinity. frequency is 10 Hz.
Chapter 6  Z-Transform  |  2.61

10 µF ∴ samples are: x ( n ) = 5e −0.05 n


S x (z ) Hence, the correct option is (b).
x (n)
1 3. The z-transform X[z] of sequence x[n] is given by
+ Sampler
− 200 kΩ (fs = 10 Hz) z-transform 0.5
5V X[z ] = . It is given that the region of conver-
1 − 2z −1
gence of X[z] includes the unit circle. The value of x[0]
is [2007]
(a) −0.5 (b) 0
(c) 0.25 (d) 0.5
11. The expression and the region of convergence of the Solution:  (0)
z-transform of the sampled signal are [2008]
0.5
5z x (z ) =
(a) 5 , z < e −5 1 − 2z −1
z −e
ROC includes unit circle left handed system
5z
(b) −0.05 , z < e −0.05
z −e x ( n ) = −(0.5)2− n u ( − n − 1)
5z x (o ) = 0
(c) −0.05 , z > e −0.05
z −e 14. A causal LTI system is described by the difference
5z equation
(d) −5 , z > e −5
z −e 2y[n] = ay[n − 2] − 2x[n] + bx[n−1]. [2004]
Solution: (c) The system is stable only if
∞ (a) a = 2, b < 2
X ( z ) = ∑ 5e −0.05 n z − n (b) a > 2, b > 2
n =0
∞ (c) a < 2, ∞ any value of b
( )
−n
= 5∑ e −0.05 z −1 (d) b < 2, any value of a
n =0

−0.05 −1 Solution: (c)
For e z < 1
⇒ |z| >e−0.05 2 y ( n ) = ay [n − 2] − 2x [n ] + bx [n − 1]

5z Taking z-transform
x(z) =
z − e −0.05
2 y ( z ) = ay ( z )z −2 − 2x ( z ) + bx ( z )z −1
Pole should be inside unit circle
y ( z )  2 − az −2  = z ( z )[bz −1 − 2]
a
<1 y ( z ) bz −1 − 2
2 = =T ⋅F
x ( z ) 2 − az −2
| a |< 2
Hence, the correct option is (c). For stable B can have only value
12. The sample x(n)(n = 0,1,2, ...) is given by [2008]
(a) 5(1−e−0.05n) a
2 − az −2 = 0 ⇒ z =
(b) 5e−0.05n 2
(c) 5(1−e−5n)
(d) 5e−5n Hence, the correct option is (c).
Solution: (b) 1 5. If the impulse response of a discrete-time system is
Using the voltage divider rule h[n] = −5nu[−n − 1], then the system function H(z) is
equal to [2002]
 
 5 −z
200 × 103 (a) and the system is stable
V R (S ) =   z −5
 200 × 103 + 1  S z
 10 × 10 −6 S  (b) and the system is stable
z −5
10
= = 5e −0.5tt −z
2S + 1 (c) and the system is unstable
z −5
2.62 | Signals and Systems

z 17. A linear discrete-time system has the characteristics


(d) and the system is unstable equation z3 − 0.81z = 0. The system [1992]
z −5
(a) is stable
Solution: (b) (b) is marginally stable
(c) is unstable
h ( n ) = −5nu [− n − 1] (d)  stability cannot be assessed from the given
z ­information
u ( n ) ←

z −1 Solution: (a)
z A system is stable when all the poles lie inside the unit
→( z −1 ) ×
u ( n − 1) ← circle in z-plane
z −1
1 z 3 − 0.81z = 0 ⇒ z ( z 2 − 0.81) = 0
u( − n − 1) ←

1− z z ( z − 0.9)( z + 0.9) = 0
1 5z z = 0, − 0.9, 0.9
5n u ( − n − 1) ←
→ ×
5 5−z
All the poles are inside unit circle hence stable.
z Hence, the correct option is (a).
−5n u ( − n − 1) ←

z −5 1 8. The z-transform of the following real exponential
The system is stable because ROC includes unit circle. sequence [1990]
Hence, the correct option is (b). X(nT) = an, nT ≥ 0
16. The z-transform of a signal is given by [1999] = 0, nT < 0, a > 0

C (z ) =
−1
1 z 1− z (
−4
)
. Its final value is
1
(a) −1 ; z > 1 (b)
1− z
1
1 − az −1
; z >a
(
4 1 − z −1 2 ) 1
(c) 1 for all z (d) ; z <a
(a) 1/4 (b) zero 1 − az −1
Solution: (b)
(c) 1.0 (d) infinity
Solution: (c) x ( nT ) = an , nT ≥ 0
= 0, nT < 0, a > 0
1 z −1 (1 − z −4 )
C (z ) =
nT )) = aa >
n
4 (1 − z −1 ) 2 xx (( nT = aan uu (( nn ),
), > 00
Final value theorem xx (( nn )) = a n
n u ( m ),
= a u ( m ), aa >> 00
zz
lim x ( n ) = lim(1 − z −1 )x ( z )
n →∞ z →1
aann uu (( mm )) == z − a ⋅⋅ || zz ||>
>|| aa ||
z −a
1 z −1 (1 − z −4 ) 11
= lim (1 − z −1 ) =
xx (( zz )) = ; | z |> a
z →1 4 (1 − z −1 ) 2 − aazz −−11 ; | z |> a
11 −
1 z −1 (1 − z −4 ) 0 Hence, the correct option is (b).
= lim −1
→ 

z →1 4 (1 − z ) 0 19. Consider the system shown in the figure below. The
transfer function Y(z)/X(z) of the system is
For apply L-hospital rule
x(k) Σ Σ y (k )
 1 
1−
lim x ( n ) = lim
1 1  z 4 
= lim
1 z −1
4
( )
z −1
n →∞ z →1 4 z  1  z →1 4 z 4 ( z − 1)
1
 z−
 
−b a
1 4z 3
lim x ( n ) = lim
n →∞ z →1 4 5z 4 − 4 z 3  [1988]
1
= ×4 =1 1 + bz −1
1 + az −1 (b)
4 (a)
1 + bz −1 1 + az −1
Final value = 1 1 + az −1 1 − bz −1
Hence, the correct option is (c). (c) −1 (d)
1 − bz 1 + az −1
Chapter 6  Z-Transform  |  2.63

Solution: (a) Solution:
y1(z) y1(z) Given, x(n) = 1 n = 0
+
x(k) Σ Σ y(k) Σ =0n≠0
+ So,  x(n) = f(n)
z −1 z −1
f (n) ←
z .T
→1
−b a −b
1
So, y2 ( z ) = H ( z ).x( z ) = H ( z ).1. = H ( z ) =
( z + 1) 2
y 1 (z ) 1
= z
x ( z ) (1 + bz −1 )  (1)
u (n) ←
Z .T

z −1
y (z )
Now, 2d  z  z
y 1 (z ) nu (n) ←
Z .T
→−  =
dz  2 − 1 ( z − 1) 2
y1(k) +
Σ y(k)
−z
+ ( −1) n nu (m) ←
Z .T

( z + 1) 2
z −1
−1
( −1) n −1 (n − 1)u (n − 1) →
a (2 + 1) 2

y2(n) = (−1)n (n − 1)u(n − 1)


y ( z ) = y 1 ( z ) + az −1 y 1 ( z )
y. = (−1)n (n − 1) ∀n ≥ 1
y (z )  (2) =0n≤0
= 1 + az −1
y 1 (z )
2. In the linear time-invariant system shown in figure,
Now blocks labelled D represent unit delay elements. Find
the expression for y(n), and also the transfer function
y (z ) y (z ) y 1 (z ) 1
= × = (1 + az −1 ) × Y (z )
x (z ) y 1 (z ) x (z ) (1 + bz −1 ) in the z- domain [1996]
X (z )
y ( z ) 1 + az −1 1 1 1
T.F = = x(n) y (n)
x ( z ) 1 + bz −1 y1(n)
Hence, the correct option is (a). 1 1
D D
4 −2

Five-marks Questions
D −3 1
1. In figure, a linear time invariant discrete system is 5
shown. Blocks labelled D represent unit delay ele-
ments. For n < 0, you may assume that x(n), y1(n), y2(n) D D
are all zero.
Solution:
+ y1(n)
x(n) Σ D D y2(n) + y1(n)
− x(n) Σ D D y2(n)


2

2
1
1
(a) Find the expression for y1(n) and y2(n) in terms of
x(n). (a) y1(n) = x(n − 1) − 2y1 (n − 1) − y1 (n − 2)
(b) Find the transfer function Y2(z)/X(z) in the y2(n) = y1(n − 1) = x(n − 2) − 2y1 (n − 2) − y2 (n − 1)
z- domain. y3(n) = y(n − 2) − 2y2 (n − 1) − y2 (n − 2)
(c) If x(n) = 1 at n = 0 = 0 otherwise
(b) y2(n) + 2y2(n − 1) + y2(n − 2) = x(n − 2)
Find y2(n). [1997] taking Z-transform of both sides.
2.64 | Signals and Systems

y2(z) [1 + 2z−1 + z−2] = z−2 × (2) y1(n) = y(n) + 4x(n − 1) + 5x(n − 3)


y ( 2) z −2 1 y(n) = y1(n) − 2y(n − 1) − 3y(n − 2)
So, H ( z ) = 2 = =
x(2) 1 + 2 z −1 + z −2 ( z + 1) 2 y(n + 2y)(n − 1) ≠ 3y(n − 2) = y1(n)
= x(n) − 4x(n − 1) + 5x(x − 3)
3. The output of a system is given in difference e­ quation taking Z-transform on both sides
form as
y(k) = ay(k − 1) + x(k) y ( z )[1 + 2 z −1 + 3 z −2 ] = x( z )[1 + 4 z −1 + 5 z −3 ]
where x(k) is the input. If x(k) = 0 for k ≠ 0,
y ( z ) 33 z 3 + 4 z 2 + 5
x(0) = 1, and y(0) = 0, find y(k) for all k. So, H ( z ) = =
Determine the range of ‘a’ for which y(k) is bounded. x( z ) 2( z 2 + 2 z + 3)
 [1988] y(−2) = −a−2
Solution: y(k) = −a−k for k ≤ −1
Given, x(k) = 0 k ≠ 0 so x(k) is a y(k) = 0 for k > −1
x(k) = 1 k = 0,
 1 1 1 1 
discrete, impulse, x(k) = f(k) y (k ) = ..., − 4 , − 3 , − 2 , − , 0, 0, 0,...
given that, y(0) = 0  a a a a ↑ 
y(k) = ay(k − 1) + x(k)
for a = 1
y(k) = ay(k − 1) + f(k)
So,
k = 1 → y(1) = ay(0) + f (1) = 0
{
y (k ) = ....., −1, −1, −1, 0, 0, 0...}

}
k = 2 → y(2) = ay(1) + f (2) − 0 for a > 1, let a = 2,
k = 0 → y(0) = ay(−1) + f (0) − ay(−1) + 1 = 0
 1 1 1 
y (k ) = ..., , − , − , 0, 0, 0...
−1  8 4 2 ↑ 
y (1) = = − a −1
a
1
k = −1 = y ( −1) = a −1 for a < 1, let a =
2
= ay ( −2) + f ( −1) = ay ( −2)
y(−2) = −a2 {
y (k ) = ...., −8, −4, −2, 0, 0, 0....

}
1 1 1 1 So for y(k) to be bounded
x(n) y (n)
|a| ≥ 1
1 1
D D
4 −2

D 5 −3 1

D D
Chapter 7
DTFT and DFT
(c) In-place computation requires storage of only 2N
One-mark Questions node data.
(d) Computation of a butterfly requires only one com-
1. Consider two real sequences with time-origin marked plex multiplication.
by the bold value, Solution: (d)
x1[n] = {1, 2, 3, 0}, x2[n] = {1, 3, 2, 1} For an N point FFT algorithm with N = 2m, computation
of a butterfly requires only one complex multiplication
Let X1(k) and X2(k) be 4-points DFTs of x1[n] and x2[n], and two complex additions.
respectively. Hence, the correct option is (d).
Another sequence x3[n] is derived by taking 4-point
n
inverse DFT of X3(k) = X1(k) X2(k). 1
3. Let x (n ) =   u (n ), y (n ) = x 2 (n ) and Y (e ) be
jw

The value of x3[2] is _______. [2015] 2


 
the Fourier transform of y(n). Then Y (e ) is [2005]
j0
Solution:  By applying the property
Multiplication in frequency domain = Convolution in 1
(a) (b) 2
time domain. 4
4
x1[n] ⊗ x2[n] = X1(k) ⋅ X2(k) (c) 4 (d)
3
Solution: (d)
n
1
1 2 3 0 Given signal x ( n ) =   u ( n )
1 1 2 3 0 2
2n
1
and x ( n ) = x ( n ) =   (u ( n ))
2 2
3 3 6 9 0
2
n
2 2 4 6 0 1
⇒ y ( n ) =   u( n )
4
1 1 2 3 0 Taking z-transform
1
y (z )
x3[n] = { 1, 5, 11, 14, 8, 3, 0} 1 -1
1 z
4
x3[2] = 11
1
Hence, the correct Answer is (10.9 to 11.1). Put z = e jw ⇒ y (e jw ) =
1 − Jw
2. For an N-point FFT algorithm with N = 2m, 1 e
4
which one of the following statement is TRUE? 1 4
At w = 0, y (e ) = = A ns
Jo
 [2010]
1 3
(a) It is not possible to construct a signal flow graph 1−
4
with both input and output in normal order.
(b) The number of butterflies in the mth state is N/m. Hence, the correct option is (d).
2.66 | Signals and Systems

Two-marks Questions ⎛ j2 ⎞
W6  = exp ⎜ – ⎟ In the figure, what should be the
1. It is desired to find a three-tap causal filter which gives ⎝ 6⎠
zero signal as an output to an input of the form value of the coefficients a1, a2, a3 in terms of W6 so the
X[1] is obtained correctly? [2019]
jπ n ⎞ ⎛ jπ n ⎞
x[n] = c1 exp ⎛⎜ – ⎟ + c2 exp ⎜⎝ ⎟ x(0) + x(3)
⎝ 2 ⎠ 2 ⎠ x[0] X[0]
x(0) – x(3) a1
Where c1 and c2 are arbitrary real numbers. The desired x[3] X[1]
three-tap filter is given by and –1
x(2) + x(4) a2
h[0] = 1, h[1] = a, h[2] = b
x[1] X[2]
h[n] = 0 for n < 0 or n > 2. x(2) – x(4)
x[4] a3 X[3]
What are the values of the filter taps a and b if the out-
put is y[n] = 0 for all n, when x[n] is as given above? x(2) + x(5)
x[2] X[4]
 [2019]
X[n] n=0 y[n] = 0 x[5] X[5]
–1 x(2) – x(5)
h[n] = {1, a, b}

(A) a = 0, b = –1 (A) a1 = 1, a2 = W62 , a3 = W6


(B) a = 1, b = 1
(B) a1 = –1, a2 = W62 , a3 = W6
(C) a = –1, b = 1
(D) a = 0, b = 1 (C) a1 = –1, a2 =W6 , a3 = W62
Solution: (D) a1 = 1, a2 =W6 , a3 = W62
 J Πn 
−
 2  Solution:  Six point DIT - DFT
X [ n] = C1e + C2 e jΠn / 2
5
n [0 ] = 1 X ( K ) = ∑ X [ n]W6Kn
n [1] = a n=0

X (1) = X ( 0) + X (1)W61 + X ( 2)W62 + X (3)W63


[ ]
n 2 =b

n=0 + X ( 4)W64 + X (5)W65


X(n) y(n)
h[n] = {1, a, b}
N
K+
Applying fourrier transform Wn 2 = −WnK [ property twiddle factor ]

H e ( ) = 1 + ae
jw − jw
+ be − j 2w
∴W63 = −W6O

Π Π W64 = −W61
−j n j n
For input X[n] = C1e 2 + C2 e 2
W65 = −W62

output y[n] = 0 if
From the graph we have
 Π
H W = −  = 0
 X (1) =  X ( 0) − X (3)  a1 +  X (1) − X ( 4)  a2
2

Π +  X ( 2) − X (5)  a3
−j
1 + ae 2 + be − iΠ = 0
on companion we get
1 − aj − b = 0
a1 = 1
1 − b − aj = 0
a2 = W61
1− b = 0 a = 0 a = W62
3
b = 1 a = 0 Hence, the correct option is (D).
Hence, the correct option is (D). 3. Let h[n] be a length-7 discrete-time finite impulse
2. Consider a six-point decimation-in-time. Fast Fourier response filter, given by
Transform (FFT) alogorithm, for which the signal-flow h[0] = 4 h[1] = 3, h[2] = 2 h[3] = 1
graph corresponding to X[1] is shown in the figure. Let h[–1] = –3, h[–2] = –2, h[–3] = –1
Chapter 7  DTFT and DFT  |  2.67

and h[n] is zero for n ≥ 4. A length-3 finite impulse Using the following identities
response approximation g[n] of h[n] has to be obtained DFT
x[n] ←⎯⎯ → X [k ]
such that
⎡ n ⎤ DFT
π x1 [n] = x ⎢ ⎥ ←⎯⎯ → X (3k ] = X 1 [k ]
( ) – G (e )
2
⎣s⎦

jω jω
E(h, g) = H e dω
−π X 1 [k ] = [12, −2 j , 0, 2 j ,12, −2 j , 0, 2 j ,12, −2 j , 0, 2 j ]

is minimized, where H(e jw) and G(e jw) are the discrete- X 1 [8] ⎛ 12 ⎞


= ⎜ ⎟ = 6.
time. Fourier transforms of h[n] and g[n], respectively. X 1 [11] ⎝ 2 j⎠
For the filter that minimizes. E(h, g), the value of 10g[–
6. A continuous time speech signal xa(t) is sampled at a
1] + g[1]. Rounded off to 2 decimal places, is _____
rate of 8 kHz and the samples are subsequently grouped
 [2019]
in blocks, each of size N. The DFT of each block is to
Solution: h [ n] = [ −1, −2, −3, 4, 3, 2,1] be computed in real time using the radix – 2 decimation
To minimize E(h, g) make a rectangular window of in frequency FFT algorithm. If the processor performs
h[n] = g[n] all operations sequentially, and takes 20 µs for comput-
On applying parseval’s theorem for DTFT. ing each complex multiplication (including multiplica-
tions by 1 and –1) and the time required for addition/
We get, subtraction is negligible, then the maximum value of N
= 10 g[–1] + g[1] is [2016]
= 10(–3) + 3 = 27 Solution:  Frequency Fm = 8KHz
Hence, the correct answer is 27. Size of block = N
4. Let X[k] = k + 1, 0 ≤ K ≤ 7 be 8-point DFT of a sequence All the operations are performed sequentially and tcomp
N −1
x[n], where X [k] = ∑ n =0 x[n]e − j 2π nk / N . = 20 µs
Each block is computed using radix 2 DIF FFT algo-
The value (correct to two decimal places) of
rithm, therefore
∑ n=0 x[2n] is _______.
3
[2018]
N = 212= 4096
X(K) = {1, 2, 3, 4, 5, 6, 7, 8}
Solution: 
Hence, the correct Answer is (4096).
3
∑ x[2n] = x[0] + x[2] + x[4] + x[6]  pn 
7. Consider a discrete time periodic signal x [n ] = sin  
n=0
 5 
=
4.5 –0.5 –0.5j – 0.5 –0.5 +0.5j . Let ak be the complex Fourier series coefficients of

=
4.5 – 1.5 = 3 x[n]. The coefficients {ak} are non-zero when k = Bm
± 1, where m is any integer. The value of B is ______.
Hence, the correct answer is 2.9 to 3.1.  [2014]
5. The Discrete Fourier Transform (DFT) of the 4 point Solution: 10
sequence Discrete time Fourier series
x[n] = {x[0], x[2], x[3]} = {3, 2, 3, 4} is n −1  2p 
jk  n
x[k] = {x[0], x[1], x[2], x[3]} = {12, 2j, 0, –2j} x ( n )∑ ake  n 

k =0
2À 2p
If x1[k] is the DFT of the 12 point sequence x1[n] = −J j n
= a−1e n
+ a0 + a1e n (1)
x [8]
{3, 0, 0, 2, 0, 0, 3, 0, 0, 4, 0, 0} the value of 1
x1 [11] 1 − J5 n 1 J p5 n
x (n) = e + e
is ____ [2016] 2j 2j
Solution: 
 pn 
x[n] = {3, 2, 3, 4} Also x ( n ) = sin  
 5 
x1[n] = {3, 0, 0, 2, 0, 0, 3, 0, 0, 4, 0, 0}  2p p 
Where N = 10,  = ⇒ N = 10 
x1[n] = X[n/3] N 5 
2.68 | Signals and Systems

1 −1 1 1 1 1 
By comparing, a1 = and a9 = a − 1 = 1 − j −1 j 
2J 2j
as, the TFS Coeff. ak is also periodic with N = 10
[p q r s] = 
1 −1 1 −1 

a1 = a11 = a21 …  
1 j −1 − j 
a1 = a9 = a19 …
Given that ak is non-zero for k = Bm ±1 = [( p + q + r + s) ( p − jq − pr + s)
( p − q + r − s) p + jq − r − js]
Bm ± 1 = 1 9…
−1 11… =  l 2 b2 g 2
d 2 
From (1) and (2)
For m = 0 for m =1
To satisfy this condition, B must be ‘10’ where ‘m’ is p + q + r + s = ( a 2 + c 2 + 2bd ) + ( 2ab + 2cd )
any integer. + (b 2 + d 2 + 2ac) + ( 2ad + 2bc)
8. The DFT of a vector [a b c d ] is the vector = (a + b + c + d )2 = a2
[a b g d ]. Consider the product.
Hence, the correct option is (a).
a b c d 9. The first six points of the 8-point DFT of a real valued
d a b c  sequence are 5, 1 −j3, 0, 3 −j4, and 3 + j4. The last two
[ p q r s ] = [a b c d ]  c d a b points of the DFT are respectively. [2011]
  (a) 0, 1 −j3 (b) 0, 1 + j3
b c d a
(c) 1 + j3, 5 (d) 1 −j3, 5
The DFT of the vector [ p q r s] is a scaled versions of Solution: (b)
 [2013] Given x(n) → real
x(k) → conjugate symmetric
a 2 b
(a) 2
g 2 d 2 
⇒ x [k ] = x ∗ [n − k ]
 a b g d
(b) n=8
 
[a + b b + d d + g g + a ]
(c) ⇒ x [k ] = x ∗ [8 ⋅ k ]
x [6] = x ∗ [8 − 6] = x ∗ [2] = 0
[a b ‡ d ]
(d)
x [7] = x ∗ [8 − 7] = x ∗ [1] = 1 + j 3
Solution: (a) Hence, the correct option is (b).
Given DFT of vector [a, b, c, d ] is [a, b, g, d] 1 0. The four-point discrete Fourier Transform (DFT) of a
i.e. discrete time sequence {1, 0, 2, 3} is [2009]
(a) [0, −2 + 2j, 2, −2 − 2j]
 l  1 1 1 1   a  a +b +c +d 
g  1 − j −1 j  b  a (b) [2, 2 + 2j, 6, 2 − 2j]
 =   =  − jb −c + jd  (c) [6, 1 − 3j, 2, 1 + 3j]
 b  1 −1 1 −1  c  a −b +c −d  (d) [6, −1 + 3j, 0, −1 − 3j]
      
d  1 j −1 − j  d  a + jb −c − jd  Solution: (d)
Given q Point DFT of sequence {1, 0, 2, 3} is given by
a b c d
d 1 1 1 1  1   1 + 2 + 3   6 
a b c  1 − j −1 j  0  1 − 2 + 3 j   −1 + 3 j 
[ p q r s ] [
= a b c d ]  c d a b
(1)    =  = 
  1 −1 1 −1  2   1 + 2 − 3   0 
b c d a       
1 j −1 − j   3  1 − 2 − 3 j   −1 − 3 j 
a2 + bd + c 2bd ab + ab + cd + cd 2ac + b 2 + d 2 2ad + 2bc 
(2) Hence, the correct option is (d).
DFT of pqrs is given as: 1 1. {x(n)} is a real-valued periodic sequence with a
period N. x(n) and X(k) form N-point Discrete Fourier
Transform (DFT) pairs. The DFT Y(k) of the sequence
 [2008]
Chapter 7  DTFT and DFT  |  2.69

1 N −1 (b)
y (n) =
N
∑ x (r ) x (n + r ) is
r =0
2

(a) |X(K)|2
1 1
1 N −1
(b) ∑ X (r )X i ( k + r ) 1/2 1/2
N r =0
1 N −1
(c) ∑ X (r )X ( k + r ) −3 −1 1 3 5 n
N r =0
(d) 0 (c)
2
Solution: (a)
DFT of y(n) = |x(k)|2
Hence, the correct option is (a). 1 1

12. A five-point sequence x[n] is given as x[−3] = 1, x[−2] 1/2 1/2


= 1, x[−1] = 0, x[0] = 5, x[1] = 1.
( )
Let X e jw denote the discrete-time Fourier transform
−6 −4 −2 0 2 n
p

∫ X (e ) d w (d)
jw
of x[n]. The value of is [2007]
2
-p

(a) 5 (b) 10 p
(c) 16 p (d) 5 + j 10 p 1 1

Solution: (b) 1/2 1/2


Given X (ejw) = e3jw + e2jw + 0 + 5 + e–jw
p
−5 −3 −1 1 3 n
∵ ∫ e qjw dw = 0 if a≠0
−p Solution: (a)
n 
p
e 3 jw e 2 jw e − jw
1
Given y ( n ) = x  − 1 for n even
⇒ ∫ X (e )dw = 0
jw
+ + 5w + 2 
3j 2j −j
−p −p =0 n odd
= 5p + 5p = 10p So, n = 0, y(0) = x(–1) = 1
Hence, the correct option is (b). n = 2, y(2) = x(0) = 2
Statement of Linked answer Question 7 and 8 n = 4, y(4) = x(1) = 1
A sequence x(n) has non-zero values as shown in the n = 6, y(6) = x(2) = ½
figure Hence, the correct option is (a).
2 x(n)
14. The Fourier transform of y(2n) will be [2005]
− j 2w
e [cos 4w + 2 cos 2w + 2]
(a)
1 1 [cos 2w + 2 cos w + 2]
(b)
1/2 1/2 e - jw [cos 2w + 2cosw + 2]
(c)
jw

−2 −1 0 1 2 n
(d) e 2 [cos 2w + 2 cos w + 2]
Solution: (c)
 n  Given y(2n) = x(n– 1)
13. The sequence y ( n ) = x  − 1 for n even will be = 0,
 2 
for n odd [2005] From graph f ( n ) = y ( 2n )

(a) 1
2
=
f ( n + 1) + f ( n ) + 2f ( n − 1)
2
1
1 1
+ f ( n − 2) + f ( n − 3)
Taking 2-transform
2
1/2 1/2
1 1
F(z ) = z + 1 + 2z −1 + z −2 + z −3
2 2
−2 0 2 4 6 n
2.70 | Signals and Systems

Put z = ejw n0w 0 + 2w k for any arbitrary integer k.


(c)
1 1 - n0w0f
(d)
F (e jw ) = e jw + 1 + 2e jw + e −2 jw + e −3 jw
2 2 Solution: (b)
 e 2 jw
+ e −2 jw

= e − jw  + e jw + e − jw + 2  y( n) = A × ( n − no ) = A sin[w o ( n − no ) + f ]
 2 
dq (w )
Given, − = no ( = t g )
f  (x) = e–Jw [cos2w + 2cosw + 2] dw
Hence, the correct option is (c). ⇒q(w) = no∫dw = –nowo + k
15. A signal x( n ) = sin(w 0 n + f) is the input to a linear So, to avoid phase change k should be an integral mul-
time-invariant system having a frequency response tiple of 2p
∴q(w) = –nowo + 2pk.
( )
H e jw . If the output of the system is Ax(n - n0), then
Hence, the correct option is (b).
the most general form of ∠H(e jw ) will be [2005]
(a) − n0w 0 + b for any arbitrary real b
(b) − n0w 0 + 2pk for any arbitrary integer k.
Chapter 8
Sampling
As we know that Nyquist sampling rate = 2 fm where fm
One-mark Questions is the maximum frequency of the input signal and for
the given signal x(t), fm = 8 Hz.
1. A continuous time sinusoid of frequency 33 Hz is mul-
Apply time multiplication and shifting property. It is
tiplied with a periodic dirac impulse train of frequency
a periodic signal so shifting does not affect the overall
46 Hz. The resulting signal is passed through an ideal
time period.
analog low pass filter with a cutoff frequency of 23 Hz.
The fundamental frequency (in Hz) of the output is Let, y(t) = x (a t + b)
_____. [2016]
a = 2.  [a > 1, so time compressor]
Solution:  Frequency fm = 33 Hz (∴ y(t) = x(2t) No effect of time shift)
Frequency fs = 46 Hz 1

T
1
∴ x(t) Nyquist sampling rate = 2fm samples/second.
y(t) Nyquist sampling rate = 2 × fsx(t) = 2 × 8 = 16
samples/sec
Hence, the correct option is (C).
–23Hz 0 23Hz f
3. An analog baseband signal, bandlimited to 100 Hz, is
fo = n. fs ± f m . 
p sampled at the Nyquist rate. The samples are quantized
n = 1, 2, …….. etc in to four message symbols that occur independently
with probabilities P1 = P4 = 0.125 and P2 = P3. The infor-
(f(o/p) = 46 + 33, 46 – 33, 92 – 33, 92 + 33 … etc. mation rate (bits/sec) of the message source is _____ .
Hence, we get frequency of o/p is 13 Hz, 79 Hz….. etc.  [2016]
But LPF has cut-off fc = 23 Hz so it allows up to 23 Hz
only. Hence only 13 Hz will pass through LPF) Solution:  Frequency fm = 100Hz
1
Probability P1 = P4 = = 0.125
fo p = ±33Hz , 46 ± 33 Hz.....etc. 8
3
f o / p =13 Hz, 33 Hz, 79 Hz .....etc. Probability P2 = P3 =
8
But fc = 23 Hz so it allows up to 23 Hz only. Number of quantization levels (L) = 4
Hence, the correct Answer is (13). And we know
2. Consider the signal x(t) = cos (6pt) + sin (8pt), where L = 2n
t is in seconds. The Nyquist sampling rate (in samples/ So, 4 = 2n
second) for the signal y(t) = x(2t + 5) is [2016]
(A) 8 (B) 12 ⇒ 22 = 2n
(C) 16 (D) 32 ⇒ n = 2
Solution:  We know that Entropy can be calculated as
x(t) = cos (6πt) + sin(8πt) H = –{P1log2P1 + P2.log2P2 + P3. log2P3 + P4 . log2P4}
and y(t) = x(2t + 5)
2.72 | Signals and Systems

⎧ 1 3 ⎫ After multiplication
= ⎨2 × .log 2 8 + 2 × .log 2 8 ⎬
⎩ 8 8 3⎭

⎧1 3 ⎫
= ⎨ × 3 + × 1.414⎬
⎩4 4 ⎭
= 1.81 bits/symbols
Information rate can be calculated as –0.2 –0.1 0 0.1 0.2 f

R = n × fm × H
=2 × 100 × 1.81 So Nyquist sampling rate = 2 × 0.2
= 200 × 1.81 = 362.0 bits/sec =
0.4 samples/sec
Hence, the correct Answer is (362.0 bits/sec). Hence, the correct Answer is (0.39 to 0.41).
4. Consider a continuous-time signal defined as
5. A sinusoidal signal of 2 kHz frequency is applied to
⎛ sin (π t 2) ⎞ ∞ a delta modulator. The sampling rate and step-size ∆
x(t) = ⎜ ⎟ ∗ ∑ δ (t − 10 n)
⎝ (π t 2) ⎠ n = −∞ of the delta modulator are 20,000 samples per second
and 0.1 V, respectively. To prevent slope overload, the
Where ‘*’ denotes the convolution operation and t is maximum amplitude of the sinusoidal signal (in Volts)
in seconds. The Nyquist sampling rate (in samples/sec) is [2015]
for x(t) is ______. [2015] 1 1
(A) (B)
2π π
sin (π 2) +∞
2
x(t) =
Solution: 
(π t 2)
⊗ ∑ δ (t −10n) (C) (D)
π
π
n = −∞
As we know that convolution in time domain is equal to Solution:  As we know that in delta modulator to
multiplication in frequency domain ­prevent slope overload
+∞ +∞
F .T 1 Δ sm(t )
So ∑ δ (t −10n) ↔ 10 ∑ δ ( f − kf s ) TS

dt
n = −∞ n = −∞  
+∞
1 Δ
= ∑ δ ( f − kf s )
10 n = −∞ TS
≥ Aω m

Where, ∆ = step size
1 1 +∞
and Fs =
Ts
= 0.1 ∑ δ ( f − 0.1k )
10 n = −∞
Ts = sampling period
 A = Amplitude of message signal.
t∞ +∞
1 m = frequency of message signal
ω
So ∑ δ ( t −10n ) ←⎯⎯
F .T

10
∑ δ ( f − 0.1k )
n = −∞ n = −∞
 0.1 × 20,000 = A × 2π × 2 × 103
sin (π t z ) 1
F .T
←⎯⎯ → ∴ A=
(π t z ) 2π 
Hence, the correct option is (A).

⎡ π⎤
6. The signal cos ⎢10π t + ⎥ is ideally sampled at a sam-
⎣ 4⎦
pling frequency of 15 Hz. The sampled signal is passed
–1
4
+1
4
f ⎛ sin (π t ) ⎞
through a filter with impulse response ⎜ cos
⎝ π t ⎟⎠
+∞ ⎛ π⎞
⎜⎝ 40π t − ⎟⎠ . The filter output is  [2015]
∑ δ (t −10n) ←⎯⎯
F .T
→ 2
n = −∞ 
15 ⎛ π⎞
(A) cos ⎜ 40π t − ⎟
2 ⎝ 4⎠
15 ⎛ sin (π t ) ⎞
1
⎛ π⎞
(B) ⎜ ⎟ cos ⎜10π t + ⎟
2 ⎝ πt ⎠ ⎝ 4⎠
0.1 0.2 0.3
Chapter 8  Sampling  |  2.73

15 ⎛ π⎞ By adding phase shift


(C) cos ⎜10π t − ⎟ π π⎞
2 ⎝ 4⎠ 15 ⎛
= cos ⎜ 40π t − + ⎟
⎝ 2 4⎠ 
15 ⎛ sin (π t ) ⎞ ⎛ π⎞ 2
(D) ⎜ cos ⎜ 40π t − ⎟
2 ⎝ π t ⎟⎠ ⎝ 2⎠ 15 ⎛ π⎞
= cos ⎜ 40π t − ⎟
2 ⎝ 4⎠ 
⎛ π⎞
Solution:  Let x(t) = cos ⎜10π t + ⎟
⎝ 4⎠ Hence, the correct option is (A).
By ignoring phase 7. Consider two real valued signals, x(t) band-lim-
ited to [−500 Hz, 500 Hz] and y(t) band-limited to
x′(t) = cos(10 π t )
[−1 Hz, 1 Hz]. For z(t) = x(t) y(t). The Nyquist sampling
1 frequency (in kHz) is _____. [2014]
X′( f ) = [δ( f – 5) + δ( f + 5)]
2 Solution: 3000
Impulse response Using multiplication property,
⎛ sin π t ⎞ ⎛ π⎞
X 1 (t ) ⋅ X 2 (t ) = X 1 (w ) ∗ X 2 (w )
h(t) = ⎜ cos ⎜ 40π t − ⎟
⎝ π t ⎟⎠ ⎝ 2⎠ 
sin c π t sin 40 π t
= So highest f2 contained by convolve signal z (t) = 1500H3
∴ Nyguist rate = 2 × 1500 = 3000H3.
1
H( f ) = rect ⊗ [δ( f – 20) – δ( f + 20)] 8. Let x(t) = cos(10p t) + cos(30p t) be sampled at 20 Hz
2j
and reconstructed using an ideal low-pass filter with
So cut-off frequency of 20 Hz. The frequency/frequencies
H(f) present in the reconstructed signal is/are [2014]
(a) 5 Hz and 15 Hz only
(b) 10 Hz and 15 Hz only
(c) 5 Hz, 10 Hz and 15 Hz only
(d) 5 Hz only
–20
Solution: (a)
20 f Given x (t ) = cos 10pt + cos 30pt
Given f S = 20 H 3 , ⇒ w S = 40p rad sec

After sampling of X′( f ) with 15 Hz the signal will X(ω )


repeat after 15 Hz.
XS1 (f )
ω
15 −30π −10π 10π 30π
2
y(w ) = ∑
k =−
X (w − kw S )

–20 –15 –10 –5 0 5 10 15 20 Hz X(w)

And the amplitude will depend upon that how much


amplitude impulse train is used to sample the x(t).
So Y( f ) = X s′ ( f ) H ( f )
 w
−70p −50p −30p −10p 10p 30p 50p 70p
By seeing the spectrum only one delta function will be
present at 20 kHz. −40p 40p

1 Applying LPF we will get 10p and 30p at o/p or 5H3,


So (δ[f – 20] – δ(f + 20)) 15kH3
2j
Hence, the correct option is (a).
15
y(t) = sin(40 π t) 9. For a given sample-and-hold circuit, if the value of the
2 hold capacitor is increased, then [2014]
15 ⎛ π⎞ (a) drop rate decreases and acquisition time ­decreases
= cos ⎜ 40π t − ⎟ (b) drop rate decreases and acquisition time i­ncreases
⎝ 2⎠ 
2
2.74 | Signals and Systems

(c) drop rate increases and acquisition time d­ ecreases Solution: (a)


(d) drop rate increases and acquisition time i­ncreases Flat-top sampling of low pass signals gives rise to aper-
Solution: (b) ture effect.
We know Hence, the correct option is (a).
=Q CV = it (1) 13. A 1.0 kHz signal is flat-top sampled at the rate of 1800
samples/sec and the samples are applied to an ideal rec-
CV tangular LPF with cut-off frequency of 1100 Hz. Then
So t = (2)
i the output of the filter contains [1995]
From (2) t lC ⇒ as capacitance will increase the ac- (a) only 800 Hz component
quisition time will increase for capacitor drop rate is (b) 800 Hz and 900 Hz components
dv (c) 800 Hz and 1000 Hz components
given as (d) 800 Hz, 900 Hz and 1000 Hz components
dt
dv Solution: (c)
i =c Given fs = 1800 Samples/sec
dt dx i
or × fm = 1000 Hz
dv i dt c
⇒ = The Spectrum of Sampled signal would have nfs ± fm
dt c
So, 1000, 1800 ± 1000 Hz, 3600 ± 1000 Hz…
As capacitor value increases, the voltage drop rate de- So, 1000, 800, 2800, 2600, 4600 Hz…
creases. The cutoff frequency of LPF is 1100 Hz. So the o/p filter
Hence, the correct option is (b). will contain 800 Hz and 1000 Hz components.
Hence, the correct option is (c).
10. A band-limited signal with a maximum frequency of
5 KHz is to be sampled. According to the sampling 14. Increased pulse-width in that flat-top sampling leads to
theorem, the sampling frequency which is not valid is  [1994]
 [2013]
(a) 5 kHz (b) 12 kHz m(I ) M(f )
(c) 15 kHz (d) 20 kHz
Solution: (a)
Given
f S min = 2f m

(f S ) min = 2 × 5 = 10 kH 3 . 0
f
−W W
So f S ≥ 10 kH 3
Hence, the correct option is (a). (a) attention of high frequencies in reproduction
(b) attention of low frequencies in reproduction
11. The transfer function of a zero-order-hold system is (c) greater aliasing errors in reproduction
 [1998] (d) no harmful effects in reproduction
1 1
(a) (
 s  1+ e
 
− sT
)
(b)  s  1− e
 
(
− sT
) Solution: (a)
X(t) X(t) = |τ sin(1+τ)|
1 + (1 / s)e − sT
1 − (1 / s)e − sT (d)
(c)
F.T
Solution: (b)
The impulse response h(t) of zero-order hold ­system is t t
h(t) = U(t) −U(t−T)
As pulse width t is increased, the width 1/T of the first
1 1 −ST (1 − e −ST ) lobe of spectrum is decreased.
So, H (s ) = − e =
s s S
S 2 +1 eS eS eS
Hence, the correct option is (b). Y (S ) = × = =
S 2 + 2S + 1 S 2 + 1 S 2 + 2S + 1 (S + 1) 2
12. Flat top sampling of low pass signals [1998] Take inverse Laplace Transform
(a) gives rise to aperture effect
(b) implies oversampling y (t ) = (t + 1)e − (t +1)
(c) leads to aliasing S ⋅ eS
(d) introduce delay distortion y ( ) = lim SY (S ) = lim =0
s →0 s → 0 (S + 1) 2
Chapter 8  Sampling  |  2.75

So at steady state, Y remains zero for all sampling f2 . |H(f)|


wS .
Hence, increased pulse-width in flat-top sampling leads K
to attenuation of high frequencies in reproduction.
Hence, the correct option is (a).

Two-marks Questions –8 –6 0 6 8 f(kHz)


1. A voice signal m(t) is in the frequency range 5 kHz to
15 kHz. The signal is amplitude-modulated to gener- The minimum sampling rate fs (in kHz) for perfect
ate as AM signal f(t) = A(1 + m(t))cos2 p fct, where fc ­reconstruction of x(t) is ________. [2018]
= 600 kHz. The AM signal f(t) is to be digitized and Solution:  Minimum sampling rate fs should be
archived. This is done by first sampling f(t) at 1.2 times
fs ≥ 5 + 8 ⇒ fs ≥ 13
the Nyquist frequency, and then quantizing each sam-
ple using a 256-level quantizer. Finally, each quantized So, fs(min) = 13 kHz
sample is binary coded using K bits, where K is the
Hence, the correct answer is 13.
minimum number of bits required for the encoding.
The rate, in Megabits per second (rounded off to deci- 3. The signal x(t) = sin (14000 pt), where t is in seconds,
mal places). Of the resulting stream of coded bits is is sampled at a rate of 9000 samples per second. The
_____ Mbps. [2019] sampled signal is the input to an ideal lowpass filter
with frequency response H(f ) as follows :
Solution: 
1, f ≤ 12 kHz
f(t)
H(f)=
0, f > 12 kHz

What is the number of sinusoids in the output and their


frequencies in kHz? [2017]
f(KHz) (A) Number = 1, frequency = 7
0 585 595 600 605 615 (B) Number = 3, frequencies = 2,7,11
(C) Number = 2, frequencies = 2,7
2 fH (D) Number =2, frequencies = 7, 11
fN =
K 4. A continuous time filter with transfer function H(s) =
 f  2s + 6
K = H  2
is converted to a discrete time filter with
 BW  s + 6s + 8
2 z 2 − 0.5032 z
615 transfer function G(z) = 2 so that the
= = 20 z − 0.5032 z + k
30 impulse response of the continuous time filter, sam-
pled at 2 Hz, is identical at the sampling instants to the
2 × 615 impulse response of the discrete time filter. The value
fN = = 61.5 KHz
20 of k is _____. [2016]
f S = 1.2 f N Solution:  To find the Inverse Laplace of transfer
= 1.2 × 61.5 KHz ­function H(s) we need to calculate the values using par-
= 73.8 KHz tial fraction formula as given below:

2 s +6
PB = nfs = 8 × 73.8 H(s) = 2
s + 6s + 8
= 590.4 kbps 

A B
H(s) = +
2. A band limited low-pass signal x(t) of bandwidth 5 ( s + 2) s + 4 
kHz is sampled at a sampling rate fs. The signal x(t)
is reconstructed using the reconstruction filter H( f ) A = H(s) ⋅ (s + 2) at s = −2
whose magnitude response is shown below: 2
A = = 1
2
2.76 | Signals and Systems

B = H(s) (s + 4) at s = −4 6. A signal m(t) with bandwidth 500 Hz is first multiplied


by a signal g(t) where
−2
B = = 1
−2
Now we have
g(t) = ∑ (−1) d (t − 0.5 ×10
k =−
k −4
k)
1 1
H(s) = + The resulting signal is then passed through an ideal low
s+2 s+4
pass filter with bandwidth 1 kHz. The output of low
h(t) = ILT {H(S)} = (e−2t + e−4t)u(t) pass fitter would be [2006]
(a) d (t ) (b) m(t)
1 1 (c) 0 (d) m (t ) d (t )
We know that, TS = =
fs 2
Solution: (b)
Substituting t = n ⋅ TS we get Given
M(t) G(t )
h(nTS) = (e −2 nTs + e −4 nTs) ⋅ u(nTS)
=
e−n ⋅ u(n) + e−2n ⋅ u(n)
H(Z) = ZT{h(nTS)} t t
−500 +500 −20 kHz +20 kHz
Z Z
H(Z) = + m(t) g(t) ↔M(t) G(t)
Z −e −1
Z − e −2
 So,
Z Z
= + M(t) × G(t )
Z − 0.367 Z − 0.135 
2
Z − 0.135Z + Z − 0.367 Z
2
= 2
Z − 0.5032 Z + 0.049  t
−20 kHz +20 kHz
2 Z − 0.5032 Z
2
=2
Z − 0.5032 Z + 0.049 low-pass filter fe = 1 kHz, so o/p = 0

∴ K = 0.049 Hence, the correct option is (b).

Hence, the correct Answer is (0.049). 7. A 1 kHz sinusoidal signal is ideally sampled at 1500
samples/sec and the sampled signal is passed through
s2 + 1 an ideal low-pass filter with cut-off ­frequency 800 Hz.
5. An LTI system having transfer function and
s + 2s + 1
2 The output signal has the frequency [2004]
input x(t) = sin(t + 1) are in steady state. The output is (a) zero Hz (b) 0.75 kHz
sampled at a rate w s rad/s to obtain the final output (c) 0.5 kHz (d) 0.25 kHz
{y(k)}. Which of the following is true? [2009] Solution: (c)
(a) y is zero for all sampling frequencies w s Given fs = 1.5 kHz,
(b) y is non-zero for all sampling frequencies w s fm = 1 kHz
(c) y is non-zero for w s > 2 but zero for w s < 2 available frequency components = nfs ± fm
(d) y is zero for w s > 2 but non-zero for w s < 2 So, 1 kHz, 2.5 kHz, 0.5 kHz.
Solution: (a) ∴LPF has fe = 0.8 kHz.
So only 0.5 kHz will be at o/p.
s2 +1 Hence, the correct option is (c).
X (s ) → H (s ) = → X (s )
s + 2s + 1
2
8. The transfer function of a zero-order hold is [1988]
x(t) = sin (t + 1) →w = 1 1− exp( −Ts )
(a) (b) 1/s
Laplace transform s
w 1 es (c) 1 (d) 1/[ − exp( −Ts)]
X (s ) = es = 2 2 es = 2
s +w
2 2
1 +s s +1 Solution: (a)
Given Impulse response of system is
⇒y(s) = H(s)*(s)
Hence, the correct option is (a). h (t ) = u (t ) − u (t − T ), T→ sampling period.
Chapter 8  Sampling  |  2.77

Taking Laplace transform. Solution: (d)


1 1 1− e−TS fs = 9000 samples/sec.
H (S ) = − e −TS = =fm1 3=
kH 3 , fm 2 6 kH 3 ,
S S S
Hence, the correct option is (a).  Then spectrum of sampled signal would have
9. A signal containing only two frequency components (3 nfs ± fm
kHz and 6 kHz) is sampled at the rate of 8 kHz, and So, 3kH3, 8 ± 3, 16 ± 3… = 3kH3, 5kH3, 11kH3, …
then passed through a low pass filter with a cut-off fre- 6kH3, 8 ± 6, 16 ± 6… = 6kH3, 2kH3, 14kH3, …
quency of 8 kHz. The filter output [1988] Cut of frequency of low pass filter = 8kH3,
(a) is an undistorted version of the original signal So filter output would have,
(b) contains only 3 kHz component 3kH3, 6kH3, 2kH3 and 5kH3
(c) contains the 3 kHz component and a spurious com- Hence, the correct option is (d).
ponent of 2 kHz
(d) contains both the components of the original signal
and two spurious components of 2 kHz and 5 kHz
Unit iII
Control Systems

Chapter 1: Basics 3.3


Chapter 2: Block Diagram and SFG 3.9
Chapter 3: Compensators and Controllers 3.18
Chapter 4: Time Response Analysis 3.25
Chapter 5: Stability Analysis 3.44
Chapter 6: Root Locus 3.55
Chapter 7: Frequency Analysis 3.62
Chapter 8: State Space Analysis 3.77
EXAM ANALYSIS
Exam Year 92 93 94 95 96 97 98 99 00 01 02 03 04 05 06 07 08 09 10 11 12 13 14-1 14-2 14-3 14-4 15 16 17 18 19
Set 1 Set 2 Set 3 Set 1 Set 2 Set 3 Set 1 Set 2
1 Marks Ques. - - 5 8 - - 9 4 1 4 4 2 2 3 2 1 2 2 3 3 1 1 2 2 1 2 3 2 3 2 2 2 2 3 2 1
2 Marks Ques. 4 2 3 - - 2 1 3 1 3 4 6 9 7 9 9 7 4 3 3 4 4 3 3 3 3 3 3 3 3 3 2 2 2 3 3
5 Marks Ques. - - 1 - 3 2 3 3 2 3 3 - - - - - - - - - 9 9 8 8 7 8 - - - - - - - - - -
Total Marks 8 2 16 8 15 14 26 25 13 25 27 14 20 17 20 19 16 10 9 9 - - - - - - 9 8 9 8 8 6 6 7 8 7

Chapter wise marks


distribution
Basics - - 1 1 - - 1 - - 3 - - 2 1 7 2 3 - 1 - 1 - - - - - 2 1 - 1 -
Block Diagram and SFG - - - 1 - 2 - - - 1 - 2 2 - - - - - 1 4 - 2 - 1 1 - 3 2 1 - 1
Compensators and 2 - - - - - - 1 - - - 1 - 2 2 4 2 1 2 - 4 - - - - - 2 - 2 - -
Controllers
Time Response Analysis - - 5 4 - - 5 3 - 1 3 2 2 4 - 2 5 2 - 1 - 2 2 1 2 3 2 3 1 1 -
Stability Analysis - 2 2 - - - 2 2 3 2 7 2 5 4 7 1 4 - - - 2 - 1 - - 2 1 3 2 1 1
Root Locus 2 - 1 - - - - 2 - 1 1 1 1 2 - 2 - 2 - 1 - - - - 2 - 3 1 - - -
Frequency Analysis 2 - 2 2 - - 2 - - 1 - 3 4 3 2 2 - 4 1 1 - 1 3 2 - 1 4 4 2 1 1
State Space Analysis 2 - - - - 2 - 3 - - - 2 4 1 2 6 2 1 4 2 2 4 2 4 2 2 1 - 1 1 1
Chapter 1
Basics
2. A system with transfer function
One-mark Questions
( s 2 + 9)( s + 2)
G ( s) =
1. The output of a standard second-order system for a unit ( s + 1)( s + 3)( s + 4)
step input is given as is excited by sin (wt). The steady-state output of the
2 ⎛ π⎞ system is zero at
y (t ) = 1 − e − t cos ⎜ 3t − ⎟ . The transfer function (a) w = 1 rad/s (b) w = 2 rad/s
3 ⎝ 6⎠
(c) w = 3 rad/s (d) w = 4 rad/s
of the system is [2015]
 [2012]
2 1
(A) (B) Solution: (c)
(
( s + 2) s + 3 ) s2 + 2s + 1
Given transfer function
3 4 ( s 2 + 9)( s + 2)
(C) 2 (D) H ( s) =
s + 2s + 3 s2 + 2s + 4 ( s + 1)( s + 3)( s + 4)
Solution: 
Given Input x(s) = sin wt
y(t) = 1 –
2
3
−t
.e .cos ( 3t − π
6 ) Taking Laplace transform
w
x ( s) =
we know s + w2 2
−ζω n t
e
c(t) = 1 – . sin(ωdt + θ) ∴ Output C(s) = X(s). H(s)
1−ζ 2
( s 2 + 9)( s + 2) w
ωd = 3 rad/sec C ( s) = . 2
( s + 1)( s + 3)( s + 4) s + w 2

ζ = cos θ = cos ( −π 6 ) = 0.866
∴ Lt c(t ) = Lt s c( s)
t →∞ s →0
ωd = ωn. 1 − ζ  2
( s 2 + 9)( s + 2) w
= Lt s. . 2
ω = ω n2 − (ζω n )
2
s →0 ( s + 1)( s + 3)( s + 4) s + w 2
d 
Lt c(t ) = 0
If s2 + w2 = s2 + 9 for t →∞
3 = ω n2 – 1
w =9
2

ω n2 = 4
w = 3 rad/sec, steady-state output will be zero.
⇒ ωn = 2 rad/sec Hence, the correct option is (c)
ω n2 Y ( s) s
∴ T(s) = 3. A system with the transfer function =
s 2 + 2ζω n s + ω n2 X ( s) s + p

4  p
∴ T(s) = has an output y(t ) = cos  2t −  for the input signal
2
s + 2s + 4   3
 p
Hence, the correct option is (D). x(t ) = p cos  2t −  . Then, the system parameter p is
 3
3.4 | Control Systems

2 which is the transfer function of notch filter.


(a) 3 (b)
3 Hence, the correct option is (d)
3 5. In the system shown below, x(t) = (sin t)u(t). In steady-
(c) 1 (d)  [2010] state, the response y(t) will be
2
Solution: (b)
y ( s) s
=
x ( s) s + p

Phase difference between input and output

f = −p −  − p  = p = 30° and w = 2 rad/s.


3  2  6 1  π 1  π
(a) sin  t −  (b) sin  t + 
From the transfer function 2  4  2  4
w 1
f = 90° − tan −1 (c) e −t sin t (d) sin t - cos t
p 2

 [2006]
−1 2
∴ 90° − tan = 30°
p Solution: (a)

2
⇒ tan −1 = 60°
p

2
⇒ = tan 60° = 3
p

2
p=
3 gives x(t) = (sin t) u(t).
Hence, the correct option is (b) ∴ In steady-stake
4. The pole-zero plot given below corresponds to a y(t) = n (t) + h(t)
∴ y(s) = x(s). H(s)
1 1
H ( jw ) = = ∠ − 45°
s +1 2
1 p
= ∠−
2 4
x(t) = sin(t) u(t)
(a) Low-pass filter (b) High-pass filter 1  p
(c) Band-pass filter (d) Notch filter [2008] y (t ) = sin  t − 
2  4

Solution: (d)
Hence, the correct option is (a)
6. Despite the presence of negative feedback, control sys-
tems still have problems of instability because the
(a) components used have nonlinearities
(b) dynamic equations of the systems are not known
exactly
(c) mathematical analysis involves approximations
(d) system has large negative phase angle at high fre-
quencies [2005]
s 2 + as + b Solution: (a)
∴ G ( s) =
s 2 + ps + q Hence, the correct option is (a)

Chapter 1  Basics  |  3.5

7. The transfer function of a tachometer is of the form step u(t) is applied at the input of the system. At steady
K state, the output has constant value of 1. The impulse
(a) Ks (b) response of this system is
s
(a) [exp (-2t) + exp (-4t)] u(t)
K K
(c) (d)  [1998] (b) [-4exp (-2t) + 12 exp(-4t)-exp(-t)] u(t)
( s + 1) s ( s + 1) (c) [-4exp (-2t) + 12 exp(-4t)] u(t)
Solution: (a) (d) [-0.5exp (-2t) + 1.5 exp(-4t)] u(t) [2008]
The tachometer input is assumed to be shaft position. Solution: (c)
Thus, it differentiates shaft position to angular rate given transfer function
and multiplies it by the gain, So, transfer function of a
k ( s + 1)
tachometer is of Ks. G ( s) =
( s + 2)( s + 4)
Hence, the correct option is (a)
8. The transfer function of a linear system is the Unit step input, R( s) = 1
(a) ratio of the output, v0(t) and input vi(t). s
(b) ratio of the derivatives of the output and the input. Output C(s) = R(s) . G(s)
(c) ratio of the Laplace transform of the output and
∴ Lt s c( s) = 1
that of the input with all initial conditions zeros. s →0
(d) none of these [1995]
s.k ( s + 1)
Solution: (c) ⇒ Lt =1
s →0 s( s + 2)( s + 4)
Given a linear system, the transfer function, G(s), of the
system is the ratio of the transform of the output to the k
⇒ =1
transform of the input with all initial conditions zero. 8
Hence, the correct option is (c)  ∴ k = 8
9. Tachometer feedback in a d.c. position control system 8( s + 1)
enhances stability. State True or False. [1994] G ( s) =
( s + 2)( s + 4)

Solution: True
Applying partial fraction
Tachometers are electromechanical devices that con-
8( s + 1) A B
vert mechanical energy into electrical energy. It is a = +
derivative feedback. So, it adds zero at origin. Hence, it ( s + 2)( s + 4) s + 2 s + 4

improves the damping characteristics of the system. 8 (s + 1) = A (s + 4) + B (s + 2)
Let S = −4
Two-marks Questions −24 = −2B ∴ B = 12
1. Negative feedback in a closed-loop control system does Let S = −2
not [2015] ⇒ −8 = 2A ∴ A = −4
(A) reduce the overall gain
(B) reduce bandwidth −4 12
∴ G ( s) = +
(C) improve disturbance rejection
s+2 s+4
(D) reduce sensitivity to parameter variation
g(t) = (−4e−2t + 12e−4t) u(t)
Solution:  Negative feedback effects in closed loop
which is the impulse response of the system.
system:
   (i) Reduces the overall gain Hence, the correct option is (c)
 (ii) Bandwidth increases 3. The frequency response of a linear time-invariant sys-
tem is given by
(iii) Reduces sensitivity to parameter variation
  (iv) Reduces noise effect 5
H( f ) = .
  (v) Improves disturbance rejection ratio 1 + j10p f
Hence, the correct option is (B). The step response of the system is
(a) 5 (1-e-5t) u (t) (b) 5 (l - e-t/5) u(t)
2. A linear, time-invariant, causal continuous time system
has a rational transfer function with simple poles at s
1
( )
(c) 1 − e −5t u(t ) (d)
5
1
( s + 5)( s + 1)
= -2 and s = -4, and one simple zero at s = -1. A unit
 [2007]
3.6 | Control Systems

Solution: (b) Taking determinant of the characteristics equation


Frequency response D of G1 = a 2 − 4b
5
H( f ) = ∴ 3-dB bandwidth = a 2 − 4b
1 + j10p f

5 5 1 s
H ( s) = = = G2 ( s) = 2
1 + 5s 1   1 s + as + b
5 + s   s + 
5   5 ∴ Determinant of the characteristics equation
1
Step response G(s) = R(s). H(s)   ∴ R( s) = D of G2 = a 2 − 4b
s
1 1 A B ∴ 3 dB-Bandwidth of G = a 2 − 4b
2
C ( s) = × = +
s  1 s  1 Hence, the correct option is (c)
s+ 5 s+ s 
    5. The unit step response of a system starting from rest is
given by
 1
⇒ 1 = A  s +  + BS c(t) = 1 - e-2t for t ≥ 0
 5

The transfer function of the system is
Let s = 0 1
(a) (b) 2
1 1 + 2s 2+s
   1 = A ∴A=5
5 1 2s
(c) (d)  [2006]
1 2+s 1 + 2s
Let S = −
5 Solution: (b)
1 c(t) = 1 − e−2t – (unit step response)
1= − B ∴ B = −5
5 1 1 s+ 2−3 2
∴ c( s ) = − = =
5 5 s s + 2 s( s + 2) s( s + 2)
C ( s) = −
s  1 1 C ( s) 2
s+ 5 ∴ R( s) = ∴ H ( s) = =
  s R( s) 2 + S
∴ c(t) = 5[1 − e−t/5] u(t) Hence, the correct option is (b)
Hence, the correct option is (b) 6. The unit impulse response of a system is
4. Consider two transfer functions h(t) = e-t, t ≥ 0.
1 s For this system, the steady-state value of the output for
G1 ( s) = 2
and G2 ( s) = 2 unit step input is equal to
s + as + b s + as + b
(a) -1 (b) 0
The 3-dB bandwidths of their frequency responses are, (c) 1 (d) ∞ [2006]
respectively Solution: (c)
(a) a 2 − 4b , a 2 + 4b Unit impulse response of a system
h(t) = e−t, t ≥ 0
(b) a 2 + 4b , a 2 − 4b
1
∴ H ( s) =
(c) 2 a − 4b , a − 4b 2
s +1

(d) a 2 + 4b , a 2 + 4b  [2006] 1
R( s) =
  s
Solution: (c) 1 1
For the given transfer function. ∴ Output C ( s) = H ( s).R( s) = .
( s + 1) s
1
G1 ( s) = Solving by partial fraction
s 2 + as + b
Chapter 1  Basics  |  3.7

.1 A B Let S = −2
⇒ = +
s( s + 1) s s + 1 2 = B(−2)(−1)

∴ 2 = 2B ∴ B = 1
1 = A (S + 1) + BS
1 1 2
Let S = 0 ∴ y ( s) = + −
s s + 2 s +1
1=A ∴A=1
Let S = −1 Taking inverse Laplace transform
1 = −B ∴ B = −1 y(t) = [1 + e−2t – 2e−t] u (t).
Hence, the correct option is (a)
1 1
Output c( s) = − 8. An electrical system and its signal-flow graph repre-
s s +1
sentations are shown in figures (a) and (b), respectively.
= (1 − e−t) u(t) The values of G2 and H, respectively, are
When t = ∞ at steady state
V0(s)
Output = 1 Z1(s) Z2(s)
Hence, the correct option is (c)
7. A system described by the following differential equa-
d2 y dy V1(s) Z3(s) Z4(s)
tion
2
+3 + 2 y = x(t ) is initially at rest. For input I1(s) I2(s)
dt dt
x(t) = 2u(t), the output y(t) is
(a) (l - 2 e-t + e-2t) u(t)
(b) (l + 2e-t -2e-2t) u(t)
(c) (0.5 + e-t +1.5e-2t) u(t)
(d) (0.5 + 2e-t + 2e-2t) u(t) [2004]
Solution: (a)
d2 y dy
Differential equations: +3 + 2 y = x (t )
dt 2 dt
Taking Laplace transform on both sides.
⇒ s2y(s) + 3sy(s) + 2y(s) = x(s)(i)
Z3 ( s) − Z3 ( s)
Input given: x(t) = 24(t) (a) ,
Z 2 ( s) + Z3 ( s) + Z 4 ( s) Z1 ( s) + Z3 ( s)
2
x ( s) = − Z3 ( s) − Z3 ( s)
5 (ii) (b) ,
Z 2 ( s) − Z3 ( s) + Z 4 ( s) Z1 ( s) + Z3 ( s)
Sub value of x(s) in equation (i)
Z3 ( s) Z3 ( s)
2 (c) ,
∴ ( s 2 + 3s + 2) y( s) = Z 2 ( s) + Z3 ( s) + Z 4 ( s) Z1 ( s) + Z3 ( s)
s
− Z3 ( s) Z3 ( s)
2 (d) ,  [2001]
y ( s) = Z 2 ( s) − Z3 ( s) + Z 4 ( s) Z1 ( s) + Z3 ( s)
s( s + 2)( s + 1)
(iii)
Solution: (c)
Using partial fraction
Applying KVL in both loop,
2 A B C
= + + for 1st loop
s( s + 1)( s + 2) s s + 2 s + 1
Vi(s) = I1(s) Z1(s) + [I1(s) – I2(s)]Z3(s)
⇒ 2 = A(s + 2)(s + 1) + B(s)(s + 1) + C(s)(s + 2) ⇒ Vi(s) = I1(s) [Z1(s) + Z3(s)] – I2(s) Z3(s)
Let S = 0 V1 ( s) I ( s) − I 2 ( s) Z3 ( s)
⇒ = 1
2 = A(2) ∴ A = 1 Z1 ( s) + Z3 ( s) ????

Let S = −1
Vi ( s) I ( s) + Z3 ( s)
2 = C(−1) (+1) I1 ( s) = + 2
Z1 ( s) + Z3 ( s) Z1 ( s) + Z3 ( s)
2 = −C ∴ C = −2 (i)
3.8 | Control Systems

In 2nd loop For a unity feedback H(s) = 1


[I2(s) – I1(s)]Z3(s) + I2(s)Z2(s) + I2(s) Z4(s) = 0 G ( s) s+4
= 2
⇒ I2(s)Z3(s) – I1(s)Z3(s) + I2(s)Z2(s) + I2(s)Z4(s) = 0 1 + G ( s) s + 7 s + 13

I 2 ( s) Z3 ( s)
⇒∴ G2 = = 1 + G ( s) s 2 + 7 s + 43
I1 ( s) Z 2 ( s) + Z3 ( s) + Z 4 ( s) ∴ =
(ii) G ( s) s+4

From signal flow graph.
1 s 2 + 7 s + 13
I1(s) = ViG1(s) + I2(s)H(s) (iii) ⇒ +1 =
G ( s) s+4
Comparing equation (i) and (iii)
.1 Z3 ( s) 1 s 2 + 7 s + 13
G1 ( s) = ; H ( s) = ⇒ = −1
Z1 ( s) + Z3 ( s) Z1 ( s) + Z3 ( s)   G ( s) s+4

Hence, the correct option is (c) 1 s 2 + 7 s + 13 − s − 41
⇒ =
9. The open-loop DC gain of a unity negative feedback G ( s) s+4

system with closed-loop transfer function
s+4 1 s2 + 6s + 9
⇒ =
is G ( s) s+4
s 2 + 7 s + 13
(a) 4 (b)
4 s+4
∴ G ( s) =
13 9 2
s + 6s + 9

(c) 4 (d) 13 [2001] For D.C. S = 0
Solution: (b) 4
∴ G ( s) =
s+4 9
Closed loop transfer function =
s 2 + 7 s + 13 Hence, the correct option is (b)
G ( s) s+4
⇒ = 2
1 + G ( s) H ( s) s + 7 s + 13

Chapter 2
Block Diagram and SFG
G1G2
One-mark Questions G=
(C)
1+ G1G2 H1
1. For the system shown in the figure Y(s)/X(s) = _______ G1G2
[2017] G=
(D)
1+ G1G2 + G1G2 H1

Solution:  Since the block diagram and its transfer


function is given, therefore, Mason formula can be

+ + ­easily applied to evaluate the closed loop gain.
X (s) G (s) = 2 y (s)
+ No. of forward paths = 1
∴ P1 = G2 G1

Individual loop gains


Solution:  Apply Mason’s gain formula between Y(s)
and X(s) L1 = –G1 H1
Number of Forward Paths = 2
L2 = –G2 G1
Forward path gain, P1 = 2, P2 = 1
Number of Loops = 1 Applying the mason’s formula we get
Loop Gain, L1 = -2 Y PΔ
∴ = 1 1
Y ( s) 2 +1 3 X Δ
∴ = = =1 Y G1G2 G1G2
X ( s) 1 − ( −2) 3 = = 
X 1 − ( −G1 H1 − G1G2 ) 1 + G1G2 + G1 H1
Hence, the correct answer is (1).
2. The block diagram of a feedback control system is Hence, the correct option is (B).
shown in the figure. The overall closed loop gain G of 3. The position control of a DC servo-motor is given in the
the system is[2016] figure. The values of the parameters are Kr = 1 N-m/A,
+ + Ra = 1 Ω, La = 0.1 H, J = 5 kg-m2, B = 1 N-m/ (rad/
X G2 G1 Y sec) and Kb = 1 V/(rad/sec). The steady-state position
– –
response (in radians) due to unit impulse disturbance
torque Td is ________. [2015]
H1
Td(s)

+ KT – 1 1
G1G2 V3(s) θ (s)
Ra + Las + Js + B S
G=
(A) –
1+ G1 H1
G1G2
G=
(B) Kb
1+ G1G2 + G1 H1
3.10 | Control Systems

θ (s) Transfer function


Let H(s) =
Solution: 
Td ( s ) z ( s) =
y ( s)
x2 ( s )
From the given Block diagram

−1 G ( s)
Forward path gain = =
s ( Js + B ) 1 + G ( s) H ( s)

∆1 = 1 1
( s + 1)
s
− K b . KT ∴ z ( s) = =
and Individual loop gain L1 = 1 s s( s + 2)
( Js + B ) ( Ra + La s) 1+ ×
s s +1
P1 Δ1
H (s) = Hence, the correct option is (d)
Δ  5. Consider the following block diagram in the figure.
1 + K b . KT ( Js + B ) ( Ra + La s) + K b KT
Δ= =
( Js + B ) ( Ra + La s) ( Js + B ) ( Ra + La s)
−1 s ( Js + B )
⇒ H (s) =
Δ 
−1 ( JS + B ) ( Ra + La s)
H (s) = × C ( s)
S ( JS + B ) ( Js + B ) ( Ra + La s ) + K b .KT The transfer function is
 R( s)
Impulse response H(s)
− ( B ) ( Ra ) G1G2
G1 G2 + G1 + 1
h(∞) = Lt S .H ( s ) = (a)
1 + G1 + G2
(b)
S →0 ( B ) ( Ra ) + K b .KT
Given KT = 1 N-m/A, Ra = 1 Ω, La = 0.1 M, J = 5 kg-m2, G1
G1 G2 + G2 + 1
(c) (d)
B = 1 N-m/ (rad/sec) and KB = 1 V 1 + G1 + G2
−1 × 1
h (∞ ) = = − 0.5 [2014]
1×1+ 1×1 Solution: (c)
Hence, the correct Answer is (–0.51 to –0.49).
4. For the following system,

Signal flow graph:


Forward path:
P1 = G1G2
P2 = G2.1
Y ( s)
when X1(s) = 0, the transfer function is P3 = 1.1 = 1
X 2 ( s)
s +1 1 C ( s)
(a) (b) So, transfer function = G1G2 + G2 + 1
2 s +1 R( s)
s+2 s +1 Hence, the correct option is (c)
(c) (d)
s( s + 1) s( s + 2)
6. The transfer function Y(s)/R(s) of the system shown is
[2014]
Solution: (d)
Chapter 2  Block Diagram and SFG  |  3.11

(a) 0 (b) 1 (c)


s +1
2 2
(c) (d)
s +1 s+3
[2010]
Solution:(b)
(d) [2001]

Solution: (d)
 1 1 
Z ( s) = Q ( s)  −
 s + 1 s + 1  ⇒

=0
So Q(s) = R(s) – 0 = R(s) Hence, the correct option is (d)
Q( s) R( s)
y ( s) = =
s +1 s +1 8. Signal flow graph is used to find
(a) Stability of the system
Y ( s) 1
= (b) Controllability of the system
R( s) s + 1 (c) Transfer function of the system

Hence, the correct option is (b) (d) Poles of the system.
[1995]
7. The equivalent of the block diagram in the figure is
Solution: (c)
given as
Transfer function of the system
Hence, the correct option is (c)

Two-marks Questions
1. The block diagram of a system is illustrated in the figure
shown, where X(s) is the input and Y(s) is the output.
Y (s)
The transfer function H(s) = is  [2019]
(a) X (s)

– s +
+ Y(s)
1
X(s) ∑ ∑
s
– 1 +
s

(b)

s2 + 1
(A) H(s) =
2s2 + 1
s2 + 1
(B) H(s) =
s3 + 2 s 2 + s + 1
3.12 | Control Systems

s +1 Break away point occurs only at the point of where


(C) H(s) = multiple poles are present. Break away point exist at
2s2 + s + 1
dk
s2 + 1 =0
(D) H(s) = ds
s3 + s 2 + s + 1 (s + 2) (2s + 2) – (S2 + 2s + 2) ⋅ 1 = 0
Solution: s2 + 4s + 2 = 0
s1 = – 0.585, s2 = – 3.414

– s Sub S1 and S2 values in characteristic equation


+ 1
X(s) ∑ ∑ Y(s) Find the valid point (k > 0)
s
1 At s = –3.414
s
k{−1.414}
1+ =0
6.8273 
Using Mason’s gain formula. k = 4.828
P1 = 1 ∆1 = 1 ∴ k > 0 at s = –3.414
1
P2 = 2 ∆2 = 1 Break point occurs at s = –3.414.
S Hence, the correct Answer is (–3.414).
Y (S ) P1∆1 + Pz ∆ 2 3. By performing cascading and/or summing/differenc-
=
X (S ) ∆ ing operations using transfer function blocks G1(s) and
G2(s), one cannot realize a transfer function of the form
1  [2015]
1.1 + 1⋅
= S2 (A)
G1(s) G2(s)
 1 1
1 − −1 − 2 − S − 
S G1 ( s )
 S  (B)
G2 ( s )
S2 +1
S2 ⎡ 1 ⎤
= G1(s) ⎢
(C) + G2 ( s )⎥
⎣ G1 ( s )
1 1
2+ 2 +S + ⎦
S S
⎡ 1 ⎤
On sum plefilation G1 ( s ) ⎢
(D) − G2 ( s )⎥
⎣ G1 ( s ) ⎦
S2 +1
= Solution:  By performing cascading/summing/differ-
2S 2 + S 3 + S + 1
encing operations using T/F blocks G1(s) and G2(s), we
Hence, the correct option is (B) G1 ( s )
2. The forward path transfer function and the feedback cannot realize transfer function of the form
G2 ( s )
path transfer function of a single loop negative feed-
back control system are given as Hence, the correct option is (B).
K ( s + 2) 4. For the signal flow graph shown in the figure, the value
G(s) =  and H(s) = 1,
s2 + 2s + 2 C (s)
of is [2015]
respectively. If the variable parameter K is real positive, R (s)
then the location of the breakaway point on the root
G1G2G3G4
locus diagram of the system is [2016] (A)
1− G1G2 H1 − G3G4 H 2 − G2G3 H 3
Solution:  From the given data
k ( s + 2) + G1G2G3G4 H1 H 2
G(s)H(s) = 2
s + 2s + 2  G1G2G3G4
(B)
1 + G(s)H(s) = 0 1+ G1G2 H1 + G3G4 H 2 + G2G3 H 3
( s 2 + 2 s + 2) + G1G2G3G4 H1 H 2
K = −
s+2 
Chapter 2  Block Diagram and SFG  |  3.13

1 Solution: (a)
(C)
1 + G1G2 H1 + G3G4 H 2 + G2G3 H 3 Using Mason’s gain formula
+ G1G2G3G4 H1 H 2 C PK ∆ K
=
1 R ∆
(D)
1 G1G2 H1 G3G4 H 2 G2G3 H 3   ∆ = 1 – [−2s−1 – 2s−2 – 4 – 4s−1]
+ G1G2G3G4 H1 H 2 2 2 4 5s 2 + 6 s + 2
∆ = 1+ + 2 +4+ =
–H3 s s s s2
1
As P1 = s −2 = 2
s
1 X1 X3 1 X4 C(s) 1
P2 =
R(s) G1 X2 G2 G3 X5 G4 s2

∆1 = 1    ∴ ∆2 = 2
–H1 –H2
1 1
×1 + ×1
Solution: From the SFG y( s) ΣPK ∆ K 2 s
= = s 2
no. of forward paths = 1 u( s) ∆ 5s + 6 s + 2
forward path gain P1 = G1 G2 G3 G4 s2
∆1 = 1 – {gain of non-touching loop gains} y ( s) s +1
⇒ =
= 1 – 0 = 1 u( s) 5s 2 + 6 s + 2
Individual loop gains:-
Hence, the correct option is (a)
L1 = –G1 G2 H1
6. The signal flow graph that DOES NOT model the plant
L2 = –G3 G4 H2
transfer function H(s) is
L3 = –G2 G3 H3 (a)
Two non-touching loop gains:-
L1 L2 = G1 G2 G3 G4 H1 H2
(b)
∆ = 1 – {L1 + L2 + L3} + L1 L2
∆ = 1 + {G1G2H1} + G3G4H2
+ G2 G3 H3 + G1 G2 G3 G4 H1H2 (c)
C ( s) P1 Δ1
=
R( s) Δ 
Hence, the correct option is (B).
(d)
5. The signal flow graph for a system is given below. The
Y ( s)  [2011]
transfer function for this system is
U ( s) Solution: (d)
Which is not equal to H(s)
100
Y ( s) 100
= =
U ( s) 100 s( s 100)

s +1 s +1
(a) 2 (b) 2
Hence, the correct option is (d)
5s + 6 s + 2 s + 6s + 2
7. The gain margin of the system under closed loop unity
s +1 s +1 100
(c) 2 (d) negative feedback is G ( s) H ( s) =
s + 4s + 2 2
5s + 6 s + 2 s( s + 10) 2

[2013]
3.14 | Control Systems

(a) 0 dB (b) 20 dB Common Data for Question 5 and 6


(c) 26 dB (d) 46 dB The input-output transfer function of a plant
[2011] 100
Solution: (c) H ( s) = . The plant is placed in a unity nega-
s( s + 10) 2
100 tive feedback configuration as shown in the figure
given G ( s) H ( s) =
s( s + 10) 2 below.
For phase cross-over frequency,
−180 = −90° − 2 tan−1 (w/10)
w = 10 rad/sec
Put s = jw
100 9. The signal flow graph of a system is shown in the fig-
G ( jw ) H ( jw ) =
jw ( jw + 100) 2 C ( s)
ure. The transfer function of the system is
R( s)
1G ( jw ) H ( jw ) w =10

100 100 1
= 2
= =
w ( w + 100) 10(200) 20

1
G.M. = = 20
G ( jw ) H ( jw )

(a) 6
  G.M. in dB = 20 log20 = 26 dB s 2 + 29 s + 6
Hence, the correct option is (c) s( s + 27)
(b)
∴ G ( s) =
8. Consider the signal flow graph shown in the figure. The s 2 + 29 s + 6
gain x5/x1 is
(c) 6 s
s 2 + 29 s + 6
(d) s( s + 2)  [2003]
s 2 + 29 s + 6
1 − (be + cf + dg ) Solution: (d)
(a)
abc
P ∆1
bedg G ( s) =
(b) 1 − (loop gain) + pair of non − touching loops
1 − (be + cf + dg )  
3 24 5 27
abcd ∴∆ = + + =
(c) s s s
1 − (be + cf + dg ) + bedg
P1 = 1
1 − (be + cf + dg ) + bedg 
(d) [2004] L1 and L3 are non-touching
abcd ( s + 27)
s
Solution: (c) ∴ G ( s) =
 3 24 2  −2 −3
P1 = abcd, ∆ = 1 1−  − − − + ×
 s s s s s
Loop again: L1 = be, L2 = cf, L3 = dg
s + 27
Non-touching loops are L1 and L3 = bedg
= s
x P1∆ 29 6
∴ 5 = 1+ +
x1 1 − (loop again) s s2

x abcd s( s + 27)
∴ 5 = ∴ G ( s) =
x 1 − (be + cf + dg ) + bedg s 2 + 29 s + 6
1
Hence, the correct option is (c) Hence, the correct option is (d)
Chapter 2  Block Diagram and SFG  |  3.15

10. In the signal flow graph of the below figure y/x equals Solution: (c)
Using Masson’s gain formulae
C PK ∆ K
=
R ∆
C G1G2G3G4
(a) 3 =
R 1 − [−G1 − G2 − G3 − G4 ] + [G1G3 + G1G4 + G2G3 + G2G4 ]
(b) 5/2
(c) 2 C G1G2G3G4
(d) None of the above [1997] =
R 1 − [−G1 − G2 − G3 − G4 ] + [G1G3 + G1G4 + G2G3 + G2G4 ]
Solution: (c)
Using Masson’s gain formula C G1G2G3G4
=
R (1 + G1 + G2 )(1 + G3 + G4 )
C PK ∆ K
=
R ∆ Hence, the correct option is (c)
C 5 × 2 ×1 10 10 1 3. In the signal flow graph shown in the below figure, X2 =
⇒ = = = =2 TX1 where T is equal to
R 1 − ( −4) 1 + 4 5

Hence, the correct option is (c)
1 1. .In the signal flow graph of the gain c/r will be

(a) 2.5 (b) 5


(c) 5.5 (d) 10 [1987]
− − −
Solution: (d)
(a) 11/9 (b) 22/15
(c) 24/23 (d) 44/23
[1991]
Solution: (d)
Using Masson’s gain formulae
Hence, the correct option is (d)
X2 = TX1
12. The C/R for the signal flow graph in the below given
figure is 0 = 1 – 05 = 0.5
x2 5 5
∴ = = = 10
x1 ∆ 0.5

− − − −
Hence, the correct option is (d)
G1G2G3G4
(a)
(1 + G1G2 )(1 + G3G4 )
14. For the system shown in figure the transfer function
G1G2G3G4 C ( s)
(b) is equal to:
(1 + G1 + G2 + G1G2 )(1 + G3 + G4 + G3G4 ) R( s)
G1G2G3G4
(c)
(1 + G1 + G2 )(1 + G3 + G4 )
G1G2G3G4
(d)
(1 + G1 + G2 + G3 + G4 )
[1989]
3.16 | Control Systems

10 10 Remaining node R5
(a) 2 (b) 2
s + s + 10 s + 11s + 10
10 10
(c) 2 (d)
s + 9 s + 10 2
s + 2 s + 10
[1987]
Solution: (b)
10 ∴node equation at l2
s( s + 1) l2 = l1t12 + l3t35t52 + l2t25t52 + l4t42
10
1+ ×s ⇒l2(1 – t25t52) = e1t12 + e3t35t52 + e4t42
C ( s) s( s + 1)
=
R( s) 10 t12 t35t52 t 42
1+ ⇒ e2 = e1 + e3 + e4
s( s + 1) 5
1 − t 25 52 1 − t 25t52 1 − t 25t52
×1
10 Reduced signal flow graph.
1+ ×s
s( s + 1)
10
∴ C ( s ) s ( s + 1) + 10 s 10
= = 2
R( s) 10 s + 11s + 10
1+
s( s + 1) + 10 s
Hence, the correct option is (b)

2. Draw a signal flow graph for the following set of alge-


Five-marks Questions braic equations:
y2 = ay1 - gy3
1. Reduce the signal flow graph shown in figure below, y3 = ey2 + cy4
to obtain another graph which does not contain the
y4 = by2 - dy4
node e5. (Also, remove any self-loop from the resulting
graph) y2 y2
Hence, find the gains y and  [1998]
1 y1
Solution:
∴ signal flow graph;

y2 a(1 + d )
Solution: ∴ =
y1 1 + d + eg + bcg + deg

y3 ae(1 + d ) + abc
∴ =
y1 1 + d + eg + bcg + deg

3. A feedback control system is shown in figure
Chapter 2  Block Diagram and SFG  |  3.17

(a) Draw the signal-flow graph that represents the sys- L3 = − G1


tem. L4 = − G1G2
(b) Find the total number of loops in the graph and de-
Touching loops, are
termine the loop-gains of all the loops.
(c) Find the number of all possible combinations of L1L2 = G1G2
non-touching loops taken two at a time. L2L3 = G1G2
(d) Determine the transfer function of the system using (d) By mason’s gain formulae
the signal-flow graph. [2001]
C ( s) PK ∆ K
Solution: (a)  =
R( s) ∆
signal flow graph representing the system
G1G2 + G1 (1 + G2 )
T.F. =
1 − (G1 − G2 − G1 − G1G2 ) + (G1G2 + G1G2 )

C ( s) G1 (1 + 2G2 )
=
R( s) 1 + 2G1 + G2 + 3G1G2
Hence, the correct option is (a)

(b) Total number of loops in the graph is 4


i.e., L1 = − G1
L2 = − G2
Chapter 3
Compensators and
Controllers
Solution:  By adding lag compensation, one pole is
One-mark Questions added which is nearer to origin will degrade the stabil-
ity of a system
1. Which of the following can be the pole-zero configu- Hence, the correct option is (D).
ration of a phase-lag controller (lag compensator)?
 [2017] 3. The magnitude plot of a rational transfer function G(s)

with real coefficients is shown below. Which of the fol-
(A) × Pole jω ×compensators
lowing Pole has such a magnitude plot?
s-planejω o Zero
× Pole o Zero
s-planejω × Pole
s-plane o Zero
O × σ s-plane o Zero
× O σ
jω × Pole O
(B) × jω σ× Pole σ
o Zero × O
s-plane o Zero

σ jω ××Pole O
σ jω
× Pole
s-plane o Zero × Pole jω
s-planejω o Zero
o Zero × Pole
(C) jω × Pole
s-plane o Zero
× O σ s-plane
(a) Lead
o Zero O × compensator
× O σ
s-plane
× O O × σ jω (b)
× Pole O ×Lag × compensator
O σ
jω × Pole × O O × σ o Zero (c) PID compensator
s-plane
o Zero (d) Lead-lag compensator [2009]
ne (D) jω
O ××Pole
× O σ
le ×
O σ s-plane o Zero Solution: (d)
ro
The given plot is for a Lead log compensator
O × × O σ
σ Hence, the correct option is (d)

Solution:  It is evident that from pole zero configura- 4. A PD controller is used to compensate a system.
tion Choice (A) satisfies phase lag compensator. Compared to the uncompensated system, the compen-
sated system has
2. Which of the following statement is incorrect?
(a) a higher type number
 [2017]
(b) reduced damping
(A) Leading compensator is used to reduce the settling
(c) higher noise amplification
time.
(d) larger transient overshoot [2003]
(B) Lag compensator is used to reduce the steady state
error. Solution: (c)
(C) Lead compensator may increase the order of a sys- When bandwidth increases then signal-to-noise ratio
tem decreases and system become more prone to noise.
(D)  Lag compensator always stabilizes an unstable
Hence, the correct option is (c)
system.
Chapter 3  Compensators and Controllers  |  3.19

5. The transfer function of a phase lead controller is (a) 2 rad/s (b) 3 rad/s
1 + 3Ts . The maximum value of phase provided by this (c) 6 rad/s (d) 1 3 rad/s
1 + Ts  [2012]
controller is Solution: (a)
(a) 90° (b) 60°
For phase to be maximum
(c) 45° (d) 30° [1998]
∂p
Solution: (d) =0

w
For lead compensator
a (1 + T ′s) 1 + 3Ts 1 1
T.F. = = a − b =0
1 + a T ′s 1 + Ts
w2 w2
1+ 2 1+ 2
So T ′ = 3T a b

  a T ′ = T 1 2
⇒ −
=0
a 3T ′ = T 1+ w 2
w +4 2

1 w2 + 4 – 2 – 2w2 = 0
a =
  3 w2 = 2
 1 w = 2 rad/sec
−1  1 − a
 −1
 1− 3 
fmax = sin   = sin   Hence, the correct option is (a)
 1+ a   1 + 1  3. A unity negative feedback closed loop system has a
 3 plant with the transfer function
1 1
= sin −1   = 30° G ( s) = and a controller Gc(s) in the feed
2 2
s + 2s + 2
forward path. For a unit step input, the transfer function
Hence, the correct option is (d)
of the controller that gives minimum steady-state error
is
Two-marks Questions s +1
Gc ( s) =
(a) ,
Common Data for Questions 1 and 2: s+2
The transfer function of a compensator is given as s+2
Gc ( s) =
(b) ,
s+a s +1
Gc ( s) =
s+b
( s + 1)( s + 4)
Gc ( s) =
(c) ,
1. Gc (s) is a lead compensator if ( s + 2)( s + 3)
(a) a = 1, b = 2
(b) a = 3, b = 2 (d) 2
Gc ( s) = 1 + + 3s,
(c) a = -3, b = -1 5
(d) a = 3, b = 1 [2012] [2010]
Solution: (a) Solution: (d)
s+a
given Gc ( s) =  
Steady-state error
 s+b S R( s)
ess = Lt
phase P = tan−1(w/a) − tan−1(w/b) s →0 1 + G ( s)Gc ( s)

for load comparators phase must be positive
r(t) = G (t)
w w
∴ > ⇒a<b 1
a b R( s) =
s
∴ a = 1, b = 2
1
Hence, the correct option is (a) s.
s 1
∴ ess = Lt = Lt
2. The phase of the above lead compensator is maximum s →0 1 + G ( s)Gc ( s) s →0 1 + G ( s)Gc ( s)
at
3.20 | Control Systems

From the options given V0 ( R2C2 s + 1) ( R1C1s + 1)


∴ = .
s +1 2 Vi (C2 s + 1) R1
For Gc ( s) = ,e =
s + 2 ss 3
∴ Controller in PID controller
s+2 1 For R
For Gc ( s) = , ess =
s +1 3
R2
( s + 1)( s + 4) 3 ∴ Z = R C s +1
For Gc ( s) = , ess = 2 2
( s + 2)( s + 3) 5
V0 R2 ( R C s + 1)
2 ∴ =− . 1 1
For Gc ( s) = 1 + + 3s, ess = 0 Vi ( R2C2 s + 1) R1
s
Hence, the correct option is (d) ∴ Controller is log compensator
4. Group I gives two possible choices for the impedance 5. A control system with a PD controller is shown in the
Z in the diagram. The circuit elements in Z satisfy the figure. If the velocity error constant Kv = 1000 and the
V0 damping ratio ς = 0.5, then the values of Kp and KD are
condition R2C2 > R1C1. The transfer function rep-
Vi
resents a kind of controller. Match the impedances in

Group-I with the types of controllers in Group-II.

(a) Kp = 100, KD = 0.09


(b) Kp = 100, KD = 0.9
(c) Kp = 10, KD = 0.09
(d) Kp = 10, KD = 0 9 [2007]
Solution: (b)
Y ( s) Gs ( k p + k D s)
Group-I = =
R( s) 1 + Gs H ( s) 100 charac-
Q.  1 + ( k p + k D s).
s( s + 10)
teristics equation 1 + G(s) H(s)
K v = Lt s G ( s) H ( s)
s →0

R.  given Kv = 1000


( k p + k D s)100
⇒ 1000 = Lt s ×
s →0 s( s + 10)

Group-II k p × 100
∴1000 =
1. PID controller 10

2. Lead compensator
∴ kp = 100
3. Lag compensator
Now characteristics equation (1 + G(s) H(s)) = 0
(a) Q - 1, R - 2 (b)
Q - l, R - 3
(c) Q - 2, R - 3 (d)
Q - 3, R - 2 ( k p + k D s)100
1+ =0
 [2008] s( s + 10)

Solution :(b)
Put kp = 100
Vi ( R1C1s + 1)1 −V0
=   ∴ given R2C2 > R1 C1 = s2 + 10s + 10000 + 100kDs = 0
1 2
V − z ( R1C1s + 1) = s2 + (10 + 100kD)s + 104 = 0 (i)
⇒ 0 =
Vi R1 Standard 2nd order equation

2 2
R C s +1 s + 2x wn s + wn = 0 (ii)
For Q, z = 2 2
C2 s + 1 Comparing equation (i) and (ii)
Chapter 3  Compensators and Controllers  |  3.21

So, wn = 100; 2 ξwn = 10 + 1000kD ⇒ 3 + 3T2w2 = 1 + 9T2w2


ξ = 0.5; 2 × 0.5 × 100 = 2ξwn ⇒ 6T2w2 = 2
100 = 10 + 100 kD 1
⇒ T 2w2 =
90 3
kD = = 0.9
100 1
⇒ Tw =
6. The open-loop transfer function of a plant is given as 3
1
G ( s) = 2 . If the plant is operated in a unity feed- 1 1
s −1 fmax = tan −1 3 × − tan −1
back configuration, then the lead compensator that can 3 3
stabilize this control system is p p p
10( s − 1) = − =
(a) (b) 10( s + 4) 3 6 6
s+2 s+2
10( s + 2) 10( s + 2) 8. A double integrator plant, G ( s) = K , H(s) = 1 is to be
(c) (d)
s + 10 s + 10 s2
[2007] compensated to achieve the damping ratio ς = 0.5, and
Solution: 
(a) an undamped natural frequency, ω0 = 5 rad/s. Which one
of the following compensator Ge(s) will be suitable?
1 1
given = G ( s) = = s + 3 (b) s + 9.9
s2 − 1 ( s + 1)( s − 1) (a)
s + 9.9 s+3
load compensator C(s) should first stabilize the plant,
s−6 s+6 
1 (c) (d) [2005 ]
i.e., remove term s + 8.33 s
( s − 1)
Solution: (a)
1 10( s − 1)
∴ G ( s) C ( s) = × ξ = 0.5
( s + 1)( s − 1) s+2
cos−1 0.5 = 60°
7. The transfer function of a phase-lead compensator is θ = 60°
given by
k
1 + 3Ts ∠G ( s) =
Ge ( s) = where T > 0 s2 s =−2.5 + 4.33 j
1 + Ts
The maximum phase-shift provided by such a compen- 4.33
= −2 tan −1 ; 120°
sator is −2.5
(a) π/2 (b) π/3
Putting s = −2.5 + 4.33j
(c) π/4 (d) π/6 [2006]
k ( s + 3) 0.5 + j 4.33
Solution: (d) = = 53°
2
s ( s + 9.9) 7.4 + j 4.33
Phase-lead compensator
1 + 3Ts 9. A process with open-loop model
Ge ( s) = where T > 0
1 + Ts − sTD
Max. phase shift G ( s) = Ke is controlled by a PID controller. For
t S +1
fm = ∠Ge ( s) this process.
−1 −1 (a) the integral mode improves transient performance
f = tan 3wT − tan wT
(b)  the integral mode improves steady-state perfor-
For maximum phase shift mance
df (c)  the derivative mode improves transient perfor-
=0
dt mance
(d) the derivative mode improves steady-state perfor-
3T T mance. [1992]
∴ 2
=
1 + (3T + w ) 1 + (Tw ) 2 Solution: (b)
⇒ 3[1+(Tw)2] = 1 + (3Tw)2 Integral mode improves steady-state performance.
3.22 | Control Systems

10. The transfer function of a simple RC network function- 2. A lead compensator network includes a parallel con-
ing as a controller is: tribution of R and C in the feed-forward path. If the
s + z1 s+2
Gc ( s) = transfer function of the compensator is Gc(s) = ,
s + p1 s+4
the value of RC is _______ [2015]
The condition for the RC network to act as a phase lead
controller is: Solution: 
Given
(a) p1 < z1 (b) p1 = 0
(c) p1 = z1 (d) p1 > z1 [1990] s+2
GC(s) =
Solution: (d) s+4 
s + z1 0.5 {1 + 0.5s} ⎧ 1 + sτ ⎫
G ( s) = G(s) = = K.⎨ ⎬

s + p1 (1 + 0.25s) ⎩1 + βτ s ⎭ 
−1  w  −1  w 
τ = 0.5 sec
∴ q = tan   − tan  
z
 1  p1  Hence, the correct Answer is (0.5).
For phase lead controller
θ>0 Five-marks Questions
w w
tan −1   − tan −1   > 0 1. A unity feedback system has the plant transfer function
 z1   p1  1
G P (s ) =
w w (s+1)(2s+1)
tan −1   ⇒ tan −1  
 z1   p1  (a) Determine the frequency at which the plant has a
phase lag of 90°.
⇒ p1 > z1 (b)  An integral controller with transfer function
k
G c (s ) = is placed in the forward path of the
Two-marks Questions s
feedback system.
1. The transfer function of a first – order controller is
given as Find the value of k such that the compensated system
has an open loop gain margin of 2.5.
K (s + a)
Gc(s) = (c) Determine the steady state errors of the compen-
s+b sated system to unit-step and unit-ramp inputs.
where K, a and b are positive real numbers. The condi-  [2002]
tion for this controller to act as a phase lead ­compensator Solution:
is [2015] (a) ∴ θ = - 90°
a < b (B)
(A) a>b ω   2ω 
K < ab (D)
(C) K > ab θ = − tan −1   − tan −1  
1
   1 

−1 −1
K (s + a) ⇒ −90 = − tan (ω ) − tan ( 2ω )
Given GC(s) =
Solution: 
(s + b) ⇒ tan −1 (ω ) + tan −1 ( 2ω ) = 90
Given it is a phase lead controller, so zero is dominating  ω + 2ω 
 1 − 2ω  = tan 10 = ∞
Im
 
⇒1 − 2ω2 = 0
1
–σ –b –a σ ω= rad/sec
2
(b) given G(s) = GP(s). GC(s)
1 k k
G ( s) = × =
∴ a < b ( s + 1)( 2 s + 1) s s( s + 1)( 2 s + 1)

Hence, the correct option is (A). ∴ ωPC = −90° − tan −1 (ω ) − tan −1 ( 2ω )

= −180°
Chapter 3  Compensators and Controllers  |  3.23

1 Solution:
ωPC = rad/sec
2 Transfer function of PD controller (G1) = Kp+ SkD

k Transfer function of power amplifier (G2) = k
| G ( jω ) |ωPC = Transfer function of process
 1  1  1 
j i + 1  2 + 1
 2  2  2  1
(G3 ) =
gain margin G(Gm) = 2.5 s( s + 1)
1 k ( k p + skD )
= Gm ∴ G ( s) = G1G2G3 =
∴ (G ( jω ) ) s( s + 1)
ω PC
1 1 G ( s)
∴ (G ( jω ) )ω = = = 0.4 Overall transfer function =
PC Gm 2.5 1 + G ( s)
k ( k p + skD )
k
= 0.4 C ( s) s( s + 1) k ( k p + sk D )
∴ = = 2
( )
∴  1   1 2
 +1 ( 2) 2 + 1 R( s) 1 + k ( k p + skD ) s + s + ( k p + skD )k
 
 2   2  s( s + 1)
1 C ( s) +kD s
R = 0.4 × ( 3 )( 3) = 2
2 R( s) s + s(1 + kk D ) + kk P

3
k = 0.4 ×
= 0.6 ∴As tw characteristics equation
2 2 2
k p = lim G ( s) H ( s) s + 2ξωn s + ωn = 0
(c)
s →0
1
∴given τ = =1
k ξ cos n
= Lt =∞
s →0 s( s + 1)( 2 s + 1)
  1
ωn =
1 ξ
ess = =0
1+ kp given ξ = 0.707

∴Poles
kv = Lt S (G ( s) H ( s)
s →0
s1 and s2 = s = ξωn 1 − ξ 2
k
= Lt = k = 0.6

s →0 ( s + 1)( 2 s + 1) s = −1 ± j1.41 1 − (0.707) 2

s= −1 ± j1.41 × 0.07

1 1 10
e=
ss = = = 1.667
s= −1 ± j
kv 0.6 6
∴s1 = −1 + j

2. For the feedback control system shown in the figure, s2= −1 – j

1
the process transfer function is G P ( s) = , and C ( s) k ( k p + k D s)
s(s + 1) = 2
(b)  R( s) s + s(1 + kk ) + kk
D P
the amplification factor of the power amplifier is K > 0.
the design specifications required for the system, time Characteristics equation
constant is 1 sec and a damping ratio of 0.707. = s2 + s(1 + k0k) + kkp
2ξωn = 1 + kk D
∴1 + kkD = 2 × 1 – 2
0⇒kkD = 1
given k = 1
1.kp = 2
(a) Find the desired locations of the closed loop poles.
kp = 2
(b) Write down the required characteristic equation
for the system. Hence determine the PD controller 1.kD =1
transfer function Gp(s) when K = 1. kD = 1
(c) Sketch the root-locus for the system. [2001]
3.24 | Control Systems

kP + kDs = 2 + s × 1 = s + 2 dk
k (kD s + k p ) As ds = 0
(c) G ( s) =
s( s +1) ∴−2s2 – s – 4s – 2 + s2 + 5 = 0
k ( s + 2) ∴s2 + 4s + 2 = 0
=
s( s + 1) −4 ± 16 − 8
∴s =
∴Number of pole(p)= 2 2
And number of Zero(z) = 1 s = −2 ± 2
∴p -z = 2 – 1 = 1 s = −0.6, −3.4
180°
∴θ = = 180°
1
Breakaway point
k ( s + 2)
1+ =0
s( s + 1)

dk ( s + 1)( 2 s − 1) − ( s 2 − 5)(1)
∴ =

ds ( s + 2) 2
Chapter 4
Time Response Analysis
( S + 1)
One-mark Questions k p = Lt p = ∞(‘P’ must be ≥ 1)
s ( s + 2)( S + 3)
s→o
1. Consider P(s) = s3 + a2s2 + a1s + a0 with all real coef- 1
e= = 6 (for unit reamp input)
ficients. It is known that its derivative pI(s) has no real ss
kv
roots. The number of real roots of p(s) is
 [2018] 1
kv = Lt SG ( S ) H ( S ) =
(A) 0 (B) 1 6
(C) 2 (D) 3 s→0
Solution: 
As per problem ( S + 1) 1
kv = Lt S . P = → for P = 1
p(s) = S + a2s + a1s + a0
3 2 S ( S + 2)( S + 3) 6
s→0
derivative of above will be pI(s) = 0 has no real roots, so
∴P =1
it is imaginary complex conjugates. (2nd order system)
It is 3rd order system so remaining one root must be Hence, the correct answer is (0.99 to 1.01).
a real root. 3. Match the inferences X, Y and Z about a system, to the
Hence, the correct option is (B) corresponding properties of the elements of first col-
umn in Routh’s table of the system characteristic equa-
2. The open loop transfer function
tion [2016]
( s + 1)
G ( s) = p X: The system is P: When all elements are
s ( s + 2)( s + 3) stable positive
where p is an integer, is connected in unity feedback Y: The system is Q: When any one element
configuration as shown in the figure. unstable is zero
Z: The test breaks R: When there is a change
G (s) down in sign of coefficients
+
− (A) X → P, Y → Q, Z → R
(B) X → Q, Y → P, Z →R
Given that the steady-state error is 0 for unit step input (C) X → R, Y → Q, Z → P
and is 6 for unit ramp input, the value of the parameter (D) X → P, Y → R, Z → Q
p is __________.  [2017] Solution:  From the RH criterion:
Solution:  X → P
1 Y → R
ess = ( For unit step input ) Z → Q
1+ kp
K p = 1 + G ( S ) H ( s) = ∞( For ess to bee ‘o’) Change in sign shows that system is unstable. The sys-
s→0 tem is stable when all the elements of Routh array are
+ve. When any row of Routh array becomes zero, then
it indicates that breakdown has occurred.
Hence, the correct option is (D).
3.26 | Control Systems

s−2 Also we have,


4. The response of the system G(s) = to the
( s + 1)( s + 3) Vin = V1 + V2
dy
unit step input u(t) is y(t). The value of at t = 0+ is If Vin = V1 : ω1 = 200 rad/sec
dt
_______. [2016] Vin = V2 : ω2 = 500 rad/sec
Solution:  We are given Case (i):
s−2
Y ( s) = Vin = 2cos(200t)
s( s + 1)( s + 3)
Applying the concept of partial function we get
s−2 A B C
= + + V01
s( s + 1)( s + 3) s s + 1 s + 3 2Ω
Z2
V1
After solving above partial fraction, we get

A = –1/3, B = 3/2, C = −5/6
−1 3 5
Y ( s) = 3 + 2 − 6 ⇒ V01 = V1 = 2 cos( 200t ) 
s s +1 s + 3 
Case (ii):
−1 3 − t 5 −3t
y (t ) = + e − e Vin = V2 = 4sin(500t)
3 2 6 
dy(t ) −3 − t 5 −3t Z1
= e + e
dt 2 2 
−3 5 2
⇒ + = =1 2Ω V02
2 2 2 
Hence, the correct Answer is (1). V2

5. In the RLC circuit shown in the figure, the input v­ oltage 2Ω


is given by
Vi(t) = 2 cos (200t) + 4 sin (500t).
V02 = V2
The output voltage V0(t) is [2016] ∴ V0 = V01 + V02.
0.25 H 100 μF
Hence, the correct option is (B).
+ + 6. For the unity feedback conrol system shown in the

10
­figure, the open loop transfer function G(s) is given as
0.4 H
Vi(t) μF V0(t) 2
G(s) = .
2Ω s( s + 1)
– – The steady state error ess due to a unit step input is
 [2016]
(A) cos(200t) + 2sin(500t)
+ e(t)
(B) 2cos(200t) + 4sin(500t) x(t) G(s) y(t)
(C) sin(200t) + 2cos(500t) –
(D) 2sin(200t) + 4cos(500t)
Solution:  The input voltage is
Vin = 2cos (200t) + 4sin(500t) (A) 0 (B) 0.5
From the above relation, (C) 1.0 (D) ∞
ω1 ⋅ t = 200t  ⇒  ω1 = 200 rad/s Solution: Open loop transfer function is
And 2
G(s) =
s ( s + 1)
ω2 ⋅ t = 500t  ⇒  ω2 = 500 rad/sec
Chapter 4  Time Response Analysis  |  3.27

For unity feedback control H(s) = 1 one sign change 1 root in RHS of S-plane and 2 roots
As we know that are on imaginary axis
∴ so only two poles exist in LHS of s-plane
S .R( s)
∴ ess = lt Hence, the correct Answer is (2).
s →0 1 + G ( s) H ( s)
 8. In the circuit shown, the initial voltages across the
1 capacitors C1 and C2 are 1 V and 3 V, respectively. The
Given R(s) = (for unit step input)
s switch is closed at time t = 0. The total energy dissi-
1 1 pated (in Joules) in the resistor R until steady state is
∴ ess = lt = =0 reached, is _____ [2015]
s →0 2 ∞
1+ t=0
s( s + 1)  R = 10 Ω

Hence, the correct option is (A). + +


7. The characteristics equation of an LTI system is given C1 = 3 F C2 = 1 F
– –
by F(s) = s5 + 2s4 + 3s3 + 6s2 – 4s – 8 = 0. The num-
ber of roots that lie strictly in the left half s-plane is
________. [2015] Solution: Vc1 (0 ) = 1 V and VC2 (0) = 3 V
Solution:  Given characteristic equation Redraw the given network in S-domain
F(s) = s5 + 2s4 + 3s3 + 6s2 – 4s – 8 = 0 10 Ω

Applying RH criterion + 1 + 3
– s I – s
s5 1 3 −4 1 1
C1 C2 s
4 3s
s 2 6 8
s3 0( 2) 0(3)
12 − 6 3 −1
s2 −8
2 S 2
I(s) = =
1 9 + 16 10 + 1 + 1 ⎛ 4⎞
s s 3s ⎜10 s + ⎟
3 ⎝ 3⎠ 
s0 −8 i(t) = ILT{I(s)}

s3: row having all zeros 1


= .e −4 30 ; for t ≥ 0
t

5
So A(s) = 0 ∞

∫ i (t ) .R.dt
2
Energy dissipated =
2s4 + 6s2 – 8 = 0 0

s4 + 3s2 – 8 = 0 1 −4 15t
=∫ .e .10.dt= 1.5 J
s4 + 3s2 – 8 = 0 25
0

Hence, the correct Answer is (1.4 to 1.6).


dA ( s )
=0 9. The natural frequency of an undamped second-order
 ds
system is 40 rad/s. If the system is damped with a
⇒ 4s + 26s = 0 3
damping ratio 0.3, the damped natural frequency in
w =0
n rad/s is______. [2014]
2s2 + 3 = 0
Solution: 38.16 rad/sec
and roots of A.E is Let s2 = y given wn = 40 rad/sec
2y2 + 6y – 8 = 0 ζ = 0.3
y2 + 3y – 4 = 0
wd = wn 1 − x 2
y = 1, –4
2
s2 = 1  and  s2 = –4 = 40 1 − 0.3 = 40 1 − 0.09

s = ±1 and s = ±2j and = 40 0.91 = 38.16 rad/sec


3.28 | Control Systems

10. For the second order closed-loop system shown in the


(c)
figure, the natural frequency (in rad/s) is

(a) 16 (b) 4
(d)
(c) 2 (d) 1 [2014]

Solution: (c)
Closed transfer function of the given system
4
C ( s) s( s + 4) 4
= = 2 (i)
R( s) 4 s + 4s + 4
1+
s( s + 4)
[2011]
Standard transfer function Solution: (a)
kwn2 Given differential equation
(ii)
s 2 + 2x wn + wn2 d2 y dy
100 − 20 + y = x (t )
Comparing equation (i) and (ii) dt 2 dt

wn2
=4 Take Laplace transform on both sides

x ( s)
wn = 2 rad/sec 100 s 2 y( s) − 20 s y( s) + y( s) =
s
Hence, the correct option is (c).
1
d2 y
dy y ( s) s
1 1. The differential equation 100 2 − 20 + y . x(t) ⇒ =
dt dt x( s) (100 s 2 − 20 s + 1)

describes a system with an input x(t) and an output y(t).
y ( s) 1 1
The system which is initially relaxed is excited by a ⇒ = =
unit step input. The output y(t) can be represented by x( s) s(10 s − 1)(10 s − 1) s(10 s − 1) 2

the waveform
1 1
∴Poles are = 0, ,
(a) 10 10
Systems are unstable, poles lie on right-hand side
Hence, the correct option is (a).
1 2. Step responses of a set of three second-order under-
damped systems all have the same percentage over-
shoot. Which of the following diagrams represents the
poles of three systems?
(b) (a)
Chapter 4  Time Response Analysis  |  3.29

(b) 6
6
wn2 = ∴ wn =
k k
1
2x wn =
k

6 1
⇒ 2 × 0.5 × =
k k
6 1
(c) ⇒ = 2
k k
1
⇒k=
6
Hence, the correct option is (c).
1 4. If the characteristic equation of a closed-loop system is
s2 + 2s + 2 - 0, then the system is
(a) overdamped
(b) critically damped
(d) (c) underdamped
(d) undamped [2001]
Solution: (c)
s2 + 2s + 2 = 0
∴ s2 + 2ζwn + wn2 = 0
1
2ζwn = 2 x=
wn
2
[2008] wn = 2
Solution: (c) ζ < 1 (underdamped)
Maximum overshoot depends on and Hence, the correct option is (c).
−xp 1−x 2 1 5. For a second-or der system with the closed-loop trans-
∴ mp = e
fer function
where x = cos −1 q 9
T ( s) = the settling time for 2 per cent
∴θ is the angle made by pole from negative real axis. 2
s + 4s + 9
For mp to be same, θ is made same.
band, in seconds, is
Hence, the correct option is (c). (a) 1.5 (b) 2.0
13. Consider a system with the transfer function (c) 3.0 (d) 4.0 [1999]
s+6 . Its damping ratio will be 0.5 when Solution: (b)
G ( s) = 2
Ks + s + 6 9
the value of K is T ( s) = 2
s + 4s + 9
(a) 2/6 (b) 3
(c) 1/6 (d) 6 [2002] ∴ wn2 = 9 ∴ h
Solution: (c)
wn = 3
s+6 s+6
G ( s) = 2 = 2ζwn = 4
k s +s+6  2 s 6
ks + +  ∴ζwn = 2
 k k
4 4
Characteristics equation ∴ ts = = =2
x wn 2
s2 + 2ζwn + wn2 given ζ = 0.5
on comparing Hence, the correct option is (b).
3.30 | Control Systems

16. If F ( s) = w , then the value of lim f (t ) , A A


→∞ ess = = =0
s +w 2 2 1+ kp 1+ ∞

{where F(s) is the L[f(t)]} Hence, the correct option is (a).
(a) cannot be determined (b) is zero
1 9. For a second order system, damping ratio, (ζ), is 0 < ζ
(c) is unity (d) is infinite
< 1, then the roots of the characteristic polynomial are
[1998] (a) real but not equal (b) real and equal
Solution: (a) (c) complex conjugates. (d) imaginary
2  [1995]
F ( s) =
s + w2 2 Solution :(c)

Damping ratio ζ is 0 < ζ < 1
s2 = w2 = 0
For under damped system, roots or poles are
∴s = ± jw
∴ poles are pure imaginary, hence final value of func- s = −x wn ± jwn 1 − x 2
tion cannot be determined. Hence, the correct option is (c).
Hence, the correct option is (a). 2( s + 1)
20. If L[f(t)] = 2
then f(0+) and f(∞) are given by
17. Consider a feedback control system with loop transfer s + 2s + 5
function (a) 0, 2 respectively
K (1 + 0.5s) (b) 2, 0 respectively
G ( s) H ( s) = (c) 0, 1 respectively
s(1 + s)(1 + 2 s)
(d) 2/5, 0 respectively.
The type of the closed loop system is [Note: 'L' stands for 'Laplace Transform of’]
(a) zero (b) one
[1995]
(c) two (d) three
Solution: (b)
[1998]
2( s + 1)
Solution: (b) L[ f (t )] = = F ( s)
s2 + 2s + 5
k (1 + 0.5s)
G ( s).H ( s) = 2s2 + 2s
s(1 + s)( s + 2 s) For f = Lt s F ( s) = Lt
(0 + ) s →∞ s →∞ s 2 + 2s + 5
∴ Type of the system is determined by number of poles
 2
at origin of open loop transfer function. s2  2 + 
 s
In this it is only 1. Lt =2
s →∞  2 5 
 1 + +
s s 2 
Hence, the correct option is (b).

18. Consider a unity feedback control system with open-
s(2 s + 2)
K For f( ∞ )* = Lt s F ( s) = Lt =0
loop transfer unction G ( s) = the s →0 s →0 s 2 + 2s + 5
s( s + 1)
Hence, the correct option is (b).
steady-state error of the system due to a unit step input
is 1
(a) zero (b) K 21. The step error coefficient of a system
with unity feedback is ( s + 6)( s + 1)
(c) 1/K (d) infinite [1998]
(a) 1/6 (b) ∞
Solution: (a) (c) 0 (d) 1
K [1995]
G ( s) =
s( s + 1) Solution: (a)

Unity feedback H(s) = 1 1
G ( s) =
k p = Lt G ( s) H ( s) ( s + b)( s + 1)
s →0

Unity feedback H(s) = 1
k
= Lt =∞ Lt G ( s) H ( s)
s →0 s( s + 1) →

Hence, the correct option is (a).
Chapter 4  Time Response Analysis  |  3.31

22. The final value theorem is used to find the (A) Four left half plan (LHP) roots
(a) steady-state value of the system output (B) One right half plane (RHP) root, one LHP root and
(b) initial value of the system output two roots on the imaginary axis.
(c) transient behaviour of the system output (C) Two RHP roots and two LHP roots.
(d) none of these. [1995] (D) All four roots are on the imaginary axis.
Solution: (a) Solution:  S4 1 1 1
The final value theorem is used to find the steady-state S23 0 0 0
value of the system. S2 1/2 1 0
23. The poles of a continuous time oscillator are…… S1  -6 0 0
[1994] S0 1 0 0
Solution: Pure imaginary. No. of sign changes = 2
The poles of a continuous time oscillator are pure No. RHP = 2
imaginary.
No of symmetric with respect of origin = 4
\ No of LHP = 2
Two-marks Questions No of imaginary axis Poles = 0
1. For a unity feedback control system with the forward Hence, the correct option is (C).
K
path transfer function G(s) = . 3. In the circuit shown, the voltage VIN (t) is described by:
s( s + 2)
The peak resonant magnitude Mr of the closed-loop  0, for t < 0
VIN (t ) 
frequency response is 2. The corresponding value of 15 volts, for ≤ 0
the gain K (Correct to two decimal places is _______.
where t is in seconds. The time (in seconds) at which the
 [2018]
current I in the circuit will reach the value 2 Amperes is
Solution:  The transfer function is given as ___________. [2017]
K
G(s) =
s( s + 2) 1Ω I
⎛ K ⎞ 2 1
CLTF = ⎜ 2 ⎟; ξ = = +
⎝ s + 2s + K ⎠ 2 K K 
peak resonant magnitude can be calculated using VIN (t) 1H 2H
1
Mr = =2
2ξ 1 − ξ 2  −
1
=2 Solution: From the given data
⎛ 1 ⎞ 1 For t < 0
2⎜ ⎟ 1 −
⎝ K⎠ K
Vin (t ) = 0 V, so I (0 − ) = 0 amp
⎛ 1 ⎞ K −1 For t > 0:-
1 = 4 ⎜ ⎟
⎝ K⎠ K At t = 0+, I (0+) = I (0-) = 0A

Simplifying the above expression we get As t  →∞

K = 4 K − 1  i(∞) = 15 A.
K2 = 16 (K – 1) 2
Leq = (2 || 1) = H
K2 – 16K + 16 = 0 3
R =1Ω
K = 14.9282 x K = 1.071
L 2
K = 14.9282 is valid. ∴ τ = = sec
R 3
Hence, the correct answer is 14 to 17. ∴ is (t ) = i(∞) + {i(0) − i(∞)}.e − t /τ
2. Which one of the following options correctly describes i(t ) = 15[1 − e −1.55t ]
the locations of the roots of the equation s4 + s2 + 1 = 0
on the complex plane? [2017]
3.32 | Control Systems

3
1× is(t )
but I (t ) =
1+ 2
= 5.[1 − e −1.5t ] Amg ∑ ak cos ( kωot + ϕk )
(C)
k =1
2
given i(t) = 2 A Then t = ?
2 = 5 [I – e-1.5t]
∑ ak cos ( kωot + ϕk )
(D)
k =1
e-1.5 t = 0.6 Solution:  If R, L, C are in series, and unit values R = 1,
t = 0.34 sec. L = 1, C = 1 then the transfer function of the system
Hence, the correct answer is (0.3 to 0.4). 1 1
4. A unity feedback control system is characterized by the H(s) = R + sL + = 1+ s +
Cs s
2( s + 1)
open-loop transfer function G ( S ) = 3 s + s2 + 1 s2 + s + 1
s + ks 2 + 2 s + 1 = =
s s 
The value of k for which the system oscillates at 2 rad/s
is ____________. [2017] At ω = kωo, H( jω) will be in the form of M < ϕ
Solution:  Given, open-loop transfer function y(t) = H( jω) x(t)
2( s + 1)
G (S ) = 3
3

s + Ks 2 + 2 s + 1 = ∑ ak M ⋅ cos ( kωo t + ϕ ) 
k =1
\ Characteristic equation is 1 + G (S) = 0
2( S + 1) 3
1+ 3 =0 = ∑ bk M .cos ( kωo t + ϕ )
S + KS 2 + 2S + 1 k =1
⇒ S 3 + KS 2 + 4 S + 4 = 0
Where ak ≠ bk.
Form Routh Table
Hence, the correct option is (A).
4
S3 1 6. A first order low pass filter of time constant T is excited
with different input signals (with zero initial conditions
3 up to t = 0), Match the excitation signals X, Y, Z with
S2 K
the corresponding time responses for t ≥ 0: [2016]
4K − 3 0 −
t
S1 X: Impulse P: 1 – e T
K
0 ⎛ −t

S 3 0 Y: Unit step Q: t – T ⎜1− e T ⎟
For the system to oscillate at ωmar = 2 rad/s, all S1 row ⎝ ⎠
−t
of elements are to be zero.
Z: Ramp R: e T
4 K mar − 3
∴ =0 (A) X → R, Y → Q, Z → P
K mar (B) X → Q, Y → P, Z → R
3 (C) X → R, Y → P, Z → Q
⇒ K ma r = = 0.75 (D) X → P, Y → R, Z →Q
4
Hence, the correct answer is (0.75). Solution:  r(t) c(t)
h(t)
5. A network consisting of a finite number of linear resis-
tor (R), inductor (L) and capacitor (C) elements, con- c(t) = r(t) ⋅ h(t)
nected all in series or all in parallel, is excited with a
source of the form. [2016] C(s) = R(s) ⋅ H(s)
3
X → R, Y → P, Z → Q
∑ ak cos (kω 0t )
k =1 Hence, the correct option is (C).
where ak ≠ 0, ω0 ≠ 0 7. The open-loop transfer function of a unity feedback
The source has nonzero impedance. Which one of the control system is given by
following is a possible form of the output measured
K
across a resistor in the network? G(s) = .
3
s ( s + 2)
∑ bk cos ( kωot + ϕk )
(A) where bk ≠ ak, ∀ k
For the peak overshoot of the closed loop system to a
k =1  
4 unit step input to be 10% the value of K is _______.
∑ bk cos ( kωot + ϕk )
(B) where bk ≠ 0, ∀ k  [2016]
k =1  
Chapter 4  Time Response Analysis  |  3.33

Solution: 
Transfer function is given as 1
9. In the feedback system shown below G(s) = .
G(S) =
K (s 2
+ 2s )
S ( S + 2) The step response of the closed loop system should
− ξπ have minimum settling time and have no overshoot.
1− ξ2
Given Mp% = e × 100  +
r k G (S) y
−ξπ
= ln (0.1) –
1 − ξ2

The characteristic eqn of this system is 1 + G(s)H(S) = The required value of gain k to achieve this is ___
0
 [2016]
i.e. s2 + 2s + k = 0

comparing it with standard equation Y ( s) k


Solution:  = 2
R( s) s + 2 s + k
s2 + 2ξωn s + ωn2 = 0
The characteristics equation can be given as
we get, the natural freq and damping ratio as given S2 + 2s + K
below
Comparing this equation with standard equation,
ξ = 0.5911 = 0.6
we get,
2ξ ωn = 2
S2 + 2ζωns +ωn2 = 0.
2 × 0.6 × ωn = 2
Minimum settling time and no over shoot occurs at
ωn = 1.66 critical case.
2ζωn = 2
K = ω n2 = K = 1.66 2 = 2.77. 
ζ = 1
Hence, the correct Answer is (2.77).
ωn = 1 rad/sec
8. The transfer function of a linear time invariant system
is given by k = ω n2 = 1 

H(s) = 2s4 – 5s3 + 5s – 2
k = 1
The number of zeros in the right half of the s-plane is
Hence, the correct Answer is (1).
_______. [2016]
1 0. Assume that the circuit in the figure has reached the
Solution:  Transfer function is
steady state before time t = 0 when the 3 Ω resistor
H(s) = 2s4 – 5s3 + 5s – 2 suddenly burns out, resulting in an open circuit. The
current i(t) (in ampere) at t = 0 is ____. [2016]
Apply RH criteria
3F 2Ω
S4 2 0 -2 1Ω
S3 -5 5
i(t)
S 2
2 -2 12 V +

S 1
0(4) 0 2F 3Ω

S0 -2

3 – sign changes hence 3 roots on right half of s – plane. Solution:  From the given data, the circuit is in steady
state before t = 0.
From the Routh array, we observed that one row is
completely zero, which shows that break down has At t = 0–, circuit is in steady state so in steady state
occurred and there are 3 sign changes in the 1st column capacitors open circuit.
indicating the instability.
Hence, the correct Answer is (3).
3.34 | Control Systems

i(o) S3 1 2k + 3
VC +
1 2Ω S2 2k 4

1Ω 2Ω
S1 2k(2k + 3) – 4 0
12 V + I=0 2k
– +
VC – 3Ω
2
S0 4

The system will only be stable when all the values of


column 1 are positive, i.e., >0
We know the first k > 0 and
12 2k(2k + 3) – 4 ≥ 0
i( 0 − ) = = 2A
6  4k2 + 6k – 4 ≥ 0
∴ Vc1(0–) = 2 × 2 = 4 V But for marginal stable system
4k2 + 6k – 4 = 0
Vc2(0–) = 3 × 2 = 6 V
or 2k2 + 3k – 2 = 0
But at t = 0+; Vc (0–) = Vc(0+)
2k2 + 4k – k – 2 = 0
for t > 0:
Redraw the given circuit 2k {k + 2} – 1 {k + 2} = 0
k + 2 = 0
i(o+)
and
4Ω 2k – 1 = 0

+ 4V

k = 1/2
i(t) and
12 V +

k = –2
+ 6V
– But k always positive so for stable system k > 0 and k >
0.5
∴ 0.5 < k < ∞
Hence, the correct option is (D).
12. A unity negative feedback system has an open-loop
10 − 6 K
i(t) = amp transfer function G(s) = . The gain K for the
4  s ( s + 10 )
=
1 amp system to have a damping ratio of 0.25 is_______.
 [2015]
Hence, the correct option is (1 amp).
K
1 1. The first two rows in the Routh table for the characteris- Solution:  Given OLTF G(s) =
s ( s + 10 )
tic equation of a certain closed loop control system are
given as: [2016] Characteristic equation s2 + 10s + K = 0
S3 1 (2K + 3) Given ζ = 0.25
2 ζωn = 10
S2 2K 4
ωn = K 
The range of k for which the system is stable is
0.5 ⋅ ωn = 10
(A) –2.0 < K < 0.5 (B) 0 < k < 0.5
ωn = 20
(C) 0 < k < ∞ (D) 0.5 < K < ∞
Solution:  The range of k can be easily obtained using K = 20 

routh array and is used as under K = 400
Applying RH criterion to given table Hence, the correct Answer is (400).
Chapter 4  Time Response Analysis  |  3.35

13. For the following feedback system Solution :  (0.49 to 0.52)


1 ess = Lt s E ( s)
G ( s) = . The 2%-settling time of the step s →0
( s + 1)( s + 2)
s.R( S )
response is required to be less than 2 seconds. ess = Lt
s →0 1 + G ( s) H ( s)

Here G ( s) = k = 4 ; H ( s) = 2 ;
s+2 s+2 s+4
1
R( s) =
Which one of the following compensators C(s) achieve s
this?
1
 1   0.03  s.
(a) 3  (b) 5 + 1 s
 s + 5   s  ess = Lt
s →0 4 2
1+ ×
 s+8 s+2 s+4
(c) 2(s + 4) (d) 4  
 s+3
1 1 1
 [2014] = = =
4 2 1+1 2
Solution: (c) 1+ ×
2 4
1
G ( s) = ∴ess = 0.5
( s + 1)( s + 2)
Hence, the correct option is (c).
H(s) = 1 15. The characteristic equation of a unity negative feed-
G ( s) 1 back system is 1 + KG(s) = 0. The open loop transfer
∴ T ( s) = = function G(s) has one pole at 0 and two poles at -1. The
1 + G ( s) H ( s) 1 + ( s + 1)( s + 2)
root locus of the system for varying K is shown in the
1 figure.
T ( s) = 2  (i)
s + 3s + 3
Standard transfer function
kwn2
(ii)
s 2 + 2x wn s + wn2

Comparing equation (1) with (ii)


2ζwn = 3
3
x wn = = 1.5
2
4 4
Settling time t s = = = 2.67
x wn 1.5
which is greater than 2 sec to make settling time (ts)
less than 2 sec, PD controller can be used
∴comparing all option, option (c) is correct.
The constant damping ratio line, forζ = 0.5, intersects
Hence, the correct option is (c). the root locus at point A. The distance from the origin
14. The steady-state error of the system shown in the figure to point A is given as 0 5. The value of K at point A
for a unit step input is_____. [2014] is______. [2014]
Solution: (0.31 to 0.41)
According to the question
k
G ( s) H ( s) = (i)
s( s + 1) 2
given ζ = 0.5
3.36 | Control Systems

As ζ = cos θ Solution: (c)


∴ = cos−1ζ = cos−10.5 = 60° w ( s) 10
=
V0 ( s) 1 + 10 s

1 + Ts = 1 + 10s
∴Ts= 10s
∴Time constant of open loop system = 10
T
Time constant of closed loop T ′ =
100
10 1
=
100 10
10
ka ×
W ( s) 1 + 10 s
=
R( s) 10
1 + ka
Let the co-ordinate of point A be (−a + jb). 1 + 10 s
Distance from origin to point A is 0.5 W ( s) 10 ka 10 ka
⇒ = =
∴OA = 0.5 R( s) 1 + 10 s + 10 ka 10 s + (1 + 10 ka)

OB
∴ cos 60° = W ( s) 10 ka
OA ⇒ =
R( s)  10 s 
(1 + 10 ka)  1 + 
∴OB = 0.25  (1 + 10 ka) 

Similarly
10 1
BA T′ = =
sin 60° = 1 + 10 ka 10
OA
= 1 + 10ka = 100
∴BA = jb = 0.433j
10ka = 99
Co-ordinates of A = −0.25 + 0.43j
99
∴As |G(s) H(s)| = 1 (ii) ka = = 9.9 ≅ 10
10
s = −0.25 + 0.43j
subvalues of s in equation (ii) Hence, the correct option is (c).

k 17. The unit step response of an under-damped second


∴ =1 order system has steady-state value of -2. Which one
s( s + 1) 2 of the following transfer functions has these properties?

∴ k = 0.37 −2.24
(a) 2
1 6. The open-loop transfer function of a dc motor is given s + 2.59 s + 1.12

w( s) 10 −3.82
as = . When connected in feedback as (b) 2
Va ( a) 1 + 10 s s + 1.91s + 1.91
shown below, the approximate value of Ka that will −2.24
reduce the time constant of the closed loop system by (c) 2
s − 2.59 s + 1.12
one hundred times as compared to that of the open-loop
system is −3.82
(d) 2
 [2009]
s − 1.91s + 1.91

Solution: (b)
Given steady-state value = −2
∴transfer function should be under damped for these
(a) 1 (b) 5 properties.
(c) 10 (d) 100 [2013]
Chapter 4  Time Response Analysis  |  3.37

From the given options (3) y(t)


solve for all transfer function to find ζ (damping ratio
−3.82 1
2
s + 1.918 + 1.91
2
∴ wn = 1.91 ∴ wn ≅ 1.4
2ζwn = 1.91
1.91
x= <1
2.8 t

Hence, the correct option is (b).


y(t)
18. Group I lists a set of four transfer functions. Group (4)
II gives a list of possible step responses y(t). Match
1
the step responses with the corresponding transfer
functions.
Group-I
25
P= 2
s + 25
36 t
Q= 2
s + 20 s + 36
(a)
P−3, Q−1,
y(t) R−4, S−2
36 (b)
P−3, Q−2, R−4, S−1
R=
s 2 + 12 s + 36 (c)
P−2, Q−1, R−4, S−3
(d)
P−3, Q−4, R−1, S−2 [2008]
36
S=
s 2 + 7 s + 49 wn2
Solution:
1 (d)  G ( s) =
s 2 + 2x wn s + wn2
Group-II
(1) 25
So p = 2
s + 25
t
∴2 ζwn = 0 ∴ P is Undamped
∴ζ=0
y(t) 36
For Q = 2
1 s + 20 s + 36
∴ wn2 = 36 ∴wn = 6

2ζ wn = 20
(2) 20
x= = 1.67
2×6
t
∴Q is over damped

For R = 36
2
s + 12 s + 36

∴ wn2 = 36 wn = 6

2ζwn =12
12
x= =1
2×6
3.38 | Control Systems

∴R is critically damped  −px 


49 overshoot, M p = exp   × 100 which one of
For s =  1− x 2 
s 2 + 7 s + 49  
the following conditions is NOT required?
∴ w 2 = 49 wn = 7
n (a) System is linear and time invariant
(b) The system transfer function has a pair of complex
7 conjugate poles and no zeroes
x= = 0.5
2×7 (c) There is no transportation delay in the system

(d) The system has zero initial conditions [2005]
∴s is under damped.
Solution: (c)
Hence, the correct option is (d).
There is no transportation delay in the system.
1 9. The magnitude of frequency response of an under
22. A ramp input applied to a unity feedback system results
damped second order system is 5 at 0 rad/sec and peaks
in 5% steady-state error. The type number and zero fre-
10 quency gain of the system are, respectively,
to at 5 2 rad/sec. The transfer function of the
3 (a) 1 and 20 (b) 0 and 20
(c) 0 and 1/20 (d) 1 and 1/20
system is
 [2005]
500 375
(a) (b) 2 Solution: (a)
2
s + 10 s + 100 s + 5s + 75
E ( s) = R( s) where 1
720 1125 R( s) = 2
(c) 2 (d) 1 + G ( s) s
s + 12 s + 144 2
s + 25s + 225
∴ ess = Lt s.E ( s)
 [2008] s →0

Solution (a): R( s)
= Lt s.
Only (a) satisfies the given conditions. s →0 1 + G ( s)
2 0.
The transfer function of a plant is
1
= Lt given (finite)
5 s →0 s(1 + G ( s))
T ( s) = . The second-order approxi-
( s + 5)( s 2 + s + 1) ∴ kv = Lt s G ( s)
s →0
mation of T(s) using dominant pole concept is
1 5 1 1
(a) (b) ess = Lt = 5% =
( s + 5)( s + 1) ( s + 5)( s + 1) s →0 s G ( s) 20

5 1 ∴k = 20; kv is finite for type 1 system


(c) (d)  [2007]
2
s + s +1 2
s + s +1 Hence, the correct option is (a).
23. A causal system having the transfer function
Solution: (d)
5 1 is excited with 10 u(t). The time at which
T ( s) = H ( s) =
( s + 5)( s + s + 1) 2 s+2

the output reaches 99% of its steady state value is
In dominant pole concept, time constant factor has to (a) 2.7 sec (b) 2.5 sec
be eliminated. (c) 2,3 sec (d) 2.1 sec [2004]
5 Solution: (c)
∴ T ( s) =
 s 1
5 1 +  ( s 2 + s + 1) H ( s) =
 5 s+2

1 r(k) = 10u(t)
=
s2 + s + 1 10
∴ R( s) =
Hence, the correct option is (d). s
2 1. In the derivation of expression for peak per cent
∴Output C ( s) = H ( s).R( s) = 1 . 10
s+2 s
Chapter 4  Time Response Analysis  |  3.39

Using partial fraction (c)


10 A B
= +
s( s + 2) s + 2 s

10 = AS + B(s + 2)
⇒ Let s = 0
10 = 2B∴ B = 5
Let s = −2
10 = −2A∴ A = −5
−5 5
∴ C ( s) = +
s+2 s (d)

∴ C(t) = 5(1 − e−2t)
At t = 0 (steady-state value)
C(t) = t(1 – e0) = 5
To reach 99% of steady-state value
0.99 × 5 = 5[1 − e−2t]
⇒ 1 − e−2t = 0.99
⇒ e−2t = 0.1
⇒ −2t = ln 0.1
⇒ −2t = −4.6
⇒ t = 2.3 sec [2003]
Hence, the correct option is (c). Solution: (b)
2 4. A second-order system has the transfer function given C ( s) = 4
2
R( s) s + 4 s + 4
C ( s) 4
= . With r(t) as the unit-step function, ∴ wn = 4 ∴ wn = 2
2
R( s) s 2 + 4 s + 4
the response c(t) of the system is represented by 2ζwn = 4
(a) ∴ζ = 1 (critical damping)
4 4
Settling time = t s = = = 2sec
x wn 1× 2

Hence, the correct option is (b).

25.
The transfer function of a system is
100 . For a unit-step input to the sys-
G ( s) =
( s + 1)( s + 100)
tem the approximate settling time for 2% criterion is
(b) (a) 100 sec (b) 4 sec
(c) 1 sec (d) 0.01 sec [2002]
Solution: (b)
100
G ( s) =
( s + 1)( s + 100)

Taking dominant pole consideration, s = −100 pole is
100
not taken. G ( s) = , it is a 1st order system
s +1
3.40 | Control Systems

(a) ae (b) (1/a)(1 e-at)


∴ t s = 4t = 4 × 1 = 4 sec (c) a(1 e-at) (d) 1 e-at [1998]
Hence, the correct option is (b). Solution: (b)
2 6. If the closed-loop transfer function T(s) of a unity nega- at
tive feedback system is given by e u(t )
 1  1 1
 0  R( s) = ∴ H ( s) =
−1 1  s + 1 0   ( s + 1) 
s + a s
 SI − A = =
( s + 1) ( s + 1)  1 s + 1  1 1  ∴ C(s) = R(s) .H(s)

 ( s + 1)
2
( s + 1)  1 1
C ( s) = .
( s + a) s

 1 
 0  Solving by partial fraction
−1 1  s + 1 0   ( s + 1) 
I − A = = 1 A B
( s + 1) ( s + 1)  1 s + 1  1 1 

= +
s( s + a ) s + a s
 ( s + 1)
2
( s + 1) 
⇒1 = AS + B(s + a) Let s = 0
then the steady-state error for a unit ramp input is
1
(a) {
f (t ) = e At = L−1 ( SI − A) } ∴2B = 1 ∴B =
a

 e −t Let s = −a
0 
e at = 
(b)  −1
te −t
e −t  −aA = 1 ∴A=
a
 x1   −1 1 0   x1   0 
        1 1 1  1
 x2  =  0 −1 0  +  x2  +  4  ∴ C ( s) = − ∴ C (t ) = [1 − e − at ]
 x3   0 0 −2   x3   0  a  s s + a  a

(c)
 x1  Hence, the correct option is (b).
   [1999] 2 8. The response of an LCR circuit to a step input is
u; y = 1 1 1  x2 
(a) Overdamped
 xs3  (b) Critically damped
Solution: (d) (c) Oscillatory
 x1   −1 1 0   x1   0  If the transfer function has
       (1) poles on the negative real axis
 x2  =  0 −1 0   x2  +  4  4
 x   0 0 −2   x3   0  (2) poles on the imaginary axis
 3  (3) multiple poles on the positive real axis
 −1 1 0  (4) poles on the positive real axis
  (5) Multiple poles on the negative real axis.
∴ A =  0 −1 0 
 [1994]
 0 0 −2 
Solution: (a) overdamped → 1 (poles on the negative
For type 2, Ramp input real axis)
kv = Lt s G ( s) k
s →0 =1
s( s + 1)

kv = ∞ (b) critically damped → 5 (multiple poles on the nega-
1 tive real axis.
ess = =0
kv ζ=1

(c) oscillatory → 2 (poles on the imaginary axis)
Hence, the correct option is (d). ξ=0
2 7. The unit impulse response of a linear time invariant s = ± jwn
system is the unit step function u(t). For t > 0, the
Hence, the correct option is (a).
response of the system to an excitation e-at u(t), a > 0
will be
Chapter 4  Time Response Analysis  |  3.41

29. Match the following codes with List-I with List-II: Solution: (a)
List-I 4(1 + 2 s)
G ( s) =
(a) Very low response at very high frequencies s 2 ( s + 2)
(b) Overshoot
(c) Synchro-control transformer output Unit ramp i.e. H(s) = 1
List-II kv = Lt sG ( s) H ( s)
s →0
(1) Low pass systems
(2) Velocity damping s 4(1 + 2 s)
= Lt
(3) Natural frequency s →0 s 2 ( s + 2)
(4) Phase-sensitive modulation
(5) Damping ratio. [1994] 4(1 + 2 s)
= Lt =∞
Solution: s →0 s( s + 2)
(a) very low response at very high frequencies → low
pass system. A A
ess = = =0
(b) over shoot → damping ratio kV ∞
(c) synchro-control transformer output → phase-sen-
Hence, the correct option is (a).
sitive modulation.
3 2. The steady-state error of a stable 'type 0' unity feedback
Solution :(c)
system for a unit step function is
Damping ratio ζ is 0 <ζ< 1 1
(a) 0 (b)
For under damped system, roots or poles are 1+ KP
Hence, the correct option is (a). 1
(c) ∞ (d)
30. A second-order system has a transfer function given by KP
x1 (t ) + 2 x1 (t ) = u(t )
[1990]
If the system, initially at rest, is subjected to a unit
Solution: (b)
step input at t = 0, the second peak in the response will
occur at The steady-state error of a stable ‘type 0’ unity feed-
(a) πsec (b) π/3sec 1
back system for a unit step function is .
(c) 2π/3 sec (d) π/2 sec. [1991] 1+ kp
Solution: (a) Hence, the correct option is (b).
25
G ( s) =
s 2 + 8s + 25 33. A critically damped, continuous-time, second order
2 system, when sampled, will have (in Z domain)
∴ wn = 25 (a) a simple pole
wn = ± 5 (b) double pole on real axis
2ζwn = 8 (c) double pole on imaginary axis
(d) a pair of complex conjugate poles. [1988]
ζ = 8 = 0.8
Solution: (b)
wd = wn 1 − x 2 = 5 1 − (0.8) 2 = 3 2nd order system when sampled
For 2nd peak n = 3 ζ=1
np 3p ∴s = −ζwn, −ζwn
tp = = = p sec
wd 3 ∴s = −wn, −wn

Hence, the correct option is (b).
Hence, the correct option is (a).
3 1. A unity-feedback control system has the open-loop 3 4. The unity feedback system shown in the figure has:
transfer function a1 = 0, a2 ≠ 0, a3 ≠ 0,. If the input to
the system is a unit ramp, the steady-state error will be
(a) 0 (b) 0.5
(c) 2 (d) Infinity [1991]
3.42 | Control Systems

(a) Zero steady-state position error (b) from figure


(b) Zero steady-state velocity error −πξ
overshoot = = 0.1
(c) Steady-state position error K units e
1−ξ 2
10
K ⇒ −πξ = 1 − ξ 2 ln (0.1)
(d) Steady-state velocity error units [1987]
10
⇒ (πξ ) 2 = (1 − ξ 2 )( 2.3) 2
Solution: (a)
k ( 2.3) 2
given G ( s) = ⇒ξ2 = = 0.349
s( s + 10) π 2 + ( 2.3) 2

H(s) = 1
⇒ ξ = 0.349 = 0.5907
∴ k p = Lt G ( s) H ( s)
s →0
Now comparing (1) with second order characteris-
k tics equation
= Lt =∞
s →0 s( s + 10) ⇒ s 2 + 2ξωn s + ωn2 = s 2 + 3s + G

A A ⇒ ωn = G
∴ ess = = =0
1+ kp 1+ ∞
⇒ 2ξωn = 3,
Hence, the correct option is (a). ⇒ 2 × 0.5907 × G = 3

G = 6.45
Five-marks Questions
(c) New value of
1. The block diagram of a feedback system is shown in G ≈ 13
the figure. ωn = 12.96 = 3.6
2ξωn = 3   (from equation (1))
3 3
ξ= = = 0.42
2ωn 2 × 3.6

ωd = ωn 1 − ξ 2

ωd = 3.6 1 − (0.42) 2

ωd = 3.27 rad/sec

ωd = = 3.27
T

Time period(T) = = 1.92 sec
(a) Find the closed loop transfer function. 3.27
(b) Find the minimum value of G for which the step 2. For figure shows the block diagram representation of a
response of the system would exhibit an overshoot, control system. The system in block A has in impulse
as shown in figure. response hA(t) = e-tu(t). The system in block B has an
(c) For G equal to twice this minimum value, find the impulse response.
time period T indicated in the figure. T = 1.96 sec.
 [2000] hB(t) = e-2tu(t). The block 'k' is an amplifier by a factor k.
For the over all system the input is x(t) and output y(t).
Solution:
(a) Close loop transfer function
G
s( s + 3) G
H ( s) = G
= 2  (1)
1+ s + 3s + G
s( s + 3)
Chapter 4  Time Response Analysis  |  3.43

y(S ) (b) when K = 0


(a) Find the transfer function when k = 1.
x(S ) Impulse response H(s)
(b) Find the impulse response when k = 0. 1
(c) Find the values of k for which the system becomes ∴ H ( s) =
( s + 1)( s + 2)
unstable. [1997]
take partial fraction
Solution:  Impulse response hA(t) = e−tu(t) of Block A
1 A B
1 = +
H A ( s) = ( s + 1)( s + 2) ( s + 1) ( s + 2)
s +1
Impulse response of Block B (hB(t) = e−2t(ut) Let S = −1
1 1 = A ∴A = 1
H B ( s) = Let S = −2
s+2
1 = −B ∴B = −1
(a) given k = 1
1 1
Y ( s) G ( s) ∴ H ( s) = −
T.F. = = s +1 s + 2
X ( s) 1 + G ( s).H ( s)
h(t) = e−tu(t) – e2tu(t)
where G(s) = HA(s) . HB(s) = (e−t − e−2t)u(t)
1 1 1 (c) For system to table
= × =
s + 1 s + 2 ( s + 1)( s + 2) 1 + GH = 0
H(s) = 1 1
∴1 + .k = 0
1 ( s + 1)( s + 2)
Y ( s) ( s + 1)( s + 2)
∴ = s2 + 3s + 2 + k = 0
X ( s) 1 1
1+ . .1 using Rough Hurwitz criterion
( s + 1) ( s + 2)
s2 1 2+k
1 1 s 3 0
1
= =
( s + 1)( s + 2) + 1 s 2 + 2 s + s + 2 + 1 s0 2 + k

for stable system
∴ Y ( s) = 1
X ( s) s 2 + 3s + 3 2+k>0
k > −2
for unstable system
k < −2
Chapter 5
Stability Analysis
One-mark Questions
1. The Nyquist stability criterion and the Routh criterion
both are powerful analysis tools for determining the
w
stability of feedback controllers. Identify which of the
following statements is FALSE:  [2018]
(A) Both the criteria provide information relative to

the stable gain range of the system
(B) The general shape of the Nyquist plot is readily
obtained from the Bode magnitude plot for all
minimum-phase systems.
(C) The Routh criterion is not applicable in the condi-
tion of transport lag, which can be readily handled
by the Nyquist criterion. Using Root locus method, the break point can be
(D) The closed-loop frequency response for a unity obtained as
feedback system cannot be obtained from the ⇒1 + G(s) = 0
Nyquist plot.
k
⇒ 1+ =0
Solution:  The closed-loop frequency response for ( s + 2)( s − 1)
a unity feedback system cannot be obtained from the
Nyquist plot. or k = −(s + 2)(s – 1)
Hence, the correct option is (D) dk
= −2 s − 1 = 0 or s = −0.5
ds
2. The forward path transfer function of a unity negative
K To have both the poles at the same direction
feedback system is given by G ( s) =
( s + 2)( s − 1) |G(s)|s = 0.5 = 1
The value of K which will place both the poles of k = 2.25.
the closed-loop system at the same location is ____. 3. If the closed-loop transfer function of a control system
 [2014] s−5
is given as T ( s) = , then it is
Solution: ( s + 2)( s + 3)
k (a) an unstable system
Given that, G ( s) = (b) an uncontrollable system
( s + 2)( s − 1)
(c) a minimum phase system
(d) a non-minimum phase system [2007]
Chapter 5  Stability Analysis  |  3.45

Solution: (d) (a) will always be unstable at high frequency


Due to the location of zero at right half, the system is a (b) will be stable for all frequency
non-minimum phase system. (c) may be unstable, depending on the feedback factor
(d) will oscillate at low frequency [2000]
Hence, the correct option is (d)
Solution: (c)
4. The open-loop transfer function of a unity-gain feed-
K For a Resistive Network feedback factor is always
G ( s) = greater than unity. So, overall gain will decrease a feed-
( s + 1)( s + 2)
back control system is given by back may lead unstability F NOT Properly applied
The gain margin of the system in dB is given by Hence, the correct option is (c)
(a) 0 (b) 1 8. The gain margin (in dB) of a system having the loop
(c) 20 (d) ∞ [2006]
2
Solution: (d) transfer function G ( s) H ( s) = is
s( s + 1)
For 2nd order system G.M. = ∞ (a) 0 (b) 3
Hence, the correct option is (d) (c) 6 (d) ∞ [1999]
5. The gain margin for the system with open-loop transfer Solution: (d)
2(1 + s) The given system is a second-order system so its gain
function G ( s) H ( s) = is
s2 margin is infinity.
(a) ∞ (b) 0 Hence, the correct option is (d)
(c) 1 (d) -∞ [2004]
9. The phase margin (in degrees) of a system having the
Solution: (d) s
loop transfer function G2 ( s) H ( s) = is
∠G ( s) H ( s) = −180 + tan −1 ( w ) 2
s + as + b

−1
(a) 45° (b) -30°
for, wf = −180 + tan w = −180 (c) 60° (d) 30° [1999]
w=0 Solution: (d)
2 1+ w2 The gain cross over frequency is defined (wg), where
So = G ( s) H ( s) = = ∞. gain is 1,
w2
2 3 2 3
1 So, = 1 or =1
G.M. = = 0 in db jw (1 + jw )
∞ w 1+ w2
Hence, the correct option is (d) Solving,
−1 −1
∠G ( jw ) H ( jw ) = −90° − tan w = −90 − tan
6. The phase margin of a system with the open-loop trans- 3
(1 − s)
fer function G ( s) H ( s) = is ∴PM = 180° - 150 = +30°.
(1 + s)(2 + s)
Hence, the correct option is (d)
(a) 0 (b) 63.4°
1 0. The number of roots of
(c) 90° (d) ∞ [2002]
s3 + 5s2 + 7s + 3 = 0 in the left half of the s-plane is
Solution: (d)
(a) zero (b) one
wg is where |G(s) H(s)| = 1 (c) two (d) three [1998]
1− s 1+ w2 Solution: (d)
So, = =1
(1 + s)(2 + s) 1+ w2 4 + w2 Using R-H criteria
2
s3 1 7
⇒ 4+ w =1 s2 5 3
S 6.4 0
w2 = −3 imaginary, so no gain cross over fg or
s0 3
P.M. = ∞
There is no sign change in 1st column, so there is no
Hence, the correct option is (d)
roots that lie in RHS of s-plane, or all three roots die in
7. An amplifier with resistive negative feedback has two left half of s-plane.
left half plane poles in its open-loop transfer function.
Hence, the correct option is (d)
The amplifier
3.46 | Control Systems

11. The open loop transfer function of a unity feedback +


G (s)
2s2 + 6 s + 5
open loop system is .
( s + 1) 2 ( s + 2) −

The characteristic equation of the closed loop


system is For the closed loop system shown, the root locus for 0
(a) 2s2 + 6s + 5 = 0 < K < ∞ intersects the imaginary axis for K = 1.5. The
(b) (s + 1)2(s + 2) = 0 closed loop system is stable for
(c) 2s2 + 6s + 5 + (s + 1)2(s + 2) = 0 (A) K > 1.5         (B)  1 < K < 1.5
(d) 2s2 + 6s + 5 - (s + 1)2(s + 2) = 0 [1998] (C) 0 < K < 1    (D)  no positive value of K
Solution: (c) Solution: 
Given from C .E = S 2 − 3S + 2 + K ( S 2 + 2S + 2) = 0
2s2 + 6s + 5 + (s + 1)2(s + 2) = 0, C .E = S 2 (1 + k ) + S ( 2k − 3) + 2 + 2k = 0
characteristics equation is given by (1 + k ) > 0 ⇒ k > −1
1 + GH = 0. 3
( 2k − 3) > 0 ⇒ k >
2
2s2 + 6 s + 5
So, 1 + =0 ( 2 + 2k ) > 0 ⇒ k > −1
( s + 1) 2 ( s + 2)
or, 2s2 + 6s + 5 + (s + 1)2(s + 2) = 0.
Hence, the correct option is (c)

Two-marks Questions −1 3/2 K

1. Consider a unity feedback system. as in the figure 3


∴k > , i.e., k > 1.5
K 2
shown, with an integral compensator and open-loop
transfer function s
Hence, the correct option is (A).
1 3. A plant transfer function is given as G(s) =
G(s) = 2
s + 3s + 2 ⎡ KI ⎤ I
⎢ K P + s ⎥ s ( s + 2) . When the plant operates in a unity
where K > 0. The positive value of K for which there are ⎣ ⎦
exactly two poles of the unity feedback system on the feedback configuration, the condition for the s­ tability
jw axis is equal to _____ (rounded off to two decimal of the closed loop system is [2015]
places) [2019]
KI
X(s) + K Y(s) KP >
(A) >0 (B) 2KI > KP > 0
Σ G(s) 2
s
– (C) 2KI < KP (D) 2KI > KP

⎛ K ⎞⎛ 1 ⎞
G(s) = ⎜ K P + I ⎟ ⎜
Solution:  .
Solution:  Charecterstic equation of the given function ⎝ s ⎠ ⎝ s ( s + 2) ⎟⎠
is
S3 + 3S2 + 2S + K = 0
G(s) =
( K I + K P s)
Applying Routh-Herrvitz critereas s 2 ( s + 2)

S3 1 2
Characteristic equation is
S2 3 K
6−K = 0 S 3 + 2S 2 + KpS + KI = 0.
6−K
S 1 0 K =6 Apply RH criterion:
3
S0 S3 1 KP
K
2. A linear time invariant (LTI) system with the transfer S2 2 KI
K ( s 2 + 2 s + 2) 2Kp – KI
function G ( s) = is connected in unity S1 0
s 2 − 3s + 2
feedback configuration as shown in the figure. [2017] S0 KI2
Chapter 5  Stability Analysis  |  3.47

It is stable only when KI > 0 and 2KP – KI > 0 Now, as2 + (k + 1) = 0


2KP > KI −aw2 + (k + 1) = 0
KI given w = 2 rad/sec
KP > >0
2 so put value of a, k = −1, 2

Hence, the correct option is (A). for k = −1, a = 0 and system will not oscillate for this
value.
4. Consider a transfer function k +1
So k = 2, a = = 0.75
ps 2 + 3 ps − 2 k +2
G p ( s) = 2 with a
s + (3 + p) s + (2 − p) Hence, the correct option is (a)
positive real parameter. The maximum value of p until 6. A certain system has transfer function
which Gp remains stable is _____. [2014]
s+8
Solution:  Given transfer function G ( s) = , where α is a parameter. Consider
s2 + a − 4
ps 2 + 3 ps + 2
G p ( s) = the standard negative unity feedback configuration as
s 2 + (3 + p) s + (2 − p) shown below.
the characteristics equation is = s2 + (3 + p)s + (2 – p)
Using RH criteria.
s2 1 (2 – p)
S (3 + p) 0
s0 (2 – p) 0 Which of the following statements is true?
For system to be stable, 2 – p ≥ 0 or Pmax = 2. (a) The closed loop system is never stable for any val-
5. The feedback system shown below oscillates at ue of α
2 rad/s when (b) For some positive values of α, the closed loop sys-
tem is stable, but not for all positive values
(c) For all positive values of α, the closed loop system
is stable
(d) The closed loop system is stable for all values of α,
both positive and negative [2008]
Solution: (c)
(a) K = 2 and a = 0.75
Closed loop gain is
(b) K = 3 and a = 0.75
G ( s) s+8
(c) K = 4 and a = 0 5 = 2
1 + G ( s ) s +a s + 4 + s +8
(d) K = 2 and a = 0.5[2012]
Solution: (a) Characteristics equation, g(s) = s2 + (α + 1)s + 4
Characteristics equation So, close loop system will be stable only for
α > −1. Therefore, for all +ve value of α system is
k ( s + 1) stable.
1 + G(1)H(s) = 0, 1 + =0
s + as 2 + 2 s + 1
3
Hence, the correct option is (c)
s3 + as2 + (2 + k)s + (1 + k) = 0 7. The number of open right half plane poles of
RH table is 10
G ( s) = 5 is
s3 1 (k + 2) s + 2 s + 3s3 + 6 s 2 + 5s + 3
4

s2 A (k + 1) (a) 0 (b) 1
(c) 2 (d) 3 [2008]
a(2 + k ) − ( k + 1)
s1 =0 Solution: (c)
9
Characteristic equation is
s° (k + 1)
g(s) = s5 + 2s4 + 3s3 + 6s2 + 5s + 3
 k +1 
for oscillation, a(2 + k ) − ( k + 1) = 0 ⇒ a =   1 g ′( z ) = 3 z 5 + 5 z 4 + 6 z 3 + 3 z 2 + 2 z + 1
9 k+2 Put s = ,
z
3.48 | Control Systems

Routh array, a = 0.75.


z5 3 6 2 Hence, the correct option is (b)
z 4
5 3 1 Common Data for Questions 6 and 7
21 7 Consider a unity-gain feedback control system whose
z3
5 5 as + 1
open-loop transfer function is G ( s) = 2
4 s
z2 1 9. The value of `a‘ so that the system has a phase-margin
3
equal toπ/4 is approximately equal to
−7 (a) 2.40 (b) 1.40
z1
4 (c) 0.84 (d) 0.74 [2006]
z0 1 Solution: (c)
Since there are two sign changes in routh array, so two p
P.M. =
poles are lying at right half of s-plane 4
Hence, the correct option is (c)
⇒ 180 + tan−1 (aw) − 180° = p
8. The positive values of 'K' and 'a' so that the system 4
shown in the figure below oscillates at a frequency of 2
⇒ aw = 1
rad/sec respectively are
for gain cross over frequency, |G(s)| = 1
1
1 + a2 w 2
= 1 ⇒ w2 = 2 ⇒ w = 24
w2
1
a= 1
= 0.84
(a) 1,0.75 (b) 2,0.75 24

(c) 1,1 (d) 2,2 [2006]
Solution: (b) Hence, the correct option is (c)
1 0. With the value of `a‘ set for phase-margin of π/4, the
k ( s + 1) value of unit-impulse response of the open-loop system
1 + G ( s) H ( s) = 1 + =0
s + as 2 + 2 s + 1
3
at t = 1 second is equal to
(a) 3.40 (b) 2.40
s3 + as 2 + 2 s + 1 + ks + k (c) 1.84 (d) 1.74 [2006]
=0
s3 + as 2 + 2 s + 1 Solution: (c)

s3 + as2 + (2 + k)s + 1 + 1 = 0 G ( s) = 0.84 s + 1


s2
s3 1 2+k H(s) = 1, R(s) = 1 ⇒ C(s) = G(s). R(s)
s2 a k+1 0.84 s + 1
⇒ C ( s) =
a(2 + k ) − (1 + k ) s2
S 0
a taking inverse Laplace transform
s0 (k + 1) −1  1 + 0.84 s 
c(t ) = L   = (t + 0.84)u(t )
a(2 + k ) − ( k + 1)  s2 
for oscillations =0
a at, t = 1,
( k + 1) c(t) = 1 + 0.84 = 1.84
⇒9=
( k + 2) Hence, the correct option is (c)
⇒ as2 + k + 1 = 0 ⇒ s = jw, s2 = −w2 = −4 1 1. The gain and phase crossover frequencies in rad/sec
are, respectively
⇒ −4a + k + 1 = 0
(a) 0.632 and 1.26 (b) 0.632 and 0.485
k +1 k +1 k +1 (c) 0.485 and 0.632 (d) 1.26 and 0.632
⇒ a= ⇒ = ⇒k=2
4 4 k +2 [2005]
Chapter 5  Stability Analysis  |  3.49

Solution: (d) 1 + G (s) H (s) = 0 ⇒ s4 + 4s3 + 5s2 + 6s + k = 0


Gain cross over frequency defined where gain is 1 RH – table,
|G(s)| = 1 s4 1 5 k
3 s 4
3
6
⇒ 1
= 1 ⇒wgc = 1.26
s2 k
w(w 2 + r ) 2
phase cross over frequency where ∠ GH = 180 7
× 6 − 4k
 wf  2
⇒ w pc = 0.682  = 2 cos 2wf  s 0
 2  7

2
Hence, the correct option is (d) s0 K
1 2. Based on the above results the gain and phase margins for system to be stable, k > 0
of the system will be
(a) -7.09 and 87.5° ⇒ 7 × 6 − 4k > 0
2
(b) 7.09 and 87.5°
7
21
(c) 7.09 dB and -87.5° ⇒k< 2
(d) -7.09 dB and -87.5° [2005] 4
Solution: (d) 21
So, 0 < k <
3 4
G.M. at wf = 1
Hence, the correct option is (a)
0.632(0.6322 + 4) 2
1 4. For the polynomial
a = 2.26
P(s) = s5 + s4 + 2s3 + 2s3 + 2s2 + 3s + 15, the number of
1
G.M. = 20 log roots which lie in the right half of the s-plane is
a (a) 4 (b) 2
1 (c) 3 (d) 1 [2004]
20 log 2.26 = −7.09 Solution: (b)
P(s) = s5 + s4 + 2s3 + 2s2 + 3s + 15
as G.M. is –ve so system is unstable, PM is also
s5 1 2 3
negative.
s4 1 2 15
P.M. = - 87.5°
s3 0(E) −12 0
Hence, the correct option is (d)
2 E + 12
13. The open-loop transfer function of a unity feedback s2 15 0
k E
system is G ( s) = 2
s( s + s + 2)( s + 3) −12(2 E + 12)
S − 15 E
The range of K for which the system is stable is E
21 s0 15
(a) > K > 0 (b) 13 > K > 0
4 Let E is a small +ve number. Then coefficient, s2,
21 2 E + 12
(c) < K < ∞ (d) -6 < K < ∞ → +ve
4 E
 [2004] −12(2 E + 12)
→ −ve
Solution: (a) E
k ∴So, two sign changes from s2 to s and s to s°
G ( s) = , H(s) = 1
2
s( s + s + 2)( s + 3) ∴Two roots on RHS of s plane.
k Hence, the correct option is (b)
1 + G ( s) H ( s) = 1 + 3 2 2
s( s + 3s + s + 3s + 2 s + 6) Common Data for Questions 12 and 13.
s 4 + 4 s3 + 5s 2 + 6 s + k The open loop transfer function of a unity feedback
= =2s
s( s3 + 4 s 2 + 5s + 6) system is given by G ( s) = 3e
s( s + 2)
3.50 | Control Systems

15. The gain margin and the phase margin of a feedback d 2


s ( s + 1) = 0 ⇒ s = 0.
system with G ( s) H ( s) = are ds
( s + 100)3
Double root on imaginary axis, so system is unstable.
(a) 0 dB, 0° (b) ∞, ∞
Hence, the correct option is (c)
(c) ∞, 0° (d) 88.5 dB, ∞
1 8. The system with the open loop transfer function
[2003]
1
Solution: (b) G ( s) H ( s) = 2
has a gain margin of
s( s + s + 1)
s
G ( s) H ( s) = (a) - 6 dB (b) 0 dB
( s + 100)3 (c) 3.5 dB (d) 6 dB [2002]

Gain margin and phase margin of system cannot be Solution: (b)
determined. 1
Given, G ( s) H ( s) =
2
Hence, the correct option is (b) s( s + s + 1)
16. The system shown in the figure remains stable when w when ∠G ( s) H ( s) = −180

−1 w ⇒ w 2pc = 1
⇒ −180 = −90 − tan 2 rad/sec.
1+ w
1
So, G ( s) H ( s) = 2 2
(a) K < -1 (b) -1 < K < 1 (1 − w 2 ) + jw
(c) 1 < K < 3 (d) K < -3 [2002]
Solution: (d) at wpc = 1 rad/sec.
k ∴G.M. = −20 log1 = 0
Y ( s) s k Hence, the correct option is (b)
Given = =
R( s)  g k  s − (3 + k ) 19. The feedback control system in the figure is stable.
1−  + 
s s −
for system to be stable. −
3 + k < 0
k < −3

Hence, the correct option is (d)
17. The characteristic polynomial of a system (a) for all K ≥ 0 (b) only is K ≥ 0
is q(s) = 2s5 + s4 + 4s3 + 2s2 + 2s + 1. The system is (c) only if 0 ≤ K < 1 (d) only if 0 ≤ K ≤1
(a) stable
(b) marginally stable [2001]
(c) unstable Solution: (c)
(d) oscillatory [2002] Given transfer function
Solution: (c) k ( s − 2)
RH – table, G1G2 ( s + 2) 2
T .F = =
s5 2(1) 4(2) 2(1) 1 + G1G2 H k ( s − 2)( s − 2)
1+
s4 1 2 1 ( s + 2) 2
s3 0 0 0 k ( s − 2)
s
2 =
( s + 2) 2 + k ( s − 2) 2

S
Characteristic equation is = s2 + 4 + 4s + ks2
s0
– 4ks + 4k
In the third row all coefficients are zero. So, different
or (1 + k)s2 + 4(4 – 4k)s + 4k + 4 = 0
rows
RH – table.
d 4
( s + 2 s 2 + 1) = 0 s = ± j, s = ± j s2 (1 + k) (4k + 4)
ds
Chapter 5  Stability Analysis  |  3.51

S k(4 – 4k) 0 12 − A
for stability > 0 ⇒ 0 < A< 12
S 4(k + 1) 3
for system to be stable.
Hence, the correct option is (c)
k(4 – 4k) > 0 ⇒ k < 1 and k ≥ 0.
also, (k + 1) > 0 ⇒ k> −1 23. An electromechanical closed-loop control system has
the following characteristic equation: s3 + 6K s2 +
so 0 ≤ k < 1
(K + 2)s + 8 = 0, where K is the forward gain of the
Hence, the correct option is (c) system. The condition for closed loop stability is:
2 0. A system described by the transfer function (a) K = 0.528 (b) K = 2
1 (c) K = 0 (d) K = -2.258
H ( s) = 3  [1990]
s + a s + Ks + 3 is stable.
2

The constraints on α and K are Solution: (a)


(a) α > 0, αK < 3 (b) α > 0, αK > 3 Given s3 + 6ks2 + (k + 2)s + 8 = 0
(c) α < 0, αK > 3 (d) α < 0, αK < 3
Apply RH,
[2000]
s3 1 k+2
Solution: (b)
s 6k 8
2
R-H table is 1 k
3 6 k 2 + 12k − 8
S
6k
∝ k −3
s° 8

for system to be stable. For stable system,
∝ k −3 ⇒k = −2.528, +0.528 as k > 0, so k⇒ 0.528
∝ > 0, > 0 or ∝ k > 3.
∝ Hence, the correct option is (a)
Hence, the correct option is (b)
24. In order to stabilize the system shown in the below
1
21. If G(s) is a stable transfer function, then F ( s) = is given figure. Ti should satisfy:
G ( s)
always a stable transfer function.
State True or False. [1994]
Solution:  False
( s + z1 )( s + z2 )
T.F. of G ( s) =
( s + p1 )( s + p2 )
Ti = -T (b)
(a) Ti = T
( s + p1 )( s + p2 )
T.F. of G ( s) = (c)
Ti < T (d) Ti > - T. [1989]
( s + z1 )( s + z2 )
Solution: (d)
for stability of G(s), poles of G(s) should be at left half
s plane, so far stable G(s) F(s) have zeros at left half of 1 + sTi 1
G ( s) = ×
s plane. So, F(s) need not to be stable. s s(1 + ST )

22. If s3 + 3s2 + 4s + A = 0, then all the roots of this equation H(s) = 1
are in the left half plane provided that
G ( s)
(a) A > 12 (b) -3 < A < 4 ∴ T ( s) =
(c) 0 < A < 12 (d) 5 < A <1s2 [1993] 1 + G ( s) H ( s)

Solution: (c) Characteristics equation
→s3 + 3s2 + 4s + A = 0 1 + G(s) H(s) = 0
Using RH (1 + sTi ) 1
1+ × ×1 = 0
s3 1 4 s s(1 + sT )

s2 3 A
= s2 (1 + sT) + (1 + sTi) = 0
12 − A
S 0 = s3T + s + sTi + 1 = 0
3
s0 A
3.52 | Control Systems

Using R-H criterion


s3 T Ti
Five-marks Questions
s 1
2
1
1. Consider the feedback control system shown in figure.
s Ti – T 0
1

s0 Ti 0
For stability 1st column should be positive
∴Ti – T > 0
Ti > T
Hence, the correct option is (d)
2 5. Consider a characteristic equation given by
s4 + 3s3 + 5s2 + 6s + K + 10
(a) Find the transfer function of the system and its
The condition for stability is characteristic equation.
(a) K > 5 (b) -10 < K (b) Use the Routh-Hurwitz criterion to determine the
(c) K > -4 (d) -10 < K < -4 range of K for which the system is stable.
[1988] [2001]
Solution: (d) Solution:
Characteristics equation
s4 + 3s3 + 5s2 + 6s + k + 10
using R – H criterion
s4 1 5 k + 10
s3 3 6 0
s 3
2
k + 10 0
−12 − 3k
s1
3 k ( s + 1)
∴ G1 ( s) =
s0 k + 10 s
For stable system, all coefficients in 1st column are 1/s 1
positive Gs ( s ) = =
1 ( s + 3)
−12 − 3k 1+ × 3
∴ >0 s
3
∴ G(s) = G1(s) × G2(s)
−12 – 3k > 0 k ( s + 1) 1
= ×
3k < −12 s ( s + 3)

k < −4 C ( s) G ( s)
∴ T.F. =
and R( s) 1 + G ( s) H ( s)

k + 10 > 0
1
k > −10 where H ( s) =
( s + 0.1)
so the value of k lies between
−10 < k < −4. k ( s + 1) 1
×
Hence, the correct option is (d) C ( s) s ( s + 3)
∴ =
R( s) 1 k ( s + 1) 1
1+ × ×
( s + 0.1) s ( s + 3)
Chapter 5  Stability Analysis  |  3.53

k ( s + 1)( s + 0.1) 2 + T > 0 ⇒ T > −2


=
s( s + 0.1) ( s + 3) + k ( s + 1) and
2kt
C ( s) k ( s + 1)( s + 0.1) ( k + 1) = >0
= 2+7
R( s) s3 + 3.1s 2 + s( k + 0.3) + k
2kt
∴Characteristic equation k +1 >
2+7
s3 + 3.1s2 + s(k + 0.3) + k = 0
Using Routh-Hurwitz criterion k > 0 and T > 0
s3 1 k + 0.3 (k + 1) (2 + 7) > 2 kT
s 3.1
2
k ⇒ 2k + 2 + Tk + T > 2 kT
⇒ 2k + 7 + 2 > kT
2.1k + 0.93
s1 0 ⇒ 2(k + 1) > T(k – 1)
3.1
0 2(1 + k )
s0 k ⇒ >T
( k − 1)

For stable system’
∴ Region in which closed-loop system is stable
k > 0 and
2(1 + k )
2.1k + 0.93 =T <
>0 ( k − 1)
3.1
3. The characteristic equation of a feedback control sys-
∴ 2.1 k + 0.93 > 0 tem is s4 + 20s3 + 15s2 + 2s + K = 0
k > −0.44 (i) Determine the range of K for the system to be sta-
∴ For stability k > 0 ble.
2. The loop transfer function of a feedback control system (ii) Can the system be marginally stable? If so, find the
is given by required value of K and the frequency of sustained
oscillations. [1998]
k ‘(s + 1)
G(s)H(s) = , k > 0. Using Routh- Solution: For
s(1 + Ts)(1+ 2s)
Hurwitz criterion, determine the region of K–T plane s4 + 20s2 + 15s2 + 2 + k = 0
in which the closed-loop system is stable. [1999] Using Routh-Hurwitz criterion
k ( s + 1) s4 1 15 K
Solution: G ( s) H ( s) =
s(1 + 7 s)(1 + 2 s) s3 20 2 0
Characteristics equation 298 0
s2 K
1 + G(s) H(s) = 0 20
k ( s + 1) k 0
1+
s(1 + 7 s)(1 + 2 s) s1 20 k × 20 0 0
2−
298
⇒ s(1 + 7s) (1 + 2s) + k(s + 1) = 0 0 0
⇒ s(1 + 2s + 7s + 27s2) + ks + k = 0 s0 K
⇒ 27s3 + 2s2 + 7s2 + s + ks + k = 0 For stable system
⇒ 27s3 + s2 (2 + 7) + s(1 + k) = 0 k>0
s3 αT 1+k 2 − 20 k × 20
and >0
s 2 + 7
2
K 298
s1 (1 + k ) − 27k 0 ⇒2>
400 k
0 2+7 298

s0 K ∴ k < 1.49
For a stable system Range of k is 0 < k < 1.49
k > 0, 2T > 0 (ii) For oscillation
T>0 For marginal stable value of k = 1.49
3.54 | Control Systems

298 2 C ( s) G (i)
s + k = 0 (from s2) ∴ = 2
20 R( s) s + 3s + G

19.9s2 + 1.49 = 0 (b) For overshoot


10s2 + 1 = 0 −πξ
1−ξ 2
−1 mp = e × 100 = 10
s2 =
10
∴ξ = 0.6
1 ωn2 = G
s± j
10 ωn = G
1 ∴2ξωn = 3 (from eq(i))
+ jωn = ± j
10 3
∴ ωn =
2 × 0.6
1
ωn = rad/sec
10 ∴ G = 2.5
4. A system having an open loop transfer function G = 6.25
k (s + 3) Using Routh-Hurwitz condition
G(s ) = is used in control system with
s (s 2 + 2s + 2) s3 1 2+k
unity negative feedback. Using the Routh-Hurwitz cri- s 2 2 3k
terion, find the range of values of k for which the feed- s 1
4 + 2k − 3k 0
back system is stable. [1996] >0
2
k ( s + 3)
Solution: G ( s) = H(s) = 1 3
s
0
s ( s 2 + 2 s + 2)
G ( s) For system to the stable
T.F. =
1 + G ( s) H ( s) 3k > 0

k>0
Characteristic equation
∴1 + G(s) H(s) = 0 4 + 2k − 3k
>0
2
k ( s + 3)
⇒ 1+ =0 4–k>0
s3 + 2 s 2 + 2 s
k<4
H(s) = 1
∴ Range of stability
C ( s) G ( s)
∴ = 0<k<4
R( s) 1 + G ( s) H ( s)

G
s( s + 3) G
=
G s( s + 3) + G
1+
s ( s + 3)
Chapter 6
Root Locus

One-mark Questions K
Given OLTF G(s) =
Solution: 
s ( s + 1) ( s + 3)
1. For the system shown in the figure, s = –2.75 lies on the
root locus if K is _______. [2015] Characteristic equation is 1 + G(s) H(s) = 0

X(S) + Y(S)
s(s2 + 4s + 3) + K = 0
s+3
∑ K
– s+2 s3 + 4s2 + 3s + K = 0
Apply RH criterion,
10 S3 1 3
S2 4 K
Solution: 
From the given block diagram 12 − k
S1 0
K ( s + 3) ( s + 2) K ( s + 3) 4
T(s) = = S0 K
10 K ( S + 3) ( s + 2) + 10 K ( s + 3)
1+ Given K > 0.
( s + 2)  It is crossing imaginary axis let S1 row all zeros.
K ( s + 3)
T(s) = 12 − k
(10 K + 1) s + (2 + 30 K )


4
= 0 ⇒  K = 12

But given S = –2.75 is on Root locus so it must satisfy Hence, the correct Answer is (12).
the magnitude condition. 3. The root locus plot for a system is given below. The
|T(s)| = 1 at S = –2.75 open loop transfer function corresponding to this plot
is given by
K (0.25)
=1
(10 K + 1) ( −2.75) + 2 + 30 K 
K (0.25) = – 27.5 K – 2.75 + 2 + 30 K
–2.5K = – 0.75
K = 0.3

Hence, the correct Answer is (0.29 to 0.31).


2. A unity negative feedback system has the open-loop
K
transfer function G(s) = . The value
s ( s + 1) ( s + 3) s( s + 1)
(a)
G ( s) H ( s) = K
of the gain K(>0) at which the root locus crosses the ( s + 2)( s + 3)
imaginary axis is _______. [2015]
3.56 | Control Systems

( s + 1)
G ( s) H ( s) = K
(b)
s( s + 2)( s + 3) 2
1
G ( s) H ( s) = K
(c)
s( s − 2)( s + 2)( s + 3)
( s + 1)
G ( s) H ( s) = K
(d)  [2011]
s( s + 2)( s + 3)
(a) only if 0 ≤ K ≤ 1
Solution: (b)
(b) only if 1 < K < 5
From the plot we can observe that one pole terminates (c) only if K > 5
at zero at −1 and three poles terminate to ∞. It means (d) if 0 ≤ K < 1 or K > 5 [2001]
there are four poles and 1 zero. There are two poles at
Solution: (d)
−3 on both sides. It means there are two poles at –3.
For overdamping roots of characteristics equation
Hence, the correct option is (b)
should lie on –ve axis and be unequal.
K Hence, the correct option is (d)
4. Given G ( s) H ( s) = , the point of inter-
s( s + 1)( s + 3) 7. If the open-loop transfer function is a ratio of a numer-
section of the asymptotes of the root loci with the real ator polynomial of degree 'm' and a denominator poly-
nomial of degree 'n,' then the integer (n-m) represents
axis is the number of
(a) -4 (b) 1.33 (a) breakaway points (b) unstable poles
(c) -1.33 (d) 4 [2004] (c) separate root loci (d) asymptotes
Solution : (c)  [1994]
ΣP − ΣZ −1 − 3 Solution: (d)
Centroide = = = −1.33
P−Z 3 Difference between poles and zeros will give number
of asymptotes.
Hence, the correct option is (c)
Hence, the correct option is (d)
5. Which of the following points is NOT on the root
locus of a system with the open-looptransfer function
G ( s) H ( s) =
K Two-marks Questions
s( s + 1)( s + 3) 1. The open loop transfer function of a unity feedback
(a) s = - j 3 (b)
s = - 1.5 control system is
(c) s = - 3 (d) s = - ∞ [2002] K
G(s) = 2 .
Solution: (b) s + 5s + 5
The value of K at the breakaway point of the feedback
control system’s root locus plot is [2016]
Solution:  Now we have transfer function as
K
G(s) = .
s 2 + 5s + 5
The characteristics equation for open loop TF is
1 + G(S)H(S) = 0
Root locus lies where number of poles and zeros to
right half of the pole is odd. S 
2
+ 5S + 5 + K = 0
∴ s = −1.5 does not lie on root locus. K = –S 2 – 5S – 5
Hence, the correct option is (b)
To find break away point differentiate the above equa-
6. The root-locus diagram for a closed-loop feedback sys- tion w.r.t s, that is
tem is shown in the figure. The system is overdamped
dK
=0
ds 
Chapter 6  Root Locus  |  3.57

⇒ –2 S – 5 = 0 s +1
(a)
( s + 2)( s + 4)( s + 7)
S = –2.5
K = –(–2.5)2 – 5(–2.5) – 5 s+4
(b)
( s + 1)( s + 2)( s + 7)
=
1.25

Break away point is that point in root locus where two s+7
(c)
complex poles are present causing the break away ( s + 1)( s + 2)( s + 4)
Hence TF has break away point at S = –2.5 at k = 1.25 ( s + 1)( s + 2) 
(d) [2014]
Hence, the correct Answer is (1.25). ( s + 7)( s + 4)
2. An open-loop transfer function of a plant in a unity
Solution: (b)
K (s + 4)
feedback configuration is given as ( s ) = . Since the root locus always emerges from the break-
( s + 8) s 2 − 9 ( ) away points, i.e., when two poles are at a part of same
The value of the gain K(>0) for which -1 + j2 lies on the root locus lie. So, σ = −1 and −2 surely be poles.
root locus is ______  [2015] We also know root locus terminates at zero. So
s = −4 is zero and s = −7 is a pole.
K (S + 4)
G(s) =
Solution:  ( s + 4)
( s + 8) ( s 2 − 9 ) ∴ T ( s) =
( s + 1)( s + 2)( s + 7)

K (s + 4) Hence, the correct option is (b)


G(s) = 4. The feedback configuration and the pole-zero locations
( s + 8) ( s + 3) ( s − 3) 
s2 − 2s + 2
Given S1 = –1 + j2 lies on root locus so of G ( s) = are shown below. The root locus
s2 + 2s + 2
|G(s)| = 1  at  S = S1
for Negative values of K, i.e. for -∞ < K < 0, has break-
K {3 + j 2} away/break-in points and angle of departure at pole P
G(S1) = 
(7 + j 2) (2 + j 2) ( −4 + j 2) (with respect to the positive real axis) equal to
+
|G(jω)| = 1 K G(s)
-
K{ 13} = 1

( 51) . 8. 20 
1m(s)
K = 25.05
o x
Hence, the correct Answer is (25 to 26).

3. In the root locus plot shown in the figure, the pole/zero Re(s)
marks and the arrows have been removed. Which one o x
of the following transfer functions has this root locus?

± 2 and 0°
(a) (b) ± 2 and 45°

(c) ± 3 and 0° (d) ± 3 and 45° [2009]


Solution : (b)
1 + G(s) H(s) = 0
k ( s 2 − 2 s + 2) s2 + 2s + 2
1+ 2
=0 ⇒k =
s + 2s + 2 s2 − 2s + 2
3.58 | Control Systems

dk (b)
= 0,
ds
So, (s2 – 2s + 2) (2s + 2) – (s2 + 2s + 2) (s – 1) = 0

⇒s=± 2
Angle of departure is

fD = 180° + f
Where f = Σq z − Σf p − 135
(c)
fD = 180 − 135 = 45°
Hence, the correct option is (b)
5. A unity feedback control system has an open-loop
K
transfer function G ( s) = 2
.
s( s + 7 s + 12)

The gain K for which s = -1 + j1 will lie on the root


locus of this system is
(a) 4 (b) 5.5 (d)
(c) 6.5 (d) 10 [2007]
Solution: (d)
G ( s)
Transfer function = ; H(s) = 1
1 + G ( s) H ( s)
for the point s = −1 + j1 to lie on root locus
1 + G(s) = 0
k
⇒1+ 2
=0
s( s + 7 s + 12)
Solution: (c)
⇒[s2 + 7s + (k)] + (12) = 0 s 2 + 3s
1 + G(s)H(s) = 0, k =
Putting s = −1 + j, 1− s
dk
⇒ (−1 + j) (1 – 2j – 1 – 7 + 7j + 12) + k = 0 For break away and break in point = 0,
ds
K = +10. dk
= (1 − s)(2 s + 3) + s 2 + 3s = 0
Hence, the correct option is (d) ds
6. A unity feedback system is given as ⇒s = 3, −1
K (1 − s) So, −1 will be break away point and 3 is break in point.
G ( s) =
s( s + 3) Hence, the correct option is (c)

Indicate the correct root locus diagram [2005] K
7. The root locus of the systemGG(s) ( s) =
( s) HH(s) has
(a) s( s + 2)( s + 3)
the break-away point located at

(a) (-0.5,0) (b) (-2.548, 0)


(c) (-4, 0) (d) (-0.784, 0)
[2003]
Solution: (d)
1 + G(s)H(s) = 0
k
1 + s( s + 2)( s + 3) = 0 ⇒ k = −1 (s3 + 5s2 + 6s)
Chapter 6  Root Locus  |  3.59

dk k
= −(3s 2 + 10 s + 6) G ( s) = ,. The root locus plot of the
ds ( s + 0.1)( s + 0.2)
dk system is of the form. [1992]
for break away point. =0 (a)
ds
or 3s + 10s + 6 = 0 ⇒ s = −0.784, −2.55
2

−2 − 3 −5
Centroid = = = −1.66
−3 3
p p
angle of asymptotes = (2k + 1) = (2k + 1) (b)
p−z 3
p 5p
= ,p ,
3 3
So break away point is (−0.784, 0) as −2.55 does not lie
on R.L.
Hence, the correct option is (d)
8. Consider the points s1 = -3 + j4 and s2 = -3 - j2 in the
s-plane. Then, for a system with the open-loop transfer
function
K
G ( s) H ( s) =
( s + 1) 4
(c)
(a) s1 is on the root locus, but not s2
(b) s2 is on the root locus, but not s1
(c) both s1 and s2 are on the root locus
(d) neither s1 nor s2 is on the root locus [1999]
Solution: (b)
s1 = −3 + 4j
k k
G ( s) H ( s) = 4
=
( −3 + 4 j + 1) ( −2 + 4 j ) 4

∠ G (s) H (s) = -4 tan -1(-2) = -4.66°≠ odd multiple of π
So, s1 does not lie on root locus
s2 = −3 – j2
(d)
k k
G ( s) H ( s) = =
( −3 − j 2 + 1) 4 ( −2 − 2 j ) 4

∠ G (s) H (s) = -4 [tan -1(1)] = - p = odd multiple of π
So, s2 lies on root locus.
Hence, the correct option is (b)
9. Given a unity feedback system with open-loop transfer
function.
3.60 | Control Systems

Solution: (a) (c)


P = 3, Z = 0, P – Z = 3.0 = 3
(2q + 1)180°
q= ⇒θ = 60°, 180°, 300°
p−z

So problem (a) will be satisfied


Hence, the correct option is (a)
1 0. The characteristic equation of a feedback control sys-
tem is given by s3 + 5s2 + (K + 6)s K = 0, where K > 0
is a scalar variable parameter. In the root-locus diagram (d)
of the system, the asymptotes of the root-loci for large
values of K meet at a point in the s-plane, whose coor-
dinates are
(a) (-3,0) (b) (-2, 0)
(c) (-1,0) (d) (2,0) [1991]
Solution : (b)
Open loop transfer function = G(s)H(s)
k ( s + 1)
=
s + 5s 2 + 6 s
3

k ( s + 1) Solution : (b)
⇒ G ( s) H ( s) =
s( s + 2)( s + 3) k
Given T.F = 2
s + (3 − k )( s + 1)
poles = 0, −2, −3, zeros = −1
( −2 − 3) − ( −1) root locus plot will be
centroid = = −2 = ( −2, 0)
3 −1
Hence, the correct option is (b)
1 1. The transfer function of a closed loop system is
K
T ( s) =
s 2 + (3 − K )( s + 1)

Where K is these forward path gain, the root locus plot
of the system is:  [1990]
(a)
Hence, the correct option is (b)
1 2. The OLTF of a feedback system is G(s) H(s)
K ( s + 1)( s + 3)
= .
s2 + 4s + 8
The root locus for the same is [1989]
(a)
(b)
Chapter 6  Root Locus  |  3.61

(b)

Fig. (b)
(c)
(a)
1 + (0.5s + 1)(10 s + 1)
K
(b)
( s + 2)( s + 0.1)
K
(c)
1 + K (0.5s + 1)(10 s + 1)
K
(d)  [1988]
K + 0.2(0.5s + 1)(10 s + 1)
(d)
Solution: (d)
k
G ( s) =
( s + 0.1)( s + 2)

K .G ( s)
T.F =
1 + K .G ( s)

k
( s + 0.1)( s + 2)
T .F . =
k
1+
Solution: (a) ( s + 0.1)( s + 2)
Answer (b) and (d) are wrong as root locus is sym- k
T .F . =
metrical about real axis, and (c) is wrong as root locus ( s + 0.1)( s + 2) + K
direction is from poles to zero.
Hence, the correct option is (a) k
T .F . =
13. Consider a closed-loop system shown in figure (a) ( s + 0.1)( s + 2) + K

below. The root locus for it is shown in figure (b). The k
closed loop transfer function for the system is T .F . =
k + 0.2(1 + 10 s)(1 + 0.5s)

R(s) + c(l)
K G(s) k
T .F . =
k + ( s + 0.1) ( s + 2)

k
=
k + (0.2) (1 + 10 s)(1 + 0.5s)
  
Fig. (a) Hence, the correct option is (d)
Chapter 7

Frequency Analysis
(A) P : Open Circuit Q : Short Circuit, R : Matched
One-mark Questions Load
(B) P : Open Circuit Q : Matched Load, R : Short
1. For an LTI system, the Bode plot for its gain is as illus- ­Circuit
trated in the figure shown. The number of system poles (C) P : Short Circuit Q : Matched Load, R : Open
Np and the number of system zeros Nz in the frequency ­Circuit
range 1 Hz ≤ f ≤ 107 Hz is [2019] (D) P : Short Circuit Q : Open Circuit , R : Matched
Gain (dB) Load
–20
100 dB/d Solution:  We know that he constant resistance circle, r
ec.
= 0 and the constant reactance circle, x = 0 pass through
–6

–4 the point ‘P’. therefore


0d

0d
B/d
B/

ec z = r + jx = 0
de

.
c.

104 105 106 107 f


0
101 102 103 –4 (in Hz) Thus point ‘P’ represents short circuit.
0d
B/d At the point ‘R’ r = ∞ and x = ∞ which gives z = ∞.
–6

ec
0d

. Therefore point ‘R’ represents open circuit.


B/
de

The constant resistance circle r = 1 passes through


c.

the point Q and x = 0.


(A) Np = 6, Nz = 3 (B) Np = 7, Nz = 4
z = r
(C) Np = 5, Nz = 2 (D) Np = 4, Nz = 2
ZL
Solution:  Addition of zero changes slope by +20 dB/ =1
Z0
decade
ZL = Z0 (Load impedance matched to the line character-
Addition of pole changes slope by -20 dB/decase
istic impedance)
Np = 6
Hence, point ‘Q’ represents matched load.
Nz = 3
Hence, the correct option is (C)
Hence, the correct option is (A)
3. A closed loop control system is stable if the Nyquist
2. The points P, Q and R shown on the smith chart (nor- plot of the corresponding open loop transfer function
malized impedance chart) in the following figure repre-  [2016]
sent: [2018] (A) Encircles the s-plane point (–1 + j0) in the coun-
terclockwise direction as many times as the num-
ber of right half s-plane poles.
(B) Encircles the s-plane point (0 – j1) in the clock-
wise direction as many times as the number of
P Q R
right half s-plane poles.
(C) Encircles the s-plane point (–1 + j0) in the coun-
terclockwise direction as many times as the num-
ber of left half s-plane poles.
Chapter 7  Frequency Analysis  |  3.63

(D) Encircles the s-plane point (–1 + j0) in the coun- jω


terclockwise direction as many times as the num-
ber of right half s-plane zeros.
Solution:  In Nyquist plot, for an open loop transfer
function, condition required is N – P = Z where N is the ω=0 ω =∞
number of encirclement of pt. (–1, 0). P = no of poles of 1 10 σ
open loop system at the right side and Z is no of poles
Hence, the correct option is (A).
of close loop system at the right side.
Hence, the correct option is (A). 6. The transfer function of a mass-spring-damper system
is given by
4. The number and direction of encirclements around the 1
point –1 + j0 in the complex plane by the Nyquist plot G(s) =
Ms 2 + Bs + K
1− s
of G(s) = is [2016]
(A) zero + 2 s
4 The frequency response data for the system are
(B) one, anti clockwise given in the following table.
(C) one, clockwise
(D) two, clockwise ω in rad/s |G( jω)| in dB Arg (G( jω)) in deg
Solution:  We are given that 0.01 –18.5 –0.2
1− s 0.1 –18.5 –1.3
G ( s) =
4 + 2s 0.2 –18.4 –2.6
The magnitude and phase can be evaluated as 1 –16 –16.9

1+ ω2 ω 2 –11.4 –89.4
M= , ϕ = − tan −1 ω − tan −1 3 –21.5 –151
16 + 4ω 2 2
5 –32.8 –167
ω=0 ω=a 10 –45.3 –174.5
1 1
∠0 ∠ − 180°
4 2 The unit step response of the system approaches a
steady state value of _______. [2015]
–1 + j.0
1
1 1 G(s) =
Solution: 
2 4 MS 2 + BS + K
1
Since, (–1,0) lies on the left and is not encircled there- for Unit step response R(s) =
s
for the system is stable and no of encirclement is 0.
1
Hence, the correct option is (A). C(s) = R(s) ⋅ G(s) =
5. The polar plot of the transfer function G(s) =
10 ( s + 1) ( 2
s Ms + Bs + K )
s + 10 Steady state value (or) F.V
for 0 ≤ ω < ∞ will be in the [2015]
= Lt s.C(s) = 1/K
S →0
(A) first quadrant (B) second quadrant
Find the ‘K’ value from the given table
(C) third quadrant (D) fourth quadrant
1
10 ( s + 1) G ( jω ) =
Given Gc(s) =
Solution:  − M ω + jBω + K 
2

( s + 10)
1
Zero is dominating, so it is a lead network. |G( jω)| =
(K − ω M )
2
+ ( Bω )
2 2
⎧ω ⎫ ⎧ω ⎫ If ω = 1
Φ = ∠Gc ( jω) = tan–1 ⎨ ⎬ − tan −1 ⎨ ⎬ any value of 
⎩1⎭ ⎩10 ⎭
0 ≤ ω ≤ ∞. 20 log |G( jω)|| = –16
Φ → positive value so, it is in 1st quadrant.
|G( jω)| = 0.158
3.64 | Control Systems

(6.329)2 = (K – M)2 + B2

40 = (K – M)2 + B2 (i)


⎧ Bω ⎫
− tan −1 ⎨ ⎬ = –16.9°
⎩K −ω M ⎭
2

B
= 0.30
K−M 
B2 = 0.0923 (K – M)2

40 = (K – M)2 + 0.09 23(K – M)2

40 = (1.0923)(K – M)2

K – M = 6.05 (a) G(s) is an all-pass filter


(b) G(s) is a strictly proper transfer function
(c) G(s) is a stable and minimum-phase transfer func-
Bω tion
= tan F
K −ω2M (d) The closed loop system is unstable for sufficiently
at ω = 2 large and positive K [2014]
2B
= tan (89.4) = 95.489 Solution: (d)
K − 4 M
and
|G( jω)|ω = 2 = 0.269

13.8 = (K – 4M)2 + 4B2 (ii)

13.8 = 9119(K – 4M)2

K – 4M = 0.038

4(K – M = 6.05)

From the above equations, we obtain K = 8.054 and


M = 2.004
But C(∞) = 1
K
C(∞) = 1 From the Nyquist plot, it can be seen that it does not
8.054  encircle the critical point (−1, 0)
C(∞) = 0.124 N = P – Z, N = 0
2nd alternate method: The closed loop-system is unstable for large and posi-
From the above analysis, we know tive k.
1 Hence, the correct option is (d).
C(∞) =
K 8. In a Bode magnitude plot, which one of the following
From the given data at ω = 0.01 rad/sec ≈ 0 slopes would be exhibited at high frequencies by a 4th
|G( jw)| in dB = –18.5 order all-pole system?
(a) -80 dB/decade (b) -40 dB/decade
∴ 20log10 1 = –18.5 (c) +40 dB/decade (d) +80 dB/decade
K
K = 100.925 = 8.4139 [2014]
1 Solution: (a)
∴ C(∞) = = 0.1188
8.4 B9 A 4th order all pole system means that the system must
Hence, the correct Answer is (0.10 to 0.13). be having no zero or s-terms in numerator and s4 terms
7. Consider the feedback system shown in the figure. The in denominator or 4 pole i.e.
Nyquist plot of G(s) is also shown. Which one of the 1
H ( s) ∝
following conclusions is correct? s4

Chapter 7  Frequency Analysis  |  3.65

One pole exhibits −20 dB/dec slope; so four pole exhib- (a)
its a slope of −80 dB/decade.
PM = 180 + ∠GH w = w
gc

from the given transfer function
 w gc  −1  gc
w  −1  gc
w 
∠G ( jw ) = − tan −1   − tan   − tan  
 0.1   1   10 
or ∠G ( jw ) = −84.289° − 45° − 5.711° = −135°
∴from equations (ii) and (iii), we get phase margins
PM = 180 – 135 = 45°.
(b)
Hence, the correct option is (a).
9. The Bode plot of a transfer function G(s) is shown in
the figure below:

(c)

The gain (20log|G(s)|) is 32 dB and -8 dB


at 1 rad/s and 10rad/s, respectively. The phase is nega-
tive for all ω. Then G(s) is
39.8 39.8
(a) (b)
s s2

32 32
(c) (d) 2  [2013]
s s
(d)
Solution : (b)
10 rad/s to 1 rad/s is 1 decade 32 – (−8) = 40 dB
So, the slope is 40 dB/decade. It means there are two
poles at origin, which means either option (b) or option
(d) is correct, put w = 1 rad/sec in both.
 39.8 
20 log  2  = 32 dB
 (1) 
 

 32 
20 log   = 30.1dB
 12  Solution: (a)
So option (b) is correct. G(jw) = 5 + jw
Hence, the correct option is (b).
G ( jw ) = 25 + w 2
10. For the transfer function G(jω) = 5 + jω, the corre-
sponding Nyquist plot for positive frequency has the at, w = 0 G (f ) = 25 = 5
form [2011]
Hence, the correct option is (a).
3.66 | Control Systems

11. For the asymptotic Bode magnitude plot shown below, (d)
the system transfer function can be

10 s + 1 100 s + 1
(a) (b) Solution: (d)
0.1s + 1 0.1s + 1
1
100 s 0.1s + 1 Let be a lag network.
(c) (d)  [2010] s +1
10 s + 1 10 s + 1
Solution: (a) At w = 0, Mag = ∞ ∠ = tan −1 0 = 0
 s  At w = ∞ Mag = 0, ∠ = − tan −1 (∞) = −90°
k 1 + 
 0.1 
T.F., G ( s) H ( s) = If in the direction of w increasing phase shift is decreas-
 s 
1 + 10  ing system is lag network.
 
Here, 20 log k = 0, k = 1 Hence, the correct option is (d).
13. In the figure, the Nyquist plot of the open-loop transfer
10 s + 1
So G ( s) H ( s) = function G(s)H(s) of a system is shown. If G(s)H(s) has
0.1s + 1 one right-hand pole, the closed-loop system is
Hence, the correct option is (a).
12. Which one of the following polar diagrams correspond
to a lag network? [2005]
(a)

(a) always stable


(b) unstable with one closed-loop right hand pole
(b) (c) unstable with two closed-loop right hand poles
(d) unstable with three closed-loop right hand poles
 [2003]
Solution: (a)
The encirclement of critical point (−1,0) in ACW direc-
tion is once.
∴ N = 1, P = 1 (given)
Z = P – N = 0
No zero is RH of s-plane. So system is stable.
(c) Hence, the correct option is (a).
14. The Nyquist plot for the open loop transfer function
G(s) of a unity negative feedback system is shown
in the figure, if G(s) has no pole in the right-half of
s-plane, the number of roots of the system characteris-
tic equation in the right-half of s-plane is
Chapter 7  Frequency Analysis  |  3.67

Hence, the correct option is (a).


1 8. The 3-dB bandwidth of a typical second-order system
C ( s) wn2
with the transfer function = 2 is
R( s) s + 2xwn s + wn2
given by

wn 1 − 2x 2
(a)

wn (1 − x 2 ) + x 4 − x 2 + 1
(b)

(a) 0 (b) 1 wn (1 − 2x 2 ) + 4x 4 − 4x 2 + 2
(c)
(c) 2 (d) 3 [2001]
Solution :(a) (d)
wn (1 − 2x 2 ) − 4x 4 − 4x 2 + 2  [1994]
N = 0, (1 encirclement in CW direction and other in
ACW). Solution: (c)
P = 0 (no pole in right half)
For example, wn (1 − 2x 2 ) + 4x 4 − 4x 2 + 2
N=P–Z
Z=P–N=0 Hence, the correct option is (c).
So, no roots on RH of s-plane.
Hence, the correct option is (a). 19. The open loop frequency response of a system at two
15. In the Bode-plot of a unity feedback control system, the particular frequencies is given by : 1.2 ∠-180° and
value of phase of G(jω) at the gain cross over frequency 1.0 ∠-190°. The closed loop unity feedback control is
is -125°. The phase margin of the system is then______ [1994]
(a) -125° (b) -55° Solution: at 180° → |GH| = 1.2
(c) 55° (d) 125° [1998]  1   1 
Solution: (c) G.M = 20 log   = 20 log   = −1.6 dB
 GH   1.2 
 
f = −125°
at |GH| = 1 → f = −190°
P. M. = 180 + φ = 180° – 125 = 55°
Hence, the correct option is (c). P.M. = 180° + f = 180 – 190 = −10°, since both GM
and PM are negative so system is unstable.
1 6. The Nyquist plot of a loop transfer function G(jω)
H(jω) of a system encloses the (1 -, j0) point. The gain
margin of the system is
(a) less than zero (b) zero Two-marks Questions
(c) greater than zero (d) infinity [1998] 1. The figure below shows the Bode magnitude and plots
Solution: (a) n0
of a stable transfer function G(s) =
A system is unstable when Nyquist plot of G(jw) H(jw) S s + d2
s2 + d1s + d0
encloses the point (−1 + j0) and gain margin of unstable
 [2018]
system is always –ve. Therefore, the gain margin of the
given system is less than zero.
G( jω )
Hence, the correct option is (a).
17. Non-minium phase transfer function is defined as the 36 dB
transfer function 20 dB
(a) which has zeros in the right-half s-plane 0 dB ω
(b) which has zeros only in the left-half s-plane + ∑ k G(s)

(c) which has poles in the right-half s-plane
∠G( jω)
(d) which has poles in the left-half s-plane [1995] 0°
Solution:  (a)
–180°
The non-minimum phase transfer function is defined as –270°
the transfer function which has zeros in the right half of
s-plane.
3.68 | Control Systems

Consider the negative unity feedback configuration jω


with gain k in the feed forward path. s-plane
+j∞
The closed loop is stable for K < K0. The maximum
value of K0 is _______. s = Rejϑ
R→∞
Solution:  Open loop transfer function OLTF = KG(s)
We know that for the system to be stable 20 log σ
0
1
> 0 dB
KG ( s) w = w
pc

{−20 log K − 20 log G( jw ) } > 0 dB


−j∞
−20 log K − 20 > 0 dB
Nyquist Path for G(s)

−20 log K − 20 > 0 dB j IM G

20 log K < – 20
+ j 5.43K G(s)-plane
log K < – 1
K < 0.1
−j ω
Hence, the correct answer is 0.1.
ω =0
2. The Nyquist plot of the transfer function Re G
−K 2K
K
G ( s) = +j ω
( s + 2 s + 2)( s + 2)
2

does not encircle the point (-1 + j0) for K = 10 but does
encircle the point (-1 + j0) for K = 100. Then the closed − j 5.43K
loop system (having unity gain feedback) is  [2017]
(A) Stable for K = 10 and stable for K = 100
(B) Stable for K = 10 and unstable for K = 100 Nyquist Plot of G(s)
(C) Unstable for K = 10 and stable for K = 100
(D) Unstable for K = 10 and unstable for K = 100 If 0<K<1, then the number of poles of the closed-
loop transfer that lie in the right-half of the s-plane is
Solution:  The above problem can be solved by RH  [2017]
­criteria. (A) 0 (B) 1
C . E = S3 + 2S2 + 2S2 + 4S + 2S + 4 + K = 0 (C) 2 (D) 3
C . E = S3 + 4S2 + 6S + 4 + K = 0 Solution:  Given open-loop transfer function
For the system to be stable 10 k ( S + 2)
24 > (4 + k) G(S ) =
S 3 + 3S 2 + 10
(4 + k) < 24
Characteristice equation is 1 + G (S) =0
0 < K < 20
10 k ( S + 2)
Hence, the correct option is (B). 1+ =0
S 3 + 3S 2 + 10
3. A unity feedback control system is characterized by the ⇒ S 3 + 3S 2 + 10 KS + ( 20 K + 10) = 0
10 K ( s + 2)
open-loop transfer function G ( s) = 3 Form Routh Table
s + 3s 2 + 10
S3 1 10 K
The Nyquist path and the corresponding Nyquist plot 3
S 2
2 K + 10
of G(s) are shown in the figure below. 10 k − 10
S 1
0
3
S 20 k + 10 0
0
Chapter 7  Frequency Analysis  |  3.69

10 K − 10
> 0 ⇒ k >1 0°
0.01 0.1 1 10 100 ω
3 rad/s
20 K + 10 > 0 ⇒ K > −1 –45°
2
\ If K > 1 → The closed-loop system is stable If 0 < K –135°
< 1 → The closed-loop system is unstable. The number
of sign changes in the first column of routh table are = –225°
2.
–270°
\ The number of poles of the closed-loop transfer
function that lie in the Right-half of S-plane are = 2 The value of p1 is _____. [2016]
Hence, the correct option is (C). Solution:  As we know that phase of the system is
⎛ω ⎞ −1 ⎛ ω ⎞ −1 ⎛ ω ⎞
4. In the feedback system below G(s) =
1 ϕ = − tan −1 ⎜ ⎟ − tan ⎜ 10 ⎟ − tan ⎜ p ⎟
( s + 1)( s + 2)( s + 3) ⎝ 0. 1 ⎠ ⎝ ⎠ ⎝ 1⎠
. [2016] Phase margin = 180 + phase of a system
At ω = 1, phase of a system – 135°
+
r k G (S) y Pm = 180 – 135° = 45°

At ω = 1,
The positive value of k for which the gain margin of the ⎛ 1 ⎞ ⎛ 1⎞ ⎛ 1⎞
loop is exactly 0 dB and the phase margin of the loop is − tan −1 ⎜ ⎟ − tan −1 ⎜ ⎟ − tan −1 ⎜ ⎟ = 45°
⎝ 0.1⎠ ⎝ 10 ⎠ ⎝ p1 ⎠
exactly zero degree is _____.
⎛ 1⎞
Solution:  Gain margin = 0 dB tan −1 ⎜ ⎟ = −135° ⇒ p1 = 1
⎝ p1 ⎠
Phase = –180° Hence, the correct Answer is (1).
On equating the gain to 0 we get 6. Consider the Bode plot shown in the figure. Assume
that all the poles and zeros are real-valued.
20 log ⎡⎢ ω 2 +1 ω 2 + 4 ω 2 + 9 ⎤
=0 ⎥⎦
⎣ 
k

(ω2 + 1) (ω2 + 4) (ω2 + 9) = k2 40 dB

⎛ω⎞ ⎛ω⎞ 40 dB/dec –40 dB/dec


tan −1 (ω ) − tan −1 ⎜ ⎟ − tan −1 ⎜ ⎟ = −180°
⎝ 2⎠ ⎝ 3⎠
 0 dB Freq. (Hz)
fL 300 900 fH
⎡ ω⎤
⎢ω + ⎥ The value of fH – fL (in Hz) is ______. [2015]
tan ⎢−1 2 ⎥ + tan −1 ⎛ ω ⎞ = 180°
⎜⎝ ⎟⎠ Solution:  From the given Bode plot
⎢ ω ⎥
2
3
⎢⎣ 1 − ⎥
2 ⎦ K ( s + 2π f L ) ( s + 2π f H )
2 2

G( jω) =
⇒ ω = 11  ( s + 2π × 300)2 ( s + 2π × 900)2
k = 12 × 15 × 20 = 60  From the Bode plot

40 − 0
Hence, the correct Answer is (60). 40 =
log (300 ) − log f L

5. The asymptotic Bode phase plot of ⎛ 300 ⎞ 300
log10 ⎜ = 1⇒ f L = = 30 Hz
k ⎝ f L ⎟⎠ 10
G(s) = , 
( s + 0.1)( s + 10) ( s + p1 ) and
with k and p1 both positive is shown below 40 − 0
−40 =
log (900 ) − log f H

1
−1 =
log (900 f H )

3.70 | Control Systems

(fH/900) = 10 9. The Bode asymptotic magnitude plot of a minimum


phase system is shown in the figure.
fH = 9000 Hz
fH – fL = 8970 Hz

Hence, the correct Answer is (8970).


7. The phase margin (in degrees) of the system G(s) =
10
is ______ [2015]
s ( s + 10 )

10
Given G(s) =
Solution: 
s ( s + 10 )
P.M = 180° + Φ If the system is connected in a unity negative feedback
configuration, the steady-state error of the closed loop
Φ = ⇒ G( jω) at ω = ωgc system, to a unit ramp input, is________.
at ω = ωgc |G( jω)| = 1 [2014]
10 Solution :
=1
ω ω 2 + 100

100 = ω2 (ω2 + 100)
ω4 + 100 ω2 – 100 = 0
Let ω2 = x
X 2 + 100X – 100 = 0
X1 = 0.99, X2 = –100
ω2 = 0.99   ω → +ve only
ωgc = 0.9948 rad/sec 20 − 0
⎧ω ⎫ −20 dB =
Φ = –90° – tan–1 ⎨ ⎬ log 0.1 − log w gc
⎩10 ⎭ 
Φ = –90° – 5.682 where 26.02 w = 0.1 = 20 log k − 20 log 0
Φ = –95.682
PM = 180° + Φ or, k = 1.99 ~ 2
P.M = 84.3178° 20( s + 2)
so, G ( s) =
2 s( s + 10)
Hence, the correct Answer is (84 to 84.5).
A
Steady-state error for ramp input is ess =
8. The phase margin in degrees of ka
⇒ ka = lim sG ( s)
10 s →0
G ( s) = calculated using
( s + 0.1) + ( s + 1)( s + 10) From equation (iii)
the asymptotic Bode plot is_____. [2014] 10( s + 2)
ka = lim s × =2
10 s →0 s( s + 10)

Solution :   s  s  s 
0.1× 10  + 1 + 1 + 1 1
 0.1  1  10  ∴ ess = = 0.5
2
10
= 1 0. The asymptotic Bode plot of a transfer function is as
 s  s  s  shown in the figure. The transfer function G(s) corre-
 0.1 + 1 1 + 1 10 + 1
    sponding to this Bode plot is
Amount of shift = 20 log k = 20 log 10 = 20 dB
Chapter 7  Frequency Analysis  |  3.71

12. The gain and phase margins of G(s) for closed loop
stability are
(a) 6 dB and 180° (b) 3 dB and 180°
(c) 6 dB and 90° (d) 3 dB and 90°
[2009]
Solution: (c)
1 1
Gain margin = 20 log = 20 log
x 0.5
= 20 log 2 = 6 dB
and phase margin = 90°
1 Hence, the correct option is (c).
(a)
( s + 1)( s + 20) 13. The asymptotic Bode plot of a transfer function is as
1 shown in the figure. The transfer function G(s) corre-
(b) sponding to this Bode plot is
s( s + 1)( s + 20)
100
(c)
s( s + 1)( s + 20)
100
(d)  [2007]
s( s + 1)(1 + 0.05s)
Solution: (c)
k
k 20
G ( s) = =
s( s + 1)( s + 20)  s 
s(1 + s) 1 + 
 20  1
(a)
Bode plot is in (1 + ST) form. ( s + 1)( s + 20)
= −20 log w + 20 log k = |60 dB|w= 0.1 1
(b)
Solving ⇒ k = 100 s( s + 1)( s + 20)
100 100 100
So G ( s) = = (c)
s( s + 1)(1 + 0.05s) s( s + 1)( s + 20) s( s + 1)( s + 20)
100
(d)  [2007]
s( s + 1)(1 + 0.05s)
Solution: (c)
k
k 20
Hence, the correct option is (c). G ( s) = =
s( s + 1)( s + 20)  s 
Common Data for Question 11 and 12 s(1 + s) 1 + 
 20 
The Nyquist plot of a stable transfer function G(s) is
shown in the figure. We are interested in the stability Bode plot is in (1 + ST) form.
of the closed loop system in the feedback configuration = −20 log w + 20 log k = |60 dB|w= 0.1
shown Solving ⇒ k = 100
11. Which of the following statement is true? 100 100
(a) G(s) is an all-pass filter So G ( s) = =
s( s + 1)(1 + 0.05s) s( s + 1)( s + 20)
(b) G(s) has a zero in the right-half plane
(c) G(s) is the impedance of a passive network
(d) G(s) is marginally stable [2009]
Solution: (b)
From the plot, G(s) has a zero in the right half of plane.
Hence, the correct option is (b).
Hence, the correct option is (c).
3.72 | Control Systems

14. The Nyquist plot of G(jω) H(jω) for a closed loop con- 0.2 k < 1 k < 5
trol system, passed through (-1, j0) point in the GH 2k > 1 k > 0.5
plane. The gain margin of the system in dB is equal to
0.5 < k < 5
(a) infinite (b) greater than zero
(c) less than zero (d) zero [2006] 1 > 8k
Solution: (d) 1
k< (negative sign only shows that it is on
1 8
G.M. = 20 log dB
a negative axis)
a = 1, G.M. = 0 Hence, the correct option is (b).
Hence, the correct option is (d). 1 6. Consider the Bode magnitude plot shown in the figure.
The transfer function H(s) is
1 5. The polar diagram of a conditionally stable system for
open loop gain K = 1 is shown in the figure. The open
loop transfer function of the system is known to be sta-
ble. The closed loop system is stable for

( s + 10)
(a)
( s + 10)( s + 100)
10( s + 1)
(b)
11 11 ( s + 10)( s + 100)
KK <<55and
(a) and << KK
and
22 88 2
(c) 10 ( s + 1)
11 11 ( s + 10)( s + 100)
KK << and
(b) and <<KK <<55
and
88 22
(d) 103 ( s + 100)  [2004]
11
KK << and
(c) and 55<< KK
and ( s + 1)( s + 10)
88
Solution : (c)
1
K < and
(d) and K < 5 [2005] 20 log k = −20 dB
8
⇒k = 10−1 = 0.1
Solution: (b)
Zero at w = 1 and poles at w = 10, 100
k ( s + 1) 100( s + 1)
H ( s) = =
 s  s  ( s + 10)( s + 100)
 10 + 1 100 + 1
  
Hence, the correct option is (c).
17. A system has poles at 0 01 Hz, 1 Hz and 80 Hz; zeros
at 5 Hz, 100 Hz and 200 Hz. The approximate phase of
the system response at 20 Hz is
(a) -90° (b) 0°
(c) 90° (d) -180° [2004]
Solution: (a)
system is stable in region −0.2 to −2 and on the left side Pole at 0.01 and 1 Hz gives −180 phase. Zero at 5Hz
of −8 as number of encirclement there is zero. gives +90 phase.
Chapter 7  Frequency Analysis  |  3.73

∴at 20 Hz −90° phase is provided.


Hence, the correct option is (a).
1 8. The approximate Bode magnitude plot of a minimum-
phase system is shown in the figure. The transfer
function of the system is

[1992]
Solution :
Corner fq = 1 rad/sec
1
=1 ⇒ T = 1
T
y = mx + c
⇒20 = 0 + c ⇒ c = 20
c = 20 log k = 20 k = 10
k 10
T.F = =
1 + TS 1 + S
( s + 0.1)3
108
(a) 2 0. The open-loop transfer function of a feedback control
( s + 10) 2 ( s + 100)
1
( s + 0.1)3 system is G ( s) ⋅ H ( s) =
107
(b) ( s + 1)3
( s + 10)( s + 100)
( s + 0.1) 2 The gain margin of the system is
108
(c) (a) 2 (b) 4
( s + 10) 2 ( s + 100) (c) 8 (d) 16 [1991]
( s + 0.1)3
109
(d)  [2003] Solution: (c)
( s + 10)( s + 100) 2 wpc ⇒ −3tan−1(wpc) = −180°
Solution: (a)
w pc = 3 rad/sec
w = 0.1 to 10, +120 dB change
∴ 3 zeros at 0.1 1 1 1
G .H = x = = =
w = 10 to 100, −40 dB 1 + w 2pc  
3 8
( 3)
2
[i.e., +60 to +20] change  1 + 
 
so two poles at w = 100
1
k ( s + 0.1)3 G.M. = = 8.
⇒ T ( s) = x
( s + 10) 2 ( s + 100)

Hence, the correct option is (c).
k 2 1. From the Nicholas chart one can determine the follow-
⇒ 20 log   = 140
 w  w =10 ing quantities pertaining to a closed loop system:

(a) Magnitude and phase
k (b) Band width
⇒ = 107
w w =10
(c) Only magnitude
(d) Only phase [1989]
∴k = 108 Solution: (a)
Hence, the correct option is (a). Nicholas chart is magnitude versus phase pilot.
19. Bode plot of a stable system is shown in figure. The Hence, the correct option is (a).
transfer function of the system is
3.74 | Control Systems

22. A system has fourteen poles and two zeros. Its high k
frequency asymptote in its magnitude plot has a slope ∴ G ( j 0) H ( j 0) = =2
p1 p2
of:
(a) -40 dB/decade (b) -240 dB/decade k = 2 p1p2
(c) -280 dB/decade (d) -320 dB/decade
k
[1987] ⇒ G ( jω ) H ( jω ) =
( jω + p1 )( jω + p2 )
Solution: (b)

Given P = 14, Z = 2, (P – Z) = 14 – 2 = 12 k
= 2
Slope = −20(P – Z) = −20(12) = −240 dB/dec. −ω + p1 p2 + jω ( p1 + p2 )

Hence, the correct option is (b). ∴ Considering imaginary axis.
10 (Real part) = Re [G(jω) H(jω)] = 0
2 3. The polar plot of G ( s) = intercepts real
s( s + 1) 2 ∴ −ω2 + p1p2 = 0

p1p2 = ω2
axis at ω= ω0. Then, the real part and ω0 are, respec-
tively, given by ω=2
(a) -2.5, 1 (b) -5, 0.5 p1p2 = 22 = 4
(c) -5, 1 (d) -5, 2 [1987] Sub value of p1p2 in equation  (i)
Solution : (c) k = 2p1p2 = 2 × 4 = 8
10 ∴ Sub value of ω, k and p1p2 in equation (ii)
G ( s) = 2
,
s( s + 1) k
∴ G ( jω ) H ( jω ) = 2
∠G = −180 = −90° − 2 tan −1 ( w ) −ω + p1 p2 + jω ( p1 + p2 )

2 tan−1(w) = 45° ⇒ w = 1 8
|G (j2) H (j2)| |= =2
wpc = 1 rad/sec 2( p1 + p2 )
thus
10 10 ∴p1 + p2 = 2 (iii)
G w=w = = =5
pc w (1 + w ) 2 1(1 + 1) ∴To find p1 and p2 values

At w = w the polar plot crosses the negative real axis at ∴ (p1 – p2)2 = Re(p1, p2)2 – p1p2
−5. ⇒ (p1p2)2 = 22 – 4 × 4
Hence, the correct option is (a). ⇒ (p1 – p2)2 = 4 – 16 = −12
⇒ p1 − p2 = j 2 3 (iv)
Five-marks Questions Adding equation (iii) and (iv)
p1 + p2 = 2
1. The Nyquist plot of an all-pole second order open-
p1 – p2 = j2 3
loop system is shown in the figure is obtain the transfer
function of the system. 2 p1 = 2 + j 2 3

= 1+ j 3
Sub value of p1 in equation (iii)
p1 + p2 = 2
⇒ 2 p1 = 2 + j 2 3
p2 = 1 − j 3
8
∴ G ( s) H ( s) =
[ s + 1 + j 3 ][( s + 1 − j 3 ]
Solution: 
System in type 0, order 2
8 8 8
k G ( s) H ( s) = = =
∴ G ( s) H ( s) = 2
( s + 1) − ( j 3 ) 2 2
s + 1 + 2s + 3 2
s + 2s + 4
( s + p1 )( s + p2 )

8 8 8
P1P2 = Poles G ( s) H ( s) = = 2 = 2
2 2
+ 1) − ( j 3 ) s + 1 + 2s + 3 s + 2s + 4
( s
Chapter 7  Frequency Analysis  |  3.75

2. Consider a feedback system with the open-loop trans- Solution:  given


K (i) Initially
fer function. Given by G(s)H(s) = . Examine
s(2s + 1) Slope = −40 dB bidecade
the stability of the closed-loop system using Nyquist 1
stability. ∴No. of poles at origin =2
s2
[1999] (ii) ω1 → 1st corner frequency
k Change on slope at ω1 = +20 dB decade
given G ( s) H ( s) =
Solution: 
s( 2 s + 1) 1
∴ zero at =1
ωi
(iii) V2 → 2nd corner frequency
Change in slope at ω2 = 20 dB/decade
1
So, poles at =1
ω2
Equation between ω1 and ω2
y = −20 log ω + k
0 = −20 log(1) + k
k=0
at ω1
20 = −20 log(ω1) + k
S1 → polar plot ⇒ 20 = −20 log(ω1) + 0
S2 → S = lim Re jθ ω1= 0.1
x →∞
at ω2
∴R = ∞
−20 = −20 log(ω2) + k
+π p
θ= to −20 = −20 log(ω2) + 0
2 2
∴ ω2 = 10
S3 → Inverse Polar plot
On initial line
S4 → S = lim Re jθ 20 = −40 log (ω1) + k1
R →0
R=0 20 = −40 log (0.1) + k1
−π +π k1 = −20
∴θ = 2 to
2 k1 = 20 log (c2)
N=0
⇒ −20 = 20 log (k2)
P=0
c2 = 0.1
N=P–Z
Z=0  2 
c2 1 + 
ω1 
T.F.G ( s) H ( s) = 
given system is stable.
3. The asymptotic Bode plot of the minimum phase open-  s 
s 2 1 + 
loop transfer function G(s) H(s) is as shown in figure.
 ω2 
Obtain the transfer function G(s) H(s)
 s 
0.11 + 
=  0.1 
 s 
s 2 1 + 
 10 
10( s + 0.1)
T.F. =
s 2 ( s + 10)

4. The loop transfer function of a single loop control sys-
tem is given by
3.76 | Control Systems

100 y = 6180
G(s)H(s) = e-sT. Using Nyquist criterion,
s(1 + 0.01s) ∴ As y = ω2
find the condition for the closed loop system to be ω2 = 6180
stable [1998] ωgc = 70.6 rad/sec
Solution: π
− tan −1 (ω gc 0.01)
φ = ω gcT −
100 / ω 1 + (0.01ω ) 2 2
G ( s) H ( s) = =1 −1
φ = −78.67 − 90° − tan (78.6 × 0.01)
ω = wge
φ = −78.67 − 90° − 38.17°

⇒ ω 1 + (0.01ω ) 2 = 100 = −78.67 − 128.17°

p.m. = 180 + φ
⇒ ω 2 (1 + (0.01ω ) 2 ) = 10000
= 180 + (−78.67 – 128.17°)
⇒ ω 2 (!+ 0.0001ω 2 ) = 10000 If pm is +ve, system becomes stable.

⇒ 0.0001ω4 + ω2 – 10000 = 0 Pm > 0
let ω2 = y 180° – 128.17° – 78.67 > 0
0.0001 y2 + y – 10000 = 0  180 
⇒ −78.67  > −51.83
 π 
−1 ± 1 + 4
∴y =
0.0002 T > 0.0115

−1 ± 5
y=
0.0002
Chapter 8
State Space Analysis
2. The state variable representation of a system is given as
One-mark Questions
⎡0 1 ⎤ ⎡1 ⎤
x = ⎢
− ⎥ x; x ( 0 ) = ⎢ 0 ⎥
 •
 ⎣ 0 1⎦ ⎣ ⎦
1. Consider the state space  x1 (t )  =
realization
•  y = [0 1] x
 x2 (t )  
0 0   x1 (t )   0 
0 −9   x (t )  +  45 u(t ), with the initial condition The response y(t) is [2015]
  2    (A) sin(t) (B) 1 – et
 x1 (0)  0  (C) 1 – cos(t) (D) 0
  =   , where u(t) denotes the unit step func- Solution:  From the given state equation
 x2 ( 0 )   0 
⎡0 1 ⎤ ⎡1⎤
tion. The Value of lim
x →∞
x12 (t ) + x22 (t ) is _________. A= ⎢
0 −1⎥ ; x (0 ) = ⎢ 0 ⎥
⎣ ⎦ ⎣ ⎦
 [2017]
Solution: 
From given state space realization, y = [0 1] x


x1 (t ) = 0 ∴ y = x2

x2 (t ) = −9 x2 (t ) + 45 u(t ) (1) x(t) = F(t) ⋅ x(0)
Apply Laplace transfrom to equation (1)
F(t) = L–1 {(SI – A)–1}
SX 1 ( s) − x1 (0) = 0 ⇒ X 1 ( s) = 0 (2)
⎡ s −1 ⎤
1 ⎢ ⎥ = (SI – A)
SX 2 ( s) − x2 (0) = −9 x2 ( s) + 45   ⎣0 s + 1⎦
5
45 1 ⎡ s + 1 1⎤
( S + 9) X 2 ( S ) = F=
S s ( s + 1) ⎢⎣ 0 s ⎥⎦

45 A B 5 5
X 2 (S ) = = + = − ⎡1 s 1 s ( s + 1)⎤
S ( S + 9) S S + 9 S S + 9 F=⎢ ⎥
⎣ 0 1 ( s + 1) ⎦ 
Apply inverse LT to equations (2) and (3)
⎡1 S 1 s ( s + 1)⎤ ⎡1 ⎤
x1 ( t ) = 0; X (s) = ⎢ ⎥
⎣ 0 1 ( s + 1) ⎦ ⎢⎣0 ⎥⎦

x2 ( t ) = 5 u ( t ) − 5e −9t u ( t )
⎡1 s ⎤ ⎡ x ⎤
X(s) = ⎢ ⎥ ⎢ 1 ⎥
Lim x12 (t ) + x22 (t ) ⎣ 0 ⎦ ⎣ x2 ⎦ 
t →∞

x(t) = 1 ⋅ x1(t) + 0 ⋅ x2(t)


lim x2 (t ) = 5
x →∞
∴ y = 0
Hence, the correct answer is (5).
Hence, the correct option is (D).
3.78 | Control Systems

1 0  Solution: (d)
dx
3. Consider the system = Ax + Bu with A = 0 1  x (t ) = 2 x(t ) + 2u(t )
(1)
dt  
 p y(t) = 0.5 x(t) (2)
and B =   where p and q are arbitrary real numbers.
q taking Laplace transform of (1)
Which of the following statements about the control- s × (s) = −2 × (s) + 2u(s)
lability of the system is true? 2u( s)
(a) The system is completely state controllable for any ⇒ x ( s) =
( s + 2)
nonzero values of p and q
(b) Only p = 0 and q = 0 result in controllability Laplace transform of (1)
(c) The system is uncontrollable for all values of p and y(s) = 0.5 × (s)
q 0.5 × 2u( s)
(d) We cannot conclude about controllability from the y ( s) =
( s + 2)
given data [2009]
Solution: (c) y ( s) 1
∴ =
1 0   p u( s) ( s + 2)

A=  ; B =  
0 1  q 
Hence, the correct option is (d)
Controllability, QC = [BAB … An – 1B] ≠ 0 6. The system mode described by the state equations
1 0   p   p  0 1  0 
A. B =    =   x =   x +   u, Y = 1 1 x is
0 1   q   q   2 −3 1 
 p p (a) controllable and observable
So Qc =   = 0 ⇒ uncontrollable, for all p and q (b) controllable, but not observable
q q (c) observable, but not controllable
Hence, the correct option is (c) (d) neither controllable nor observable [1999]
4. A linear system is equivalent represented by two sets of Solution: (a)
state equations. X = AX + BU and W = CW + DU.
Q0 = [BABA2B − An–1B]
The eigen values of the representations are also com-
puted as [λ] and [μ]. Which one of the following state- 0 1
A=  
ments is true?  2 −3 

(a) [λ] = [μ] and X = W
(b) [λ] = [μ] and X ≠ W 0 
B =   ; C = [1 1]
(c) [λ] ≠ [μ] and X = W  1
(d) [λ] ≠ [μ] and X ≠ W [2005]  1
Solution: (a,b) AB =  
 3
Eigen value of A = [λ]
Eigen value of W = [μ] 0 1
∴ Q0 =   ≠0
The eigen value of a system are always unique.  1 −3 2× 2

So [λ] = [μ] ∴ Order = 2, rank = 2
But a system can be represented by different state mod- Controllable
els having different sets of state variables 2
T T T T T
x=w Q0 = [C A C C A ...]
x ≠ aw 1 0 2 
Both are possible conditions. C T =   , AT =  

1
  1 −3
5. The transfer function Y(s)/U(s) of a system described
by the state equations x (t ) = −2 x )(t ) + 2u(t ) and y(t) = 1 2 
∴ Q0 =  ≠0
0 5x(t) is 1 −2 

(a) 0.5/(s − 2) (b) 1/(s − 2)
rank 2 observable.
(c) 0.5/(s + 2) (d) 1/(s + 2) [2002]
Hence, the correct option is (a)
Chapter 8  State Space Analysis  |  3.79

⎡ −4 −1.5⎤ ⎡2⎤
Two-marks questions X = ⎢ ⎥ X + ⎢ ⎥U , 
⎣4 0 ⎦ ⎣0 ⎦
1. Let the state-space representation of an LTI system be
X (t) = Ax(t) + Bu(t), y(t) = Cx(t) + du(t) where A, B, C Y = [1.5 0.625] x.
are matrices, d is a scalar. u(t) is the input to the system
and y(t) is its output. Let B = [0 0 1]T and d = 0. Which The transfer function representation of the system is:
one of the following options for A and C will ensure  [2018]
that the transfer function of this LTI system is H(s) = 3s + 5 3s − 1.875
(A) 2 (B)
1 s + 4s + 6 s2 + 4s + 6
?  [2019]
s 2 + 3s 2 + 2 s + 1 4 s + 1. 5 6s + 5
(C) 2 (D)
⎡ 0 1 0 ⎤ s + 4s + 6 s2 + 4s + 6
(A) A = ⎢ 0 0 1 ⎥ and C = [0 0 1] T.F = C(SI – A)–1 ⋅ B + D
Solution: 
⎢ –3 –2 –1⎥
⎣ ⎦ −1
⎡ S + 4 1.5⎤
(SI – A) = ⎢
–1
⎡ 0 1 0 ⎤
⎣ −4 S ⎥⎦
(B) A = ⎢ 0 0 1 ⎥ and C = [0 0 1] 
⎢ –1 –2 –3⎥
⎣ ⎦ 1 ⎧S −1.5 ⎫
= ⎨ ⎬
S ( S + 4) + 6 ⎩ 4 S + 4 ⎭
⎡ 0 1 0 ⎤ 
(C) A = ⎢ 0 0 1 ⎥ and C = [1 0 0]
⎢ –1 –2 –3⎥ ⎡ S −1.5 ⎤
⎣ ⎦ ⎢ S 2 + 4S + 6 S 2 + 4S + 6 ⎥
 =
⎢ ⎥ = [1.5 0.625]
⎡ 0 1 0 ⎤ ⎢ 4 S+4 ⎥
(D) A = ⎢ 0 0 1 ⎥ and C = [1 0 0] ⎢⎣ S 2 + 4 S + 6 S 2 + 4 S + 6 ⎥⎦
⎢ –3 –2 –1⎥
⎣ ⎦
⎡ S −1.5 ⎤
Solution: X (t ) = Ax (t ) + Bu (t ) ⎢ S 2 + 4S + 6 S2 + 4S + 6 ⎥ ⎡ 2⎤
⎢ ⎥
Y (t ) = Cx (t ) + Du (t ) ⎢ 4 S + 4 ⎥ ⎢⎣0 ⎥⎦
⎢⎣ S 2 + 4 S + 6 S 2 + 4 S + 6 ⎥⎦
0  
B = 0  d = 0 ⎡ 2S ⎤
⎢ S 2 + 4S + 6 ⎥
1  =
[1.5 0.625] ⎢ ⎥

⎢ 8 ⎥
1 ⎢⎣ S 2 + 4 S + 6 ⎥⎦
H ( s) = 3 2 
S + 3S + 2S + 1
3S + 5
From the standard four =
S + 4S + 6 
2
Y (S ) b0
= Hence, the correct option is (A)
U ( S ) a3 S + a2 S 2 + a4 S + a0
3
3. A second-order LTI system is described by the follow-
ing state equations,
 0 1 0  0 d
    x1 (t ) − x2 (t ) = 0
A=  0 0 1  ; B =  0  ; C = [1 0 0 ] dt
 − a0 − a1 − a2   b0  d
x2 (t ) + 2 x1 (t ) + 3 x2 (t ) = r (t )
dt
On complaring we get
where x1(t) and x2(t) are the two state variables and r(t)
0 1 0 denotes the input. The output c(t) = x1(t). The system is
A =  0 0 1  C = [1 0 0 ]  [2017]
 −1 −2 −3 (A) undamped (oscillatory)
 (B) underdamped
Hence, the correct option is (C). (C) critically damped
2. The state equation and the output equation of a control (D) overdamped
system are given below:
3.80 | Control Systems

Solution: 
From given state equation  −1 1 0  0 
•    
x1 (t ) = x2 (t ) A =  0 −1 0  , B =  4  , C = 1 1 1
•  0 0 −2   0 
x 2 (t ) = −2 x1 (t ) − 3 x2 (t ) + 1r (t )
Controllability
0 
i
  X (t ) +   r (t )  0 4 −8
x (t ) =  0 1  1  
Q0 =  B : AB : A B  =  4 −4 4 
2 
 −2 −3  
   0 0 0 

 
A= 0 1  |Q0| = 0
 −2 −3 ∴ uncontrollable.
 
Characterstic
  is S I − A = 0
  equation Observability:
 S 0 − 0 1  =0 1 −1 1 
 0 S   −2 −3  
S 0   0 1  = 0 T T T 2T
Q0 = [C : C C : C C ] = 1 0 −1T
 − 
 0 S   −2 −3 1 −2 4 
   
|Q0| = 1 ∴ observable
 
 S −1  = 0 Hence, the correct option is (b)
 2 S + 3
 S −1  5. An unforced linear time invariant (LTI) system is rep-
  =0
 2 S + 3 resented by
  equation is : S2+ 3S + 2 = 0
Charactersitic
 x   −1 0   x1 
Roots of characterstics equation are, S = -1, -2  1 =    .
\ The system is overdamped, ∵Poles are (-)ve real  x2   0 −2   x2 
and unequal. If the initial conditions are x1(0) = 1 and x2(0) = -1, the
Hence, the correct option is (D). solution of the state equation is
(a) x1(t) = –1, x2(t) = 2
4. Consider the state space model of a system, as given
(b) x1(t) = –e–t, x2(t) = 2e–t
below.
(c) x1(t) = –e–t, x2(t) = –2e–t
 x1   −1 1 0   x1   0  (d) x1(t) = –e–t, x2(t) = –2e–t [2014]
       
 x2  =  0 −1 0  +  x2  +  4  Solution: (c)
 x3   0 0 −2   x3   0   x1   −1 0   x1 
  =   
 x1  x 0 −2   x2 
 2 
u; y = [1 1 1]  x2 
 
x1 = − x1 − 2 x2 (1)

 xs3 
x2 = −2 x2 (2)

The system is
(a) controllable and observable Applying Laplace transform in equations (1) and (2)
(b) uncontrollable and observable ⇒ sx1 = 1(0) −x1(s) {x1(0) = 1}
(c) uncontrollable and unobservable 1
(d) controllable and unobservable[2014] So, x1 ( s) =
s +1
Solution: (b)
or e−t = x1(t)
 x1   −1 1 0   x1   0  ˙u s
⇒ sx ) = = x2(0) = −2x2(s)
x(2s(s) {x2(0) = −1}
       ˙s +
 x2  =  0 −1 0   x2  +  4  4 2u1( s)
 x   0 0 −2   x3   0  ⇒ x2( s( )s)== x (t) = e−2t
 3  ( s + 2)
2 2
 x1  Hence, the correct option is (c)
 
y = 1 1 1  x2  6. Consider the state space system expressed by the signal
 x3  flow diagram shown in the figure.

Chapter 8  State Space Analysis  |  3.81

7. The state equation of a second-order linear system is


given by
x (t ) = Ax(t ), x(0) = x0
1  e −t  0 
For x0 =   , x(t ) =  −t  and for x0 =   ,
 −1  −e  1 
The corresponding system is
 e −t − e −2t 
(a) always controllable x (t ) =  .
−t −2t
(b) always observable  −e + 2e 
(c) always stable
(d) always unstable [2014] 3
When x0 =   , x(t) is
Solution: (a) 5 
The stage equation from the state diagram.  −8e −t + 11e −2t   −t −2t 
(a)   (b)   11e − 8e 
x1 = 0

 8e −t − 22e −2t   −11et + 16e −2t 
x2 = 0

 −t −2t   −t −2t 
(c)  3e − 5e  (d)   −5e + 6e 
x3 = a3 x3 + a2 x2 + a1 x1 + 4

 −3e −t + 10e −2t   −5e −t + 6e −2t 
also y = c1x1 + c2x2 + c3x3 [2014]
Thus, state matrix. Solution: (b)
 x1   0 0 0   x1  0  Given x (t ) = Ax(t ), x(0) = x0
      
 x2  =  0 0   x2  +  0 
0 Taking Laplace transform
 x   a a2 a3   x3  1  sx(s) – x(0) = Ax(s)
 3  1
[SI – A] x(s) = x(0)
 x1 
  x(s) = [SI – A]−1x(0)(1)
y = c1 c2 c3   x2 
 x3  1  e −t 
for, y0 =   , x(t ) =  
 −1  −e −t 
System is controllable if the rank of controllabil-
ity matrix Qc is equal to the rank of state matrix A. 0   e −t − e −2t 
However, if Qc is a square matrix the condition is |Qc| ≠ for y0 =   , x(t ) =  −t 
0 1   −e + 2e −2t 
Qc = [B : AB : A2B] using linearity property in equation (1)
0 0 0 k1x1(t) = L−1[SI – A]−1x1(0) k1
 
Qc = 0 0 0 k2x2(t) = L−1[SI – A]−1x2(0) k2
  Using Linearity Property
1 a3 a32 

k1 x1 (t ) + k2 x2 (t ) = L−1[SI − A]−1 (2)
rank of Qc = rank if A = 1 controllable system.
[k1x1(0) + k2x2 (0)]
Observability: The system is said to be observable if
the rank of observability matrix Q0 is equal to rank of again x3(s) = [SI – A]−1x3(0)
state matrix A, for square Q0 the observability condi- 1  0  3
tion is |Q0| ≠ 0. So, k1   + k2   =  
 −1  1  5 
Q0 = [CT : ATCT : (AT)2CT]
 c1 a1c3 a1a3c3   k 0 + 0 k 2  3 k1 = 3
⇒ 1 = ⇒
   −k1 + k2  5 k2 = 8
∴ Q0 = c2 c2 c3 a2 a3c3 
 c3 a3c3 a3 a3c3  From (2) x(t) = k1x1(t) + k2x2(t)

rank of Q0 ≠ Rank of A, also |Q0| = 0 the system is not  e −t   e −t − e −2t 
= 3  +8 
observable.
 −e −t   −e −t + 2e −2t 
Hence, the correct option is (a)
3.82 | Control Systems

 11e −t − 8e −2t  x2 = x1 − x2 + 4


= 
 −11e −t + 16e −2t  y = x2 = x1 − x2 + 4
 x   −1 0   x1   −1
Hence, the correct option is (b)  1 =   + 4
 x2   1 −1  x2   1 
8. The state transition matrix ϕ(t) of a system
 x1  0 1   x1   −1 0   −1
  =  or x =  x+ 4
   is
 x2   0 0   x2   1 −1 1
Hence, the correct option is (a)
t 1  1 0 
(a)
  (b)
  10. The state transition matrix eAt of the system shown in
1 0  t 1  figure above is
0 1 1 t   e −t 0   e −t 0 
(c)
  (d)
  [2014] (a)  −t  (b)  −t 
 1 t  0 1 te −t
e   −te e −t 
Solution: (d) e −t 0  e −t −te −t 
 −t
(c)  (d)
 
 x  0 1   x1  e e −t   0 e −t 
given,  1  =   
 x2   0 0   x2   [2013]
0 1   s −1 Solution: (a)
A=   ,  SI − A =  
0 0  0 s   −1 0 
A=    1 
[SI – A] = s2  1 −1  0 
1  s + 1 0   ( s + 1) 
[SI − A]−1 =   =
 1  s 1   1 +  ( s + 1)( s + 1)  1 s + 1  1 1 
f (t ) = L−1 [ SI − A]−1 = L−1

 2  =    2 
( s + 1) 
 s 0 s   0 1   ( s + 1)
 1 
9. The state-variable equations of the system in the figure 0 
− 1  s + 1 0   ( s + 1) 
above are 1
[SI − A] =   =  1
 −1 0   −1 ( s + 1)( s + 1)  1 s +
1 1 
(a) X =   X +  u  2 
( s + 1) 
 1 −1 1  ( s + 1)
y = 1 −1 X + u {
f (t ) = e At = L−1 ( SI − A) −1 }
 −1 0   −1  e −t 0 
X = 
(b)  X +  u e At =  
 −1 −1 1 te −t e −t 
y =  −1 −1 X + u
Hence, the correct option is (a)
 −1 0   −1 11. The state variable description of an LTI system is given
X = 
(c)  X +  u by
 −1 −1 1
y =  −1 −1 X − u  x1   0 0   x1   0 
a1
      
 x2  =  0 a2   x2  +  0  u
0
 −1 −1  −1  x   a 00   x3   1 
X = 
(d)  X +  u  [2013]  3  3
 0 −1 1  x1 
y = 1 −1 X − u
y = (1 0 0 )  x2 
x 
Solution: (a)  3
where y is the output and u is the input. The system is
controllable for
(a) a1 ≠ 0, a2 = 0, a3 ≠ 0
a1 = 0, a2 ≠ 0, a3 ≠ 0
(b)
So, x1 − x1 − 4 a1 = 0, a2 ≠ 0, a3 = 0
(c)
x2 = −( x2 + x1 ) = −( x2 − x1 − 4) a1 ≠ 0, a2 ≠ 0, a3 = 0 
(d) [2012]
Chapter 8  State Space Analysis  |  3.83

Solution: (d) x = [2]x + [1]u;


(a) y = 1 2  x
 x1   0 a1 0   x1  0  1 
       x = [−2]x + [1]u;
(b) y = x
 x2  =  0 0 a2   x2  + 0  u
2
 x3   a3 0 0   x3  1 
 −2 0  1
x = 
(c)  x +   u; y = 1 2  x
 x1   0 −2  1
 
y = 1 0 0   x2 
2 0 1  1 
 x3  (d)
x =   x +   u; y = x [2011]
 0 2  2 2
QC = [BABA2B](1)
Solution: (b)
0 a1 0  0   0 
     y1 ( s) 1 y ( s) 2
AB =  0 0 a2  0  =  a2  = ; 2 =
 a3 u( s) s + 2 u( s) ( s + 2)
0 0  1   0 

y1 ( s) x1 ( s) 1
0 a1 0   0   a1a2  . =
     x1 ( s) u( s) s + 2
2
AAB = A B =  0 0 a2   a2  =  0 

 a3 0 0   0   0  y2 ( s ) y2 ( s )
=
2
y2 ( s) u( s) ( s + 2)
0 0 a1a2 
  x1 ( s) 1 x ( s)
Qc = 0 a2 0  ⇒ = and 1 =1
1 0 0  u( s) s + 2 x1 ( s)

for controllable system |Q0| ≠ 0 y2 ( s ) 1 y ( s)
= and 2 =2
(0 − a1aa2 )
≠0 u( s) s+2 u( s)

sx1 + 2x1(s) = u( )
or
and y1(s) = x1(s)
or a1 ≠ 0
sx2(s) + 2x2(s) = u(s)
a2 ≠ 0
y2(s) = 2x2(s
Hence, the correct option is (d) x1 (t ) + 2 x1 (t ) = u(t ) )
Statement for Linked Answer Questions 12 and 13: y2(t) = 2x2(t)
The state diagram of a system is shown below described T T 1 
So y =  y1 y2  = 1 2  =  
by the state-variable equations. 2
X = AX + Bu;
x2 ( t ) + 2 x2 ( t ) = u ( t )
y = CX + Du 

x1 (t ) + 2 x2 (t ) = u(t )
 x1   −2 0   x1  1
  =     +   u (t )
x −2 0   x2  1
 2 
12. The block diagram of a system with one input u and
or x = [−2]x + [1]4
two outputs y1 and y2 is given below.
Hence, the correct option is (b)

Common Data for Questions 13 and 14:


The signal flow graph of a system is shown below:

A state space model of the above system in terms of the


T
state vector x and the output vector y =  y1 y2  is
3.84 | Control Systems

13. The state variable representation of the system can be 15. A signal flow graph of a system is given below.
 1 1 0 
x = 
(a)  x +  u
 −1 0  2
y = 0 0.5 x

 −1 1  0 
x = 
(b)  x +  u
 −1 0  2
y = 0 0.5 x

 1 1 0 
x = 
(c)  x +  u
 −1 0  2 The set of equations that correspond to this signal flow
y = 0.5 0.5 x graph is

 −1 1  0   x1   b −g 0   x1  1 0 
x = 
(d)  x +  u  [2010] (a) d        u 
 −1 0  2 x2 = g a 0   x2  +  0 0   1 
dt    u
y = 0.5 0.5 x  x3   −a −b 0   x3  0 1   2 

Solution: (b)  x1  0 a g   x1  0 0 
d        u 
 1 1 0 
(b)  x2  = 0 −a −g   x2  + 0 1   1 
dt   u
x=  x +  u  x3  0 b − b   x3  1 0   2 
 1 0 2
y = [0 0.5]x  x1   −a b 0   x1  1 0 
d        u 
(c) x2 = − b −g 0   x2  +  0 1   1 
Hence, the correct option is (b) dt    u
 x3   a g 0   x3  0 0   2 
14. The transfer function of the system is
 x1   −g 0 b   x1  0 1 
s +1 s −1 (d)  x2  =  g
d     u 
(a) 2 (b) 0 a   x2  +  0 0   1 
s +1 s2 + 1 dt    u
 x3   − b 0 −a   x3  1 0   2 
s −1 [2008]
(c) s + 1 (d) 
2
2
s + s +1 s + s +1 Solution: (d)
 [2010]  x1   −g 0 b   x1  0 1 
d        u 
x2 = g 0 a   x2  +  0 0   1 
dt   
Solution: (c)
u
x −b 0 −a   x3  1 0   2 
forward path gain  3  
 1  1  1 Hence, the correct option is (d)
P1 = 2     (0.5) = 2
s
   s s

Common Data for Questions 13 and 14:
1 1
P2 = 2   (1)(0.5) = Consider a linear system whose state space rep-
s
  s
resentation is x (t ) = Ax(t ) . If the initial state
Δ1 = 1 1
vector of the system is x(0) =   , then the
Δ2 = 1  −2 
 1 1 1 1  e −2t 
∆ = 1 − − − 2  = 1 + + 2 system response is x(t ) =   . If the initial
 s s  s s  −2e −2t 
P1∆1 + P2 ∆ 2 s +1 1
T.F = = 2 state vector of the system changes to x(0) =   .
∆ s + s +1 then the system response becomes  −1
Hence, the correct option is (c)  e −t 
x (t ) =  
 −e −t 

Chapter 8  State Space Analysis  |  3.85

16. The eigen-value and eigen-vector pairs (λi, vi) for the Solution: (a)
system are  dw 
 1   1    dt   −1 1   w   0 
(a)  −1,    and  −2,     =  + 4
 1   2   dia   −1 −10  ia  10 
 dt 
  1    1 
 −2,    and  −1,   
(b) dw
⇒ = −w + ia (1)
  −1    −2   dt
dia
  1    1  = −w − 10ia + 10u (2)
 −1,    and  −2,   
(c) dt
  −1    −2   taking Laplace transform (1) and (2)
sw(s) = −w(s) + Ia(s)
  1    1 
⇒ (s + 1) w(s) = Ia(s) (3)
 −2,  −1  and 1,  −2  
(d)  [2007]
      ⇒ sIa(s) = −w(s) – 10Ia(s) + 10 u(s)
⇒ [s2 + 11s + 11]w(s) = 10 u(s)
Solution: (a)
w ( s) 10
Sum of the eigen value = trace of the principle diagonal =
u( s) ( s 2 + 11s + 11)
matrix.
Sum = −3, only (a) satisfies.
Hence, the correct option is (a)
Hence, the correct option is (a) 19. A linear system is described by the following state
equation
17. The system matrix A is
 0 1
 0 1 X (t ) = AX (t ) + BU (t ),
A 
1 1  −1 0 
(a)
  (b)
 
 −1 1  −1 −2  The state-transition matrix of the system is
1 1 0 1  cot t sin t 
(c)
  (d)
  (a)  
 −1 −2   −2 −3  − sin t cos t 
 [2007]  − cos t sin t 
(b)
 
Solution: (d)  − sin t − cos t 
Multiplication of eigen values = determinant of matrix
 − cos t − sin t 
∴ Det = 2 satisfies by last option. (c)
 
 − sin t cos t 
Hence, the correct option is (d)
cos t − sin t 
18. The state space representation of a separately excited
(d)   [2006]
cos t sin t 
DC servo motor dynamics is given as
Solution: (a)
 dw  −1 −1
 dt   −1 1  w   0  f (t ) = L [SI − A]
 =   +  u −1 −1
 dia   −1 −10  ia  10   s 0  0 1 
−1 −1  s −1
 dt  = L  −  = L  
 0 s   −1 0   1 s 
where w is the speed of the motor, ia is the armature  s 1 
current and u is the armature voltage. The transfer  2 
2
s + 1 s + 1  cos t sin t 
w( s) = L−1  [ D = s 2 + 1] =  
function of the motor is  −1
U ( s)
s   − sin t cos t 
 2 2 
 s + 1 s + 1
10 1
(a) 2 (b) Hence, the correct option is (a)
s + 11s + 11 2
s + 11s + 11 20. The state variable equations of a system are:
1 1. x1 = −3 x1 − x2 + u
(c) 10 s + 10 (d) 2
 [2007]
2
s + 11s + 11 s + s +1 2. x2 = 2 x1 y = x1 + u
3.86 | Control Systems

The system is  x1  1 0   x1   x1 (0)  1 


(a) controllable but not observable   =     and   =   is
 x2  1 1   x2   x2 (0)  0 
(b) observable but not controllable
(c) neither controllable nor observable tet  et 
  (b)
(a)  
(d) controllable and observable [2004]  t   t 
Solution: (d)  et  t 
(c)  t  (d)
 t [2003]
 x1   −3 −1  x1   1 te  te 
  =   + u
 x2   2 0   x2  0  Solution: (c)

 x   1 ( SI − A) =  s 0  − 1 0  =  s − 1 0
y = 1 0   1  +   u      
 x2   0  0 s  1 1  −1 s − 1

( s − 1) 0  1 0
 −3    ( s − 1) 
QC = [BAB], AB =   1 ( s − 1)  
 2 ( SI − A) −1 = = 
( s − 1) 2  1 1 
 1 −3  2 
( s − 1) 
QC =  ≠0  ( s − 1)
 0 2
 et 0
L−1[SI ℘A]−1 e At  t
∴ Controllable, t
˙ 
 1  −3 2 
Q0 = C T AT C T  : C T =   , AT =    et 0   1  et 
 
0   −1 0  ⇒ x(t ) = e At [ x(0)] =    =  
tet 0  0  tet 
 −3
AT C T =   Hence, the correct option is (c)
 −1
23. For the system described by the state equation
 1 −3
∴ Q0 =  ≠0  0 1 0 0 
0 −1    
x =  0 0 1  x + 0  u
Observable. 0.5 1 2  1 

Hence, the correct option is (d) If the control signal is given by u = [-0.5 -3 -5] x + v,
1 0  then the eigen values of the closed-loop system will be
21. Given A =   , the state transition matrix (a) 0, -1,-2 (b) 0, -1, -3
0 1  (c) -1, -1,-2 (d) 0, -1, -1 [1999]
eAt is given by
 0 e −t  Solution: (a)
et 0 
(a)  −t  (b)    0 1 0 0 
e 0   0 et  x =  0 0 1 x + 0  [0.5 − 3 − 5]x + v
   
e −t 0   0 et 
(c)   (d)  t  [2004] 0.5 1 2   1
 0 e −t  e 0  0 1 0
Solution: (b)  
∴ x = 0 0
 1 x + v
 s 0  1 0   s − 1 0  0 −2 −3
[SI − A] =  − = 
0 s  0 1   0 s − 1
Char. equation
 1  = λ3 + 3λ2 + 2λ + 0 = 0
 s −1 0   t
e 0 λ = 0, –1, –2.
e At = [SI − A]−1 = = 
 0 1   0 et 
 Hence, the correct option is (a)
s − 1 
24. A certain linear time invariant system has the state and
Hence, the correct option is (b) the output equations given below:
22. The zero-input response of a system given by the state-
space equation
Chapter 8  State Space Analysis  |  3.87

 x1  1 −1  x1  0   −1 0   1  −1


  =   + u ⇒ AT C T =    =  
 x2  0 1   x2  1   0 −2   2   −4 

x   1 −1
y = [1 1]  1  is Q0 = C T AT C T  =  ⇒
 
 x2   2 −4 

Q0 = −4 − ( −2) = −2 ≠ 0
dy
xl(0) = 1, x2(0) = -1, u(0) = 0 then is So, the given system is observable
dt t =0
(a) 1 2 6. A liner second-order single-input continuous-time
(b) -l system is described by following set of differential
(c) 0 equations
(d) None of the above [1997] x1 (t ) = −2 x(t ) + 4 x2 (t ); x2 (t ) = 2 x1 (t ) − x2 (t ) + u(t )
Solution: (a) x1 (t ) = −2 x(t ) + 4 x2 ( t ); x2 (t ) = 2 x1 (t ) − x2 (t ) + u(t ) , Where x1(t) and x2(t) are
x1 = x1 − x2
 the state variables and u(t) is the control variable. The
system is
x1 (0) = x1 (0) = x2 (0) = 1 − ( −1) = 2
 (a) controllable and stable
(b) controllable but unstable
x2 = x2

(c) uncontrollable and unstable
[1991]
x2 (0) = x2 (0) = −1
 (d) uncontrollable but stable.
Solution: (b)
y = x1 + x2
 x1   −2 4   y1  0 
dy dy
= x1 (0) + x2 (0) = 2 − 1 = 1   =     +   u (t )
= x1 + x2 , dt x +2 −1  y2   1
 2 
dt t =0
Qc =  B AB 
Hence, the correct option is (a)
25. A linear time-invariant system is described by the state 0   −2 4  0   4 
B =   ; AB =    =  
variable model  1  +2 −1  1  −1
 x1   −1 0   x1  0  0 4 
  =    +  u Q0 =  
x 0 −2   x2  1 
 2   1 −1
x  Q = 0−4 = 4 ≠ 0
Y = [1 2]  1  0
 x2 
Hence, the given system is controllable characteristics
(a) The system is completely controllable. equation
(b) The system is not completely controllable. [SI – A] = 0
(c) The system is completely observable.
(d) The system is not completely observable. [1992] ( s + 2) −4 
 =0
Solution: (b, c)  −2 s + 1

 −1 0  0  (s + 2) (s + 1) – 8 = 0 s = 4.37, 1.37
A=  , B =  
 0 −2   1 Since, pole lies at right half of s plane so system is
unstable.
 −1 0  0   0 
AB =    =  
 0 −2   1  −2  Hence, the correct option is (b)

27. Given the following state-space description of a system
0 0 
Qc =  B AB  =  ⇒  x1   −2 0  0 
 1 −2    =  s+  u
x 0 −4 1 
 2  
|Qc| = 0 – 0 = 0
So, system is not controllable. x 
y = [0 1]  1 
 −1 0  T 1   x2 
AT =   C = [1 2], C =  2 
 0 −2    Find the state-transition matrix. [1988]
3.88 | Control Systems

2. A certain linear, time-invariant system has the state and


Solution: A =  −2 0 
  output representation shown below:
 0 −4 
s + 2 0  i 
SI − A =    x1  =  −2 1   x1   1  u
 0 s + 4  1
 i   0 −3  x   0 
0
 x2    2 
1 s + 4 0  s + 2 
[SI − A]−1 =  =  x 
( s + 2)( s + 4)  0 s + 2   0 1  y = 1 1  1 
 s + 4   x2 
 1 0
1  s + 4 0   s+2  (a) Find the eigen values (natural frequencies) of the
[SI − A]−1 =  = 
( s + 2)( s + 4)  0 s + 2   0 1  system.
 s + 4  (b) If u(t) = δ(t) and x1(0+) = x2(0+) = 0, find x1(t), x2(t)
State transition matrix and y(t), for t > 0.
f (t ) = e At = L−1{[SI − A]−1} (c) When the input is zero, choose initial conditions
x1(0+) and x2(0+) such that y(t) = Ae-2t for t > 0.
e −2t 0  [2000]
f (t ) = e At =  − 
 0 e t4
 x1   −2 1   x1   1 
Solution:    =   + 4
 x2   0 −3  x2   0 
Five-marks Questions x 
y = 1 1  1 
 x2 
1. The block diagram of a linear time invariant system is
given in the figure is (a) |S – A| = 0
 s + 2 −1 
 =0
 0 s + 3

⇒ (s + 2) (s + 3) – 0 = 0
⇒ (s + 2) (s + 3) = 0
∴ s = −2, −3
(b) State transition matrix
 s + 2 −1 
(a) Write down the state variable equations for the sys-  S − A =  
 0 s + 3
tem in matrix form assuming the state vector to be
[x1(t)x2(t)]T. −1 1 s + 3 1 
⇒  S − A =  
(b) Find out state transition matrix. ( s + 2)( s + 3)  0 s + 2
(c) Determine y(t), t ≥ 0, when the initial val-  1 1 
ues of the state at time t = 0 are x1(0) = 1,and  s + 2 ( s + 2)( s + 3) 
x2(0) = .1 [2002] = 
 1 
Solution: From the figure given  0 
 s + 3 
x2′ (t ) = −2 x2 (t ) − u(t ) φ (t ) = e = L ( s − A) −1 
AT −1 
 
′ = − x1 (t ) + x2 (t ) + u(t )
x1 (t ) e −2 x . e −2t − e −3t 
y(t) = x1(t) + x1(t) e AT =  
 0 e −3t 
State equation
∴As x(t ) =  L {S − A}  x(0) + L ( S − A) f s .4( s) 
−1 −1 −1 −1
 x   −1 1   x1   1 
∴ 1  =   + 4
x 0 −2   x2   −1  1 1 
 2   ( s + 2)
−1 ( s + 2)( s + 3)  1 
∴ x1(t) = 2e−t − e−2t ⇒  S − A . B =   
 1  0 
x2(t) = e−2t  0 
 ( s + 3) 

y(t) = x1(t) + x2(t) = 2e−t − e−2t + e−2t
y(t) = 2e−t
Chapter 8  State Space Analysis  |  3.89

 1  Apply KVL in Loop 1


= s+ 2 diL
= e(t ) − 2i1 = e(t ) − 2(iL − i2 )
 

 0  dt 1

diL1
L−1{( S − A) −1 . B} = [e −2t ] = e(t ) = 2iL1 − 2i2  (i)
dt
e −2t  diL1
x(t ) = l At x(0) +  ∴ i2 = − VC2  (ii)
 dt
 0 
4
So sub eq (ii) in (i)
e −2t   x1 
x (t ) =  −  diL1  diL − VC2 
 0   x2  = e(t ) − 2iL1 − 2  1 
dt  4
 
So, x1(t) = e−2t
x2(t) = 0 diL1 1 diL1 VC2
∴ = e(t ) − 2iL1 − −
e −2t  dt 2 dt 2
−2t
y(t ) = [11]  =e
 0  3 diL1 1
⇒ = e(t ) − 2iL1 + Vc2
2 dt 2
e −2t e −2t − e −3t   x1 (0) 
x (t ) = 
(c)   diL1 4 1 2
 0 e −3t   x2 (0)  ⇒ = − iL1 + VC2 + e(t )  (iii)
dt 3 3 3
 x (t ) 
y(t ) = 1 1  1  = x1 (t ) + x2 (t ) In 3rd loop
 x2 ( t ) 
dVC2
y(t) = e−2t x1(0) + (e−2t − e−3t) x2(0) + e−3tx2(0) ⇒ = C2 − iL3
dt
y(t) = Ae−2t
So, x1(0) = A Sub value of i2 in above equation
x2(0) = 0 diL1
dvC2 − VC2
= dt − iL3
dt 4
dVC2 1 4 1 2  1
3. ⇒ = − iL1 + VC2 + l ( p)  − VC2 − iL3

4 3 3 3
dt  4
dVC2 1 1 1
⇒ = − iL1 − VC2 − iL3 + l (t )
For the circuit shown in the figure choose state vari- dt 3 6 6
 (iv)
ables as x1, x2, x3 to be iL1(t), vc2(t), iL3(t). Write the state
equations diL3
⇒ = VC2 − 4iL3 (v)
i  dt
 x1  x  4. Obtain a state space representation in diagonal form for
i   1 the following system [1996]
 x 2  = A  x1  + B[e(t )]  [1997]
i    d3 y dy d2
x   x1  3
+6 2
+11
+ 6y = 6u(t )
 
3 dt dt dt
Solution: The given system is
Solution: From the circuit given
d3 y d2 y dy
3
+6 2
+ 11 + 6 y = 6 u (t )
dt dt dt

⇒ s3 y( s) + 6 s 2 y( s) + 11sy( s) + 6 y( s) = 6u( s)

⇒ y(s) [s3 + 6s2 + 11s + 6] = 6u(s)
y ( s) 6
T.F. = = 3 2
u( s) s + 6 s + 11s + 6

3.90 | Control Systems

6  x1 
⇒  
( s + 1)( s + 2)( s + 3) y( s) = 1 1 1  x2 

 x3 
Taking partial Fraction.
6 A B C 5. From the signal flow graph shown in figure obtain the
⇒ = + +
( s + 1)( s + 2)( s + 3) s + 1 s + 2 s + 3 state space model with x1, x2, x3 and x4 as state variables

and write the transfer function directly from the state
⇒ 6 = A(s + 2) (s + 3) + B(s + 1) (s + 3) + C(s + 1) (s + space model.
2)
⇒ μt s = −1
6 = A(1)(2) ∴ A = 3
let s = −2
6 = B(−1)(1) ∴ B = −6
let s = −3
6 = C(−2)(−1), ∴C=3
∴ T.F. = y( s) = A + B + C Solution: Consider the system given
u( s) s + 1 s + 2 s + 3
x1′ = x2
y ( s) 3 6 3
= − +
u( s) s + 1 s + 2 s + 3 x2′ = x3

x(t) = x1(t) + x2(s) + x3(s) x4′ = x4

y(t) = x1(t) + x2(t) + x3(t) x4 = G ( s) − 2 x 4 − 5 x3 − 7 x2 − 9 x1

3
x1 ( s) = u( s) ∴ y = x1 + 3x2 + 4x3 + 6xy
s +1
∴ State equation representation as given by
take inverse laplace transform
 x1   0 1 0 0   x1   0 
x1 = 3 u(t) – x1′ (t)(i)       
−6  x2  =  0 0 1 0   x2  +  0  [ 0 ]
x2 ( s ) = u( s)  x3   0 0 0 1   x3   0 
s+2       
 x   −9 −7 −5 −2   x4   1 
 4 
Taking inverse laplace transform
x2′ – 2x2 = 6u(t)  (ii)  x1 
 
x
x3 ( s) =
3
4( s ) [ y ] = 1 3 4 6   2 
s+3  x3 
 
 x4 
Taking inverse laplace transform
x3′ = −3x3 + 3u(t)(iii) y ( s) 6 s 2 + 4 s 2 + 3s + 1
T.F. = = 4
from equation (i), (ii) and (iii) x ( s) s + 2 s3 + 5s 2 + 7 s + 9

 x1   −1 0 0   x1   3 
      
 x2  =  0 −2 0   x2  +  −6 
 x   0 0 −3  x3   3 
 3 
Unit iV
Electronic Devices
And Circuits
Chapter 1: Basic 4.3
Chapter 2: PN Junction 4.22
Chapter 3: Bipolar Junction Transistor 4.35
EXAM ANALYSIS
Exam Year 92 93 94 95 96 97 98 99 00 01 02 03 04 05 06 07 08 09 10 11 12 13 14-1 14-2 14-3 14-4 15 16 17 18 19
Set 1 Set 2 Set 3 Set 1 Set 2 Set 3 Set 1 Set 2
1 Marks Ques. - - 4 11 - 1 2 1 - 2 1 5 3 3 4 2 4 2 2 3 1 3 3 3 3 3 2 4 2 2 1 0 3 3 4 1
2 Marks Ques. 3 1 - 2 1 4 1 1 - - - 5 7 3 4 3 4 1 4 3 3 - 3 6 4 4 1 4 1 4 3 1 4 3 3 4
5 Marks Ques. - - - - - - - - - 1 - - - - - - - - - - - - - - - - - - - -
Total Marks 6 2 4 15 2 9 4 3 0 2 1 15 17 9 12 8 12 4 10 9 7 3 9 15 11 11 4 12 4 10 7 2 11 9 10 9

Chapter wise marks


distribution
Basics 2 - 1 4 - 7 - - - - 5 7 4 6 1 4 2 5 - 1 1 3 2 8 - - - -
PN Junction - 2 - 3 - - 4 - - - 1 7 4 3 5 7 3 - 2 2 1 1 2 2 1 - 5 8 9 4 5
BJT 4 - 3 8 2 2 - 3 - - 3 6 2 1 - 5 2 3 6 1 3 4 4 - 3 4 - -
Chapter 1
Basic
One-mark Questions  N .N 
V0 = VT  n  A 2 D 
 ni 
1. A bar of Gallium Arsenide (GaAs) is doped with 1× 1018 × 105
Silicon such that the Silicon atoms occupy Gallium and V0 = 25 × 10-3  n = 0.173 V
1× 10 20
Arsenic sites in the GaAs crystal. Which one of the fol-
lowing statements is true?  [2017] Hence, the correct option is (D).
(A) Silicon atoms act as p-type dopants in Arsenic 3. The output V0 of the diode circuit shown in the figure is
sites and n-type dopants in Gallium sites connected to an aveaging DC voltmeter. The reading on
(B) Silicon atoms act as n-type dopants in Arsenic the DC voltmeter in Volts, neglecting the voltage drop
sites and p-type dopants in Gallium sites across the diode, is _________.  [2017]
(C) Silicon atoms act as p-type dopants in Arsenic as
well as Gallium sites +
(D) Silicon atoms act as n-type dopants in Arsenic as
well as Gallium sites 10 sin ω t 1kΩ Vo
Solution:  Si is a IVth group element, so it acts like f= 50 Hz
p-type dopant in Vth group sites p, As ....etc.) and it is

acts like a n-type dopant like in IIIrd Group sites (B, Al,
Ga,… etc).
4. A small percentage of impurity is added to an intrinsic
Hence, the correct option is (A).
semiconductor at 300 K. Which one of the following
2. An n+-n Silicon device is fabricated with uniform and statements is true for the energy band diagram shown
non-degenerate donor doping concentrations of ND = in the following figure?  [2016]
1
1 × 1018 cm-3 and ND = 1 × 1015 cm-3 corresponding to
2
the n+ and n regions, respectively. At the operational Conduction band
Ec
temperature T, assume complete impurity ionization,
kT /q = 25 mV, and intrinsic carrier concentration to be New energy level 0.01eV
ni = 1 × 1010 cm-3. What is the magnitude of the built-in
potential of this device?  [2017]
(A) 0.748 V (B) 0.460 V Ev
Valence band
(C) 0.288 V (D) 0.173 V
Solution:  Form the given data (A) Intrinsic semiconductor doped with pentavalent
ND = 1 × 1018 atoms/cm3 atoms to form n-type semiconductor.
1
ND = 1 × 1015 atoms/cm3 (B) Intrinsic semiconductor doped with trivalent at-
2
VT = 25 mv oms to form n-type semiconductor.
V0 = ? (C) Intrinsic semiconductor doped with pentavalent
atoms to form p-type semiconductor.
ni2 (D) Intrinsic semiconductor doped with trivalent at-
NA =
N D1 oms to form p-type semiconductor.
Solution:  Here Fermi energy level is lying just below
1× 10 20
= =1× 105 atoms/cm3 the conduction band which shows that it is n-type semi-
1× 1015
4.4 | Electronic Devices And Circuits

conductor doped, i.e., an intrinsic semiconductor is forward voltage


doped with pentavalent impurity to form n-type.
V = 208 mV
Hence, the correct option is (A).
Ih= ?
5. The I–V characteristics of three types of diodes at the
room temperature, made of semiconductors X, Y and Z
are shown in the figure. Assume that the diodes are uni-
I = I 0 eV( nVT
− 1 ,)
formly doped and identical in all respects except their But,
materials. If Egx, Egy and Egz are the band gaps of X, Y
I = Ih
and Z respectively, then [2016]
I Consider n = 1 for small currents
q.DP .Pn
Io ≈ .A
LP

X Y Z ni2 10 20
Pno = = = 105
N D 1 × 1015

LP = DP .τ P = 36 × 10 −4
V 
LP = 6 × 10–2 cm
(A) Egx > Egy > Egz Ih
(B) Egx = Egy = Egz Hole current density =
A
(C) Egx < Egy < Egz
(D) no relationship among these band gaps exists q. pno .DP
Jh = . ⎡⎣eVD VT
− 1⎤⎦
Solution:  The energy band gap of a semiconductor LP

is directly proportional to the cut in voltage. More the −19 5
1.6 × 10 × 10 × 36 208 26
cut in voltage, more will be the energy band gap of the
semiconductor, therefore the relation of Eg of three
Jh =
6 × 10 −2
e −1 { }

semiconductors X, Y, Z is
Jh = 9.6 × 10 × 2980
–12

Egz > Egy > Egx.


Jh = 28.608 nA/cm2
Hence, the correct option is (C).
Hence, the correct Answer is (28 to 30).
6. For a silicon diode with long P and N regions, the
7. A dc voltage of 10 V is applied across an n-type silicon
acceptor and donor impurity concentrations are 1 × 1017
bar having a rectangular cross-section and a length of
cm–3 and 1 × 1015 cm–3, respectively. The lifetimes of
1 cm as shown in figure. The donor doping concentra-
electrons in P region and holes in N region are both 100
tion ND and the mobility of electrons µn are 1016 cm–3
µs. The electron and hole diffusion coefficients are 49
and 1000 cm–2V–1s–1, respectively. The average time
cm2/s and 36 cm2/s, respectively. Assume kT/q = 26 mV,
(in µs) taken by the electrons to move from one end of
the intrinsic carrier concentration is 1 × 1010 cm–3, and
the bar to other end is ______. [2015]
q = 1.6 × 10–19 C. When a forward voltage of 208 mV
is applied across the diode, the hole current density 10 V
(in nA//cm2) injected from P region to N region is
_________. [2015]
Solution:  From the given data n – Si

NA = 1 × 10 cm  and ND = 1 × 10 cm
17 –3 15 –3 1 cm

τP = τn = 100 µs Solution: 


10 V
Dn = 49 cm2/s and DP = 36 cm2/s
VT = 26 mV
n. – Si
ni = 1 × 1010 cm–3
1 cm
q = 1.6 × 10–19 C
Chapter 1  Basic | 4.5

ND = 1016 cm–3 and µn = 1000 cm2/V-sec 0.1


μV 1000 × 10 Jn(0) = 1015 × 1.6 × 10–19 × 36 × × 1
Vd = µE = = = 104 cm/sec 0.026
L 1
∴ J n (0 )(diff ) = 2.2 × 10 −2 A/cm2
We know
distance L and
drift velocity = =
time τ 
Jn(drift) = n(x) . q . µn . E
L 1 cm
∴ τ= = 4 = 10–4 sec D
Vd 10 cm/sec = VT
μ
∴ τ = 100 µ sec 
Dn 36
Hence, the correct Answer is (95 to 105). μn= = = 1384.61 cm 2 /V-S
VT 0.026
8. The energy band diagram and the electron density pro- 
file n(x) in a semiconductor are shown in the figures. Jn(0)(drift) = 1015 × 1.6 × 10–19 × 1384.5 × Ex
(qα x kT )
Assume that n(x) = 1015 e cm–3, with α = 0.1 V/ − KT 1 dn ( x )
Ex = . . = –α
kT q n ( x ) dx
cm and x expressed in cm. Given = 0.026 V, Dn =
q
D kT =
–0.1 V/cm
36 cm2 s–1, and = . The electron current density
μ q ∴ Jn driff = –2.2 × 10–12 A/cm2
(in A/cm2) at x = 0 is [2015]
J = Jn(driff) + Jn(driff)
E(eV) log(n(x))
=
–2.2 × 10–12 + 2.2 × 10–12
Slope = –0.1eV/cm
= 0
Hence, the correct option is (C).
Ec
9. A silicon bar is doped with donor impurities ND = 2.25
× 1015 atoms cm–3. Given that the intrinsic carrier con-
Ev
centration of silicon at T = 300 K is n1 = 1.5 × 1010
x=0 x    x = 0 x cm-3 and assuming complete impurity ionization, the
equilibrium electron and hole concentrations are
(A) –4.4 × 10–2 (B) –2.2 × 10–2 (a) n0 = 1.5 × 1016 cm-3, p0 = 1.5 × 105 cm-3
(C) 0 (D) 2.2 × 10–2 (b) n0 = 1.5 ×1010 cm-3, p0 = 1.5 × 1015 cm-3
Solution:  From the given data electron density profile (c) n0 = 225 × 1015 cm-3, p0 = 1.5 × 1010cm-3
(d) n0 = 2.25 × 1015 cm-3, p0 = 1 × 105 cm-3 [2014]
⎛ q∝x⎞
⎜ ⎟ Solution: (d)
n(x) = 1015. e ⎝ KT ⎠
cm–3 n ≈ ND = 2.25 × 1015 cm3
α = 0.1 V/cm 2
ni
KT p= = 105 cm −3
= VT = 0.026 V = 26 mV n
q Hence, the correct option is (d)
Dn = 36 cm2/sec 1 0. A thin p-type silicon sample is uniformly illuminated
We know with light which generates excess carriers. The recom-
dn bination rate is directly proportional to
Jn = q . Dn . = diffusion current density (a) the minority carrier mobility
dx (b) the minority carrier recombination lifetime
d ( )
qα x (c) the majority carrier concentration
∴ jn (x) = q . Dn . (1015 × e KT ) (d) the excess minority carrier concentration
dx
 [2014]
q ∝ q∝ x KT Solution: (d)
⇒ Jn(x) = 1015 × 1.6 × 10–19 × 36 × .e
KT  Hence, the correct option is (d)
at X = 0
11. At T = 300 K, the hole mobility of a semiconductor µp
kT
= 500 cm2/V-s and = 26 mV . The hole
q
4.6 | Electronic Devices And Circuits

diffusion constant Dp in cm2/s is_____. [2014] 1.24


Solution: 13 cm2/s. λ≤ λ ≤ 1.12µ m
1.1
By Einstein relation, Dp = µPVT = 500 × 26 × 10-3 = 13
cm2/s. ∴λmax = 1.12 µm.
12. At T = 300 K, the band gap and the intrinsic carrier con- 15. In IC technology, dry oxidation (using dry oxygen) as
centration of GaAs are 1.42 eV and 106 cm-3, respec- compared to wet oxidation (using stem or water vapour)
tively. In order to generate electron hole pairs in GaAs, produces
which one of the wavelength (λC) ranges of incident (a) superior quality oxide with a higher growth rate
radiation is most suitable? (Given that: Plank’s constant (b) inferior quality oxide with a higher growth rate
is 6.62 × 10-34 J-s, velocity of light is 3 × 1010 cm/s and (c) inferior quality oxide with a lower growth rate
charge of electron is1.6 × 10-19 C.) (d) superior quality oxide with a lower growth rate
(a) 0.42 µm < λC < 0.87 µm  [2013]
(b) 0.87 µm < λC < 1.42 µm Solution: (d)
(c) 1.42 µm < λC < 1.62 µm The answer is superior quality oxide with a lower
(d) 1.62 µm < λC < 6.62 µm [2014] growth rate.
Solution: (a) Hence, the correct option is (d)
1.24 16. Drift current in semiconductors depends upon
E g (eV ) ≤
λ ( µ m) (a) only the electric field

(b) only the carrier concentration gradient
1.24 1.24 (c) both the electric field and the carrier concentration
λc ≤ = λ ≤ 0.87 µ m
Eg 1.42 (d) both the electric field and the carrier concentration

gradient
Hence, the correct option is (a)  [2011]
13. In the figure, ln (ρi) is plotted as a function of 1/T, Solution: (c)
where ρi is the intrinsic resistivity of silicon, T is the Drift current = J.A. = Nq µ nE
temperature, and the plot is almost linear. ∴ I ∝ E, I ∝ N.
Constant
ln (p1) Hence, the correct ear (E )
Sub-lin option is (c).
c

(E ) f
17. Thin gate oxide in a CMOS process is preferably
(a) grown (b)
using (E ) v
(a) wet oxidation (b) dry oxidation
(c) epitaxial deposition Linear(d) ion implantation
 [2010] (E )c
(c) (E ) (d)
1/T Solution: (b) f

λE
DryMedium
oxidationLarge
provides better isolation from impurities. (E )
The slope of the line can be used to estimate Small E
v
E
E Hence, the correct option is (b).
(a) band gap energy of silicon (Eg)
(b)  sum of electron and hole mobility in silicon 18. In an n-type silicon crystal at room temperature, which
(µn + µp) of the following can have a concentration of 4 × 1019
(c) reciprocal of the sum of electron and hole mobility cm-3?
in silicon (µn + µp)-1 (a) Silicon atoms
N-well Implant source - drain diffusion (b) Holes
(d) intrinsic carrier concentration of silicon (ni)[2014]
Solution: (a) (c) Dopant atoms
(d) Valence electrons [2009]
Hence, the correct option is (a)
Solution: (c)
14. The cut-off wavelength (in µm) of light that
Passivation can be used
metallization
for intrinsic excitation of a semiconductor material of For a highly doped silicon, ratio is 1:10
3

band gap Eg = 1.1 eV is_______. ∴ Dopant atoms = 1/10 × 5 × 10 = 5 × 1019


3 22

[2014] ≈ 4 × 1019 cm–3.


1V
Solution: 1.12 Hence, the correct option is (c).
1 3
1.24 N = 10 /cm 19. The ratio of the mobility to the diffusion coefficient in
λ ≤
( µ m) E g (eV ) a semiconductor has the unit
x=0 x = 1 pm (a) V-1 (b) cm V-1
(c) V cm-1 (d) V s [2009]
Chapter 1  Basic | 4.7

Solution: (a) (c) directly proportional to the intrinsic concentration.


According to the Einstein theorem, (d) inversely proportional to the intrinsic concentra-
n n
tion. [2006]
D = VT ⇒ µ = 1/ VT Solution: (b)
µn Dn 2
ni
µ Minority concentration =
∴ has the unit of V-1. Majority concentration
D
∴ Minority concentration is inversely proportional to
Hence, the correct option is (a).
majority concentration.
20. Which of the following is true?
Hence, the correct option is (b).
(a) A silicon wafer heavily doped with boron is a p+
substrate. 24. Under low-level injection assumption, the injected
(b) A silicon wafer lightly doped with boron is a p+ minority carrier current for an extrinsic semiconductor
substrate. is essentially the
(c) A silicon wafer heavily doped with arsenic is a p+ (a) diffusion current (b) drift current
substrate. (c) recombination current (d) induced current
(d) A silicon wafer lightly doped with arsenic is a p+  [2006]
substrate. [2008 ] Solution: (a)
Solution: (a) The current is by diffusion as there is concentration
gradient due to low-level injection.
Boron is a p-type material and arsenic is an n-type
material. Hence, the correct option is (a).
Hence, the correct option is (a). 25. The band gap of silicon at room temperature is
21. A silicon wafer has 100 nm of oxide on it and is (a) 1.3 eV (b) 0.7 eV
inserted in a furnace at a temperature above 1000° C (c) 1.1 eV (d) 1.4 eV [2005]
for further oxidation in dry oxygen. The oxidation rate Solution: (c)
(a) is independent of current oxide thickness and tem- At 300 k, band gap of silicon is 1.1 eV.
perature. Hence, the correct option is (c).
(b) is independent of current oxide thickness but de-
pends on temperature. 26. The primary reason for the widespread use of silicon in
(c) slows down as the oxide grows. semiconductor device technology is
(d) is zero as the existing oxide prevents further oxida- (a) abundance of silicon on the surface of the Earth.
tion. [2008] (b) larger band gap of silicon in comparison to germa-
nium.
Solution: (d) (c) favourable properties of silicon-dioxide (SiO2).
The oxide layer of 100 nm acts as an insulator for fur- (d) lower melting point. [2005]
ther oxidation. Solution: (a)
Hence, the correct option is (d). Silica is abundant on the Earth. This is one of the major
22. The electron and hole concentrations in an intrinsic reasons for using Si.
semiconductor are ni per cm3 at 300 K. Now, if accep- Hence, the correct option is (a).
tor impurities are introduced with a concentration of NA
per cm3 (where NA >> ni), the electron concentration per 27. The impurity commonly used for realizing the base
cm3 at 300 K will be region of a silicon n-p-n transistor is
(a) ni (b) ni + NA (a) Gallium (b) Indium
ni2 (c) Boron (d) Phosphorus
(c) NA - ni (d) [2004]
NA Solution: (c)
[2007]
Solution: (d) Boron is used in n-p-n transistor.
2
Hence, the correct option is (c).
ni
n= 28. N-type silicon is obtained by doping silicon with
NA (a) Germanium (b) Aluminium
Hence, the correct option is (d). (c) Boron (d) Phosphorus [2003]
Solution: (d)
2 3. The concentration of minority carriers in an extrinsic
semiconductor under equilibrium is Phosphorus has excess electrons and thus n-type semi-
(a) directly proportional to the doping concentration. conductor is formed.
(b) inversely proportional to the doping concentration. Hence, the correct option is (d).
4.8 | Electronic Devices And Circuits

29. The band gap of silicon at 300 K is 34. In a p-type Si simple, the hole concentration is 2.25 ×
(a) 1.36 eV (b) 1.10 eV 1015 cm–3. The intrinsic carrier concentration is 1.5×10–
(c) 0.80 eV (d) 0.67 eV [2003] 10
cm-3, and the electron concentration is
Solution: (b) (a) Zero (b) 1010 cm–3
(c) 105 cm–3 (d) 1.5 × 1025 cm–3
At 300 k, band gap of silicon is 1.1eV.
 [1995]
Hence, the correct option is (b).
Solution: (c)
30. The intrinsic carrier concentration of silicon sample at
ni2=n*p [as per mass action law]
300 K is 1.5 × 1016 rn–3. If after doping, the number
2
of majority carriers is 5 × 1020/m3, the minority carrier ni 2.25 × 10 20
∴n = = = 105 atoms cm -3
density is P 2.25 × 1015
(a) 4.50 × 1011 m–3 (b) 3.33 ×104 m–3
(c) 5.00 × 1020 m–3 (d) 3.00 ×10×-5 m–3 35. A small concentration of minority carries is injected
 [2003] into a homogeneous semiconductor crystal at one
Solution: (a) point. An electric field of 10 V/cm is applied across the
crystal and this moves the minority carriers a distance
ni = 1.5 × 1016 m–3 of 1 cm in 20 µsec. The mobility (in cm2 V-sec) will be
Let majority carrier be represented by N. (a) 1,000 (b) 2,000
N = 5 × 1020 m–3 (c) 5,000 (d) 500,000 [1995]
2 Solution: (c)
ni 2.25 × 1032
∴ Minority carrier density = =
N 5 × 10 20 E = 10 V/cm; d = 1 cm t = 20 µ sec.
= 4.5 × 1011m–3
d 1 106
Hence, the correct option is (a). Vd = = = cm/ sec.
t 20 × 10 −6 20
31. The units of q/kT are
(a) V (b) V-1 Vd 106 105
µ= = = = 5000 cm 2 /V- sec
(c) J (d) J/K [1997] E 20 × 10 20
Solution: (b) 36. A p-type silicon sample has a higher conductivity
q/kT = 1/VT (VT = thermal voltage) compared to an n-type silicon sample having the same
q /kT has the units V - 1. dopant concentration. Is it True or False? [1994]
Hence, the correct option is (b). Solution: False
32. The drift velocity of electrons in silicon: No, it is false because n-type sample will be having
(a) is proportional to the electric field for all values of a large number of elections whose mobility is high as
electric field compared to holes.
(b) is independent of the electric field.
(c) increases at low values of electric field and de- Two-marks question
creases at high values of electric field exhibiting
negative differential resistance. 1. A dc current of 26 µA flows through the circuit shown.
(d) increases linearly with electric field at low values The diode in the circuit is forward biased and it has an
of electric field and gradually saturates at higher ideality factor of one. At the quiescent point, the diode
values of electric field. [1995] has a junction capacitance of 0.5 nF. Its neutral region
resistances can be neglected. Assume that the room
Solution: (d)
temperature thermal equivalent voltage is 26 mV.
Drift velocity increases linearly with electric field at
low values, sub-linearly and at medium values and gets
saturated at high values.
Hence, the correct option is (d). 100 Ω
5 sin (ω t) mV
33. The probability that an electron in a metal occupies the V
Fermi level at any temperature (> K) is:
(a) 0 (b) 1
(c) 0.5 (d) 1.0 [1995]
Solution: (c) For w = 2 × 106 rad/s, the amplitude of the small-signal
The asked probability is 0.5. component of diode current (in µA, correct to one deci-
Hence, the correct option is (c). mal place) is _______. [2018]
2ε s  1 1 
W =  + V j Chapter 1  Basic | 4.9
q  NA ND 
Solution:  1
∴ CT α
Voltage VT = 26 mV W
Current ID = 2.6 µA CT1 W2
=
Resistance CT2 W1
V 26 mA
rd = T = = 1 KΩ 1 1
I D 26 μA CT1 + 16
 = 1016
10
Impedance CT2 1 1
1 +
Xcj = = 1 kΩ 1014 1014
ωcj
1014 1
= 16
=
Xcj 10 10
∴ CT2 = 10.CT1

Hence, the correct answer is (10).


1 kΩ
3. The dependence of drift velocity of electrons on elec-
tric field in a semiconductor is shown below. The semi-
conductor has a uniform electron concentration of n = 1
5 sin (ω t) mV i 100 Ω
× 1016 cm-3 and electronic charge q = 1.6 × 10-19C. If a
bias of 5 V is applied across a 1 mm region of this semi-
conductor, the resulting current density in this region,
in kA/cm2, is________. [2017]
Drift velocity (cm/s)

⎛ 1 ⎞
Z = ⎜ rd  ⎟ + 100 Ω = 600 – j500 Ω constant
⎜ j ωcj ⎟⎠ 107

The amplitude of the small signal component of diode linear
is
5
= 6.4 μA
100 61
Hence, the correct answer is 6.2 to 6.6.
0 5×105 Electric field (V/cm)
2. As shown, two silicon (Si) abrupt p-n junction diodes
are fabricated with uniform donor doping concentra-
tions of N D 1 = 1014 cm −3 and N D 2 = 1016 cm −3 in the Solution: 
n-regions of the diodes, and uniform acceptor d­ oping Drift velocity (cm/s)
concentrations of N A1 = 1014 cm −3 and NA = 1016 cm-3
2
in the p-regions of the diodes, respectively. Assuming
constant
that the reverse bias voltage is >> built-in potentials 107
of the diodes, the ratio C2/C1 of their reverse bias
capacitances for the same applied reverse bias, is linear
____________.  [2017]
p n p n
1014
10 14
1016
1016
cm−3 cm−3 cm−3 cm−3
C1 C2 0 5×105 Electric field (V/cm)
Diode 1 Diode 2

εA Form the given data


We know s C =
Solution: 
W  V = 5 Volts, L = 1 mm
V
2ε s  1 1  E = = 5 × 106 V/m. = 5 × 10 4 V/cm
W =  + V j L
q  NA ND 
1
∴ CT α
W
CT1 W2
=
CT2 W1
4.10 | Electronic Devices And Circuits

We Know ∂p
  J p.diff = − q.D p .
Vd = µE ∂x

J = n.q.Vd  J ∂   x 
p . diff = −q.D p .  pno + GLo .τ P 1 −  
∂x   L 
from the given data
∂  x 
10 7 − 0   J p.diff = − q.D p .GL 0 .τ p 1 −  
µ=m= = 20 ∂x  L  
5 × 105 − 0
q.D p .τ p .GL0
∴ J = 1× 1016 × 1.6 × 10 −19 × 20 × 5 × 1014 ∴ J P .diff =
L
= 0.032 × 5 × 10 = 1.6 × 10 A/cm
4 3 2 1.6 × 10 −19 × 100 × 10 −4 × 1017
J P .diff =
0.1× 10 −4
J = 1.6 KA/cm 2
1.6 × 10 −3
J P .diff = = 16 A/cm2
Hence, the correct answer is (1.5 to 1.7). 10 −4
4. As shown, a uniformly doped silicon (Si) bar for length Hence, the correct answer is (15.9 to 16.1).
L = 0.1 mm with a donor concentration ND = 1016 cm-3, is
5. A MOS capacitor is fabricated on p-type Si (Silicon)
illuminated at x = 0 such that electron and hole pair are
where the metal work function is 4.1 eV and electron
 x
generated at the rate of GL = GL 0 1 −  , 0 ≤ 0 X ≤ L, affinity of Si is 4,4 eV. Ec - EF = 0.9 eV, where Ec and
 L EF are the conduction band minimum and the Fermi
where GL0 = 1017 cm-1s-1. Hole lifetime is 10-4 s, elec- energy levels of Si, respectively. Oxide ∈r = 3.9. ∈o
= 8.85 × 10-14 F/cm, oxide thickness tox = 0.1 mm and
tronic charge q = 1.6 × 10-19 C, hole diffusion coeffi-
electronic charge q = 1.6 × 10-19 C. If the measured flat
cient Dp = 100 cm2/s and low level injection condition
band voltage of this capacitor is −1 V, then the mag-
prevails. Assuming a linearly decaying steady state
nitude of the fixed charge at the oxide-semiconductor
excess hole concentration that goes to 0 at x = L, the
interface, in nC/cm2, is _________. [2017]
magnitude of the diffusion current density at x = L/2, in
A/cm2, is________. [2017] 6. An electron (q1) is moving in free space with velocity
105 m/s towards a stationary electron (q2) far away. The
Light closest distance that this moving electron gets to the
stationary electron before the repulsive force diverts its
Si (ND = 1016 cm−3) path is________ × 10-8 m.
[Given, mass of electron m = 9.11 × 10-31 kg, charge of
electron e = -1.6 × 10-19 C, and permittivity
x=0 L = 0.1 μm
ε 0 = (1/ 36π ) × 10 −9 F/m]  [2017]
Solution: 
From the given data
N D = 1016 cm −3 Solution: 
5
=10 m/s
L = 0.1 µm = 1× 10 −5 cm q1 V → 
→ r ←

 x i moving i stationary
GL = GLO 1 −  , 0 ≤ x ≤ L
 L
GLO = 10 cm −3s −1
17
1  qq 
mv 2 =  1 2  ⇒ Force Per Distance
tp = 10 s -4 2  4πε 0 r 
q = 1.6 × 10-19c and Dp = 100 cm2/sec
2 × −1.6 × 10 −19 × −1.6 × 10 −19
Jp = ?  r =
10 −9
P = P0 + ∆P 4π × × 9.11× 10 −31 × (105 ) 2
36π
∆P
GL = ⇒   r = 5.058 × 10−8 m
  τp
∆P = GL .τ p Hence, the correct answer is (4.55 to 5.55).
7. An abrupt pn junction (located at x = 0) is uniformly
 x doped on both p and n sides. The width of the deple-
  P = Pno + GLo .τ P  1 −
 L  tion region is W and the electric field variation in the
Chapter 1  Basic | 4.11

x-direction is E(x). Which of the following figures rep- the n – side. No external voltage is applied to the diode
resents the electric field profile near the pn junction? Given: kT/q = 26 mV, ni = 1.5 × 1010 cm–3, εsi = 12ε0, ε0
 [2017]
= 8.85 × 10–14F/cm, an and q = 1.6 × 10–19 C. The charge
(A) E(x)
per unit junction area (nC cm–2) in the depletion region
on the p–side is _____. [2016]
n-side p-side Solution:  Number density of acceptor atoms NA =
1017 cm–3
Charge q = 1.6 10–19 C
X
(0,0) Number density of donor atoms ND = 1016 atoms cm–3
W Number density of intrinsic charge carriers ni = 1.5 ×
1010 cm–3
(B) E(x) Voltage VT = 26 mV
εsi = 12εo
n-side p-side
2ε s ⎡ 1 1 ⎤
W = ⎢ + ⎥ ⋅V j
q ⎣ N A ND ⎦

(0,0) X dw
CT = q ⋅ NA ⋅ A ⋅
W dv 

(C)
Qp = –q ⋅ NA . Wp A
E(x)
⎡N ⋅N ⎤
V j = VT  n ⎢ A 2 D ⎥ V .
⎣ ni ⎦ 
n-side p-side
Qp
= − q⋅ N A ⋅ wp
A 
W
X ⎡ 10 × 1016 ⎤
17
V j = 26 × 10 −3 × ln ⎢ 20 ⎥
(0,0)
⎣ 2.25 × 10 ⎦ 

= 26 × 10 −3 × ln ⎡⎣ 4.44 × 1012 ⎤⎦

=
0.757 V

(D) E(x)
2 × 12 × 8.85 × 10 −14 ⎡ 1 1 ⎤
W = ⎢1017 + 1016 ⎥ × 0.757
1.6 × 10 −19 ⎣ ⎦ 
n-side p-side
W = 1.1054 × 10 −9 

W = 3.324 ×10 −5 cm


W ND
X Wp = ⋅W = 3.022 × 10–6 cm.
N A + ND
(0,0)
Q
= −1.6 × 10 −19 × 1017 × 3.022 × 10 −6
A 
= –48.36 nC/cm2.
Hence, the correct Answer is (-48).
8. Consider a silicon p–n junction with a uniform accep-
9. The figure shows the doping distribution in a p type
tor doping concentration of 1017 cm–3 on the p side and
semiconductor in log scale.
a uniform donor doping concentration of 1016 cm–3 on
4.12 | Electronic Devices And Circuits

δρ
NA (cm–3) 1016 Generation rate =
τ ρo

1014
1 2
∆p = Gopt × τpo = 1.5 × 1020 × 0.1 µ
∆p = 1.5 × 1013 cm–3
Position ( μ m)
Pt = p0. exp (–t/τp)
⎛ −0.3 × 10 −6 ⎞
The magnitude of the electric field (in kV/cm) in the =
1.5 × 1013 ⋅ exp ⎜ ⎟
semiconductor due to non uniform doping is _____. ⎝ 0.1 × 10 −6 ⎠

 [2016] =
7.47 × 1011 cm–3
Solution:  Now using the relation
Hence, the correct option is (A).
∂P
q ⋅ DP ⋅ = q ⋅ µP ⋅ PE 1 1. A voltage VG is applied across a MOS capacitor with
∂X
metal gate and p-type silicon substrate at T = 300k.
P = NA The inversion carrier density (in number of carriers per
VT ∂P unit area) for VG = 0.8 V is 2 × 1011 cm–2. For VG = 1.3 V,
E = ⋅ the inversion carrier density is 4 × 1011 cm–2. What is
N A ∂X 
the value of the inversion carrier density for VG = 1.8
d V? [2016]
⇒ ε = VT ⋅ ln {NA(X)}
dx (A) 4.5 × 1011 cm–2 (B) 6.0 × 1011 cm–2
(C) 7.2 × 1011 cm–2 (D) 8.4 × 1011 cm–2
Hence, the correct Answer is (1010 to 1.25).
Solution:  The following relation can be given for com-
1 0. Consider a silicon sample at T = 300 K, with a uniform puting the value of inversion carrier density.
donor density Nd = 5 × 1016 cm–3, illuminated uniformly
such that the optical generation rate is Gopt = 1.5 × 1020 Qinv = K(VGS–VT), VGS > Vt
cm–3s–1 throughout the sample. The incident radiation
is turned off at t = 0. Assume low level injuection to be Qinv = q ⋅ Ni

valid and ignore surface effects. The carrier lifetimes
are τpo = 0.1 µs and τno = 0.5 µs. Ni → Inversion carrier density
from the given data
(i) K (0.8 – VT) ) = q × 2 × 1011
(ii) K(1.3 – VT) = q × 2 × 1011
n-type Si From (i) and (ii)
1 0.8 − VT
=
The hole concentration at t = 0 and the hole concentra- 2 1.3 − VT

tion at t = 0.3 µs, respectively, are [2016] VT = 0.3 V
(A) 1.5 × 1013 cm–3 and 7.47 × 1011 cm–3
(B) 1.5 × 1013 cm–3 and 8.23 × 1011 cm–3 and
(C) 7.5 × 1013 cm–34 and 3.73 × 1011 cm–3 K = 4.q × 1011
(D) 7.5 × 1013 cm–3 and 4.12 × 1011 cm–3
Solution:  Donor density Nd = 5 × 1016 cm–3 1.6 × 10–19 × Ni = 4 × 1011 × 1.6 × 10–19 × 1.5
Optical generation rate Gopt = 1.5 × 1020 cm–3 s–1
Carrier lifetimes τpo = 0.1 µsec Ni = 6 × 1011 cm–2
Carrier lifetimes τno = 0.5 µs Hence, the correct option is (B).
Now using the relation
1 2. Consider the avalanche breakdown in a silicon P +
( )
2
ni2 1.5 × 1010 N junction. The n–region is uniformly doped with a
Na = = donor density ND. Assume that breakdown occurs when
Nd 5 × 1016  the magnitude of the electric field at any point in the
=
4.5 × 10 atoms/cm
3 3 device becomes equal to the critical field Ecrit. Assume
Ecrit to be independent of ND. If the built in voltage of
Chapter 1  Basic | 4.13

the P + N junction is much smaller than the breakdown 2ε ⎛ 1 1 ⎞


voltage VBR, The relationship between VBR and ND is W = ⎜ + ⎟ Vo 
q ⎝ NA ND ⎠
given by, [2016]
and (but it is devoid of electrons therefore)
VBR ×
(A) N D = constant
q.Nd .W
E=
ND × VBR = constant
(B) ∈s

ND × VBR = constant
(C) E ∈S
W =L=
N q.N d
(D) D = constant 
VBR 50 × 10 × 11.7 × 8.85 × 10 −14
3
= 
2ε ⎛ 1 1 ⎞ 1.6 × 10 −19 × 1017
W =
Solution:  ⎜⎝ + ⎟ Vo = 32.35 nm.
q NA ND ⎠
From above relation we conclude that Hence, the correct Answer is (32.35 nm).
1 4. The I–V characteristics of the zener diodes D1 and
W ∝ doping
D2 are shown in figure I. These diodes are used in the
and circuit given in figure II. If the supply voltge is varied
from 0 to 100 V, then the breakdown occurs in [2016]
W ∝ breakdown voltage
I
1 –80 V
Break down voltage ∝ –70 V D1
doping V

VBR × ND = constant 0-100 V D2


D1 D2
Hence, the correct option is (C).
1 3. Consider a region of silicon devoid of electrons and
holes, with an ionized donor density of N a+ = 1017 cm–3. Figure I Figure II
The electric field at x = 0 is 0 V/cm and the electric field
at x = L is 50 kV/cm in the positive ­x-direction. Assume (A) D1 only
that the electric field is zero in the y and z directions. (B) D2 only
(C) both D1 and D2
(D) none of D1 and D2
Nd+ = 1017 cm–3 Solution:  Voltage Vz1 = 80 V
Voltage Vz2 = 70 V
x=0 x=L Vin → 0 to 100 V only
When we will vary the Vin > 80 V
Given q = 1.6 × 10–19 coulomb, ∈0 = 8.85 × 10–14 F/
D1 → break down.
cm, ∈r = 11.7 for silicon, the value of L in nm is ____.
 [2016] When 0 < Vin < 70, none of the Zener diode reaches
breakdown but when 70 < Vin < 80, only D1 and D2 will
Solution:  Number density Nd = 10 cm17 –3
conduct but D2 will not break down. When Vin > 80 the
For silicon ∈r = 11.7 diode D1 reaches breakdown. Hence, option (A) is cor-
Permittivity of free space ∈0 = 8.85 × 10–14 F/cm rect because the max voltage across it will remain 80 V
Now we know relation between field and potential is which will not decrease to cause the breakdown of D2.
V Hence, the correct option is (A).
E=
x 1 5. Consider a long channel NMOS transistor with source
Also E = 0  at  x = 0 and body connected together. Assume that the electron
mobility is independent of VGS and VDS. Given,
E = 5 0KV/cm  at  x = L
gm = 0.5 µA/V
V
= 50 KV/cm
L  for VDS = 50 mV  and  VGS = 2 V,
V gd = 8 µA/V
L=
50 KV  for VGS = 2 V  and  VDS = 0 V,
4.14 | Electronic Devices And Circuits

Where, Δp
∂I D ∂I D ⇒ ∆g = 
gm = and  gd = . τp
∂VGS ∂VDS
  ∴ ∆p = ∆g ⋅ τp = 1020 × 10–6 = 1014 = 10x
The threshold voltage (in volts) of the transistor is
_____. [2016] ∴ x = 14
Solution:  We are given that Hence, the correct Answer is (14).
gm = 0.5 µA/V 1 7. A piece of silicon is doped uniformly with phosphorous
with a doping concentration of 1016/cm3. The expected
for VDS = 50 mV  and  VGS = 2 V value of mobility versus doping concentration for sili-
gd = 8 µA/V con assuming full dopant ionization is shown below.
The charge of an electron is 1.6 × 10–19 C. The con-
for VGS = 2V  and  VDS = 0V) ductivity (in S cm–1) of the silicon sample at 300 K is
______. [2015]
∂I D ∂I D
gm = and gd =
∂VGS ∂VDS Hole and electron mobility in silicon at 300 k

(for NMOS operating in active region) 1400


Electron Hole
⎡ 1 2 ⎤ 1200
I D = K 1 ⎢(VGS − VT ) ⋅VDS − VDS

Mobility (cm2·V–1·s–1)

⎣ 2 ⎦ 1000

∂I D 800
gd =
∂VDS
{
= K 1 (VGS − VT ) ⋅1 − VDS } 600

400
Substituting the given values, we get
200
8 × 10–6 = K1 {(2 – VT) – 0}
0

1 8 × 10 −6 1. E+13 1. E+14 1. E+15 1. E+16 1. E+17 1. E+18 1. E+19 1. E+20


K = (i) Doping concentration (cm–3)
2 − VT

Also we have, Solution:  From the given data
∂I D ND = 1016 cm–3
gm = = K 1 {(1 − 0 ) ⋅VDS − 0} = K 1 ⋅VDS
∂VGS q = 1.6 × 10–19 C

K1 VDS = 0.5 × 10–6 from the graph at ND = 1016 cm–3

µ = 1200 cm2/V-sec
0.5 × 10 −6
1
K = (ii) ∴ σ = ND ⋅ q ⋅ µn
50 × 10 −3
From Eq. (i) and Eq. (ii) we get =
1016 × 1.6 × 10–19 × 1200
8 × 10 −6 0.5 × 10 −6 8 × 50 × 10 −3 σ = 1.92 S/cm
= 2 – V =
2 − Vt 50 × 10 −3
T
0.5 
Hence, the correct Answer is (1.8 to 2.0).
VT = 2 – 0.8 = 1.2 Volts 18. In the circuit shown, assume that diodes D1 and D2 are
ideal. In the steady state condition, the average voltage
Hence, the correct Answer is (1.2 Volts).
Vab (in Volts) across the 0.5 µF capacitor is _________.
1 6. An n-type silicon sample is uniformly illuminated with  [2015]
light which generates 1020 electron-hole pairs per cm3
1 μF
per second. The minority carrier lifetime in the sample
is 1 µs. In the steady state, the hole concentration in the
sample is approximately 10x, where x is an integer. The D1 D2
50 sin(ω t)
value of x is _____.  [2015]
0.5 μF
Solution:  From the given data n-type Si sample b a
– +
e-h pair generation = 1020 pairs/cm–3/s = ∆g Vab
τp = 1 µs Solution:  Given circuit is voltage doubler.
For first half cycle
D1 is on and D2 is off
Chapter 1  Basic | 4.15

1 μF
+
+
– +
Vm vin vout
– 0 0.5 T T
Vm –

+

(A)
0.5 μF 0 0.5 T T
For next half cycle.
1 μF
(B)
– + 0 0.5 T T
Vm

Vm 0
(C) 0.5 T T

+
0.5 μF
(D)
– + 0 0.5 T T
2 Vm
So Vab = 2 × 50
Solution:  Where positive input cycle is applied then
=
100 V circuit can be redrawn as
19. A silicon sample is uniformly doped with donor type + +
impurities with a concentration of 1016/cm3. The elec-
tron and holemobility in the sample are 1200 cm2/V-s Vin Vout
and 400 cm2/V-s respectively. Assume complete ioniza- – –
tion of impurities. The charge of an electron is 1.6 ×
Vout = Vin
10–19 C. The resistivity of the sample (in Ω-cm) is
______. [2015] Vout = –Vin
Solution:  From the given data
When negative input cycle is applied then circuit can
ND = 1016 cm–3 be redrawn as
Vout
µn = 1200 cm2/V-s
µp = 400 cm2/V-s – +
q = 1.6 × 10 –19
C Vi/p Vout = 0V

ρ = ? + –

ρ = 1/ σ  Vout
σ = (n . µn + p µp)q
But n >> p (n-type)
σn ≈ n µn . q
n = ND
0 0.5T
σn ≈ ND . µn . q T

σn = 1016 × 1200 × 1.6 × 10–19


Hence, the correct option is (C).
=
1.92 /cm
1 21. The doping concentrations on the p-side and n-side of a
∴ ρ= = 0.52 Ω-cm
1.92 silicon diode are 1 × 1016 cm3 and 1 × 1017cm-3, respec-
Hence, the correct Answer is (0.50 to 0.54). tively. A forward bias of 0.3 V is applied to the diode.
At T = 300 K, the intrinsic carrier concentration of sili-
2 0. For the current with ideal diodes shown in the figure, con ni = 1.5 × 1010cm-3
the shape of the output (vout) for the given sine wave
input (vin) will be [2015]
4.16 | Electronic Devices And Circuits

and
kT
= 26 mV . The electron concentration at 27 − 9 x = x x = 2.7 µ am
q
WN = 2.7 µ m
the edge of the depletion region on the p-side is ∴
(a) 2.3 × 109 cm-3 (b) 1 × 1016 cm WP = 0.3 µ m

(c) 1 × 10 cm
17
(d) 2.15 × 106cm-3[2014]
q
Solution: (a) Eg max = WN N D
ε
V
n po e 1.6 × 10 −19
kt = × 2.7 × 10 −4 × 1016 = 4.15 × 105 V/cm
1.04 × 10 −12
2
ni 24. Consider a silicon sample doped with ND = 1 × 1015
where n p 0 = in p-type cm–3 donor atoms. Assume that the intrinsic carrier
NA
concentration ni = 1.5 × 1010 cm–3. If the sample is addi-
2.25 × 10 20
∴ n p0 = = 2.25 × 10 4 tionally doped with NA = 1 × 1018 cm–3 acceptor atoms,
16
10 the approximate number of electrons cm–3 in the sam-
0.3 ple, at T = 300 K, will be_____.
−3
2.25 × 10 4 e 26×10 [2014]
n p =
Solution: 225.22
np = 2.306 × 109 cm–3
ND = 1015 cm–3
Hence, the correct option is (a).
NA = 1018 cm–3
2 2. Assume electronic charge q = 1.6 × 10-19C, kT/q = 25
mV and electron mobility µn = 1000 cm2/V-s. If the ∴ Net holes = (1018 - 1015) cm–3
concentration gradient of electrons injected into a = 999 × 105 cm–3
p-type silicon sample is 1 × 1021 cm4, the magnitude of 2
ni 2.25 × 10 20
electron diffusion current density (in A/cm2) is [2014] Now n = = = 225.22 cm-3
p 999 × 1015
Solution: (4000)
dn 25. An N-type semiconductor having uniform doping is
J = qDn biased as shown in the figure
dx
V
Dn
= VT ⇒ Dn = VT µ n
µn
dn N-type semiconductor
J = qµnVT = 1.6 × 10 −19 × 1000 × 25 × 10 −3 × 10 21
dx
J = 4000 A/cm2 If Ec is the lowest energy level of the conduction band,
23. When a silicon diode having a doping concentration of Ev is the highest energy level of the valance band and
NA = 9 × 1016 cm-3 on p-side and ND = 1 × 1016 cm-3 EF is the Fermi level, which one of the following rep-
on n-side is reverse biased, the total depletion width is resents the energy band diagram for the biased N-type
found to be 3 µm.Constant
Given that the permittivity of silicon semiconductor?
ln (p1) is 1.04 × 10-12 F/cm, the depletion widthb-loneathe
r p-side (Ec) (Ec)
Su in
and the maximum electric field in the depletion region, (Ef)
respectively, are (a) (b) (Ef)
(a) 2.7 µm and 2.3 × 105 V/cm (Ev) (Ev)
(b) 0.3 µm and 4.15 × 105 V/cm
(c) 0.3 µm and 0.42 × 105 V/cm Linear
(Ec)
(d) 2.1 µm and 0.42 × 105 V/cm [2014] (c) (Ef) (d)
1/T
Solution: (b) λE (Ec)
Medium Large (Ev) (Ef)
WPNA = WNND Small E E
E
Let WN = x. µm (Ev)
then WP = (3 - x) µm [2014]
16 16 Solution: (c)
(3 − x ) × 9 × 10 = x × 10
n-type SC Ef lies near to conduction band.
N-well Implant source - drain diffusion
Hence, the correct option is (c)

Passivation metallization
N-well Implant source - drain diffusion

Passivation metallization
Chapter 1  Basic | 4.17

26. The magnitude of the electric field at x = 0.5 µm is


1V
(a) 1 kV/cm (b) 5 kV/cm
(c) 10 kV/cm (d) 26 kV/cm [2010] N = 101 /cm3
Solution: (c)
E = V/x x=0 x = 1 pm
At x = 0.5 µm V = 0.5 V 29. The majority carriers in an n-type semiconductor have
0.5 an average drift velocity v in a direction perpendicu-
∴E = = 10 kV/cm
0.5 × 10 −4 lar to a uniform magnetic field B. The electric field E
induced due to the Hall effect acts in the direction
Hence, the correct option is (c)
(a) v × B (b) B×v
27. The magnitude of the electron drift current density at x
(c) along v (d) opposite to v [2006]
= 0.5 µm is
(a) 2.16 × 104 A/cm2 (b) 1.08 × 104 A/cm2 Solution: (b)
(c) 4.32 × 10 A/cm
3 2
(d) 6.48 × 102 A/cm2 E = B × v according to the Hall Effect.
 [2010] Hence, the correct option is (b)
Solution: (a) 30. A heavily doped n-typed semiconductor has the follow-
Election drift current = J.A ing data:
Let A = 1. Hole-electron mobility ratio: 0.4
J = σ E = N D qµn E Doping concentration: 4.2 × 108 atoms/m3
J = 1016 × 1.6 × 10-19 × 1350 × 104 = 2.16 × 104 Intrinsic concentration: 1.5 × 104 atoms/m3
A/cm2 The ratio of conductance of the n-type semiconductor
Hence, the correct option is (a) to that of the intrinsic semiconductor of same material
and at the same temperature is given by
28. Silicon is doped with boron to a concentration of 4 ×
(a) 0.00005 (b) 2,000
1017 atoms cm–3. Assume the intrinsic carrier concen-
(c) 10,000 (d) 20,000 [2006]
tration of silicon to be 1.5 × 1010 cm–3 and the value
of kT/q to be 25 mV at 300 K. Compared to undoped Solution: (d)
silicon, the Fermi level of doped silicon: µp
(a) goes down by 0.13 eV = 0.4 n = 4.2 × 108 atoms/cm 3
µ n
(b) goes up by 0.13 eV
(c) goes down by 0.427 eV ni = 1.5 × 104 atoms cm–3
(d) goes up by 0.427 eV [2008] σn nqµn n µn
Solution: (c) = =
σ i ni q( µn pµ p )  µp 
NA = P = 4 × 1017 atoms cm–3, ni = 1.5 × 1010 cm–3 ni µn 1 + 
 µn 
kT
= 25 mV at 300 K σn 4.2 × 108
q = = 20, 000
σ 4
In n- type, i 1.5 × 10 (1 + 0.4)
KT  NA  −3
 4 × 1017  Hence, the correct option is (d)
Shift = loge  e
 = 25 × 10 log  
q  ni   1.5 × 10
10
 31. A silicon sample A is doped with 1018 atoms/ cm3 of
boron. Another sample B of identical dimensions is
= 0.427 eV.
doped with 1018 atoms cm–3 of phosphorus. The ratio of
Hence, the correct option is (c) electron to hole mobility is 3. The ratio of conductivity
Common Data Questions 9 and 10 of the sample A to B is
The silicon sample with unit cross-sectional areas (a) 3 (b) 1/3
shown below is in thermal equilibrium. The following (b) 2/3 (d) 3/2 [2005]
information is given: T = 300 K, electronic charge = 1.6 Solution: (b)
× 10-19 C, thermal voltage = 26 mV and electron mobil- For n type σ = nqµn sample A → p-type
ity = 1350 cm2/V-s.
For p type σ = pq µp sample B → n-type
4.18 | Electronic Devices And Circuits

Solution: (b)
Now σ n = nµ n Constant
σ p pµ p lnx = ) µm, n = 1014 cm–3, D = 25cm2/sec.
(p10.5
n Sub-lin
ear
14
σB σn dn 10
J = qDn = 1.6 × 10 −19 × 25 ×
= = 3 /1 dx 6.5 × 10 −4
σ σ
A p
= 8 A/cm2
σA Linear
= 1/ 3 Hence, the correct option is (b)
σ B 35. If P is Passivation, Q is
1/Tn-well implant, R is metalliza-
λE
Hence, the correct option is (b) tion and S is source/drain diffusion, then the order inMedium Large
Small
3 2. The reistivity of a uniformly doped n-type silicon sam- which they are carried out in a standard n-wellE
CMOS E E

ple is 0.5 Ω cm. If the electron mobility (µn) is 1250 fabrication process is
cm2/V-sec and the charge of an electron is 1.6 × 10-19 (a) P-Q-R-S (b) Q-S-R-P
Coulomb, the donor impurity concentration (ND) in the (c) R-P-S-Q (d) S-R-Q-P [2003]
sample is Solution: (b)
(a) 2 × 1016 cm–3 (b) 1 × 1016 cm–3 N-well Implant source - drain diffusion
(c) 2.5 × 10 cm
15 –3
(d) 2× 1015 cm–3
 [2004]
Solution: (c)
Passivation metallization
ρ = 0.5 Ω-cm, µn = 1250 cm2/v-sec, q = 1.6 × 10 - 19c
1 1
σ = 0.5Ω − cm, ⇒ = Hence, the correct option is (b)
ρ N D qµn
3 6. The intrinsic carrier 1density
V at 300 K is 1.5 × 1010 cm–3
ρ 0.5 in silicon for n-type silicon doped to 2.25 × 1015 atoms
ND = = = 2.5 × 1015 / cm3 1 3

qµ n 1.6 × 10 −19 × 1250 N = 10


cm–3, the equilibrium /cm
electron and hole densities are
(a) n = 1.5 × 10 p = 1.5 × 1010 cm–3
15,
Hence, the correct option is (c)
(b) n = 1.5x×=10
0 10, p = 2.25 x×=10
1 pm
15
cm–3
3 3. The longest wavelength that can be absorbed by sili- (c) n = 2.25 × 10 , p = 1.0 × 10 cm–3
15 5

con which has the band gap of 1.12 eV is 1.1 µm. If (d) n = 1.5 × 1010, p = 1.5 × 1010 cm–3 [1997]
the longest wavelength that can be absorbed by another
Solution: (c)
material is 0.87 µm, then the band gap of this material
is ni = 1.5 × 1010/ cm3. ND = 2.25 × 1015 atoms/cm3
(a) 1.416 eV (b) 0.886 eV n ≈ ND
(c) 0.854 eV (d) 0.706 eV [2004] ∴ ni = 2.25 × 1015/ cm
Solution: (a) 2
ni 2.25 × 10 20
1.24 P= = = 105 / cm3
∴ Ea = n 2.25 × 1015
λ
Hence, the correct option is (c)
where Ea is in eV and λ in µm.
37. The electron concentration in a sample of uniformly
∴ Ea ∝ 1/ λ doped n-type silicon at 300 K varies linearly from 1017
x = 1.416 eV. cm–3 at x = 0 to 6 × 1016 cm–3 at x = 2 µrn. Assume a situ-
Hence, the correct option is (a) ation that electrons are supplied to keep this concentra-
tion gradient constant with time. If electronic charge
34. The neutral base width of a bipolar transistor, biased
is 1.6 × 10-9, Coulomb and the diffusion constant Dn =
in the active region, is 0.5 µm. The maximum electron
35cm2/s, the current density in the silicon, if no electric
concentration and the diffusion constant in the base are
field is present, is
1014 cm–3 and Dn = 25 cm2/sec, respectively. Assuming
(a) zero (b) 120 A/cm2
negligible recombination in the base, the collector
(c) +1120 A/cm (d)
2
-1120 A/cm2
current density is (the electron charge is 1.6 × 10-19
 [1997]
coulomb)
(a) 800 A/cm2 (b) 8 A/cm2 Solution: (d)
(c) 200 A/cm 2
(d) 2 A/cm2 [2004] n1 = 1017/ cm3 x1 = 0
n2 = 6 × 10 / cm3 x2 = 2µm
16
Chapter 1  Basic | 4.19

q = 1.6 × 10-19c (c) p-type with carrier of 2 × 1016 cm–3


Dn = 35 cm2/ sec (d) n-type with a earner concentration of 2 × 1016 cm–3
 [1991]
dn  6 × 1016 − 1017 
J = qDn = 1.6 × 10 −19 × 35 ×  −4  Solution: (b)

dx  2 × 10  As n < p (or) NA > ND, ∴ semiconductor is p-type with
J = – 1120 A / cm2 carrier concentration of 1016 cm–3.
Hence, the correct option is (d) Hence, the correct option is (b)
3 8. An n-type silicon bar 0.1 cm long and 100 µrn2 in cross- 41. Under high electric fields, in a semiconductor with
sectional area has a majority carrier concentration of 5 increasing electric field,
× 1020/m3 and the carrier mobility is 0.13 m2/V-s at 300 (a) the mobility of charge carriers decreases
K. If the charge of an electron is 1.6 × 10-19 coulomb, (b) the mobility of the carries increases
then the resistance of the bar is (c) the velocity of the charge carriers saturates
(a) 106 Ω (b) 104 Ω (d) the velocity of the charge carriers increases[1990]
c) 10-1 Ω (d) 10-1 Ω [1997] Solution: (a) and (c)
Solution: (a)
Constant
l = 0.1 cm, A = 100 µm2, n = 5 ln× (10
p ) /m µ = 0.13 m / vs
20 3, 2
ear (Ec)
1
Sub-lin
0.1 (Ef)
l= = 10 −3 m A = 100 × 10 −12 m (a)
100
(Ev)
R = ρl / A
Linear
1 1 1 (Ec)
ρ= = =
σ nqµ 5 × 10 20 × 1.6 × 10 −19 × 0.13 1/T
(c) (Ef)
λE
ρ = 5 / 52 Ω-m Small
Medium Large (Ev)
E E
−3 E
5 10
Now R = × ≅ 106 Ω
52 10 −10
From figure at high electric field, mobility of the carrier
Hence, the correct option is (a) decreases and drift velocity saturated.
3 9. A semiconductor is irradiated with light such that car- Hence, the correct option is (a) and (c)
riers are uniformly generated throughout its volume. source - drain diffusion
N-well Implant
The semiconductor is n-type with ND = 1019 cm–3. If the 42. Due to illumination by light, the electron and hole con-
excess electron concentration in the steady state is ∆n = centrations in a heavily doped N-type semiconductor
1015 cm–3 and if τp = 10 µsec (minority carries life time), increase by ∆n and ∆p, respectively. If n1 is the intrinsic
the generation rate due to irradiation Passivation concentration
metallization then,
(a) is 1020 e – h pairs/cm3/s (a) ∆n < ∆p (b) ∆n > ∆p
(b) is 1024 e – h pairs/cm3/s (c) ∆n = ∆p (d) ∆n × ∆p = η2
(c) is 1010 e – h pairs/cm3/s  [1989]
(d) cannot be determined, as the given data is 1 V Solution: (c)
insufficient. [1992]
N = 10 /cm The light results in breaking of covalent bonds. Thus,
1 3

Solution: (a) ∆n = ∆p.


∆p
Generation rate in an-type semiconductorx == 0 x = 1 pm
Hence, the correct option is (c)
Tp
∆p = ∆n = 10 /cm 15 3 4
3. The concentration of ionized acceptors and donors in
a semiconductor are NA, ND, respectively. If NA > ND
105 20 3.
So, generation rate = = 10 e − h pairs/cm and n1 is the intrinsic concentration, the position of the
10 × 10 −6 Fermi level with respect to the intrinsic level depends
Hence, the correct option is (a) on
40. A silicon sample is uniformly doped with 10 phos- 16 (a) NA – ND (b) NA + ND
phorus atoms cm–3 and 2 × 1016 boron atoms cm–3. If all N - ND
(c) A (d)
n1
the dopants are fully ionized, the material is ni2
(a) n-type with carrier concentration of 1016 cm–3 [1989]
(b) p-type with carrier concentration of 1016 cm–3
4.20 | Electronic Devices And Circuits

Solution: (c) For intrinsic semiconductor


For an n-type semiconductor n = p = ni
 NAND  n ∝T3/2
E f = Ec − kT loge 
 n2  Hence, the correct option is (c)
 i 
46. According to the Einstein relation, for any semiconduc-
NAND tor the ratio of diffusion constant to mobility of carriers
Ec − E f = kT loge
ni2 (a) Depends upon the temperature of the semiconduc-

tor
NAND (b) Depends upon the type of the semiconductor
∴ Ec − E f α loge
ni2 (c) Varies with life time of the semiconductor
(d) Is a universal constant [1987]
Hence, the correct option is (c)
Solution: (a)
4 4. Consider two energy levels: E1 E2 eV above the Fermi
level and E2, E eV below the Fermi level. P1 and P2 are, D Dn D p T ( o K)
respectively, the probabilities of E1 being occupied by Einsteis equation → = = = VT =
µ µn µ p 11600
an electron and E2 being empty. Then
(a) P1 > P2 where
(b) P1 = P2
(c) P1 < P2 T = temperature in °K
(d) P1 and P2 depend on the number of free VT = thermal voltage
electrons [1987] D = diffusion constant
Solution: (c) µ = mobility
Given that Dn = electron diffusion constant
P1 = E1 Fermi level probability Dp = hole diffusion constant
P2 = E2 Fermi level probability µn = electron mobility
1 µp = hole mobility
f(E) =
(E-E f )KT VT = T /11600
1+ e
Dn
where ∴ = T /11600
µn
f(E) = Fermi-Dirac probability function
EF = Fermi level ∴ It depends upon the temperature of the semiconductor.
P1 < P2 Hence, the correct option is (a)
Hence, the correct option is (c) 47. Direct band gap (DBG) semiconductors
(a) Exhibit short carrier life time and they are used for
4 5. In an intrinsic semiconductor, the free electron concen- fabricating BJTs
tration depends on (b) Exhibit long carrier life time and they are used for
(a) Effective mass of electrons only fabricating BJTs
(b) Effective mass of holes only (c) Exhibit short carrier life time and they are used for
(c) Temperature of the semiconductor fabricating lasers
(d) Width of the forbidden energy band of the semi- (d) Exhibit long carrier life time and they are used for
conductor [1987] fabricating BJTs [1987]
Solution: (c) Solution: (c)
By massN A -action
N D law DBG semiconductors exhibit short carrier life time.
n × p = ni2 They are used for fabricating lasers.

ni = intrinsic carrier concentration In DBG semiconductor during the recombination, the
p = hole concentration energy is released in the form of light.
Hence, the correct option is (c)
N An- =N electron
D concentration
ni ∝ T
2 3

ni ∝ T3/2.
Chapter 1  Basic | 4.21

du
Five-marks Questions Substitute
dp
in equation (1)

1. An n-type silicon sample, having electron mobility µn


 n2 
twice the hole mobility µp, is subjected to a steady illu- µ p + µn  − i 2 =0
mination such that the electron concentration doubles  p 
 
from its thermal equilibrium value. As a result, the con-
ductivity of the sample increases by a factor of [1991] uu
p = ni
Solution: Given sample in n-types silicon SC. µp
 ……(3)
∴ σn = nq µn from equation (2) and (3)
n'=2n up
σ'n = n'qµn = 2nqµn n = ni A
µn  ……(4)
n
σ ′ = 2σ
σq [nµn+pµp]
∴New conductivity is twice the initial value.
 up u 
2. Show that the minimum conductivity of an extrinsic σ min = q  ni µn + ni u µ p 
µn µp 
silicon sample occurs when it is slightly p-type calcu-  
late the electron and hole concentration when where
the conductivity is minimum given that µn = 1350 cm σ min = 2ni q µn µ p

W-sec, µp = 450 cm-sec, and the intrinsic carrier con-
centration, ni = 1.5 × 1010 cm-3. [1991] from equation (2) and given data
Solution: Given up 450
n = ni = 1.5 × 1010
μn = 1350 cm2/volt – sec µn 1350

μp = 450 cm2 /volt – sec P = 2.59 × 1010/cm3
ni = 1.5 × 1010 / cm3 from equation (4)
σ = nq μn + pq μp = conductivity
up 450
σ = q(n μn + p μp) n = ni = 1.5 × 1010
µn 1350
for σ to be minimum
dσ n = 0.866 × 1010/cm3
=0
dp we know that for a semiconductor mobility of e− is
always greater than mobility of holes μn > μp
 du 
q  µ p + µn =0 µp un

dp  h = ni P = ni
µn µp
du
µ p + µn =0 ∴P>n
dp  ……(1)
∴ It proves that minimum conductivity of extrinsic sili-
By mass action law
con occurs when it is slightly p type.
np = ni2  ……(2)
2
dx ni
=− 2
dp p
Chapter 2
PN Junction
Solution: Fermi level for n & p type semiconductor
One-mark Questions are:
1. Which one of the following options describes correctly N 
E F − N = EC − KT ln  C 
the equilibrium band diagram at T = 300 k of a Silicon  N 
pnn+p++ configuration shown in the figure?  [2019]
N 
p n n+ P++ E F − P = EV − KT ln  V 
 P 

(A) EC
Flour the above equations we notice that when
in n-side, Fermi level is closer to EC
EF in P-side, Fermi level is closer to EV
in P++, Fermi level penetrates into valence bond
EV Hence, the correct option is (D).
2. In a p-n junction diode at equilibrium, which one of the
following statements is NOT TRUE? [2018]
(B) (A) The hole and electron diffusion current components are
EC in the same direction.
(B) The hole and electron drift current components are in
the same direction.
(C) On an average, holes and electrons drift in opposite di-
EF
rection.
(D) On an average, electrons drift and diffuse in the same
EV
direction.
Solution:  From the given statements
(C) (i) Idrift ⎫
Idiffusion ⎪ are in the same direction
EC ⎬
EF ⎪
(Ip, In) ⎭
(ii) 
electrons and holes movement in opposite
direction.
Hole direction
EV
e– (opposite direction)

(D) EC Hole direction

e– direction

EV
Hence, the correct option is (D)
EF
3. The electric field profile in the depletion region of a p-n
junction in equilibrium is shown in the figure. Which
one of the following statements is NOT TRUE?[2015]
Chapter 2  PN Junction | 4.23

E 2p sin ωtV
=
(V/cm)
104 For negative half cycle.

10 kΩ
–0.1 0 0.5 1.0 x( μ m) + –
6π sin ω t Vab
(A) The left side of the junction is n-type and the right side π < ω t < 2π 10 kΩ 20 kΩ
is p-type
(B) Both n-type and p-type depletion regions are uniformly
doped
6π sin ω t × 10
(C) The potential difference across the depletion r e - Vab = = 3π sin ω t V
gion is 700 mV 20
(D) If the p-type region has a doping concentration of 1015 T
1
f (t ) dt
T ∫0
cm-3, then the doping concentration in the n-type region Vavg =
will be 1016 cm–3 
Solution:  ω ⎡ π 2π

= ⎢∫ 2π sin ω t dt + ∫ 3π sin ω t dt ⎥
E(V/cm) 2π ⎢⎣ 0 π ⎥⎦

104
ω ⎡ ( − cos ω t )π 3π ⎤
+ ( − cos ω t )π ⎥

= ⎢ 2π 0
2π ⎢ ω ω ⎥
⎣ ⎦
=
2 + 3

–0.1
0
0.5 1.0 x( μ m) = 5 V
Hence, the correct Answer is (4.85 to 5.15).
From the given options 5. The built-in potential of an abrupt p-n junction is
0.75 V. If its junction capacitance (CJ ) at a reverse bias
1 (VR) of 1.25 V is 5 pF, the value CJ (in pF) when VR =
Built in potential VJ = × 104 × 102 × 1.1 × 10–6 = area
2 7.25 V is _______. [2015]
= 0.55 volts = 550 mV
Solution:  From the given data
So option c is NOT correct
Built-in voltage of an abrupt p-n junction is
Hence, the correct option is (C).
4. In the circuit shown, assume that the diodes D1 and D2 Vj = 0.75 V
are ideal. The average value of voltage Vab (in Volts), If VR = 1.25 V, CJ = 5 pF
across terminals ‘a’ and ‘b’ is _________.
 [2015] then VR = 7.25 V, CJ = ?

we know
εA 1
D1 D2 C = ;C α
10 kΩ W W 
6π sin(ω t) a + – b

10 kΩ
vab
20 kΩ
W∝ (VR +V j )
C J1 V j + VR2
=
Solution:  For positive half cycle circuit can be drawn CJ2 V j + VR1
as 
2
C J 2 = 5 pF ×
8
Vab
+ – C J 2 = 2.5 pF
6π sin ω t
10 kΩ
0 < ωt < r 10 kΩ 20 kΩ Hence, the correct Answer is (2.4 to 2.6).
6. When the optical power incident on a photodiode is 10
6π sin ω t × 10 μW and the responsivity is 0.8 A/W, the photocurrent
⇒ Vab =  generated (in μA) is______. [2014]
30
4.24 | Electronic Devices and Circuits

Solution: 8 μA 9. The i-v characteristics of the diode in the circuit given


Photo current = incident power × responsivity below are
= 10 × 10-6 × 0.8 = 8 μA.  v − 0.7
 A, v ≥ 0.7 V
7. In the figure, assume that the forward voltage drops of i =  500
 v < 0.7 V
the PN diode D1 and Schottky diode D2 are 0.7 V and + 0 A 1 kΩ
0.3 V, respectively. If ON denotes conducting state of
1kΩ
the diode and OFF denotes non-conducting state of the v0
I
diode, then in the circuit,
+
+
1 kΩ 20 kΩ – _ 10 V
_v 10 V
1 kΩ 20 kΩ

The current in the circuit is


(a) 10 mA (b) 9.3 mA
10 V D1 D2 (c) 6.67 mA (d) 6.2 mA [2012]
10 V D1 D2
Solution (d)
From the circuit
(a) both D1 and D2 are ON - 10 + 1000i + v = 0
(b) D1 is ON and D2 is OFF i = 10 - v / 1000
(c) both D1 and D2 are OFF
v - 0.7/500 = 10 - v/1000
(d) D1 is OFF and D2 is ON [2014]
Solution (c) + 2 v -1.4 = 10 - v
3v = 11.4
Let D1 is off and D2 is ON. Then I
D1
1 kΩ 20 kΩ v = 11.4/3 10 V
1 IkΩ V1 20 kΩ
I 11.4
V1 10 −
i= 3 = 6.2 mA
1000
D2 2
10 V 0.3 V Hence, the correct option is (d)
10 V 0.3 V 10. A silicon p – n junction is forward biased with a con- 10k I
V
+ stant current at room temperature.
20 V When 20 V
the temperature is increased by 10°C, the forward bias
2 voltage across the p – n junction
2 10 − 0.3
I k=
10 I = 0.461 mA (a) increases by 60 mV
10 k I 21k (b) decreases by 60 mV
20 V
20 V ∴ V1 = 9.539 V (c) increases by 25 mV
Both D1 and D2 are ON. (d) decreases by 25 mV [2011]
Hence, the correct option is (c) Solution (b)
8. In a forward biased p – n junction, the sequence of For 1°C rise in temperature, forward voltage decreased V1
events, that best describes the mechanism of current by 2.5 mV
flow is So, forward voltage decreases by 25 mV.
(a) injection, and subsequent diffusion and recombina- Hence, the correct option is (b)
R 1
tion of minority carriers 1

V1 R 11. A Zener diode, when used in voltage stabilization cir-


and subsequentV driftV0and generation of
1
(b) injection,
V1 1 kΩ 0 cuits, is biased in
minority carriers
1 kΩ 1 kΩ +
R 2 (a) reverse bias region below the breakdown voltage
(c) extraction, and subsequent
R 1kΩ diffusion and genera- 2
2
(b) reverse breakdown region
tion of minority carriers
(c) forward bias region
(d) extraction, and subsequent +drift and recombination 1 kΩ
– 2V (d) forward bias
5V constant current mode
V [2011]
of minority carriers +– 2V
A
[2013]
+ 5V Solution (c)
Solution (a)
The Zener diode is reversed biased when used as a volt-
First injection, then subsequent diffusion, and recombi- –
age regulator. 1 kΩ
nation of minority carriers take place.
VA Hence, the correct option is (c)
Hence, the correct option is (a)
Vi


1 kΩ
1 kΩ

Vi
Vi P+ -region N -region
1 kΩ X 16 3
Chapter 2  PN Junction | 4.25

12. Which of the following is Not associated with a p-n Solution (b)
junction ? εA
(a) Junction capacitance C=
d
(b) Charge storage capacitance
(c) Depletion capacitance C ε 11.7 × 8.85 × 10 −12
(d) Channel length modulation [2008] = = ≈ 10 µ
A d 10 × 10 −6
Solution (d)
This is associated with FET. Hence, the correct option is (b)
Hence, the correct option is (d) 1 7. Choose proper substitutes for X and Y to make the
following statement correct. Tunnel diode and ava-
13. In a p-n junction diode under reverse bias, the magni- lanche photodiode are operated in X bias and Y bias,
tude of electric field is maximum at respectively.
(a) the edge of the depletion region on the p-side (a) X: reverse, Y: reverse
(b) the edge of the depletion region on the n-side (b) X: reverse, Y. forward
(c) the p-n junction (c) X: forward, Y: reverse
(d) the centre of the depletion region on the n-side (d) X. forward, Y forward [2003]
 [2007]
Solution (c)
Solution (c)
Tunnel diode → forward biased
Electric field is always max at the junction.
Avalanche photodiode → Reverse biased.
Hence, the correct option is (c)
Hence, the correct option is (c)
14. The values of voltage (VD) across a tunnel-diode cor-
18. In the figure, silicon diode is carrying a constant cur-
responding to peak and valley currents are Vp and Vv,
rent of 1 mA. When the temperature of the diode is
respectively. The range of tunnel-diode voltage VD for
20°C, VD is found to be 700 mV. If the temperature rises
which the slope of its I-VD characteristics is negative
to 40°C, VD becomes approximately equal to
would be
(a) 740 mV (b) 660 mV
(a) VD < 0 (b) 0 ≤ VD < Vp
(c) 680 mV (d) 700 mV [2002]
(c) Vp ≤ VD < Vv (d) VD ≥ Vv [2006]
Solution (c) +
The slope is negative when Vp ≤ VD < Vv 1kΩ
v0
Hence, the correct option is (c) I
+
15. A silicon p – n junction at a temperature of 20°C has +
– _ 10 V
a reverse saturation current of 10 pico-amperes (pA). _v
The reverse saturation current at 40°C for the same bias Solution (b)
is approximately For 1°C rise in temp, VD decreases by 2 mV.
(a) 30 pA (b) 40 pA
∴ for 20°C rise in temp, VD decreases by 40 mV.
(c) 50 pA (d) 60 pA [2005]
∴ VD = 660 mV.
Solution (b)
T2 −T1 Hence, the correct option is (b)
I (T 2) = I (T 1) 2 10 19. For small signal ac operation, a practical forward biased
diode can be modelled as
20 (a) a resistance and a capacitance
I ( 40°) = 10 × 2 10 = 40 PA
(b) an ideal diode and resistance in parallel
Hence, the correct option is (b) (c) a resistance and+an ideal diode in series
(d) a resistance [1998]
16. A silicon p – n junction diode under reverse bias has Solution (d) I
D
depletion region of width 10 μm. The relative permit- 1

tivity of silicon, εr = 11.7 and the permittivity of free For small signal ac operation, a practical forward biased
space ε0 = 8.85 × 10-12 F/m. The depletion capacitance diode can be modelled as a resistance.
of the diode per square metre is Hence, the correct option is (d)
(a) 100 μ (b) 10 μ 20. The static characteristic Dof2 an adequately forward
(c) 1 μ (d) 20μ [2005] biased p--n junction is a straight line, if the plot is of
V
+ 20 V
4.26 | Electronic Devices and Circuits

(a) log I vs. log V (b) log I vs. V Solution (b)


(c) I vs. log V (d) I vs. V If τ ⋅ I 0 (eVD /ηVT − 1)
[1998] CD = τ ⋅ g = τ ⋅ =
ηVT ηVT
Solution (b)
Also C ∝ 1/W
I = I 0 eVD /ηVT
&W ∝ VRB + V0 → reverse bias
VD = hVT ln (I/I0)
∴ As reverse bias increases, W ↑ and hence C ↓.
∴ Plot of log I vs V gives a straight line.
Hence, the correct option is (b)
Hence, the correct option is (b)
21. The diffusion potential across a p-n junction
(a) decreases with increasing doping concentration Two-mark Questions
(b) increases with decreasing band gap 1. In an ideal pn junction with an ideality factor of 1 at
(c) does not depend on doping concentrations T = 300 K, the magnitude of the reverse-bias voltage
(d)  increases with increase in doping concentration required to reach 75% of its reverse saturation current,
 [1995] rounded off to 2 decimal places, is _____ mV.
Solution (d) [K = 1.38 × 10-23 jK-1, h = 6.625 × 10-34 J, s, q = 1.602
The diffusion potential across a p-n junction increases × 10-19 C] [2019]
with increase in doping concentration. Solution: For p - n junction diode
Hence, the correct option is (d)
 VBE 
22. A Zener diode works on the principle of
I = I S  e VT − 1
(a) tunnelling of charge carriers across the junction  
(b) thermionic emission
(c) diffusion of charge carriers across the junction I = −0.75 I S
(d) hopping of charge carriers across the junction
 [1995]  VBE 
Solution (a) −0.75 I S = I S  e VT − 1
 
A Zener diode works on the principle of tunnelling of
charge carriers across the junction. VBE
Hence, the correct option is (a) VT
1 − 0.75 = e
23. The depletion capacitance, Ci, of an abruptly p – n V
junction with constant doping on either side varies with ln 0.25 = BE
VT
R.B.VR as
(a) CJ ∝ VR (b) CJ ∝ VR-1 VBE = VT ln 0.25

(c) CJ ∝ VR -1/2
(d) CJ ∝ VR-1/3
−1.38 × 10 −23 × 300

[1995] VBE = −1.386 ×
Solution (c) 1.6 × 10 −19
1 V = 35.87 mV
Cj ∝ BE
w
Hence, the correct answer is 35.87.
w ∝ √v 2. A Germanium sample of dimensions 1 cm × 1 am is
Cj ∝ 1/ √v illuminated with 20 mW, 600 nm laser light source as
Hence, the correct option is (c) shown in the figure. The illuminated sample surface
24. In a junction diode has a the 100 nm of loss-less. Silicon dioxide layer
(a) the depletion capacitance increases with increase that reflects one-fourth of the incident light. From the
in the reverse bias remaining light, one-third of the power is reflected
(b) the depletion capacitance decreases with increase from the Silicon dioxide-Germanium interface, one-
in the reverse bias third is absorbed in the Germanium layer. and one-third
(c) the depletion capacitance increases with increase is transmitted through the other side of the sample. If
in the forward bias the absorption coefficient of Germanium at 600 nm is 3
(d) the depletion capacitance is much higher than the × 104 cm–1 and the bandha is 0.666 eV, the thickness of
depletion capacitance when it is forward biased the Germanium layer., rounded off to 3 decimal places,
 [1990] is _____ mm. [2019]
Chapter 2  PN Junction | 4.27

20 mW, 600 nm 20 − 0
IL = = 20 mA
1 cm 1
1 cm I R = I z + I L = 60 + 20 = 80 mA

Silicon dioxide 100 mm Vimin = V2


( RL + R1 )
RL
Germanium T
12
= × 20
Solution: 10
Pi = 20 mV = 24 volt
Pi I R = 80 mA
Pr1 =
4
(Si − 02 )
Vimax = 200 × 80 + 20
1 P = 16 + 20
Pt =  Pi − i 
3 4 = 36 volt

Pi
= Hence, the correct option is (B)
4
4. The quantum efficiency (h) and respectively (R) at
1 P
a wavelength l (in mm) in a p-i-n photodetector are
Pa =  Pi − i 
3 4 related by [2019]
P1 1.24 × λ η×λ
= (A) R = (B) R =
4 η 1.24
2 Pi λ 1.24
P4 E = Pt + Pa = (C) R = (D) R =
4 η × 1.24 η×λ
Pt = Po e − at
Pi 2 Pi − at Ip ng nl
= e Solution: R = = =
4 4 Po nf 1.24

Hence, the correct option is (B).
1 − 2e − at = 0
5. A p-n step junction diode with a contact potential of
e − at = 0.5 0.65 V has a depletion width of 1µm at equilibrium.
− ln ( 0.5) The forward voltage (in volts, correct to two deci-
a= cm
mal places) at which this width reduces to 0.6 µm is
3 × 10 4
______. [2018]
Hence, the correct answer is 0.231 mm.
Solution: 
3. In the circuit shown, the breakdown voltage and the
contact potential V01 = 0.65V :
maximum current of the Zener diode are 20 V and 60
mA, respectively. The values of R1 and RL are 200 W depletion width W1 = 1 mm
and 1 kW, respectively. What is the range of V1 that will forward voltage V02 = ?
maintain the Zener diode in the ‘on’ state?” [2019] reduced width W2 = 0.6 mm
R1 We know that
2ε s ⎡ 1 1 ⎤
Vi + RL
W= ⎢ + ⎥ Vj
– q N
⎣ A N D⎦

W∝ Vj
(A) 20 V to 28 V (B) 24 V to 36 V
(C) 18 V to 34 V (D) 22 V to 34 V
W1 Vo1
Solution: =
W2 Vo 2
R1 = 200 Ω
1 0.65
60 mA =
0.6 Vo 2
Vi +
– 20 V RL = 1 kΩ
4.28 | Electronic Devices and Circuits

0.65 8. A junction is made between P-Si with doping density


Vo2 = = 0.234 V
2.77 NA1 = 1015 cm–3 and P Si with doping density NA2 = 1017
But, V02 = V01 = VFB cm–3
Given: Boltzmann constant K = 1.38 × 10–23J. K–1, elec-
0.234 = 0.65 – VFB
tronic charge q = 1.6 × 10–19 C.
VFB = 0.416 V At room temperature (T = 300K), the magnitude of
Hence, the correct answer is 0.4 to 0.43. the built-in potential (in volts, correct to two decimal
6. Red (R), Green (G) and Blue (B) Light Emitting Diodes places) across this junction will be ________ . [2018]
(LEDs) were fabricated using p – n junctions of three Solution: 
different inorganic semiconductors having different Boltzmann constant K = 1.38 × 10–23J. K-1
band-gaps. The builtin voltages of red, green and blue
electronic charge q = 1.6 × 10–19 C
diodes are VR, VG and VB, respectively. Assume donor
and acceptor doping to be the same (NA and ND, respec- Room temperature T = 300K
tively) in the P and n sides of all the three diodes. Now using the relation
Which one of the following relationships about the ⎧P ⎫
VO = VT ln ⎨ 1 ⎬
built-in voltages is TRUE? [2018] ⎩ P2 ⎭ 
(A) VR > VG > VB (B) VR < VG < VB
(C) VR = VG = VB (D) VR > VG < VB ⎧1017 ⎫
VO = 25.6 × 10–3 × ln ⎨ 15 ⎬
Solution:  For Red VR = 1.8V ⎩10 ⎭ 
=
0.119 V
For Green VG = 2.2V
For Blue VB = 5V Hence, the correct answer is 0.11 to 0.13.
From the above data 9. For a particular intensity of incident light on a silicon
VR < VG < VB. pn junction solar cell, the photocurrent density (JL) is
2.5 mA/cm2 and the open-circuit voltage (Voc) is 0.451
Hence, the correct option is (B) V. Consider thermal voltage (VT) to be 25 mV. If the
7. A solar cell of area 1.0 cm2, operating at 1.0 sun inten- intensity of the incident light is increased by 20 times,
sity, has a short circuit current of 20 mA, and an open assuming that the temperature remains unchanged, Voc
circuit voltage of 0.65 V. Assuming room temperature (in volts) will be ___________. [2017]
operation and thermal equivalent voltage of 26 mV, the 10. A region of negative differential resistance is observed
open circuit voltage (in volts, correct to two decimal in the current voltage characteristics of a silicon PN
places) at 0.2 sun intensity is _______. [2018] junction, if [2015]
(A) both the P-region and the N-region are heavily doped
Solution: 
(B) the N-region is heavily doped compared to the P-region
Solar cell area = 1cm2 (C) the P-region is heavily doped compared to the N-region

short circuit current Isc1 = 20 mA (D) an intrinsic silicon region is inserted between the P-
region and the N-region
open circuit voltage Voc1 = 0.65V
Solution:  The tunnel diode is having negative resist-
thermal equivalent voltage VT = 26mV
ance characteristics.
Let open circuit voltage be Voc2 at 0.2 sun intensity It is a heavily doped diode.
⎧I ⎫ ∴ both sides of P and N-region are heavily doped.
Open circuit voltage Voc = VT ln ⎨ sc ⎬
⎩ Io ⎭ Hence, the correct option is (A).
Io → Intensity
1 1. In the circuit shown below, the Zener diode is ideal and
⎧I ⎫ the Zener voltage is 6 V. The output voltage Vo (in Volts)
Voc2 – Voc1 = VT ln ⎨ SC2 ⎬
⎩ I SC1 ⎭  is _______ [2015]
0.2 1 kΩ
Voc2 = Voc1 – 26 × 10–3 ln
1 
=
0.65 – 0.0418 +
10 V 1 kΩ V0
=
0.608 V –

Hence, the correct answer is 0.59 to 0.63.


I
1 D1
V
+ 102V 0.3 V

+
2
D2 2
1 kΩ VA I Junction | 4.29
10k2  PN
V 5V Chapter
5V + 20 V
20 V
Solution: 
From the given data Solution (c)

VZ = 6 V 1 kΩ

If zener diode is in ON position


Vi
Vo = VZ = 6 V.
1 kΩ
Check whether the zener diode is ON or OFF V0
R1
If diode is ON, 0.7 V
V1 V0
1k × Vin 1 kΩ
+
P -region V 1NkΩ =
-region
L
= 6 V
20 kΩ
NA >> ND X N = 10 /cm 2k
n D
16 3
2V R2 1 kΩ

Vin = 12 V
1
+ 2V
The minimum required input voltage is 12 V, but given i = 0 when diode does not conduct. So, V0 = - 5 V. –
10voltage
V D1 is in OFF mode. D2
is 10 V. So,zener diode +
Now, when diode conduct, then imax
Vin × 1k Vin 2
∴ Vo = = = 5 V.  5 − 2.7 2.3
2k 2 = = = 1.15 mA
1 kΩ V2 2
Hence, the correct Answer is (5). 5V A

5V ∴ V0 = 1.15 + 2.7 = 3.85 V.


12. Consider an abrupt p – n junction (at T = 300 K) shown Hence, the correct option is (c).
in the figure. The depletion region width xn on the 14. The –donor and acceptor impurities in 1kan
Ω abrupt junc-
N-side of the junction is 0.2 μrn and the permittivity of tion silicon diode are 1 × 1016 cm-3 and 5 × 1018 cm-3,
silicon (∈sj) is 1.044 × 10-12 F/c m. At the junction, the respectively. Assume that the intrinsic carrier con-
approximate value of the peak electric20field Vi
1 kΩ kΩ (in kV/cm)
I centration in silicon n1 = 1.5 × 1010 cm-3 at 300K,
is________. V [2014] 1 kΩ
1
kT = 26 mV and the permittivity of silicon ∈ = 1.04
q si V0
× 10 - 12 F/cm. The built-in potential and the depletion
0.7 V
width of the diode under thermal equilibrium condi-
10 V
P+ -region N -region 0.3 V tions, respectively, are
Xn ND = 1016/cm3 2V
NA >> ND (a) 0.7 V and 1 × 10-4 cm
(b) 0.86 V and 1 × 10-4 cm
(c) 0.7 V and 3.3 × 10-5 cm
2 Solution: 30.651 (d) 0.86 V and 3.3 × 10-5 cm [2014]
10k Iq Solution (d)
ε max = WN N D
20 V ε N AN D
Vbi = VT ln 2
ni
1.6 × 10 −19 −4 16
= × 0.2 × 10 × 10 = 30.651 kv/cm
1.044 × 10 −2 1016 × 5 × 108
26 × 10 −3 ln = 0.86 V
13. The diode in the circuit shown has Von = 0.7 V but is 2.25 × 10 20
ideal otherwise. If Vi = 5 sin(wt) V, the minimum and
maximum values of V0 (in volts) are, respectively, 2ε  1 1 
W =  +  (V )
q  N A ND 
R1
V1 V0
1 kΩ
2 × 1.04 × 10 −12  1 1 
= −19  1016 +  0.86
R2 1 kΩ 1.6 × 10  5 × 1018 
= 3.3 × 10-5 cm.
+ 2V Hence, the correct option is (d).

+ 15. Compared to a p – n junction with NA = ND = 1014 cm–3,
which one of the following statements is TRUE for a
p – n junction with NA = ND = 1020 cm–3?
-5 and 2.7
(a) (b) 2.7 and 5 (a) Reverse breakdown voltage is lower and depletion
VA
-5 and 3.85
(c) (d) 1.3 and 5 [2014] capacitance is lower


1 kΩ

Vi

1 kΩ
+

4.30 | Electronic Devices and Circuits 1kΩ


v0
I
(b) Reverse breakdown voltage is higher and depletion 3. Early effect +
+
capacitance is lower 4. Flat-band–voltage _ 10 V
_v 10 V
(c) Reverse breakdown voltage is lower and depletion (a) P-3, Q-1, R-4, S-2
capacitance is higher (b) P-1, Q-4, R-3, S-2
(d) Reverse breakdown voltage is higher and depletion (c) P-3, Q-4, R-1, S-2
capacitance is higher [2010] (d) P-3, Q-2, R-1, S-4 [2007]
Solution (d) Solution (c)
1
Breakdown voltage ∝ Population inversion takes place in LASER
doping
C ∝ doping Pinch-off voltage is characteristic of JFET
∴ As doping increases, reverse breakdown voltage Early effect in BJT
decreases whereas depletion capacitance will increase. Hence, the correct option is (c)
Hence, the correct option is (d). 19. A p+n junction has a built-in potential of 0.8 V. The
16. Consider the following assertions. + depletion layer width at a reverse bias of 1.2 V is 2 μrn.
S1: For Zener effect to occur, a very abrupt junction I is For a reverse bias of 7.2 V, the depletion layer width
required D be
will
1

S2: For quantum tunnelling to occur, a very narrow en- (a) 4 μm (b) 4.9 μm
ergy barrier is required. (c) 8 μm (d) 12 μm [2007]
Which of the following is correct? Solution (a)
(a) Only S2 is true
WD∝ 2
V0 + VRB 2
(b) S1 and S2 are both true but S2 is not a reason for S1
V 10k
(c) S1 and S2 are both true and S2 is a reason for S1
+∴ 2 = 0.8 + 1.2 20 V
(d) Both S1 and S2 are false [2008] 20 V

x 0 .8 + 7 .2
Solution (a)
For quantum tunnelling, very narrow depletion width is 2 2 1
desired. So, only S2 is true. = = x 4
x 8 4

Hence, the correct option is (a).
∴ Width = 4 μm
17. Group I lists four types of p – n junction diodes. Match
each device in Group I with one of the options in Group Hence, the correct option is (a)
II to indicate the bias condition of that device in its nor- 20. In the circuit shown below, the switch was connected to
mal mode of operation. position 1 at t < 0 and at t = 0, it is changed to position
  Group I      Group II 2. Assume that the diode has zero voltage drop and a
P. Zener diode 1. Forward bias storage time ts. For 0 < t ≤ ts, vR is given by (all in volts)
Q. Solar cell 2. Reverse bias 1
R. LASER diode
S. Avalanche photodiode
+
(a) P-1, Q-2, R-1, S-2
(b) P-2, Q-1, R-1, S-2 2
(c) P-2, Q-2, R-2, S-1
(d) P-2, Q-1, R-2, S-2 [2007] 1 kΩ V
5V A

Solution (b) 5 V

Solar cell and laser are forward biased.



Hence, the correct option is (b)
18. Group I lists four different semiconductor devices.
Match each device in Group I with its characteristic (a) VR = – 5 (b) VR = + 5
Vi
property in Group II. (c) 0 ≤ V R
< 5 (d) -5 < VR < 0
 [2006]
Group I
P. BJT Q. MOS capacitor Solution (a)
R. LASER diode S. JFET Due to change in polarity, only current direction
Group II +
P -region
changes. N -region
N >> N X 5
16
N = 10 /cm3

1. Population inversion A Initial


D = I = 5 mA
n D

1k
2. Pinch-off voltage
Chapter 2  PN Junction | 4.31

∴ VR = 5 V 1 4
Now new VR = -5 V ∴ ε1 = x = 1 = 0.5 pF
x 1 2

Hence, the correct option is (a)
Hence, the correct option is (d)
21. Find the correct match between Group 1 and Group 2
24. Match items in Group-1 with items in Group-2, most
Group 1 suitably.
E. Varactor diode F. PIN diode
G. Zener diode H. Schottky diode Group-1
P. LED
Group 2 Q. Avalanche Photodiode
1. Voltage reference R. Tunnel diode
2. High-frequency switch S. LASER
3. Tuned circuits
4. Current controlled attenuator Group-2
(a) E-4, F-2, G-1, H-3 1. Heavy doping
(b) E-2, F-4, G-1, H-3 2. Coherent radiation
(c) E-3, F-4, G-1, H-2 3. Spontaneous emission
(d) E-1, F-3, G-2, H-4 [2006] 4. Current gain
(a) P-1; Q-2; R-4; S-3
Solution (c) (b) P-2; Q-3; R-1; S-4
Varactor diode is used in tuning circuits as it can act as (c) P-3; Q-4; R-1; S-2
capacitor. (d) P-2; Q-1; R-4; S-3 [2003]
PIN diode can be used as current controlled attenuator. Solution (c)
Hence, the correct option is (c) LED-spontaneous emission
22. In an abrupt p – n junction, the doping concentrations LASER-coherent radiation.
on the p-side and n-side are NA = 9 × 1016 cm–3 and Hence, the correct option is (c)
ND = 1 × 1016 cm–3, respectively. The p – n junction
is reverse biased and the total depletion width is 3 μm. 2 5. At 300 K, for a diode current of 1 mA, a certain ger-
The depletion width on the p-side is manium diode requires a forward bias of 0.1435 V,
(a) 2.7 μm (b) 0.3 μm whereas a certain silicon diode requires a forward bias
(c) 2.25 μm (d) 0.75 μm [2004] of 0.718 V. Under the conditions stated above, the clos-
est approximation of the ratio of reverse saturation cur-
Solution (b) rent in germanium diode to that in silicon diode is
WNND = WPNA (a) 1 (b) 5
Let WN = x μm (c) 4 × 103 (d) 8 × 103 [2003]
WP = (3 - x) μm Solution (c)
16
x × 10 = (3 − x ) × q × 10
16

(
I = I 0 eVD h VT − 1 )
x = 27 - 9x For Ge → = 1
x = 2.7 mm Si → h = 2.
∴ Wp = 0.3 μm (VD1 h VT −1)
= I 0( Si ) e( D 2
Now I V h VT −1)
0 (Ge ) e
Hence, the correct option is (b)
 0.718 
2 3. Consider an abrupt p junction. Let Vbi be the built-in  −1

potential of this junction and VR be the applied reverse  2×26×10 −3 
I 0 (Ge) e 992211.23
bias. If the junction capacitance (Ci) is 1 pF for Vbi + VR = =
I 0 (Si )  0.1435  248
 −1
= 1 V, then for Vbj + VR = 4 V, Ci will be  26×10 −3 

e
(a) 4 pF (b) 2 pF
(c) 0.25 pF (d) 0.5 pF [2004] ≈ 4000.
Solution (d) Hence, the correct option is (c)
1 2 6. A particular green LED emits light of wavelength 5490
Cj ∝ Å. The energy band gap of the semiconductor material
VRB + V0
used there is (Planck’s constant = 6.626 × 10-34 J-s)
(a) 2.26 eV (b) 1.98 eV
(c) 1.17 eV (d) 0.74 eV [2003]
4.32 | Electronic Devices and Circuits

Solution (a) 30. Referring to the below figure, the switch S is in position 1 kΩ 20
1.24 1.24 1 initially and steady-state condition exists from time t I V1
EG = = = 2.26 eV = 0 to t = t0. The switch is suddenly thrown into position
λ ( µ m) 5490+× 10 −4
(eV ) 2. The current I through the 10 K resistor as a function
I
D of time t, from t = 0 is? (Give the sketch showing the
Hence, the correct option is (a) 1
magnitudes of the current at t = 0, t = t0 and t 10 = ∞).
V
27. A p – n junction in series with a 100 ohms resistor is  [1991]
forwarded biased so that a current of 100 mA flows. If
the voltage across this combination is instantaneously
reversed at t = 0, current through
D2 diodes is approxi- 2
mately given by 10k I
(a) 0 mA V
(b) 100 mA
+ 20 V 20 V
(c) 200 mA (d) 50 mA [1998]
Solution (b)
Due to change in polarity of voltage, only direction of
current flow changes but magnitude remains the same. Solution: 2 mA
Hence, the correct option is (b) Due to change in polarity, J =  20  = 2 mA
10 × 1000 
28. The built-in potential (diffusion potential) in a p – n 31. In a uniformly doped abrupt p – n junction, the doping
junction level of the n-side is four (4) times the doping level R1
(a) is equal to the difference in the Fermi-level of the of the p side. The ratio of the depletion layer width
two sides, expressed in volts V1 of V0
n-side verses p-side is 1 kΩ
(b) increases with the increase in the doping levels of (a) 0.25 (b) 0.5 R2 1 kΩ
the two sides (c) 1.0 (d) 2.0 [1990]
(c) increases with the increase in temperature 1
(d) is equal to the average of the Fermi levels of the Solution (a)
+ 2V
WNND = WPNA –
two sides [1993]
+
Solution (a) and (b) WN NA 1
2 = = = 0.25
kT N N WP ND 4
VT = log e A 2 D
q ni kΩ
1Hence, the correct
V option is (a)
5V A

∴ VT ↑ as NA, ND ↑
5V 3 2. The switching speed of P+ N junction (having a heavily
doped P region) depends primarily on
Also, VT is equal to the difference in Fermi-level of two (a) the mobility
sides, expressed in volts. – of minority carriers in the P+-region.
1 kΩ
(b) the lifetime of minority carriers in the P+-region
Hence, the correct option is (a) and (b) (c) the mobility of majority carriers in the N-region
29. The small signal capacitance of an abrupt P + n junc- (d) the lifetime of majority carriersVi in the N-region
tion is 1 nf/cm2 at zero bias. If the built-in voltage is  [1989] 1 kΩ
1 volt, the capacitance at a reverse bias voltage of 99 Solution: (d) V0
volts is Switching speed depends upon the injected minority 0.7 V
(a) 10 (b) 0.1 carriers. So, injected minority carries in N region.
(c) 0.01 P -region (d) 100
+ [1991]
N -region
X 16
N = 10 /cm 3 Injected minority carriers are majority carriers. 2V
Solution (b) N >>A N D
n D

Hence, the correct option is (d)


1
C∝ 33. In a Zener diode
VRB + V0 (a) only the P-region is heavily doped
(b) only the N-region is heavily doped
C1 VRB 2 + V0
= (c) both P- and N- regions are heavily doped
C2 VRB1 + V0 (d) both P- and P- regions are lightly doped
 [1989]
1 100 Solution (c)
= = x = 0.1 nF/cm 2
x 1 Both regions are heavily doped.
Hence, the correct option is (b) Hence, the correct option is (c)
Chapter 2  PN Junction | 4.33

34. For p-n junction, match the type of breakdown with Diode D1 is in forward bias
phenomenon Diode D2 is in reverse bias
1. Avalanche breakdown
So, the current through diode D1 is forward current If
2. Zener breakdown
and current through diode D2 is
3. Punch through
A. Collision of carriers with crystal ions reverse current l0.
B. Early effect So, total current ≈ I = lf + lo
C. Rupture of covalent bond due to strong electric
I = I o  e d T - 1 = I o e d T - I o
V /nV V /nV
field.
 
(a) 1-B. 2-A. 3-C


(b) 1-C. 2-A. 3-B
(c) 1-A. 2-B. 3-C
+
(
I = I o eVd /hVT - I o + I o ) 1 kΩ 20 kΩ

1kΩ
(d) 1-A. 2-C. 3-B v0 [1988] Vd /hVT
I =I I o e
Solution (d) +
+ I
1-A, 2-C, 3-B – _ 10 V eVd hV T
_v = 10 V D1
Io
Avalanche breakdown → Collision of carriers with
crystal ions. Vd  I 
= ln  
Zener breakdown → Rupture of covalent bond due to hVT  Io 
strong electric field.
 I 
Punch through → Early effect. Vd = hVT ln  
 Io 
Hence, the correct option is (d)
35. In the circuit shown below, the current voltage relation- KT  I 
Vd = ln   {For Ge,h = 1}
ship when D1 and D2 are identical is given by (Assume q  Io 
Ge diodes) 1 kΩ 20 kΩ
∴ Vd = V I
V1
+ KT  I 
V= ln  
q  Io 
I
D1
Hence, the correct option is (b) 10 V

36. The diffusion capacitance of a p – n junction


(a) decreases with increasing current and increasing
temperature
D2
(b) decreases 2with decreasing current and increasing
temperature 10k I
V
+ 20(c) 
V i ncreases with
20 V increasing current and increasing
temperature
(d) does not depend on current and temperature[1987]
KT I Solution (b)
(a)
V= sinh  
q 2 Decreases with decreasing current and increasing
temperature.
KT  I  τ
(b)
V= ln  
q Diffusion capaci tan ce = CD = τ g =
 Io  r R1

ηVT V1 V0
KT I
(c)
V= sinh-1   r= 1 kΩ
q 2 lf R2 1 kΩ

KT Tl f Tl f
1 CD = =
(d)
V= [exp(-I ) - 1]  [1988]
q ηVT η kT + 2V

CD ∝ If +
Solution (b) 2 CD ∝
KT  I  Hence, the correct option is (b)
V= ln   1 kΩ VA
q  Io  5V
5V


1 kΩ

Vi
4.34 | Electronic Devices and Circuits

Five-marks Question
e −V1 / uVT − 1 = −e −V2 .nVT + 1
1. For the circuit shown in figure. D1 and D2 are identical eV1 / uVT + e 0V2 / nVT = 2
diodes with utility factor of unity. The thermal voltage
VT = 25 mV V1 + V2 = 50 mV  …….. given
(a) Calculate V1 and V2. V2 = 50 – V1
(b) If the reverse saturation current of D1 and D2 are 1
and n = 1 .……. given
pA then compute the current I through the circuit

D1 D2 ∴ eV1 /VT + e −( 50 −V1 )/VT = 2
V /V V /V −50 /VT
+ − + − e 1 T + e 1 T .e =2
V1 V2
e
V1 /VT
[1 + e −50 / 25 ] = 2
V1 /VT 2
e =
50 mV 1 + e −2

V1  2 
= ln   = 0.566
VT  1 + e −2 
[2001]
V1 = VT × 0.566 = 25 × 10−3 × 0.566
Solution: n = 1 (for Ge diode)
V1 = 14.15 mV
Diode D1 is forward bias and diode D2 is reverse bias.
V2 = 50 – 14.15 = 35.85 mV
Cut in voltage of Ge diode
Vy = 0.2 V I = I 0 eV1 / nVT − 1
(b)
 
∴ 0.2750 mV
So no biasing I 0 = 10 −12 Amp 

Since both diodes are in series ∴ forward current of D1 n =1  given
is equal to reverse current of D2 VT = 25 mV 

∴ ID =ID
F1 r2
I = 0.76 p Amp
I 0 eV1 / nVT − 1 = I 0 e −V2 / nVT − 1
   
Chapter 3
Bipolar Junction Transistor
Hence, there is a decrease in High frequency cut off
One-mark Questions frequency.
1. For a narrow base PNP BJT, the excess minority car- Hence, the correct option is (D).
rier concentrations (∆nE ⋅for emitter, ∆pB ⋅for base, ∆nC ⋅ 3. An npn bipolar junction transistor (BJT) is operating
for collector) normalized to equilibrium minority car- in the active region. If the reverse bias across the base-
rier concentrations ( nE 0 for emitter, pBo for base. nC0 collector junction is increased, then
for collector) in the quasi-neutral emitter, base and col- [2017]
lector regions are shown in the figure. Which one of the (A) the effective base width increases and common-
following biasing modes is the transistor operating in? emitter current gain increases
 [2017] (B) the effective base width increases and common-
Carrier Concentration

emitter current gain decreases


ΔpB
Normalized Excess

pBo
(C) the effective base width decreases and common-
ΔnC
≈ emitter current gain increases
nCo (D) the effective base width decreases and common-

0 emitter current gain decreases


ΔnE −1
nEo 4. If fixed positive charges are present in the gate oxide of
Emitter (P) Base (N) Collector (P) an n-channel enhancement type MOSFET, it will lead
 X and Y axes are not to scale to
(A) Forward active (B) Saturation (a) a decrease in the threshold voltage
(b) channel length modulation
(C) Inverse active (D) Cut off
(c) an increase in substrate leakage current
Solution:  From the diagram given in problem, emitter- (d) an increase in accumulation capacitance
base junction is in reverse- and collector-base junction
[2014]
is in forward bias so it is operating in reverse active
region. Solution (a)
Hence, the correct option is (C). This decreases the threshold voltage
2. The Miller effect in the context of a Common Emitter Hence, the correct option is (a)
amplifier explains [2017] 5. An increase in the base recombination of a BJT
(A) an increase in the low-frequency cutoff ­frequency increases
(B) an increase in the high-frequency cutoff f­ requency (a) the common emitter DC current gain β
(C) a decrease in the low-frequency cutoff frequency (b) the breakdown voltage BVCEO
(D) a decrease in the high-frequency cutoff frequency (c) low temperature dry oxidation
Solution:  From the high frequency response analysis (d) low energy ion-implantation
1 [2014]
of CE amplifier, we can say that f H =
2π Cin Rin Solution: (b)
Cin = Cπ + Cµ [1 + g m Rc ] Base recombination will lead to decrease in β and
hence increase in BVcε0.
1+ g m Rc is known as miller multiplier due to miller Hence, the correct option is (b)
­effect in CE amplifier input capacitance increases and
4.36 | Electronic Devices and Circuits

6. In CMOS technology, shallow P-well or N-well regions Solution: (b)


can be formed using ∂I D
(a) low pressure chemical vapour deposition gm = = 2k (VGs − VT )
∂VGS
(b) low energy sputtering
(c) low temperature dry oxidation Hence, the correct option is (b)
(d) low energy ion-implantation 1 1. The phenomenon known as ‘Early Effect ’ in a bipolar
[2014] transistor refers to a reduction of the effective base-
Solution (d) width caused by
Wells are formed using low level ion implantation. (a) electron-hole recombination at the base
(b)  the reverse biasing of the base-collector
Hence, the correct option is (d) junction
7. In MOSFET fabrication, the channel length is defined (c) the forward biasing of emitter-base junction
during the process of (d) the early removal of stored base charge during sat-
(a) isolation oxide growth uration-to-cutoff switching
(b) channel stop implantation [2006]
(c) poly-silicon gate patterning
(d) lithography step leading to the contact pads Solution (b)
[2014] Electron hole recombination is responsible for early
effect.
Solution (c)
Hence, the correct option is (b)
Poly-silicon gate patterning defines the length.
12. If for a silicon n-p-n transistor, the base-to-emitter
Hence, the correct option is (c) voltage (VBE) is 0.7 V and the collector-to-base voltage
8. In a MOSFET operating in the saturation region, the (VCB) is 0.2 V, then the transistor is operating in the
channel length modulation effect causes (a) normal active mode
(a) an increase in the gate-source capacitance (b) saturation mode
(b) a decrease in the transconductance (c) inverse active mode
(c) a decrease in the unity-gain cut-off frequency (d) cutoff mode
(d) a decrease in the output resistance [2004]
[2013] Solution: (a)
Solution (d) VBε > VCB
Channel length modulation affects the output resistance. ∴ Normal active mode.
Hence, the correct option is (d) Hence, the correct option is (a)
9. At room temperature, a possible value for the mobility 13. Consider the following statements S1 and S2.
of electrons in the inversion layer of a silicon n-channel
MOSFET is S1 The β of a bipolar transistor reduces if the base width
(a) 450 cm2/V-s (b) 1350 cm2/V-s is increased.
(c) 1800 cm /V
2 -s
(d) 3600 cm2/V-s S2 The β of a bipolar transistor increases if the doping
[2010] concentration in the base is increased.
Solution (b) Which one of the following is correct?
(a) S1 is False and S2 is True
This is a theoretical value of 1350 cm2/V-s. (b) Both S1 and S2 are True
Hence, the correct option is (b) (c) Both S1 and S2 are False
10. The drain current of a MOSFET in saturation is given (d) S1 and True and S2 is False
by lD = K (VGS -VT)2 where K is a constant. The magni- [2004]
tude of the trans-conductance gm is Solution (d)
As base width increases, Ib increases.
K (V - V ) 2
(a) GS T (b) 2K (VGS - VT) β = I c ∴ β will decrease.
VDS Ib
I K (VGS - VT ) 2
(c) D (d) Also, if doping in base is increased, then Ib ↑ ∴ β will
VGS - VDS VGS reduce.
[2008] Hence, the correct option is (d)
Chapter 3  Bipolar Junction Transistor  |  4.37

14. For an n-channel enhancement type MOSFET, if the (c) base-emitter junction is forward biased and base-
source is connected at a higher potential than that of collector junction reverse biased
the bulk (i.e., VSB > 0), the threshold voltage VT of the (d) both the junctions are forward biased
MOSFET will [1995]
(a) remain unchanged (b) decrease
Solution (d)
(c) change polarity (d) increase
[2003] Both the junctions are forward biased.
Solution (d) Hence, the correct option is (d)
NA 2qN Aε si 19. The Ebers Moll model is applicable to
Where ϕs = VT ln and V e =
Vie Cox (a) Bipolar junction transistors (BJT)
(b) NMOS transistors
As VSB Positive, therefore voltage VT increases.
(c) Unipolar junction transistors
Hence, the correct option is (d) (d) Junction field-effect
[1995]
15. MOSFET can be used as a
Solution (a)
(a) current-controlled capacitor
(b) voltage-controlled capacitor BJT are modelled by EM model
(v) current-controlled inductor Hence, the correct option is (a)
(d) voltage-controlled inductor
20. The early-effect in a bipolar junction transistor is
[2001]
caused by
Solution: (b)
(a) Fast-turn-on
The MOSFET is a voltage-controlled capacitor. (b) Fast-turn-off
Hence, the correct option is (b) (c) large collector-base reverse bias
(d) large emitter-base forward bias
16. The effective channel length of a MOSFET in satura-
 [1995 & 1999]
tion decreases with increase in
(a) gate voltage (b) drain voltage Solution (c)
(c) source voltage (d) body voltage As VCB ↑, more reverse-biasing causes the base width to
[2001] reduce, so if VCB is changed, base width can be altered.
This process is called early- effect
Solution (b)
Hence, the correct option is (c)
Increase in drain voltage decreases the channel length.
Hence, the correct option is (b) 21. The threshold voltage of an n-channel MOSFET can be
increased by
17. The break down voltage of a transistor with its base (a) increasing the channel dopant-concentration
open is BVCEO and that with emitter open is BVCBO, then (b) reducing the channel dopant concentration
(a) BVCEO = BVCBO (c) reducing the GATE oxide thickness
(b) BVCEO > BVCBO (d) reducing the channel length
(c) BVCEO < BVCBO [1994]
(d) BVCEO is not related to CBO Solution (a)

[1995] VT = VT 0 + γ  2φ f + VSB − 2φ f 
 
Solution (c)
2qN Aε 2qN Aε
1 γ = = to x
β vCEO = β vCBO n Cox 3.45 × 10 −11
β

So, as NA ↑ γ ↑ and hence VT ↑
Where β > 1
Hence, the correct option is (a)
∴ βVCEO < βVCBO.
Hence, the correct option is (c) 22. The transit time of the current carriers through the
channel of a JFET decides its_____ characteristic.
18. A BJT is said to be operating in the saturation region if (a) source (b) drain
(a) both junctions are reverse biased (c) GATE (d) source and drain
(b) base-emitter junction is reverse biased and base [1994]
collector junction is forward biased
4.38 | Electronic Devices and Circuits

Solution (b) 10-13 mA. The transconductance of the BJT (in mA/V)
The drain decides the transit time. is____. [2014]
Hence, the correct option is (b) Solution: 5.76
2 3. Channel current is reduced on application of a more Ic I E
gm = ≈
positive voltage to the GATE of the depletion mode VT VT

n-channel MOSFET. State True/ False.
 VBε 
[1994] I E = I s  e VT 
Solution: False  
 
Because for + ve Vas current will increase due to for- 0.7×1000
ward biasing of Gate. −13
= 10 × e
25

= 0.144 mA
Two-marks Questions 0.144
∴ gm = × 103 = 5.76 mA/V
1. The injected excess electron concentration profile in 26
the base region of an NPN BJT, biased in the active
region, is linear, as shown in the figure. If the area of 3. A depletion type N-channel MOSFET is biased in its
the emitter base junction is 0.001 cm2, µn = 800 cm2/ linear region for use as a voltage controlled resistor.
(V – s) in the base region and depletion layer widths are Assume + threshold voltageGateVTH = -0.5 V, VGS = 2.0 V, VDS
negligible, then the collector current Ic(in mA) at room = 5 V, W/L
VGS
= 100, C ox
= 10-8
F/cm2 and µn = 800 cm2V/-
+
P
gm
temperature is _________. s. The value of the resistance of the voltage controlled
resistor– (in Source
Ω) is___. tch N Drain
(Given: Thermal voltage VT = 26 mV at room tempera- [2014]
ture, electronic charge q = 1.6 × 10–19C)  [2016] P
+
Solution (500)
IB
vG 1
n p n γ ds =
1014 cm–3 Excess w
µnC0 × (VGs − VT )
electron L 1 m
IE profile IC
1
= −8
= 500Ω G
800 × 10 × 100( 2.5)
1 m
D
0 gm 4. For the n-channel MOS transistor shown in figure, the
1nm
threshold voltage VTh is 0.8 V. Neglect channel length
0.5 μm 0.2 m
S 0.2 Vm = 1.6 V,
D When the drain voltage
modulation effects. D
Solution:  Area of the emitter base junction A = the drain current
0.2 Im
D
wasp substrate
found to 0.2
be 0.5
m mA. If VD is
0.001 cm2, v
adjusted to be 2 V by changing the values of R and VDD,
B
µn = 800 cm2/V–S G the new value of ID (in mA) is
Charge q = 1.6 × 10-19C
VDD
VDD
Voltage VT = 26 mV
gm R
As we know that the current density for NPN t­ ransistor
is R=
D
dn Ι
J = q ⋅ Dn ⋅ =
dx A  v G
G
−19
I = J ⋅ A = 1.6 × 10 × 800 × 26 × 10 −3 VDD
gm S M
1014 
× × 0.001
0.5 × 10 −14
= 6.656 mA (a) 0.625 (b) 0.75
Hence, the correct Answer is (6.656 mA). (c) 1.125 (d) 1.5 [2014]
Solution (c)
2. A BJT is biased in forward active mode. Assume VBE =
0.7 V, kT/q = 25 mV and reverse saturation current
vG Is = I D = kn′ (VGS − VT ) 2 VDD = + 12V

R2 10 k
gm
1 m
D
1nm

0.2 m 0.2 m
D S
0.2 m p substrate 0.2 m

B
Chapter 3  Bipolar Junction Transistor  |  4.39

VDD for
0.5 = kn′ (1.6 − 0.8) 2 V saturation region
DD
kn 2
R ID = VGS − VT 
25
kn′ = mA/V 2 2
R = 10 k
32 D
kn
ID = = VGS − VT 
Now I D′ =
25 2
(2 − 0.8 ) = 1.25 mA 2VDD 
G 32
VDD ∂ ID k 10 −2
Hence, the correct option
S is (c) M = n = = 10 −1 × 0.707 = 0.0707 A / V
∂VGS 2 2
5. For the MOSFET shown in the fig-
∂ ID kn| = 102 −2 V and
ure, the threshold voltage |V
= 1 = = 10 −1 × 0. 707 = 0.0707 A / V
∂VGS 2 2
1 W  7. An ideal MOS capacitor has boron doping-concentra-
K = µC   = 0.1 mA / V 2 . The value of ID
2  L tion of 1015 cm-3 in the substrate. When a gate voltage
(in mA) is____. [2014] is applied, a depletion region of width 0.5 µm is formed
+ Gate
with a surface (channel) potential of 0.2 V. Given that
VDD = + 12V
vG ε0 = 8.854 × 1014 F/cm and the relative permittivities of
VGS P
+
silicon and silicon dioxide are 12 and 4, respectively,
gm
– Source tch N
Drain the peak electric field (in V/µm) in the oxide region
R2 10 k
is______. [2014]
+
P Solution: 2.4
vG We know that peak electric field
εs
Emax = ⋅εs
ε ox
vG 1 m
R2 10 k ID
0.2 × 2 4
G Es = −16
= × 106 V / m
0.5 × 10 5
1 m
D
gm 1nm 12 4
Emax = × = 2.4 V/µ m
G
0.2 m m
0.2 4 5
D S
0.2 m p substrate 0.2 m 8. For the MOSFET M1 shown in the figure, assume W/L
VSS = – 5V B = 2, VDD = 2.0 V, µn COX = 100 µA/V2 and VTH = 0.5
vG
V. The transistor M1 switches from saturation region to
linear region when Vin (in volts) is_____.
W Drain Solution (0.9)
VDD
ID1 = ID2 VDD
gm
ID = 0.1 (0 - ( - 5) - 2)2 R

= 0.1(9) = 0.9 mA R = 10 k
D
G 6. The slope of the ID vs. VGS curve of an n-channel
MOSFET in linear region is 10-3 Ω-1 at VDS = 0.1 V. For VDD
vG
the same device, neglecting channel
G length modulation,
the slope of the I D vs. VGS (in A /V) under satura- VDD
gm S [2014] M
tion region is approximately______.
Solution: 0.0707
 V 2 DS 
I D = kn (VGS − VT )VDS − 
 2 
[2014]
∂I D Solution (1.5)
= kn [VDS ] = 10 −3
v ∂VGS VDD = + 12V
ID in saturation region
G
kn × 0.1 = 10-3
kn′
kn = 10-2 = (VGs − VT ) 2
R2 2
10 k
gm

vG
R2 10 k ID
4.40 | Electronic Devices and Circuits

100 Solution (d)


= × 2(VGs − VT ) 2
2 Volume = area × depth
At verge of linear region, VGs - VT = VDs = 10-8 × 10-4 = 10-12 cm3
VDS = V0 ∴ND = 1019 atoms/cm3
2
ID = 100 V02 µA = 100 × 10-6 V02 (A) ni 10 20
= P = = 10 atoms/cm 3
Now VDD = 104 I + V0 N D 1019

VDD = 2 V Number of holes in 10-12 cm3 volume P = 10 × 10-12


4 −4 2
2 = 10 × 10 V0 + V0 =10-11 ≈ 0.
+ Gate is (d)
V02 + V0 - 2 = 0 Hence, the correct option
(V0 + 2) (V0 - 1) Common
VGS Data for PQuestions
+ 11 and 12
gm= 0 V0 = 1 V
Vin - VT = 1 In the
– three-dimensional
Source tch N
view
Drain of a silicon n channel
MOS transistor shown below, δ = 20 nm. The transistor
Vin = 1 + 0.5 = 1.5 V is of width 1 µm. The P depletion width formed at every
+

9. Consider two BJTs biased at the same collector cur- p--n junction is 10 nm. The relative permittivities of Si
rent with area A1 = 02 µm × v0.2
G µm and A = 300 µm.
2 and SiO2, respectively, are 11.7 and 3.9, and ε0 = 8.9 ×
Assuming that all other device parameters are identical 10-12 F/m.
kT/q = 26 mV, the intrinsic carrier concentrations is 1 ×
1 m
1010cm-3, and q = 1.6 × 10-19C, the difference between
the base-emitter voltages (in mV) of the two BJTs (i.e., G
VBE1 - VBE2) is______. [2014]
1 m
Solution: 380.28g D
1nm
m
Is ∝ Area
0.2 m 0.2 m
D S
Now Ic1 = Ic2
VBE1 0.2 m p substrate 0.2 m
η VT VBE2 /η VT
I s1e = I s2e vG B

VBE1 −VBE 2 12. The source-body junction capacitance is approximately


VT I S 2  300 × 300  VDD(b) 7 fF
e = = (a) 2 fF
I S1  0.2 × 0.2 
VDD
gm (c) 2 pF (d) 7 pF [2012]
R
 300 × 300  Solution (a)
VBE1 − VBE 2 = 26 ln   = 380.28 mV R = 10 k
 0.2 × 0.2  Source body Ixn capacitance
D

εA
10. A BJT in a common-base configuration
vG is used to Cj = VDD
amplify a signal received by a 50 Ω antenna. Assume dG
kT/q = 25 mV. The value of the collector bias current ε = εγ (Si) × εo = 11.7 × 8.85 × 10-12 f/m VDD M
g S
(in mA) required tom match the input impedance of the A = 1 µm × 0.2 µm = 0.2 × 10-12 m2
amplifier to the impedance of the antenna is_______.
d = 10 nm = 10-8 m
[2014]
11.7 × 8.85 × 10 −12 × 0.2 × 10 −12
Solution: 0.5 ∴C j =
10 −8
γC = characteristics impedance = 50 Ω
VT 25 C j ≅ 2 ff

Ic = = × 10 −3 = 0.5 mA V = + 12V
Ve 50 vG Hence, the correct optionDD
is (a)
11. The source of a silicon (n1 = 1010 per cm3) n-channel 1 3. The gate-source overlap capacitance is approximately
MOS transistor has an area of 1 sq µrn and a depth of (a) 0.7 fF (b) 0.7 pF
1 µm. If the dopant density in the source is 1019 cm–3, (c) 0.35RfF
2 10 k (d) 0 .24 pF [2012]
the number of gholes
m in the source region with the above Solution (a)
volume is approximately
εA
(a) 107 (b) 100 Cg =
(c) 10 (d) 0 [2012] d
vG
R2 10 k ID

VG
Gate
Chapter 3  Bipolar Junction Transistor  |  4.41

where ε = ε ( SiO2 ) × ε o Solution: (c)


Channel resistance ∝ channel width
A = 1 µm × δ = 1 µm × 20 nm = 20 × 10-15 m
VGs = 0 V, W = 1 V
d = 10-9 m
VGs = -3 V, W = ?
3.9 × 8.85 × 10 −12 × 20 × 10 −5
Cg = = 0.7 ff 1 −1
10 −9 = x = 2 µm
x −3 − 1
Hence, the correct option is (a)
∴ Net thickness for conduction will be 6 µm.
14. For a BJT, the common-base current gain α = 0.98 and
the collector base junction reverse bias saturation cur- Now, tch1 × R1 = tch2 × R2
rent ICO = 0.6 µA. This BJT is connected in the common for Q. 3.41. 10 × 600 = 8 × R2
emitter mode and operated in the active region with a R2 = 750 Ω
base drive current IB = 20 µA. The collector current IC Q. 3.42. 10 ×600 = 6 × R2
for this mode of operation is
(a) 0.98 mA (b) 0.99 mA R2 = 1000 Ω
(c) 1.0 mA (d) 1.01 mA [2011] Hence, the correct option is (c)
Solution (d) 17. In a uniformly doped BJT, assume that NE, NB and Nc are
Ic = βIb + (1 + β) Ic0 the emitter, base and collector dopings in atoms cm–3,
α 0.98 respectively. If the emitter injection efficiency of the
β= = = 49 BJT is close to unity, which one of the following condi-
1 − α 0.02
tions is TRUE?
Ic = 49 × 20 + 50 × 0.6 (a) NE = NB = NC
= 980 + 30 = 1010 µA = 1.01 mA (b) NE >> NB and NB > NC
Hence, the correct option is (d) (c) NE = NB and NB < NC
Common Data for Questions 14 and 15. (d) NE < NB < NC [2010]
The channel resistance of an n-channel JFET shown in Solution: (b)
the figure below is 600 Ωwhen the full channel thick-
ness (tch) of 10 µrn is available for conduction. The Emitter is heavily doped to provide carriers.
built-in voltage of the gate P+N junction (Vbi) is -1. Hence, the correct option is (b)
When the gate to source voltage (VGS) is 0 V, the chan-
nel is depleted by 1 µrn on each side due to the built-in 18. Consider the following two statements about the inter-
voltage and hence the thickness available for conduc- nal conditions in an n-channel MOSFET operating in
tion is only 8 µm. the active region
S1: The inversion charge decreases from source to drain
+ Gate
S2: The channel potential increases from source to drain
VGS P
+
Which of the following is correct?
– Source Drain (a) Only S2 is true
tch N
(b) Both S1 and S2 are false
+
P (c) Both S1 and S2 are true, but S2 is not a reason for S1
G
(d) Both S1 and S2 are true, and S2 is a reason for S2
 [2009]
Solution (d)
15. The channel resistance when VGS = 10 V
m is
(a) 480 Ω (b) 600 Ω Both are true but both have no relation.
G
(c) 750 Ω (d) 1000 Ω [2011] Hence, the correct option is (d)
Solution: (c)D 1 m
1nm 19. The measured trans-conductance gm of an NMOS tran-
Hence, the correct option is (c) sistor operating in the linear region is plotted against
0.2 m resistance when V = -3 V
16. The channel the gate voltage VG at a constant drain voltage VD.
D GS S 0.2is m
(a) 360 Ω 0.2 m p substrate (b) 917
0.2 m
Ω Which of the following figures represents the expected
(c) 1000 Ω (d) 3000 Ω [2011] dependence of gm on VG? [2008]
vG B

VDD
VDD
R

R = 10 k
D
R2 10 k
gm

4.42 | Electronic Devices and Circuits vG


R2 10 k
++
+ + Gate
Gate
GateGate
(a) geometrical parameters and doping levels remaining
VV
VGS
GS PP
++
+
gg
gmm + VGS
GS the same), P thenP the ratio between the mutual transcon-
Gate
+
m gm
–– ductances of the initial
Drainand Vthe modified JFET is
Source Drain
– Source
–Source ttch NN + Drain Drain
VGS Source tch ch N tchP N G
gm Gate
– ++ Drain
Source P +
PP P +
tch N +
P
vvvGG v +
G G P

vG Source n W Drain
11
1 mm
m1 m
(b)
GG
G G1 m
+
P
11
1 mm
m1 m
DD
D D Gate G
gg
gmm 1nm
1nm
1nm1nm
m gm VG
0.2
0.2 m 1 m
0.2 m0.2
m Dm D 0.2 mm
0.2
gm
D
D D 1nm S 0.2
SS m
S 0.2 m
0.2 mm pp substrate 0.2 m  1− 2 / VP 
0.2
0.2 m m
0.2 psubstrate
substrate 0.2
0.2 m0.2
p substrate m m  
0.2 m (a) 4
S 0.2 (b) m
 1−
vvvGG v
D
BB  1 /( 2VP ) 
G B B
G 0.2 m p substrate 0.2 m

(c) vG 1  1B
(c) 
− 2 / VP 
VV  (d)
1 − 2 / VP ( )
( ( ))
V DD V
2  1 − 1 /( 2DD
DD
VP )DD 1 − 1/ 2 VP VV
VDD V
DD
DD
DD
gg
gmm RR
m gm R VRDD
VDD
RR
R== 10 k
R k=k10 k
=10
10
gm DD
D R
[2008]
D
Solution (b) R=V
10
V
V
k
vvvGG v D DD V
DD
DD
DD
G GG 
G V −V 
G
G
gm = k N ′ 1 − bi Gs  VV
V VDD
vG DD V
DD MM
S S Vp
DD
(d) gg
gmm SS  DD M M
m gm G
VDD
gm kN′ = trans conductance
S parameter M
kN′ ∝ w
 Vbi − VGs1 
1− 
gm1 k Vp1
∴ = n1  
gm 2 k  12VVbi − VGs 2

VV =+++ 12
−=V+V 12
n 2==
vvvGG v
G
VDD V
DD
DD DD112 V 
G
 Vp2 
Solution (a) VDD = + 12V
vG
qN D a 2
RR
R
Vp =
10
10 kk 2a = width of tranistor
2εk
2 2 R2 10 k10
2
gg
gmm
m gm
R∴
2 Vp10
will
k be four times
gm
 0 − ( −2) 
1 − 
gm1 1  Vp 
vvvGG v =
  IDIID
G G RR 10
gm210
R2 22 R 10kkk10
2 k 0 − ( −2) D ID
2
 1− 
vG  4Vp 
R2 10 k ID

Initially gm increases as VG increases but as more and


1  1 − 2 / Vp 
more VG is increased,Vmobility
V
VGG
will get decreased. So, =  
G VG 2  1 − 1 / 2Vp 
gm will start
Gate
Gate reducing.
GateGate
Hence, the correct option
P+ + Gis
PP
+ V
+
(a) Hence, the correct option is (b)
Gate P
20. The cross section of a JFET is shown in the following VV
V ==
=–– 5VV
–55 V 5V
2 1. An MOS capacitor made SS V
SS
SS
SS = – p-type
using substrate is in
figure. Let VG be -2 VPand + let Vp be the initial pinch- the accumulation mode. The dominant charge in the
offSource
Source
Source
Source voltage. If the width
n
nn nW W isW doubled
W
W Drain
Drain
Drain (with other
Drain
VSS = – 5V
channel is due to the presence of

Source n W Drain
++ +
PP
P P+
Gate
Gate
GateGate VV
PVGG
+
G VG

Gate
VG
Chapter 3  Bipolar Junction Transistor  |  4.43

(a) holes I D1 (VGS1 − VT ) 2


(b) electrons =
(c) positively charged ions I D2 (VGS 2 − VT ) 2

(d) negatively charged ions [2005] 2
1  900 − 400  1
Solution (a) =  =
x  1400 − 400  4
The dominant charge depends on the substrate type.
∴ ID2 = 4 mA
Hence, the correct option is (a)
Hence, the correct option is (d)
22. Consider the following statements S1 and S2.
25. An n-channel JFET has lDSSS = 2 mA and V = -4 V. It
S1: The threshold voltage (VT) of a MOS capacitor
has transconductance gm (in mV/V). An applied GATE
decreases with increase in gate oxide thickness
to source voltage VGS of -2 V is
S2: The threshold voltage (VT) of a MOS capaci- (a) 0.25 (b) 0.5
tor decreases with increase in substrate doping (c) 0.75 (d) 1.0 [1999]
concentration
Solution (b)
Which one of the following is correct?
(a) S1 is False and S2 is True −2 I DSS  Vqs 
gm = 1 − VP 
(b) Both S1 and S2 are True VP  

(c) Both S1 and S2 are False
(d) S1 is True and S2 is False [2004] −2 × 2  2
= 1 − 4  = 1 / 2 = 0.5 mV/V
Solution (c) (− 4 )  

VT = VT 0 + γ  2φ f + VSB − 2φ f  Hence, the correct option is (b)


 
2 6. In a bipolar transistor at room temperature, if the emit-
2qN Aε 2qN Aε ter current is doubled, the voltage across its base-emit-
γ = = tox ter junction

Cox 3.45 × 10 −11 (a) doubles
As tox↑ γ ↑ VT ↑ (b) halves
(c) increases by about 20 mV
NA ↑ γ ↑ VT ↑
(d) decreases by about 20 mV [1997]
∴ Both statements are false.
Solution (c)
Hence, the correct option is (c)
I = I E 0 [eVBε /ηVT − 1]
23. The drain of an n-channel MOSFET is shorted to the
gate so that VGS = VDS. The threshold voltage (VT) of for transistors, x = 1
MOSFET is 1 V. If the drain current (ID) is 1 mA for VGS
= 2 V, then for VGS = 3 V, ID is  VBε1 
(a) 2 mA (b) 3 mA I E 0 e ηVT − 1
 
(c) 9 mA (d) 4 mA [2004] I1  
= I 2 = 2 I1
Solution: (d) I2  V B ε 2 
I E 0 e ηVT − 1
ID∝ (VGs - VT)2  
 
1 ( 2 − 1) 2
= I D 2 = 4 mA VBε1
I D2 (3 − 1) 2 −1
1 e VT
= V ε
Hence, the correct option is (d) 2 B 2 −1
VT
2 4. When the gate-to-source voltage (VGS) of a MOSFET e
with threshold voltage of 400 mV, working in satura- VBε1
tion, is 900 mV, the drain current is observed to be 1 1 e VT
mA. Neglecting the channel width modulation effect ≈ V ε
2 B 2
and assuming that the MOSFET is operating at satura- VT
e
tion, the drain current for an applied VGS of 1400 mV is
VBε1 −VBε 2
(a) 0.5 mA (b) 2.0 mA 1 VT
(c) 3.5 mA (d) 4.0 mA [2003] =e
2
Solution (d)
ID = k(VGS - VT)2
4.44 | Electronic Devices and Circuits

VB ε1 − VB ε 2 Solution: (A → 3, B → 3, C → 4)
−0.693 = qN D a 2
VT (i) Vp =

VT = 26 × 10-3 V.
So, as ND ↓, VP ↓
VB ε 2 − VB ε1 ≅ 18 mV. (ii) gm ↑ means conductivity increases.
∴VBε increases by about 20 mV. 1
(iii) Transit time ∝
Hence, the correct option is (c) length of channel
27. If a transistor is operating with both of its junctions for- 30. In a transistor having finite β, forward bias across the
ward biased, but with the collector base forward bias base emitter junction is kept constant and the reverse
greater than the emitter-base forward bias, then it is bias across the collector base junction is increased.
operating in the Neglecting the leakage across the collector base junc-
(a) forward active mode tion and the depletion region generations current, the
(b) reverse saturation mode base current will ___ (increase/decrease/remains con-
(c) reverse active mode stant). [1992]
(d) forward saturation anode [1996] Solution: (decrease)
Solution: (b) As the reverse bias increases at CB (collector base)
Hence, the correct option is (b) junction, the collector current (Ic) increases and the
28. In a bipolar junction transistor (match the following). effective base width decreases. So the recombination
in base decreases.
List-I
A. The current gain increases As reverse bias ↑, base width ↓, hence IB will decrease.
B. The collector breakdown voltage increases 31. An n-channel JFET has a pinch-off voltage Vp = -5 V,
C. The cut off frequency increases VDS(max) = 20 V, and gm = 2 mA/V. The min 'ON' resist-
List-II ance is achieved in the JFET for
1. The base doping is increased and the base width is (a) VGS = -7 V and VDS = 0 V
reduced (b) VGS = 0 V and VDS = 0 V
2. The base doping is reduced and the base width is (c) VGS = 0 V and VDS = 20 V
increased (d) VGS = -7 V and VDS = 20 V [1992]
3. The base doping the base width are reduced Solution (b)
4. The emitter area is increased and the collector area For minimum γds, Vas should be +ve and large and VDS
is reduced should be minimum.
5. The base doping and the base width are increased
Hence, the correct option is (b)
 [1995]
32. The ‘Pinch-off’ voltage of a JFET is 5.0 volts, its ‘cut-
Solution: (A → 3, B → 1, C → 1)
off’ voltage is
The current gain increases with the base doping base (a) (5.0)1'2 V (b) 2.5 V
width. (c) 5.0 V (d) (5.0)3/2 V [1990]
The collector breakdown voltage increase causes the Solution: (c)
base doping increase and the base width is reduced.
Cut off voltage = pinch - off voltage = 5 V.
The cut off frequency increases causes the base doping
Hence, the correct option is (c)
to increase and the base width is reduced
33. Which of the following effects can be caused by a rise
29. In a JFET
in the temperature
List-I (a) increase in MOSFET current (IDS )
A The pinch-off voltage decreases (b) increase in BJT current (IC )
B. The transconductance increases (c) decrease in MOSFET current (IDS )
C. The transit time of the carriers in the channel is re- (d) decrease in BJT current (IC ) [1990]
duced
Solution: (b) and (c)
List-II
Ic - bIb + (1 + b) ICO
1. The channel doping is reduced
2. The channel length is increased As temperature increases, ICO also increases. So, the
3. The conductivity of the channel is increased current (IC) increases in BJT with rise in temperature.
4. The channel length is reduced Mobility decreases as temperature increases.

5. The GATE area is reduced [1995] T ↑ → m ↓
Chapter 3  Bipolar Junction Transistor  |  4.45

As temperature increases, Ic increases due to increase 36. In MOSFET devices, the n-channel type is better than
in leakage current whereas IDS decreases due to reduc- the P-channel type in the following respects:
tion in mobility of carriers. (a) it has better noise immunity
Hence, the correct option is (b) and (c) (b) it is faster
(c) it is TTL compatible
34. In a MOSFET, the polarity of the inversion layer is the
(d) it has better drive capability [1988]
same as that of the
(a) charge on the GATE-EC-electrode Solution (b)
(b) minority carriers in the drain Due to high mobility of elections n-channel type
(c) majority carriers in the substrate MOSFET is faster.
(d) majority carriers in the source [1989] Hence, the correct option is (b)
Solution (d) 37. The pinch off voltage for an n-channel JFET is 4 V.
In MOSFET, majority carriers are responsible for When VGS = 1 V, the pinch-off occurs for VDS equal to
action. (a) 3 V (b) 5 V
Hence, the correct option is (d) (c) 4 V (d) 1 V
35. In an n-channel JFET, VGS is held constant. VDS is less [1987]
than the breakdown voltage. As VDS is increased Solution (a)
(a) conducting cross-sectional area: of the channel ‘S' Given that,
and the channel current density ’J’ both increase Vp = 4 V
(b) 'S' decreases and ‘J’ decreases
VGS = 1 V
(c) 'S' decreases and ‘J’ increases
(d) 'S' increases and ‘J’ decreases [1988] |VDS| = |Vp| - |VGS|
Solution (c) |VDS| = 4 - 1 = 3V
As VDS is increased, reverse biasing increases. So, Hence, the correct option is (a)
cross-selection area S decreases and hence current den-
sity increases.
Hence, the correct option is (c)
Unit V
analog electronics

Chapter 1: Operational Amplifiers 5.3


Chapter 2: Diodes Applications 5.34
Chapter 3: BJT Analysis 5.48
Chapter 4: FET and MOSFET Analysis 5.79
Chapter 5: Frequency Response of Amplifier 5.100
Chapter 6: Feedback Amplifiers 5.104
Chapter 7: Oscillator Circuits 5.109
Chapter 8: Power Amplifiers 5.113
Chapter 9: Multi-vibrators and 555 Timers 5.115
EXAM ANALYSIS
Exam Year 92 93 94 95 96 97 98 99 00 01 02 03 04 05 06 07 08 09 10 11 12 13 14-1 14-2 14-3 14-4 15 16 17 18 19
Set 1 Set 2 Set 3 Set 1 Set 2 Set 3 Set 1 Set 2
1 Mark Questions - 6 5 10 2 3 13 4 8 3 4 5 4 3 2 2 1 - 3 2 2 2 4 3 3 3 4 3 4 3 3 6 3 3 3 4
2 Mark Questions 6 2 2 - 5 2 - 4 2 5 5 8 5 11 7 11 8 8 3 4 3 6 3 - 2 2 4 3 2 3 3 5 3 4 5 6
5 Mark Questions 1 1 2 2 2 4 4 1 1 2 - - - - - - - - - - - - - - - - - - - - - - - - - -
Total Marks 17 15 19 20 22 27 33 18 17 23 14 21 14 25 16 24 17 16 9 10 8 14 10 3 7 7 12 9 8 9 9 16 9 11 13 16

Chapter-wise marks
distribution
Operational 6 2 1 2 1 3 1 1 6 5 3 6 4 5 2 8 4 2 3 1 2 3 5 1 2 1 8 4 3 3 -
Amplifiers
Diode Applications 2 1 - - 2 - 1 2 - 2 2 3 4 2 6 3 1 6 - 4 1 4 - - 1 1 1 6 1 3 2
BJT Analysis 2 4 4 2 4 2 7 1 5 1 2 4 1 7 6 4 4 2 6 3 3 2 3 1 3 5 5 4 4 - -
FET and MOSFET 2 - 1 1 - - 2 - - - 4 2 - 8 1 8 4 4 - 2 2 2 2 1 - - 6 7 3 2 8
Frequency Response - 1 - 2 3 - 1 4 - 2 - - 2 - - - - - - - - 2 - - - - - - - - -
Feedback Amplifiers - 1 - 1 - 2 1 3 - - 1 2 1 1 1 1 - - - - - 1 - - 1 - - - 1 - -
Oscillator Circuits - - 2 - 2 - - - 1 2 2 2 2 - - - - - - - - - - - - - - 1 - - -
Power Amplifiers - 1 1 2 - - - 1 - - - - - - - - - - - - - - - - - - - - - - -
Multi-vibrators and - - - - - - - - - 1 - 2 - 2 - - 4 2 - - - - - - - - - 1 - - -
555 Timers
Chapter 1
Operational Amplifiers
2. For the operational amplifier circuit shown in the fig-
One-mark Questions ure, the output saturation voltage are ±15 V. The upper
and lower threshold voltages for the circuit are, respec-
1. In the circuit shown below, the op-amp is ideal and tively, [2017]
Zener voltage of the diode is 2.5 volts. At the input, unit
+ −
step voltage is applied, i.e., vIN(t) = u(t) volts. Also, at Vin
Vout
t = 0, the v­ oltage across each of the capacitors is zero. − +
 [2018] 10 KΩ
1 μF

5 KΩ
1 KΩ 1 μF
1V – +
Vin(t) Vout(t) − 3V
+
0V
t=0

The time t, in milliseconds, at which the output voltage (A) +5 V and -5 V (B) +7 V and -3 V
vOUT crosses -10V is (C) +3 V and -7V (D) +3 V and -3 V
(A) 2.5 (B) 5 Solution: 
By KCL
(C) 7.5 (D) 10 15 − VUTP VUTP − 3
  =
Solution:  10 K 5K
1 μF 15 − V UTP = 2VUTP − 6
+ Vc – + 3VUTP = 21
1 KΩ 1 μF  
2.5 V
1V – –
Vin(t) I 0V Vout(t) = –10 V
in + ∴VUTP = 7 V
0V
t=0
by KCL
Input current Iin = 1 mA
−15 − VLTP VLTP − 3
Now the voltage can be calculated as =
10 K 5K
1 −15 − VLTP = 2VLTP − 6
Vc = ∫ idt
C  3VLTP = −9
1 VLTP = −3V .
1× 10 −6 ∫
Vc = 1× 10 −3 t

⇒ Vc = 1000t ∴ Upper threshold voltage = +7 V
1000 t = 7.5 Lower threshold Voltage= -3 V.
t = 7.5 msec Hence, the correct option is (B).
Hence, the correct option is (C) 3. Consider the constant current shown in the figure
below. Let β represent the current gain of the transistor
5.4 | Analog Electronics

+VCC 4. The following signal Vi of peak voltage 8 V is applied


to the non inverting terminal of an ideal opamp. The
R transistor has VBE = 0.7 V, β = 100; VLED = 1.5 V, Vcc =
10 V and –Vcc = –10 V
10 V
Vref –
10 V 100 Ω
+ R2

RL 8 kΩ +VCC LED
R1

– 15 kΩ
+ VB
2 kΩ

Vin –VCC
The load current I0 through RL is [2016]

⎛ β + 1⎞ Vref
(A) I0 = ⎜ Vi
⎝ β ⎟⎠ R
6v
⎛ β ⎞ Vref 4v
I0 = ⎜
(B)
⎝ β + 1⎟⎠ R 2v

⎛ β + 1⎞ Vref –2v
I0 = ⎜
(C)
⎝ β ⎟⎠ 2 R –4v
–6v
⎛ β ⎞ Vref
I0 = ⎜
(D)
⎝ β + 1⎟⎠ 2 R
The number of times the LED glows is _____.
Solution: 
Re-draw the given circuit  [2016]
Solution: 
+
VCC R IE 10 V

Vref 10 V 100 Ω
IB

β 8 kΩ +VCC LED
+ R2
Vx – 15 kΩ
RL I0 = IC
R1 + VB
2 kΩ

Vin –VCC

I E = I B + I C = (1 + β ) ⋅ I B If VB is positive then only the transistor is in ON state



Vref ∴ Vin > Vx : LED glows
= (1 + β ) ⋅ I B
R 10 x 2
∴ Vx = =2V
IC = β ⋅ I B 10 
From the given Vin
I
IB = C
β Vin > 2 V, VB = +Vsat

⎛ β ⎞ Vref Then LED glows


∴ IC = IO = ⎜ ⋅
⎝ 1 + β ⎟⎠ R

Hence, the correct option is (B).
Chapter 1  Operational Amplifiers  |  5.5

Vin Solution:  Here two diodes D1 and D2 are connected


8V LED ON LED ON LED ON back to back. In case when diodes are connected back
to back, the device is prevented since this condition in-
1 troduces the stability avoiding the runoff or saturation
2 3
2V of the device.
Hence, the correct option is (A).
6. In the circuit shown, assume that the op-amp is ideal. If
the gain (vo/vin) is -12, the value of R (in kΩ) is ______
 [2015]
–8V 10 kΩ 10 kΩ

vin 10 kΩ R
From the above data LED glows 3 times. –
Hence, the correct Answer is (3).
v0
+
5. Consider the oscillator circuit shown in the figure. The
function of the network (shown in dotted lines) consist-
ing of the 100 kΩ resistor in series with the two diodes
connected back to back is to  [2016] Solution: 
10 kΩ Va 10 kΩ
InF 158 kΩ

R
Vi/p 10 kΩ
158 kΩ –
InF +VCC 0V VO
+
+
– +
–VCC 22.1 kΩ

D1 Write KCL at Va
Va Va − Vo V
100 kΩ + + a = 0
R 10 k Ω 10 k Ω
D2

10 kΩ ⎡ 1 1⎤ V
⇒ Va ⎢ + ⎥ = o  (a)
⎣ R 5 ⎦ 10
KCL at 0 V
0 − Vi 0 − Va
+ =0
10 k Ω 10 k Ω 
Va = –Vi(b)

From both equations (a) and (b)


10 kΩ
⎛ 5 + R ⎞ Vo
–Vi ⎜ =
⎝ 5 R ⎟⎠ 10
Vout

Vo
but = –12
Vi
(A) Introduce amplitude stabilization by preventing the
op amp from saturating and thus producing sinusoi- (5 + R)10 = 12
dal oscillation of fixed ­amplitude. 5R

(B) Introduce amplitude stabilization by forcing the
opamp to swing between positive and negative ⇒ R = 1 k Ω
saturation and thus producing square wave oscil- Hence, the correct Answer is (1).
lations of fixed amplitude.
7. Assuming that the opamp in the circuit shown below
(C) Introduce frequency stabilization by forcing the
is ideal, the output voltage Vo (in volts) is _________.
circuit to oscillate at a single frequency.
 [2015]
(D) Enable the loop gains to take on a value that pro-
duces square wave oscillations.
5.6 | Analog Electronics

2 kΩ IC = 200 ma + 0.4 mA = 200.4 mA


12 V
1 kΩ PQ1 (max ) = 14 × 200.4 × 10–3 = 2.805 W


Hence, the correct Answer is (2.7 to 2.9).
+ V0
9. In the low-pass filter shown in the figure, for a cut-off
1V
frequency of 5 kHz, the value of R2 (in kW) is _____.
–12 V
 [2014]
R2
Solution:  V– = 0
V+ = 1 V C

V+ > V– 10 nF
1 kΩ
Vi −
so Vo = + Vsat = +12 V V0
R1
Hence, the correct Answer is (11 to 12). +
8. For the voltage regulator circuit shown, the input volt-
age (Vin) is 20 V ± 20% and the regulated output volt-
age (Vout) is 10 V. Assume the opamp to be ideal. For a Solution: 
3.183 kΩ
load RL drawing 200 mA, the maximum power dissipa-
tion in Q1 (in Watts) is _______. [2015] Output to input voltage ratio or gain

Vin Q1 Vout V0 Zf
=
Vi Z1
Vref 4 V + R1 RL
 1 
–  R2 || 
sC 
=−
R2 = 10 kΩ
R1
 R2 
  1
=  sC  ×
1
PQ1 ( max ) = VCE (max ) × I C (max )
Solution:   R2 +  R1
 sC 
VC max = 20 + 20% = 24 V  R2  1
= − ×
 1 + sCR2  R1
VE = 10 V
VCE max = 14 V R2

200.4 C = 10 nF
20 ± 20% Q1 mA 10 V
Vin
0.4 mA 200 mA
1 kΩ
Vref 4V + Vi −
R1 RL R1 V0
– +
4V1 = V1

10 kΩ
1
Cut-off frequency = rad/sec.
CR2
10 × 10 kΩ
⇒ V′ = =4V
10 kΩ  1
2p × 5k =
R1 = 15 k Ω CR2
10 − 4
Current across R1 = = 0.4 mA 1
15  ⇒ 10pk =
10 × 10 −9 R2
Chapter 1  Operational Amplifiers  |  5.7

10 4 (a) A bandpass filter


⇒ R2 = (b) A voltage controlled oscillator
p
(c) An amplitude modulator
= 3.183 kΩ (d) A monostable multivibrator
10. In the differential amplifier shown in the figure, the Solution: (d)
magnitudes of the common-mode and differential- The given circuit represents a mono stable multivibra-
mode gains are Acm and Ad, respectively. If the resist- tor.
ance RE is increased, then [2014]
C2 12 V
Vcc Vi −
Rc Rc Vo
R2 +
Vo −12 V
P−i
+ − −2 V
C1
+
12. In the circuit shown below what is the output voltage
Vi
(Vout) if a silicon transistor Q and an ideal op-amp are
− used? [2013]

RE Io

+15 V Q
−VEE
1kΩ

(a) Acm increases Vout
+ +
(b) common-mode rejection ratio increases 5V

(c) Ad increases
−15 V
(d) common-mode rejection ratio decreases
Solution: (b) − 15 V
(a) (b) − 0.7 V
The magnitudes of the common-mode and + 0.7 V
(c) (d) + 15 V
­differential-mode gains are ACM and Ad, respectively.
Solution: (b)
− RC Since the transistor Q is a silicon transistor
ACM =
2 Re
Vout + VBE = 0
If Re is increased, then ACM decreases Vout = −VBE
1 = −0.7 V.
Ad =
g m Rc Hence, the correct option is (b).
2
If Re is increased, then Ad does not get affected 13. The circuit below implements a filter between the input
current ii and output voltage v0. Assume that the op-amp
Ad 1  2R 
CMRR = = g m Rc  − e  is ideal. The filter implemented is a [2011]
ACM 2  RC  L1
= − g m Re
R1
If Re is increased, then CMRR increased
Hence, the correct option is (b). −
11. The circuit shown represents [2014] + +
Vo
C2 + 12 V −
Vi −
Vo
R2 +
(a) Low pass filter
−12 V
R1 (b) Band pass filter
−2 V (c) Band stop filter
C1 (d) High pass filter
5.8 | Analog Electronics

Solution: (d)
30 kΩ
When w = 0 10 kΩ
inductor acts as a SC −
⇒ V0 = 0 +
and when w = ∞,
Ideal operational amplifier
inductor acts as an OC
⇒ V0 = i1R1
So, it acts as a high pass filter
Hence, the correct option is (d). R1
14. Assuming the op-amp to be ideal, the voltage gain of
the amplifier shown below is[2010] 30
(a) kΩ (b) 10 kW
4
R1
− (c) 40 kW (d) infinite
V0
+ Solution: (b)
R2
30 k
Vi +

R3 10 k

+

R R
− 2 (b)
(a) − 3
R1 R1
Ri
R R R + R3
− 2 3 (d)
(c) − 2 Connect a Vs voltage source across inverting terminal of
R1 R1
op-amp
Vs
Solution: (a) Ii =
Applying nodal analysis at non-inverting input 10 k

30 k
0 − Vi 0 − V0
+ =0
R1 R2
Ii 10 k
Vs −
R1 0V
Vi − +
V0
+
R2

Ri
R3
Vs
I/P resistance, Ri =
Ii

Vi V0 = 10 kW
⇒ + =0 Hence, the correct option is (b).
R1 R2
1 6. An ideal op-amp is an ideal [2004]
R2 (a) Voltage controlled current source
⇒ V0 = −Vi
R1 (b) Voltage controlled voltage source
(c) Current controlled current source
V0 − R2 (d) Current controlled voltage source
⇒ voltage gain =
Vi R1
Solution: (b)
Hence, the correct option is (a). An ideal op-amp is an ideal voltage controlled voltage
1 5. The input resistance Ri of the amplifier shown in the source.
figure is [2005]
Chapter 1  Operational Amplifiers  |  5.9

17. The circuit in the figure is [2004] Input −


Output
Vref = 2 V +

R R Vout Vi = 4 sin wt
Vin +
If Vi = 2 V
2 = 4 sin wt
1
⇒ sin wt =
2
(a) low-pass filter (b) high-pass filter
(c) band-pass filter (d) band-reject filter p . 5p
wt =
Solution: (a) 6 6
At w = ∞, tON 5p / 6 − p / 6 4p / 6
duty cycle = = =
V0 T 2p 2p
=0
Vin
1
=
3
− Hence, the correct option is (b).
R R Vout
Vin +
19. If the differential voltage gain and the common mode
voltage gain of a differential amplifier are 48 dB and 2
dB, respectively, then its common mode rejection ratio
is [2003]
V0 = 0 (a) 23 dB (b) 25 dB
At w = 0, (c) 46 dB (d) 50 dB
V0 Solution: (c)
=1 Common mode rejection ratio is given by
Vin
V0 = Vin Ad
CMRR =
So, the given circuit is a low-pass filter. Ac
Hence, the correct option is (a).
Ad = 48 dB
18. If the input to the ideal comparator shown in the figure Ac = 2 dB
is a sinusoidal signal of 8V (peak to peak) without any (CMRR)dB = 20 log Ad – 20 log Ac
DC component, then the output of the comparator has a =48 − 2
duty cycle of [2003] =46 dB
1 1 Hence, the correct option is (c).
(a) (b)
2 3 20. A 741-type op-amp has a gain-bandwidth product of
1 MHz. A non-inverting amplifier using this op-amp
1 1
(c) (d) and having a voltage gain of 20 dB will exhibit a −3 dB
6 12 bandwidth of [2002]
Input − (a) 50 kHz (b) 100 kHz
Output
Vref = 2 V + 1000 1000
(c) kHz (d) kHz
7.07 7.07
Solution: (b)
V
Solution: (b)
4V Gain × BW = 1 × 106
2V 1 × 106 106
BW = =
ωt Gain 10
π /6 5π/6 2π
= 105
−4 V Hence, the correct option is (b).
5.10 | Analog Electronics

21. The ideal OP AMP has the following characteristics: C


[2001]
(a) Ri = ∞, A = ∞, R0 = 0 C
(b) Ri = 0, A = ∞, R0 = 0 −
V1 sin ωt V0
(c) Ri = ∞, A = ∞, R0 = ∞ +
V2 sin ωt
(d) Ri = 0, A = ∞, R0 = ∞ C

Solution: (a) C C


The ideal op-amp has the following characteristics: V0 = V1 sin wt   − V2 sin wt  
C C
In put impedance, Ri = ∞
= (V1 + V2 ) sin wt
Voltage gain, A = ∞
Output impedance, R0 = 0 If Vi is a triangular wave, then V0 will be a square wave.
Hence, the correct option is (a). Hence, the correct option is (d).
22. In the circuit of the figure, Vo is [2000] 24. Assume that the op-amp or the figure is ideal. If Vi is a
+15 V triangular wave, then V0 will be [2000]

Vo
+1 V + R
C
R Vi −
−15 V
V0
+
R

− 1 V
(a) (b) 2 V (a) square wave
+ 1 V
(c) (d) + 15 V (b) triangular wave
(c) parabolic wave
Solution: (d) (d) sine wave
Since I/P applied at the terminal O/P is +ve, and since
feedback is positive, O/P voltage gets saturated. Solution: (a)
Op amp gain is given by
+15 V


 −R
V0 = Vi 
Vo  X c 
+1 V +
R −Vi R
−15 V =
Xc
R
dVi
V0 = −RC
∴V0 = + 15 V dt
Hence, the correct option is (d).
R
23. If the op-amp in the figure is ideal, then Vo is [2000]

C C
Vi −
V0
sin ωt C +
V1
V0
V2 +
sin ωt C Hence, the correct option is (a).

25. The most commonly used amplifier in sample and hold


(a) Zero (b) (V1− V2) sin wt circuits is [2000]
(c) − (V1 + V2) sin wt (d) (V1 + V2) sin wt (a) a unity gain inverting amplifier
(b) a unity gain non-inverting amplifier
Solution: (d) (c) an inverting amplifier with a gain of 10
Applying super position theorem, (d) an inverting amplifier with a gain of 100
Chapter 1  Operational Amplifiers  |  5.11

Solution: (b) (a) 6 dB (b) 8 dB


The most commonly used amplifier in sample and hold (c) 12 dB (d) None of the above
circuits is a unity gain non-inverting a­ mplifier. Solution: (a)
Each amplifier has a noise figure of 3 dB

∴the noise figure of the cascade combination = 3 dB +
V0 3 dB
Vi + = 6 dB
C A A
Control
gate 50 50
Hence, the correct option is (b).
26. The first dominant pole encountered in the frequency
response of a compensated op-amp is approximately at
Hence, the correct option is (a).
 [1999]
(a) 5 Hz (b) 10 kHz 29. The circuit shown in the figure is that of[1996]
(c) 1 MHz (d) 100 MHz Vin −
Vo
Solution: (a) +
 The first dominated pole encountered in the fre-
quency response of a compensated op-amp is R1
approximately at 5 Hz R2
Hence, the correct option is (a).
27. One input terminal of high gain comparator circuit is
connected to ground and a sinusoidal voltage is applied (a) a non-inverting amplifiers
to the other input. The output of comparator will be (b) an inverting amplifier
 [1998] (c) an oscillator
(a) a sinusoid (d) a Schmitt trigger
(b) a full rectified sinusoid Solution: (d)
(c) a half rectified sinusoid The given circuit has a positive feedback, so it has a
(d) a square wave very high O/P gain.
Solution: (d) Any periodic waveform applied to I/P results in a square
One I/P terminal of high gain comparator circuit is wave so wave. So, this circuit is a Schmitt trigger.
connected to ground and a sinusoid voltage is applied Vin −
to the other I/P. The O/P of a comparator will be a Vo
square wave due to the fact that Op. Amp has a very +
high gain, i.e. O/P voltage gets saturated.
Hence, the correct option is (d). R1
R2
28. An amplifier A has 6 dB gain and 50 Q input and output
impedances. The noise figure of this amplifier as shown
in the figure is 3 dB. A cascade of two such amplifiers Hence, the correct option is (d).
as in the figure will have a noise figure of[1997]
30. In the given circuit figure, if the voltage inputs V− and
A V+ are to be amplified by the same amplification factor,
the value of ‘R’ should be
50 Ω 50 Ω 22 KΩ

10 KΩ
V− −
A A V0
V+ +
15 KΩ Va
50 Ω 50 Ω Ideal op-Amp
R=?

[1995]
5.12 | Analog Electronics

Solution:  33 kW SR = VP × 2pfmax
Gain at inverting terminal SR
fmax =

Rf 2p VP
A = −
R1
12 × 106
= = 159 kHz
−22 K 2p × 12
A− = = − 2.2
10 k
32. The frequency compensation is used in op-amps to
22 K
increase its_____. [1994 ]
10 K
Solution:  Stability
V− − The frequency compensation used in Op Amp is used
Va V0 to increase the stability of op-amps.
V+ +
15 K
Two-marks Questions
R=?
1. The amplifier circuit shown in the figure is implemented
using a compensated operational amplifier (op-amp),
and has an open-loop voltage gain A0 = 105 V/V and an
A− = 2.2 open-loop cut-off frequency, fc = 8 Hz. The voltage gain
of the amplifier at 15 kHz, in V/V, is ________.[2017]
 R  R2 = 79 KΩ
Va = V + 
 R + 15 


Gain at non-inverting terminal
R1 = 1 KΩ
Rf −
A+ = 1 +
R1 + VO

V0 Rf Vo
=1+ =
Va R1  R 
V+
Vi ∼
 R + 15 
V0  22   R 
= 1 +  ×  
V+  10   R + 15  R1 1
β=
Solution:  =
R1 + R2 50
 R 
A+ − 3.2 
 R + 15  AoL 1
AcL = =
3.2 R 1 + AoL β 50
= 2.2
R + 15 Where AcL: closed loop gain
3.2R = 2 − 2R + 33 AoL: open loop gain
R = 33 kW
105
31. An op-amp is used as a zero-crossing detector. If ∴ AcL =  80
1
maximum output available from the op-amp is ± 12 1 + (105 )
Vp−p and the slew rate of the op-amp is 12 V/m sec 80
then the maximum frequency of the input signal that  = 10,008 Hz
can be applied without causing a reduction in the P−P 79 KΩ
output is [1995]
Solution: 
159 kHz
1 KΩ
VPP = ± 12 V −
SR = 12V/MSec VO
+
Let V0 = VP sin wt
dV0 ∼
SR = = V pw cosw t = V pw max
dt max
max Vin


Chapter 1  Operational Amplifiers  |  5.13

AcL At inverting terminals


Gain at f = 15 KHz = 15000 Hz is Af =
2 Vx − V2 Vx − V4 V − VN Vx − V0
 f  + + ..... + x + =0
1+   10 k 10 k 10 k 10 k
 f cL 
80 ⎡ N⎤
=  44.4 ∴ V2 + V4 + … + VN =Vx ⎢1 + ⎥ − V0 (ii)
2 ⎣ 2⎦
 15000 
1+   But form (i) and (ii) we get
 10008 
V0 = {V1 + V3 + ……+ VN – 1} –
Hence, the correct answer is (43.3 to 45.3).
2. An ideal opamp has voltage sources V1, V3, V5, VN – 1 {V2 + V4 + ….+ VN}
connected to the non inverting input and V2, V4, V6, VN
connected to the inverting input as shown in the fig- 1
Given; V j = ; j → odd
ure below (+Vcc = 15 volt, –Vcc = –15 volt) the volt- j 
1 1 −1 1 −1 −1
ages V1, V2, V3, V4, V5, V6, are 1, – , , – , , Vj = ; j → even
2 3 4 5 6 j
volt respectively. As N approaches infinitely, the output 
voltage (in volt) is _____  [2016] ⎧ 1 1 1 1 1⎫
∴ V0 = ⎨1 + + + + + ....... + ⎬
⎩ 2 3 4 5 ∞⎭ 
10 kΩ
V2
∴ V0 = +Vsat = 15 V
10 kΩ 10 kΩ
V4
Hence, the correct Answer is (14 V).
+VCC
VN – 3. A p – I – n photodiode of responsivity 0.8 A/W is
10 kΩ V0
connected to the inverting input of an ideal opamp as
+
1 kΩ –VCC shown in the figure +Vcc = 15 V, –Vcc = –15 V, Load
V1
resistor RL = 10 k Ω. If 10 µW of power is incident
1 kΩ on the photodiode, then the value of the photocurrent
V3
1 kΩ (in µA) through the load is[2016]
VN–1
1 kΩ 1 MΩ

V2
+VCC
10 kΩ
V4 –
10 kΩ V0
VN – +
10 kΩ Vx 1 MΩ
–VCC
+ V0 10 kΩ

1 kΩ +VCC
V1 Vx
1 kΩ Solution:  Responsivity of photodiode R = 0.8 A/W
V3
1 kΩ Photon current Ip = ?
1 kΩ
VN–1
We know that
Ip
Solution:  On applying virtual GND concept we have R =
po
V+ = V– = Vx 
At non-inverting terminal (applying KCL) Ip = R ⋅ Po = 0.8 × 10 μA

Vx − V1 Vx − V3 V − VN −1 Vx Vo − 0
+ + ..... + x + =0 = Ip
1k 1k 1k 1k  1M 
N V0 = 1M × 8 μ = 8 V
V1 + V3 + V5 + … + VN – 1 = Vx + ⋅Vx
2 
The value of the photocurrent (in µA) through the load
⎡ N⎤
⎢1 + 2 ⎥ ⋅Vx (i)
= is
⎣ ⎦
5.14 | Analog Electronics

V0 8 VN = 10 – Vc
∴ IL = = 4 = 800 μ A 
10 k 10 VN = –10 + 20. e–t.
Hence, the correct Answer is (800 µA). Apply Virtual GND Concept
4. In the op-amp circuit shown, the Zener diodes Z1 and Z2 VN = Vx
clamp the output voltage V0 to +5 V or –5 V. The switch
S is initially closed and is opened at time t = 0. –10 + 20 ⋅ e–t = –1
+10 V 20 ⋅ e–t = 9
S 100 μF +10 V t = 0.798 sec.
t=0 – 470 Ω Hence, the correct Answer is (0.798 sec).
V0
10 kΩ +
5. An op-amp has a finite open loop voltage gain of 100.
–10 V z1 Its input offset voltage Vios (= +5 mV) is modelled as
–10 V shown in the circuit below. The amplifier is ideal in all
4 kΩ z2
1 kΩ other respects. Vinput is 25 mV.
0V 0V 1 kΩ 15 kΩ
The time t = t1 (in seconds) at which V0 changes state is
_____.[2016]

Solution:  From the given data switch is closed ­initially
A0 = 100
and opened at t = 0.
+
for t < 0 :
+
at t = 0 − : – Vios = 5 mV
10 V +
– 470 Ω – Vinput
V0 = –5 V
+
The output voltage (in millivolts) is ____.[2016]
Vx Solution: 
4 kΩ 1 kΩ 15 kΩ

1 kΩ


A V0
−5 × 1
Vx = = –1 V +
5
+
Vc (0–) = 0 V – Vin = 25 mV
+
For t > 0: – Vios = 5 mV
100 μF
+ A = 100,
+ –
VC 10 kΩ we know for low gain op-amps
+
– VN (since it is a non-inverting op-amp, therefore the gain
10 V 10 V
– + can be given as)

V0 =
{1 + R /R } × V
f 1

t → ∞ 1+
{1 + R /R1}
f
i

Vc (∞) = 20 V A 
Vc(t) = 20 + [0 – 20] ⋅ e–t/τ 16
= × 30 mV
τ = R ⋅ C = 10 kΩ × 100 µF 16
1+
τ = 1 sec 100 
∴ Vc(t) = 20 – 20. e–t V0 = 413.8 mV
Hence, the correct Answer is (413.8 mV).
Chapter 1  Operational Amplifiers  |  5.15

6. For the circuit shown in the figure, R1 = R2 = R3 = 1 Ω, 1


L = 1 µH and C = 1 µF. If the input Vin = cos(106t), then Cut-off frequency f =
Solution: 
2π RC
the overall voltage gain (Vout/Vin) of the circuit is[2016]
1
=
L R1 R3
2 × π × 10 × 10 3
× 0.1 × 10 −6 
– – = 159.15 Hz
Vout
+ R2 C + Hence, the correct Answer is (159 to 160).
Vin 8. In the circuit shown, assume that the opamp is ideal.
The bridge output voltage V0 (in mV) for δ = 0.05 is
______[2015]

Solution:  Resistance R1 = R2 = R3 = 1 Ω , 100 Ω +


1V
Inductance L =1μ H 250 (1 + δ ) Ω 250 (1 – δ ) Ω –
Capacitance C = 1μ F – +
The input voltage is Vin = cos106t V V0

Angular frequency ω = 106 rad/sec 250 (1 – δ ) Ω 250 (1 + δ ) Ω


R1 R3 100 Ω
50 Ω
SL
– 1 –
V0 Solution: 
+ V R +
1 100 Ω + 1V
sC
Vin –
262.5 Ω 237.5 Ω

1 – + 1
2 V0 2
After considering the virtual ground effect, we get 237.5 Ω 262.5 Ω
(let V is denoted by Vx) then
1V
100 k I
Vx ⎛ R⎞ 50 Ω
= 1+ 1
Vin ⎜⎝ sL ⎟⎠ I
Vo − R3
= From the given circuit V+ = V– = 1 V
Vx ⎛ 1 ⎞
⎜⎝ R2 + ⎟
sC ⎠ 1
Current through 50 Ω is = Amp = I
− R3 ⎛ R⎞ 50
Vo = × ⎜1 + 1 ⎟ × Vin 1
⎛ 1 ⎞ ⎝ sL ⎠ Vo = [262.5 – 237.5]
⎜⎝ R2 + ⎟ 100
sC ⎠
Sub given values, we get 25
= = 250 mV
100
Vo
= −1 Hence, the correct Answer is (249 to 251).
Vin
9. The circuit shown in the figure has an ideal opamp. The
Hence, the correct Answer is (-1).
oscillation frequency and the condition to sustain the
7. In the circuit shown using an ideal opamp, the 3-dB oscillations, respectively are[2015]
cut-off frequency (in Hz) is _______.  [2015]
R1
10 kΩ 10 kΩ
Vi +
R2
V0 –
0.1 μF –
Vout
10 kΩ 10 kΩ +

2C C 2R
R
5.16 | Analog Electronics

1 But Rf = R1 and R3 = R2


(A) and R1 = R2
CR
∴ R1 = 4R2
1
(B) and R1 = 4R2
CR Hence, the correct option is (D).
1 1 0. In the circuit shown, V0 = V0A for switch SW in position
(C) and R1 = R2
2CR
A and V0 = V0B for SW in position B. Assume that the
1
(D) and R1 = 4R2 V0 B
2CR opamp is ideal. The value of is ________. [2015]
V0 A
Solution:  The given circuit is Wein Bridge Oscillator
1 kΩ
Rf

R3 5V 1 kΩ
– –
Vout V0
+ C B A 1 kΩ
1
Rf +
SW 1 kΩ
C2 R1
R2
1 kΩ
1V

Rf
The gain of the op–amp is A = I + and feedback
R3 ⎛ −1 ⎞ ⎛ −1 ⎞
Vf Solution:  VOB = 5 ⎜ ⎟ + 1⎜ ⎟ = –6 V
factor B = ⎝ 1 ⎠ ⎝ 1 ⎠
Vo
⎛ 1 × 1⎞ ⎛ 1⎞ ⎛ 1 ⎞
Let Z1 = R1 + 1/SC1 and Z2 = R2 || 1/SC2 VOA = ⎜ ⎜1 + ⎟ 5 ⎜ ⎟ = − 4 V
⎝ 1 + 1⎟⎠ ⎝ 1⎠ ⎝ 1 ⎠ 
Z 2 × V0 Z2
Vf = ∵B= VOB −6
Z1 + Z 2 Z1 + Z 2 = = 1.5
 VOA −4

Sub all values mB we get finally Hence, the correct Answer is (1.5).
jω R 2 C1 1 1. In the bistable circuit shown, the ideal opamp has satu-
B=
1 + jω ( R1C1 + R 2 C2 + R 2 C1 ) − ω 2 R1 R2C1C2 ration levels of ± 5 V. The value of R1 (in k Ω) that gives
a hysteresis width of 500 mV is ____.[2015]
But B is real quantity so equate
R2 = 20 kΩ
1 – ω2 R1R2C1C2 = 0
Thus the frequency of oscillation, R1
+
1 1
ω0 = ω0 = Vout
R1 R2C1C2 2RC vin + –

From the given data
R1 = 2R, R2 = R
C1 = C, C2 = 2C Solution:  Hysteresis width = VTH – VTL
⎛ R ⎞ ⎛ R ⎞
And B=
R2C1 500 mV = 5 ⎜ 1 ⎟ − ( −5) ⎜ 1 ⎟
⎝ 20 k Ω ⎠ ⎝ 20 k Ω ⎠
R1C1 + R2C2 + R2C1 

Substitute all values R1 = 1 k Ω
RC 1 Hence, the correct Answer is (1).
B= =
2 RC + 2 RC + RC 5 
For sustained oscillation AB = 1 12. In the voltage regulator circuit shown in the figure, the
op-amp is ideal. The BJT has VBE = 0.7 V and b = 100,
1 Rf and the Zener voltage is 4.7 V. For a regulated output of
A = ∴ 1+ =5
B    R3
 9 V, the value of R (in W) is______. [2014]
Chapter 1  Operational Amplifiers  |  5.17

V1 = 12 V V0 = 12 V − I2(R1 + R2)
(a)
(b)
I2R2
1 kΩ 1 kΩ (c)
I1R2
+
− − I1 ( R1 + R2 )
(d)

Solution: (c)
V2 = 47 V R
Vout = I1R2
Current does not enter the op-amp.
Solution: R2

Given
VBE = 0.7 V R1

B = 100 Vout
I1
Vz = 4.7 V +
I2
Vi = 12 V ie V0 = 9 V

1k
1k
Hence, the correct option is (c).
+
1 4. Assuming that the op-amp in the circuit shown is ideal,

Vo is given by [2014]
V
VZ = 4.7 V 3R
R

R
V1
V2 Vo
Vi + 1ki − 4.7 = 0 +
12 + i − 4.7 = 0
⇒ i = 7.3 mA
2R R
Since B is large enough
I ≅ ie = 7.3 mA
V = 4.7 V (virtual ground)
V − ieR = 0 5 5
(a) V1 − 3V2 (b)
2V1 − V2
V 2 2
⇒R=
ie 3 7 11
4.7 − Vi + V2 (d)
(c) −3V1 + V2
= 2 2 2
7.3 Solution: (d)
⇒R = 0.64 kW Applying nodal analysis,
13. In the circuit shown, the op-amp has finite input imped- V2 − V1 V2 V2 − V0
ance, infinite voltage gain and zero input–offset volt- + + =0
R 2R 3R
age. The-output voltage Vout is [2014]
R2 3R

R1 I1 R
− V1 −
Vout Vo
+ V2 +
I2 2R R
5.18 | Analog Electronics

6V2− 6V1 + 3V2 + 2V2− 2V0 = 0 Solution: (b)


Non-inverting terminal gain
⇒ 11V2− 6V1− 2V0 = 0
 
11  R2 
⇒ V0 = −3V1 + V2 V0 ( s) = −  V ( s)
2 1 i
 R1 + 
Hence, the correct option is (d).  sc 
15. In the circuit shown below, the op-amps are ideal. Then
R2
Vout in volts is [2013]
+5 V
C R1
1 kΩ 1 kΩ
−2 V Vi −
Vo
+15 V +
+15 V

+ −5 V
+ Vout

1 kΩ −15 V
−15 V
− sCR2
+1 V V0 ( s) = Vi ( s)
1 kΩ 1 + sCR1
1 kΩ

1
thus cut-off frequency is
(a) 4 (b) 6 (c) 8 (d) 10 R1C

Solution: (c) at w  = 0
V0 = 0
V1 = 1(1 + 1) −2(−1) and, at w  = ∞
= 2+2
= 4V R2
V0 = −Vi
Vout = 2V1 R1
= 2×4 ∴it acts as a high-pass filter.
= 8V Hence, the correct option is (b).
Hence, the correct option is (c).
17. The transfer characteristic for the precision rectifier
16. The circuit shown is a [2012] circuit shown below is (assume ideal op-amp and prac-
R2 tical diodes) [2010]

+5 V 20 V
C R1 +
+ − Output R
Input −
− + D2
4R

−5 V
Vi −
R Vo
+
D1
1
(a) low pass filter with    f 3dB = rad / s
( R1 + R2 )C
1
(b) high pass filter with f 3dB = rad / s (a) V0
R1C
10
1
(c) low pass filter with    f 3dB = rad / s
R1C
1 Vi
(d) high pass filter with f 3dB = rad / s −10 −5 0
( R1 + R2 ) C
Chapter 1  Operational Amplifiers  |  5.19

(b) the feedback in the circuit is positive or negative and


V0
determine the voltage V at the output of the op-amp.
 [2009]
10 V
5

Vi 5 kΩ
−10 −5 0

(c) V0
V +

5
5V
1.4 kΩ
Vi
0 +5

(d) V0

10 (a) Positive feedback, V = 10 V


(b) Positive feedback, V = 0 V
(c) Negative feedback, V = 5 V
Vi (d) Negative feedback, V = 2 V
0 +5
Solution: (d)
Solution: (b) V2 = 5 V (virtual grounds)
At Vi = −10 V 10 − 5I − V2 = 0
V0 − 0 0 − 20 0( −10) ⇒ 5 = 5I
= + ⇒ I = 1 mA
R 4R R
20 V R 10 V

D2 I
4R
5 kΩ
Vi −
R Vo
+ V2
D1
V +

V0 = −5 + 10
= 5V +
5V
At Vi = −5 V 1.4 kΩ V3

V0 − 0 0 − 20 0( −10)
= +
R 4R R
V0 = –5 + 5
= 0V ⇒ V −VBE −V3 = 0
For Vi > –5 V, both diodes are conducting. So, V0 = 0 V ⇒ V = VBE + V3
∴transfer characteristics is = 0.6 + 1.4k × 1m
V0 = 0.6 + 1.4
⇒V=2V
Hence, the correct option is (d).
5
19. Consider the following circuit using an ideal op-amp.
Vi
−10 −5 0 The I–V characteristics of the diode is described by the
Hence, the correct option is (b).  V 
1 8. In the circuit shown below, the op-amp is ideal, the relation I = I 0  eVT − 1  [2008]
 
transistor has VBE = 0 6 V and b  = 150. Decide whether
5.20 | Analog Electronics

where VT = 25 mV, I0 = 1 μA and V is the voltage Solution: (b)


across the diode (taken as positive for forward bias). At w = 0
D 4 kΩ
C

Vi = −1 V

100 kΩ V0
+ R2

Vi −
R1 L V0
+
For an input voltage Vi = −1V, the output voltage V0 is
(a) 0 V (b) 0.1 V
(c) 0.7 V (d) 1.1 V
R2
Solution: (b) V0 = −Vi
R
   1
1 = 10 (eV/Vt−1) At w = ∞
V0 = 0
0 − ( −1)
= ∴the circuit shown represents a low-pass filter.
100 k
Hence, the correct option is (b).
D 4k 2 1. For the op-amp circuit shown in the figure, V0 is
[2007]
Vi = −1 V 2 kΩ

100 kΩ V0
+ 1 kΩ

Vi V0
+
 V  1 1 kΩ
⇒ 10 −6 e −3
− 1 = 5
 25 × 10  10 1 kΩ

⇒ V = 0.06 V
⇒V = 0.06 V
V −V 1 (a) −2 V (b) −1 V (c) −0.5 V (d) 0.5 V
⇒ 0 =
4k 100 k Solution: (c)
Nodal analysis at non-inverting terminal
⇒V0 = 0.1 V
V −1 V
Hence, the correct option is (b). + =0
1 1
2 0. The op-amp circuit shown below represents a 2V = 1
[2008]
2k
C
1k

1V V V0
+
R2
1k
Vi −
R1 L V0 1k
+

V = 0.5 V
(a) high-pass filter
Nodal analysis at inverting terminal
(b) low-pass filter
V − 1 V − V0
(c) band-pass filter + =0
(d) band-reject filter 1 2
Chapter 1  Operational Amplifiers  |  5.21

2V − 2 + V – V0 = 0 ln 2 = (V01−V02)/ VT
3V = 2 + V0 ⇒V01−V02 = VT ln2
⇒ 3 × 0.5 = 2 + V0 Hence, the correct option is (b).
⇒ 1.5 = 2 + V0 Linked Answer Questions 12 and 13.
⇒ V0 = −0.5 V Consider the op-amp circuit shown in the figure.
Hence, the correct option is (c).
R1
22. In the op-amp circuit shown assume that the diode cur-
rent follows the equation I = Isexp(V/VT). For Vi = 2
R1
V, V0 = V01, and for Vi = 4 V, V0 = V02. The relationship
between V01 and V02 is [2007] Vi V0
+
R
C
Vi −
2 kΩ V0
+

23. The transfer function V0(s)/Vi (s) is [2007]


1 − sRC 1 + sRC
V02 = 2V01 (b)
(a) V02 = e2V01 (a) (b)
1 + sRC 1 − sRC
V02 = V01ln 2
(c) (d) V01 − V02 = VT ln 2
1 1
Solution: (d) (c) (d)
1− sRC 1+ sRC
For Vi = 2 V
Solution: (a)
V0 = −VD
Nodal analysis at non-inverting input
 i
V0 = −VT ln   V − Vi
 1s  + VsC = 0
R
VD
+ −
1  V
V  + sC  = i
R  R
Vi − R1
2k i V0
+
R1
Vi −
V V0
Vi +
2 = 2i
R
⇒ i = 1 mA C
 1m 
⇒ V0 = −VT ln   = V01 (1)
 1s 
 i ⇒V = Vi/(1 + sCR) (1)
 V0 = −VD = −VT ln  
 1s 
4  = 2i V − Vi V − V0
+ =0
i  = 2 mA R1 R1

 2m 
 V0 = −VT ln  = V02 (2) ⇒ 2V = Vi + Vo
 1s 
1m 2Vi
= e −V01 /VT ⇒ = Vi + V0
1s 1 + sCR

2m  2 
⇒ − 1 V = V0
and
1s
= e −V02 /VT  1 + sCR  i
taking the ratio,  2 − 1 − sCR 
⇒ V = V0
1 (V − V01 )
= e 02  1 + sCR  i
2 VT
5.22 | Analog Electronics

t=0
 1 − sCR 
⇒ V = V0 C = 1 µF
 1 + sCR  i S

V0 1 − sCR 1 kΩ
⇒ = −
Vi 1 + sCR P
+
Hence, the correct option is (a).
10 V
24. If Vi = V1 sin (wt) and Vo = V2 sin (wt + f), then the
minimum and maximum values of f (in radians) are,
respectively [2007] Applying kCL at node P
p p p
− and
(a) (b) 0 and 10 dV 10 −6 × VC
2 2 2 =C C =
1k dt 1 × 10 −3
p ⇒ VC = 10 V
−p and 0
(c) (d) − and 0
2 Hence, the correct option is (d).
Solution: (c)
26. The voltage e0 indicated in the figure has been meas-
Vi = V1 sin wt ured by an ideal voltmeter. Which of the following can
V0 = V2 sin (wt+ f) be calculated? [2005]
1 MΩ
V0 ( s) 1 − sCR
= −
Vi ( s) 1 + sCR
e0
+
V0 1 − sCR
∠ ( s) = ∠ 1 MΩ
VI 1 + sCR

q = −tan−1wCR –tan−1wCR
q = −2tan−1wCR (a) Bias current of the inverting input only
Minimum value of q = −p (at w → ∞). (b) Bias current of the inverting and non-inverting in-
Maximum value of q = 0 (at w = 0). puts only
Hence, the correct option is (c). (c) Input offset current only
(d) Both the bias currents and the input offset current
25. For the circuit shown in the following figure, the capac- Solution: (c)
itor C is initially uncharged. At t = 0, the switch S is
closed. The voltage VC across the capacitor at t = 1 mil- V1 = −IB1 × 1M
lisecond is______. V2 = V1 = −IB1× 1M (due to virtual ground)
C = 1 µF Drop in feedback register 1M = IR2 × 1M
S
e0 = V2 + IB2 × 1M
− +
VO e0 = −IB1× 1M + IB2 × 1M
− e0 = (IB2 − IB1) × 1M
1 kΩ
+ where (IB2 − IB1) is offset current.
Hence, the correct option is (c).
10 V
27. The op-amp circuit shown in the figure is a filter. The
type of filter and its cut-off frequency are respectively
 [2005]
In the figure shown above, the op-amp is supplied with
+15 V.[2006] 10 kΩ
10 kΩ
(a) 0 V (b) 6.3 V

(c) 9.45 V (d) 10 V +
Vi
1 µF
Solution: (d) 1 kΩ
Capacitor is initially uncharged. At t = 0, switch ‘S’ is
closed.
Chapter 1  Operational Amplifiers  |  5.23

(a) high pass, 1000 rad/sec. R1


(b) low pass, 1000 rad/sec.
R1
(c) high pass, 10000 rad/sec. Vs −
(d) low pass, 10000 rad/sec. V V0
+
Solution: (c) R2
Since O/P is taken across 10 KW, it is a high filter
R2
iL RL
10 kΩ

10 kΩ Nodal analysis at non-inverting terminal



V0
+ V V V − V0
+ + =0
Vi 1 µF R2 RL R2
1 kΩ
 2 1  V0
V + = (2)
 R2 RL  R2
at w = 0
From equations (1) and (2)
V0 = 0
at w = ∞  2 1  2V − VS
V + =
V0 = 2Vi  R2 RL  R2
I/P is at non-inventing point
2V V 2V VS
1 + = −
so, frequency = R2 RL R2 R2
RC
V −V
1 ⇒ = S (3)
= RL R2
1 × 10 × 1 × 10 −6
3

V = iL RL
= 1000 rad/sec
Hence, the correct option is (c). V −V
⇒ iL = = s
2 8. In the op-amp circuit given in the figure, the load cur- RL R2
rent iL is [2004] Hence, the correct option is (a).
R1 R1 2 9. If the op-amp in the figure is ideal, the output voltage
Vs Vout will be equal to [2003]

− 5 kΩ
1 kΩ
+ 2V −
R2 Vout
3V +
R2 1 kΩ
RL 8 kΩ
iL

V Vs (a) 1 V (b) 6 V (c) 14 V (d) 17 V


− s (b)
(a)
R2 R2
Solution: (b)
V Vs Nodal analysis at non-inverting terminal of op-amp
− s (d)
(c)
RL R1 V −3 V
+ =0
Solution: (a) 1 8
Nodal analysis at inverting input 8V – 24 + V = 0
V − VS V − V0 9V = 24
+ =0 24
R1 R1 V =
9
⇒ 2V = VS + V0(1)
5.24 | Analog Electronics

5k 31. An amplifier using an op-amp with a slew-rate SR =


1 V/μsec has a gain of 40 dB. If this amplifier has
1k
2V − to faithfully amplify sinusoidal signals from dc to
V
V
Vout 20 kHz without introducing any slew-rate induced
3V +
distortion, then the input signal level must not
1k
8k exceed______. [2002]
(a) 795 mV (b) 395 mV
(c) 79.5 mV (d) 39.5 mV
Nodal analysis at non-inverting terminal of op-amp
Solution: (c)
V − 2 V − Vout
+ =0 Slew rate = A. 2pfVm
1 5
V = AVmsin wt
5V – 10 + V − Vout = 0
6V = 10 + Vout dV
SR = = AVm2pfm
dt max
24 = 10 + V
6× out
9
SR 1 × 106  20 log A = 40 
16 = 10 + Vout Vm = =  A = 100 
A.2pf m 100 2p × 20 × 103  
Vout = 6V = 79.5 mV
Hence, the correct option is (b). Hence, the correct option is (c).
30. Three identical amplifiers with each one having a volt- 32. The inverting op-amp shown in the figure has an open-
age gain of 50, input resistance of 1 kW and output loop gain of 100. The closed-loop gain V0/Vs is
resistance of 250 W are cascaded. The open circuit volt-
[2001]
age gain of the combined amplifier is 2003]
(a) 49 dB (b) 51 dB R2 = 10 kΩ
(c) 98 dB (d) 102 dB
R1 = 1 kΩ
Solution: (c) + −
Vs V1
V − + V0
AV = 4
V1

250 V2 250 V3 250

(a) −8 (b) −9 (c) −10 (d) −11

+ 50 V + + Solution: (b)
V1 1k 1k V
− 1 − 50 V2 1k − 50 V3 4
Open loop gain
V0
= 100
−Vi
V4 V3 V2
⇒ AV = × × 10 k
V3 V2 V1
1k
Voltage across 1k after first stage + −
Vs Vi V0
− +
1000 × 50 V1
= = 40
1250
V3
Similarly, = 40
V2 Nodal analysis at non-inverting terminal of the op-amp
∴Av = 40 × 40 × 50 = 8 × 104
AV in dB = 20 log (8 × 104) = 98 dB. Vi − VS Vi − V0
+ =0
Hence, the correct option is (c). 1 10
Chapter 1  Operational Amplifiers  |  5.25

10Vi − 10Vs + Vi – V0 = 0 −1
CR ∫
V0 = V dt
11Vi = 10 Vs + V0
−1  − cos 100t 
−11 V0 = = + 10 cos 100t V
= 10Vs + V0 10 m 100  100 
100
Hence, the correct option is (a).
−111
V0 = 10 Vs
100 34. If the op-amp in the figure has an input offset voltage
of 5 mV and an open-loop voltage gain of 10,000, then
V0 −1000
⇒ = ≅ −9 V0 will be [2000]
Vs 111
+15 V
Hence, the correct option is (b).

33. In the figure assume the op-amps to be ideal. The out- V0
+
put V0 of the circuit is [2001]
−15 V
10 mH 10 µF

10 Ω (a) 0 V (b) 5 mV
− 100 Ω (c) +15 V or −15 V (d) +50 V or −50 V
Vs = 10 cos(100t) 1 −
+ 2 3
+ V0 Solution: (c)
I/P offset voltage = 5 mV
Open loop gain = 104

(a) 10 cos(100t) 15 V
t

10∫ cos(100t ) dt
(b) −
0
V0
t +
10 −4 ∫ cos(100t ) dt
(c)
0
−15 V
d −4
(d)
10 cos(100t )
dt
O/P voltage = 5 m × 104
Solution: (a)
= 50 V
10 mH 10 µF = 15 V (∴O/P voltage can’t go beyond ± 15 V)
Hence, the correct option is (c).
10 Ω
− 100 Ω 3 5. The output voltage V0 of the circuit shown in the figure
Vs = 10 cos(100t) −
+ V V0 is [1997]
+
10 K

5K

Output of first op-amp Vo
− L dVs +
V= +
2V -
R dt
100 K
−10 m 10 K
= × 10 × 100 sin (100t )
10
=
+ sin(100t)
Output of 2nd op-amp (a) −4 V (b) 6 V
(c) 5 V (d) −5.5 V
5.26 | Analog Electronics

Solution: (d) By calculating gain at the non-inverting terminal of the


Applying nodal analysis, op-amp
10 K
 100   99 
(V0)2 = 4 ×  × 1+
5K  100 + 100   100 
2V −
Vo =
2 × 1.99
+

V =
3.98 V
100 K 99
10 K
100
V1 = 4 V −
V0
V V − V0 +
100
+ =0
10 100 V2 = 4 V
11V = V0 (1) 100
V − 2 V − V0
+ =0
5 10
Total output voltage is
2V− 4 + V −V0 = 0
3V = 4 + VO (2) V0 = (V0)1 + (V0)2
3V0 = −3.96 + 3.98
= 4 + V0 = 0.02 V.
11
8V 37. An op-amp has an offset voltage of 1 mV and is ideal
⇒− 0 =4 in all other respects. If this op-amp is used in the circuit
11
shown in the figure, the O/P voltage will be (Select the
⇒ V0 = −5.5 V nearest value) [1992]
Hence, the correct option is (d). 1 KΩ
36. For the ideal op-amp circuit of figure, determine the
1 KΩ
output voltage V0. [1993]

Vo
+
100 Ω 99 Ω


+
100 Ω (a) 1 mV (b) 1 V (c) ±1 V (d) 0 V
4V
+ Vo Solution: (a)
- 100 Ω
I/P offset voltage = 1 mV
This voltage is applied to the terminal because off
set is an error and if should be positive
Solution:  By calculating gain at the inverting terminal
1K
of the op-amp
1K
 −99 
(V0)1 = 4 ×  −
 100  Vo
+
=
−3.96 V
99

100  Rf 
− V0 (off) = Vi (off) 1 + 
 R1 
+
 1k 
100 100 Vo 1 m 1 + 
=
4V
+  1k 

=
2 mV
Chapter 1  Operational Amplifiers  |  5.27

1K 39. Assume that the operational amplifier in figure is ideal,


the current I through the 1 K ohm resistor is________.
1K  [1992]

Vo(off)
+ 2 KΩ
Vi(off)

Hence, the correct option is (a). −


+ 1 KΩ
38. The circuit of the figure uses an ideal op-amp for small
2 mA
positive values of Vin, the circuit works as [1992] 2 KΩ

R

Vin Vout
+ Solution:  If input current at inverting terminal of op-
amp is 2 mA, applying nodal analysis at node P,

2K if

(a) a half-wave rectifier


(b) a differentiator p
− i
(c) a logarithmic amplifier V
+ 1K
(d) an exponential amplifier iL
2 mA
Solution: (c) 2K
Vout +VD = 0
Vout = −VD (1)
VD
+ − 0 −V
− 2m = 0
2k
R
− V = −4V
Vin i Vout
+ V = 2iL = −4
As current does not enter the op-amp
⇒iL = −2 mA
iL = i −if
 hVVd  ⇒ i = iL+ if
Diode current, I D ≈ I S  e T − 1
  = −2m − 2m
i = −4 mA
approximating, lD ≈ lS eVD /hVT
40. The op-amp of figure shown below has a very poor
l  open loop voltage gain of 45 but is otherwise ideal. The
⇒VD = hVT ln  D  (2) gain of the amplifier equals: [1990]
l  S

Output voltage of op-amp is 8K

l  2K
Vout = −VD = −hVT ln  D  (3)
 ls  −

Input voltage can be written as + Vout


Vin
Vin = iR = IDR [∴I = ID] (4)
From equations (3) and (4) (a) 5 (b) 20 (c) 4 (d) 4.5
V  Solution: (d)
Vout = −hVT ln  in 
 lS R  Vout
= 45 (1)
∴the circuit acts as a logarithmic amplifier. Vin − V
Hence, the correct option is (c).
5.28 | Analog Electronics

8K Non-inverting gain is given by

2K V0
− A2 = =0
V V2 V1
+ Vout
Vin  R f   100 
= 1 +    = 90.09
 R1   100 + 1
Gain of the op-amp is given by
A2 − A1 90.09 − ( −90)
Vout 8k Ad = = = 90
= 1+ = 5 (2) 2 2
V 2k
Ac = A1+A2 = −90 + 90.09 = 0.09
V 1
= b = = 0.2
Vout 5 Ad 90
CMRR = = = 1000
Vout Ac 0.09
A= = 45
Vin − V Hence, the correct option is (c).
AB = 9 (comparable to 1) 42. If the input to the circuit of figure is a sine wave the
A 45 output will be [1990]
∴ Af = = = 4.5
1 + AB 1 + 9 Input −
output
Hence, the correct option is (d). +

41. The CMRR of the differential amplifier of the fi


­ gure
shown below is equal to [1990]
90 K (a) A half-wave rectified sine wave
(b) A full-wave rectified sine wave
1K (c) A triangular wave
V1 − (d) A square wave
Vo
+ Solution: (d)
1K
V2 If I/P applied is sine wave
O/P = A (0 − sinwt)
100 K
= −A sinwt
Input −
output
+
(a) ∞ (b) 0 (c) 1000 (d) 1800
Solution: (c)
Inverting gain of the op amp is given by
Since, op-amp was a very high voltage gain A, there-
V fore the O/P voltage saturates and results in a square
A1 = 0 =0
V1 V2 wave.
Hence, the correct option is (d).
−90 k
= = −90 43. Refer to figure shown below: [1989]
1k
R2
90 K +
Vo
1K −
V1 −
R1
Vo
V2 + −
1K + +
100 K Vi

Chapter 1  Operational Amplifiers  |  5.29

R2 L R
(a) For Vi > 0, V0 = − Vi
R1
R C
(b) For Vi > 0, V0 = 0 V1 −
R V0
(c) For Vi < 0, V0 = − 2 Vi +
R1
(d) For Vi< 0, V0 = 0
Solution: (c) p
p
(a) (b)
Case 1: Vi > 0 2
∴ D1 is on
3p
(c) (d) 2p
R2
Vo
2
Solution: (c)

R1 D1 D2 L 
− R= 
C  given
+ + V

1 
Vi w=
− LC 
L R

V is SC (assuming op-amp ideas) R C


D2 is OFF Vi −
Vo
V0 = 0 V +
R2
Vo
Output to input voltage ratio is
R1
Vi −  
+ Vo Zf  R + jwL 
=− = −
Vi Z1 j 
 R− 
 wc 
R 2 + w 2 L2 wL 1
Case 2: Vi < 0 = × ∠ tan −1 + tan −1 +p
1 R wCR
D1 is OFF, D2 is ON R2 +
w 2C 2
− R2 Therefore the phase angle
V0 = × Vi
R1
 wL 1 
R2 f =  tan −1 + tan −1 +p
Vo  R w CR 
R1  wL 1 
Vi −  + 
= tan −1  R wCR  + p
+ w L 1
 1 − × 
 R wCR 
 
Hence, the correct option is (c). −1
 w 2 LC + 1 1 
= tan  × +p
44. The op-amp shown in the figure below is ideal.  wCR 1−
L 
 CR 2 
R − L / C . The phase angle between V0 and V1 at  
w = 1/ LC  [1988] 
−1  1 + 1 1 
= tan  × +p
 wCR 1 − L 
 CR 2 
 2 LC 1 
= tan −1  × +p
 CR 0 
p
= +p
2
3p
 
 w 2 LC + 1 −1 1 
= tan  × +p
 wCR 1−
L 
 CR 2 
 

−1  1 + 1 1 
= tan  × +p
 wCR 1 − L 
5.30 | Analog Electronics
 CR 2 
 2 LC 1 
= tan −1  × +p
 CR 0  A+ − A− 100 − ( −100)
Ad = = = 100
p 2 2
= +p
2 CMRR = 60 dB
3p = 103
=
2
Ad
Hence, the correct option is (c). CMRR = 103 =
Ac
45. In the figure shown below, if the CMRR of the opera-
tional amplifier is 60 dB, then the magnitude of the out- O/P voltage is given by,
put voltage is: [1987]
V0 = Ac Vc + Ad Vd

+ 100 k Ad 100
R R = × 1 + 0 = 3 = 0.1 V
2V 10 3
10
− 1k

+
+
R R
1k
V0

Five-marks Questions
100 k
1. Assume that the op-amp in the figure, ideal

Solution: 
0.1 V
R
+ 100 k + − B
2V R R Vs A +
− V1 1 k − + V
V− − o

V2 V0
V+ +
R R
1k
100 k (a) Obtain an expression for Vo in terms of Vs, R and the
reverse saturation current Is of the transistor.
(b) If R = 1 Ω, Is = 1 pA and the thermal voltage VT =
Apply nodal analysis at node voltages V1 and V2, 25 mV, then than what is the value of the output
voltage Vo for an input voltage Vs = 1V?
V1 − 2 V1 (c) Suppose that the transistor in the feedback path is
+ =0 replaced by a p − n junction diode with a reverse
R R
saturation current of Is the p-side of the diode is
V1 = 1 V connected to node A and the n-side to node B. Then
V2 − 2 V2 what is the expression for Vo in terms Vs, R and Is?
+ =0
R R  [2001]
V2 = 1 V Solution: (a)  Current in resistance R is 1

Vd = V2 − V1 = 0 V VS − 0 VS
i= =
R R
V2 + V1 1 + 1
VC = = =1V i = iC = ie = I S eVbe/VT
2 2
VS V
− V −Rf = I S e be/VT
A = 0 = = −100 R
V1 V2 = 0
R1
Vbe + V0 = 0
V0 V +
 R f   100   100  V0 = −Vbe
A+ = = 1 + R  =  100 + 1 1 + 1  = 1000
V2 V1 = 0
V2 1
VS
= I S e −V0 /VT
R
Chapter 1  Operational Amplifiers  |  5.31

V 
Vd
V0 I VS I
− = ln  S  I S eVT − = −
VT  RI S  S R S
Vd V 
= ln  S 
V  VT  RI S 
V0 − VT ln  S 
 RI S 
V 
Vd = VT ln  S 
 1   RI S 
(b) V0 = −25 × 10 −3 ln 
 1 × 10 −12 
RIS – Vd – V0 = 0
V0 = −0.69 V
 V 
V Vd = RI S − V0 = VT ln  s 
(c) i = S  RI S 
R
i = i f = I f eVd /VT = Vs /R KT  VS   KT 
V0 = RI S − ln   VT = q 
q  RI S 
V 
Vd
= ln  S 
VT  RI S  (b) Using equation of V0 of part (a)
V0 + Vd = 0
 Vi 
V0 = −Vd 0 = (100 × 103 ) × (1 × 10 −6 ) − 25 × 10 −3 ln 
 100 × 103 × 1 × 10 
V 
V0 = −VT ln  S   Vi 
 RI S  0 = (100 × 103 ) × (1 × 10 −6 ) − 25 × 10 −3 ln 
 100 × 103 × 1 × 10 

2. Consider the circuit given in figure using an ideal oper-
ational amplifier. ln 10 Vi = 4

e4
R
Vi =
10
+
+ V
Vi + − o Vi = 5.46 V
− + RIs

3. Shown that the system in figure is a double integrator.

The characteristics of the diode are given by the rela- In other words, prove that the transfer gain is given by
tion I = ls [eqV/KT −1] Vo (s) 1
= assuming ideal op-amp.
Where V is the forward voltage across the diode. Vi (s) (CRs )2
(a) Express Vo as a function of Vj assuming Vj > 0.
R R
Vs −
KT Op-amp Vo
(b) If R = 100 kΩ, Is = 1mA and = 25 mV. Find the
q +
2C
Vi for which Vo = 0. [1997]
Solution:
V − RI S C C
(a) I = S
R
 Vd  V R/2
I f = I s e VT − 1 = I = S − I S
  R
[1995]
5.32 | Analog Electronics

Solution: 1 kΩ 2 kΩ 4 kΩ 8 kΩ
1V
R VA R VB
Vs − − −
Vo Vo
i + 1V +
+
2C
7 kΩ
1 kΩ
i C C

 [1994]
R/2 Solution: By virtual short VC = 1
1K V
VA = V0 . = 0
1K + 7 K 8
By virtual short: V0 (1)
VB = 0 VA =
KCL at node A 8

VA − VS V V − VB 1k VA 2k VB 4 k VC 8k
+ A + A =0 1V
R 1 R
2CS − −
Vo
+ 1V +
2VA + 2RCs VA = VS
VA
7k
VS 1k
VA = (1)
2( RCS + 1)

Apply KCL at node C


Apply KCL at mode A
VC − 0 VC − VB VC − V0
+ + = 0 (VB = 0) VA − 1 VA − VB
R 1 1 + =0
1K 2K
2 Cs Cs
2VA − 2 + VA − VB = 0
RCS V0
VC = V0
2( RCS + 1) From equation (1) VA =
8
VA − 0 0 − VC 3V0
i= = = −CS VC VB = −2
R 1/CS 8
Apply KCL at node C
Vs
VA = − RCS VC =
2( RCS + 1) VC − VB VC − V0
+ =0
4 8
From equation (1)
2VC − 2VB + VC − V0 = 0
Vs  RCS V0 
= − RCS   3V0
2( RCS + 1)  2( RCS + 1)  VC = 1 V and VB = −2
8
V0 1  3V 
- ∴ 3 × 1 − 2  0 − 2 − V0 = 0
VS ( RCS )2  8 

4. Find the output voltage of the following circuit assum- V0 = 4 V


ing ideal op-amp behaviour.
5. Find the output, V0 in the following circuit figure, as
summing that the op-amps are ideal.
Chapter 1  Operational Amplifiers  |  5.33

1 kΩ 2 kΩ +10 V

2 kΩ
R2
2 kΩ
− 2 kΩ R1
Ra Rd A
A −
+ B C Vo −
+ Vo
+
B R1
Rb Rc R2
1V 2V

[1993]
 [1992: 5 Marks]
Solution: By virtual short
Solution:
VA = 1 V VC = 2 V
Rb 100
Apply KCL at node A (a) VA = 10 × = 10 × = 5V
Ra + Rb 100 + 100
0 − VA VA − VB VA − VC Rb 100
= + VA = 10 × = 10 × = 5V
2k 2k 1k R +R 100 + 100
a b
0 − 1 1 − VB 1 − 2
= + VB = 5V
2k 2k 1k
Given op-amp is a differential amp
VB = 0
R2 12k
Apply KCL at node C So V0 = (VB − VA ) = (5 − 5) = 0
R1 10k
VC − VB VC − VA VC − V0
+ + =0 Rb
2k 1k 2k (b) VA = 10 × = 5V
Rd + Rb
V0 = 6 V
VB = 9.55 V
6. Consider the circuit shown in figure. The circuit uses an
ideal operational amplifier. Assuming that the imped- 12k
V0 = (4.55 − 5) = −0.55V
ances at nodes A and B do not load the preceding bridge   10k
circuit, calculate the output voltage V0.
(a) when Ra = Rb = Rc = Rd = 10 Ω.
(b) when Ra = Rb = Rc = 10 ohms and Rd = 120 Ω.
R2 = 12 kΩ
R1 = 10 kΩ
Chapter 2
Diodes Applications
1 
T /2 T
One-mark Questions VL (avg) =  ∫ 4 at + ∫ −10 at 
T  0 T /2 
1. In the circuit shown Vs is a square wave of period T
with maximum and minimum values of 8 V and −10 V. 1 T  T 
=  4 × − 10 T −  
respectively. Assume that the diode is ideal and R1 = R2 T 2  2 
= 50 W. 1 3T
  The average value of VL is _____ volts (rounded off
=
T
[ 2T − 5T ] = −
T
= −3 V

to 1 decimal place). [2019]
2. In the figure, D1 is a real silicon pn junction diode with
R1 a drop of 0.7 V under forward bias condition and D2
is a Zener diode with breakdown voltage of –6.8 V.
+
The input Vin (t) is a periodic square wave of period T,
+8 whose one period is shown in the figure. [2017]
0 Vs +

R2 VL
Vin(t) 10 μF
T T
–10 2 – +14 V
D1
Solution: 0 10 Ω Vout(t)
R1 t (seconds) D2
−14 V
+
8V
+ R2 V2 0 Assuming 10 t << T, where t is the time constant of
the circuit, the maximum and minimum values of the
-10V
-
output waveform are respectively.
(A) 7.5 V and -20.5 V (B) 6.1 V and -21.9 V
For the half cycle debde is reverse biased (C) 7.5 V and -21.2 V (D) 6.1 V and -22.6 V
R1 = 50Ω 3. The figure shows the I–V characteristics of a solar cell
R2
+
VL = VS ⋅ illuminated uniformly with solar light of power 100 m
R1 + R2
W/cm2. The solar cell has an area of 3 cm2 and a fill fac-
8V + R2Ω 80 tor of 0.7. The maximum efficiency (in %) of the device
= 8⋅
50 + 50 is ____. [2016]
VL = 4 V I
-
Isc = 180 mA
For -ve half cycle debde is forward biased
+

-10V + R2 = 50Ω V2
VL = VS V
(0, 0) Voc = 0.5 V
= −10 V
-
Chapter 2  Diodes Applications  |  5.35

Solar light power = 100 mW/cm2


Solution:  5. The diode in the circuit given below has VON = 0.7 V
Solar cell area = 3 cm2 but is ideal otherwise. The current (in mA) in the 4 k Ω
resistor is ______. [2015]
Total power incident on the solar cell will be,
Pin = 3 × 100 = 300 mW; 2 kΩ 3 kΩ
D 1 kΩ
Now, 1 mA
I mVm
Fill factor = 4 kΩ 6 kΩ
I SC .VOC

Ι mVm
= 0.7
180 mA × 0.5 V  Solution: 
Assume that diode is ON then

Ι mVm = 63 mW 
1 kΩ
I mVm 2k 3k 1/3
Maximum efficiency = ×100
Pin 1k kΩ 0.5
 1m 1m

i kΩ
63 mW 4k 6k
= × 100 = 21% 4 kΩ 6 kΩ
300 mW 
Hence, the correct Answer is (21%).
4. The diodes D1 and D2 in the figure are ideal and ⇒ current across 4 k Ω is given by
the capacitors are identical. The product RC is very
large compared to the time period of the AC voltage.
=
(
1 × 10 −3 × 6 + 1
2 )
( )( )
Assuming that the diodes do not breakdown in the
6+ 1 + 4+ 1
reverse bias, the output voltage V0 (in volt) at the steady 2 3 
state is ___. [2016]
3
D1 = = 0.6 mA
5
+ Hence, the correct Answer is (0.59 to 0.61).
C
10 sin ω t 6. The figure shows a half-wave rectifier. The diode D is
R V0
ideal. The average steady-state current (in amperes)
ac – through the diode is approximately___________.
C
 [2014]
D2
D

Solution: 
Consider the figure given below
D1 10 sin wt
100 Ω R C 4 mF
I = 50 Hz

C +
10 sin ω t
R V0

ac
Solution:  (0.1 A)

C
D

D2
10 sin wt
Now for positive peak diode D1 is ON and diode D2 is 100 Ω R C 4 mF
I = 50 Hz
ON, therefore
Vo = +Vc – Vc = 0
At steady state, capacitor is 0 C.
for negative peak diodes D1 and D2 are OFF, therefore
Steady-state current
Vo = 0 V
10
= = 0.1 A
Hence, the correct Answer is (0 V). 100
5.36 | Analog Electronics

7. Two silicon diodes, with a forward voltage drop of 0.7 8. The diodes and capacitors in the circuit shown are
V, are used in the circuit shown in the figure. The range ideal. The voltage v(t) across the diode D1 is
of input voltage Vi for which the output voltage Vo = Vi [2012]
is
[2014] C1 D2
V(t)
R
+ + +
cos (wt ) D1 C2
D1 D2

Vi Vo
+ +
−1 V − 2V −
− −

(a) −0.3 V < Vi < 1.3 V (a) cos (w t) −1


(b) −0.3 V < Vi < 2 V (b) sin (w t)
(c) −1.0 V < Vf < 2.0 V
(d) −1.7 V < Vf < 2.7 V (c) 1 − cos (w t)
(d) 1 − sin (w t)
Solution: (d)
Solution: (a)
R
+ +
C1 D2
V(t)
D1 D2
Vi Vo
+
+ +
−1 V − 2V − cos (wt ) D1 C2

− −

Case 1:
Vi < − 1.7
D1 → ON
When excited cos wt the clamping section clamps the
D2 → OFF
positive peak to 0 V and negative peak to −2 V.
R
+ + So whole cos wt is lower by −1 V
I/p

Vi Vo
+ +1
−1 V − p/2 p
− − cos wt
3p/2 2p
−1
V0 = −1.7 V
Case 2:
− 1.7 < Vi  < 2.7 V(t )
D1 → OFF
D2 → OFF 0 cos (wt ) − 1

R −1
+ +

−2
Vi Vo

− −
Hence, the correct option is (a).
9. In the following limiter circuit, an input voltage Vi =
V0 = Vi 10sin100pt is applied. Assume that the diode drop is
∴ the range of I/P voltage Vi for which the O/P voltage 0.7 V when it is forward biased. The Zener breakdown
V0 = Vi is −1.7 V <Vi < 2.7V. voltage is 6.8 V. [2008]
Hence, the correct option is (d).
Chapter 2  Diodes Applications  |  5.37

1 kΩ Solution: (c)
The correct full wave rectifier circuit is
D1
Vi D2 Vo

Z 6.8 V
Input
Output
The maximum and minimum values of the output volt-
age respectively are
(a) 6.1 V, −0.7 V Hence, the correct option is (c).
(b) 0.7 V, −7.5 V 11. The circuit shown in the figure is best described as a
 [2003]
(c) 7.5 V, −0.7 V
(d) 7.5 V, −7.5 V

Solution: (c)
Output
During +ve part of Vi > 6.8 V D1 will be forward biased.
Thus net voltage = 6.8 + 0.7
=7.5 V (max)
During −ve part of Vi D2 will be forward biased
(a) bridge rectifier
D1 will be reverse biased
(b) ring modulator
Thus net voltage = −0.7 (min.)
(c) frequency discriminatory
Hence, the correct option is (c).
(d) voltage doubler
10. The correct full wave rectifier circuit is
Solution: (d)
[2007] The circuit shown in the figure is best described as a
(a) voltage doubler.

Input
Output Output

(b)
Hence, the correct option is (d).
1 2. For full wave rectification, a four diode bridge rectifier
Input
is claimed to have the following advantages over a two
Output diode circuit. [1998]
1. Less expensive transformer
(c) 2. Smaller size transformer and
3. Suitability for higher voltage application of these
(a) only (1) and (2) are true
Input (b) only (1) and (3) are true
Output (c) only (2) and (3) are true
(d) (1), (2), as well as (3) are true
(d) Solution: (d)
A four diode bridge rectifier uses the smaller size of
transformer, which is less expensive transformer and
Input these rectifiers are suitable for higher voltage applica-
Output tions, because of low P/V rating required of each diode.
Hence, the correct option is (d).
5.38 | Analog Electronics

13. The wave shape of V0 in the figure is[1993] V0


5.9
4.1 V 4.1 V
V0 0 t

−5.9

10 sin 314 t 10 KΩ V0
Hence, the correct option is (a).

(a) 5.9
Two-marks Questions
V0 0 1. In the circuit shown, Vs is 10 V square wave of period,
T = 4 ms with R = 500 W and C = 10 mF. The capacitor
−5.9 is initially uncharged at t = 0, and the diode is assumed
(b) 4.1
to be ideal. The voltage across the capacitor (Vc) at 3
V0 0 ms is equal to _____ volts (rounded off to one decimal
place).[2019]
−4.1
R
(c) 4.1 +10 +
V0 0 0
T T Vs +
– C VC
–10 2
−4.1 t=0 –
(d) 5.9
Solution:
V0 0
= 17.5 W VL
500 Ω
O+
Solution: (a) 0mF 10
Case 1: During +ve half cycle VS + VC
T/2 T
diode DA is forward bias, so DA is short circuit.
diode DB is reverse bias, so Diode DB is in conducting O- -10

state when Vi > 4.1 V


For +ve half cycle D is forward biased
4.1 V 4.1 V
V0 ∴VC = V0 = 10 V
For -ve half cycle D is reverse biased
∴VC = V0 1 − e − t / RC 
10 sin 314 t 10 K V0
 −2 ×10 −3 
= 10 1 − e 5×10 
−3

 
VC
 

5.9 V = 10 1 − e −0 .4  = 3.3 V



Hence, the correct answer is 3.3 V
t
2. The circuit shown in the figure is used to provide regu-
lated voltage (5 V) across 7the 1kW resistor. Assume
Case 2: During −ve half cycle that the Zener diode has a constant reverse breakdown
diode DB is forward bias, so DB is short circuit. voltage for a current range, starting from a minimum
Diode DA is reverse bias, so DA is in conducting state required Zener current, IZmin = 2 mA to its maximum
when |Vi| > 4.1 V allowable current. The input voltage VI may vary by
5% from its normal value of 6 V. The resistance of the
VC
diode in the breakdown region is negligible.  [2018]
I1 R

V1 5V 1 kΩ
5.9 V
Chapter 2  Diodes Applications  |  5.39

The value of R and the minimum required power RMS current Irms (in mA) through the diode is _____.
dissipation rating of the diode, respectively, are Solution:  AC voltage source V = 10 sin(t)
(A) 186 W and 10 m W Let RMS current Irms = I
(B) 100 W and 40 mW
Resistance R1 = 3 k Ω
(C) 100 W and 10 mW
(D) 186 W and 40 m W Resistance R2 = 6 k Ω
Solution:  Resistance R3 = 9 k Ω,
R
I I I
+
IL V=
3
[ R1 ] + [ R2 ] + R3
6 3 
V1 5V 1 kΩ
I I I
– =
3
[3] + 6 [6] + 3 [9]

Current IS(min) = 7 mA V = I [5Ω]

Current IL = 5 mA
V
Current IZ = 2 mA = 5Ω
I 
We know that
VI(min) − 5 ⎛ Vm ⎞
IS(min) = ⎜⎝ R ⎟⎠
Rmax I rms =

2 
5.7 − 5 ⎛ 10 ⎞
Rmax = = 100 Ω 
7 ⎜⎝ ⎟⎠
5
= = 1 mA
Now we have 2 
6.3 − 5 But key given
IS(max) = = 13 mA 1
100 = = 0.7
And 2

IZ(max) = Is(max) – IL = 13 – 5 = 8 mA Hence, the correct Answer is (0.7).


4. Assume that the diode in the figure has Von = 0.7 V, but
minimum power dissipation rating is otherwise ideal
PZ(min) = VZ . IZ(max)
R1
= (5 × 8) mW
= 40 mW 2 kΩ i2

Hence, the correct option is (B) + 2V R2



6 kΩ
3. An AC voltage source V = 10 sin(t) volts is applied to
the following network assume that R1 = 3 kΩ, R2 = 6 kΩ
and R3 = 9 kΩ, and that the diode is ideal [2016]
The magnitude of the current i2 (in mA) is equal to
e R3 h ____. [2016]
Solution:  Let the diode is in ON state as shown below
R2 R2 R3 in figure
R3 + –
f R2 g iD
d c 0.7 V
R2 R1 2 kΩ i1 i2
R1 R2
R2
+ 6 kΩ
2V –
a b
R1
AC

Let, i2 = i1 + iD =
V = 10 sin(t)
5.40 | Analog Electronics

Applying KVL For HWR


2 − 0.7 Vm
i2 = = 0.216 mA Vdc = = 3.18 V
6 π 
0.7 Vdc
i1 = mA = 0.35 mA Rc = = 3.18 Ω
2 I dc

i2 < i1. 1 1
f = = 3 = 1 kHz
The Diode is forward biased and it required positive T 10 
current to flow through it for its conduction. For a HWR shunt capacitor filter
So, diode current iD = –0.134 mA Vac ( rms) 1
iD is negative so, Diode is in OFF position, γ = =
Vdc 2 3 f c .RL
2 γ = 0.19
∴ i2 = mA = 0.25 mA
2+6
We know that
Here, we obtained negative iD so, the diode will remain
off. Vac = 0.19 × 3.18 = 0.6 V
Hence, the correct Answer is (0.25 mA). V u
Vac = r =
5. The figure shows a half wave rectifier with a 475 µF 2 3 2 3
filter capacitor. The load draws a constant current I0 = ⇒ u = 0.6 × 2 3
1 A from the rectifier. The figure also shows the input 
=2.07 V
voltage Vi, the output voltages Vc and the peak to peak
voltage ripple u on Vc. The input voltage Vi is a triangle Hence, the correct Answer is (2.07 V).
wave with an amplitude of 10 V and a period of 1 ms. 6. If the circuit shown has to function as a clamping cir-
 [2016] cuit, then which one of the following conditions should
be satisfied for the sinusoidal signal of period T  ?
 [2015]
C
+ + –
Vi + 475 μF VC IO = 1 A

V R

+ 10 V Vi

0 t RC << T (B)
(A) RC = 0.35 T
RC ≈ T (D)
(C) RC >> T
– 10 V Solution:  For clamping circuit RC >> T
VC Hence, the correct option is (D).
u 7. In the circuit shown below, the knee current of the ideal
Zener diode is 10 mA. To maintain 5 V across RL, the
0 t
minimum value of RL in Ω and the minimum power rat-
ing of the Zener diode in mW, respectively, are [2013]
The value of the ripple u (in volts) is ____.
Solution: 
100 Ω
ILoad
+ 10 V
Vin + 470 μF VC IO = 1 A

– VZ = 5 V RL

Amplitude of input voltage Vm = 10 V


Chapter 2  Diodes Applications  |  5.41

(a) 125 and 125 (b) 125 and 250


(c) 250 and 125 (d) 250 and 250 1 kΩ

Solution: (b) W Y X
Z

100 Ω
ILoad + 1 kΩ −

10 V
During the half cycle,
RL
all diodes are OFF & hence
VZ = 5 V
V1 V2
− + + −

5
RL min =
I L max + V −

10 − 5 5 |V1| = |V2|
I100 = = = 50 mA
100 100 V = 0 V
ILmax = I100 − Iknee During −ve half cycle

=
40 mA
5
RL min = × 100 V
40
= 125 Ω V = 0 V
Hence, the correct option is (d).
Minimum power rating of Zener should be = 50 mA ×
5 V = 250 mW Statement for Linked Answer Questions 9 and 10.
Hence, the correct option is (b). In the circuit shown below, assume that the voltage
drop across a forward biased diode is 0.7 V. The thermal
8. A voltage 1000 sinωt volts is applied across YZ. voltage Vt = kT/q = 25 mV. The small signal input Vi =
Assuming ideal diodes, the voltage measured across V cos(ωt) where Vp = 100 mV.
WX in volts, is [2013]
9900 Ω

1 kΩ
+
127 V −
W Y X IDC + Iac VDC + Vac
Z
Vi

+ 1 kΩ −

(a) sinwt 9. The bias current IDC through the diodes is [2011]
(b) (sinwt + |sin wt|)/2 (a) 1 mA (b) 1.28 mA
(c) (sinwt − |sin wt|)/2 (c) 1 5 mA (d) 2 mA
(d) 0 for all t
Solution: (a)
Solution: (d)
Drop of voltage at one diode is 0.7 V.
VYZ
12.7 − (0.7 + 0.7 + 0.7 + 0.7)
I DC =
9900
0 wt = 1 mA
5.42 | Analog Electronics

9900 Ω + V − 1Ω

+ 1Ω
127 V −
IDC + Iac VDC + Vac Vi +
− 1A
Vi

V = Vi ; Vi < 0
Case 2:
Hence, the correct option is (a).
Vi > 0
1 0. The AC output voltage vac is [2011]
(a) 0.25 cos (wt) mV (b) 1 cos (wt) mV + V −

(c) 2 cos(wt) mV (d) 22 cos (wt) mV 1Ω 1Ω


Solution: (b) +
Vi 1A
AC dynamic resistance, −

hVT
gd =
ID
V = 1 V; Vi > 0
1 × 25 mV ∴V = min (Vi, 1)
= = 25 Ω
1 mA Hence, the correct option is (b).
The AC dynamic resistance offered by each diode = 25 Ω Common Data for Questions 12 and 13.
Consider a silicon p-n junction at room t­emperature
 4 × 25 Ω 
∴Vac = Vi ( ac)   having the following parameters:
 9900 + 25  Doping on the n-side = 1 × 1017 cm−3
 100  Depletion width on the n-side = 0.1 μm
= 100 × 10 −3 Cos(wt )   Depletion width on the p-side = 1.0 μm
 9925  Intrinsic carrier concentration = 1.4 × 1010cm−3
Vs = Cos wt mV Thermal voltage = 26 mV
Hence, the correct option is (b). Permittivity of free space = 8.85 × 10−14F-cm−1
11. In the circuit given below, the diode is ideal. The volt- Dielectric constant of silicon = 12
age V is given by[2009] 12. The built-in potential of the junction [2009]
+V
(a) is 0.70 V
1Ω 1Ω (b) is 0.76 V
(c) is 0.82 V
Vi + 1A
− (d) cannot be estimated from the data given
Solution: (c)

kT  N A N D 
V0 = ln
(a) min (Vi, 1) (b) max (Vi, 1) q  ni 2 
(c) min (−Vi, 1) (d) max(−Vi, 1) N N 
= VT ln  A 2 D 
Solution: (b)  ni 
Case 1:
NAxp0 = NDxn0
Vi < 0
+ V − 1017 × 0.1
⇒ NA = = 1016 cm −3
1Ω 1Ω 1

+  1017 × 1016 
Vi
− 1A V0 = 0.026 ln  14 2 
 (1.4 × 10 ) 
= 0.28 V
Hence, the correct option is (c).
Chapter 2  Diodes Applications  |  5.43

13. The peak electric field in the device is [2009] Common Data for Questions 15 and 16.
(a) 0.15 MVcm−1, directed from p-region top-region A regulated power supply, shown in figure below, has
(b) 0.15 MVcm−1, directed from n-region top-region an unregulated input (UR) of 15 volts and generates a
(c) 1.80 MVcm−1, directed from p-region top-region regulated output Vout. Use the component values shown
(d) 1.80 MVcm−1, directed from n-region top-region in the figure
15 V(UR) Q1
Solution: (b)
q 1 kΩ 12 kΩ +
e= N d xn 0
∈ + 12 kΩ Vout

−q −
= N a x p0 −

1.6 × 10 −19 × 1017 × 0.1 × 10 −4 6V 24 kΩ
=
12 × 8.85 × 10 −14
= 0.115 MVcm −1
15. The power dissipation across the transistor shown in
Directed from n-region to p-region.
the figure is [2006]
Hence, the correct option is (b).
14. For the Zener diode shown in the figure, the Zener volt- (a) 4.8 watts (b) 5.0 watts
age at knee is 7 V, the knee current is negligible and the (c) 5.4 watts (d) 6.0 watts
Zener dynamic resistance is 10 Ω. If the input voltage Solution: (c)
(Vi ) range is from 10 to 16 V, the output voltage (V0 ) Diode voltage
ranges from[2007] VA = 6 V
+ 6 × 36
200 Ω Vout = =9V
24
Vi V0 9
I out = = 0.9 A
10

VA 6
(a) 7.00 to 7.29 V (b) 7.14 to 7.29 V I= = = 0.25 mA
24 k 24 k
(c) 7.14 to 7.43 V (d) 7.29 to 7.43 V
Power dissipation across transistor = VCE × iC
Solution: (b)
VCE = 15 − 9 = 6 V
In addition to Zener diode voltage, some voltage drop
∴iC = I + iout = 0.9 A
also takes place due to dynamic resistance (10 Ω)
PD = 6 × 0.9 = 5.4 W
200 Ω
iE = I + I out  0.9 A
+
Hence, the correct option is (c).
Vi V0 1 6. If the unregulated voltage increases by 20%, the power
dissipation across the transistor Q1
− [2006]
(a) increases by 20%
For Vi = 10 V (b) increases by 50%
 10  (c) remains unchanged
V0 = 7 + (10 − 7) 
 200 + 10  (d) decreases by 20%
= 7.14 V Solution: (b)
If unregulated power supply increases by 20% then
For Vi = 16 V VCe = 18 − 9 = 9 V
 10  ic = 0.9 A
V0 = 7 + (16 − 7) 
 200 + 10  PD = 8.1 W
8.1 − 5.4 
= 7.43 V % increase in power =  = 50%
 5.4 
Hence, the correct option is (b).
Hence, the correct option is (b).
5.44 | Analog Electronics

17. For the circuit shown below, assume that the Zener current of 0.5 mA. The maximum load current drawn
diode is ideal with a breakdown voltage of 6 volts. The from this circuit ensuring proper functioning over the
waveform observed across R is [2006] input voltage range between 20 and 30 V is
6V [2005]
1 kΩ
+

12 sin wt R VR
VL
VL = 5.8 V Load
20 – 30

(a) 6V
(a) 23.7 mA (b) 14.2 mA
(b) 6V (c) 13.7 mA (d) 24.2 mA
Solution: (a)
VL = 5.8 V
Iz 1k Ω IL
−12 V
(c) 12 V Iz
Vi
20 – 30 VL = 5.8 V Load

−6 V

(d) Maximum load current will be when Vi = Vmax


−6 V =30 V
Current through 1k resistor
Solution:  (b) 30 − 5.8
When I/P voltage increases from 0 to 6 V, diode is OFF. = IL + IZ
1k
6V ⇒ 24.2 mA = IL + IZ
IL = 24.2 mA − 0.5 mA
+
= 23.7 mA
12 sin wt R VR Hence, the correct option is (a).
19. In the voltage regulator shown in the figure, the load
− current can vary from 100 to 500 mA. Assuming that
the Zener diode is ideal (i.e., the Zener knee current
When I/P is > 6 V, Zener is under break down. is negligibly small and Zener resistance is zero in the
∴VR = Vinput− 6 = 12 sinωt − 6 breakdown region), the value of R is [2004]
Maximum, it can go up to 6 V when I/P is −ve, di- (a) 7 Ω (b) 70 Ω
ode is FB. In this case, O/P voltage follows I/P. 70
(c) Ω (d) 14 Ω
Vo 3
Solution: (d)
6V 12 − V
= IL + IZ
t R
R

−12 V

Variable load
12 V +
− 5V
Hence, the correct option is (b). 100 to 500 mA

18. The Zener diode in the regulator circuit shown in the


figure has a Zener voltage of 5.8 V and a Zener knee
Chapter 2  Diodes Applications  |  5.45

When IL = 100 mA (a) 3 V (b) 6 V


12 − 5 (c) 9 V (d) 12 V
⇒ ≥ 100 mA
R Solution: (c)
7 × 1000 As voltage at non-inverting terminal is 3 V due to Zener
R≤
100 diode, voltage at inverting terminal will be 3 V because
of virtual ground.
R = (7 × 1000/100) = 70
When IL = 500 mA, +
12 − 5
≥ 500 mA 1k Ω
R
7 × 1000 15 V DC +
R≤ = 14 Ω
500 Unregulated VZ = −
Power source 3V 40k Ω
R = 14 Ω (choosing minimum one)
Hence, the correct option is (d). regulated
20k Ω
DC Output
20. In a full-wave rectifier using two ideal diodes, Vdc and −
Vm are the dc and peak values of the voltage respec-
tively across a resistive load. If PIV is the peak inverse 3
voltage of the diode, then the appropriate relationships So, current in 20 k is
20k
for this rectifier are [2004]
3
V = mA
(a) Vdc = m , PIV = 2Vm 20
p
V 3
(b) Vdc = 2 m , PIV = 2Vm ∴V0 = × 60 k = 9 V
p 20 k
Vm Hence, the correct option is (c).
(c) Vdc = 2 , PIV = Vm
p 22. A Zener diode regulator in the figure is to be designed
V to meet the specifications: IL = 10 mA, V0 = 10 V and
(d) Vdc = m , PIV = Vm Vin varies from 30 V to 50 V. The Zener diode has Vz
p
= 10 V and Izk (knee current) = 1 mA. For satisfactory
Solution: (b) operation [2002]
In a full-wave rectifier using two ideal diodes, Vdc and Vm
R
are the dc and peak values of the voltage, respectively, +
IZ IL = 10 mA
across a resistive load. If P/V is the peak inverse voltage
of the diode, then the appropriate relationship of this Vo RL
Vin
rectifier is DZ
2Vm −
Vdc =
p
and P/V = 2Vm (a) R ≤ 1800 Ω
Hence, the correct option is (b). (b) 2000 Ω ≤ R ≤ 2200 Ω
(c) 3700 Ω ≤ R ≤ 4000 Ω
21. The output voltage of the regulated power supply (d) R > 4000 Ω
shown in the figure is [2003]
Solution: (a)
+
Vin − v0
≥ I z + I L = I1
1 kΩ R
I1 R
15 V DC + +
VZ = 3 V IZ IL = 10 mA
Unregulated −
Power source
Vin DZ Vo RL

10 kΩ
Regulated −
20 kΩ
DC Output

5.46 | Analog Electronics

When Vin = 30 V, 24. A DC power supply has a no-load voltage of 30 V, and


a full-load voltage of 25 V at a full-load current of 1 A.
30 − 10
≥ (10 + 1) mA Its output resistance and load regulation, respectively,
R are [1999]
20
≥ 11 mA (a) 5 Ω and 20% (b) 25 Ω and 20%
R
(c) 5 Ω and 16.7% (d) 25 Ω and 16.7%
⇒ R ≤ 1818 Ω (1) Solution: (a)
When Vin = 50 V, no load voltage-full load voltage
voltage regulation=
40 full load volttage
≤ 11 × 10 −3
R VNL − VFL
=
R ≤ 3636 (2) VFL
From equations (1) and (2) 30 − 25 1
= = = 20%
⇒ R ≤ 1818 Ω 25 5
Hence, the correct option is (a).
VDCNL − VDCFL
23. The transistor shunt regulator shown in the figure has R0 = =5Ω
a regulated output voltage of 10 V, when the input I DC
varies from 20 to 30 V. The relevant parameters for
the Zener diode and the transistor are: Vz = 9 5, VBE Hence, the correct option is (a).
= 0.5 V, b = 99. Neglect the current through RB. Then 2 5. A Zener diode in the circuit shown below has a knee
the maximum power dissipated in the Zener diode current of 5 mA, and a maximum allowed power dis-
(Pz) and the transistor (PT) is sipation of 300 mW. What are the minimum and maxi-
[2001] mum load currents that can be drawn safely from the
circuit, keeping the output voltage Vf constant at 6 V?
20 Ω
 [1996]
IZ IC
50 Ω
VZ +
Vin V0 = 10 V +
20-30 V + +
RB VBE 9V Load


Pz = 75 mW, PT = 7.9 W
(a)
Pz = 85 mW, PT = 8.9 W
(b) (a) 0 mA, 180 mA (b) 5 mA, 110 mA
Pz = 95 mW, PT = 9.9 W
(c)
(c) 10 mA, 55 mA (d) 60 mA, 180 mA
Pz = 115 mW, PT = 11.9 W
(d)
Solution: (c)
Solution: (c) Current through 50 Ω resistance = I
Vcc = 10 V
9−6 3
Case 1: I= = = 60 mA
50 50
20 − 10 Given that
= I = 0.5 A
20 Iz, min = 5 mA
Ic + Ib = 0.5 A, also Ic = b Ib Pz max = Iz maxVz = 300 mW
On solving, we get Ic = 0.495 A 300 × 10 −3
PT = (Vcc × Ic) 4.95 W (not given in option) I z max =
Vz
Case 2:
300 × 10 −3
= = 50 mA
30 − 10 6
= I = 1A
20
I = Iz min + IL max = Iz max + IL min
Ic + Ib = 0.5 A, also Ic = bIb 60 = 50 + IL min
On solving, we get Ic = 0.99 A means PT = 9.9 W
Hence, the correct option is (c). I L min = 10 mA
Chapter 2  Diodes Applications  |  5.47

60 = 5 + IL max 1k
+
I L max = 55 mA
D1 D2
Hence, the correct option is (c). 1k Vo
2 6. The 6 V Zener diode shown below has zero Zener 1V 2V
resistance and a knee current of 5 mA. The minimum −
value of R so that the voltage across it does not fall
below 6 V is [1992] 1 V and 2 V can not be in parallel and its violation of
50 Ω KVL
So this circuit does not exist.
(B) V0 ≤ 2 V (D2 f, B and D1 f, B)
10 V 6V R 6V 1k 2V 2V 2V
+
I2 Ii o I3
+
10 Ω
(a) 1.2 kΩ (b) 50 kΩ Vi 1 k Vo
− 1V 2V
(c) 80 kΩ (d) 0 kΩ

Solution: (c)
Knee current, Ik = 5 mA Apply KCL
For R to be minimum, IL should be maximum and current I + I1 = I2 + I3
across the Zener diode should be minimum i.e., Ik
Vi − 2 2 −1 2 − 0
Applying KVL, + I1 = +
1k 10 1k
10 − 50I − 6 = 0
4 104 − Vi
⇒I = A = 80 mA i1 =
50 1000
IL = I − IK i ≥ 0
= 80 m - 5 m 104 − Vi
= 75 mA ≥0
1000
6 6
R= = = 80 Ω Vi ≤ 104
I L 75m
Vi ≤ 104V ⇒ V0 = 2V
Hence, the correct option is (c). Case 2: Vi ≥ 104 V, so V0 ≥ 2 V Diode D1 is forward bias
Diode D2 is reverse bias
Five-marks Questions 1 kΩ Vo Vo

1. (A) Draw the transfer characteristics of the circuit of +


figure, assuming both D1 and D2 to be ideal + 10 Ω
(B) How would the characteristics change if D2 is ideal, Vi 1 k Vo
but D1 is no-ideal and has a forward resistance of − 1V 2V
10 Ω and a reverse of infinity. −
1 kΩ Apply KCL
+
V0 − Vi V0 − 1 V0 − 0
+ D1 D2 + + =0
1K 10 1K
Vi 1 kΩ Vo
− 1V 2V Vi + 100
V0 =
− 102
[1998] Vo

Solution: 
(A) Both the Diodes are ideal whom V0 is
between 1 V to 2 V   1 V ≤ V0 ≤ 2 V in this 2V
case both the diodes D1 and D2 are forward
Vi
bias 104 V
Chapter 3
BJT Analysis
VCC
One-mark Questions
R1
I2
1. Consider the circuit shown in the figure. Assume base-
to-emitter voltage VBE = 0.8 V and common-base cur-
rent gain (a) of the transistor is unity. [2017] Q1 Q2

+18 V
I1 R2

4 KΩ
44 KΩ

The value of R2 (in Ω) for which I2 = 100 µA is ____.


 [2016]
Solution:  Q1 and Q2 are perfectly matched, thus
16 KΩ 2 KΩ I1  =  I2 and forms a current mirror. So base current
IB ≅ 0.
Also,
I1 ≅ I2 = 1 mA.
The Value of the collector-to-emitter voltage VCE (in
volt) is _____________. Now using the relation for collector current
2. The Ebers Moll of a BJT is valid [2016] VCC − VCE
(A) only in active mode = Ic
(B) only in active and saturation modes R2
(C) only in active and cut off modes Substituting collector emitter VCE = 0.2 V
(D) in active, saturation and cut off modes
Solution:  The Ebers Moll model is a model developed 5.98
R2 = = 598 Ω
for BJTs. It is valid for all operating modes, active, sat- 1 mA
uration and cut off modes.
Hence, the correct Answer is (598 W).
Hence, the correct option is (D).
4. Which one of the following statements is correct about
3. Resistor R1 in the circuit below has been adjusted so an AC coupled common emitter amplifier operating in
that I1 = 1 mA. The bipolar transistors Q1 and Q2 are the mid band region? [2016]
perfectly matched and have very high current gain, so (A) The device parasitic capacitances behave like open
their base currents are negligible. The supply voltage circuits, whereas coupling and bypass capacitanc-
Vcc is 6 V. The thermal voltage kT /q is 26 mV. es behave like short circuits.
Chapter 3  BJT Analysis  |  5.49

(B) The device parasitic capacitances, coupling capac- Voltage VBE1 = VEB2 = 0.7 V
itances and bypass capacitances behave like open
circuits. Apply KVL, we get
(C) The device parasitic capacitances, coupling capac- –2.5 + 0.7 + 0.7 + 10K(IB) + 1 = 0
itances and bypass capacitances behave like short
circuits. The base current will be
(D) The device parasitic capacitances behave like
short circuits, whereas coupling and bypass ca- 2.5 − 0.7 − 0.7 − 1 0.1
IB = = mA
pacitances behave like open circuits. 10 k 10
Solution:  And the collector current will be
Low Mid-band High frequency
IC = β IB
frequency region region
Coupling Parasitic capacitances Junction Since β = 50 since IC flowing across Q2 will be
capacitors behaves like open circuit capacitors
coupling and by pass 50 × 0.1
IC = mA
capacitances behaves like 10 
short circuit
=
0.5 mA
In the mid band region, an AC coupled amplifier per-
forms different operations for different capacitors. The The required voltage is
coupling capacitance and bypass capacitance becomes
VC2 = I C ×1k
short circuited in the mid band region. While the para- 
sitic capacitance behaves as open circuited. The junc-
tion capacitance is not considerable in mid band gap. =
0.5 mA × 1K = 0.5 V.
Hence, the correct option is (A). Hence, the correct Answer is (0.5 V).
5. Consider the circuit shown in the figure. Assuming VBE1
= VEB2 = 0.7 volt, the value of the DC voltage Vc2 (in 6. An npn BJT having reverse saturation current IS = 10–15
A is biased in the forward active region with VBE =
volt) is ____. [2016]
700 mV. The thermal voltage (VT) is 25 mV and the
Vcc = 2.5 V current gain (β) may vary from 50 to 150 due to manu-
facturing variations. The maximum emitter current
(in µA) is _______. [2015]
β1=100
Q2 Solution:  IS = 10–15 A = Io
β1= 50
10 kΩ VBE = 0.7 V
Vc2
1V VT = 25 mV = 0.25 V
1 kΩ
β → 50 to 150

⇒ IE(max) = ?
Solution: 
Consider the figure given below
ηVT
2.5 V
I ≈ Io. eVBE = IC 
VBE1 = VBE2 = 0.7 V Consider for small currents n = 1
IC = 10–15 × e0.7/0.025
2.5 V β1 =100
Ic = 1.446 mA
Q1
Q2
10 kΩ IE = (β + 1) IB
β2 = 50 1V
IB IE =
(β + 1) .I
Vc2 C
IC β 
1 kΩ IE max when β = 50
5.50 | Analog Electronics

IE = 1.02 × 1.446 × 10–3 resistance Rin, and output resistance Ro for A1 and A2 are
as follows:
IE = 1.475 mA
A1: AVO = 10, Rin = 10 kΩ, Ro = 1 kΩ
Hence, the correct Answer is (1465 to 1485). A2: AVO = 5, Rin = 5 kΩ, Ro = 200 kΩ
7. In the ac equivalent circuit shown, the two BJTs are The approximate overall voltage gain vout/vin is
biased in active region and have identical parameters [2014]
with β >> 1. The open circuit small signal voltage gain
+ +
is approximately _______. [2015]
A1 A2 RL 1 KΩ Vout
Vin

− −

Vout Solution: (34.72)

+
Vin +

A1 A2 Vout RL = 1 KΩ
Vin

− −

Solution: 
0.7 +
R01 R02

+

+ +
Vin Rin1 AV01 V1 Rin2 AV02 R2 Vo
− −
Vin V2 RL
Vout −

V0 RL
=
Vin AV02 × V2 RL + R0 2
+
0.7 – AV01 × Vin Rin L
where V2 =
Rin 2 + R01
From equations (1) and (2), we get
V0 RL Rin 2
= AV02 × AV01 × ×
⇒ Vin = 0.7 Vin RL + R0 2 Rin 2 + R01
V0 1K 5K
Vout = –0.7 ⇒ = 5 × 10 × ×
Vin 1K + 0.2 K 5 K + 1K
Vout −0.7 V0
= = –1 ⇒ = 34.72
Vin +0.7 Vin

Hence, the correct Answer is (–1.1 to –0.9). 10. In the circuit shown, the PNP transistor has |VBE| = 0 7V
8. A good current buffer has [2014] and b = 50. Assume that RB = 100 kΩ. For V0 to be 5 V,
(a) low input impedance and low output impedance. the value of RC (in kΩ) is_____. [2014]
(b) low input impedance and high output impedance.
(c) high input impedance and low output impedance. RC
(d) high input impedance and high output impedance.
Vo
Solution: (b)
A good current buffer has low input impedance and
high output impedance. RB
Hence, the correct option is (b). VEE = 10 V

9. A cascade connection of two voltage amplifiers A1 and


A2 is shown in the figure. The open-loop gain Avo, input
Chapter 3  BJT Analysis  |  5.51

(RC = 1.07 kΩ)


Solution:  (a) 250 Ω
(b) 27.5 Ω
(c) 25 Ω
RC
(d) 22.5 Ω
Vo
Solution: (c)
We know that
ib B C
RB VEE = 10 V

rπ r0

Appling KVL in input loop, we get E


10 = 0.7 + I B RB
9.3
IB = 0.93 mA rp = ( b + 1)re
100 k
Hence, k = bIB = 4.65 mA VT
rp = ( b + 1)
V −0 5 Ie
where I C = 0 =
RC RC
VT
rp = ( b + 1)
5 ( b + 1) I b
∴ RC =
4.65
VT
RC = 1.07 kΩ rp =
Ib
11. If the emitter resistance in a common-emitter voltage
amplifier is not bypassed, it will [2014] where I is dc current through base so I = 1mA,
b b
(a) reduce both the voltage gain and the input imped-
−3
ance. VT = 2.5mV at room temperature So, rp = 25 × 10 = 25Ω
(b) reduce the voltage gain and increase the input im- 1 × 10 −3
25 × 10 −3
pedance. rp = = 25Ω
(c) increase the voltage gain and reduce the input im- 1 × 10 −3
Hence, the correct option is (c).
pedance.
(d) increase both the voltage gain and the input imped- 13. In the circuit shown below, capacitors C1 and C2 are
ance. very large and shorts at the input frequency, vi is a
Solution: (b) small input. The gain magnitude |v0/v1| at 10 M rad/s is
For an bypassed R  [2011]
e
Ri = b re + (1 + b ) Re 5V

AI RL
and AV =
Ri 10 µH 2 KΩ 1 µF

Hence, the correct option is (b).


12. The current i b through the base of a silicon NPN +
C2
transistor is 1+0.1 cos (10,000 p t)mA. At 300 K,
the r p in the small signal model of the transistor is Q1
 [2012] + Vo 2 KΩ
2.7 V -
ib B C
2 KΩ
Vi C1
rπ r0 -

E (a) maximum (b) minimum


(c) unity (d) zero
5.52 | Analog Electronics

Solution: (a) Io
In a parallel RLC circuit R = 9.3 KΩ
L = 10 mH and C = 1nF
1
wg = Q1 Q2
LC (b1 = 700) (b2 = 715)
1
=
10 × 10 −6 × 10 −9
= 10 7 rad/s −10 V

~ 10 M rad/s
I0
So that for a tuned amplifier, gain is maximum at reso- I1 =
2
nant frequency.
⇒ I 0 = 2 I1
5V
Now KVL for Q1
9.3 K (I1) + 0.7 = 10
10 µH 2K 1 µF
⇒ I1 = 1 mA
C2 I 0 = 2 I1 = 2 mA

+
Hence, the correct option is (b).
Q1 15. The amplifier circuit shown below uses a silicon tran-
+ 2K Vo sistor. The capacitors CC and CE can be assumed to be
2.7 V - short at signal frequency and the effect of output resist-
2K C1 ance r0 can be ignored. If CE is disconnected from the
Vi circuit, which one of the statements is TRUE? [2010]
-
Vcc = 9V

Hence, the correct option is (a).


RB = 800 kΩ RC = 2.7 kΩ
1 4. In the silicon BJT circuit shown below assume that the
CC
emitter area of transistor Q1 is half that of transistor
CC V0
Q2. [2011]
b = 100
V0
Io
R = 9.3 KΩ Vs
RE = 0.3 kΩ CE
R0
R
Q1 Q2
(b1 = 700) (b2 = 715)

(a) The input resistance Ri increases and the magni-


tude of voltage gain Av decreases
−10 V (b) The input resistance Ri decreases and the magni-
tude of voltage gain Av increases
The value of current Io is approximately [2010] (c) Both input resistance Ri and the magnitude of volt-
(a) 0.5 mA (b) 2 mA age gain Av decrease
(c) 9.3 mA (d) 15 mA (d) Both input resistance Ri and the magnitude of volt-
age gain Av increase
Solution: (b) Solution: (a)
Since the emitter area of Q1 is half of transistor Q2, Given circuit after removing Ce will behave as current
series feedback. Overall voltage gain will decrease as
Chapter 3  BJT Analysis  |  5.53

feedback signal comes into picture and since it is cur- Solution: (a)


rent − series feedback, input impedance increases. 3V

Vcc = 9V IC

1 kr

RB = 800 k RC = 2.7 k
IB
V0
CC
CC
V0 b = 100

Vs
RE = 0.3 k CE
Collector current is given by
R1
3 − 0.2
IC = = 2.8 mA
1k
I C 2.8
Rs IB = = = 56 mA
b 50
Thus, IBmin = 56 μA is necessary to drive the transistor in
RB bre RC RL saturation.
Hence, the correct option is (a).
18. Assuming that the b of the transistor is extremely large
and VBE = 0.7 V, IC and VCE in the circuit shown in the
Hence, the correct option is (a).
figure are [2004]
16. The cascade amplifier is a multistage configuration of 5V
 [2005] IC
(a) CC−CB (b) CE−CB
4 kΩ 2.2 kΩ
(c) CB−CC (d) CE−CC
Solution: (b) +
Cascade amplifier is a multistage configuration of CE− VCE
CB. −
Hence, the correct option is (b).
1 kΩ 300 Ω
17. Assuming VCEsat = 0.2 V and b = 50, the minimum base
current (IB) required to drive the transistor in the figure
to saturation is [2004]
(a) IC = 1 mA, VCE = 4.7 V
3V (b) IC = 0.5 mA, VCE = 3.75 V
IC (c) IC = 1 mA, VCE = 2.5 V
(d) IC = 0.5 mA, VCE = 3 V.
1 kΩ
Solution: (c)
IB
5V
IC

4k 2.2 k

A
(a) 56 mA (b) 140 mA
(c) 60 mA (d) 3 mA
1k 30 C
5.54 | Analog Electronics

Applying voltage divider at node A (b)  AV


dB
5 ×1
VA = =1V 0
5 –3
1 − 0.7 0.3
Ie = = = 1 mA
300 300 f
40 Hz 0.5 kHz
IC = Ie as b is large
(c)  AV
⇒ IB = 0
dB
∴Vce = 5 − 1(2.2 + 0.3)
= 2.5 V 0
Hence, the correct option is (c). –3
19. Choose the correct match for input resistance of various
amplifier configurations shown below:  [2003]
f
Configuration Input resistance 40 Hz 1 kHz
CB: Common Base LO: Low (d)  AV
CC: Common Collector MO: Moderate dB
CE: Common Emitter HI: High
0
(a) CB−LO, CC−MO, CE−HI
–3
(b) CB−LO, CC−HI, CE−MO
(c) CB−MO, CC−HI, CE−LO
(d) CB−HI, CC−LO, CE−MO f
40 Hz 2 kHz
Solution: (b)
Configuration I/P resistance
CB low
Solution: (b)
CC high Frequency response of individual transistor amplifies
CE moderate AV dB
20. Generally, the gain of a transistor amplifier falls at high
frequencies due to the [2003] 0
(a) internal capacitances of the device. –3
(b) coupling capacitor at the input.
(c) skin effect.
(d) coupling capacitor at the output. f
20 Hz 1 kHz
Solution: (a)
The gain of the transistor falls at high frequency due to
D = 21/ n − 1
internal capacitances of the device.
= 21/ 3 − 1
21. Three identical RC-coupled transistor amplifiers are
cascaded. If each of the amplifiers has a frequency = 1.25 − 1 − 0.25
response as shown in the figure, the overall frequency = 0.5
response is as given in [2002]
f c 20
A fc′ = = = 40 Hz
dB D 0.5
0 fH′ = fH × D
–3 = 1 × 0.5
= 0.5 kHz
Overall frequency response of cascaded amplifier:
f AV dB
20 Hz 1 kHz
(a)  AV
dB 0
0 –3
–3

f
40 Hz 0.5 kHz
f
20 Hz 1 kHz
Hence, the correct option is (b).
Chapter 3  BJT Analysis  |  5.55

22. If the transistor in the figure is in saturation, then VCC


 [2002]
C R R
IC
Vin1 Vin2
IB
bdc denotes the
B dc current gain

IEE

−VEE
E

IC is always equal to bdcIB


(a) (a) zero (b) infinite
IC is always equal to −bdcIB
(b) Vin1 + Vin2
(c) indeterminate (d)
IC is greater than or equal to bdcIB
(c) 2VT
IC is less than or equal to bdcIB
(d) Solution: (a)
Solution: (d)  −R 
A common mode =  c 
For transistor in saturation,  2 Re 

I  VCC
IB ≥  c 
b

C R R
IC
Vin1 Vin2

IB bdc denotes the


dc current gain
IEE

−VEE

Given Re = ∞
Hence, the correct option is (d). thus, ACM = 0
Hence, the correct option is (a).
23. The current gain of a BJT is [2001]
25. Introducing a resistor in the emitter of a common emit-
gm ter amplifier stabilizes the dc operating point against
(a) gmro (b)
ro variations in [2000]
(a) only the temperature
gm
(c)
gmrp (d) (b) only the b of the transistor
rp (c) both temperature and b
(d) none of the above
Solution: (c)
Solution: (c)
Current gain of BJT is gmgp Both variations in temperature as well as variation in b
24. In the differential amplifier of the figure, if the source 26. The current gain of a bipolar transistor drops at high
resistance of the current source IEE is infinite, then the frequencies because of [2000]
common-mode gain is [2000] (a) transistor capacitances
(b) high current effects in the base
5.56 | Analog Electronics

(c) parasitic inductive elements Thus it is equal to gm.


(d) the early effect Hence, the correct option is (a).
Solution: (a) 28. The circuit of the figure is an example of feedback of
At high frequency, due to transistor capacitances, gain the following type [1998]
starts reducing.
Ve 4.3 +Vcc
∴ Ie = =
Re 4.30 Vo
Ie = Ic = 10 mA
Hence, the correct option is (a). Vi
27. In the cascade amplifier shown in the figure, if the com-
mon-emitter stage (Qt) has a transconductance gm1, and
the common base stage (Q2) has a transconductance
gm2, then the overall transconductance g( = i0/Vi) of the (a) current series
cascade amplifier is  [1999] (b) current shunt
Q2 (c) voltage series
i0
Vo (d) voltage shunt

Solution: (d)
+Vcc
RL
Vi Q1 Vo

Vi

(a)
gm1 (b)
gm2

g m1 (d)
(c) g m2
The circuit of the figure is an example of feedback of
2 2
the type voltage shunt.
Solution: (a) Hence, the correct option is (d).
Q2 i0
Vo 29. From the measurement of the rise time of the O/P pulse
of an amplifier whose input is a small amplitude square
wave, one can estimate the following parameter of the
amplifier: [1998]
RL (a) gain-bandwidth product
Vi Q1 (b) slew rate
(c) upper-3-dB frequency
(d) lower 3-dB frequency
Solution: (c)
i0 0.35
gm = g m = , i ≅ Ie2 = Ic1 BW = f H =
Vi 0 tr
IC1 = bIB, Ie2 = IC1
I0 ≅ bIB1, Vi = IB1gp Upper 3- dB frequency
Hence, the correct option is (c).
i0 bI 3 0. A distorted sinusoid has the amplitude, A1, A2, A3, … of
= B1 = gm1
Vi I B1rp the fundamental, second harmonic, third harmonic,…
respectively. The total harmonic distortion is [1998]
I c1
= ( I c1 = bI B1 ) A + A3 + .....
Vi (a) 2
A1
In this cascade connection overall transconductance gm
is equal to transconductance of the first stage. A2 + A32 + .....
(b) 2
A1
Chapter 3  BJT Analysis  |  5.57

33. In a series regulated power supply circuit the voltage


A2 + A32 + .....
(c) 2 gain Av of the ‘pass’ transistor satisfies the condition
A12 + A22 + A32  [1998]
(a) Av → ∞ (b) 1 << Av << ∞
A2 2 + A32 + ... (c) Av = 1 (d) Av << 1
(d)
=
A1
Solution: (c)
Solution: (b) In series regulated power supply, pass transistor is emit-
ter follower type.
A2 2 + A32 + ...
Total harmonic distortion = Thus, gain is nearly equal to me.
A1 Hence, the correct option is (c).
where A1, A2, A3… are the fundamental, second har- q
monic, third harmonic, … respectively. 34. The unit of are [1998]
kT
Hence, the correct option is (b).
(a) V (b) V−1
31. The emitter coupled pair of BJTs gives a linear transfer (c) J (d) J/K
relation between the differential O/P voltage and the
differential input voltage Vid only when the magnitude Solution: (b)
of Vid is less ‘a’ times the thermal voltage, where ‘a’ is kT
 [1998] Thermal voltage = VT =
q
(a) 4 (b) 3 (c) 2 (d) 1
q
Solution: (d)
So, → V −1
kT
The emitter coupled pair of BJTs gives a linear transfer
Hence, the correct option is (b).
relation between the differential O/P voltage and the
differential I/P voltage Vid only. 35. In the BJT amplifier shown in the figure the transistor is
When the magnitude of Vid is less ‘a’ times the thermal biased in the forward active region. Putting a capacitor
voltage, then a is ‘1’. across RE will [1997]
Hence, the correct option is (d). VCC
32. A multistage amplifier has a low-pass response with +
three real poles at s = −w1, −w2 and w3. The approxi- RL
mate overall bandwidth B of the amplifier will be given Rbias
by [1998] +
(a) S = −w1 + w2 + w3
Vout
Vin
1 1 1 1 RE
(b) = + +
B w1 w 2 w 3 − −
1 (a) decrease the voltage gain and decrease the I/P im-
B = (w 1 + w 2 + w 3 )3
(c)
pedance.
(b) increase the voltage gain and decrease the I/P im-
(d)
B = w 12 + w 22 + w 33 pedance.
(c) decrease the voltage gain and increase the I/P im-
Solution: (b) pedance.
(d) increase the voltage gain and increase the I/P im-
1 1 1 1 pedance.
= + +
B w1 w 2 w 3 Solution: (b)

VCC
Cascading of amplifier results in decrease of ­higher cut- +
off frequency and increase in lower ­cut-off ­frequency.
RL
So, fH ↓ and fL↑ Rbias
Bw = fH − fL +
So, BW↓.
Hence, the correct option is (b). Vout
Vin
RE
− −
5.58 | Analog Electronics

Putting a capacitor across will increase the voltage gain Solution:  (3.75 mA)
and decrease the I/P impedance. 12 V
This capacitor will be by pass Re thus ‘ac’ signals will
pass through capacitor Ce. 1K
Hence, the correct option is (b).

36. A cascade amplifier stage is equivalent to [1997] 1K


(a) a common emitter stage followed by a common 10 K
base stage.
(b) a common base stage followed by an emitter fol-
lower.
(c) an emitter follower stage followed by a common 1K
base stage.
(d) a common base stage followed by a common emit-
ter stage. Ic cannot be 5.32 mA because
Ic = 5.32 mA will make Vcε negative which implies tran-
Solution: (a)
sistor is in saturation.
A cascade connection is where common emitter con-
Through kVL,
figuration is followed by common base.
12 IB + 2 Ic = 11.2
Hence, the correct option is (a).
10 IB − Ic = 0.6
37. A BJT is said to be operating in the saturation Region Upon solving
if [1995] Ic ≅ 3.75 mA
(a) both the junctions are reverse biased. 39. In order to reduce the harmonic distortion in an ampli-
(b) base-emitter junction is reverse biased and base- fier its dynamic range has to be___________ [1994]
collector junction is forward biased. Solution: (Compressed)
(c) base-emitter junction is forward biased and base- In order to reduce the harmonic distortion is an ampli-
collector junction is reverse-biased. fier its dynamic range has to be compressed.
(d) both the junctions are forward biased.
40. A Common Emitter transistor amplifier has a collector
Solution: (b) current of 1.0 mA. When its base current is 25 mA at
Both the junctions are forward biased. the room temperature, its input resistance is approxi-
When emitter base (e-B) junction and collector base mately equal to_______ [1994]
(C-B) junction both are forward biased, then the BJT is Solution: (1 k  )
said to be operating in saturation region. For common emitter configuration
Hence, the correct option is (b). I/P resistance = Ri

38. A transistor having a = 0.99 and VBE = 0.7 V is used in VT 25 × 10 −3


= = = 1 k'
the circuit of the figure. The value of the collection cur- I B 25 × 10 −6
rent will be [1995]
41. The bandwidth of an n-stage tuned amplifier, with each
+12 V
stage having a bandwidth of B, is given by
 [1993]
1K
B B
(a) (b)
n n
1K
10 K B
(c) B 21/ n − 1 (d)
2 −1
1/ n

Solution: (c)
The overall band width of an n- stage turned amplifier
1K is
BWn = B 21/ n − 1
Hence, the correct option is (c).
Chapter 3  BJT Analysis  |  5.59

42. For the amplifier circuit of the figure, the transistor has midband voltage gain magnitude, Av = | v0 / v1 | V /V , is
V0 ____________. [2017]
a b of 800. The mid-band voltage gain of the circuit
Vi VCC = 12 V
will be____ [1993] 73 KΩ 2Ω
RC
R1 10 μF
+15 V +
10 μF C2
C1 RL
V0
V1 2 KΩ C 8 KΩ
470 Ω RE E
200 kΩ 47 KΩ R2
100 μF

+
+ 6.4 µF
4.7 µF Vout Solution:  DC analysis: Capacitors are open circuited
Vin 100 kΩ ac source is short circuited.
− − Vcc R2
− VBE
R1 + R2
Ic =
(a) 0 (b) < 1 (c) ≈ 1 (d) 800 RE
Solution: (c) 12 × 47
− 0.7
I c = 120
+15 V
2 KΩ
I c = 2 mA
470 Ω
200 kΩ Ic 2 mA 2
gm = = =
4.7 µF + Vt 25 mV 25
+
6.4 µF
Vout
12 V
Vin
100 kΩ

− −
2 KΩ
37 KΩ
Vi

100 K 200 K

47 KΩ 2 KΩ
Vo

470 Ω

A.C. analysis:
V Capacitors are short circuited DC source is also short
AV = 0
Vi circuited.
lB (1 + h fe ) Re iC
=
lB [hie + (1 + h fe )]Re + V0
≅1 + +
R2 Vbe 2 KΩ 8 KΩ
Hence, the correct option is (c). Vin ∼ R1 47 KΩ

73 KΩ −

Two-marks Questions
V0 = -ic(Rc || RL)  Vin = nbe
1. For the DC analysis of the common-Emitter amplifier
shown, neglect the base current and assume that the V0
emitter and collector currents are equal. Given that VT Voltage Av = = gm (Rc || RL)
Vin
= 25 mV, VBE = 0.7 V and the BJT output resistance
2
r0 is practically infinite. Under these conditions the \ Av = [ 2 K || 8K ]
25
5.60 | Analog Electronics

(A) Current gain will increase.


2/  2 × 8 K 
\ Av = (B) Unity gain frequency will increase.
25  10/ 5  (C) Emitter-base junction capacitor will increase.
\ Av = 128. (D) Early Voltage will increase.
Hence, the correct answer is (127). Solution:  If Base with in BJT is increased then early
voltage also increases
2. In the figure shown, the npn transistor acts as a switch.
+5 V ⎡ V ⎤
IC = IS ⎢1+ CE ⎥ .eV VT as Base width increases
4.8 kΩ
⎣ VA ⎦
Vin(t)
2V 12 kΩ ⇒ IB↑ and IC↓
+ ⇒ VA↑
Vin(t)
T −
0V
Hence, the correct option is (D).
t (in seconds) C
2
4. In the circuit shown in the figure, the BJT has a cur-
For the input Vin (t) as shown in the figure, the transis- rent gain (β) of 50. For an emitter-base voltage VEB =
tor switches between the cut-off and saturation regions 600 mV, the emitter-collector voltage VEC (in Volts) is
of operation, when T is large. Assume collector-to- _______. [2015]
emitter voltage at saturation VCE(sat) = 0.2 V and base-to- 3V
emitter voltage VBE = 0.7 V. The minimum value of the
common-base current gain (a) of the transistor for the
switching should be ________.  [2017]
Solution:  Given that 60 kΩ
500 kΩ
VCE ( sat ) = 0.2 V
VBE = 0.7
Solution:
2 − 0.7
IB = = 108.33 µA
12 K Ω
5 − 0.2 4.8 8V
IC = = = 1 mA 500 Ω
4.8K Ω 4.8 KΩ E
P
IC 1 mA n
 b = = = 9.2310 B
I B 108.33 µA β = 50

β 9.2310 C
VEB
+ P
  a = = = 0.9022 60 kΩ 60 kΩ
β + 1 1 + 9.2310
500 kΩ
∴ a = 0.9022
+5V
3V

4.8 kΩ Given β = 50 and VEB = 600 mV = 0.6 V


VEB = VE – VB = 0.6 V
12 kΩ +
0.2 But VE = 3 V
+
+ −
0.7
VB = 2.4 V
2V − 2.4
∴ IB = mA 
− 60
IB = 0.04 mA
Hence, the correct answer is (0.89 to 0.91).
IC = β ⋅ IB = 2 mA
3. If the base width in a bipolar junction transistor is dou-
bled, which one of the following statements will be VC = RC ⋅ IC
TRUE? [2015] =
500 × 2 × 10–3
Chapter 3  BJT Analysis  |  5.61

= 1 V
VEC = VE – VC 5 KΩ
R1 33 kΩ
= 3 – 1 Vo
1 µF
= 2 V 1 µF
Vi
Hence, the correct Answer is (2).
5. In the circuit shown, I1 = 80 mA and I2 = 4 mA. R2 11 kΩ
RS 10 kΩ
Transistors T1 and T2 are identical. Assume that the
thermal voltage VT is 26 mV at 27°C. At 50°C, the value
of the voltage V12 = V1 – V2(in mV) is _______ [2015]
RE 1 kΩ C 1 µF
Vs
I2 I1
– +
V2 V1
V12 Solution: (−223.6)

RC = 5 KΩ
T2 T1
R1 = 33 k
Vo
1 µF
1 µF
Vi
Solution:  From the given data
I1 = 80 mA and I2 = 4 mA
R2 11 k
RS = 10 k
VT = 26 mV at 27°C

At 50°C, the value of the voltage V12 = V1 – V2 = ?


We know RE = 1 kΩ C = 1 µF

{
I = Io eV ηVT − 1
} 


{
I≈I e o
V ηVT
}
12 V
12 V
I1
= e( 1 2 ) T
V −V η.V
I2
 5K 5K
We know 33 k
T
VT = 8.25 k
11600 
at T = 273 + 50° = 323° K
323 11 k 3V
1.01 k 1.01 k
VT = = 27.844 mV
11600 
⎛I ⎞
V12 = ηVT ⋅ ln ⎜ 1 ⎟ V
⎝ I2 ⎠
 By applying KVL in input loop, we get
Let η = 1 (NOT given)
3 = 8.25 I B + 0.7 + 1.10 KIe
⎛ 80 ⎞
V12 = 27.844 × 10–3 ⋅ ln ⎜ ⎟
⎝ 4⎠  8.25K 
 2.3 = I e  + 1.01K 
=
83.413 mV  201 
Hence, the correct Answer is (83.5 to 84.0). 2.3 = I e × 1 − 051K
6. For the amplifier shown in the figure, the BJT param-
le = 2.2 mA
eters are VBE = 0 7 V, b = 200, and thermal voltage VT =
25 mV. The voltage gain (v0/vi) of the amplifier is____.
 [2014]
5.62 | Analog Electronics

AC equivalent model of amplifier is given as Solution: (b)


10 V
Ib
Vi Vo RC

R1||R2 bre = 2.27 k bIb 5k VBF = 0.7 V


50 k
5V VCE(sat) = 0.2 V
RB b = 50

0.01 KΩ
Assume the transistor is in active region
5 − 0.7
IB active = = 86 mA
50 k
where re is given by IC active = bIB active
= 50 × 86 mA = 4.3 mA
VT Assume Re = 1 kΩ
re =
Ie 10 V
25 mV
re = = 11.36Ω 4.3 mA
2.2 mA
V0 = −5bI b  (1) 1K

OFF
Vi = 2.27K I b + 0 − 01K ( I b + bI b ) (2) VC = 5.7 V
50 kΩ
From equations (1) and (2), we get 5V

V0 −5K × 200 Vb = 0.7 V


= = −233.6
Vi 4 − 28K
It is in active.
7. In the circuit shown, the silicon BJT has b − 50. Assume Rc = 3kΩ
Assume VBF = 0.7 V and VCE(sat) = 0.2 V. Which one of
the following statements is correct? [2014] 10 V

4.3 mA
10 V
3K
RC
ON
VC = −2.9 V
50 kΩ 50 k
5V 5V
RB
VB = 0.7 V

It is in saturation.
(a) For RC = 1 kΩ, the BJT operates in the saturation Hence, the correct option is (b).
region.
(b) For RC = 3 kΩ, the BJT operates in the saturation 8. Consider the common-collector amplifier in the figure
region. (bias circuitry ensures that the transistor operates in for-
(c) For RC = 20 kΩ, the BJT operates in the cut-off ward active region, but has been omitted for simplicity)
region. Let IC be the collector current, VBE be the base-emitter
(d) For RC = 20 kΩ, the BJT operates in the linear re- voltage and VT be the thermal voltage. Also, gm and ro
gion. are the small-signal transconductance and output resist-
ance of the transistor, respectively. Which one of the fol-
lowing conditions ensures a nearly constant small signal
voltage gain for a wide range of values of RE? [2014]
Chapter 3  BJT Analysis  |  5.63

9. For the common collector amplifier shown in the figure


the BJT has high b, negligible VCE(sat), and VBF = 0 7 V.
The maximum undistorted peak-to-peak output voltage
Vin
vo (in volts) is______. [2014]
Vout
VCC = +12 V
RE

R1 5 kΩ
gm RE << 1
(a) (b) IC RE >> VT
gm ro>1 (d)
(c) VBE>>VT 1 µF
Vi
Solution: (b) 1 µF
R2 10 kΩ Vo
RE 1 kΩ

Vin

Vout
Solution:  (9.4 V)
RE DC analysis
VCC = 12 V

AC equivalent circuit for given common collector am-


plifier is R1 = 5 kΩ
+ 1 µF
Vi
bre bIb
1 µF
R2 = 10 kΩ Vo
Vi Vo RE = 1 kΩ
RE

Applying voltage divider
VCe × R2 12 × 10 KΩ
(VB ) q = =
Vin = b re I b + (1 + b ) I b Re R1 + R2 15 KΩ
V0 = (1 + b ) Re I b =8V
Then, (Ve )Q = VB − 0 − 7
= 8 − 0.7 = 7.3 V
V0 (1 + b ) Re
= (VCe ) = (VC )Q − (Ve )Q
Vin b re + (1 + b ) Re
= 12 − 7.3 = 4.7 V
V0 bRe Re
= =
Vin b re + bRe re + Re

The condition for small signal voltage gain to be nearly 4.7 V
constant is (VCe )q ∆VCe = V0
Re >> re 4.7 V
V
Re >> T
IC
V0 ( p − p ) = 2 × 4 − 7
I C Re >> VT
= 9.4 V
Hence, the correct option is (b).
5.64 | Analog Electronics

10. In the circuit shown below, the silicon npn transistor Q Solution: (d)
has a very high value of b. The required value of R2 in
13.7 v
kQ to produce IC = 1 mA is [2013]
12 k
VCC = 3V
C Vo
100 k
C
R1 = 500 Ω IC
10 k b = 100

Q Vi

R2 RE = 500 Ω
KVL in input loop,

13.7 − ( I C + I B )12k − 100 k ( I b ) − 0.7 = 0


(a) 20 (b) 30 (c) 40 (d) 50 ⇒ I B = 9.9 mA

Solution: (c) I C = b /B = 0.99 mA

VCC = 3V 1e = 1 mA

26 mA
∴ re = = 26Ω
R1 = 60 K
Ie

Zi = bre = 2.5k Ω
Q

 100 k || 12k 
R2 RE = 500 ∴ AV =   = 412
 26

 100 k 
Zi1 = Zi 
 1 + 412 
R2 = 221Ω
3⋅ = 1.2
60 + R2
Zd 1
R2 = 40 k Ω AVS = AV
Zi1 + RS
Hence, the correct option is (c).
 221 
= 412 
11. The voltage gain Av of the circuit shown below is  221 + 10 k 
 [2012]
AVS ≈ 10
13.7 volts
Hence, the correct option is (d).
12 kΩ
1 2. For the BJT Q1 in the circuit shown below, b = ∞, VBEon
= 0.7 V, VCEsat = 0.7 V. The switch is initially closed. At
C Vo time t = 0, the switch is opened. The time t at which Q1
100 Ω
leaves the active region is  [2011]
C

b = 100 5v
10 Ω
Vi
0.5 mA

−5 v
Q1 5 µF
(a) |Av| ≈ 200 (b) |Av| ≈ 100 t=0
(c) |Av| ≈ 20 (d) |Av| ≈ 10
4.3 kΩ
−10 v

Chapter 3  BJT Analysis  |  5.65

(a) 10 ms (b) 25 ms 1 3. The resistance seen by the source Vs is [2010]


(c) 50 ms (d) 100 ms (a) 250 Ω (b) 1258Ω
Solution: (c) (c) 93 kΩ (d) ∞
5v
Solution: (b)
Base current
0.5 mA
10 − 0.7
IB =
93k
−5 v
Q1 5 µF
= 100 mA
t=0
I c = bI B
4.3 k
= 100 × 100 mA
−10 v I e = 10 − 1 mA
Apply KVL at the BE junction 26 mV
ge=
−5 − 0.7 + 10 10 − 1 mA
1e =
4.3 KΩ = 2.57Ω
4.3 b re = 100 × 2.57 = 257Ω
= = 1 mA
4.3 KΩ Ri = RB || b re
Always, 1e = 1 mA = 93K Ω || 257Ω
At collector junction = 257Ω
I cap + (0.5 mA ) = 1 mA (∵ b = ∞; I e = I C )
∴RS (seen form RS) = 1k Ω + 257 Ω = 1257 Ω
Icap = 1 − 0.5 = 0.5mA always constant Hence, the correct option is (b).
VCe = VC − Ve 1 4. The lower cut-off frequency due to C2 is [2010]
⇒ VC = VCe + Ve (a) 33.9 Hz (b) 27.1 Hz
= 0.7 + 4.3K × 1 × 10 −3 (c) 13.6 Hz (d) 16.9 Hz
= 0.7 + 4.3 (∵Ve = 1e Re ) Solution: (b)
Lower cut off frequency is given by
VC = 5V = Vcap
1
t f LC =
Vcap = I cap 2
2p ( R0 + RL )C2
C
1
Vcap (c) 5 × 5 × 10 −6 =
⇒t = = = 50 mA 2p ( 250Ω + 1K Ω)4.7 mF
I cap 0.5 × 10 −3
(∵ R0 ~ RC = 250Ω)
Hence, the correct option is (c).
= 27.1 Hz
Common Data for Questions 13 and 14. Hence, the correct option is (b).
Consider the common emitter amplifier shown below with
the following circuit parameters: 15. A small signal source vi (t) = A cos 20t + B sin 106t is
b = 100, gm = 0.3861 A/V, ro = ∞, rp = 259 Ω, RS = 1 kΩ, RB applied to a transistor amplifier as shown below. The
= 93 kΩ, RC = 250 Ω, RL = 1 kΩ, C1 = ∞ and C2 = 4.7 μF. transistor has b = 150 and hie = 3 kΩ. Which expression
best approximates v0(t)? [2009]
+10 v
12 V
RB RC
100 kΩ 3 kΩ
C2 VD (t )
100 nF
RS C1 + Vi (t )
V0 RL 100 nF
+ −
100 kΩ
VS
− 20 kΩ 900 kΩ 10 µF
5.66 | Analog Electronics

v0(t) = −1500 (A cos 20t + B sin106 t)


(a) 16. The value of DC current IE is [2008]
v0(t) = −150 (A cos 20t + B sin106t)
(b) (a) 1 mA (b) 2 mA
v0(t) = −1500 B sin106t
(c) (c) 5 mA (d) 10 mA
v0(t) = −150 B sin106t
(d) Solution: (a)
Solution: (d) VCC = 9 V

12 V 20 k 3k

100 k 3k CC
2
VD (t )
100 nF CC1 3k
100 nF
Vi (t)

10 k 2.3 k CE
100 kΩ
20 k 900 k 10 µF

Design is independent of IB as b is very high,


Gain
RL 9 × 10
AV = − h fe I B  0, VB = =3V
hie 20 + 10

3 kΩ Thus applying Kirchhoff’s law in base emitter, we get


= −150 ×
3 kΩ 3 = 0.7 + Ie× 2.3 × 103.
AV = −150 ⇒ Ie = 1 mA
∴ V0 = −150 Vi(t) Hence, the correct option is (a).
but Vi (t ) = A cos 20t + B sin 106 t 17. The mid-band voltage gain of the amplifier is approxi-
mately [2008]
where for A cos 20wt ,
(a) −180 (b) −120
20 (c) −90 (d) −60
f =
= 3.18 Hz
2p
Solution: (d)
Since the coupling capacitors block low frequency sig-
nals, A cos 20t will not appear in output for RL ′
(3.53). Mid-band voltage gain = −
10 6 re
B sin 106 t , f =
160 kHz Ie = 1 mA
2p
∴ B sin 106t appear or −150 B sin 106t 25
re = = 25 Ω
Hence, the correct option is (d). Ie
Statement for Linked Answer Questions 16 and 17.
In the following transistor circuit, VBE = 0.7 V, re = 25 mV/IE, RL′ = RC ||RL
and b and all the capacitances are very large. = 3 k||3 k
= 1.5 k
VCC = 9 V
−1500
20 kΩ 3 kΩ Thus, AV = = −60
25
Hence, the correct option is (d).
CC2

CC1
18. The DC current gain (b) of a BJT is 50. Assuming that
IE the emitter injection efficiency is 0.995, the base trans-
10 kΩ 2.3 kΩ CE 3 kΩ port factor is [2007]
(a) 0.980 (b) 0.985
(c) 0.990 (d) 0.995
Chapter 3  BJT Analysis  |  5.67

Solution: (b) B − C junction = 5 V ⇒FB


Relation in alpha and beta is ∴ saturation.
Hence, the correct option is (b).
( b − 1) 49
a = =
b 50 Common Data for Questions 20, 21 & 22.
= 0.98 In the transistor amplifier circuit shown in the figure below,
a = Rb the transistor has the following parameters:
where b = base transport factor bDC = 60, VBE = 0.7 V, hie→∞, hfe→∞
R = emitter injection efficiency The capacitance CC can be assumed to be infinite.
b = 0.98/0.995 12 V
= 0.985
12 V
1 KΩ
Hence, the correct option is (b).
53 KΩ
19. For the BJT circuit shown, assume that the b of the 1K
+
transistor is very large and VBE = 0.7 V. The mode of 5.3 KΩ
53 K IC + IB
operation of the BJT is [2007] Vc
Vc
CC IC

V5.3
s
K
10 KΩ
CC
+ 10 V Vs
- In the above figure, the ground has been shown by the
+
2V - 1 KΩ symbol
20. Under the DC conditions, the collector-to-emitter volt-
age drop is [2006]
(a) 4.8 volts (b) 5.3 volts
(c) 6.0 volts (d) 6.6 volts
(a) cut-off (b) saturation Solution: (c)
(c) normal active (d) reverse active
12 V
Solution: (b)
1K

53 K IC + IB
10 K
Vc
IC
+ 5.3 K
10 V
-
CC
+ Vs
2V - 1K

12 = (IC + IB) 1k + IB × 53k + VBe


11. 3 = (61k + 53k)IB
VBe = 0.7 V IB = 99 μA
VB = 2 V IC = 5.95 mA
∴Ve = VB − VBe = 1.3 V VCe ≈ 12 − (1 × 5.95)
Ve 1.3 ≈6V
Ie = = = 1.3 mA  I C Hence, the correct option is (c).
Re k
21. If bDC is increased by 10%, the collector-to-emitter
∴Vc = 10 − IC Rc voltage drop [2006]
= 10 − (1.3 mA) (10 kΩ) (a) increases by less than or equal to 10%
VC = −3 V (b) decreases by less than or equal to 10%
∴VBC = VB − VC = 2 − (−3) = 5 V (c) increases by more than 10%
∴B − E junction = 0.7 V ⇒FB (d) decreases by more than 10%
5.68 | Analog Electronics

Solution: (b) 23. For an npn transistor connected as shown in the figure,


Now bdc = 66 VBE = 0.7 volts. Given that reverse saturation current of
 11.3  the junction at room temperature 300°K is 10−13 A, the
IB =  mA
 120 
emitter current is [2005]
IC
 11.3 
IC =  × 66
 120 
IC = 6.215 mA
VCe = 12 − 6.215
= 5.78 V VBE
Thus vce decreases less than or equal to 10%.
Hence, the correct option is (b).
(a) 30 mA (b) 39 mA
22. The small-signal gain of the amplifier VC/VS is (c) 49 mA (d) 20 mA
[2006]
(a) −10 (b) −5.3 (c) 5.3 (d) 10 Solution: (c)
Solution: (b)
1e = 6 mA IC
26 mV 26 mv
∴ re = = = 4.33 Ω +
Ie 6 mA
VBE −
−( Re || z2 )
Now An =
re
Since collector is connected to base, it is acting as for-
53K ward bias diode
where Z 2 =  53 kΩ
1
1−
AV ∴ I e = I s [eVBE /hVT − 1]

using Miller’s theorem, = 10 −13 [e 0.7 /1× 26 mv − 1]

−(1k || 53k ) = 49 mA
Av = Hence, the correct option is (c).
4.33
= −226.5 24. In an ideal differential amplifier shown in the figure, a
large value of (RE) [2005]
zi
Now AVs = AV VCC
zi + Rs
where zi = z1||bre RC RC

53 k
Z1 = (using Miller’s theorem)
1 − AV

53 k V1 V2
Z1 = = 233 Ω
227.5 RE
233(60 × 4.33)
∴ Z1 = −VEE
233 + (60 × 4.33)
= 122.8 Ω (a) increases both the differential and common-mode
gains.
 122.8 
⇒ Ans = −226.5  (b) increases the common-mode gain only.
 122.8 + 5.3 k  (c) decreases the differential-mode gain only.
   = −5.13 (d) decreases the common-mode gain only.
Ans  −5.13

Hence, the correct option is (b).


Chapter 3  BJT Analysis  |  5.69

Solution: (d) Emitter current is given by


VCC Ie = IC + IB
= (b + 1) IB
RC RC z0 = 430 × IB + 0.7 + 51 × IB
19.3 = 481 IB
V1 V2 IB = 40.1 μA
IC = bIB
= 50 × 40.1 μA
RE = 2 mA
Collector voltage is given by
−VEE VC = VCC − ICRC
Common mode gain is given by = 20 − (2 mA × 2 kΩ)
−2 RC = 16 V
ACM =
Re Hence, the correct option is (b).
Differential gain is given by 2 6. Assuming that the b of the transistor is extremely large
and VBE = 0.7 V, IC and VCE in the circuit shown in the
1 figure are [2004]
Ad = gmRC
2 5V
Large value of Re will decrease common mode gain and IC
differential gain is not affected.
Hence, the correct option is (d). 4 kΩ 2.2 kΩ

25. The circuit using a BJT with b = 50 and VBE = 0.7 V +


is shown in the figure. The base current IB and collector VCE
voltage VC are, respectively, [2005] −
1 kΩ
20 V 300 Ω
430 kΩ 2 kΩ
VC

10 µF (a) IC = 1 mA, VCE = 4.7 V


(b) IC = 0.5 mA, VCE = 3.75 V
1 kΩ 40 µF (c) IC = 1 mA, VCE = 2.5 V
(d) IC = 0.5 mA, VCE = 3.
Solution: (c)

(a) 43 mA and 11.4 volts 5V


(b) 40 mA and 16 volts
(c) 45 mA and 11 volts IC
(d) 50 mA and 10 volts 4k 2.2 k

Solution: (b)
20 V A

430 k 2k 1k
300
Vo

10 µF

Vi 1k 40 µF Applying voltage divider at node A


5 ×1
VA = =1V
5
5.70 | Analog Electronics

1 − 0.7 0.3 28. In the circuit of the figure, assume that the transistor is
Ie = = = 1 mA in the active region. It has a large p and its base-emitter
300 300
voltage is 0.7 V. The value of Ic is  [2000]
IC = Ie as b is large
⇒IB = 0
15 V
∴Vce = 5 − 1(2.2 + 0.3)
= 2.5 V
10 Ω RC
Hence, the correct option is (c).
lC
27. In the amplifier circuit shown in the figure, the values
of R1 and R2 are such that the transistor is operating at
VCE = 3V and IC = 1 5 mA when its b is 150. For a tran-
sistor with b of 200, the operating point (VCE, IC) is 5Ω 430 Ω
 [2003]
VCC = 6 V

R1 R2
(a) Indeterminate since Rc is not given
(b) 1 mA
(c) 5 mA
(d) 10 mA
Solution: (d)

15 V
(a) (2 V, 2 mA)
(b) (3 V, 2 mA)
(c) (4 V, 2 mA) 10 kΩ RC
(d) (4 V, 1 mA) IC

Solution: (a) B
VCC = 6 V IE

R2 5 kΩ 430 Ω

Applying potential divider at B


At the collector
15 × 5
V − Vce VB = =5V
Rz = cc 5 + 10
Ic
Since b is large, I B  0,
6−3
= Ve = VB − 0.7
1.5 × 10 −3
= 2 kΩ = 5 − 0.7
Ic = 4.3 V
IB = = 0.01 mA Hence, the correct option is (d).
b
29. A Darlington stage is shown in the figure. If the
b = 150 transconductance of Q1 is gm1 and Q2 is gm2, then the
b changes to 200, IC = b × IB = 200 × 0.01 mA overall transconductance given by gm is given by
= 2 mA  [1996]
Vce = 6 − 2m × 2 kΩ = 2 V
Hence, the correct option is (a).
Chapter 3  BJT Analysis  |  5.71

VCC 4. T
 he base doping concentration is increased keep-
ing the ratio of the emitter doping concentration to
IC bade doping concentration constant.
Q1 5. The collector doping concentration is reduced.

Q2 Solution: (A−2, B−3, C−5)


As the base width of the BJT is reduced, then the re-
combination current (base current IB) decreases as a re-
Vbe sult collector current (Ic) increases. So, the gain of the
BJT increases.
(a)
gm1 (b) 0.5 gm1
Ic
(c)
gm2 (d) 0.5gm2 a = .
Ie
Solution: (d) If the emitter doping concentration to base doping con-
I centration ratio is reduced, then the emitter injection
gm = e 2
Vbe efficiency decreases, so the current gain (a) of BJT re-
duces.
Overall transconductance If the collector doping concentration is increased then
Ie2 the breakdown (VBR) of a BJT will be reduced.
gm =
2Vbe 32. a cut-off frequency of a bipolar junction transistor
 [1993]
1 Ie2 1
= = gm 2 (a) increases with the increase in base width.
2 Vbe 2
(b) increases with the increase in emitter width.
gm = 0.5 gm2 (c) increases with the increase in collector width.
Hence, the correct option is (d). (d) increases with decrease in the base width.

30. Match the following. [1996] Solution: (d)


(a) Cascade amplifier As base width decreases, recombination decreases. So
(b) Differential amplifier collector current Ic increases
(c) Darlington pair common-collector amplifier
(1) Does not provide current gain Ic
a = .
(2) Is a wide band amplifier Ie
(3) Has very low input impedance emitter amplifier
So, a also increases.
and very high current gain
Hence, the correct option is (d).
(4) Has very high input impedance and very high cur-
rent gain 33. If the transistor in the figure has high value of b and VBE
(5) Provides high common mode voltage rejection of 0.65, the current I flowing through the 2 kilo ohms
resistance will be________ [1992]
Solution: (a−2, b−5, c−4)
Cascade amplifier−It provides a wide band amplifier. I
Differential amplifier−Provides high common mode 2 KΩ
6.5 KΩ
voltage rejection.
Darlington pair common-Collector amplifier−Has very
high I/P impedance and very high current gain
31. Match the following: [1994] +
1.85 KΩ 10 V
List-I −
A. The current gain of a BJT will be increased.
B. The current gain of a BJT will be reduced.
C. The break-down voltage of a BJT will be ­reduced.
1.65 KΩ
List-II 1 KΩ
1. The collector doping concentration is ­increased.
2. The base width is reduced.
3. The emitter doping concentration to base doping
concentration ratio is reduced.
5.72 | Analog Electronics

Solution: 
(1 mA) Solution:  (5 V)
I
Given that b is very large
10 V
2 KΩ
6.5 KΩ

10 V 5 mA 1 KΩ

VDC = ?
1.85 KΩ

1.65 KΩ
1 KΩ

So, Ic = Ie
Given that b is very large, So, the current through 1 kΩ
so IB = 0 resistance
10 × 1.65 I = 5 mA
So, V1.65 =
1.65 + 1.85 + 6.5 VDC = 10 − IR
= 1.65 V = 10 − 5 × 10−3 × 1 × 103
VDC = 5 V.
Apply KVL at I/P mesh
V1.65 = Vbe+ IeRe 35. For good stabilized biasing of the transistor of the CE
= 0.6 + Ie× 1k amplifier of figure, we should have [1990]
= 1.65 + VCC

Ie = 1 mA RC
R2
b is very large
so, Ic≅Ie +
+
I = Ic = 1 mA
R1 || R2 = RB
Vin R1 Vo
34. In the figure all transistors are identical and have a RE
high value of beta. The voltage VDC is equal to______. − −
 [1991]
10 volts R
(a) E << 1
RB

R
5 mA 1 KΩ (b) E >> 1
RB
VDC = ?
RE
(c) << hFE
RB
RE
(d) >> hFE
RB

Solution: (b)
R1||R2 = RB
Chapter 3  BJT Analysis  |  5.73

VCC b
S=
RC  Re 
b
 RB + Re 
Rth
RB
+ S = 1+
Re
Vbe −
Vin Re For better stability, S ≅ 1
RB
So, << 1
Re
+ VCC Re
>> 1.
RB
R2 RC
Hence, the correct option is (b).
+
+
36. Which of the following statements are correct for basic
transistor amplifier configurations?
R1
[1990]
Vin Vo
RE (a)  CB amplifier has low input impedance and low cur-
− −
rent gain.
(b) CC amplifier has low output impedance and a high
Simplified self bias circuit using Thevenin’s theorem. current gain.
Thevenin open circuit voltage (c)  CE amplifier has very poor voltage gain but very
Vcc R1 high input impedance.
Vth = (d) The current gain of CB amplifier is higher than the
R1 + R2
current gain of CC amplifiers.
Thevenin interval resistance Solution:  (a, b)
R1 R2 In common base (CB) amplifiers I/P impedance (zi) is
Rth = R1 || R2 = = RB low and current gain (a) is also low and current gain
R1 + R2
(a) is also low.
Applying KVL to I/P mesh I
Vth = IBR + Vbe + IeRe a = c.
Ie
Put Ie = IBR + Vbe + IeRe
In common collector (CC) amplifiers O/P impedance
Vth = IBR + VB RB + Vbe + (IB + IC) Re (z0) is low and current gain (g) is high
Differentiate wrt Ic, keeping B and Vbe constant Ie
g = = 1+ b
∂I B Ib
0 = ( RB + Re ) + Re + 0
∂I c Hence, the correct options are (a, b).
∂I B − Re 37. Of the four biasing circuits shown in figure, for a BJT,
=
∂I C RB + Re indicate the one which can have maximum bias stabil-
ity [1989]
1+ b
S=
∂I VCC
VCC VCC
1− b b
∂I c
1+ b VCC
S=
 Re 
1+ b 
 RB + Re 
b >> 1
Re
b >> 1
RB + Re
(a) (b) (c) (d)
5.74 | Analog Electronics

Solution: (a) So, overall b of the composite transistor


In the BIT self-bias circuit or potential circuit provides
the maximum bias stability. IC 2
b′ = = b (1 + b )
I B1
VCC
= 100 × (1 + 100) = 10,100

Hence, the correct option is (c).


40. The amplifier circuit shown below uses a composite
transistor of a MOSFET and BIPOLAR in cascade. All
capacitance are large. gm of the MOSFET = 2 mA/V, and
hfe of the BIPOLAR = 99. The overall transconductance
gm of the composite transistor is  [1988]

38. The quiescent collector current IC of a transistor is


increased by changing resistances. As a result
 [1988]
C
(a) gm will not be affected C
(b) gm will decrease
(c) gm will increase
(d) gm will increase or decrease depending upon bias
stability. Vin

Solution: (c)
Ic (a) 198 m A/V (b) 9.9 m A/V
gm = (c) 4.95 m A/V (d) 1.98 m A/V
VT
Solution: (d)
So, if Ic↑ then gm↑
gm∝Ic
So, if the quiescent collector current Ic increases then
transconductance gm also increases.
Hence, the correct option is (c).
39. Each transistor in the Darlington pair (see the figure
below) has hFE = 100. The overall hFE of the composite C
transistor neglecting the leakage currents is [1988] C
C
Vin

B
Vin = Vgs
1c
gm ′ =
Vgs
E

(a) 10,000 (b) 10,001 b


I c = aI e = Ie
(c) 10,100 (d) 10,200 1+ b
Solution: (c)  b  Ie bI e
gm ′ =   =
 1 + b  Vgs (1 + b )Vgs
I e 1 = (1 + b ) I B1
ID = Ie
I e 1 = (1 + b ) I B1 b I b
gm ′ = × D = gm
= I B2 1 + b Vgs 1 + b
I c 2 = bI B 2 99
= b (1 + b ) I B1 = × 2 mA/V
1 + 99
Chapter 3  BJT Analysis  |  5.75

gm′ = 1.98 mA/V 42. The conf iguration of cascade amplif ier is
Hence, the correct option is (d).  [1987]
41. The transistor in the amplifier shown below has the fol- (a) CE−CE (b) CE−CB
lowing parameters: (c) CC−CB (d) CC−CC
hfe = 100, hie = 2 kΩ, hre = 0, hoe = 0.05 mhos. C is very Solution: (b)
large. The output impedance is [1988] Cascade amplifier is the common emitter followed by
VCC common base configuration.
Hence, the correct option is (b).
58 k 5k

C
V0 Five-marks Questions
100 nF 1. An emitter-follower amplifier is shown in the figure, Zi
is the impedance looking into the base of the transistor
10 k and Z0 is the impedance looking into the emitter of the
1k C
transistor [2001]
Vcc
(a) 20 KΩ (b) 16 KΩ
(c) 5 KΩ (d) 4 KΩ Rc

Solution: (d)
Rs
VCC
Z0

5k + V0
58 k
Vs Zi
V0 RL
C −
100 nF
−VEE
10 k
1k C (A) Draw the small signal equivalent circuit of the am-
plifier.
(B) Obtain an expression for Zi.
Output admittance, (C) Obtain an expression for Z0.
h fe hre
y0 = h0 e − Solution:
hie + Rs (A)  Replace all the sources with internal impedance
and short circuit all cap then draw small signal h
model.

1/h0e 5k Iin Ib
10 k 58 k

hie bIb
+ Ia
Vin RB
Z01 = 20 k Z0 − +
RE V
IE −
0

100 × 0
= 0.05 × 10 −3 −
2 × 103 + 10 × 103 IE = Ib + bIb = (1 + b)Ib
y0 = 0.05 × 10−3 Vin = Ibhie + (1 + b) IbRE [KVL in Loop 1]
1 1 V0 = (1 + b) I0IE
z01 = = = 20 k'
y0 0.05 × 10 −3
V0 (1 + b ) I b RE
AV = =
Output impedance, Vin I b [hie + (1 + b ) RE ]

z0 = z01|| 5k = 20k||5k = 4 kΩ (1 + b ) RE
=
Hence, the correct option is (d). hie + (1 + b ) RE
5.76 | Analog Electronics

∵ (1 + b)RE < hie Solution:


AV = 1 V0 − RC
(A) Voltage gain = AV = =
(B) Input Impedance Zin Vi Re
Iin + Ia + Ib gm
re =
Ie
Vin
Z in =
I in  1+ b  1 + 100
Ie =  IC = × 1.3 mA
Vin Vin  p  100
I in = +
RB hie + (1 + b ) RE Ie = 1.313 mA
Vin 1 26 × 10 −13  VT 
Z in = = re = = 19.8 Ω  re = 
Z in 1 1 1.313 × 10 −13  Ie 
+
RB hie + (1 + b ) RE
− RC
AV = = −101
(C) Output impedance Z0 rc
Short circuit the input source and place 1 V source (B) Voltage gain AV when Ce is removed
at V0 and find I0.
Apply KCL − Re
AV =
Ib + bIb + I0 = IE
Re
AV = −4
 1 1
(1 + b )  −  + I 0 = (C) Output voltage, V0
 hie  RE
V0 = A0 · VS
1 (1 + b ) V0 = −1.01 sin wt
I0 = +
RE hie
3. A bipolar junction transistor amplifier circuit is shown
1 in the figure is. Assume that the current source Ibias is
Z0 =
I0 ideal, and the transistor has very large b, rb = 0, and r0
→ ∞. Determine the ac small-signal midband voltage
1 gain (Vo/Vs). input resistance (R1), and output resistance
Z0 =
1 1+ b (R0) of the circuit. Assume VT = 26 mV.
+
RE hie
Vcc
Ibias
2. For the amplifier of given figure, lc = 1.3 mA, Rc = 2 kΩ, 0.5 mA
C2 = ∞
RE = 500 Ω, VT = 26 mV, b = 100, VCC =15 V, Vs = 0.01 V0
sin(wt) and Cb = Ce = 10 μF. R0
RB 100 kΩ RL 1 kΩ
+Vcc

R1 RC
IC
Cb V0 50 Ω
C1 = ∞
Rs
+
R1 Vs

Vs R2
RE CE
[1999]
4. In the circuit of figure. Determine the resistance R0
seen by the output terminals ignore the effects of R1
and R2.
(A) What is the small-signal voltage gain, Av = Vo/Vs?
(B) What is the approximate Av if Ce is removed?
(C) What will Vo be if C b is short circuited?
 [2000]
Chapter 3  BJT Analysis  |  5.77

+Vcc VCC = 9 V

RL
R1 R1
+
1k VC2
b = 99
re = 1 k 10 µF

R2 +
Vs R2 1k R0 VC1

R2 = 18 K
[1998] RE = 200 Ω

Solution: 
Replace transistor with h – parameter mod-
el.
[1997]
Ib
Solution:  Q current = IC = 1 mA
1k 99Ib
b is very large ∴ Ib = 0
+ VCC = 9 V
1k I0
− +
I2 1k 1V 1 mA
− +
R1 VR1 RL
− VC2 = 6 V

1V R2
R0 = VC1 = 3 V
I0

+ +
0 −1 Vbe
I0 = = −0.5 mA 18 K VR3 −
2k Re Ie = 1 mA

Ib + 99 Ib + I0 = IL = 100 Ib + I0 = 1 mA
I0 = 51 mA

1
R0 = = 20 Ω VCC − VC2 9−6
51 mA RL = = = 3 kΩ
1 mA 1 mA

5. In the cascade amplifier circuit shown below, deter- Apply KVL


mine the values of R1 R2 and RL. Such that the quies-
cent current through the transistors is 1 mA and the VCC = VR1 + Vbe + VC1 = VR1 + 0.7 + 3
collector voltage Vc1 = 3 V, and Vc2 = 6 V. Take VBE =
0.7 V. Transistor b to be high and base currents to be VR1 = 5.3V
negligible
5.78 | Analog Electronics

Apply KVL +VCC = 5 V

VRS = Vbe + I e Re = 0.9V RC


+
VR3 Rs − Vo
I R3 = = 50 m Amp
R3
Vs
∴ Ib = 0 RE CE
I R1 = I R2 = I R3 = 50 m Amp

VR1 = I R1 × R1 The N/W components have following values, Rc =



2 kΩ, Rs = 1.4 k Ω, RE = 100 Ω. The transistor has
VR1 specifications, b = 100 and base spreading resistance
R1 = = 106 kΩ
I R1 rbb′ = 100Ω.

VCC = VR1 + VR2 + VR3 KT
Assume = 25 mV.
q
VR2 = VCC − VR1 − VR3 = 2.8 Evaluate input resistance R1, for two cases. At a fre-

quency of 10 kHz.
VR3 = R2 I R2
(A) CE, the bypass capacitor across RE is 25 μF.
(B) The bypass capacitor CE is removed leaving RE un-
2.8
∴ R2 = = 56 kΩ bypassed. [1997]
50 × 10 −6

6. The transistor in the circuit shown in the figure is so


biased (dc biasing N/W is not shown) that the dc col-
lector current Ic = 1 mA. Supply is Vcc   = 5V
Chapter 4
FET and MOSFET Analysis
Solution:
One-mark Questions
C
1. The correct circuit representation of the structure
shown in the figure is [2019]
B
B E C E

n++ Hence the correct option is (B)


p+ 2. The figure shows the high-frequency C-V curve of a
n++
MOS capacitor (at T = 300K) with fms = 0 V and no
n
oxide charges. The flat-band, inversion, and accumu-
n+ lation conditions are represented, respectively, by the
points [2019]
(A) C C

P Q
B
R

E 0 VG
(B) C
(A) Q, R, P (B) P, Q, R
(C) Q, P, R (D) R, P, Q
B Solution:
C
P
E Q
(C) C
R

B 0
VG

Vus -ve ⇒ accumulation ∴ P is accumulation


E
Vus inverses ⇒ inversion layer ∴ R is inversion
(D) C Hence, the correct option is (A)
3. A standard CMOS inverter is designed with equal rise
B and fall times (bn = bp). If the width of the pMOS tran-
sistor in the inverter is increased, what would be the
effect on the LOW noise margin (NML) and the HIGH
E noise margin NMH? [2019]
5.80 | Analog Electronics

(A) NML decreases and NMH increases. VD D


(B) No change in the noise margins.
(C) Both NML and NMH increase.
Vbias
(D) NML increases and NMH decreases. M2
Solution: 
VDD

G M1
Vin Vout
S

Which of the following statements about estimates for


gm and r0 is correct? [2018]
Given, rise time = full time
(A) gm ≈ gm1 ⋅ gm2 ⋅ r02 and r0 ≈ r01 + r02
Bn = BP
(B) gm ≈ gm1 + gm2 and r0 ≈ r01 + r02
NK L = VIH − VOL (C) gm ≈ gm1 and r0 ≈ r01 ⋅ gm2 ⋅ T02
NM H = VOH − VIH (D) gm ≈ gm1 and r0 ≈ r02
VOH = VDD Solution:  Consider the figure given below
VOL = O VD

VDD + Vtop + K r ( 2VO + VTOP )


VIH = Vbias ID2
1 + Kr M2
KP
Kr = 1 +
Kn
ID1
2VO − | Vtop | − VDD + K rVTON G
VIL = M1
1 + Kr

S
as width increases KP increases & VIL increases, VIH ↑.
Thus increasing NML & decreasing NMH. ∂iD ∂i
gm = = D1 ; gm1
∂vGS ∂vGS
Hence, the correct option is (D).
4. Two identical nMOS transistors M1 and M2 are con- Ix
nected as shown below. The circuit is used as an
amplifier with the input connected between G and S
terminals and the output taken between D and S termi- +
nals. Vbias and VD and so adjusted that both transistors Vgs2 gm2Vgs2 r02
are in saturation. – +
The transconductance of this combination is defined as Vx
∂i Ix –
g m = D while the output resistance is
∂vGS r01
∂v
r0 = DS , where iD is the current flowing into the
∂iD

drain of M2. Let gm1, gm2 be the transconductances and
r01, r02 be the output resistances of transistors M2 and By Applying kirchoffs Voltage law in given circuit we
M2, respectively. get
Chapter 4  FET and MOSFET Analysis  |  5.81

Vx = (Ix – gm2Vgs2) r02 + Ix r01 (B) If the threshold voltage is reduced, the resistance
decreases.
Vx = Ixr02 – gm2Vgs2 r02 + Ix r01
(C) If the device length L is increased, the resistance
Vx = Ixr02 – gm2Ix r01 r02 + Ix r01 increases.
\ r0 = Vx/Ix = r01 + r02 + r01 r02gm (D) If VGS is increased, the resistance increases.
Solution:  Drain to source resistance of NMOS transis-
r0 ≈ r01 r02 gm
tor iVDS = 50 mV
Hence, the correct option is (C)
1 )
5. Consider an n-channel MOSFET having width W, rds =
w
length L, electron mobility in the channel mn and oxide μn ⋅ cox ⋅ (VGs − VT )
L
capacitance per unit area Cox. If gate-to-source voltage
rds = vds/ids
VGS = 0.7 V, drain-to-source voltage VDS = 0.1 V, ( µ nCox ) ⋅
1
= 100 mA/V2, threshold voltage VTH = 0.3 V and (W/L) rds ∝
VGs
= 50, then the trans conductance gm (in mA/V) is 
_____________. Therefore, as VGS increases, rds decreases,
 [2017] Hence, the correct option is (D).
6. An n-channel enhancement mode MOSFET is ­biased 9. Transistor geometries in a CMOS inverter have been
at VGS > VTH and VDS > (VGS − VTH ) , where VGS is the adjusted to meet the requirement for worst case charge
gate-to-source voltage, VDS is the drain-to-source and discharge times for driving a load capacitor C. This
voltage and VTH is the threshold voltage. Considering design is to be converted to that of a NOR circuit in the
channel length modulation effect to be significant, the same technology, so that its worst case charge and dis-
MOSFET behaves as a [2017] charge times while driving the same capacitor are simi-
(A) Voltage source with zero output impedance lar. The channel lengths of all transistors are to be kept
(B) Voltage source with non-zero output impedance unchanged. Which one of the following statements is
(C) Current source with finite output impedance correct? [2016]
(D) Current source with infinite output ­impedance
VDD VDD
7. Consider the following statements for a Metal Oxide
Semiconductor Field Effect Transistor (MOSFET):
In 2
P:  As channel length reduces, OFF state current In Out In 1
­increases. Out
Q:  As channel length reduces, output resistance C
C
­increases.
R:  As channel length reduces, threshold voltage
­remains constant. (A) Widths of PMOS transistors should be ­doubled,
S: As channel length reduces, ON current increases. while widths of NMOS transistors should be
halved.
Which of the above statements are incorrect?
(B) Widths of PMOS transistors should be ­doubled,
 [2016]
while widths of NMOS transistors should not be
(A) P and Q (B) P and S
changed.
(C) Q and R (D) R and S
(C) Widths of PMOS transistors should be halved,
while widths of NMOS transistors should not be
Solution:  As we know that the drain current of MOS-
changed.
⎛W ⎞ ⎡ VDS 2 ⎤
FET is I d = μ nCox ⎜ ⎟ ⎢(VGS − Vt ) VDS −
2
(D) Widths of PMOS transistors should be ­unchanged,
⎝ L⎠⎣ 2 ⎥⎦ while widths of NMOS transistors should be
But r ∝ Id si halved.
Hence, the correct option is (C). Solution:  From the given data
8. A long channel NMOS transistor is biased in the linear I D 2 = 2 I D1 

region with VDS = 50 mV and is used as a resistance.
⎛W ⎞ ⎡ VDS ⎤ 2
Which one of the following statements is NOT cor- Id = μ nCox ⎜ ⎟ ⎢(VGS − Vt ) VDS −
2

rect? [2016]
⎝ L⎠⎣ 2 ⎥⎦ 
(A) If the device width W is increased, the resistance ID ∝ W
decreases.
5.82 | Analog Electronics

i.e., the width will increase with increase in drain 1 1 + λ (5 )


current. =
1.02 1 + λ (6 )
So width of PMOS transistors should be doubled, and 
widths of NMOS transistors should not be changed. ⇒ 1 + 6 λ = 1.02 + 5.1 λ
Hence, the correct option is (B). 0.9 λ = 0.2
10. The figure shows the band diagram of a Metal Oxide λ = 0.022 V–1
Semiconductor (MOS). The surface region of this
MOS is in [2016] Hence, the correct Answer is (0.018 to 0.026).
1 2. In the circuit shown, both the enhancement mode
NMOS transistors have the following characteristics:
kn = µnCox(W/L) = 1 mA/V2; VTN = 1 V. Assume that
the channel length modulation parameter λ is zero and
body is shorted to source. The minimum supply volt-
SIO2
age VDD (in volts) needed to ensure that transistor M1
EM operates in saturation mode of operation is ________.
ϕB
Ec
 [2015]
ϕB EFS VDD
Ei

EV
M2

(A) inversion (B) accumulation


2V M1
(C) depletion (D) flat band
Solution:  The given figure shows the band diagram of
a MOS where the Fermi level is lying just below the Solution:  From given circuit
conduction band with Fermi voltage of fB. this fB de-
cides that surface region of this MOS is in inversion. VDD =VD2 = VG2
Hence, the correct option is (A). The transistor M1 to be operated in saturation region
11. The current in an enhancement mode NMOS transistor So VDS =VGS − Vth 
biased in saturation mode was measured to be 1 mA at
a drain-source of 5 V. when the drain-source voltage For M1 transistor VS = 0
was increased to 6 V while keeping gate-source volt-
VS – VD = VS – VG – Vth
age same, the drain current is increased to 1.02 mA.
Assume that drain to source saturation voltage is
⇒ VD = VG + Vth
much smaller than the applied drain-source voltage.
The channel length modulation parameter λ (in V–1) is =
2 + 1
________. [2015]
=
3 V
Solution:  From the given data
Hence, the correct Answer is (2.9 to 3.1).
NMOS transistor is in saturation mode
13. A MOSFET in saturation has a drain current of 1 mA
So ID = K [VGS – VT]2 (1 + λ .VDS) for VDS = 0.5 V. If the channel length modulation coef-
ID = 1 mA at VDS = 5 V ficient is 0.05 V–1, the output resistance (in k Ω) of the
MOSFET is _____ [2015]
ID = 1.02 mA at VDS = 6 V
Solution:  Under channel modulation
From the given data ID = IDsat (1 + λVDS)
VGS1 – VT = VGS2 – VT
Output resistance
∴ ID = K1(1 + λ.VDS) ∂ VDS
ro =
I D1 1 + λ VDS1 ∂ ID
= 
I D 2 1 + λVDS2 ∂ ID
 = I Dsat λ
∂ VDS

Chapter 4  FET and MOSFET Analysis  |  5.83

1 1 Solution:  VD = VG so FET is operating in saturation


⇒ ro = =
λ I Dsat 0.05 × 1 × 10 −3 So VGS = VSS – VDD

105 kn′
= = 20 kΩ ⇒ ID = (V – Vth)2
5  2 GS
Hence, the correct Answer is (19 to 21). kn′
1 4. For the MOSFET in the circuit shown, the threshold ID =
(V – VDD – Vth)2
2 SS
voltage is Vth, where Vth > 0. The source VSS is varied k′
from 0 to VDD. Neglecting the channel length modula- If VSS = 0 then ID is max ID= n (VDD + Vth)2 and ID ∝
(VSS – VDD – Vth)2 2
tion, the drain current ID as a function of VSS is repre-
sented by [2015] So, it varies non-linearly.
VDD Hence, the correct option is (A).
1 5. In MOS capacitor with an oxide layer thickness of 10
nm, the maximum depletion layer thickness is 100 nm.
The permittivities of the semi conductor and the oxide
layer are εs and εox respectively. Assuming εs/εox = 3,
the ratio of the maximum capacitance to the minimum
VSS capacitance of this MOS capacitor is ______. [2015]
Solution:  From the given data
tox = 10nm
(A) ID
The maximum depletion thickness = 100 nm
εs
=3
ε ox



Cmax
=
(ε ox tox )
VSS
VDD – Vth Cmin ε ε
ox
× s
tox Dmax
(B) ID
ε ox ε
+ s
tox Dmax

Cmax ε ox Dmax
∴ ≈1+
C tox ε s
min 
VSS
Vth 1 100 × 10 −9
=1+ × = 4.33
3 10 × 10 −9
(C) ID
Hence, the correct Answer is (4.3 to 4.4).
16. In the following circuit employing pass transistor logic,
all NMOS transistors are identical with a threshold
voltage of 1 V. Ignoring the body-effect, the output
voltages at P, Q and R are, [2014]
VSS
VDD – Vth 5V 5V 5V

(D) ID
5V

P Q R′

VSS (a) 4 V, 3 V, 2 V
VDD – Vth (b) 5 V, 5 V, 5 V
(c) 4 V, 4 V, 4 V
(d) 5 V, 4 V, 3 V
5.84 | Analog Electronics

Solution: (c) 18. Two identical FETs, each characterized by the param-


5V 5V 5V eters gm and rd, are connected in parallel. The composite
FET is then characterized by the parameters.[1998]

5V g
(a) m   and 2rd
2
P Q R
g r
(b) m and d
2 2
VT = IV
For proper operation rd
(c) 2gm and
VDS = VGS − VT 2
(d) 2gm and 2rd
VD − VS = VG − VS − VT
⇒ VD = VG − VT Solution: (c)
At p G D
+
⇒ VD = VG − VT
Vgs gm Vgs rd
= 5 −1

VD = 4V (at P)

VD at Q = VG − VT If two FETs are connected in parallel,

= 5 −1 = 4 V rd
rd′ =
VD at R = VG − VT 2
gm ′ = 2 gm
= 5 −1 = 4 V
∴ VD ( P ) =4V G
+
D

VD ( S ) = 4 V Vgs gm Vgs gm Vgs rd rd


VD ( R ) = 4 V

5
Hence, the correct option is (c).
17. An n channel depletion MOSFET has following two G D
points on its ID − VGS curve: +
(i) VGS = 0 at ID = 12 mA and Vgs 2gm Vgs rd/2
(ii) VGS = −6 volts at Z0 = ∞
Which of the following Q-points will give the highest −
trans-conductance gain for small signals? [2006] 5
(a) VGS = −6 V
Hence, the correct option is (c).
(b) VGS = −3 V
(c) VGS = 0 V 19. In the MOSFET amplifier of the figure, the signal out-
(d) VGS = 3 V put V1 and V2 obey the relationship [1998]
Solution: (d)
Since given device is depletion MOSFET n-­channel the
RD
current will flow even for positive values of VGS. We
+
can see that for positive value of VGS slope will be more.
Hence current option is VGS = 3 V. + Ω1
+ V2 −
ID V1 RD /2 −

12 mA
V2 V
(a)
V1 = V1 = − 2
(b)
VGS 2 2
−6V
V1 = 2V2 (d)
(c) V1 = −2V2
Hence, the correct option is (d).
Chapter 4  FET and MOSFET Analysis  |  5.85

Solution: (c) Vout

Vdd
RD
+ Vdd
V1 2
+
+ V2 −
Vi RD /2 −
− Vdd Vdd
V1 = Vin
2

V1 = 2V2 Solution: 
Vdd = 3 V
V1 = I D RD
mnCox = 100 m A / V 2
IS = ID
Vtn = 0.7 V
RD
V2 = I S
2 m p Cox = 40 m A / V 2
I D RD V1
= =
2 2 | Vtp | = 0.9 V

So, V1 = 2V2 using the current equation we get
Hence, the correct option is (c).
1 W
( )
2
20. An n-channel JFET has IDSS = 1 mA and VP = −5 V. Its I DS = mC V − Vt
2 n ox L mS
maximum transconductance is_______  [1995]
Solution: (0.4 ms) 1 W 
( ) 1 W 
( )
2 2
mC V −V = m C V − Vtp
2 n ox  L  n ms tn 2 p ox  L  p mS
2 I DSS
g m ,max =
VP W 
 L  40 × (1.5 − 0.9)
2
n
2 × 1 × 10 −3 =
= = 0.4 ms W  100 × (1.5 − 0.7)
2
−5  L 
p

21. The transit time of current carriers through the channel of W 


an FET decides its ______ ­characteristics. [1994]  L 
n 4×9
Solution: (switching) = = 0.225
W  16 × 10
The transit time of current carriers through the channel  L 
of an FET decides its switching characteristics. p

Hence, the correct answer is (0.225).


Two-marks Questions 2. In the circuit shown, the threshold voltages of the pMOs
1. A CMOS inverter, designed to have a mid-point volt- ( )
Vtp and nMOS (Vtn) transistors are both equal to 1 V.
age V1 equal to half of Vdd. As shown in the figure, has All the transistor have the same output resistance rds of
the following parameters: 6 MW. The other parameter are listed below.
Vdd = 3 V
W
mnCox = 100 mA/V2 ; Vtn = 0.7 V for nmos mnCox = 60 mA/v2 ; nMOS = 5
L
mpCox = 40 mA/V2 ; |Vtp| = 0.9 V for pMOS W
mnCox = 30 mA/v2 ; nMOS = 10
⎛W ⎞ ⎛W ⎞ L
The ratio of ⎜ ⎟ to ⎜ ⎟ is equal to _____ (round
⎝ L ⎠n ⎝ L ⎠P mn and mp are the carrier mobilities, and Cox is the oxide
off to 3 decimal places). [2019] capacitance per unit area. Ignoring the effect of channel
length modulation and body bias, the gain of the circuit
is _____ (rounded off to 1 decimal place) [2019]
5.86 | Analog Electronics

Vdd = 4 V Vdd

W = 10 W = 10 W = 40
L L L
Vout
Vdd Iout
Vin V1 W =5 W =5 V2
L L
1 mA

Solution:  W =2 W =3
L L
VDD = 4 V

Solution: 
M1 M3
Vdd
Vout
M2 M4 Vin
W = 10 W = 10
L L

W  Vdd V1 W =5 W =5 V2 Iout
mnCox = 60µA/V 2 ;   = 5 L L
 L n
W  1 mA
m pCox = 30µA/V 2 ;   = 10
 Lp
W =2 W =3
| V |= Vtn = 1 V L L
tp

rds = 6 MΩ
Iref = 1 mA
1 W 
I DC = µ pCox   VSG − | Vtp |2 
Current hours in the ratio  
2  Lp W
 L
1 W 
× 30 × 10 × ( 2 − 1)
2
=
2  L 
I ref 1 1 2
= 150 µA = = =
Io W  Io 3
n
 L 
W  2
gm = 2 × 1DC × µn Ccox  
 L I o = 1.5 mA
= 2 × 150 × 60 × 5 I out 40
=
= 300 mA/V Io 10

Av = − gm ( rd || rd ) = −300 (6.1116) 40
I out =
× 1.5
= ( −300)(3) = −900 V/V 10
= 6 mA
3. In the circuit shown, V1 = 0 and V2 = Vdd. The other rel- Hence, the correct answer is (6 mA).
evant parameters are mentioned in the figure. Ignoring
the effect of channel length modulation and the body 4. In the circuits shown, the threshold voltage of each
effect, the value of Iout is ______ mA (rounded off to 1 nMOS transistor is 0.6 V, Ignoring the effect of channel
decimal place). [2019] length modulation and body bias, the values of Vout1
and Vout2. [2019]
Chapter 4  FET and MOSFET Analysis  |  5.87

3V to two decimal places). [e0 = 8.854 × 10–14 F/cm,


est = 11.9] [2019]
3V + Vout1 Solution:

3V + 1 W 2
– I DS = mnCox Vms − VT 
2 L
1
= × 800 × 3.45 × 10 −7 × 10 [5 − 0.7]
2
3V 3V 3V
2
Vout2
= 25.5162 mA
3V +
– Hence, the correct answer is 25.562 mA.
6. In the circuit shown below, the (W/L) value for M2 is
twice that for M1. The two nMOS transistors are other-
(A) 1.8 and 2.4 (B) 2.4 and 1.2 wise identical. Threshold voltage VT for both transistors
(C) 1.8 and 1.2 (D) 2.4 and 2.4 is 1.0 V. Note that VGS for M2 must be > 1.0V.
Solution:
3.3 V
3V

M1
2.0 V
M2
M2
3V + Vo1
– 3V +

Vx

M1
3V 3V 3V

M3 M4 M5 Vo2

3V +
– Current through the nMOS transistors can be modelled
as:
⎛W ⎞ 1 2
IDS = µCox ⎜ ⎟ ((VGS – VT) VDS – V ) for VDS ≤
⎝L⎠ 2 DS
For M1
VGS – VT
V1 = 3 − 2.6 = 2.4 V.
⎛W ⎞
IDS = µCox ⎜ ⎟ (VGS – VT)2/2 for VDS ≥ VGS – VT
For M2 ⎝L⎠
Vo1 = V1 − Vt = 2.4 − 0.6 The voltage (in volts, accurate to two decimal places) at
V = 1.8 V Vx is _______. [2018]
o1
For M3 Solution:  Consider the figure given below
3.3 V
V3 = 3 − 0.6 = 2.4 V

For M4
2V M2
V4 = 3 − 0.6 = 2.4 V.

For M2 Vx
V5 = 3 − 0.6 = 2.4
V = 2.4 V. M1
o 2
Hence the correct option is (A)
5. Consider a long-channel MOSFET with a channel
length 1 mm and width 10 mm. The device param- From the above figure we have
eters are acceptor concentration NA = 5 × 1016 cm–3, M2: VGS2 = 2 – Vx
electron mobility mn  = 800 cm2/V–s, oxide capaci- VDS2 = 3.3 – Vx
tance/area Cox = 3.45 × 10–7 F/cm2. Threshold volt-
age VT = 0.7 V. The drain saturation current (IDsat) VDS2 ≥ VGS2 – Vt
for a gate voltage of 5 V is _____ mA (rounded off 3.3 – Vx ≥ 1 – Vx
5.88 | Analog Electronics

Therefore M2 operates in saturation region. VS = I D , VD = 8 − I D


M1: VGS1 = 2 VDD = 8 V

VDS1 = Vx 1 KΩ
3 MΩ RD
Vx ≤ 1
ID
VD = 8 − ID
We know that M1 operates in linear region
VG
ID1 = ID2
Vs = ID

⎛ω ⎞ V 2 5 MΩ
μ n cox ⎜ ⎟ ⎡⎣(VGS1 − VT ) ⎤⎦ VDS − DS 1 KΩ
⎝ ⎠
L 2
1 ⎛ω ⎞
μ n cox ⎜ ⎟ ⎡⎣(VGS2 − VT ) ⎤⎦
2
=
2 ⎝ ⎠
L
Thus ID is given by
⎛ω ⎞ V2
μ n cox ⎜ ⎟ [( 2 − 1) ]Vx − x 1 W
⎝L⎠ 2 I D = µ nCox (VGS − Vtn ) 2
2 L
1 ⎛ 2ω ⎞
= μ n cox ⎜ ⎟ [ 2 − Vx − 1]2 1
I D = × 1× (5 − I D − 1) 2
2 ⎝ L ⎠ 2
3Vx2 − 6Vx + 2 = 0 1
I D = (4 − I D )2
2
1 1
Vx = 1 ± V ID = (4 − I D )2
3 2
1 1 16 + I D2 − 8 I D = 2 I D
Vx cannot be 1 + V. So, Vx should be 1 – = 0.42
V 3 3 I D2 − 10 I D + 16 = 0
Hence, the correct answer is 0.41 to 0.435. I D2 − 2 I D − 8 I D + 16 = 0
7. For the circuit shown, assume that the NMOS transis- I D (II D − 2) − 8( I D − 2) = 0
tor is in saturation. Its threshold voltage Vtn = 1V and
W  ∴ ( I D − 8)( I D − 2) = 0
its trans conductance parameter µ nCox   = 1 mA/V 2
L
. Neglect channel length modulation and body bias ∴ I D = 2 mA, 8 mA
effects. Under these conditions, the drain current ID in
mA is ______. [2017] Given that nmos transistor is in saturation so
VDD = 8 V VDS > VGS − Vtn
RD 1 KΩ
R1
ID
For I D = 2 mA
3 MΩ
VGS = 3, Vtn = 1
VDS = VD − VS = 6 − 2 = 4
VDS > VGS − Vtn
R2
4 > 2 nmos transistor is in saturation for
5 MΩ Rs
1 KΩ
I D = 8 mA
VGS = 5 − 8 = −3 VDS = VD − VS
Solution:  Vtn = 1 V VDS = 0 − 8
VDD × R2 VDS > VGS − Vtn VDS = −8
VGS = VG − VS VG =
R1 + R2 −8 > −4 False
8× 5 ∴ nmos transistor is not in saturation region.
∴VGS = 5 − I D VG = =5V
8 ∴ Drain current ID = 2 mA
Hence, the correct answer is (1.9 to 2.1).
Chapter 4  FET and MOSFET Analysis  |  5.89

8. Assuming that transistors M1 and M2 are identical and ∂I D


have a threshold voltage of 1 V, the state of transistor gd = ; drain conductance
∂VDS
M1 and M2 are respectively
∂I D 1 W
= kn ⋅ × ⎡(VGS − VT ) (0 + λ ⋅1)⎤
2
3V gd =
∂VDS 2 L ⎣ ⎦

1
× 70 × 10 −6 × 4 ⎡(1.5) (0 + 0.09 × 1)⎤
2
gd =
2 ⎣ ⎦
2.5 V M2
140 × 10 −6 [0.2025] = 28.35 × 10–6 
=

Hence, the correct Answer is (28.35).


1 1. Figure I and II show two MOS capacitors of unit area.
2V M1 The capacitor in Figure I has insulator materials X (of
thickness t1 = 1 nm and dielectric constant ε1 = 4) and
Y (of thickness t2 = 3 nm and dielectric constant ε2 =
20). The capacitor in figure II has only insulator mate-
rial X of thickness tEq If the capacitors are of equal
(A) Saturation, Saturation
capacitance, then the value of tEq (in nm) is _________.
(B) Linear, Linear
 [2016]
(C) Linear, Saturation
(D) Saturation, ­Linear Metal Metal
t2 ε2
9. Two n-channel MOSFETs, T1 and T2 are identical in t1 ε1 tEq
all respects except that the width of T2 is double that ε1
of T1. Both the transistors are biased in the saturation Si Si
region of operation. but the gate overdrive voltage (VGS
-VTH) of T2 is double that of T1. where VGS and VTH
are the gate-to-source voltage and threshold voltage Solution: 
Given
of the transistors, respectively. If the drain current and t1 = 1 nm and ε1 = 4
transconductance of T1 are ID1 and gm1 respectively, the
corresponding values of these two parameters for T2 t2= 3 nm and ε2 = 20
are  [2017]
Capacitor in figure 2 has insulator material X of thick-
(A) 8ID1 and 2gm1 (B) 8ID1 and 4gm
1 ness tEq.
(C) 4ID1 and 4gm1 (D) 4ID1 and 2gm
1
The oxide capacitance of MOS is
10. Consider an n–channel metal oxide semiconductor
field effect transistor (MOSFET) with a gate to source ε ε .ε
Cox = ox = r o
W tox tox
voltage of 1.8 V. Assume that = 4, µn Cox = 70 × 10–6
L εo
AV–2, the threshold voltage is 0.3 V, and the channel Cox1 =
⎡⎛ t1 ⎞ t 2 ⎤
length modulation parameter is 0.09 V–1. In the satura- ⎢⎜ ⎟ + ⎥
tion region, the drain conductance (in micro Siemens) ⎣⎝ ε1 ⎠ ε 2 ⎦ 
is ______. [2016] ε ×ε
Solution:  Gate to source voltage VGS = 1.8 V And Cox2 = 0 1
t Eq
W 
= 4 , Given Cox1 = Cox2
L
µncox = 70 × 10–6 A/V2 tE q ⎡t t ⎤
=⎢ 1 + 2⎥
Threshold voltage VT = 0.3 V ε1 ⎣ ε1 ε 2 ⎦
Channel length modulation parameter λ = 0.09V −1 ⎡1 3 ⎤
t E q = 4 ⎢ + ⎥ nm
Given transistor is in the Saturation region so ⎣ 4 20 ⎦
1 W ⎡ 3⎤
I D = kn ⋅ (VGS − VT ) ⋅ (1 + λ ⋅VDS ) t E q = ⎢1 + ⎥ nm
2

2 L  ⎣ 5⎦
But t E q = 1.6 nm
VDS = VGS – VT = 1.5 V Hence, the correct Answer is (1.6 nm).
5.90 | Analog Electronics

12. In the circuit shown in the figure, the channel (A) –gm1(r01||r02||r03)
length modulation of all transistors is non-zero (λ
⎛ 1 ⎞
≠ 0). Also, all transistors operate in saturation and (B) –gm1 ⎜ r01 || || r03 ⎟
have negligible body effect. The AC small signal ⎝ gm3 ⎠
voltage gain (V0/Vin) of the circuit is [2016] ⎛ ⎛ 1 ⎞ ⎞
VDD (C) –gm1 ⎜ r01 || ⎜ || r02 ⎟ || r03 ⎟
⎝ ⎝ gm2 ⎠ ⎠
⎛ ⎛ 1 ⎞ ⎞
M3 M2
(D) –gm1 ⎜ r01 || ⎜ || r03 ⎟ || r02 ⎟
⎝ ⎝ gm3 ⎠ ⎠
VG
Solution:  Apply AC analysis and replace all the tran-
sistor with small signal equivalent model
V0

Vin M1

S D G D
– +
Vgs2
Vgs3 gm3 Vgs3 gm2 Vgs2
r03 r02
+ –
G S

S D
V0
Vi +

Vgs1 gm1 Vgs1 r01


G
S

⎡ ⎛ 1 ⎞⎤ VDD
Vont = − gm1Vgs1 ⎢ r01 r03 ⎜ r02 ⎟ ⎥ 
⎢⎣ ⎝ gm 2 ⎠ ⎦⎥
Vin = Vgs1 1 kΩ

Vout ⎛ ⎛ 1 ⎞ ⎞ V0
= gain = − gm1 ⎜ r01 ⎜ r02 ⎟ r03 ⎟ 
Vin ⎝ ⎝ gm 2 ⎠ ⎠ 50 pF
Vin M1
Hence, the correct option is (C).
5 kΩ
1 3. In the circuit shown in the figure, transistor M1 is in
saturation and has transconductance gm = 0.01 siemens.
Ignoring internal parastitic capacitances and assuming
the channel length modulation λ to be zero, the small
Solution:  Given gm = 0.01
input pole frequency (in kHz) is [2016]
Voltage gain Av = –gm RD
=
–0.01 × 1000 = –10
Chapter 4  FET and MOSFET Analysis  |  5.91

Due to the presence of capacitance of 50 pF, Miller Solution: (b)


effect will takes place resulting in input and output VB
capacitance, Cin and Cout respectively. ID
By using Miller’s theorem.
Cin = C(1 – Av)
M
and
1
Cout = C(1 – )
Av
 ∂I D
Cin = (50 × 10–12) (1 + 10) = 550 pF gm =
∂VGS
5 kΩ Vout VB = VG
+
VS = 0
Cin Vgs gMVgs Cout 1 kΩ VGS = VG − VS
= VG − 0 = VG = VB
– ∂VB 1
=
∂I D g m
VD = VG
Input frequency will be
VD − VS = VG − VS
1
f input = VDS = VGS
2π RC
1 VDS > VCS − VT
= = 57.88 kHz
2π × 5 × 10 × 550 × 10 −12
3
So, the given MOSFET is in saturation region
Hence, the correct Answer is (57.88 KHz).
1 4. Which one of the following processes is preferred to W
g m = mn Coy (VGS − VT )
form the gate dielectric (SiO2) of MOSFETs? L
 [2015] g m = 40 × 10 −6 × ( 2 − 1) = 4 × 10 −5
(A) Sputtering ∂VB 1
(B) Molecular beam epitaxy =
(C) Wet oxidation ∂I D g m
(D) Dry oxidation 1
=
Solution:  Dry oxidation method is preferred to form 4 × 10 −5
the gate dielectric (Sio2) of MOSFETS 100 × 103
=
Hence, the correct option is (D). 4
= 25 kΩ
15. The small-signal resistance (i.e., dVB/dID) in kΩ offered
by the n-channel MOSFET M shown in the figure below, Hence, the correct option is (b).
at a bias point of VB = 2 V, is (device data for M:device
16. In the CMOS circuit shown, electron and hole mobili-
transconductance parameter kN = mnC′ox(W/L) = 40
ties are equal, and M1 and M2 are equally sized.
mA/V2, threshold voltage VTN = 1 V, and neglect body
The device M1 is in the linear region if
effect and channel length modulation effects) [2013]
 [2012]
VB
ID
5V

M1
− VTP = 1 V
M
Vin

VTn = 1 V
(a) 12.5 (b) 25 (c) 50 (d) 100 M2
5.92 | Analog Electronics

(a) Vjn < 1.875 V Solution: (c)


(b) 1.875 V < Vin< 3.125 V
(c) Vjn 3.125 V 6V
(d) 0 < Vin < 5 V
Solution: (a) 5V
W/L = 4
The voltage transfer characteristics of the CMOS is
Vx

5V
W/L = 1
M1 − VTP = 1 V

Vin Vo
W
The transistor which has =4
M2 VTn = 1 V L
VDS = 6 − Vx , and ,
V0 VGS = 5 − Vx
VoH = VDD VGS = −VT = 5 − Vx − 1
M1 : triode = 4 − Vx
VOS VGS − VT
M2 : triode
So that transistor is in saturation region, the transistor
Vi
VIL VIH VDD W
which has = 1 drain is connected to gate so that tran-
L
1 sistor in saturation VDS > VGS > VT (∴ VDS = VGS).
where VIL = (3VDD + 2Vt ) The current flow in both the transistor is same
8
2
= 2.125 V  W   (V ) − V 
mnCox    GS 1 T 
Hence, approximately, Vin < 1.875 V. So the device M1 is  L 1  2 
in the linear region.
2
Hence, the correct option is (a).  W   (V ) − V 
= mn Cox    GS 2 T 
 L 2  2 
17. In the circuit shown below, for the MOS transistors, μn
Cox = 100 μA/V2 and the threshold voltage VT = 1 V. The 4(5 − Vx − 1) 2 1(Vx − 4) 2
voltage Vx at the source of the upper transistor is[2011] = (∵VGS = Vx − 0)
2 2

6V ( )
4 Vx 2 − 8Vx + 16 = Vx 2 − 2Vx + 1

⇒ 3Vx 2 − 30Vx + 63 = 0
5V
W/L = 4
⇒ Vx = 3V
Vx
Hence, the correct option is (c).
W/L = 1 Statement for Linked Answer Questions 18 and 19.
Consider the CMOS circuit shown, where the gate voltage
VG of the n-MOSFET is increased from zero, while the gate
voltage of the p-MOSFET is kept constant at 3 V. Assume
that, for both transistors, the magnitude of the threshold
(a) 1 V (b) 2 V
voltage is 1 V and the product of the transconductance pa-
(c) 3 V (d) 3.67 V
rameter and the (W/L) ratio, i.e. the quantity μCoX (W/L), is
1 mA. V−2.
Chapter 4  FET and MOSFET Analysis  |  5.93

5V Since the threshold voltage is IV,  for small i­ncrease in


Va beyond IV, n-MOSFET is in saturation and p-MOS-
3V FET is in triode region.
Hence, the correct option is (b).
V0 2 0. For the circuit shown in the following figure, transistors
M1 and M2 are identical NMOS transistors. Assume that
VG M2 is in saturation and the output is unloaded. [2008]

VDD
1 8. Estimate the output voltage V0 for VG = 1.5 V.
Ibias RE
[2009]
Vout
1 1 Va Ix
(a) 4 − 4+
V (b) V
2 2 M1 M2
Is
(c) 3 (d) 3
4− V 4+ V
2 2
Solution: (d) The current Ix is related to Ibias as
Ix = Ibias + Is
(a)
1 1
(Vgs − Vt )n2 = (Vgs − Vt ) p VDS − VDS 2 Ix = Ibias
(b)
2 2
Ix = Ibias− Is
(c)
1 1
⇒ (1.5 − 1) 2 = ( −2 + 1) × VDS − VDS 2
2 2  V 
Ix = Ibias− VDD − out 
(d)
⇒ VDS = 4.875 V  RE 
Solution: (b)
 3
⇒ V0 = VDS = 4.875 V =  4 + Given circuit is current mirror, since MOSFETS are
V
 2  identical

Hence, the correct option is (d). VDD

19. For small increase in VG beyond 1 V, which of the fol- Ibias RE


lowing gives the correct description of the region of Vout
operation of each MOSFET? [2009]
Va Ix
(a) Both the MOSFETs are in saturation region
(b) Both the MOSFETs are in triode region M1 M2
(c)  n-MOSFET is in triode and p-MOSFET is in satu- Is
ration region
(d) n-MOSFET is in saturation and p-MOSFET is in
triode region
W  W 
Solution: (b)   =  
L 2 L 1
5V ∴ I x = I bias
Hence, the correct option is (b).
3V
21. Two identical NMOS transistors M1 and M2 are con-
V0 nected as shown below. Vbias is chosen so that both tran-
+
VG VDS sistors are in saturation. The equivalent gm of the pair is

∂I
defined to be out at constant Vout
∂Vi
5.94 | Analog Electronics

Iout 23. The maximum depletion layer width in silicon is


Vout  [2007]
Vbias M2
(a) 0.143 mm
(b) 0.857 mm
(c) 1 mm
(d) 1.143 mm
Vi M1 Solution: (b)
Series capacitance is given by

The equivalent gm of the pair is [2008] 1 1 1


= +
(a) the sum of individual gm’s of the transistors Ca Coy Csi
(b) the product of individual gm’s of the transistors
(c) nearly equal to the gm of M1 toy
Coy = ⋅A
(d) nearly equal to gm/g0 of M2 toy
Solution: (c) t si
Since the current in both transistors are equal, Csi = ⋅A
xd
gm is decided by M1
Iout 1012 1012 × 10 −4
1012 = +
Vout 7 xd max
Vbias M2
xd max = 0.857 mm.

Hence, the correct option is (b).


Vi M1 24. The gate oxide thickness in the MOS capacitor is
 [2007]
(a) 50 μm (b) 143 μm
Hence, the correct option is (c). (c) 350 μm (d) 1 μm
Common Data Question 22, 23 and 24.
The figure shows the high-frequency capacitance-voltage Solution: (a)
(C–V) characteristics of a Metal/SiO2/silicon (MOS) capac- C
itor having an area of 1 × 10−4 cm2. Assume that the permit- 7pF
tivities (e0 er) of silicon and SiO2 are 1 × 10−12 F/cm and 3.5
× 10−13 F/cm respectively.
1pF
0 V
C
7pF
Capacitance is given by
1pF
0 V tt A
C = too t rr A
22. Consider the following statements about the C–V char- C= d
acteristics plot: given, dC = 7 pF
given, C =−47 pF
S1: The MOS capacitor has as n-type substrate. A = 1 × 10 −4
S2: If positive charges are introduced in the oxide, the A = 1 × 10
t0 t r = 3.5 × 10 −−13
C–V plot will shift to the left. t0 t r = 3.5 × 10 13
Then which of the following is true? [2007] tt A
thus, d = t00 t rr A
(a) Both S1 and S2 are true thus, d = C
(b) S1 is true and S2 is false C nm
= 50
(c) S1 is false and S2 is true = 50 nm
(d) Both S1 and S2 are false Hence, the correct option is (a).
Solution: (b) 25. In the CMOS inverter circuit shown, if the transcon-
The MOS capacitor has a p-type substrate. If positive ductance parameters of the NMOS and PMOS transis-
charges are introduced in the oxide, the C–V plot will W W
tors are K0 = Kp = m0Cox n = m pCox P = 40 mA / V
2
shift to the right. Ln LP
Hence, the correct option is (b).
Chapter 4  FET and MOSFET Analysis  |  5.95

and their threshold voltages are VTHn = |VTHp| = 1 V, the (a) 1.875 mS and 3.41
current I is  [2007] (b) 1.875 mS and −3.41
5V (c) 3.3 mS and −6
(d) 3.3 mS and 6
PMOS Solution: (a)

2.5 V I  VGSQ 
g m = g mo 1 − ,
 VP 
NMOS
where

(a) 0A (b) 25 mA 2 I DSS 2 × 10


g mo = = = 2.5
(c) 45 mA (d) 90 mA VP 8
Solution: (c)  2
g m = 2.5 1 −  = 1.875 ms
5V  8
 20 
PMOS voltage gain = − g m ( ra || RD ) − 1.875  
 11 
= 3.41
2.5 V

NMOS
Hence, the correct option is (a).

27. I D and V DS under DC conditions are, respectively


VGS for each MOS is 2.5V  [2005]
VT = IV, device parameter, (a) 5.625 mA and 8.75 V
(b) 7.500 mA and 5.00 V
A (c) 4.500 mA and 11.00 V
k = 40 m
V2 (d) 6.250 mA and 7.50 V
k Solution: (a)
so, I D = (VGS − VT ) 2
2 Drain current is given by
= 20( 2.5 − 1) 2
2
= 45 mA  V 
I D = I DSS 1 − GS 
Hence, the correct option is (c).  VP 
Common Data Questions 26, 27 and 28. VP = −8
Given rd = 20 kΩ, IDSS = 10 mA, Vp = −8V I DSS = 10 mA
VGS = − Z
20V
−2
I D = 10 m 1 −  = 5.625 mA
 −8 
2kΩ
D VDS = VD − 2 K I D = 8.75 V

G
Hence, the correct option is (a).
S
2MΩ 28. Zi and Z0 of the circuit are respectively [2005]
V1 V0
− (a) 2 MΩ and 2 kΩ
2V
+ 20
(b) 2 MΩ and kΩ
11
Z1 Z0 (c) infinity and 2 MΩ
20
(d) infinity and kΩ
26. Transconductance in milli-Siemens (mS) and voltage 11
gain of the amplifier are respectively [2005]
5.96 | Analog Electronics

Solution: (d) Solution: (b)
Given From graph,
20V ID

2k

+ +
VGS
2M IV
Vi V0
− VD = 5V
− 2V −
+

VG = 3V
Zi Z0

rd = 20 KΩ
rd =
I DSS = 10 KΩ
20 mA VS =1V

V =
I DSS = 10
−8 mA
V
P
= Vth = 1 V
VZP = (−R8DV|| ∞)
i
= VGS = 2 V
Zi = (RRD=|| 2∞m
)Ω
G
VDS = 4 V
Z0 == (RRGD=|| 2rdm) Ω
Z 0 = ( RD || rd ) 20 And VDS ≥ (VGS − VT )
= ( 2 K || 20 K ) = 20 KΩ
= ( 2 K || 20 K ) = 11 KΩ ∴ MOSFET is in saturation.
11 Hence, the correct option is (b).

Hence, the correct option is (d). 30. The action of a JFET in its equivalent circuit can best
be represented as a [2003]
29. For an n-channel MOSFET and its transfer curve shown (a) Current Controlled Current Source
in the figure, the threshold voltage is  [2005] (b) Current Controlled Voltage Source
(c) Voltage Controlled Voltage Source
ID (d) Voltage Controlled Current Source
Solution: (d)
Transfer
characteristics
JFET is voltage controlled current source. Input imped-
ance of JFET is very high. Thus input current is zero.
Hence, the correct option is (d).
IV VGS 31. Consider the following statements in connection
with the CMOS inverter in the figure, where both the
VD = 5V
MOSFETs are of enhancement type and both have a
threshold voltage of 2V.
D
Statement 1: T1 conducts when Vi > 2V
Statement 2: T1 is always in saturation when V0 = 0 V.
VG = 3V
+5V
G

T2

S V1 V0
VS =1V
T1
(a) 1 V and the device is in active region
(b) −1 V and the device is in saturation region
(c) 1 V and the device is in saturation region Which of the following is correct? [2002]
(d) −1 V and the device is in active region (a) Only Statement 1 is TRUE
(b) Only Statement 2 is TRUE
Chapter 4  FET and MOSFET Analysis  |  5.97

(c) Both the statements are TRUE (a) 150 Ω (b) 470 Ω
(d) Both the statements are FALSE (c) 560 Ω (d) 1 kΩ
Solution: (a) Solution: (a)
Statement 2 is false because VDS will be less than VGS– Using the formula for drain current,
VT
IDS
If V0 = 0
5V
10V

T2 RS
Vi
V0

T1
2
 V 
I DS = I DSS 1 − GS 
Hence, the correct option is (a).  Vp 

V0  V 2
2
32. The voltage gain Av = of the JFET amplifier shown 6.4 × 10 −−33 = 10 × 10 −−33 1 − VGS 
Vi 6.4 × 10 = 10 × 10 1 − −GS5 
in the figure is  −5 
2
VDD = +10V  V 2
0.64 = 1 + VGS 
ID = 1mA 0.64 = 1 + 5GS 
Rd  5 
(3 kΩ) C2 V
⇒ 1 + VGS = 0.8
Ct + ⇒ 1 + 5GS = 0.8
V0 5
+ VGS
⇒ VGS = −0.2
Rg ⇒ 5 = −0.2
Vi 5
(1 kΩ) Rs
Cs ⇒ VGS = −1 V
(2.5 kΩ)
− ⇒ VGS = −1 V
⇒ VGS = − I DS RS
 [2002] ⇒ VGS = − I DS RS
IDSS = 10 mA  Vp = −5 V −1 = −6.4 × 10 −−33 RS
−1 = −6.4 × 10 RS
(Assume C1, C2 and C3 to be very large) 1
RS = 1 = 156Ω
(a) +18 (b) –18 (c) +6 (d) –6 RS = 6.4 × 10 −−33 = 156Ω
6.4 × 10
Solution: (d) RS ≅ 150Ω
RS ≅ 150Ω
VG = 0 ⇒ VS = IDRS = 1 mA × 2.5 K = 2.5 V
VGS = VG − VS = 0 − 2.5 = −2.5 V
Five-marks Questions
2 × 10 × 10 −3   −2.5  
gm = 1 −  −3   = 2 ms 1. A JFET with Vp = −4V and lDSS = 12 mA is used in the
151   
circuit shown in figure. Assuming the device to be oper-
A v = − g m RD = −2 × 10 −3 × 3 K = −6 V. ating in saturation, determine lD, VDS and VGS. [1996]
VDD = 20 V
33. The JFET in the circuit shown in the figure has an IDSS
= 10 mA and VP = −5V. The value of the resistance RS
for a drain current IDS = 6.4 mA is (Select the Nearest 400 kΩ 2 kΩ
value)[1992]
D +
G
IDS VDS
+
VGS − S −
+
10V 100 kΩ 2 kΩ

RS
5.98 | Analog Electronics

Solution: Solution:
VDD = 20 V 1
SCgd
+ +
2k +
1 1
Rth = 400 k || 100 k Vin R1|| R2 Vgs rds 5k 5k Vo
+ - SCgs
gm Vgs
SCds
- -
VDS
+
VGS − − V0 V V
4V Vth (V0 − Vin ) S Cgd + g m Vgs + + V0 s cos + 0 + 0 = 0
2k rds 5k 5k

 1 2
V0  SCgd + + s cos +  = Vin ( S Cgd − g m )
100  rds 5k 
Vth = 20 × = 4V
100 + 400
V0
8 AV =
40 0 × 100 Vi
Rth = 400 || 100 K = = 80 K
500 K SCgd − g m
∴ I4 = 0 =
1 2
SCgd + + SCas +
Apply KVL to input mesh rds 5k
4 = Vgs + 2 × 103 × ID
VGS = G − 2 ID (1) Upper cut off frequency.
1 1
 V 
2
fH = =
I D = I DSS 1 − GS  2pt 2p (rds || 5k || 5k )(Cgd + Cds )
 VP 
2
rds = ∞
IIDD ==12  (4 − 2 I D ) 
12 1 −  f H = 2.89 MHz
 −4 
ID = 5.33 mA, 3 mA
3. In the MOSFET amplifier shown in the figure below
Apply KVL to output mesh
has μ = 50, rd = 10 kΩ, Cgs = 5 pF, Cgd = 1 pF and Cds = 2
20 = 2ID + VDS + 2IDS pF. Draw a small signal equivalent circuit for the ampli-
VDS = 20 − 4IDS(2) fier for midband frequencies and calculate its midband
Substitute IDS in equation (2) voltage gain. [1994]
VDS1 = 20 − 4 IDS1 = 20 − 4 × 5.33
UDS1 = −1.33V VDD

RD = 20 kΩ
But VDS should be positive, So IDS ≠ 5.33 mA
Substitute IDS = 3 mA in equation (2)
50 µF
VDS2 = 8V rsrc = 100 kΩ
+ RL = 20 kΩ
∴ VGS = 4 − 2ID = 4 − 2 × 3 = −2V Vs
Rs = 20 kΩ

VGS = −2V
2. In the JFET circuit shown in figure assume that R1||R2 Solution:
= 1MΩ and the total stray capacitance at the output to
100 Ω
be 20 pF The JFET used has gm = 2 mA/V, Cgs = 20 pF.
and Cgd = 2 pF. Determine the upper cut-off frequency +
of the amplifier. [1995] +
Vs VGS 20 k 20 k
gmVgs
+Vcc −

R2 RD = 5 kΩ 1k
1 µF

50 µF
Rs = 1 kΩ RL = 5 kΩ μ = 50
R1
Vs
2 kΩ 20 µF rd = 10 kΩ
Chapter 4  FET and MOSFET Analysis  |  5.99

μ = gm rd = gm × 10 × 103 = 50 V0 = −gm Vgs (10 k)


gm = 5 mA/V
V0 0 g m Vgs [10 k ] − g m [10 k ]
Vin − Vgs + gm Vgs × 1k = =
Vin Vgs + g m Vgs 1k 1 + g m × 1k
V0 = −gm Vgs (20||20)
V0
= −8.3
Vin
Chapter 5
Frequency Response
of Amplifier
Solution: (a)
One-mark Questions
fT = b f b
1. The fT of a BJT is related to its gm, Cp and Cm as fol- fT = 200 × 1
lows: [1998] = 200 MHz
Cp + C m
(a) fT = fb
gm fa = = (1 + b ) f b
1−a
2p (Cp + C m ) fa = (1 + 200)
fT =
(b)
gm = 201 MHz
gm
(c) fT = Hence, the correct option is (a).
Cp + C m
gm 3. An amplifier has an open-loop gain of 100 and its lower
fT =
(d) and upper-cut-off frequency of 100 Hz and 100 kHz,
2p (Cp + C m )
respectively, a feedback network with a feedback fact
of 0.99, is connected to the amplifier. The new lower-
Solution: (d)
and upper-cut-off frequencies are at_____ and _____
Unity gain bandwidth product of the CE transmitter
 [1995]
gm
fT = Solution:  (1 Hz)
2p (Cp + Cm )
gm 1 + Ab = 1 + 100 × 0.99
= = 1 + 99
2p (Ce + Cc )
Hence, the correct option is (d). = 100
2. An n-p-n transistor has a beta cut-off frequency fb of 1 fL 100
f L1 = = = 1 Hz
MHz and Common Emitter short circuit low frequency 1 + Ab 100
current gain b0 of 200. Its unity gain frequency fT and
the alpha cut-off frequency fa, respectively, are [1996] 4. An RC-coupled amplifier is assumed to have a single-
(a) 200 MHz, 201 MHz pole low frequency transfer function. The maximum
(b) 200 MHz, 1999 MHz lower-cut-off frequency allowed for the amplifier to
(c) 199 MHz, 200 MHz pass 50 Hz square wave with no more than 10% tilt
(d) 201 MHz, 200 MHz is______. [1995]
Chapter 5  Frequency Response of Amplifier  |  5.101

Solution:  (1.59 Hz)


RD = 10 KΩ
pf
% tilt = L × 100%
f
f × % tilt Vi M C = 1µF
fL =
p × 100 RL = 10 KΩ
50 × 10
=
p × 100
f L = 1.59 Hz (a) 8 (b) 32 (c) 50 (d) 200
Solution: (a)
5. In a multi-stage RC-coupled amplifier the coupling
capacitor: [1993] RD = 10 KΩ
(a) Limits the low frequency response
(b) Limits the high frequency response
(c) Does not affect the frequency response Vi 1µF
(d) Blocks the dc components without affecting the RL = 10 KΩ
frequency response.
Solution: (d)
The low frequency of operation of a multistage RC
Lower cut off frequency is given by
coupled amplifier is limited by the coupling capacitor.
Hence, the correct option is (d). 1
fL =
2pRC
Two-marks Questions =
1
2 × 3.14 × (10 k + 10 k ) × 1 mF
1. In the circuit shown, transistors Q1 and Q2 are biased at
a collector current of 2.6 mA. Assuming that transistor = 8 Hz
current gains are sufficiently large to assume collector Hence, the correct option is (a).
current equal to emitter current and thermal voltage of 3. A bipolar transistor is operating in the active region
V with a collector current of 1 mA. Assuming that the b
26 mV, the magnitude of voltage gain 0 in the mid-
Vs of the transistor is 100 and the thermal voltage (VT) is
band frequency range is _________ (upto second deci- 25 mV, the transconductance (gm) and the input resist-
mal place). [2017] ance (rp) of the transistor in the common emitter con-
5V figuration are [2004]
(a) gm = 25 mA/V and rp = 15.625 kΩ
1 KΩ (b) gm = 40 mA/V and rp = 4.0 kΩ
(c) gm = 25 mA/V and rp = 2.5 kΩ
VO (d) gm = 40 mA/V and rp = 2.5 kΩ
Q1 Solution: (d)
VS ∼ I
gm = C
VT
Q2 1 mA
=
RB2 25 mV
= 0.04
−5 V = 40 mA/V
h fe = gm rp
2. The ac schematic of an NMOS common-source stage is
shown in the figure below, where part of the biasing cir- ⇒ h fe = b
cuits has been omitted for simplicity. For the n-channel
b
MOSFET M, the transconductance gm = 1 mA/V, and rp =
body effect and channel length modulation effect are to gm
be neglected. The lower cut-off frequency in Hz of the 100
= = 2.5 kΩ
circuit is approximately at [2013] 40 × 10 −3
Hence, the correct option is (d).
1
fT =
2pRC
1 I
5.102 | Analog Electronics = gm = C
R VT
4. An npn BJT has gm = 38 mA/V, Cμ = 10−14F, Cp = 4 × IC
10−13F, and DC current gain b0 = 90. For this transistor fT =
2pV T × C m
fT and fb are [2001]
(a) fT = 1.64 × 108 Hz and fb = 1.47 × 1010 Hz 1 mA
(b) fT = 1.47 × 1010 Hz and fb = 1.64 × 108 Hz Cm =
2p × 26 mV × 400 × 106
(c) fT = 1.33 × 1012 Hz and fb = 1.47 × 1010 Hz
(d) fT = 1.47 × 1010 Hz and fb = 1.33 × 1012 Hz C m = 15 pF
Solution: (a)
Hence, the correct option is (a).
gm
fT = 7. In the circuit shown in the figure, N is a finite gain
2p (C m + Cp ) amplifier with a gain of K, a very large input imped-
ance, and a very low output impedance. The input
38 × 10 −3 impedance of the feedback amplifier with the feedback
=
2p (10 −14 + 4 × 10 −13 ) impedance Z connected as shown will be
[1996]
38 × 10 −3
= Z
2p × 10 −13 ( 4.11)

= 1.47 × 1010 Hz
+ +
f
fb = T Vi Vo
b0 N
− −

1.47 × 10 10
=
90

= 1.64 × 108 Hz ( b 0 = h fc )  1
(a) Z 1 − 
Hence, the correct option is (a).  K
5. An amplifier is assumed to have a single-pole high- (b) Z(1 − K)
frequency transfer function. The rise time of its output
response to a step function input is 35 nsec. The upper  Z 
(c)
 K − 1
−3 dB frequency (in MHz) for the amplifier to a sinu-  
soidal input is approximately at [1999]
 Z 
(d)
(a) 4.55 (b) 10 (c) 20 (d) 28.6 1− K 
 
Solution: (b)
Solution: (d)
tr × BW = 0.35
Miller theorem states that if a series impedance Z is
0.35 connected between the input and output terminal of a
BW = = 10.
35 × 10 −9 network, then it can be replaced with a shunt imped-
ance Z1 in the side and by a shunt impedance Z2 in the
Hence, the correct option is (b). output section of the network
6. An n-p-n transistor (with C = 0.3 pF) has a unity – gain Z
cut-off frequency fT of 400 MHz at a dc bias current Ic
= 1 mA. The value of its Cμ (in pF) is approximately (VT
= 26 mV) [1999]
(a) 15 (b) 30 (c) 50 (d) 96 + +
Solution: (a) Vi Vo
N
1 − −
fT =
2pRC
1 I
= gm = C
R VT If Vi > V2 then I1 flows
IC
fT =
2pV T × C m

1 mA
Cm =
2p × 26 mV × 400 × 106

C m = 15 pF
Chapter 5  Frequency Response of Amplifier  |  5.103

VCC
V1 −
V −VV2
II11 =
= 1Z 2
Z Rc 10 kΩ
V1 [[11 −
25 pF
=
V −VV2 // V
V1 ]]
= 1 Z2 1 Vo
Z Cc
[11 −
[ − AV ]]
A
=
=VV11 Z V 10 kΩ
VB 2Q
Z
V RB
V11 = Z
II1 = Z1
1
1
Z
= Z
= −A
11 − AV Transistor data gm = 5 mA/V, rp = 20 kΩ,
V
Z
Z Cp = 1.5 pF and Cμ = 0.5 pF determine the upper cut-off
Z11 =
Z = 1− K
1− K frequency fH of the amplifier. [1996]
Solution:  Net feedback, Cf = C + Cm = 25.5 μF
Apply Miller’s theorem to Cf ,
+ +
K = −gm  RC = −50
Vi Z1 Z2 Vo
N
− − Cm = Cf (1 − k) = 1300.5 pF
1
fH =
2p Cm (R f || rp )

Hence, the correct option is (d). f H = 18.38 kHz

Five-marks Questions
1. A common-emitter amplifier with an external capacitor
Cc connected across the base and the collector of the
transistor is shown in figure.
Chapter 6
Feedback Amplifiers
One-mark Questions 20 Ω +15 V

20 Ω
1. A good trans impedance amplifier has  [2018] Vout
+
(A) low input impedance and high output impedance. 5Ω −15 V
(B) high input impedance and high output impedance.
(C) high input impedance and low output impedance. VP
(D) low input impedance and low output impedance.
Solution: 
Hence, the correct option is (D) Q1 Q2 Q3 Q32
2. A good trans conductance amplif ier should have
 [2017]
(A) high input resistance and low output resistance.
The output voltage Vout (in volts) is ____________.
(B) low input resistance and high output resistance.
(C) high input and output resistances. 4. In the ac equivalent circuit shown in the figure, if iin is
(D) low input and output resistances. the input current and RF is very large, the type of feed-
back is [2014]
Solution:  For a trans conductance amplifier input re-
sistance is high and output resistance is also high. trans
conductance amplifier can also be called voltage con- RD
RD
trolled current source, i.e., vccs. An amplifiers is VC, Vout
when input resistance is high and an amplifies is cc M2
when output resistance is high. M1
Hence, the correct option is (C).
RF
3. In the voltage reference circuit shown in the figure, the
op-amp is ideal and the transistors Q1, Q2, …., Q32 are Small signal Iin
identical in all respects and have infinitely large val- input

ues of common-emitter current gain (b ). The Collector


current (Ic) of the transistors is related to their base- (a) voltage-voltage feedback
emitter voltage (VBE) by the relation Ic = Is exp (VBE/ (b) voltage-current feedback
VT), where Is is the saturation current. Assume that the (c) current-voltage feedback
voltage Vp shown in the figure is 0.7 V and the thermal (d) current-current feedback
voltage VT = 26 mV. [2017] Solution: (b)
Sampling → current
Mixing → voltage
Chapter 6  Feedback Amplifiers  |  5.105

Solution: (a)
RD
The desirable characteristics of a transconductance am-
RD
Vout
plifier are high input resistances and high output resist-
ances.
M2

M1
Hence, the correct option is (a).
7. In a voltage-voltage feedback as shown below, which
RF
one of the following statements is TRUE if the gain k is
increased? [2013]
Small signal Iin
input
+V Vi + A0 + Vout
− in − −
So, the given feedback is of type voltage–current feed-
back.
Hence, the correct option is (b).
5. The feedback topology in the amplifier circuit (the base
Vi = kVout +
− K +

bias circuit is not shown for simplicity) in the figure is
 [2014]
VCC (a) The input impedance increases and output imped-
ance decreases
RD Io (b) The input impedance increases and output imped-
Vo ance also increases
(c) The input impedance decreases and output imped-
RS ance also decreases
RE (d) The input impedance decreases and output imped-
VS
ance increases
Solution: (a)
The given configuration is a voltage–series feedback
(a) Voltage shunt feedback configuration. So, the input impedance increases
(b) Current series feedback Rif = Ri (1 + A0 k ) and the output impedance decreases.
(c) Current shunt feedback
(d) Voltage series feedback
+V Vi + A0 + Vout
Solution: (a) − in − −
The feedback topology in the amplifier circuit is cur-
rent series because
VCC

Vi = kVout K +
RC Io

Vo

Rof = Ri (1 + A0 k )
RS

RE
Hence, the correct option is (a).
VS
8. In a transconductance amplifier, it is desirable to have
 [2007]
Sampling → current (a) a large input resistance and a large output ­resistance
Mixing → voltage (b) a large input resistance and a small output ­resistance
So, current series feedback. (c) a small input resistance and a large output ­resistance
Hence, the correct option is (a). (d)  a small input resistance and a small output
6. The desirable characteristics of a transconductance ­resistance
amplifier are [2014] Solution: (a)
(a) high input resistance and high output resistance Transconductance amplifier has voltage at input end and
(b) high input resistance and low output resistance current at output end. Thus, input resistance must be high
(c) low input resistance and high output resistance and output resistance must be high.
(d) low input resistance and low output resistance Hence, the correct option is (a).
5.106 | Analog Electronics

9. The input impedance (Zi) and the output impedance 12. In a negative feedback amplifier using v­ oltage-series
(Zo) of an ideal trans-conductance (voltage controlled (i.e. voltage-sampling, series ­mixing) feedback.[2002]
current source) amplifier are [2006] (a) Ri decreases and Ro decreases
(a) Zi = 0, Z0 = 0 (b) Ri decreases and Ro increases
(b) Zi = 0, Z0 = ∞ (c) Ri increases and Ro decreases
(c) Zi = ∞, Z0 = 0 (d) Ri increases and Ro increases
(d) Zi = ∞, Z0 = ∞ (Ri and Ro denote the input and output resistances, re-
spectively)
Solution: (d)
For a voltage controlled current source input is voltage. Solution: (c)
So input impedance is to be as high as possible output Voltage – series negative feedback increases input im-
is current so output impedance is to be kept as high as pedances and decreases output impedance,
possible so that full current can go to load
Rif = Ri (1 + Ab )
i. e., Z i = ∞ R0
Rof =
Z0 = ∞ 1 + Ab
Hence, the correct option is (c).
Hence, the correct option is (d).
13. Negative feedback in an amplifier [1999]
10. The effect of current shunt feedback in an amplifier is
(a) reduces gain.
to [2005]
(b) increases frequency and phase distortions.
(a) increase the input resistance and decrease the out- (c) reduces bandwidth.
put resistance (d) increases noise.
(b) increase both input and output resistances
Solution: (a)
(c) decrease both input and output resistances.
(d) decrease the input resistance and increase the out- A
put resistance Af = (for −ve feedback)
1 + Ab
Solution: (a)
With current shunt negative feedback, Thus, gain reduces.
Hence, the correct option is (a).
Ri
Rif = 14. In a shunt–shunt negative feedback amplifier, as com-
1 + Ab
pared to the basic amplifier. [1998]
and Rof = R0 (1 + Ab ) (a) both input and output impedance decrease.
(b) input impedance decreases but output impedance
Hence, the correct option is (a). increases.
11. Voltage series feedback (also called series-shunt feed- (c) input impedance increases but output impedance
back) results in [2004] decreases.
(d) both input and output impedance increase.
(a) increase in both input and output impedances
(b) decrease in both input and output impedances Solution: (a)
(c) increase in input impedance and decrease in output In a shunt–shunt negative feedback amplifier, as com-
impedance pared to the basic amplifier, both input and output im-
(d) decrease in input impedance and increase in output pedances decrease
impedance
Ri
Solution: (c) Rif =
Voltage series feedback results in increase in input im- 1 + Ab
pedance and decrease in output impedance, R0
Rof =
1 + Ab
Rif = Ri (1 + Ab )
and Hence, the correct option is (a).
R0 15. To obtain very high input and output impedances in a
Rof =
1 + Ab feedback amplifier, the mostly used is [1995]
(a) voltage-series
Hence, the correct option is (c). (b) current-series
Chapter 6  Feedback Amplifiers  |  5.107

(c) voltage-shunt 1 − VA VA − V0
(d) current-shunt =
1 31
Solution: (b) The required voltage is VA = 0.5 V
Current series feedback amplifiers have very high input Hence, the correct answer is 0.4 to 0.6.
and very high output impedances
2. Consider the circuit shown in the figure.
Rif = Ri (1 + Ab ) = Ri (1 + Gmb )
− +
Rof = R0 (1 + Ab ) = R0 (1 + Gmb )
3 i0
Hence, the correct option is (b).
P
16. Negative feedback in amplifiers [1993] 1Ω
(a) improves the signal to noise ratio at the input
(b) improves the signal to noise ratio at the output + 1Ω
10 V 1Ω

(c) does not affect the signal to noise ratio at the output
(d) reduces distortion 1Ω
Q
Solution:  (a, d)
Negative feedback in amplifiers
The thevenin equivalent resistance (in W) across P-Q is
(b) improves the signal to noise ratio at the output
__________. [2017]
(d) reduces distortion.
Hence, the correct options are (a and d). 3. An amplifier without feedback has a voltage gain of
50, input resistance of 1 kΩ and output resistance of
Two-marks Questions 2.5 kΩ. The input resistance of the current-shunt nega-
tive feedback amplifier using the above amplifier with
1. An op-amp based circuit is implemented as shown a feedback factor of 0.2 is [2003]
below. (a) 1/11 kΩ (b) 1/5 kΩ
31 kΩ
(c) 5 kΩ (d) 11 kΩ
Solution: (a)
+15 V
1 kΩ Ri
+ – Rif =
A 1 + Ab
+
b = 0.2, A = 50
1V +
Vo
1k
– –15 V

⇒ Rif =
1 + 50 × 0.2
1
In the above circuit, assume the op-amp to be ideal. The = kΩ
11
voltage (in volts, correct to one decimal place) at node
A, connected to the negative input of the op-amp as Hence, the correct option is (a).
indicated in the figure is _______.  [2018] 4. An amplifier has an open-loop gain of 100. An input
Solution:  Consider the figure given below impedance of 1 kΩ, and an output impedance of 100
31 kΩ
Ω.A feedback network with a feedback factor of 0.99 is
connected to the amplifier in a voltage series feedback
+15 V mode. The new input and output impedances, respec-
1 kΩ tively, are [1999]
+ –
A (a) 10 Ω and 1 Ω
+
(b ) 10 Ω and 10 Ω
1V + (c) 100 kΩ and 1 Ω
Vo
(d) 100 kΩ and 1 kΩ
– –15 V

Solution: (d)
Feedback mode is voltage series – input impedances
If we apply Kirchhoff’s current law in above circuit we increases. Output impedance decreases.
get
5.108 | Analog Electronics G × BW = constant
A1 × BW1 = A2 × BW2
Z if = Z i (1 + Ab ) A1 × BW1 1 × BW1
BW2 = =
Z0 100 A2 20
Z of = = = 1Ω
1 + Ab 1 + 99 BW2
=
A = 100, b = 0.99, Z i = 1 kΩ, Z 0 = 100 Ω 20
Z if = 1k (1 + 99) = 100 k Ω
So, as compared to the unity gain amplifier with gain
Hence, the correct option is (d). twenty has less BW.
5. Negative feedback in Hence, the correct option is (c).
(1) voltage series configuration 7. The feedback amplif ier shown in the f igure has:
(2) current shunt configuration [1997]  [1989]
(a) increases input impedance (a) current-series feedback with large input ­impedance
(b) decreases input impedance and large output impedance.
(c) increases closed loop gain (b) voltage-series feedback with large input ­impedance
(d) leads to oscillation. and low output impedance.
Solution: (a) (c) voltage-shunt feedback with low input ­impedance
Voltage series configuration – increases the impedance and low output impedance.
Rif = Ri (1 + Ab ) (d) current-shunt feedback with low input i­mpedance
and output impedance.
Current shunt configuration – decreases the input im- Solution: (c)
Ri
pedances Rif = VCC
1 + Ab
Hence, the correct option is (a).
6. Two non-inversing amplifiers, one having a unity gain
and the other having a gain of twenty, are made using
identical operational amplifiers. As compared to the
unity gain amplifier, the amplifier with gain twenty has
 [1991]
(a) less negative feedback
(b) greater input impedance
(c) less bandwidth
(d) none of the above.
Solution: (c)
 For identical operational amplifier, gain-bandwidth
product is constant i.e., Emitter is output node, so it is voltage sampler voltage
G × BW = constant shunt feedback.
A1 × BW1 = A2 × BW2 Hence, the correct option is (c).
A1 × BW1 1 × BW1
BW2 = =
A2 20
BW2
=
20
Chapter 7
Oscillator Circuits
1 kW 2.1 kW
One-mark Questions

1. The configuration of the figure is a[2000] Vout
+
R1 R2 C

1 kW


V0
+ 1 kW C
R

C 1
(a) mF (b) 2pμF
2p
R C 1
(c) mF (d)
2p 6 mF
2p 6
(a) Precision integrator
Solution: (a)
(b) Hartley oscillator I
1 kW 2.1 kW
(c) Butterworth high pass filter
(d) Wien-bridge oscillator
Solution: (d) −
Vout
The given configuration is a Wien-bridge oscillator. +
C
R1 R2
1 kW


V0 1 kW C
+
R

C
Given circuit is a Wien-bridge oscillator oscillation fre-
quency given by
R C 1
W =
RC
1
⇒C =
WC
1
=
Two-marks Questions 2p 10 × 1 × 103
3

1
1. The value of C required for sinusoidal oscillations of = mF
frequency 1 kHz in the circuit of the figure is[2004] 2p
Hence, the correct option is (a).
5.110 | Analog Electronics

2. The oscillator circuit shown in the figure has an ideal (a) R2 = 5R1 (b)
R2 = 6R1
inverting amplifier. Its frequency of oscillation (in Hz) R1 R
is[2003] (c)
R2 = (d)
R2 = 1
6 5
Solution: (a)
R2

R1
C C C −
+ +
R R R

+ Network Vo(f )
Vi (f ) B(f )


1 1
(a) (b)
( 2p 6 RC ) ( 2pRC )
1 1 1
(c) (d) b =
( 6 RC ) 6 ( 2pRC ) 6
R1
and b =
Solution: (a) R1 + R2
bV0 − 0 bV0 − V0
+ =0
R1 R2
 R + R2  1
⇒b 1  =
C C C  R1 R2  R2
R1
R1 b =
R R R b = R1 + R2
R1 + R2
1 R1
1 R1 =
= 6 R1 + R2
6 R1 + R2
Frequency of oscillation for this oscillator is ⇒ R1 + R2 = 6 R1
⇒ R1 + R2 = 6 R1
1 ⇒ R2 = 5 R1
f = where RC = phase shift oscillator. ⇒ R2 = 5 R1
2p 6 RC
Hence, the correct option is (a). Hence, the correct option is (a).
3. The circuit in the figure employs positive feedback and 4. The oscillator circuit shown in the figure is [2001]
is intended to generate sinusoidal oscillation. If at a fre- −VCC
quency f0,
LC
Vf ( f ) 1
B( f ) = = ∠0 then to sustain oscillation at
V0 ( f ) 6
this frequency [2002] CC L = 10 µH
R1
R2 Vo

R1
− C1 = 2 pF C2 = 2 pF
+ + R2
Re Ce

+ Network Vo(f )
Vi (f ) B(f )


(a) Hartley oscillator with foscillation = 79.6 MHz
(b) Colpitts oscillator with foscillation = 50.3 MHz
Chapter 7  Oscillator Circuits  |  5.111

(c) Hartley oscillator with foscillation = 159.2 MHz Solution: (b)


(d) Colpitts oscillator with foscillation = 159.2 MHz 100 K

Solution: (b) 5K
Given oscillator is a Colpitts oscillator −
Vo
+
−VCC
R
LC

0.01 µF 10 mH 1K

CC L = 10 µH
R1
Vo
R Vf
+

C1 = 2 pF C2 = 2 pF +
Vo 1K 10 mH 0.01 µF Vf
R2 -
Ro Co
-

1
w=
1 LC
f = 1 1
f = 2p 1LCer w=
f = 2p LCer
2pC CLCer 2 × 2 10 × 10 × 0.01 × 10 −6
−3

Cer = C11C22 = 2 × 2
Cer = CC +CC = 2 ×4 2 w = 105 rad/sec
Cer = C11 1+ C2 22 = 4
1 + C2
= 1Cpf 4 Apply nodal analysis
= 1 pf
= 1 pf 1 V f − V0 Vf Vf
f = 1
f = 2p 10 × 10 1 −−66 × 10−−12
12 + + + V f CS = 0
f = 2p 10 × 10 × 10 R 1K LS
−6 −12
21p× 10
10 × 10 × 10
9
1 1   1  V0
= 1 × 10 9 = 50.3 MHz ⇒ Vt  +
= 12p× 10190 = 50.3 MHz  + Vf WC − WL  = R
= 2p 10 = 50.3 MHz  R 1K   
2p 10
1
for oscillation, WC − =0
Hence, the correct option is (b). WL
5. Value of R in the oscillator shown in the given figure is  1 1 1 V10  V
Vf  V+ =
chosen in such a way that it just oscillates at an angular  R + 1R
 R f1K = 0
K  R

frequencies of ‘w’. The value of ‘w’ and the required 1
value of R will, respectively, be 1
Vf V R
[1996] ⇒ b =⇒ b== f = R
V0 1 11 1
V0+
100 KΩ R KR+K
Rf 100 k
5 KΩ A = 1 +A = 1=+1R + f = 1 +=100
21k = 21
− Ri 5k
Vo R 5k i
+ 1 1
R b = b= = 1 = 1
A 21A 21
1 1 1 1
0.01 µF 10 mH 1K = R =R R R
1 =1 1 R1+ 1=K R + 1K
+
R 1KR + 1RK× 1K R × 1K
R ± 1k =
R±211k× 1=k 21 × 1k
(a) 105 rad/sec, 2 × 104 Ω R = 20RkΩ = 20 kΩ
(b) 2 × 104 rad/sec, 2 × 104 Ω = 2 × 10=42Ω× 10 4 Ω
(c) 2 × 104 rad/sec, 105 Ω
(d) 105 rad/sec, 105 Q Hence, the correct option is (b).
5.112 | Analog Electronics

6. Match the following [1994] Apply KCL at node A

List - I List - II VA − O VA − VC
+ =0
(a) Hartley (1) L  ow-frequency oscillator R R′
(b) Wien-bridge (2) H  igh-frequency oscillator
 R′ 
(c) Crystal (3) Stable-frequency oscillator VC = VA 1 +  (1)
 R
(4) Relaxation oscillator
(5) N  egative Resistance Apply KCL at node B.
­oscillator VA = VB

Solution:  (a-2, b-1, c-3)  R′ 


VC = VB 1 +  (2)
 R
(a) Hartley oscillator (2) High-frequency
oscillator VB − O VB − VD
+ =0
1 R
(b) Wien-bridge oscillator (1) Low-frequency
oscillator SC
(c) Crystal oscillator (3) Stable-frequency VD = VB (1 + RCs)(3)
oscillator Apply KCL at node D

VD − VC VD VD − VB
+ + =0
1 R R
Five-marks Questions SC
1. Find the value of R′ in the circuit of figure. For generat- VD (2 + RCS) = VC RCS + VB(4)
ing sinusoidal oscillations. Find the frequency of oscil- From equations (1), (3) and (4) we have
lations. [1998]
 R1 
R′
VB (1 + RCS )(2 + RCS ) = VB 1 +  ( RCS ) + VB
 R
R  R1 
− R 2 + 3RCS + R 2 C 2 S 2 − 1 +  RCS − 1 = 0
 R
+
C
R C
 R1 
1 + R 2 C 2 S 2 +  2 −  RCS = 0
 R
So for oscillations.

Solution: R1
2− =0
R
R′
R1 = 2R
R
− C R for frequency of oscillation
D
A
B + C 1 + R2C2S2 = 0
R C Put S = jw
1 – R2C2 w 2 = 0
1
w=
RC
By virtual short
VB = VA
Chapter 8
Power Amplifiers
+15 V
One-mark Questions
Ic
1. Crossover distortion behaviour is characteristic of
 [1999]
Q2 Vcε
(a) Class A output stage
(b) Class B output stage
(c) Class AB output stage Q1
(d) Common-base output stage 8Ω

Solution: (b)
Crossover distortion behaviour is a characteristic of −15 V
class B output stage amplifier and this distortion can be
reducing by adopting push-pull configuration. Solution:  (15 V)
Hence, the correct option is (b). 15 V
2. A power amplifier delivers 50 W output at 50% effi-
Ic
ciency. The ambient temperature is 25 °C. If the maxi-
mum allowable junction temperature is 150 ° C, then
the maximum thermal resistance qgc that can be toler- Q2 Vcε
ated is______ [1995] 0.7 + +
− −
Solution: (2.5) Q1 0.7
Power dissipated (PD) = 50 × 50% 8Ω

Po 1
=
Pin 2 15 V

Pin = 2Po = 2 × 50 = 100 W 15


IC = A
8
15
T j − TA I = A
PD = Pin − Po = VCce = 158 V
q
Vec1eB1==150.V
7V
T j − TA 150 − 25 125 Vee11B1− =VB01 .7= V
0.7 V
q= = =
PD 50 50 Vee11 −= V0B.17 =V0.7[∵VVB1 = 0]
q = 2.5 °C/W Ve1 = 0.7 V [∵ VB1 = 0]
3. The circuit shown in the figure supplies power to on 8 Ve1 = VB2 = 0.7 V
Ω speaker, LS. The values of IC and VCE for this circuit
will be IC = ____ and VCE  [1995] VB2e 2 = 0.7 V
VB2 − Ve 2 = 0.7 V
0.7 − Ve 2 = 0.7
Ve 2 = 0 V
5.114 | Analog Electronics

Apply KVL at emitter terminal (c) there is no even harmonics in the output
(d) there is no odd harmonics in the output
0 = I e Re − 15
Solution: (c)
15 15
Ie = = The output of push pull amplifier consists of only odd
Re 8 harmonics terms.
Vce = VC − Ve = 15 − 0
I o = 2k[ B1 cos wt + B3 cos 3wt + B5 cos 5wt + …]
= 15 V
Hence, the correct option is (c).
4. A Class A transformer coupled, transistor power ampli-
fier is required to deliver a power output of 10 watts. Two-marks Questions
The maximum power rating of the transistor should not
be less than [1994] 6. In case of class A amplifiers the ratio (efficiency of
(a) 5 W (b) 10 W (c) 20 W (d) 40 W transformer coupled amplifier)/(efficiency of a trans-
Solution: (c) former less amplifier) is [1987]
(a) 2.9 (b) 1.36 (c) 1.0 (d) 0.5
PD max
=2 Solution: (a)
PAC max Class A amplifier ratio
For class A transformer coupled amplifier, efficiency of transformer coupled amplifier
=
PD max = 2 × PAC max = 20 W ∴ Power rating ≥ 20 W. efficiency of transfoormer less amplifier
5. In a transistor push-pull amplifier [1993] 50%
= =2
(a) there is no dc present in the output 25%
(b) there is no distortion in the output Hence, the correct option is (a).
Chapter 9
Multi-vibrators
and 555 Timers
2. Consider the following two statements:
One-mark Questions Statement 1: A stable multi-vibrator can be used for
generating square wave.
1. In the Astable multivibrator circuit shown in the figure, Statement 2: Bi-stable multi-vibrator can be used for
the frequency of oscillation (in kHz) at the output pin 3 storing binary information. [2001]
is ____. [2016] (a) Only statement 1 is correct
(b) Only statement 2 is correct
Vcc
(c) Both the statements 1 and 2 are correct
(d) Both the statements 1 and 2 are incorrect
RA = 2.2 kΩ 8 4
Solution: (c)
Vcc Res
 A stable multi-vibrator can be used for generating
7 square wave Bistable multi-vibrator can be used for
Disch
storing binary information.
RB = 4.7 kΩ
555 Timer Hence, the correct option is (c).
6 3
Thresh Out

Two-marks Questions
2
Tring
Gnd 1. In the following astable multi-vibrator circuit, which
C 0.022 μF 1 properties of v0(t) depend on R2?
R1

Resistance RA = 2.2 k Ω
Solution:  −
V0(t )
Resistance RB = 4.7 k Ω  +
R3
C
Capacitance C = 0.022 μ f 
R2 R4
For Astable multivibrator frequency can be expressed as
1  [2009]
f =
0.69 ( RA + 2 RB ) C (a) Only the frequency
1 (b) Only the amplitude
= (c) Both the amplitude and the frequency
0.69 × 11.6 × 103 × 0.022 × 10 −6
(d) Neither the amplitude nor the frequency
=
5.67 kHz Solution: (a)
In the given a stable multi-vibrator circuit, only the fre-
Hence, the correct Answer is (5.67 kHz). quency of V0(t) depends on R2.
5.116 | Analog Electronics

R1 3. An astable multi-vibrator circuit using IC 555 timer


is shown below. Assume that the circuit is oscillating
steadily.

V0(t ) 9 kΩ
+ 30 kΩ
R3
C
4 8
(Reset) (Supply)
R2 R4 6 (Threshold)

10 kΩ (Output) 3

Hence, the correct option is (a). 2 (Trigger) (Gnd)


(Discharge) 1
2. Consider the Schmidt trigger circuit shown below: 7
R1
12 kΩ
+15V
10 kΩ Vc 0.01 µF
V1 −
V0
+

10 kΩ
The voltage VC across the capacitor varies between
 [2008]
10 kΩ (a) 3 V and 5 V
−15V (b) 3 V and 6 V
(c) 3.6 V and 6 V
(d) 3.6 V and 5 V
A triangular wave which goes from −12 V to 12 V is
applied to the inverting input of the op-amp. ­Assume Solution: (b)
that the output of the op-amp swings from +15 V to 9k
−15 V. The voltage at the non-inverting input switches 30 k
between [2008]
4 8
(a) −12 V and +12 V (Reset) (Supply)
(b) −7.5 and +7.5 V
6 (Threshold)
(c) −5 V and +5 V
(d) 0 V and 5 V 10 kΩ (Output) 3

Solution: (c) 2 (Trigger) (Gnd)


Let VA be the voltage at non-inverting terminal KCL at (Discharge) 1
this node, 7
+15V
10 k
12 kΩ
Vi − Vc 0.01 µF
V0
+

10 k 2Vcc
The voltage VC varies between Vce to given that,
Vcc = 9 V 3
10 k ∴, voltage Vc varies between 3 V and 6 V.
−15V
Vc

6
VA − 15 VA + 15 VA − V0
+ + =0 3
10k 10k 10k
V t
⇒ VA = 0
3 Hence, the correct option is (b).
Since output voltage swings between +15 V and –15 V, 4. Given the ideal operational amplifier circuit shown in
VA swings between +5 V and –5 V. the figure indicates the correct transfer characteristics
Hence, the correct option is (c). assuming ideal diodes with zero cut-in voltage.
Chapter 9  Multi-vibrators and 555 Timers  |  5.117

+10V V0
Vi − +10V
V0
+
−10V
−5V +8V
Vi
2 kΩ

0.5 kΩ −10V
2 kΩ

2
V0T = 10 × = 8V
(a) V0 2 + 0.5
+10V
⇒ if Vi > 8 then V0 is negative

−8V −5V
Vi
When upper diode is ON,

Ve1 = VB2 = 0.7 V


  −10V
VB2e 2 = 0.7 V
(b)  +10V
V0 Hence, the correct option is (a).
V − V = 0.7 V
B2 e2
5. An ideal saw-tooth voltage waveform of frequency
−5V +8V
Vi
0.7 − Ve32 V=is0.generated
500 Hz and amplitude 7 by charging a
capacitor of 2 μF in every cycle. The charging requires
 Ve 2 = 0 V [2003]
−10V (a) constant voltage source of 3 V for 1 ms
(b) constant voltage source of 3 V for 2 ms
(c)  +5V
V0
(c) constant current source of mA for 1 ms
(d) constant current source of 3 mA for 2 ms
−5V +5V
Vi Solution: (d)
0 = I e Re − 15
−10V
15 15
Ie = =
(d)  +10V
V0
Re 8
Vce = VC − Ve = 15 − 0
−5V +5V = 15 V
Vi

−5V  [2005]
Solution: (a)
When lower diode is ON, Hence, the correct option is (d).
+10V
Vi −
+
V0
Five-mark Questions
−10V
1. Implement a monostable multi-vibrator using the timer
2k
circuit shown in figure. Also determine an expression
for ON time T of the output pulse.
0.5 k [1998]
2 kΩ
5.118 | Analog Electronics

Vcc Vcc

R
Comparator-1 RA
− 4 8 Discharge
Reset Vcc
Threshold +
S Q output output Trigger
R 3 IC 555 7 RB
2 Comparator-2
S Q

GND Threshold
Trigger + 5 0.01 µF
R Control
Discharge R
0.05 µF
GND
GND Evaluate values of RA and RB if the capacitor has the
values of 0.01 μF for the configuration chosen. If neces-
 R  sary you can suggest modifications in the external cir-
 R+ 2  3 cuit configuration.[1997]
Solution: Vth = VCC   = VCC Solution:  f = 10 kHz
R + R + R 5
 2  1
T= = 100 m sec
f
2VCC
Vtrig = Ton + Toff = 100 μ sec
5
100
3 VCC 3VCC Toff = Ton = = 50 m sec
Ton = time to change from 0 to = VCC [1 − e − ti / RA C A
] 2
5 5
3VCC TH  = 0.7 (RA + RB)C
= VCC [1 − e − ti / RAC A ]  TL  = 0.7 RBC
5
3 Since it is not possible to get square wave with this con-
e − ti / RAC A = 1 − = 0.4 figuration.
5
∴ Diode is connected across RB
−ti
= ln(0.4) = −0.916
RA C A
RC

Ton = 0.916 RACA


RB

2. An IC-555 chip has been used to construct a pulse


C
generator. Typical pin connections with components is
shown below is figure for such an application. However
it is desired to generate a square pulse of 10 kHz. Since Diode on during charging and diode off during
discharging.
TH = 0.7 RAC
TC = 0.7 RAC
50 × 10 −6
RA = RB = = 7.14kΩ
0.7 × 10 −8
Unit VI
Digital Circuits

Chapter 1: Number Systems 6.3


Chapter 2: Boolean Algebra 6.6
Chapter 3: Logic Gates 6.13
Chapter 4: Combinational Circuits 6.26
Chapter 5: Sequential Circuits 6.42
Chapter 6: Logic Families 6.64
Chapter 7: Memories 6.73
Chapter 8 ADC and DAC 6.78
EXAM ANALYSIS
Exam Year 92 93 94 95 96 97 98 99 00 01 02 03 04 05 06 07 08 09 10 11 12 13 14-1 14-2 14-3 14-4 15 16 17 18 19
Set 1 Set 2 Set 3 Set 1 Set 2 Set 3 Set 1 Set 2
1 Marks Ques. 4 4 6 6 1 9 9 4 3 3 3 5 6 2 - 2 - 1 2 3 4 1 1 4 2 2 1 2 1 2 7 2 2 2 3 3
2 Marks Ques. - - 1 - - - 1 1 4 4 3 7 7 5 6 7 8 6 2 2 1 2 2 1 3 4 3 2 1 7 4 4 3 3 3 3
5 Marks Ques. - 1 - 2 4 1 1 1 1 1 1
Total Marks 4 09 08 16 21 14 16 17 16 16 21 19 20 12 12 16 16 13 6 7 6 5 5 6 8 10 7 6 3 16 15 10 8 8 9 9

Chapter wise marks


distribution
Number Systems - 1 - - - 1 1 - - 1 1 - 5 1 2 1 2 - - - - - - 1 - 0 1 0 0 0
Boolean Algebra - - - - - - 2 3 - - - 3 2 2 - 3 - 2 - - 1 2 3 1 - 2 0 1 1 0
Logic Gates - - 2 2 1 - 2 2 3 3 3 - 3 2 2 - 2 - 2 1 - - 1 2 - - 5 2 1 0 2
Combinational Circuits 1 - - 1 - 1 - 2 - 2 2 5 2 1 2 2 2 6 2 1 1 - - 3 3 1 4 5 1 0
Sequential Circuits 1 1 1 2 - 1 3 2 4 2 - 3 4 4 4 4 4 4 2 5 3 - - 1 5 4 3 0 1 1
Logic Families 1 - 2 - - 4 2 2 - 2 - 3 1 4 - 2 2 1 - - 1 - - - - 1 0 1 2 2 0
Memories 1 - 2 1 1 - - 2 - 3 - 2 - 2 - - - - - - - 2 - - 2 - 0 0 2 1 0
ADC and DAC - - 1 1 - - 1 1 4 - 1 3 1 - 2 4 4 - - - - - - - - - 1 1 0 0 0
Chapter 1
Number Systems
Solution: (b)
One-mark Questions (43)10
(i) BCD: 4 →0100 3→0011
1. The number of bytes required to represent the deci-
mal number 1856357 in packed BCD (Binary Coded ⇒ ( 43)10 = (0100 0011) BCD
Decimal) form is________. [2014] (ii) Hexadecimal: 43 = 32 + 11 ⇒ (2B)16
Solution: (2B, 0100 0011)
Given number is (1856357)10. Hence, the correct option is (b).
In BCD one digit is represented by 4 bits ,so 4. The range of signed decimal numbers that can be rep-
resented by 6-bit 1’s complement number is
(a) −31 to +31 (b) −63 to + 63
(c) −64 to + 63 (d) −32 to + 3
 [2004]
2. X = 01110 and Y = 11001 are two 5-bit binary numbers Solution: (a)
represented in 2’s complement format. The sum of X 2, s complement form, MSB is sign bit and reset of 5 bit
and Y represented in 2’s complement format using 6 is mag bits.
bits is ⇒ range = −(25 – 1) to +(25 – 1)
(a) 100111 (b) 001000
= −31 to 31
(c) 000111 (d) 101001 [2007]
Hence, the correct option is. (a)
Solution: (c)
5. 4-bit 2’s complement representation of a decimal num-
2’s complement
ber is 1000. The number is
 X = 01110 MSB = 0 ⇒ + ve number = +14
 (a) +8 (b) 0
Y = 11001 MSB = −1 ⇒ + ve no
(c) −7 (d) −8 [2002]
Now as Y is –ve , sum of X & Y = X + ( −Y ) Solution: (d)
= X + 2’sc (Y ) 2’s complement = 1000——, MSB = 1 ⇒ – ve number.
Sum = X + 2’s comp of Y Take inverse of 2’s comp:
+ 11001 Subt 1: 1000
Neglect ← 100111 = 000111 −1
↓ 0111
     +ve result (0 added to make 6th bit) Take 1’s comp = 1000 = 8 (mag) and sign = –ve
Hence, the correct option is (c). ⇒ number of (–8)
3. Decimal 43 in hexadecimal and BCD number system Hence, the correct option is (d).
is, respectively, 6. The 2’s complement representation of −17 is
(a) B2, 01000011 (b) 2B, 01000011 (a) 101110 (b) 101111
(c) 2B, 0011 0100 (d) B2, 0100 0100[2005] (c) 111110 (d) 110001 [2001]
6.4 | Digital Circuits

Solution: (b) In 2’s complement form, if MSB 1 ⇒ –ve number


2’s complement of –17 = 0 ⇒ +ve number
+ve 17 = 010001(6 bit representation) ∴To extend it to 16-bit word, copy the original byte to
Take 1’s comp = 101110 the lower byte (less significant) of word and copy the
MSB (sign bit) of original byte in higher byte of word
add 1 + 1
Hence, the correct option is (c).
101111 = (–17)
9. 2’s complement representation of a 16-bit number (one
Hence, the correct option is (b).
sign bit and 15 magnitude bits) is FFFF. Its magnitude
7. An equivalent 2’s complement representation of the 2’s in decimal representation is
complement number 1101 is (a) 0 (b) 1
(a) 110100 (b) 001101 (c) 32,767 (d) 65, 535 [1993]
(c) 110111 (d) 111101 [1998]
Solution: (b)
Solution:(d)
2’s complement 16 bit no (1 sign + 15mag bit) = FFFF
2’s complement representation = 1101 = 1111 1111 1111 1111
Now to extend the number into more number of bytes, 2’s complement of 111 1111 1111 1111 is 000 0000
the MSB (sign) is copied to left side 0001 and its decimal equivalent is 1 with negative sign.
∴ Decimal number is –1 and its mag is 1.
Hence, the correct option is (b).
1 0. The subtraction of a binary number Y from another
binary number X, done by adding the 2’s complement
of Y to X, results in a binary number without overflow.
This implies that the result is:
Hence, the correct option is (d). (a) negative and is in normal form.
8. A signed integer has been stored in a byte using the 2’s (b) negative and is in 2’s complement form.
complement format. We wish to store the same integer (c) positive and is in normal form.
in a 16 bit word. We should: (d) positive and is in 2’s complement form.
(a) copy the original byte to the less significant byte  [1987]
of the word and fill the more significant byte with Solution: (b)
zeros. For X  Y operation, using 2’s component form, if result
(b) copy the original byte to the more significant byte of has
the word and fill the less significant byte with zeros. i) overflow, then it is discarded and result is positive
(c) copy the original byte to the less significant byte of the and in normal format.
word and make each bit of the more significant byte ii) no overflow, then result is negative and in 2’s com-
equal to the most significant bit of the original byte. plement form.
(d) copy the original byte to the less significant bytes
well as the more significant byte of the word. Hence, the correct option is (b).
 [1997]
Solution: (c) Two-marks Questions
In 2's complete form , if MSB =1⇒ -ve number
1. The two numbers represented in signed 2’s complement
   =0 ⇒+ve number
form are P = 11101101 and Q = 11100110. If Q is sub-
∴To extended it to 16-bit word , copy the original byte tracted from P, the value obtained in signed 2’s comple-
(less significance) of word and copy the MSB (sign bit) ment form is
of original byte in higher byte of word (a) 10000011 (b) 00000111
(c) 11111001 (d) 111111001 [2008]
Solution: (b)
P = 11101101 ( − ve) = −19
Q = 11100110 ( − ve) = −26
Then P − Q = ( − ve) | P | −( − ve | Q |) = −ve | P | + | Q |
Chapter 1  Number Systems  |  6.5

=| Q | − | P | (mag) 100 010 011 001


Then for (make pair of 3-bits)
= 26 − 19 = 7 ↓ ↓ ↓ ↓
4 2 3 1
= 000111
(100010011001)5 = ( 4231)10
Hence, the correct option is (b).
Hence, the correct option is (d).
2. A new Binary Coded Pentary (BCP) number system
is proposed in which every digit of a base-5 number is 3. 11001, 1001 and 111001 correspond to the 2’s comple-
represented by its corresponding 3-bit binary code. For ment representation of which of the following sets of
example, the base-5 number 24 will be represented by number?
its BCP code 010100. In this numbering system, the (a) 25, 9 and 57, respectively
BCP code 100010011001 corresponds to the following (b) −6, −6 and −6, respectively
number in base-5 system: (c) −7, −7 and −7, respectively
(a) 423 (b) 1324 (d) −25, −9 and −57, respectively [2004]
(c) 2201 (d) 4231 [2006] Solution: is (c)
Solution: (d) 11001 inv
 2;s
→ −00111 = −7
BPC (base –5) 1001 inv
 2;s
→ −0111 = −7
Carry digit → 3-bit binary code
⇒ 2 → 010 111001  → −000111 = −7
( 24)10 = (010 100)5 *(For inv. 2’s complement, subtract (+1) and then take
4 → 100
1’s comp)
⇒ (6) –7, –7, –7
Hence, the correct option is (c).
Chapter 2
Boolean Algebra
One-mark Questions (A) (X + Y + Z)(X + Y + Z )( X + Y + Z )

1. A function F(A, B, C) defined by three Boolean vari-


(
(B) X + Y + Z )( X +Y + Z )( X +Y + Z )
ables A, B and C when expressed as sum of products is
given by
( X +Y + Z )
(C) ( X +Y + Z )( X +Y + Z )( X +Y + Z )
F = A⋅ B ⋅C + A⋅ B ⋅C + A⋅ B ⋅C
Where, A, B and C are the complements of the respec-
( X +Y + Z )
tive variables. The product of sums (POS) form of the
function F is. [2018]
(D) ( X +Y + Z )( X +Y + Z )( X +Y + Z )
F = (A + B + C) (A + B + C) ( A + B + C)
(A)
( X +Y + Z )
F = ( A + B + C ) ( A + B + C ) (A + B + C )
(B) F(X, Y, Z) = xyz + xy z + xy z + xyz
Solution: 
=
∑m(2, 4, 6, 7) =∏M(0, 1, 3, 5)
F = (A + B + C ) ⋅ (A + B + C ) ⋅ ( A + B + C )
(C) 
(
( A + B + C) A + B + C ) = (X + Y + Z)(X + Y + Z ) (X + Y + Z )( X + Y + Z
)
F = ( A + B + C) ⋅ ( A + B + C) ⋅ (A + B + C)
(D) 
(A + B + C ) (A + B + C) Hence, the correct option is (A).
Solution: sum of products function is given as 3. A function of Boolean variables X, Y and Z is expressed
in terms of the min-terms as
F = A ⋅ B ⋅C + A ⋅ B ⋅C + A⋅ B ⋅C
F(X, Y, Z) = Σ (1, 2, 5, 6, 7)
The K- map for the above function is given below
Which one of the product of sums given below is equal
BC to the function F(X, Y, Z)? [2015]
A 00 01 11 10

0 1 0 0 1 ( )( )(
(A) X + Y + Z . X + Y + Z . X + Y + Z )
1 1 0 0 0 (B) ( X + Y + Z ).( X + Y + Z ).( X + Y + Z )

From the above K- map the product of sums (POS)


(C) ( X +Y + Z )⋅( X +Y + Z )⋅( X +Y + Z )⋅
form of the function F is ( X +Y + Z )⋅( X +Y + Z )
F = (A + B + C)⋅(A + B + C)⋅
( A + B + C) ⋅( A + B + C) ⋅( A + B + C)
(D) ( X +Y + Z )⋅( X +Y + Z )⋅( X +Y + Z )⋅
Hence, the correct option is (C)
( X +Y + Z )⋅( X +Y + Z )
2. The Boolean expression F(X, Y, Z) = XY Z + X Y Z + F(x, y, z) = ∑m(1, 2, 5, 6, 7)
Solution: 
XY Z + XYZ converted into the canonical product of
= ∏ M (0, 3, 4)
sum (POS) form is [2015]
Chapter 2  Boolean Algebra  |  6.7

= (x + y + z) (x + y + z ) (x + y + z)

Hence, the correct option is (B).

4. The Boolean expression


( X + Y )( X + Y ) + ( XY ) + X simplifies to
(a) X
(b) Y
(c) XY
Hence, the correct option is (b).
(d) X + Y [2014]
8. The number of distinct Boolean expressions of 4 vari-
Solution: (a)
ables is
y = ( x + y )( x + y )( x y + x ) (a) 16 (b) 256
xx = x (c) 1024 (d) 65536 [2003]
= ( x ⋅ x + y ⋅ x + x y + y ⋅ y )( x( y + 1))
yy = 0 Solution: (d)
= ( x + xy + x y )( x ) y +1 = 1 For 4 variables:
= x (1 ) x = x n
For n var 22 function
4
Hence, the correct option is (a). ⇒22 = 216 = 1024 × 26 = 65536
5. For an n-variable Boolean function, the maximum Hence, the correct option is (d).
number of prime implicants is 9. The logical expression Y = A + AB is equivalent to
(a) 2(n – 1) (b) n/2 (a) y = AB (b) y = AB
(c) 2n (d) 2(n-1) [2014] (c) y = A + B (d) y = A+ B [1999]
Solution: (d) Solution: (d)
Hence, the correct option is (d). Y = A + AB by distribution law:
6. In the sum of products function f(X, Y, Z) = ∑(2, 3, 4, 5)
A + AB = ( A + A)( A + B )
the prime implicates are ↓
1
(a) XY , XY
(b)
XY , XY Z , XYZ Y = ( A + A)( A + B)
(c)
XYZ , XYZ , XY Y = A+ B
(d)
XYZ , XYZ , XY Z , XYZ  [2012] Hence, the correct option is (d).
Solution: (d) 1 0. The K-map for a Boolean function is shown in the fig-
ure. The number of essential prime implicants for this
function is

F(x, y, z) = {(1, 2, 3)}


⇒ PI = xy + x y

Hence, the correct option is (d).
7. The Boolean function Y = AB + CD is to be realized
using only 2-input NAND gates. The minimum number
of gates required is
(a) 2 (b) 3
(c) 4 (d) 5 [2007] (a) 4 (b) 5
Solution: (b) (c) 6 (d) 8 [1998]
y = AB + CD
6.8 | Digital Circuits

Solution: (a)
Two-marks Questions
1. Which one of the following gives the simplified sum
of products expression for the Boolean function F = m0
+ m2 + m3 + m4, where m0, m2, m3 and m5 are minterms
corresponding to the inputs A, B and C with A as the
MSB and C as the LSB ?  [2017]
A B+ABC+A B C
(A)

(B) AC + A B + A B C
Quad : 1
Pair : 3 (C) AC + A B + A B C
∴ to cover all 1’s, Quad + 3 Pair A B C + AC + A B C
(D)
⇒ 4 PI
Hence, the correct option is (a). Solution:  Given that
  F = m0 + m2 + m3 + m5
11. Two 2’s complement numbers having sign bits x and ∴ F = I + II + III
y are added and the sign bit of the result is z. Then,
the occurrence of overflow is indicated by the Boolean I = AB
function. II = AC
(a) xyz (b) xyz III = ABC
(c) ( X Y Z + XY Z ) (d) xy + yz + zx
∴ Simplified F = A B + AC + ABC .
 [1998]
Solution: (c) 00 01 11
2n 2’s complement, if no having sign bit X & Y are m0 m3 m2
added, then overflow occurs; if X and Y are the same 1 10
(same sign) and resultant sign changes 1 1
0
e.g., X = Y = 0 (+ve both), then Z should be 0 if Z
  = 1 ⇒ overflow
1
X =Y = 1 (–ve) ⇒ (–ve) + (–ve): –ve (Z = 1) but Z 1
  = 0 overflow
II
⇒ overflow: ( X Y Z + XY Z ) I
III
Hence, the correct option is (c). Hence, the correct option is (B).
12. The number of Boolean functions that can be generated 2. Identify the circuit below [2016]
by n variables is equal to:
(a) 22n-1 (b) 22n
OP0 IP0
(c) 2n−1 (d) 2n [1990] OP1 IP1
X2 Y2
Solution: (b) OP2 IP2
3.8 OP3 IP3 8.3
For n variable function, as n can be 0 or 1 X1
Decoder OP4 IP4 Encoder
Y1
⇒ 2n input combination is possible OP5 IP5
X0 Y0
OP6 IP6
2 n
OP7 IP7
I/P

n
⇒ 22 function possible (A) Binary to gray code converter
(As if n = 2 (A, B), 4 input: AB , AB , AB, AB ) (B) Binary to XS3 converter
(C) Gray to binary converter
For 4 input, 24 function can be made (16).
(D) XS3 to binary converter
Hence, the correct option is (b).
Chapter 2  Boolean Algebra  |  6.9

Solution: 
Consider the table given below From the given K-maps
Decoder Encoder F(PQRSX) = S .Q X + S .QX
X2 X1 X0 Output Input Y2 Y1 Y0
Hence, the correct option is (B).
0 0 0 OP0 IP0 0 0 0
(from the given K-maps
0 0 1 OP1 IP1 0 0 1
When X = 0
0 1 0 OP2 IP3 0 1 1
F1(PQRSX) = SQ′X ′
0 1 1 OP3 IP2 0 1 0
1 0 0 OP4 IP6 1 1 0 and F2(PQRSX) = S′QX
1 0 1 OP5 IP7 1 1 1 Hence, F(PQRSX) = SQ′X ′+ S′QX
1 1 0 OP6 IP4 1 0 0
1 1 1 OP7 IP5 1 0 1
options with conditions of X = 0 and X = 1 and K-Maps
option (B) gives the correct result)
Binary to gray code converter except last 2 conditions. Hence, the correct option is (B).
Hence, the correct option is (A). 4. Consider the Boolean function
3. Following is the K–map of a Boolean function of five F ( w , x, y, z ) = wy + xy + wxyz + wxy + xz + x y z .
variables P, Q, R, S and X. The minimum sum of prod-
Which one of the following is the complete set of
uct (SOP) expression for the function is  [2016]
essential prime implicants?
PQ PQ (a) w , y, xz , x z (b) w, y, xz
RS 00 01 11 10 RS 00 01 11 10
00 0 0 0 0 00 0 1 1 0 (c) y , x y z (d) y, xz , x z  [2014]
01 1 0 0 1 01 1 0 0 1 Solution:(d)
f ( w , x, y, z ) = wy + xy + wxyz + wxy + xz + x yz
11 1 0 0 1 11 1 0 0 1
10 0 0 0 0 10 0 1 1 0
For essential PI, we reduce f
x=0 x=1
f = wy + xy + zx(1 + wy ) + xwy + x y z
=1
(A)
P Q S X + P Q S X + Q R S X + QR S X
(B)
QS X +Q S X = w y + xy + xz + xw y + x y z

(C)
QSX+Q S X = y( w + xw ) + xy + xz + x y z

(D)
QS + QS = yw + y x + xy + xz + x y z
= y( w + x + x ) + xz + x y z
Solution: 
Given, =1
PQ
RS
= y + xz + x y z = xz + ( y + y )( y + x z )
00 01 11 10 =1
00
f = xz + y + x z
01 1 1
⇒ PI : y, xz , x z
11 1 1

10 Hence, the correct option is (d).


X=0 5. In the circuit shown below, Q1 has negligible collector-
to-emitter saturation voltage and the diode drops neg-
ligible voltage across it under forward bias. If Vcc is +5
PQ
RS
V, X and Y are digital signals with 0 V as logic 0 and Vcc
00 01 11 10
as logic 1, the Boolean expression for Z is
00 1 1

01 1

11 1

10 1 1
X=1


6.10 | Digital Circuits

(a) XY (b) XY Solution: (d)


(c) XY (d) XY  [2013] y = ABCD + ABC D + ABCD + ABC D
Solution: (b)
= ( A + A) BCD + ABC D + ABCD

y = BC D + ABC D + ABCD
Hence, the correct option is (d).
8. The Boolean expression for the truth table shown is

Input x and y output: z


a) x = 0 y = 0 ⇒ Q = is OFF z = 0
b) x = 0 y = 5 ⇒ Q : OFF z = 5
c) x = 5 y = 0 ⇒ Q : ON z = 0
d) x = 5 y = 5 ⇒ Q : ON z = 0
X Y Z (a) B (A + C) ( A + C )
0 0 0
B (A + C )( A + C)
(b)
0 1 1 ⇒ Z = XY
(c) (A + C )( A + C)
1 0 0
1 1 0 B ( A + C )( A + C ) 
(d) [2005]

Hence, the correct option is (b). Solution: (a)


6. If X = 1 in the logic equation From truth table f = ABC + ABC
[ X + Z {Y + ( Z + XY )}]{ X + Z ( X + Y )} = 1, = B( AC + AC )
Then or
(a) Y=Z
(b) Y =Z f = B( AC + AC + AA + CC )
(c) Z=1
  = B( A + C )( A + C ) = { AA 0=
CC 0}
(d) Z = 0 [2009]
Solution:(d) Hence, the correct option is (a).

 { }
 x + z y + ( z + x y)   x + z( x + y)  = 1
  
9. The Boolean expression AC + BC is equivalent to
(a) AC + BC + AC
put x = 1 (b) BC + AC + BC + ACB
   (c) AC + BC + BC + ABC
 
⇒ 1 + z { } 0 + z (1 + y )  = 1
 
    (d) ABC + ABC + ABC + ABC  [2004]
=1 
 =1   
Solution: (d)

⇒ z =1⇒ z = 0 y = AC + BC
Hence, the correct option is (d). By k-MAP:
7. The Boolean expression

Y = ABCD + ABCD + ABCD + ABCD can be mini-


mized to
Y = ABCD + ABC + ACD
(a)
Y = ABCD + BCD + ABCD
(b)
Y = ABCD + BCD + ABCD or
(c)
Y = ABCD + BCD + ABCD 
(d) [2007] y = AC ( B + B) + BC ( A + A)
Chapter 2  Boolean Algebra  |  6.11

11. The minimized form of the logical expression


y = ABC + ABC + ABC + ABC ( ABC + ABC + ABC + ABC ) is
Hence, the correct option is (d). (a) AC + BC + AB
1 0. If the functions W, X, Y and Z are as follows (b) AC + BC + AB

W = R + PQ + RS (c) AC + BC + AB
(d) AC + BC + AB  [1999]
X = PQRS + PQRS + PQRS Solution: (a)
Y = RS + PR + PQ + PQ Y = ( ABC + ABC + ABC + ABC )t

Z = R + S + PQ + PQR + PQS = A( B + B)C + ABC + ABC B + B =1


= AC + ABC + ABC = A(C + BC ) + ABC
then
(a) =W Z= W = Z, X = y
, X Z (b) = A(C + B)(C + C ) + ABC (distribution law )
(c) W = Y (d) W = Y= Z = AC + AB + ABC = 1
 [2003]
= C ( A + AB) + AB = C ( A + A)( A + B) + AB
Solution: (a)
W = R + PQ + RS Y = C A + CB + AB
X = PQRS + PQRS + PQRS Hence, the correct option is (a).

Y = RS + PR + PQ + PQ
Five-marks Questions
Z = R + S + PQ + PQR + PQS
Now 1. Given the Boolean function F in three variables R, S
and T as
W = R + RS + PQ = ( R + R)( R + S ) + PQ (1)
=1
F = RST + RRT + RST
W = R + S + PQ
(a) Express F in the minimum sum-of-products form.
X = PQRS + ( P + P )QRS = PQRS + QRS (2) (b) Express F in the minimum product-of-sums form.
(c) Assuming that both true and complement forms of
= RS (Q + PQ ) = RS (Q + Q )(Q + P )
=1
the input variables are available, draw a circuit to
implement F using the minimum number of 2-in-
= RSQ + RSP [1996]
Y = RS + PR + Q( P + P ) = RS + PR + Q = RS + ( P + P )( P + R) put NAND gates only.
=1 =1
Solution: (a) F = RST + RST + RST
= RS + P +
Y = RS + PR + Q( P + P )Y= =RSRS+ +PR R = RS + PR ⋅ Q = SR + ( R + P ) Q = RST + RT ( S + S )
=1 PR+ +QQ=( PRS+ +P )( P= +RSP )( P + R)
=1 + PR + Q = RS + ( P + P )( P + R)
= Y = SR + RQ + PQ=1 =1 = RST + RT
= RS + P + R = RS + PR=⋅ QRS= +SR + ( R + P ) Q
P + R = RS + PR ⋅ Q = SR + ( R + P )Q (b) F = RST + RST + RST
= Y = SR + RQ + PQ = Y = SR + RQ + PQ ∑m(2, 5, 7)

3
Z = R +ZS =+ P
R (+QS++SP) (+QPQR
+ S ) + PQR
= R + S =+ P
R (+SS++QP)((P
S +Q
QR)()P + QR)
= R + S =+ (RP++SSQ PP+ +SQ
+ ()( Q )(+PR+
) Q + R)
= R + S =+ PS
R +QS ++ PQ ++
PS Q + SQR
PRPQ + PR + SQR
= R + S =+ P
R (+QS++RP) (=QR++RS) =+ PQ
R + S + PQ To possible solves
Solution 1 → F = ( R + S )( R + T )( R + T )
+ PR = R + S =+ PQ
+ PR R + S + PQ (P + 1 = 1) 4
From (1) (2) (4) w = z. From (2) and (4) x = z . Solution 2 → F = ( S + T )( R + T )( R + T )
Hence, the correct option is (a). (c) NAND – NAND → AND-OR
F = RST + RT
6.12 | Digital Circuits

Hence, the correct option is (a)


Chapter 3
Logic Gates
2. In the circuit shown. A and B are the inputs and F is the
One-mark Questions output. What is the functionality of the circuit? [2019]
Vdd
1. In the circuit shown, what are the values of F for EN =
0 and EN = 1, respectively? [2019]
Vdd

EN

F
F

A B
(A) 0 and 1 (B) Hi-Z and D (A) XNOR (B) SRAM Cell
(C) Hi-Z and D (D) 0 and D (C) XOR (D) Latch
Solution: Solution:
VDD

X
EN

D
Y

X = EN ⋅ D
A B
Y = EN + D On simplification

When EN = 0 When EN = 1
B
X = 0⋅ D = 1 X = 1⋅ D = D
A
Y = 0+ D = 0 Y = 1+ D = D
∴ F is Hight-z ∴ F is D
B A
Hence, the correct option is (B).
A B
6.14 | Digital Circuits

A B F (C) Either X = 1, Y = 1 or X = 0, Y = 0
(D) X = 0, Y = 0
0 0 1
Solution:  When p = 0, Q = 0 ⇒ x = 1, y = 1
0 1 0
1 0 0 when p = 1, Q= 1 ⇒ x = 1, y = 0
1 1 1 (or)
when p = 1, Q = 1 ⇒ x = 0, y = 1
∴ F is an XOR circuit.
p=01
3. The logic function f (X, Y) realized by the given circuit x = 10
is [2018]
VDD

X X

Y Y

f(X, Y)
Q=01 y=101

Hence, the correct option is (B).


5. The minimum number of 2-input NAND gates required
to implement a 2 input XOR gate is  [2016]
(A) 4 (B) 5
(A) NOR (B) AND
(C) 6 (D) 7
(C) NAND (D) XOR
Solution: 
For XOR gate
Solution: 
VDD
f = AB + A. B

X X
For the implementation of XOR gate, number of NAND
gates required can be obtained as
Y Y

f(X, Y)

A
f
B

f(X, Y) = XY + XY = X  Y = X ⊕ Y
Hence, the correct option is (D)
4. In the latch circuit show, the NAND gates have non- From above figure to design a XOR gate minimum 4
zero, but unequal propagation delays. The present NAND gates are required.
input condition is: P = Q = 0, If the input condition is
changed simultaneously to P = Q = 1, the outputs X and Hence, the correct option is (A).
Y are  [2017] 6. A universal logic gate can implement any Boolean
function by connecting sufficient number of them
P
X appropriately. Three gates are shown. [2015]
X F1 = X + Y X F2 = X ∙ Y

Y Y

Y Gate 1 Gate 2
Q

X F3 = X + Y
(A) X = 1, Y = 1
Y
(B) Either X = 1, Y = 0 or X = 0, Y = 1
Gate 3
Chapter 3  Logic Gates  |  6.15

Which one of the following statements is TRUE?


(A) Gate 1 is a universal gate.
20 ns 20 ns
(B) Gate 2 is a universal gate.
(C) Gate 3 is a universal gate. t=0
(D) None of the gates shown in a universal gate.
A
Solution:  A Universal logic gate can implement any B 20 ns
Boolean function by connecting sufficient number of
gates. 20 ns delay
20 ns of NOT gate
{AND, NOT}, {OR, NOT} are universal gates B

X AB 20 ns delay
X+0=X 40 ns of AND gate
0

NOR gate C 20 ns
20 ns delay
X X 40 ns of XOR gate
AB ⊕ C
0 X+Y=X+Y
Y Hence, the correct Answer is (40).
OR gate 8. A 3-input majority gate is defined by the logic func-
tion M(a, b, c) = ab + bc + ca. Which one of the
X X+Y following gates is represented by the function

Y
X + Y + 0 = XY
( ( ) )
M M ( a, b, c ), M a, b, c , c ? [2015]
0
(A) 3-input NAND gate
AND gate (B) 3-input XOR gate
(C) 3-input NOR gate
Hence, the correct option is (C). (D) 3-input XNOR gate
7. All the logic gates shown in the figure have propagation Solution:  3 input Majority gate M (a, b, c) = ab + bc +
delay of 20 ns. Let A = C = 0 and B = 1 until time t = 0. ac.
At t = 0, all the inputs flip (i.e., A = C = 1 and B = 0)
and remain in that state. For t > 0, output Z = 1 for a
( ( ) )
M M ( a, b, c ), M a, b, c , c = ?

duration (in ns) of [2015] =


M ( a, b, c) . M (a, b, c )
A + M (a, b, c) c + M (a, b, c ).c
B Z
= (ab + bc + ac)′(ab + bc′ + ac′)
C + (ab + bc + ac)′c. + (ab + bc′ + ac′) c
= (a′ + b′) (b′ + c′)(a′ + c′)(ab + bc′ + ac′)
Solution:  All the logic gates have same propagation + (a′ + b′)(b′ + c′)(a′ + c′) c
delay = 20 ns. + (ab + bc′ + ac′) c
0→1 = (a′c′ + b′) (a′ + c′) (ab + bc′ + ac′)
A
+ (a′c′ + b′) (a′ + c′).c + abc
B Z
= (a′b′ + a′c′ + b′c′) (ab + bc′ + ac′)
1→0 0→1
C + (a′b′ + a′c′ + b′c′)c + abc
0→1
=
a′bc′ + ab′c′ + a′b′c + abc
6.16 | Digital Circuits

=  ∑m (1, 2, 4, 7) = a ⊕ b ⊕ c → 3 input XOR ⇒A B Y


gate
0 0 0
Odd no. of 1’s in the minterms = XOR gate
Hence, the correct option is (B). 0 1 1
1 0 1
9. In the circuit shown in the figure, if C = 0, the expres- 1 1 0
sion for Y is  
  ⇒ Y = AB + AB
  ⇒ Y = A ⊕ B
Hence, the correct option is (c).
1 1. The output Y in the circuit below is always `1’ when

(a) Y = AB + AB (b) Y = A + B
(c) Y = A + B (d) Y = AB [2014]
Solution: (a) (a) two or more of the inputs P, Q, R are `0’
(b) two or more of the inputs P, Q, R are `1’
(c) any odd number of the inputs P, Q, R is `0’
(d) any odd number of the inputs P, Q, R is `1’
[2011]
Solution: (b)

(C = 0)
Y = ( A + B + C/ + AB)(C ) = 1 Y = PQ ⋅ QR + PR = PQ + QR + PR
=0
Y = PQ + QR + PR
Y = ( A + B + AB)
For y = 1, either PQ or QR or PR = 1 (at least any one)
(
Y = A + B + AB ) ⇒ 2 or more of P, Q, R = 1.
= ( A + B) AB Hence, the correct option is (b).

1 2. Match the logic gates in Column-A with their equiva-
Y = ( A + B)( A + B ) = AB + AB. lents in Column-B.
Hence, the correct option is (a). Column-A Column-B
10. A bulb in a staircase has two switches, one switch being
at the ground floor and the other one at the first floor.
The bulb can be turned ON and also can be turned OFF
by any one of the switches irrespective of the state of
the other switch. The logic of switching of the bulb
resembles
(a) an AND gate (b) an OR gate
(c) an XOR gate (d) a NAND gate [2013]
(a) P-2, Q-4, R-1, S-3
Solution: (c)
(b) P-4, Q-2, R-1, S-3
(1) Bulb can be turned ON/OFF by any one of switch (c) P-2, Q-4, R-3, S-1
  ⇒ for (0,1) and (1,0) input, output = 1, else zero (d) P-4, Q-2, R-3, S-1 [2010]
Chapter 3  Logic Gates  |  6.17

Solution: (d) 14. The figure shows the internal schematic of a TTL
AND-OR-lnvert (AOI) gate. For the inputs shown in
the figure, the output Y is

(a) 0 (b) 1
P : NOR= AB = A + B ( 4)
(c) AB (d) AB  [2004]

Q : NAND( A + B) = A + B = ( AB) ( 2) Solution:(a)
R: X – OR = (3)
S: X – NOR = (1)
( A ⊕ B ) = A B + AB = AB + AB = A ⊕ B

A ⊕ B = AB + AB = A ⊕ B
⇒ P – 4, Q – 2, R – 3, S – 1.
Now for TTL, floating input considers 1.
Hence, the correct option is (d).
13. For the output F to be 1 is the logic circuit shown, the y = (1 ⋅1) + ( A ⋅ B ) = 1 + AB AB + 1 = 1
input combination should be y =1= 0

y = 0.
Hence, the correct option is (a).
15. If the input to the digital circuit (in the figure) consist-
ing of a cascade of 20 XOR gates is X, then the output
Y is equal to
(a) A = 1, B = 1, C = 0 (b) A = 1, B = 0, C = 0
(c) A = 0, B = 1, C = 0 (d) A = 0, B = 0, C = 1
 [2010]
Solution: (d) (a) 0 (b) 1
(c) X (d) X [2002]
Solution: (b)

F = A ⊕ B⊕A⊕B⊕C X ⊕ 1 = X ⋅1 + X ⋅1 = X + X ⋅ 0 (1st X -OR )


Now
If f = 1 ⇒ odd number of input = 1

If C = 0 ⇒ both
A ⊕ B & A ⊕ B = 1 → ( not possible) X ⊕ 1 = X
For example 2nd X-OR:
C = 1 ⇒ either A ⊕ B = 1 or A⊕B = 1
 A = 0, B = 1 X ⊕X = X⋅X + X⋅X = X⋅X + XX = X +X

 or A = 1, B = 0 =1
⇒ either C =1 
 or A = 1, B = 1 ∴ For X-OR of odd number, output = X
or A = 0, B = 0
 Even number of output = 1
⇒ ( A = 0 = B , C = 1) ⇒ for y = output of 20th X-OR, y = 1.
Hence, the correct option is (d). Hence, the correct option is (b).
6.18 | Digital Circuits

16. For the ring oscillator shown in the figure, the propaga- Solution: (c)
tion delay of each inverter is 100 pico sec. What is the
fundamental frequency of the oscillator output? Z = ABC ,

(a) 10 MHz (b) 100 MHz


(c) 1GHz (d) 2GHz [2001]
Solution: (c)

Hence, the correct option is (c).


TPD = 100 p sec (1 inv) 19. For the identity AB + AC + BC = AB + AC , the dual
1 form is
As f = (N = n umber of inv in ring oscillator) (a) ( A + B)( A + C )( B + C ) = ( A + B)( A + C )
2 Ntpd
1
= ( N = 5) ( A + B )( A + C )( B + C ) = ( A + B )( A + C )
(b)
2 × 5 × 100 × 10 −12
1 1 ( A + B)( A + C )( B + C ) = ( A + B )( A + C )
(c)
= −12
= −9
1000 × 10 10 (d) AB + AC + BC = AB + AC  [1998]
9
f = 10 = 1 GHz Solution: (a)
Hence, the correct option is (c). AB + AC + BC = AB + AC
1 7. For the logic circuit shown in the figure, the required In duel form: •↔ +
input condition (A, B, C) to make the output (X) = 1 is
⇒ ( A + B ) ⋅ ( A + C ) ⋅ ( B + C ) = ( A + B ) ⋅ ( A + C ) (a)
Hence, the correct option is (a).

20. The output of the logic gate in figure is

(a) 1, 0, 1 (b) 0, 0, 1
(c) 1, 1, 1 (d) 0, 1, 1 [2000] (a) 0 (b) 1
(c) A (d) A [1997]
Solution: (d)
Solution: (c)

Now F = AO + A ⋅ O
For x = 1,
(a) c = 1   = A ⋅1 + O
(b) A⊕ B =1 ⇒ B ≠ A
F=A
(c) B ⊕ C = 1 ⇒ B = C = 1  (from a)
c = 1, B = 1, A ≠ B = 0  (from b) Hence, the correct option is (c).

⇒ ( A, B, C ) = (0, 1, 1) (from d) 21. The Boolean function A + BC is a reduced form of


Hence, the correct option is (d). (a) AB + BC (b) (A + B) . (A + C)
18. The minimum number of 2-input NAND gates required (c) AB + ABC (d) (A + C)⋅ B
to implement the Boolean function Z = ABC , assum-  [1997]
ing that A, B and C are available, is Solution: (b)
(a) two (b) three
(c) five (d) six [1998] y = A + BC
1
f = TPD = propagam delay of 1 inverter
2 Ntpd
1
⇒ TPD =
2 Nf Chapter 3  Logic Gates  |  6.19
N =5
Checking all equations: 1
⇒ TPD =
(a) AB + BC = B ( A + C ) ≠ A + BC 2 × 50 × 106
(b) ( A + B)( A + C ) = ( A + BC ) 1
= 7 = 10 −7 = 100 × 10 −9 sec
(c) AB + ABC ≠ A + BC 10
(d) ( A + C ) B ≠ A + BC TPD = 100 n sec

Hence, the correct option is (b). 2 5. Boolean expression for the output of XNOR (equiva-
2 2. The minimum number of NAND gates required to lence) logic gate with inputs A and B is
implement the Boolean function A + AB + ABC is (a) A B + A B (b) AB + AB
equal to (c) ( A + B)( A + B ) (d) ( A + B )( A + B)
(a) Zero (b) 1  [1993]
(c) 4 (d) 7 [1995]
Solution: (b) and (c)
Solution: (a)
XNOR
y = A + AB + ABC
A⊕B (b)
= = AB + AB
For minimum NAND gates reduce it AB + AB
y = A + AB(1 + C ) (1 + C = 1) AB + AB
= A + AB = ( A + B )( A + B ) (c)

y = A(1 + B) = A Hence, the correct option is (b) and (c).


Y=A 2 6. For the logic circuit shown in the figure, the output is
equal to
⇒ number of NAND is required is zero.
Hence, the correct option is (a). y = AB + BC + A + C
23. The output of a logic gate is ‘1’ when all its inputs are = A+ B + B +C + A+C ( A + A = A)
at logic ‘0’. The gate is either
(a) a NAND or an EX-OR gate y = A+ B +C
(b) a NOR or an EX-NOR gate
Hence, the correct option is (b).
(c) an OR or an EX-NOR gate
(d) an AND or an EX-OR gate [1994] 2 7. Indicate which of the following logic gates can be used
to realized all possible combinational Logic functions:
Solution: (b)
(a) OR gates only
Output y = 1 when all inputs = 0 (b) NAND gates only
Then (c) EX-OR gates only
a) y = EXNOR (for all input same, output = 1) (d) NOR gates only [1989]
b) y = NAND (NAND =1, when all input = 0) Solution: (b) and (d)
c) y = NOR (NOR = 1, input =0)
For realizing all logic function: UNIVERSAL GATES
(NOR and EXNOR) used
Hence, the correct option is (b). ⇒ NAND and NOR.
24. A ring oscillator consisting of 5 inverters is running at a Hence, the correct option is (b) and (d).
frequency of 1.0 MHz. The propagation delay per gate
28. For the circuit shown below the output F is given by
is_____n sec. [1994]
Solution:
For ring oscillator of 5 inverters (N = 5) mini frequency
of operation is
1
f = TPD = propagam delay of 1 inverter
2 Ntpd
(a) F = 1 (b) F = 0
1
⇒ TPD = (c) F = X (d) F = •X [1988]
2 Nf
 N =5
1
⇒ TPD =
2 × 50 × 106
1
= 7 = 10 −7 = 100 × 10 −9 sec
10
TPD = 100 n sec
6.20 | Digital Circuits

Solution: (b) E1
D1

D2
E2
D3
E3
V0

1 kΩ

10 V

(A) 3-input OR gate (B) 3-input NOR gate


Then x ⊕ 0 = x.0 + 0.x (C) 3-input AND gate (D) 3-input XOR gate
x ⊕ 0 = x.0 + 0.x
Solution:  If any of the input E1, E2, E3, is zero. The
= 00 +
= + xx ⋅⋅ 11
corresponding diode gets forward biased, and outputs
⊕ 00 =
xx ⊕ = xx V0 = Voltage Across Forward biased diode = 0.
⇒F
⇒ F= = xx ⊕
⊕ xx So Any input = 0, output = 0.
F= = 00 If No input = 0, i.e., All input are 1, All diodes will be
F reverse biased, V0 = Logic 1,
Hence, the correct option is (b). So it is a 3 input AND gate.
2 9. Minimum number of 2-input NAND gates required to Hence, the correct option is (C).
implement the function, F = ( X + Y )( Z + W ) is 2. In the figure shown, the output Y is required to be Y =
(a) 3 (b) 4 AB + C D . The gates G1 and G2 must be, respectively.
(c) 5 (d) 6 [1988]
 [2015]
Solution: (b)
A
F = ( x + y )( z + w )
G1
For NAND gate implementation ( AB) B
G2 Y
F = ( x ⋅ y )( z + w ) C

= x yz + x yw D

(A) NOR, OR (B) OR, NAND


(C) NAND, OR (D) AND, NAND
Solution: 
A
G1
AB
B
G2 Y = AB + C D
C C + D = CD
D
− −
NOR gate output is C + D = C D
⇒ NAND gates This term present in output of G2 so we can assume G2
Hence, the correct option is (b). is OR gate, and other input of G2 is AB.
For G1 gate, inputs are A, B, and output is AB, so G1 is
Two-marks Questions NOR gate A + B = AB
1. In the circuit shown, diodes D1, D2 and D3 are ideal, Hence, the correct option is (A).
and the inputs E1, E2 and E3 are “0 V” for logic ‘0’ and
“10 V” for logic ‘1’. What logic gate does the circuit 3. The output F in the digital logic circuit shown in the
represent? [2015] figure is
Chapter 3  Logic Gates  |  6.21

Z = P ⊕ Q and M1 = Z ⊕ R = P ⊕ Q ⊕ R.
Hence, the correct option is (d).
5. The point P in the following figure is stuck-at-1. The
output f will be

(a) F = XYZ + XYZ


F = XYZ + XYZ (b) (a) ABC (b) A
(c) F = XYZ + XYZ (d)
F = XYZ + XYZ (c) [2006]
ABC (d) A
 [2014]
Solution: (d)
Solution: (a)

F = ( A ⊕ B) ⋅ ( A ⊕ B ⊕ Z ) If P is stuck at 1 ⇒ output of NAND (5) = 0


⇒ For NAND (7), output = 1
= ( A ⊕ B) (( A ⊕ B) ⋅ Z + A ⊕ B ⋅ Z )
⇒ f = ( A.1) ⋅ A = AA = A
= A ⊕ B ⋅ Z + ( A ⊕ B)( A ⊕ B) ⋅ Z
=0 f =A
.
= ( A ⊕ B) Z Hence, the correct option is (d).
F = ( AB + AB ) Z 6. The number of product terms in the minimized sum-
F = ( ABZ + ABZ ) of-product expression obtained through the following
K-map is (where ‘d’ denotes don’t care states)
F = XYZ + X Y Z .
Hence, the correct option is (a).
4. Which of the following Boolean expressions correctly
represents the relation between P, Q, R and M1?

(a) 2 (b) 3
(c) 4 (d) 5 [2005]
Solution: (a)
(a) M1 = (P OR Q) XOR R ∴For SOP expression:
(b) M1 = (P AND Q) XOR R ⇒1 quad + 1 pair is taken as two product terms.
(c) M1 = (P NOR Q) XOR R
(d) M1=(P XOR Q) XOR R [2008]
Solution: (d)

X = P ⋅Q
Y = P+Q Hence, the correct option is (a).
7. A Boolean function f of two variables x and y is
Z = X ⋅ Y = PQ( P + Q )
defined as follows: f(0, 0) = f(0, 1) = f(1, 1) = 1;
= ( P + Q )( P + Q ) f (1, 0) = 0. Assuming complements of x and y are not
available, a minimum cost solution for realizing f using
= PQ + PQ
6.22 | Digital Circuits

only 2-input NOR gates and 2-input OR gates (each Solution: (b)
having unit cost) would have a total cost of
(a) 1 unit (b) 4 unit
(c) 3 unit (d) 2 unit [2004]
Solution: (d)

f (0,0) = f (0,1) = f (1,1) = 1 ; f (1,0) = 0

X Y F
0 0 1
(a) Before to (t < to)
0 1 1
Vi = 0, G1 = 1
1 1 1 G2 = 01(1⊕ 0)
1 0 0 ( b) t > to, Vi = 1
Now, output of G1 becomes 1 + 1 = 0 (after 10 ns)
f = x( y + y ) + xy
t +t 
= x + xy ∴Till t0 −  0 1  V0 remains ( G2 = 0 ⊕ 1 = 1 ) after
 t2 
f = x+ y t0+ t1, G1 = 0 (after TPD = 10 ns)
As X is not available ⇒ G2 = 0 ⊕ 1 =1, but after TPD = 20ns
(t0 + t1 ) + 20 ns
⇒ After , V0 = 1
t3
V0 remain 0 from (t0 + t1) – (t0 + t3)
2 gates ⇒ 2 unit. (t2 – t3)→V0 = 0
Hence, the correct option is (d). ⇒ V0 t < t0 = 1
8. The gates G1 and G2 in the figure have propagation t0 < t < t 2 = 1
delays of 10 nsec and 20 nsec, respectively. If the input t 2 < t3 = 0
Vi makes an abrupt change from logic 0 to 1 at time t =
t3 < t = 1
t0, then the output waveform V0 is


(a)
Hence, the correct option is (b).
9. In the figure, the LED

(b)

(c)

(d) (a) emits light when both S1 and S2 are closed.


(b) emits light when both S1 and S2 are open.
(c) emits light when only of S1 and S2 is closed
(d) does not emit light, irrespective of the switch posi-
[2002] tions. [2001]
Chapter 3  Logic Gates  |  6.23

Solution: (d)
Five-marks Questions
1. The truth table for the output Y in terms of three inputs
A, B and C are given in Table. Draw a logic circuit reali-
zation using only NOR gates.
A 0 1 0 1 1 1 0 1
B 0 0 1 1 0 0 1 1
C 0 0 0 0 1 1 1 1
1 1 1 0 1 0 0 0
Y
LED glows when n side is low (P → +5)
⇒ output of NAND = 0  [1993]
⇒ Output AND and XOR = 1 Solution: y = πm (3, 5, 6, 7) = y (CBA) = y(ABC)
⇒ Input of AND = 1, 1
Input of XOR = 0, 1 or 1, 0
For AND gate: S1 and S2 should be open (input = high
(+5 Vcc)) but when S1 and S2 are open, then input to
X-OR = 121 ⇒ output = 0
∴ If S1S2 both open → NAND ≠ 0 
 LED does not glow
S1S2 both close → NAND ≠ 0 

y = ( A + B )( B + C )(C + A)
∴ If S1S2 both open → NAND ≠ 0 
 LED does not glow
S1S2 both close → NAND ≠ 0 

Hence, the correct option is (d).
10. For the logic circuit shown in the figure, the simplified
Boolean expression for the output Y is

2. The operating condition (ON = 1, OFF = 0) of three


(a) A + B + C (b)
A pumps (x, y, z) are to be monitored. x = 1 implies that
(c) B (d) C [2000] pump X is on. It is required that the indicator (LED) on
Solution: the panel should glow when a majority of the pumps
fail.

y = (( AB B B ) + AB B + C )
Now AB. B = ( A + B) B = AB + B = ( A +1)( B )

=B=B A +1 = 1
Figure (a) Figure (b)
⇒ y = ( B ⋅ B) + B + C = O + B + C = 87 (' B ⋅ B ) = 0
(a) Enter the logical values in the K-map in the format
y = 1 + BC = 1 = 0 shown in figure (a). Derive the minimal Boolean
y= 0 sum-of-products expression whose output is zero

when a majority of the pumps fail.
6.24 | Digital Circuits

(b) The above expression is implemented using logic = xy + yz + xz


gates and point P is the output of this circuit, as when majority of pumps fails P is at ‘0’ therefore cur-
shown in figure (b). P is at 0 V when a majority of rent ‘I’ flows through the ckt.
the pumps fail and is at 5 V otherwise. Design a cir-
cuit to drive the LED using this output. The current I = 10 mA  given

through the LED should be 10 mA and the using VD = 1V 
this output. The current through the LED should be
10 mA and the voltage drop across it is 1 V. Assume 5 −1 4
R= = = 400Ω
that P can source or sink 10 mA and a 5 V supply is 10 m 10 × 10 −3
available. [2000]
Solution: 3. For digital block shown in Figure (a), the output Y =
f(S3, S2, S1 S0) where S3 is MSB and S0 is LSB. Y is given
in terms of minterms as
X Y Z P
0 0 0 0 0
Y = Σm(1, 5, 6, 7, 11, 12, 13, 15) and its complements
Y = lm(0, 2, 3, 4, 8, 9, 10, 14).
1 0 0 1 0
2 0 1 0 0
3 0 1 1 1
4 1 0 0 0
5 1 0 1 1
6 1 1 0 1
7 1 1 1 1
P (x, y, z) = ∑m (3, 5, 6, 7)
(a)
Figure (a) Figure (b)

(a) Enter the logical values in the given Karnaugh map


[figure (b)] for the output Y.
(b) Write down the expression for Y in sum-of-prod-
ucts from using minimum number of terms.
(c) Draw the circuit for the digital logic boxes using
four 2-input NAND gates only for the each of the
boxes. [2001]
Solution:
(a)

P(x, y, z) = xy + yz + zx
(b)
Chapter 3  Logic Gates  |  6.25

(b)
y = S3 S1S0 + S3 S2 S1 + S3 S2 S1 + S3 S1S0
In solution quad has not been included because it is
Reduntant term.
(c). y = S3 S2 S1 + S3 S2 S1 + S3 S1S0 + S3 S1S0

= S2 ( S3 S1 + S3 S1 ) + S0 ( S3 S1 + S3 S1 )

= S2 ( S3 ⊕ S1 ) + S0 ( S3 ⊕ S1 ) = S2 S + S0 SA

s
Chapter 4
Combinational Circuits
Y = ABC ⊕ AB ⊕ BC
One-mark Questions
=
⎣ ( )
ABC ⊕ ⎡ AB BC + AB ( BC )⎤
⎦
1. Consider the circuit shown in the figure[2017]
ABC ⊕ ⎡⎣ ABC + ABC ⎤⎦
=
Y 0

ABC ⊕ B ( A ⊕ C )
=

MUX 0

0 1 MUX F
(
= ABC ⎡⎣ B + AC + A C ⎤⎦ + ABC ABC + A. BC

)
= ABC + ABC + ABC 
1
= BC + AB = B (C + A)
X 
Hence, the correct option is (C).
Z
3. A 4 : 1 multiplexer is to be used for generating the out-
The Boolean expression F implemented by the circuit put carry of a full adder. A and B are the bits to be
is added while Cin is the input carry and Cout is the output
X Y Z + XY + Y Z (B)
XY Z + XZ + Y Z carry. A and B are to be used as the select bits with A
(A)
being the more significant select bit.  [2016]
(C) XY Z + XY + Y Z (D)
X Y Z + XZ + Y Z
I0
2. The output of the combinational circuit given below is I1 4:1 Cout
[2016] I2 Mux
I3 S1 S0
A

A B

B C Y Which one of the following statements correctly


describes the choice of signals to be connected to the
inputs I0, I1, I2, and I3 so that the output is Cout?
(A) A + B + C (B) A(B + C) (A) I0 = 0, I1 = Cin, I2 = Cin and I3 = 1
(C) B(C + A) (D) C(A + B) (B) I0 = 1, I1 = Cin, I2 = Cin and I3 = 1
(C) I0 = Cin, I1 = 0, I2 = 1 and I3 = Cin
Solution:  (D) I0 = 0, I1 = Cin, I2 = 1 and I3 = Cin
A AB Solution:  In a 4:1 multiplexer, the no of select lines
ABC
will be 2 which are A and B, observing the carry output
C Y will make it easier to find the min terms.
B
Carry output of full adder will have min terms
AB ⊕ BC
BC ∑ m (3, 5, 6, 7)

To implement it, we can use tabular method:


Chapter 4  Combinational Circuits  |  6.27

AB I0 I1 I2 I3 So, Q, S, R are inputs S2 S1 S0 of Demultiplexer


Cin 00 01 10 11 P is Din,
0 0 2 4 (6) P, Q, R, S = Din, S2, S0, S1
1 1 (3) (5) (7) Hence, the correct option is (D).
0 Cin Cin 1 5. In a half-subtractor circuit with X and Y as inputs, the
borrow (M) and difference (N = X − Y) are given by
I0 = 0 (a) M = X ⊕ Y, N = XY
I1 = I2 = Cin (b) M = XY, N = X ⊕ Y
(c) M = XY , N = X ⊕ Y
I3 = 1
(d) M = XY , N = X ⊕ Y  [2014]
Hence, the correct option is (A).
Solution: (c)
4. A1-to-8 demultiplexer with data input Din, address
HALF SUBT.:
inputs S0, S1, S2 (with S0 as the LSB) and Y 0 toY 7 as
N = X – Y
the eight demultiplexed outputs, is to be designed using
two 2-to-4 decoders (with enable input E and address X Y N M
inputs A0 and A1) as shown in the figure. Din, S0, S1 and 0 0 0 0
S2 are to be connected to P, Q, R and S, but not neces- 0 1 1 1
sarily in this order. The respective input connections to
1 0 1 0
P, Q, R and S terminals should be [2015]
1 1 0 0
P
1E 1Y0 Y0 N = X ⊕ Y 
Q 2-to-4  
1Y Y1 M = X. Y 
R 1A0
Decoder 1  .
1Y2 Y2
Hence, the correct option is (c).
S 1A1 1Y3 Y3
6. Consider the multiplexer based logic circuit shown in
the figure.
2E 2Y0 Y4
2-to-4
2Y Y5
Decoder 1
2A0
2Y2 Y6
2A1 2Y3 Y7

(A) S2, Din, S0, S1 (B) S1, Din, S0, S2


(C) Din, S0, S1, S2 (D) Din, S2, S0, S1
Solution:  Which one of the following Boolean functions is real-
P ized by the circuit?
1E 1Y0 Y0 (a) F = WS1S2
Q
1Y1 Y1 (b) F = WS1 + WS2 + S1 S2
R 1A0
1Y2 Y2 (c) F =W + S1 + S2
S 1A1 1Y3 Y3 (d) F = W ⊕ S1 ⊕ S2 [2014]
Solution: (d)

2E 2Y0 Y4
2Y1 Y5
2A0
2Y2 Y6
2A1 2Y3 Y7

The OR gate output is zero, when inputs are zero. Then


decoders will be enabled.
Output × of MUX 1: X = S1W + S1W = S1 ⊕ W
P = Q = 0, 1st Decoder will work with inputs S, R (A1 A0)
P = 0, Q = 1, 2nd Decoder will work with input S, R (A1 F = S2 X + S2 X = S2 ⊕ X = S1 ⊕ W ⊕S2 .
A0) Hence, the correct option is (d).
6.28 | Combinational Circuits

7. The output Y of a 2-bit comparator is logic 1 whenever 9. The Boolean function f implemented in the figure using
the 2-bit input A is greater than the 2-bit input B. The two input multiplexers is
number of combination for which the output is logic 1
is
(a) 4 (b) 6
(c) 8 (d) 10 [2012]
Solution: (b)

(a) ABC + ABC (b) ABC + ABC


(c) ABC + ABC (d) ABC + ABC
Y = 1 when (A > B)  [2005]
If B = 00. Then A can be: 01, 10, 11 Solution: (a)
B = 01, A = 10, 11
B = 10 A = 11
B = 11, then A cannot be >B
⇒ A = (01, 10, 11), (10, 11), (11) ⇒ 6 Combination.
B = (00) (010) (10)
Hence, the correct option is (b).
8. The logic function implemented by the circuit below is
(ground implies a logic `0’) ⇒ From MUX 1: E = C B + CB
f = E ⋅0 + E ⋅ A
= 0 + (C B + CB ) A

f = ABC + ABC .
Hence, the correct option is (a).

10. Without any additional circuitry, an 8:1 MUX can be


used to obtain
(a) F = AND (P, Q) (a) some but not all Boolean functions of 3 variables
(b) all functions of 3 variables but none of 4 variables
(b) F = OR (P, Q)
(c) all functions of 3 variables and some but not all of
(c) F = XNOR (P, Q) 4 variables
(d) F = XOR (P, Q) [2011] (d) all functions of 4 variables [2003]
Solution: (d) Solution: (c)
8:1 MUX
(3)
⇒ 3 select lines (2 )
⇒ All function of 3 & some 4 variable function.
Hence, the correct option is (c).

11. A 2-bit binary multiplier can be implemented using


(a) 2 input ANDs only
(b) 2 input X-ORs and 4-input AND gates only
F = PQI 3 + PQI 2 + PQI1 + PQI 0 (c) Two (2) input NORs and one XNOR gate
(d) XOR gates and shift registers. [1997]
= PQ ⋅ 0 + PQ ⋅1 + PQ ⋅1 + PQ ⋅ 0
Solution: (a)
F = PQ + PQ = P ⊕ Q 2-bit binary multiplier can be implemented using 2
F = XOR( P , Q ). input AND gote only.

Hence, the correct option is. (a)
Hence, the correct option is (d).
Chapter 4  Combinational Circuits  |  6.29

12. The output of the circuit shown in figure is equal to F = C ( AB + AB) + C ( AB + AB )


= C A( B + B) + CA( B + B)
= C A + CA
F = C ⊕ A.
So, the answer is (b).
Hence, the correct option is (b).
14. The minimal function that can detect a ‘divisible by 3’
(a) 0 (b) 1
8421 BCD code digit (representation is D8 D4 D2 D1) is
(c) AB + AB (d) ( A ⊕ B) ⊕ ( A ⊕ B) given by
 [1995 (a) D8 D1 + D4 D2 + D8D2D1
Solution: (b) (b) D8D1 + D4D2 D1 + D4 D2 D1 + D8 D4 D2 D1
(c) D8D1 + D4D2 + D8 D4 D2 D1
(d) D4 D2 D1 + D4D2D1 + D8D4D2D1 [1990]
Solution: (b)
To detect a number divisible by three:
D8 D4 D2 D1 Y
0 0 0 0 0 1
⇒ F = X ⊕X 1 0 0 0 1 0
F =1 2 0 0 1 0 0

3 0 0 1 1 1
If A ⊕ B = X ( AB + AB)
4 0 1 0 0 0
Then A ⊕ B = X , also 5 0 1 0 1 0
= ( AB + AB ) (X-NOR of same number = 1) 6 0 1 1 0 1
Hence, the correct option is (b). 7 0 1 1 1 0
13. The logic realized by the circuit shown in figure is 8 1 0 0 0 0
9 1 0 0 1 1
0 is divisible

Y = D 8 D 4 D 2 D1 + D 8 D 4 D2 D1 + D 8 D4 D2 D1 + D8 D 4 D1 .
Y = D 8 D 4 D 2 D1 + D 8 D 4 D 2 D 1 + D 8 D4 D2 D1 + D8 D 4 D1 .
(BCD – till a)
Hence, the correct option is (b).

(a) F = A ⊕ C (b) F = A ⊕ C Two-marks Questions


(c) F = B ⊕ C (d) F = B ⊕ C [1992] 1. A four-variable Boolean function is realized using 4 ×
Solution: (b) 1 multiplexers as shown in the figure [2018]
The minimized expression for F (U, V, V, X) is

I0 I0 F(U, V, W, X)

I1 4 × 1 I1 4 × 1
VCC
MUX MUX
I2 I2

I3 S1 S0 I3 S1 S0

U V W X
For 4 × 1 MUX,
6.30 | Combinational Circuits

(A) (UV + UV )W 3. The functionality implemented by the circuit below is


 [2016]
UV + UV )(WX + WX )
(B) P
Q
(C) (UV + UV )W R
S
UV + UV )(WX + WX )
(D) Y
O0
Solution: 
Consider the figure given below
C1 O1
I0 I0 2:4
C0 Decoder O2
I1 4 × 1 I1 4 × 1
VCC F(U, V, W, X) O3
MUX MUX
I2 I2
Enable = 1
I3 S1 S0 I3 S1 S0
is a tristate buffer
U V W X
(A) 2 to 1 multiplexer
From the above figure, the minimized expression for F (B) 4 to 1 multiplexer
(U, V, V, X) will be (C) 7 to 1 multiplexer
(D) 6 to 1 multiplexer

(
F = UV + UV W )( ) Solution: 

Hence, the correct option is (C)


2. Figure I shows a 4-bit ripple carry adder realized using Y = A if EN = 1,
full adders and Figure II shows the circuit of a full-
Y = High impedance if EN = 0,
adder (FA). The propagation delay of the XOR, AND EN
and OR gate in Figure II are 20 ns, 15 ns and 10 ns,
C1C0 of decoder will activate (=1), one of the output
respectively. Assume all the inputs to the 4-bit adder
O0 to O3. So that corresponding tristate buffer will
are initially reset to 0.
give output same as input, other tristate buffers will be
At t = 0, the inputs to the 4-bit adder are changed to disabled.
X3X2X1X0 = 1100, Y3Y2Y1Y0= 0100 and Z0 =1. The out-
put of the ripple carry adder will be stable at t (in ns) = C1 C0 O0 O1 O2 O3 Y
____________. [2017] 0 0 1 0 0 0 P
0 1 0 1 0 0 Q
Y3 X3 Y2 X2 Y1 X1 Y0 X0
1 0 0 0 1 0 R
Z3 Z2 Z1 1 1 0 0 0 1 S
Z4 FA FA FA FA Z0
The two inputs of decoder are handling 4 outputs
S3 S2 S1 S0 simultaneously disabling the unused buffers therefore;
it will behave as multiplexer.
Xn Hence, the correct option is (B).
Yn Sn 4. For the circuit shown in the figure, the delays of NOR
gates, multiplexers and inverters are 2 ns, 1.5 ns and
1 ns respectively. If all the inputs P, Q, R, S and T are
Zn+1 applied at the same time instant, the maximum prop-
agation delay (in ns) of the circuit is __________.
 [2016]
Zn
Chapter 4  Combinational Circuits  |  6.31

Solution:  NAND output = 0, when Q2, Q0 = 1, 1


P So Q2 Q Q0 = 101 or 111, in UP counter 101 state occurs
first, so counter counts up 101, at the same moment it
Q go to 000 state, so total states we can see are from 000
0 0
to 100 only. Modulo-5-up counter
R MUX MUX Y Hence, the correct option is (A).
S 1 1
S0 S0 6. In the circuit shown, W and Y are MSBs of the control
inputs. The output MSBs is given by
T

Since, the multiplexer has two input lines therefore, the


number of select lines is 1, i.e., T.
Hence, the MUX works in 2 cases;
(i) when T = 0
 (delay is caused by two multiplexers and one
inverter)
delay = tNOR + tMUX + tMUX = 2 + 1.5 + 1.5 = 5 ns (a) F = WX + WX + YZ
(ii) when T = 1 (b) F = WX + WX + YZ
(delay is caused by both the multiplexers, inverter
T and a NOR gate) (c) F = WXY + WXY
delay = tinv + tMUX + tNOR + tMUX = 1 + 1.5 + 2 (d) F = (W + X )YZ  [2014]
+ 1.5 = 6 ns
So the maximum delay = 6 ns. Solution: (c)
Hence, the correct Answer is (6 ns).
5. The circuit shown consists of J-K flip-flops, each with
an active low asynchronous reset Rd input . The ( )
counter corresponding to this circuit is [2015]
(A) a modulo-5 binary up counter
(B) a modulo-6 binary down counter
(C) a modulo-5 binary down counter
(D) a modulo-6 binary up counter
Q0 Q1 Q2 VCC = +1
1 J Q 1 J Q 1 J Q
Clock
gmd = 0
1 K Rd 1 K Rd 1 K Rd W1Y → MSB of S.L.

⇒ Q = WX ⋅ 0 + W X ⋅1 + WX ⋅1 + W ⋅ X ⋅ 0
Q = W X + WX
Solution:  All the flip-flops are in toggle mode Qn+1 =
Qn Q =W ⊕ X
F = (Y Z + YZ )(Q ) + (YZ + YZ ).0
J = 1, K = 1, Qn+1 = Q n ,

so for every clk pulse flip-flop will change its state. = Y ⋅ Q = Y (W X + W X )


Clk input is connected Q0 (LSB). F = W X Y +W X Y .
Sequence will be Q2 Q1 Q0
Hence, the correct option is (c).
Q is connected to falling edge so it’s n up counter
NAND gate is connected to Rd input (reset) 7. If X and Y are inputs and the difference (D = X − Y) and
So when NAND output is zero all flip-flops will go the borrow (B) are the outputs, which one of the fol-
back to reset state. lowing diagrams implements a half-subtractor?
6.32 | Combinational Circuits

(a) Solution: (a)


For 2 bit subtraction: Diff
D = X ⊕Y = ( X −Y )

Borrow B = XY
Checking all MUX:
(a) Y

X
Y

(b) (b)
D = Y X +Y X 
B = YD + Y X ≠ Borrow 
(c)
B = X Y + Y X = X ⊕ Y ≠ Borrow 
(d) B = Y X + XY ≠ Borrow 
Hence, the correct option is (a).
8. An 8-to-1 multiplexer is used to implement a logical
function Y as shown in the figure. The output

(c)

(a) Y = ABC + ACD


(b) Y = ABC + ABD
(c) Y = ABC + ACD

(d) (d) Y = ABD + ABC


[2014]
Solution: (c)

[2014]
Chapter 4  Combinational Circuits  |  6.33

(c) F = ∑m(1, 2, 4, 5, 11, 14, 15)


= ABC ⋅ 0 + ABC ⋅1 + ABC ⋅ D + 0 + ABC ⋅ D +
(d) F = ∑m(2, 3, 5, 7, 8, 9, 12) [2010]
ABC ⋅ D + ABC ⋅ D + 0
Solution: (d)
= ABC + ABCD + ABC ⋅ 0 + ABCD
= BC ( A + X ) + ACD( B + B )
=1

Y = ABC + ACD.
Hence, the correct option is (c).
9. A 16-bit ripple carry adder is realized using 16 identical
full adders (FA) as shown in the figure. The carry-prop-
agation delay of each FA is 12 ns and the sum-propaga-
tion delay of each FA is 15 ns. The worst case delay (in
ns) of this 16-bit adder will be_________. f = ABI 3 + ABI 2 + ABI1 + ABI 0

= AB(C D ) + ABC + ABD + ABC


Now f should be converted in canonical form
f = ABC D + ABCD + ABC D + ABCD + ABCD + ABCD + ABC D
(1100) (1001) (1000) (0111) (0101) (0011) (0010)

f = ABC D + ABCD + ABC D + ABCD + ABCD + ABCD + ABC D


[2014] (1100) (1001)
(1000) (0111) (0101) (0011) (0010)

Solution: ⇒ f = m12 + ma + m8 + m7 + m5 + m3 + m2
In ripple carry adder, carry out of 1 FA becomes carry
in of next FA. f = ∑ m(2,3,5, 7,8, 9,12) .
∴ For generation of G5, P.d. (carry) of all 16 FA added: Hence, the correct option is (d).
= 16 × 12 ns
Common Data for Question 11 and 12.
= 192 ns.
Two products are sold from a vending machine, which
For sum generation, output sum of a FA uses carry out
has two push buttons P1 and P2. When a button is
of previous
pressed, the price of the corresponding product is dis-
FA ⇒ 16 FA P.d. (sum) added played in a 7-segment display.
= 16 × 15 ns If no buttons are pressed, ‘0’ is displayed, signifying
= 240 ns ‘Rs. 0’
As final sum pd > carry pd ⇒ Worst case delay If only P1 is pressed, ‘2’ is displayed, signifying ‘Rs. 2’
= Pd sum If only P2 is pressed, ‘5’ is displayed, signifying ‘Rs. 5’
= 240 ns. If both P1 and P2 are pressed, ‘E’ is displayed, signify-
10. The Boolean function realized by the logic circuit ing ‘Error.’
shown is The names of the segments in the 7-segment display,
and the glow of the display for ‘0’,
‘2’, ‘5’ and ‘E’ are shown below.

Consider
(i) push button pressed/not pressed in equivalent to
(a) F = ∑m(0, 1, 3, 5, 9, 10, 14) logic 1/0, respectively.
(b) F = ∑m(2, 3, 5, 7, 8, 12, 13) (ii) a segment glowing/not glowing in the display is
equivalent to logic 1/0, respectively.
6.34 | Combinational Circuits

11. If segments a to g are considered as functions of P1 and Solution: (d)


P2, then which of the following is correct? Logic of driver for 7-seg display:
(a) g = P1 + P2, d = c + e (1) g = P1 + P2
(b) g = P1 + P2, d = c + e
(2) c = P1 P2 + P2 P1 = P1
(c) g = P1 + P2, e = b + c
(3) e = P1 P2 + P1 P2 + P1 P2 = P2 + P1 P2 = P2 + P1
(d) g = P1+ P2, e = b + c [2009]
Solution: (4) a = P1 P2 + P1 P2 + P1 P2 + P1 P2 = 1 = d

(5) b = P1 P 2 + P1 P2 = P2
(6) f = P1 P + P P + P P = P P
2 2 1 1 2 1 2
⇒ P1 & P2 req
12. What are the minimum numbers of NOT gates and (2 NOT gates)
2-mput OR gates required to design the logic of the g, e, f → 10R each (3 OR)
driver for this 7-segment display? 2 NOT & 3 OR.
(a) 3 NOT and 4 OR (b) 2 NOT and 4 OR Hence, the correct option is (d).
(c) 1 NOT and 3 OR (d) 2 NOT and 3 OR 14. For the circuit cuit shown in the following figure, I0−I3
 [2009] are inputs to the 4 : 1 multiplexer. R (MSB) and S are
Solution: (a) control bits.

For g, it gives, when P2 pressed (P1 not) for 5 = P2 P1 , ,


g = P1 P 2 + P1 P2 + P1 P2

= P1 P 2 + P2
= ( P1 + P2 )( P2 + P2 )
=1

when P1 pressed (not Q2) for 2 = P1 P2 when both


pressed (E) = P1P2
g = P1 + P2
Similarly for d: for 0 ( P1 P2 ) , for 2, 5, E
The output Z can be represented by
⇒ d = P1 P2 + P1 P2 + P1 P2 + P1 P2 (a) PQ + PQS + QRS
also, for C: for 0, 5
(b) PQ + PQR + PQS
⇒ c = P1 P2 + P2 P1 (c) PQR + PQR + PQRS + QRS
for e: 0, 2, E
(d) PQR + PQRS + PQRS + QRS  [2008]
ee =
=P P2 +
P1 P +P P2 +
P1 P +PP1 P
P2
1 2 1 2 1 2
⇒ c+e = P P +P P +PP +P P +P P Solution: (a)
⇒ c + e = P11 P22 + P11 P22 + P11 P22 + P11 P22 + P22 P11
+ cc =
ee + =P P2 + P1 P2 + P2 P1 + P2 P1 = d
P1 P
1 2 + P1 P2 + P2 P1 + P2 P1 = d

⇒ d = c + e, g = P1 + P2 .
Hence, the correct option is (a).
1 3. What are the minimum number of 2-to-1 multiplexers
required to generate a 2-input AND gate and a 2-input
Ex-OR gate?
(a) 1 and 2 (b) 1 and 3
(c) 1 and 1 (d) 2 and 2 [2009]
Chapter 4  Combinational Circuits  |  6.35

I2 S1 = A ⊕ B (from 1)
z = RSI 3 + RS PQ + RSI1 + RSI 0
= ABC + ABC + ABC + ABC
= RSP + RSPQ + RSP + RS ( P + Q )
Hence, the correct option is (a).
= RSP + RSPQ + RSP + P RS + QRS
16. An I/O peripheral device shown in the figure below is
Solving by k-map to be interfaced to an 8085 microprocessor. To select
Quad PQ the I/O device in the I/O address range D4 H-D7 H, its
Pair : RSQ, PQS chip-select (CS ) should be connected to the output of
the decoder shown in the figure
⇒ z = PQ + PQS + QRS (a)

PQ
RS

(a) output 7 (b) output 5


(c) output 2 (d) output 0 [2006]
Solution: (b)

Hence, the correct option is (a).


1 5. In the following circuit, X is given by

A 7 6 5 4 3 2 1 0
(O4) 4 = 1 1 0 1 0 1 0 0
(D7) 4 = 1 1 0 1 0 1 1 1
Input of decoder = 101 (for D4 to D7)
=5
(a) X = ABC + ABC + ABC + ABC Output
⇒ 5 to CS
(b) X = ABC + ABC + ABC + ABC (b)
(c) X = AB + BC + AC Hence, the correct option is (b).
(d) X = AB + BC + AC  [2007] 17. The circuit shown in the figure converts
Solution: (a)
Output of MUX (1):
S1 = AB I 3 + ABI 2 + ABI1 + AB I 0
=0 =0

S1 = ( AB + AB)
Output of MUX (2):
X = S1S0 I 3 + S1S0 I 2 + S1S0 I1 + S1S0 I 0
=0 =1 =1 =0 (a) BCD to binary code
= S1S0 + S1S0 (b) Binary to excess –3 code
⇒ X = S1 ⊕ S = A ⊕ B ⊕ C (c) Excess –3 to Gray code
(d) Gray to Binary code [2003]
S0 = C (given)
6.36 | Combinational Circuits

Solution: (d) (c) 4-bit subtractor giving Q − P


(d) 4-bit adder giving P + Q + R [2003]
Solution: (b)
y = P⊕Q⊕R z = RQ + PR + QP
For example, P =1101 Q = 1001
⇒ yn = Pn ⊕ Qn ⊕ Rn , z = Rn Qn + P n Rn + Qn P n t
& Rn +1 = zn
z4 = y5 ( MSB )


X4 = A4 and output y5 y4 y3 y2 y1 = 00010 ⇒
X 3 = A4 ⊕ A3 = X 4 ⊕ A3 4-bit subtractor
X 2 = A2 ⊕  X 3 ( A4 + A3 )  Q → input (4-bit)

R → Cin
X1 = A1 ⊕ X 2 ( A4 + A3 + A2 )
By drawing T. T. find outputs z → borrow out
For example: For A4A3A2A1 = 1010, output = 1100 Complete truth table can be checked for all 23 (4-bit
(GARY CODE) . inputs)
Hence, the correct option is (d). Hence, the correct option is (b).
18. The minimum number of 2-to-1 multiplexers required 20. If the input X3, X2, X1, X0 to the ROM in the figure are 8
to realize a 4-to-1 multiplexer is 4 2 1 BCD numbers, then the outputs Y3, Y2, Y1 Y0 are
(a) 1 (b) 2
(c) 3 (d) 4 [2004]
Solution: (c)
(2 × 1) MUX to realize (4 × 1) MUX
(4 × 1) MUX: 2 select lines, 4 input
(A, B) (I0 – I3)

(a) gray code numbers (b) 2 4 2 1 BCD numbers


(c) excess-3 code numbers (d) none of the above
Hence, the correct option is (c).  [2002]
1 9. The circuit shown in the figure has 4 boxes each Solution: (b)
described by inputs P, Q, R and outputs Y, Z with 4 2 1 (6, 7, (D4 – (D2, 3, (1, 3, 4,
8, 9) D 9) 8, 9) 7, 9)
Y = P ⊕ Q ⊕ R Z = RQ + PR + QP
X3 X2 X1 X0 Y3 Y2 Y1 Y0
D0 0 0 0 0 0 0 0 0
D1 0 0 0 1 0 0 0 1
D2 0 0 1 0 0 0 1 0
D3 0 0 1 1 0 0 1 1
D4 0 1 0 0 0 1 0 0
D5 0 1 0 1 0 1 0 1
D6 0 1 1 0 1 1 0 0
D7 0 1 1 1 1 1 0 1

The circuit acts as a D8 1 0 0 0 1 1 1 0


(a) 4-bit adder giving P + Q D9 1 0 0 1 1 1 1 1
(b) 4-bit subtractor giving P − Q
Chapter 4  Combinational Circuits  |  6.37

BCD – (0 to 9) Solution: (c)


⇒ 9 is given by y3y2y1y0 = 1111 Half subsector: (A – B)
1 A B D X
1 → 000 y →1
1 0 0 0 0 0
1 ⇒ D = AB + AB 
2 → 000 0 y1 → 2 1 0 1 0  (c)
2 X = AB 
0 1 1 1
4 → 0 − 00 y2 → 4
1 1 0 0
6 → 1 1 00 y2 → 4 Hence, the correct option is (c).
4
⇒ y3 → 2
⇒ y3y2y1y0 → 2421. Five-marks Questions
Hence, the correct option is (b).
1. A ROM is to be used to implement the Boolean func-
21. In the TTL circuit in the figure, S2 and S0 are select lines tions given below:
and X7 and X0 are input lines. S0 and X0 are LSBs. The
output Y is F1 (A, B, C, D) = ABCD + ABCD

F2 (A, B, C, D) = (A + B) + (A + B + C)
F3 (A, B, C, D) = ∑13,15 + ∑ 3, 5
φ

(a) What is the minimum size of the ROM required?
(b) Determine the data in each location of the ROM.
 [1995]
Solution: 4 inputs (ABCD) and 3 outputs
(a) indeterminate (F1 F2 F3)
(b) A ⊕ B So size of ROM = 24 × 3 = 48
(b) f1 ( A, B, C , D ) = ABCD + ABCD
(c)
A⊕ B
= ∑m(0, 15)
(d) C ( A ⊕ B ) + C(A ⊕ B) [2001]
f ( ABCD ) = ( A + B)( A + B + C )
Solution:(c) 2
For 8 × 1 MUX: *(For TTL, flowing input = 1) = ( A + B + C )( A + B + C )( A + B + C )

Output y ⇒ input to S2 = C + (1) = C + 1 = 1 = ( A + B + C + D )( A + B + C + D )

⇒ from 8 outputs, lower 4 are 0 ( S 2 = 0)
( A + B + C + D )( A + B + C + D )
⇒ y = S2 ( ABI 3 + B AI 2 + BAI1 + B AI 0 )
=1 0 0 =1 = ( A + B + C + D )( A + B + C + D )

= 1( AB + AB )
= πm(0, 1, 2, 3, 12, 13)

y = A⊕B
=∑m (4, 5, 6, 7, 8, 9, 10, 11, 14, 15)
F3(ABCD) = ∑m(13,15) + ∑(3,5)
Hence, the correct option is (c).
2 2. For a binary half-subtractor having two inputs A and B, A B C D F1 F2 F3
the correct set of logical expressions for the outputs D 0 0 0 0 0 1 0 0
(= A minus B) and 1 (= borrow) are 1 0 0 0 1 0 0 0
(a)
D = AB+ AB, X = AB 2 0 0 1 0 0 0 0
(b)
D = AB + AB + AB , X = AB 4 0 1 0 0 0 1 0

(c)
D = AB + AB , X = AB 5 0 1 0 1 0 1 1
6 0 1 1 0 0 1 0
(d)
D = AB + AB, X = AB  [1999]
7 0 1 1 1 0 1 0
8 1 0 0 0 0 1 0
6.38 | Combinational Circuits

9 1 0 0 1 0 1 0 8 1 0 1 1 1 0 0 0
10 1 0 1 0 0 1 0 9 1 1 0 0 1 0 0 1
11 1 0 1 1 0 1 0
W = ∑m(11, 12) + ∑d(5, 6, 7, 13, 14, 15)
12 1 1 0 0 0 0 0
X = ∑m(4, 8, 9, 10) + ∑d(5, 6, 7, 13, 14, 15)
13 1 1 0 1 0 0 1
Y = ∑m(2, 3, 9, 10) + ∑d(5, 6, 7, 13, 14, 15)
14 1 1 1 0 0 1 0
Z = ∑m(1, 3, 8, 10, 12) + ∑d(5, 6, 7, 13, 14, 15)
15 1 1 1 1 1 1 1
(a) W
2. A 'code converter' is to be designed to convert from
the BCD (5421) to the normal BCD (8421). The input
BCD combinations for each digit are given below. A
block diagram of the converter is shown in figure.

BCD (5421)
Decimal
A B C D
0 0 0 0 0
1 0 0 0 1
2 0 0 1 0
3 0 0 1 1
4 0 1 0 0
5 1 0 0 0
6 1 0 0 1
X
7 1 0 1 0
8 1 0 1 1
9 1 1 0 0

(a) Draw K-maps for outputs W, X, Y and Z.


(b) Obtain minimized expression for the output W, X,
Y and Z. [1995]
Y
Solution:
Decimal BCD(5421) BCD(8421)
A B C D W X Y Z
0 0 0 0 0 0 0 0 0
1 0 0 0 1 0 0 0 1
2 0 0 1 0 0 0 1 0
3 0 0 1 1 0 0 1 1
4 0 1 0 0 0 1 0 0
5 1 0 0 0 0 1 0 1
6 1 0 0 1 0 1 1 0
7 1 0 1 0 0 1 1 1
7 1 0 1 0 0 1 1 1
Chapter 4  Combinational Circuits  |  6.39

Z w

(b) W = AB + ACD
X = AB + ABC + ABD
or
AB + ABC + ACD
or
AB + ACD + ABD

Y = AC + ACD + CD

Z = AD + AD
Solution: (a) y
Decimal BCD (5421) BCD (8421)
A B C D W X Y Z
0 0 0 0 0 0 0 0 0
1 0 0 0 1 0 0 0 1
2 0 0 1 0 0 0 1 0
3 0 0 1 1 0 0 1 1
4 0 1 0 0 0 1 0 0
5 0 1 0 1 0 1 0 1
6 0 1 1 0 0 1 1 0
7 0 1 1 1 0 1 1 1
8 1 0 1 1 1 0 0 0
9 1 1 0 0 1 0 0 1 z

W = ∑m(11, 12) + ∑d(8, 9, 10, 13, 14, 15)


X = ∑m(4, 5, 6, 7) + ∑d(8, 9, 10, 13, 14, 15)
Y = ∑m(2, 3, 6, 7) + ∑d(8, 9, 10, 13, 14, 15)
Z = ∑m(1, 3, 5, 7, 12) + ∑d(8, 9, 10, 13, 14, 15)
6.40 | Combinational Circuits

W=A F3:

X = AB
Y = AC
Z = AB + BD + ACD + BCD
Hence, the correct option is (a).
3. It is desired to generate the following three Boolean F3 = ab + bc (3)
functions.
From the OR gate array
F1 = abc + abc + bc F1 = P1 + P2 (4)

F2 = P2 + P3 (5)
F2 = abc + ab ± abc
F3 = P4 + P5 (6)
F3 = abc + abc ± ac Comparing (1) and (4)

By using an OR gate array as shown in figure where P1 (2) and (5)
to P5 are the product terms in one or more of the vari- (3) and (6)
ables a, a, b, c and c. P1 = ab
Write down the terms Pv P2, P3, P4 and P5 P2 = ac

P3 = bc
Solution: First simplifying F1F2F3

P = bc, P5 = ac
or
P4 = ac , P5 = bc
4. In certain application, four inputs A, B, C, D (both true
and complement forms available are fed to logic cir-
cuit, producing an output F which operates a relay).
The relay turns on when F(ABCD) = 1 for the follow-
ing states of the inputs (ABCD): '0000', '0010', '0101',
'0110', '1101' and '1110'. States '1000' and '1001' do
not occur, and for the remaining states, the relay is off.
Minimize F with the help of a Karnaugh and realize
it using a minimum number of 3-input NAND gates.
 [1999]
F1 = ab + ac (1) Solution:
F1:

F2:

F = ABD + BCD + BCD


F1 = b ’c + ac (2) To realize with NAND gates only first draw AND-OR
ckt then replace all AND OR gates with NAND gates
Chapter 4  Combinational Circuits  |  6.41

line, D0 and D1 are the input data lines and Y is the


output line. The function table for the multiplexer
is in given table. [2002]
Solution:
(a) Solving k map for minterm(POS)

X = ABC + ABC + ABC


Solving k map for max term (POS)

X = ( A + B + C )( A + B + C )( A + B + C )( A + B + C )( A + B + C )
X = ( A + B + C )( A + B + C )( A + B + C )( A + B + C )( A + B + C )
5. The inputs to a digital circuit shown in the figure is
are the external signals A, B and C. ( A, B and C are not (b)  X = AC + AB = A( B + C )
available). The +5V power supply (logic 1) and the
ground (logic 0) are also available. The output of the y1 = C .1 + C . B = C + BC = (C + C )(C + B)
circuit is X = AB + ABC . y1 = B + C

y2 = Ay1 + A0 = Ay1

y2 = A( B + C ) = X

(a) Write down the output function in its canonical


SOP and POS forms.
(b) Implement the circuit using only two 2:1 multiplex-
ers shown in the figure where S is the data-select
Chapter 5
Sequential Circuits
TCIK
One-mark Questions CLK1
CLK1 D Q
D-Latch Output
1. In the circuit shown, the clock frequency, i.e., the fre- CLK2 CK
quency of the CLK signal, is 12 kHz. The frequency of CLK2
the signal at Q2 is _____ kHz [2019] TCIK/5

TCLK TCLK 3TCLK


TON =
Solution:  − =
2 5 10
D1 Q1 D2 Q2
3TCLK
Clk Q1 Clk Q2 TON 10 = 3
12 kHz = =
∴ Output Duty Cycle
TCLK TCLK 10
Solution:
3
∴ Output Duty Cycle in % = × 100% = 30%
10
D1 Q1 D2 Q2

TCIK
CLK1 D Q
Clk Q1 Clk Q2 CLK1 D-Latch Output
 12 kHz CLK2 CK
CLK2
D1 = Q1 ⋅ Q2 TCIK/5
D = Q1
2
Hence, the correct answer is (29.9 to 30.1).
Clk Present State Input Next State
3. Assume that all the digital gates in the circuit shown
Q1 Q2 D1 D2 Q1+ Q2+
in the figure are ideal, the resistor R = 10 kΩ and the
0 0 0 1 0 1 0 supply voltage is 5 V. The D flip flops D1, D2, D3, D4
1 1 0 0 1 0 1 and D5 are initialized with logic values 0, 1, 0, 1, and 0,
2 0 1 0 0 0 0 respectively. The clock has a 30% duty cycle. [2016]
3 0 0 – – – –
∴ There are only 3 states.
12 R = 10 kΩ
So, output frequency will be = 4 KHz D Q D Q D Q D Q D Q
3 D1 D2 D3 D4 D5
Hence, the correct answer is (4).
Clock
2. Consider the D-Latch shown in the figure, which is
transparent when its clock input CK is high and has 0
propagation delay. In the figure, the clock signal CLK1 The average power dissipated (in mW) in the resistor R
has a 50% duty cycle and CLK2 is 1/4 period delayed is ______ .
version of CLK1. The duty cycle at the output of the Solution:  All the flip flops are provided same positive
latch in percentage is _______________. [2017] edge clock and it will behave as a counter.
Chapter 5  Sequential Circuits  |  6.43

Clk Q1 Q2 Q3 Q4 Q5 Q3 + Q5 = Y
0 0 1 0 1 0 0 Set Q

1 0 0 1 0 1 1 5V
2 1 0 0 1 0 0
3 0 1 0 0 1 1 Q
Reset
4 1 0 1 0 0 1
(A) NOR gates to NAND gates
5 0 1 0 1 0 0
(B) Inverters to buffers
(C) NOR gates to NAND gates and inverters to buffers
(D) 5 V to ground
ClK
Solution: 
TTL implements NAND gates.
0 T 2T 3T 4T 5T
Set
Y Q

Average power can be calculated using

1 V (t )
T 2 Reset Q

T ∫0 R
PAvg= dt
Reset
Q

1 ⎛ 25 ⎞
T 2T 4T 5T
25
= ⎜ ∫ + ∫ 0 dt + ∫ + ∫ 0 dt ⎟ mW
5T ⎝ 0 10 T 10 4T ⎠
2T
Set Q
1 ⎛ 25 25 ⎞
= ⎜ + × 2⎟ = 1.5 mW For NOR gate Reset, set are connected as inputs to Q,
5 ⎝ 10 10 ⎠
Q NOR gates.
Hence, the correct Answer is (1.5 mW). In the given circuit set, Reset are connected to Q, Q
4. A three bit pseudo random number generator is shown. gates, so by replacing NOR gates with NAND gates,
Initially, the value of output Y = Y2 Y1 Y0 is set to 111. we can get the correct functionality of SR latch
The value of output Y after three clock cycles is[2015] Hence, the correct option is (A).
Y2 Y1 Y0 6. The figure shows a binary counter with synchronous
clear input. With the decoding logic shown, the counter
works as a [2015]
D2 Q2 D1 Q1 D0 Q0
Binary
counter Q3
CLK
Q2
CLK
Q1
(A) 000 (B) 001
Q0
(C) 010 (D) 100
CLR
Solution: 
D2 D1 D0
Clk Q2 Q1 Q0 Q1 ⊕ Q 0 Q2 Q1
0 1 1 1 0 1 1
(A) mod-2 counter (B) mod-4 counter
1 0 1 1 0 0 1 (C) mod-5 counter (D) mod-6 counter
2 0 0 1 1 0 0
Solution: 
3 1 0 0
Binary
Hence, the correct option is (D). counter
5. An SR latch is implemented using TTL gates as shown Q3
in the figure. The set and reset pulse inputs are provided Clk Q2
Q1
using the push-button switches. It is observed that the Q0
circuit fails to work as desired. The SR latch can be CLR
made functional by changing [2015]
6.44 | Digital Circuits

Given circuit is Binary Counter with Synchronous 8. The circuit shown in the figure is a
clear.

CLK Q3 Q2 Q1 Q0 CLK = Q3  Q2
0 0 0 0 0 1
1 0 0 0 1 1
2 0 0 1 0 1
3 0 0 1 1 1 (a) toggle Flip Flop
4 0 1 0 0 0 (b) JK Flip Flop
5 0 0 0 0 1 (c) SR Latch
(d) Master-Slave D Flip Flop [2014]
So no. of states = Modulus = 5 Solution: (d)
Hence, the correct option is (C).

7. Five JK flip-flops are cascaded to form the circuit


shown in the figure. Clock pulses at a frequency of 1
MHz are applied as shown. The frequency (in kHz) of
the waveform at Q3 is_____.
D1 – LATCH
D1 = input 0
clk → +ve
D2 – LATCH
D2 = Q1 (output of D1)
clk → −ve (of D1)
⇒ When D changes, Q1, changes on +ve clock cycle.
[2014] and as Q1 changes (D2), Q2 change, but on –ve clock
Solution: ⇒ input D goes to Q2 only once during a complete clk
cycle
⇒ This is MS-FF
Hence, the correct option is (d).
9. Consider the given circuit

Output of Q0 = clk of Q1

Q1 = clk of Q2 and so on

& clk → Q0

⇒ ASYN counters ⇒ 2n Asyn counter of clk freq = f

then freq of In this circuit, the race around
Q1 = f / 2 (a) does not occur
(b) occurs when CLK = 0
Q2 = f / 2 × 2 (f / 2 2)
(c) occurs when CLK = 1 and A = B = 1
f clk 1 MHz 1000 K (d) occurs when CLK = 1 and A = B = 0 [2012]
f3 = = =
8 8 8 Solution: (a)
f3 = 6.25 KHz

f
Q3 = ( f/23 )
2× 2× 2
Q4 = f / 24
Q5 = f / 25
Chapter 5  Sequential Circuits  |  6.45

RACE AROUND condition Solution: (c)


Because when output toggles between 0 and 1 at any Master-slave FF:
instance, we cannot decide the output (1 or 0) –  Change in input does not reflect output change
(a) From figure, we can see it is R-S FF immediately
So in RS FF: – Change in output depends on state change in slave
R S Qn+1
0 0 Qn
0 1 1 ⇒No race around condition Hence, the correct option is (c).
1 0 0 1 2. Choose the correct one from among the alternatives A,
B, C, D after matching an item from Group 1 with the
1 1 Not valid most appropriate item in Group 2.
Hence, the correct option is (a). Group-1
10. When the output Y in the circuit below is ‘1’, it implies P. Shift register
that data has
Q. Counter
R. Decoder
Group-2
1. Frequency division
2. Addressing in memory chips
3. Serial to parallel data conversion
(a) changed from ‘0’ to ‘1’ (a) P − 3, Q − 2, R − 1 (b) P − 3, Q − 1, R − 2
(b) changed from ‘1’ to ‘0’ (c) P − 2, Q − 1, R − 3 (d) P − 1, Q − 2, R − 2
(c) changed in either direction  [2004]
(d) not changed [2011]
Solution: (b)
Solution: (a)
P. Shift Reg → serial to || data conv. (3)
Q. Counter → freq division (1)
R. Decoder → Memory Chips (2)
⇒ P – 3, Q – 1, R – 2
y = 1 ⇒ Q1 . Q2 = 1 (Q1 = 1 = Q2) Hence, the correct option is (b).
As Qn+1 = Dn (for D FF) 13. A 0 to 6 counter consists of 3 flip flops and a combina-
⇒ Before clk pulse, both D1 and D2 =1 tion circuit of 2 input gate(s). The combination circuit
consists of
D1 = 1 D2 = 1
(a) one AND gate
As D2 = Q1 & D2 = 1 (before clk) (b) one OR gate
(c) one AND gate and one OR gate
(d) two AND gates [2003]
⇒ Q1 = 1 & Q1 = 0
Solution: (d)
As Q1 = 0, this means D1 was D (before going to 1)
0 to 6 counter ⇒ Resets at 7 (111)
⇒ Data has changed from 0 to 1.
Hence, the correct option is (a). ⇒ 31/P AND gate reg:
1 1. A master-slave flip-flop has the characteristic that but with 21/P gate:
(a) change in the input immediately reflected in the
output
(b) change in the output occurs when the state of the
master is affected
(c) change in the output occurs when the state of the
slave is affected ⇒ 2 AND req.
(d) both the master and the slave states are affected at Hence, the correct option is (d).
the same time [2004]
6.46 | Digital Circuits

14. Figure shows a mod-K counter. Here K is equal to Solution: (d)


J = Q, K = 1

(a) 1 (b) 2
(c) 3 (d) 4 [1998]
Solution: (c)

Initially cleared Q = 0


Ck J K Q Q
Initially cleared: Q0 Q1 = 00 0 – – 0 1
Q 0 Q1 = 11 1 1 1 1 0 (toggle)
Q 0 2 0 1 0 1
→ 10 →
3 1 1 1 0
Next clk: J0K0 = 11 ⇒ Q0 = 1
4 0 1 0 1
J1K1 = 01 ⇒ Q1 = 0, Q1 = 1 5 1 1 1 0
Next clk: J0K0 = 11 ⇒ Q0 = 0 (toggle) 6 0 1 0 1
Q0 → 10 → 01 ⇒ Q = 010101.
J1K1 = 11 ⇒ Q1 = 1 (toggle) Hence, the correct option is (d).
Next clk: J0K0 = 01 ⇒ Q0 = 0 (reset) 1 6. An R-S latch is
J1K1 = 01 ⇒ Q1 = 0 (reset) (a) combinatorial circuit
(b) synchronous sequential circuit
⇒ (c) one bit memory element
(d) one clock delay element [1995]
Mod – 3 counter.
Solution: (c)
Hence, the correct option is (c).
R–S latch
15. In a J-K flip-flop we have J == Q, and K = 1 (see figure). (a) Combinational Circuit – No (seq)
Assuming the flip-flop was initially cleared and then (b) Syn. Seq. Ckt – No (No clk, ∴ not sync)
clocked for 6 pulses, the sequence at the Q output will (c) 1 – bit mem. Element – Yes
be (d) 1 clk delay – No (No clk in latch)
(a) 010000 (b) 011001 Hence, the correct option is (c).
(c) 010010 (d) 010101
17. A switch-tail ring counter is made by using a single D
flip-flop. The resulting circuit is a
(a) SR flip-flop (b) JK flip-flop
(c) D flip-flop (d) T flip-flop [1995]
Solution: (d)
* Switch Tail → Ripple counter like

[1997]
Chapter 5  Sequential Circuits  |  6.47

Now for On,


D = Qn and Qn+1 = D = Qn

Toggle ⇒ TFF
Hence, the correct option is (d).
1 8. Synchronous counters are _______than the ripple
counters. [1994] Q0 n +1 = Q2 n ⊕ Q3n
Solution: Hence, the correct option is (c).
Synchronous counter: All FF work simultaneously 2 1. An S–R FLIP–FLOP can be converted into a T FLIP-
(have individual clk connection) FLOP by connecting _____to Q and ________ to Q
Ripple counter: output of 1 FF acts as clk to next FF  [1991]
(output ripples through FF connections) Solution:
∴Synch. Are FASTER than ripple. S–R FF to T FF
19. A pulse train with a frequency of 1 MHz is counted
using a modulo-1024 ripple counter built with J-K flip
flops. For proper operation of the counter, the maxi-
mum permissible propagation delay per flip flop stage
is _______n sec. [1993]

Solution:
T = SQ n + RQn
Modulo 1024 ripple counter
⇒ 210, i.e., 10 FF (N = 10)
As fclk = 1 mHz
1
and we know f clk < (N = number of FF)

NTp ⋅ d
1
or TPD < For T FF,
N ⋅ f clk
1 T = SQ n + RQn
⇒ TPDmax = = 10 −7 = 100 × 10 −9
10 × 1× 106 ⇒ R is conn. to Q
= 100 n sec.
S to Q .
2 0. The initial contents of the 4-bit serial-in-parallel-out,
22. A 4-bit modulo-16 ripple counter uses JK flip-flops. If
right-shift, Shift Register shown in the figure is 0110.
the propagation delay of each FF is 50 ns, the maxi-
After three clock pulses are applied, the contents of the
mum clock frequency that can be used is equal to:
Shift Register will be
(a) 20 MHz (b) 10 MHz
(c) 5 MHz (d) 4 MHz [1990]
Solution: (c)
For ripple counter, total delay
= Ntpd
= 4 × 50 ns
(a) 0000 (b) 0101
(c) 1010 (d) 1111 [1992] and max clock frequency:
Solution: (c) 1
f <
NTPD
1 1
f max = −9
=
4 × 50 × 10 200 × 10 −9
= 5 × 106
f = 5 MHz.

Hence, the correct option is (c).
6.48 | Digital Circuits

23. A ripple counter using negative edge-triggered D-flip 25. Choose the correct statements relating to the circuit of
flops is shown in figure below. The flip-flops are cleared figure
to ‘0’ at the R input. The feedback logic is to be designed
to obtain the count sequence shown in the same figure.
The correct feedback logic is:

(a) For Vi = −2v, P = 0


(b) For Vi = +3v, P = 0
(c) For Vi = 0v, P = 0 always
(d) For Vi = 0v, P can be either 0 or 1. [1987]
Solution: (b)

(a) F = Q2 Q1Q0 (b) F = Q2 Q1Q0


(c) F = Q2 Q1Q0 (d) F = Q2 Q1Q0
 [1987]
Solution: (a)
Ripple counter sequence: For comparator: output is +1, if V+ > V– 0, if V– > V+
⇒ In case a) Vi = 2V, ∴ for compl, V+ = V– ⇒ NO
output.
Vi = +3V ⇒ For Comp 1, V+ > V– ⇒ output = +1
This means, after 101, next state 110 should be
( Q 2Q1Q0 ) Comp 2, V– > V+ ⇒ output = 0
cleared (000)
∴ For P: P → due to 1 from comp 1,
(mod-6 counter)
P=0
⇒ Q0Q1Q2 in NAND gate (to R I/P)
Vi = 0 ⇒ C1: output = 0
C2: output = 0
But output P will depend on previous state output
24. The circuit given below is a Vi = 0 P → on previous state
Hence, the correct option is (b).

Two-marks Questions
1. A traffic signal cycles from Green to Yellow, Yellow to
(a) J–K flip-flop Red and Red and Red to Green. In each cycle, Green
(b) Johnson’s counter is turned on for 70 seconds, Yellow is turned on for 5
(c) R–S latch seconds and the Red is turned on for 75 seconds. This
(d) None of above [1988] traffic light has to be implemented using a finite state
Solution: (c) machine (FSM). The only input to this FSM is a clock
of 5 seconds period. The minimum number of flip-flops
If
required to implement this FSM is _______. [2018]
A B Qn+ 1 Solution:  We know that in order to turn Green for 70
0 0 Qn (previous o/p) seconds we need 2 flip flops
0 1 0 We know that in order to turn Red for 75 seconds we
need 2 flip flops
1 0 1
1 flipflop is required to turn yellow colour totally we
1 1 Not valid
need 5 such flipflops to have traffic signal cycles from
∴ From PS-NS table, it is RS LATCH. Green to Yellow, Yellow to Red and Red to Green
Hence, the correct option is (c). Hence, the correct answer is 5.
Chapter 5  Sequential Circuits  |  6.49

2. In the circuit shown below, a positive edge – triggered and clock periods are equal and the value of DT/Tck =
D Flip-flop is used for sampling input data Din using 0.15, where the parameters DT and TCK are shown in
clock CK. The XOR gate outputs 3.3 volts for logic the figure. Assume that the Flip and the XOR gate are
HIGH and 0 volts for logic LOW levels. The data bit ideal.

X
Din D Q TCK

D Flip-Flop CK
CLK Din

CK ΔT ΔT ΔT

If the probability of input data bit (Din) transition in Present State DA DB next State
each clock period is 0.3, the average value (in volts,
QA QB QA QB
accurate to two decimal places) of the voltage at note
X, is ______  [2018] 0 0 0 1 0 1
0 1 1 1 1 1
Solution: 
⎡ 1 1 0 1 0 1
⎡ ΔT ⎤ ⎤
Vx(avg) = ⎢0.3 × 3.3 ⎢1 − ⎥ ⎥ × (0.7 × 0) V 0 1 1 1 1 1
⎣ ⎣ TCK ⎦ ⎦
1 1 0 1 0 1
=
0.3 × 3.3 × [1 – 0.15]
\ If X in = 0 the given circuit will have two states,
01, 11.
=
3.3 × 3.3 × 0.85 = 0.8415 V
Hence, the correct option is (D).
Hence, the correct answer is 0.82 to 0.86. 4. A 4-bit shift register circuit configured for right-shift
3. A finite state machine (FSM) is implemented using the operation, i.e., Din → A, A → B, B → C , C → D, is
D flip-flops A and B, and logic gates, as shown in the shown. If the present state of the shift register is ABCD
figure below. The four possible states of the FSM are = 1101, the number of clock cycles required to reach
QA QB = 00, 01, 10 and 11. the state ABCD = 1111 is _____________. [2017]

QA
D Q D Q QB
A B
XIN
CK Q CK Q
Din A B C D
CLK
Clock

Assume that XIN is held at a constant logic level Din → A, A → B, B → C, C → D


Solution: 
throughout the operation of the FSM. When the FSM is A B C D
initialized to the state QA QB = 00 and clocked, after a 1 1 0 1
few clock cycles, it starts cycling through. [2017] 1 0 1 1 0
(A) all of the four possible states if XIN = 1 2 0 0 1 1
(B) three of the four possible states if XIN = 0 3 1 0 0 1
(C) only two of the four possible states if XIN = 1 4 0 1 0 0
(D) only two of the four possible states if XIN = 0
5 0 0 1 0
Solution:  From the given circuit diagram. 6 0 0 0 1
D A = Q A ⊕ QB 7 1 0 0 0
8 1 1 0 0
DB = Q. X in
9 1 1 1 0
Let us take Xin = 0 10 1 1 1 1
6.50 | Digital Circuits

Y=1
X = 0, Y = 0, Z = 0
Y=1
A B
Y = 0, Z = 0

X = 0, Z = 1 X = 1, Y = 1,
Z=1 Z=1
1 1 0 1
Din X = 1, Y = 0 Y = 0, Z = 1
Clock

\ 10 clock pulses are required to get state of ABCD = C


1111
Z=0
Hence, the correct answer is (10).
5. The state diagram of a finite state machine (FSM)
designed to detect an overlapping sequence of three Which one of the following statements is correct?
bits is shown in the figure. The FSM has an input ‘In’ (A) Transitions from state A are ambiguously ­defined.
and an output ‘Out’. The initial state of the FSM is S0. (B) Transitions from State B are ambiguously ­defined.
Xn (C) Transitions from State C are ambiguously d­ efined.
Yn Sn
(D) All of the state transitions are defined
­unambiguously.
Solution:  xyz = 001 condition is not defined so transi-
tions from state C are ambiguously defined.
Zn+1
Hence, the correct option is (C).
7. For the circuit shown in the figure, the delay of the
Zn
bubbled NAND gate is 2 ns and that of the counter is
assumed to be zero. [2016]
In = 0 If the clock (CLK) frequency is 1 GHz, then the coun-
00 Out = 0 ter behaves as a
S0
In = 1 (A) mod 5 counter (B) mod 6 counter
Out = 0 (C) mod 7 counter (D) mod 8 counter
In = 0 In = 1
01 Q0(LSB)
Out = 0 Out = 0 S0
S1 In = 0
3-bit Q1
Out = 0 Synchronous
In = 1
counter Q2(MSB)
10 Out = 0
S2 Clk
In = 0 In = 1
RESET
Out = 0 Out = 1

11
S3
Solution:  Here, NAND gate is used, which will behave
If the input sequence is 10101101001101, starting with to reset the counter. The counter will begin with all bits
the left-most bit, then the number of times ‘Out’ will be as 0. The clock used is positive edge triggered, thus the
1 is ___________. [2017] change in bit will occur during transition from 0 to 1.
The MSB Q0 will be complemented in every clock pulse
6. The state transition diagram for a finite state machine
and Q1 will change when a transition of 0 to 1 occurs in
with states A, B and C, and binary inputs X, Y and Z is
Q0. The LSB Q3 will change due to transition from 0 to
shown in the below figure. [2016]
1 occurring in Q2. Hence, the counter will start counter
Chapter 5  Sequential Circuits  |  6.51

until all the bits become 1. As soon as all bits are 1, the (a)
output of NAND will become 0 and the bits will be reset.
The counter will behave as mod 8 counter. So Option D.
Hence, the correct option is (D).
8. A mod-n counter using a synchronous binary
­up-counter with synchronous clear input is shown in
the figure. The value of n is _______. [2015] (b)

4-Bit binary QA QA
counter QB QB
CLOCK CLK
QC QC
CLEAR QD QD

(c)

Solution: 
4-Bit binary QA QA
up counter QB QB
Clk
QC QC
QD QD (d)
Clear

The NAND gate shown is connected to synchronous


clear, i.e., clear will be applied after clock pulse.
Solution: (d)
Clk QA QB QC QD Clear
0 0 0 0 0 1
1 0 0 0 1 1
2 0 0 1 0 1
3 0 0 1 1 1
4 0 1 0 0 1
5 0 1 0 1 1 Stall diagram
6 0 1 1 0 0
Q can be 0 or 1
7 0 0 0 0
(a) If A = 0, (i) Q = 0 ⇒ X1 = 0
As clear is synchronous input, clear will be zero in 6th X0 = 0 =1
clock pulse, but output is cleared in next clock pulse so ⇒ Y = 1 (= X0)
total no. of states = Modulus = 7. ⇒ D = y = 1 and Q = D = 1 (Q inserts)
Hence, the correct answer is (7). A = 0 (ii) Q = 1 ⇒ X1=1, X0 = 0
9. The state transition diagram for the logic circuit shown ⇒ D = Y = X0 = 0
in ⇒ Q = 0 (inserts)

(b) If A = 1
(i) Q = 0 ⇒ X1 = 0
Y0 = 1
6.52 | Digital Circuits

and Y = X1 = 0
⇒ D = Y = 0
⇒ Q = D = 0 (no change)
(ii) Q = 1 ⇒ X1= (1), X0 = 0
and Y = X1 = 1
⇒ D = Y = 1 & Q = D = 1 (no change)

When XOR → replaced by XNOR, then A = ?


From the diagram, 2b input D2 remains same
10. The digital logic shown in the figure satisfies the given ⇒ (Q2, D1, 2Q1)
state diagram when Q1 is connected to input A of the For X–OR: D2 = A ⊕ S = Q1 ⊕ S = Q1(s) + Q1 S  (1)
XOR gate.
X–NOR: D2 = A ⊕ S = AS + A( S ) (2)
Comparing 1 and 2: A = Q1 (Weff of S)
or A = Q1 (S)
⇒ A is connected to Q1 (d)
11. In the circuit shown, choose the correct timing diagram
of the output (Y) from the given waveforms W1, W2, W3
and W4.

Suppose the XOR gate is replaced by an XNOR gate.


Which one of the following options preserves the state
diagram?
(a) Input A is connected to Q2
(b) Input A is connected to Q2
(c) Input A is connected to Q1 and S is complemented
(d) Input A is connected to Q1  [2014]
Solution: (d)

(a) W1 (b) W2
(c) W3 (d) W4 [2014]
Chapter 5  Sequential Circuits  |  6.53

Solution: Solution: (d)

J1 = Q 2 J 2 = Q1
K1 = Q2 K 2 = Q1
Q1 = Q2 = 0 (intially)

PS NS
clk Q1 Q2 J1 K1 J 2 K2 Q1 Q2
0 0 0
1 0 0 10 01 1 0
Q1 ⋅ 01100
2 1 0 10 10 1 1
3 1 1 01 10 0 1
4 0 1 01 01 0 0
Hence, the correct option is (d).
1 3. The output of a 3-stage Johnson (twisted-ring) counter
(1) At clk 1, X1 = 0, X2 = 0 is fed to a digital-to-analogue (D/A) converter as shown
⇒ Q1 Q2 = 0 ⇒ Y = 0 in the figure below. Assume all states of the counter to
(2) X1 = 1, X2 = 0 ⇒ Q2 = 0 be unset initially. The waveform which represents the
⇒ Y = Q1 . Q2 = 0 D/A converter output V0 is
(3) X1 = 1, X 2 = 1 ⇒ Y = Q1 ⋅ Q2 = 1
(4) X = 1, X2 = 1 ⇒ Y = Q1 Q2 = 0
Q1 = 1, Q2 = 1
1 2. The outputs of the two flip-flops Q1 Q2 in the figure
shown are initialized to 0, 0. The sequence generated at
Q1 upon application of clock signal is

(a)

(b)
(a) 01110... (b) 01010...
(c) 00110… (d) 01100... [2014]
6.54 | Digital Circuits

(c) (c) D A = (Q A QB + Q A QB ), DB = Q A
(d) D A = (Q A QB + Q A QB ), DB = QB
[2011]
Solution: (d)
D FF synchronous counter:
(d) DFF: Qn+1 = Dn
Qn QA : 00 → 11 → 01 → 10 (PS)
It this is PS then NS QBQA:
11 → 01 → 10 → 00 (NS)
QB Q A (PS) QB Q A (NS) ⇒ DB D A
[2011] 00 11 11
Solution: (a) ⇒ 11 01 01
01 10 10
10 00 00
⇒ D B = QB Q A + QB Q A = QB ( Q A + Q A ) = D B = Q B
D A = Q B Q A + QB Q A

Hence, the correct option is (d).
1 5. Assuming that all flip-flops are in reset conditions ini-
tially, the count sequence observed at QA in the circuit
shown is

(a) 0010111... (b) 0001011...


Johnson counter is finished using counter with states: (c) 0101111... (d) 0110100... [2010]
Q2 Q2 Q2 (= D2 D1 D0) V0 Solution: (d)
0 0 0 0
1 1 0 4
1 1 1 6
1 1 1 7
0 1 1 3
0 0 1 1 As D FF copies the input D to Q (at clk)
0 0 0 0
Hence, the correct option is (a).
14. Two D flip-flops are connected as a synchronous coun-
ter that goes through the following QB QA sequence 00
® 11 ® 01 ® 10 ® 00 ®...
QA = 01101000………….(d)
The connections to the inputs DA and DB are
Output = QA, initially QA QB QC = 0
(a) DA = QB, DB = QA
(b) D A = Q A , DB = QB
Chapter 5  Sequential Circuits  |  6.55

QB ⊕ QC
clk DA DB DC QA QB QC Q1 = 0 = 1
0 0 0 0 0 0 0 Q = P2 ⋅ Q1 = 1 ⋅1 = 0 (Q1 = 1, LS )
2
1 1 0 0 1 0 0
(Q1, Q2) = (1, 0)
2 1 1 0 1 1 0
NOR: P1 = 1 ⇒ Q1 = 0
3 0 1 1 0 1 1
P2 = 1 ⇒ Q2 = 0 (Q, 0)
4 1 0 1 1 0 1
⇒ (C) NAND: (1, 0) then (1, 0).
5 0 1 0 0 1 0
NOR (1, 0) then (0, 0).
6 0 0 1 0 0 1
7 0 0 0 0 0 0 Hence, the correct option is (c).
17. What are the counting states (Q1, Q2,) for the counter
shown in the figure below?
Hence, the correct option is (d).
1 6. Refer to the NAND and NOR latches shown in the
figure. The inputs (P1 P2) for both the latches are first
made (0, 1) and then, after a few seconds, made (1, 1).
The corresponding stable outputs (Q1, Q2) are

(a) 11, 10, 00, 11, 10, .....


(a) NAND: first (0, 1) then (0, 1) NOR: first (1, 0) then (b) 01, 10, 11, 00, 01, ....
(0, 0) (c) 00, 11, 01, 10, 00, .....
(b) NAND: first (1, 0) then (1, 0) NOR: first (1, 0) then (d) 01, 10, 00, 01, 10, .... [2009]
(1, 0) Solution: (a)
(c) NAND: first (1, 0) then (1, 0) NOR: first (1, 0) then
(0, 0)
(d) NAND: first (1, 0) then (1, 1) NOR: first (0, 1) then
(0, 1) [2009]
Solution: (c)

Clk initially: Q1Q2 = 00 (Q1Q2 = 11) 00 → 11


(1) ⇒ J1 K1 = 1 & Q1 toggle to 1 (Q1 = 0)
Initially: P1P2 = (0, 1) NAND: As P1 = 0, ⇒ J 2 = 1 (LS of Q1 ), x2 = 1 ⇒ Q2 = 1(Q2 = 0)
Q1 = 1 (Q1, Q2) = (1, 0)
(2) Q1 = 0 ⇒ J 2 = 0 ⇒ Q1 = 1 (no change)
Now P2 ⋅ Q1 = 1 ⋅1 = 0 = Q2 00 → 11 → 10
NOR: P2 = 1 ⇒ Q2 = 0
q2 = 0 ⇒ J1 K1 = 0 Q2 = 0 (reset)
(Q1, Q2) = (1, 0)
(1) Last state Q1Q2 = 10
Q1 = P1 + Q2 = 0 = 1
Q2 = 1 J1 K1 ⇒ Q1 toggle to 0
Then P1P2 = (1, 1) NAND: Q1 = P1 − Q2 = Q2 &
Q1 = 0 = J 2 , K 2 = 1 ⇒ Q2 reset to 0

Q2= 0 (Case state)
6.56 | Digital Circuits

clk J1 K1 J 2 K2 Q1Q2 Solution: (b)


0 11 11 00
1 11 11 11
2 00 01 10
3 11 01 00

00 → 11 → 10 → 00 → 11… so on.
Hence, the correct option is (a). At JK = 1, FF toggles for each edge of clock.
1 8. For each of the positive edge-triggered J-K flip flop ∴ Q0 toggles at ΔT delay T = 2T (+1ΔT).
used in the following figure, the propagation delay is Q1 toggles according to Q0 (Q0 is clk for Q1) + ΔT
ΔT. T = 4T (+2 ΔT).

Which of the following waveforms correctly represents


the output at Q1?
(a)
(T = 2T, delay = ΔT)
(T = 4T, delay = 2ΔT)
Hence, the correct option is (b).
1 9. For the circuit shown in the figure, D has a transition
from 0 to 1 after CLK changes from 1 to 0. Assume
(b)
gate delays to be negligible

(c)

Which of the following statements is true?


(a) Q goes to 1 at the CLK transition and stays at 1.
(b) Q goes to 0 at the CLK transition and stays at 0.
(c) Q goes to 1 at the CLK transition and goes to 0
(d) when D goes to 1.
(d) Q goes to 0 at the CLK transition and goes to 1
when D goes to 1. [2008]
Solution: (c)

[2008]
Chapter 5  Sequential Circuits  |  6.57

At clk transition 1 → 0, (D = 0)

D + Clk = 1 ⇒ Q = 0

D + Clk = 0 Q =1

After that, at D transition: D = 1 (Clk = 0)

D + Clk = 0 + 0 = 1
& D + Clk = 1 + 0 = 0, Q = 1 = 0 = Q = 1 ⇒ Q = 1
(a) 00, 01, 10, 11, 00 …
⇒ Q goes to 1 at clk transition, and goes’ to Q at D (b) 00, 01, 10, 00, 01 …
transition. (c) 00, 01, 11, 00, 01 ...
Hence, the correct option is (c). (d) 00, 10, 11, 00, 10 ... [2007]
20. The following binary values were applied to the X and Solution: (b)
Y inputs of the NAND latch shown in the figure in the
sequence indicated below:
X = 0, Y = 1;
X = 0, Y = 0;
X = 1, Y = 1.
The corresponding stable P, Q outputs will be

Initially, Q0 Q1 = 00
and D0 = Q0 + Q1 = Q 0 Q1 D1 = Q0
D0 D1 Q0 Q1
(a) P = 1, Q = 0; P = 1, 0 0 0 0
Q = 0; P = 1, Q = 0 or P = 0, Q =1 1 0 1 0
(b) P = 1, Q = 0; P = 0, ⇒ Q1Q0 : 00, 01,10, 00
0 1 0 1
Q = 1 or P = 0, Q = 1; P =0, Q = 1
(c) P = 1, Q = 0; P = 1, 0 0 0 0
Q = 1; P = 1, Q = 0 or P = 0, Q = 1 1 0 1 0
(d) P = 1, Q = 0; P = 1, Q = 1; P = l, Q =1 Hence, the correct option is (b).
[2007] 2 2. For the circuit shown in the figure below, two 4-bit
Solution: (c) parallel-in serial-out shift registers loaded with the data
NAND later shown are used to feed the data to a full adder. Initially,
all the flip-flops are in clear state. After applying two
clk X Y P Q
clock pulses, the outputs of the full adder should be
1 0 1 1 0
:
2 0 0 1 1
3 1 1 Q P

⇒ If PQ (earlier) = 10 then PQ = QP = 10
If PQ (previous state) = 01 ⇒ PQ = 01
⇒ If X4 = 11, PQ depends on last state 1, 0 or 01
Hence, the correct option is (c).
21. For the circuit shown, the counter state (Q1 Q0) follows
the sequence (a) S = 0 C0 = 0 (b) S = 0 C0 = 1
(c) S = 1 C0 = 0 (d) S = 1 C0 = 1 [2006]
6.58 | Digital Circuits

Solution: (d) Seq Q1Q0 = 00 → 0.1 → 11 → 10 → 00 →


Initially Q0Q1 = 00
at Clk 1, Q0 = 1 ⇒ D0 = 1 ⇒ D0 →Q1 (as Q 1 = 0)
2, Q0 = 1 ⇒ D0 = 1 again = Q1
⇒ D0 = Q1
Clk 1: Q1 = 0 ⇒ D1 = 0 ⇒ Q0
Clk 2: Q1 = 1 = ⇒ D1 = 1
⇒ D1 → Q0
⇒ (a) (Q1,Q0).
Initially Q1Q2 = 0 Hence, the correct option is (a).
Q3 = 0 24. The present output Qn of an edge triggered JK flip-flop
S0 = 0 is logic 0. If J = 1, then Qn+1
C0 = 1 (a) cannot be determined
At 1st clk pulse: (b) will be logic 0
(c) will be logic 1
D1 = 1, Q1 = 1 (d) will race around [2005]
D2 = 1, Q2 = 1 Solution (c)
(C0 = 1) D3 = 1, Q3 = 1 Qn = 0 (present output)
⇒S=1+1+1=1 Now J = 1 (set), but k = ?
C0 = 1
(a) If k = 1 ⇒ JK = 11 toggle ⇒ Qn +1 = Q n = 1
After 2nd pulse
⇒ for J = 1 (Qn = 0)
D1 = D2 = 1 ⇒ Q1 = Q2 = 1
(b) k = 0 ⇒ JK = 10 (set)
As C0 W as 1, D3 = 1 = Q3
Qn+1 = 1.
⇒ 5 = 1 + 1 + 1 = 1, C0 = 1.
Qn+1=1
Hence, the correct option is (d).
Hence, the correct option is (c).
23. Two D-flip-flops, as shown below, are to be connected
25. The given figure shows a ripple counter using positive
as a synchronous counter that goes through the follow-
edge triggered flip-flops.
ing Q1Q0 sequence
If the present state of the counter is Q2Q1Q0 = 011, then
00  → 01  →11  →10  → 00  → ....... its next state (Q2Q1Q0) will be
The inputs D0 and D1, respectively, should be connected
as

(a) 010 (b) 100


(c) 111 (d) 101 [2005]
(a) Q1 and Q0 (b) Q0 and Q1 Solution: (b)
Q2Q1Q0 = 011 (ripple counter)
(c) Q1Q0 and Q1Q0 (d) Q1Q0 and Q1Q0
Next state: Q0 → toggles 1 to 0 = 0 (the edge trigger)
 [2006]
Q0 = 0 to 1 (–1)
Solution: (a)
For Q1: Q0 is clk (0 to 1) ⇒ + ve edge clock
⇒ Q1 toggles (1 to 0) =0.
Q2: Q1 is clk (0 to 1) ⇒ + ve edge
⇒ Q2 (0 to 1) = 1
⇒ Q2Q1Q0 = 100.
1 2 3 4 Hence, the correct option is (b).
Chapter 5  Sequential Circuits  |  6.59

26. In the modulo-6 ripple counter shown in the figure, 28. The digital block in the figure is realized using two pos-
the output of the 2-input gate is used to clear the J–K itive edge triggered D-flip-flops. Assume that for t < t0,
flip-flops. Q1 = Q2 = 0. The circuit in the digital block is given by:

(a)

(b)
The 2-input gate is
(a) a NAND gate (b) a NOR gate
(c) an OR gate (d) an AND gate [2004]
Solution: (c) (c)
Mod–6 ⇒ 0 to 5 (101)
At CBA, counter is reset.
110 (d)
⇒ A = 0 A =1

B = 1 B=0

C = 1 C =0 [2001]
Solution: (c)

⇒ C OR B = CB = CLR Q1Q2 = 00 (t < t0)



Y Q1 = D1 = 1 (when clk arrives)
CLR = CB (required)
When no clk ⇒ Q = D ⇒ Q1 = 1 always
⇒ an OR gate.
∴ Q2 D2 = 1 (fixed, as clk (Q1) = 1 always)
Hence, the correct option is (c).
2 7. A 4 bit ripple counter and a 4 bit synchronous counter
are made using flip flops having a propagation delay of
10 ns each. If the worst case delay in the ripple counter
and the synchronous counter be R and S respectively, X when clk, Q1 = D1 = 1, Q1 = 0 (clk for Q2)
then
(a) R = 10 ns, S = 40 ns ∴Q2 (–ve edge triggered) = 1 always
(b) R = 40 ns, S = 10 ns (Both the edge triggered) at the edge,
(c) R = 10 ns, S = 30 ns Q1 = D1(=1) = 1
(d) R = 30 ns, S =10 ns [2003] For D2, D2 = Q1 = 0 (at the edge)
Solution: (b) ⇒ Q2 = 0
Ripple counter: delay all 4 added But initially Q1 = 0 ⇒ Q1 = 0 D2
⇒ R = 4 × 10 = 40 ns ∴ At 1st pulse (t0), Q2 = 1, then goes to 0
Synch counter: Only 1 delay (as all FF works
simultaneously)
=5
⇒ R = 40 ns, S = 10 ns.
Hence, the correct option is (b).
Same as (c),
6.60 | Digital Circuits

But it is –ve edge triggered (a) 0.833 kHz (b) 1.0 kHz
(c) 0.91 kHz (d) 0.77 kHz [2000]
Solution: (b)
JK = 11, for all 4 FF.
Hence, the correct option is (c)
⇒ All 4 FF toggle for each clk pulse.
2 9. A sequential circuit using D Flip-Flop and logic gates is
shown in the figure, where X and Y are the inputs and Z For output frequency, find number of states
is the output. The circuit is IF all are initially cleared (0000), then
Q1 Q4
0 0 0 0
clk 1 1 0 0 0
2 0 1 0 0
3 1 1 0 0
(clk at falling edge: 1 → 0)
(a) S − R Flip-Flop with inputs X = R and Y = S Now C LR = Q4 Q2 ⇒ CLR = Q4 Q2 = 11
(b) S − R Flip-Flop with inputs X = S and Y = R ⇒ clears at Q4Q3Q2 = 1010
(c) J − K Flip-Flop with inputs X = J and Y = K
0000 → 0001 → 0110 → 0011 → … 1010
(d) Q1 − K Flip-Flop with inputs X = K and Y = J
⇒ 0 to 9 (10 states) ↓ cleared
[2000]
F 10 K
Solution: (d) ⇒ f output = = = 1 KHz.
10 10
Hence, the correct option is (b).

31. The ripple counter shown in the figure works as a

X Y Qn +1
0 0 Zn
0 1 Zn + Zn = 1
(a) mod-3 up counter
1 0 0+0 = 0 (b) mod-5 up counter
1 1 Z +0 = Z (c) mod-3 down counter
(d) mod-5 down counter [1999]
⇒ when XY = 01, Q = 1 (set) Y = set Solution: (d)
10, Q = 0 (reset) X = Reset
11 Q = z toggle ⇒ JK FF
⇒ X = Reset = K
Y = J (set).
Hence, the correct option is (d).
3 0. In the figure, the J and K inputs of all the four Flip-
Flips are made high. The frequency of the signal at out-
PR Q A Q B Q C
put Y is
⇒ at 010 counter is reset.

32. In the figure given below, A = 1 and B = 1. The input B


is now replaced by a sequence 101010..... the outputs x
and y will be
Chapter 5  Sequential Circuits  |  6.61

Solution:

(a) fixed at 0 and 1, respectively


(b) x = 1010 .....while y = 0101 ......
(c) x = 1010......and y = 1010......
(d) fixed at 1 and 0, respectively [1998]
Solution: (a)

Unused states are


101 
To check lock out condition if we give unused 110 
111 
I/P B → 101010…
states as input to the ckt and corresponding next state is
→ when A = 1, B = 1
one of the used state the ckt is said to be lock out free
X = A ⋅Y = Y or No. Lock out condition exist”
For 101:
nly Y = B ⋅ X = X
→ When A = 1 (fixed) B = 0, θ2 θ1 θ0 J0 K0 J1 K1 J2 K2 θ
2+
θ+ θ
1 0+

Y = O ⋅ X = (1) 1 1 0 0 1 1 1 0 1 0 0 0

used state
X = A ⋅ Y = (0)
∴No Lock out
→ A = 1 X = Y = 1 = (0) X = 0 fixed
⇒ For 111:
B = 1 Y = X = 0 = (1) Y = 1 fixed

θ2 θ1 θ0 J0 K0 J1 K1 J2 K2 θ θ+ θ
2+ 0+
Hence, the correct option is (a). 1

1 1 1 0 1 1 1 1 1 0 0 0
 used state
Five-marks Question  No Lock out
∴Counter is lock out free.
1. The mod-5 counter shown in figure counts through
(b) Minimum time Required = 2tF + tA
states Q2 Q1 Q0 = 000, 001, 010, 011 and 100.
1
Max clock rate =
2t F + t A

2. A sequence generator is shown in figure. The counter


status (Q0 Q1 Q2) is initialized to 010 using preset/clear
inputs.

(a) Will the counter lockout if it happens to be in any


one of the unused states ?
(b) Find the maximum rate at which the counter will
operate satisfactorily. Assume the propagation
delays of flip-flop and AND gate to be tF and tA.
 [1998]
6.62 | Digital Circuits

The clock has a period of 50 ns and transitions take θ0 θ1 θ2 θ4


place at the rising clock edge. 1 0 0 0
(a) Give the sequence generated at Q0 till it repeats.
1 clk 0 0 0 1
st
(b) What is the repetition rate for the generated se-
quence? [1997] 2nd clk 0 0 1 1
Solution:(a) D0 = θ 2 3rd clk 0 1 1 0
4th clk 1 1 0 1
D1 = θ 0
5th clk 1 0 1 0
D2 = θ1 .θ 0
6th clk 0 1 0 0
D0 D1 D2 θ0 θ1 θ2
7th clk 1 0 0 0
1 1 0 0 1 0
So after 7th clk pulse pattern 1000 will reappear on ϑ0 ϑ1
1 0 0 1 1 0
ϑ2 i3
1 0 1 1 0 0
(b)
0 0 1 1 0 1
0 1 0 0 0 1
θ0 θ1 θ2 θ3
1 1 0 0 1 0 1 1 0 0
So sequence at ϑ0 is 1 0 0 1
∴1100 should be their initially so that after 1st clk 1001
patterns comes
4. A state machine is required to cycle through the fol-
lowing sequence of states:
(b) No. of states = 5
ABC
Talk = 50 n sec
1 1
Repetition rate = =
5 × talk 5 × 50 × 10 −9
= 4 MHz One possible implementation of the state machine is
shown below. Specify what signals should be applied
3. A 4-bit shift register, which shifts 1 bit to the right at to each of the multiplexer inputs.
every clock pulse, is initialized to values (1000) for (Qo
Q1 Q2 Q3). The D input is derived from Q0, Q2 and Q3
through two XOR gates as shown in figure.

[1996]
(a) Write the 4-bit values (Q0 Q1, Q2 Q3) after each
clock pulse till the pattern (1000) reappears on (Q0
Q1 Q2 Q3).
(b) To what values should the shift register be initial-
ized so that the pattern (1001) occurs after the first
clock pulse?
Solution: (a)
D = θ 0 ⊕ θ 2 ⊕ θ3 [1996]
Chapter 5  Sequential Circuits  |  6.63

Solution: DC = I0 I1 I2 I3
Present State Next State FF input C 0 (2) (4) 6

A B C A B C DA DB DC C 1 (3) 5 7
0 1 C 0
0 0 0 0 1 0 0 1 0
0 1 0 1 1 1 1 1 1
1 1 1 1 0 0 1 0 0
1 0 0 0 1 1 0 1 1
0 1 1 1 0 1 1 0 1
1 0 1 0 0 0 0 0 0
DA = ∑m(2, 3, 7) + ∑d(1, 6)
DB = ∑m(0, 2, 4) + ∑d(1, 6)
DC = ∑m(2, 3, 4) + ∑d(1, 6)

DA = I0 I1 I2 I3
C 0 (2) 4 6
C 1 (3) 5 (7)
0 1 0 C

DB = I0 I1 I2 I3
C (0) (2) (4) 6
C 1 3 5 7
C C C 0
Chapter 6
Logic Families
Solution: 
One-mark Questions
A B Y M1 M2 Y
1. For the circuit shown in the figure P and Q are the
0 0 0 OFF ON GND
inputs and Y is the output. [2017]
0 1 0 ON OFF A
1 0 0 OFF ON GND
PMOS 1 1 1 ON OFF A

∴ From the above truth table output = AB ⇒ AND


Y
gate
Hence, the correct option is (D).
P NMOS
3. The output (Y) of the circuit shown in the figure is
Q

The logic implemented by the circuit is


(A) XNOR (B) XOR
(C) NOR (D) OR
2. The logic functionality realized by the circuit shown
below is [2016]
B
B

A
Y

(a) A + B + C (b) A + B ⋅ C + A ⋅ C
(c) A + B + C (d) A ⋅ B ⋅ C  [2014]
(A) OR (B) XOR
(C) NAND (D) AND
B
B

M1
A
Y
M2
Chapter 6  Logic Families  |  6.65

→ CMOS logic 5. The full forms of the abbreviations TTL and CMOS in
reference to logic families are
⇒ y = A. B. C
(a)  Triple Transistor Logic and Chip Metal Oxide
= A+ B +C Semiconductor
(b) Tristate Transistor Logic and Chip Metal Oxide
y = A + B + C. Semiconductor

Hence, the correct option is (a). (c) Tristate Transistor Logic and Complementary Met-
al Oxide Semiconductor
4. In the circuit shown (d) Tristate Transistor Logic and Complementary Met-
al Oxide Silicon [2009]
Solution: (c)
TTL → Tristate Transistor logic
CMOS → Complementary metal oxide S.C.
Hence, the correct option is (c).
6. Given figure is the voltage transfer characteristic of

(a) an NMOS inverter with enhancement mode tran-


sistor as load
(a) Y = AB + C (b) Y = ( A + B)C
(b) an NMOS inverter with depletion mode transistor
(c) Y = ( A + B )C (d) Y = AB + C  [2012]
as load
Solution: (a) (c) a CMOS inverter
(d) a BJT inverter [2004]
Solution: (c)

At Vin = 0 at Vin = High


Vout = High Vout = 0
⇒ Very close to be ideal inverter ⇒ CMOS.
Hence, the correct option is (c).
→ A + B (as A || B) 7. The output of the 74 series GATE of TTL gates is taken
→ C series with (A + B) in N channel. from a BJT in
(a) totem pole and common collector configuration
⇒ C (A + B) (reverse in P)
(b) either totem pole or open collector configuration
⇒ Y = C (A + B) (c) common base configuration
(d) common collector configuration [2003]
y = C ( A + B)
Solution: (b)
= C + A+ B The output of the 74 series GATE of TTL gates is
taken from a BJT in either totem pole or open collector
y = C + AB configuration.
Hence, the correct option is (a). Hence, the correct option is (b).
6.66 | Logic Families

8. A Darlington emitter-follower circuit is sometimes Solution: (a)


used in the output stage of a TTL gate in order to
(a) increase its IOL
(b) reduce its IOH
(c) increase its speed of operation
(d) reduce power dissipation [1999]
Solution: (c)
Darlington EF: It has low R0 which results in short
time constant for charging for capacitive load (at out-
put). Small charging time results in higher speed of
operation.
Hence, the correct option is (c).
9. Commercially available ECL Gates use two ground
lines and one negative supply in order to Vth = 2V.
(a) reduce power dissipation
(b) increase fan-out For inverter, Vi = ?
(c) reduce loading effect = Vi V0
(d) eliminate the effect of power line glitches or the −5 0
biasing circuit. [1999] 0 −5 inverter (a)
Solution: (d) If Vi = −5 ⇒ N → ON (VD > Vth)
ECL uses two ground lines and one negative supple to P → OFF
eliminate the power line glitches or effects of biasing.
and V0 = 5 – 5 = 0
Hence, the correct option is (d).
Vi = 0 ⇒ P → ON (VD < Vth)
10. The noise margin of an TTL gate is about
(a) 0.2 V (b) 0.4 V N → OFF
(c) 0.6 V (d) 0.8 V [1998] V0 = −5
Solution: (b) Hence, the correct option is (a).
TTL noise margin ~ 0.4 V. 12. In standard TTL the ‘totem pole’ stage refers to
Hence, the correct option is (b). (a) the multi-emitter input stage.
(b) the phase splitter.
11. The threshold voltage for each transistor in figure is 2 (c) the output buffer.
V. For this circuit to work as an inverter, Vi must take (d) open collector output stage. [1997]
the values
Solution: (c)
TTL: An output stage having two transistors connected
such that only one is ON at a time is called TOTEM
POLE stage.
80 Tetempole → output buffer.
Hence, the correct option is (c).
13. The inverter 74 AL S01 has the following specifications:
IOH max = −0.4 mA, IOL max = 8 mA, IIH max = 20µA,
IIL max = −0.1 mA.
The fan out based on the above will be
(a) 10 (b) 20
(c) 60 (d) 100 [1997]
(a) −5 V and 0 V (b) –5 V and 5 V
(c) 0 V and 5 V (d) −3 V and 3 V
 [1998]
Chapter 6  Logic Families  |  6.67

Solution: (b) ⇒ F = A(B + C) + DE.


74 AL SOL IOH = −0.4 mA IOL max = 8 mA Hence, the correct option is (c).
IiH max = 20 μA IiL max = −0.1 mA 15. The gate delay of an NMOS inverter is dominated by
FANOUT: charge time rather than discharge time because
FO for high level: (a) the driver transistor has a larger threshold voltage
than the load transistor
I OH max 0.4 m 0.4 × 10 −3 0.4
= = = × 1000 (b)  the driver transistor has larger leakage currents
I IH max 20 m 20 × 10 −6 20 compared to the load transistor
(c) the load transistor has a smaller W/L ratio com-
I DL max 8m pared to the driver transistor
(FO)low level = = = 80
I IL max 0.1 m (d) none of the above. [1997]

Solution: (c)
F.O. = min [(F))H, (FO)L] = min (20, 80)
In NMOS inverter, if load transistor has smaller (W/L)
F.O = 20.
ratio as compared to driver, then inverter is dominated
Hence, the correct option is (b). by charge time.
1 4. For the NMOS logic gate shown in figure, the logic Hence, the correct option is (c).
function implemented is
16. Figure shows the circuit of a gate in the Resistor
Transistor Logic (RTL) family. The circuit represents a

(a) ABCDE
(a) NAND (b) AND
(b) ( AB + C ) ⋅ ( D + E ) (c) NOR (d) OR [1992]
(c) A⋅(B + C) + D ⋅ E
Solution: (d)
(d) ( A + B) ⋅ C + D ⋅ E  [1997]
Solution: (c)
NMOS Logic:

RTL logic family


Vi Vi V0
1 2
(1) LOW  LOW  T1, T2 – OFF ⇒ LOW V0
(2) LOW  HIGH  T2 – ON ⇒ HIGH V0
(3) H  L  T1 – ON ⇒ V0 HIGH
D and E → series ⇒ DE (4) L  H  T1, T2 → ON, HIGH
Only B||C → B + C ⇒ V0 is low only when both
and A series (B||C) T1 and T2 are off ⇒ No = (Vi 1 + Vi2 ) ⇒ (d) or gate.
⇒ A(B + C) Hence, the correct option is (d).
⇒ A(B + C) + DE 1 7. The CMOS equivalent of the following nMOS gate
and upper NMOS → inverter (figure) is (draw the circuit).
6.68 | Logic Families

When clock is HIGH,


a) For N-channel MOS, It is ON
b) P-MOS-off
∴Circuit can be redrawn as

[1991]
Solution:

From Stage 1: 2t is (A + B). C and stage 2 is inverter


(CMOS)
⇒ output = (A + B) C.
19. Among the digital IC-families ECL, TTL and CMOS:
(a) ECL has the least propagation delay
(b) TTL has the largest fan-out
(c) CMOS has the biggest noise margin
(d) TTL has the lowest power consumption
[1989]
Solution: (a)
In ECL TTL, CMOS
least prop delay: ECl is fastest logic family.
18. In the figure, the Boolean expression for the output in Hence, the correct option is (a).
terms of inputs A, B and C when the clock ‘ck’ is high 20. A logic family has threshold voltage VR = 2 V, mini-
is given by mum guaranteed output high voltage VOH = 4 V, mini-
mum accepted input high voltage VIH = 3 V, maximum
guaranteed output low voltage VOL = 1 V, and maxi-
mum accepted input low voltage VIL = 1.5 V. Its noise
margin is
(a) 2 V (b) 1 V
(c) 1.5 V (d) 0.5 V [1989]
Solution: (d)
VR = 2V (threshold)
VOH = 4V
[1991] VOL = 1V
Solution: VIH = 3V
VIL = 1.5V
Noise margin is calculated as (NM)4 = VOH – VIH = 4 – 3
= 1V (noise m is High level voltage)
(NM)L = NM in low voltage level = VIL – VOL = 1.5 – 1 =
0.5V
NM = min [NMH – NML] = min [1], 0.8] = 0.5 V.
(NM represents the amplitude of noise voltage that may
cause the logic level to change.)
Hence, the correct option is (d).
Chapter 6  Logic Families  |  6.69

21. Fill in the blanks of the statements below concerning Solution:  Consider the circuit diagram given below
the following Logic Families: X0
Standard TTL (74 XXLL), low power TTL (74L XX),
low power schottky Wired AND
TTL (74L SXX), schottky TTL(74 SXX), emitter cou- X1
pled logic (ECL), CMOS. X2 Y

(a) Among the TTL Families, _________ family re-


X3
quires considerably less power than the standard
TTL (74 XX) and also has comparable propagation
From the above figure the output will be
delay.
(b) 
Only the _________ family can operate over a Y = X 0 ⎡⎣( X1 ⊕ X 2 ) ⋅ X 0 ⎤⎦ + X 3
wide range of power supply voltages. [1987]
Y = X3
Solution: Output Y will be high when Input X3 is High.
TTL family list: Therefore there are 8 such possible combinations.
Hence, the correct answer is 8.
FAMILY DELAY POWER
(ns) (mw)
2. The logic function implemented by the following cir-
1. Basic 10 10
cuit at the terminal OUT is
2. Low-power (L) 35 1
3. SCHOTTKY(S) 3 18
4. Low-power Schottky (LS) 9 2
5. Advance scholtky (AS) 1.5 1Z
6. Adv. low-power schottky (ALS) 4 1

ECl: power dissipation ~ 25 mw

CMOS: 0.002 mv

Prop delay ~ 2ns

Now Low Power (LS) has delay comp. to basic & power (a) P NOR Q (b) P NAND Q
less than it (c) P OR Q (d) P AND Q [2008]
(a) Low Power (LS) schottky Solution: (d)
(b) CMOS

Two-marks Questions
1. The logic gates shown in the digital circuit below use
strong pull-down nMOS transistors for LOW logic level
at the outputs. When the pull-downs are off, high-value
resistors set the output logic levels to HIGH (i.e. the
pull-ups are weak). Note that some nodes are intention-
ally shorted to implement “wired logic”. Such shorted
nodes will be HIGH only if the outputs of all the gates
whose outputs are shorted are HIGH. [2018]

X0

X1
X2
Y
X3
6.70 | Logic Families

From the circuit, Q2 : Active


ln NMOS → PQ → series Q4 : Cutoff
P → P||Q Q3 : Sat.
⇒ P AND Q. Hence, the correct option is (b)
Hence, the correct option is (d). 4. The transistors used in a portion of the TTL gate shown
3. The circuit diagram of a standard TTLNOT gate is in the figure have a β = 100. The base-emitter voltage
shown in the figure. When Vi = 2.5 V, the modes of is 0.7 V for a transistor in active region and 0.75 V for a
operation of the transistors will be transistor in saturation. If the sink current I = 1 mA and
the output is at logic 0, then the current IR will be equal
to

(a) Q1: reverse active;


Q2: normal active;
Q3: saturation; (a) 0.65 mA (b) 0.70 mA
Q4: cut-off (c) 0.75 mA (d) 1.00 mA [2005]
(b) Q1: reverse active Solution: (c)
Q2: saturation;
Q3: saturation;
Q4: cut-off
(c) Q1: normal active;
Q2: cut-off;
Q3: cut-off;
Q4: saturation
(d) Q1: saturation;
Q2: saturation;
Q3: saturation;
Q4: normal active [2007]
Solution: (b)
β = 100
VBE = 0.7 V (active region)
VBE sat = 0.75 V
Isat = 1 mA
Output = O(logic), IR = ?
Jsink = 1 mA = Ic sat
(i.e. BJT in sat mode)
VBE sat = 0.75
and 0.75 = JR. 1k
0.75
JR = = 0.75 mA
1000
Vi = 2.5 V (TTL NOT gate)
Hence, the correct option is (c).
Q1 : EB : RB
5. Both transistors T1 and T2 shown in the figure have a
BC : FB Reverse Active
threshold voltage of 1 volts. The device parameters K1
Chapter 6  Logic Families  |  6.71

and K2 of T1 and T2 are, respectively, 36 µA/V2 and 9 (P)  (Q)  (R)  (S)


µA/V2. The output voltage V0 is Fanout is minimum  DTL  DTL  TTL CMOS
Power consumption TTL CMOS ECL DTL
is minimum
Propagation delay CMOS ECL TTL TTL
is minimum
The correct column is
(a) P (b) Q
(c) R (d) S [2003]
Solution: (b)
CMOS: Power Consumption is lowest
ECL: Program delay is minimum
⇒ (Q).
(a) 1 V (b) 2 V Hence, the correct option is (b).
(c) 3 V (d) 4 V [2005] 7. The circuit in the figure has two CMOS NOR-gates.
Solution: (c) This circuit functions as a-

(a) flip-flop
(b) Schmidt trigger
(c) monostable multi-vibrator
(d) astable multi-vibrator [2002]
Solution: (c)

Vth = 1 V
K1 = 36 μA/V2
K2 = 9 μA/V2
V0 = ?
As ID1 = ID2 (sat mode)
⇒ K1 (Vas1 – Vt)2 = K2(Vas2 – Vt)2
This circuit will act like a mono-stable multi-vibrator.)
or k1 (Va s1 − Vt = k2 Va s2 − Vt
Hence, the correct option is (c).
Vas1 = 5 – V0, Vas2 = (V0 – 0)gmd
8. Given that for a logic family,
K1(Vas1 – Vt)2 = K2 (Vas2 – Vt)2 Vt = 1
VOH is the minimum output high-level voltage.
4
36 (5 − V − 1) 2 = 9 (V − 1) 2 Vas1 = 5 – V0 VOL is the maximum output-low-level voltage.
0 0
4(4 – V0)2 = (V0 – 1)2 VGs2 = (V0 – 0) VIH is the minimum acceptable input high-level voltage
and
2(4 – V0) = (V0 – 1)
8 – 2V0 = V0 – 1 VIL is the maximum acceptable input low-level voltage.
3V0 = 9 The correct relationship is
(a) VIH > V0H > VIL > VOL
V0 = 3V.
(b) VOH > VIH > VIL> VOL
Hence, the correct option is (c). (c) VIH > VOH > VOL> VIL
6. The DTL, TTL, ECL and CMOS family GATE of digi- (d) VOH > VIH > VOL > VIL [1987]
tal ICs are compared in the following 4 columns
6.72 | Logic Families

Solution: (b) collector of the pull-down transistor. The purpose of


VOH: (V HIGH) min value (2.4 V) min value V IH: this diode is to isolate the output node from the power
(V in high) min ( ~ 2 V) min value. supply Vcc. [1994]
VOL: (Vo low) max (0.4V) max value V IL: Solution:
(V in low) max (0.8 V) max value This is TRUE, to isolate output from power Vcc, a
From general value: VOH > VIH > VIL > VOL . diode is attached bho E of pull-up and C of pull-down
transfer (TRUE)
Hence, the correct option is (b).
9. In the output stage of a standard TTL, we have a diode
between the emitter of the pull-up transistor and the
Chapter 7
Memories
SRAM Cell:
One-mark Questions
1. In a DRAM, [2017]
(A) periodic refreshing is not required
(B) information is stored in a capacitor
(C) Information is stored in a latch
(D) Both read and write operation can be performed
simultaneously

2. An 8085 microprocessor based system uses a 4 K × 8 bit


RAM whose starting address is AA00 H. The address of
the last byte in this RAM is
(a) 0FFFH (b) 1000H 4-transistor for storing each bit on SRAM + 2 transis-
(c) B9FFH (d) BA00H tors for controlling read from or writing into cell.
 [2001] Hence, the correct option is (a).
Solution: (d) 4. The minimum number of MOS transistors required to
4k × 8 bit RAM, starting add: AAOOH make a dynamic RAM cell is
(a) 1 (b) 2
8085 → 16 and lines total.
(c) 3 (d) 4 [1995]
4k → 212, 12 odd lines : A0 – A11
Solution: (b)
4k = 4 × 1024 = 4096 bytes
From 7.3, it can be seen that DRAM cell required one
A15 – A9 are fixed (1010 101) transistor.
= 1000 H Hence, the correct option is (b).
Starting add : AAOO 5. A dynamic RAM consists of
+ number of bytes : 1000 (a) 6 transistors
Last bytes BA00H. (b) 2 transistors and 2 capacitors
(c) 1 transistor and 1 capacitor
Hence, the correct option is (d)
(d) 2 capacitors only [1994]
3. Each cell of a static Random Access Memory contains
Solution: (c)
(a) 6 MOS transistors
(b) 4 MOS transistors and 2 capacitors A dynamic RAM consists of 1 transistor and 1 capacitor.
(c) 2 MOS transistors and 4 capacitors DRAM : A Dynamic RAM cell (MOS) is:
(d) 1 MOS transistor and 1 capacitors [1996]
Solution:(a)
Each cell of a static RAM contains six MOS transistors.
Each bit on static RAM is stored on four transistors out
of which two are PMOS and two are NMOS, that from
cross-coupled inverters. Remaining two transistors are
⇒ 1 transistor + 1 capacitor.
used to control reading from or writing into cell.
Hence, the correct option is (c).
6.74 | Digital Circuits

6. A PLA can be (c)


(a) as a microprocessor
(b) as a dynamic memory
(c) to realize a sequential logic
(d) to realize a combinational logic. [1994]
Solution: (d)
PLA is a type of fixed architecture logic devices with
programmable AND gates followed by programma-
ble OR gates. The PLA is used to implement complex
combinational circuits.
PLA → complex complain cpt as arrays of (d)
prog AND & OR.
Hence, the correct option is (d).
7. Choose the correct statements (s) from the following:
(a) PROM contains a programmable AND array and a
fixed OR array.
(b) PLA contains a fixed AND array and a program-
mable OR array.
(c) PROM contains a fixed AND array and a program-
mable OR array
(d) PLA contains a programmable AND array and a [2014]
programmable OR array. [1992] Solution: (b)
Solution: (c) & (d) For the construction of SRAM we require 4 MOSFET
(a) PROM → Prog. AND and fixed OR  (has fusible (2 PMOS and 2 NMOS) with interchanged outputs
diodes at each inter seem) ⇒ Prog. OR connected to each CMOS inverter, This condition is
(b) PLA → fixed AND, Prog. OR  (Prog. AND & followed in option (b)
prog OR) Hence, the correct option is (b).
(c) PROM → fixed AND, Prog. OR 
2. There are four chips each of 1024 bytes connected to a
(d) PLA → Prog. AND & OR 
16 big address bus as shown in the figure below. RAMs
Hence, the correct option is (c) and (d). 1, 2, 3 and 4 respectively are mapped to address

Two-marks Questions
1. If WL is the Word Line and BL the Bit Line, an SRAM
cell is shown in
(a)

(b)
(a) 
0C00H-0FFFH, 1C00H-1FFFH, 2C00H-2FFFH,
3C00H-3FFFH
(b) 
1800H-1FFFH, 2800H-2FFFH, 3800H-3FFFH,
4800H-4FFFH
(c) 
0500H-08FFH, 1500H-18FFH, 3500H-38FFH,
5500H-58FFH
(d) 
0800H-0BFFH, 1800H-1BFFH, 2800H-2BFFH,
3800H-3BFFH [2013]
Chapter 7  Memories  |  6.75

Solution: (d) (a) 0100 – 02 FF (b) 1500 – 16 FF


(c) F900 – FAFF (d) F800 – F9FF [2005]
Solution: (d)

S1 = A13
S0 = A12
A10 – A15 → decoder input
A0 – A9 → add lines for RAM
⇒ A15 A14 = 00
A11 A10 = 10
(a) A12 A13 = 00 ⇒ S1 S12 = 00 → R1 selected. For chip 1 : A9A8 = 01
A15 A14 A13 A12 A11 A10 A9 ………….A0 A7 – A0 = OO – FF
0 0 0 0 1 0 0 …………...0 ⇒ A9 – A0 : 01 0000 0000 to
1 0 11 1 01 FFFF FFFF
(R1) i.e. 0800H – 08FF H For chip 2, CS = A9 A8
(b) A13A12 = 01 (S1S2 = 01) → R2 ⇒ A9 A8 = 10
⇒ A9 – A0 = 10 0000 0000 to
10 FFFF FFFF
0 0000 0000 to 10 FFFF FFFF add are converted
R2 ⇒ 1800A – 18FF by chip 1 and chip 2
(c) R3 : A13 A12 = 10 ⇒ A15 A14 A13 A12 A11 A10 = ∴ F800 – F9FF are not represented.
0010  10  0   0 Hence, the correct option is (d).
     1   1 4. In the circuit shown in the figure, A is parallel-in, parallel-
⇒ 2800H – 2BFFH R3 out 4 bit register, which loads at the rising edge of the
(d) R4 : A13 A12 = 1 ⇒ 0011 100 0 clock C. The input lines are connected to a 4 bit bus, W.
    0011 101 1 ⇒ 3800H – 3 BFFH] Its output acts as the input to a 16 × 4 ROM whose output
is floating when the enable input E is 0. A partial table of
Hence, the correct option is (b). the contents of the ROM is as follows
3. What memory address range is NOT represented by
chip 1 and chip 2 in the figure. A0 to A15 in this figure Address Data
are the address lines and CS means Chip select. 0 0011
2 1111
4 0100
6 1010
8 1011
10 1000
11 0010
14 1000
6.76 | Digital Circuits

∴ At address 1010 ( = 10), data = 1000.


Hence, the correct option is (c).
5. In the DRAM cell in the figure, the Vt of the NMOSFET
is 1 V. For the following three combinations of WL and
BL voltages.

(a) 5 V; 3 V; 7 V (b) 4 V; 3 V; 4 V
(c) 5 V; 5 V; 5 V (d) 4 V; 4 V; 4 V
 [2001]
Solution:(b)
The clock to the register is shown, and the data on the
W bus at time t1 is 0110. The data on the bus at time t2
is
(a) 1111 (b) 1011
(c) 1000 (d) 0010 [2003]
Solution: (c)

Vt = 1V (VMOS)
Hence, the correct option is (b).
6. If CS = A15 A14 A13 is used as the chip select logic of
a 4K RAM in an 8085 system, then its memory range
will be
(a) 3000 H – 3 FFF H
(b) 7000 H – 7 FFF H
ROM Add: Data
(c) 5000 H – 5 FFF H and 6000 H – 6 FFF H
(d) 6000 H – 6 FFF H and 7000 H – 7 FFF H
0 0011
[1999]
2 1111
4 0100 Solution:(d)
(6) → 1010 CS = A15 A14 A13 of 4k RAM (8085)
8 1011 4k ⇒ 212, i.e. 12 add lines A0 − A11
(10) → 1000
and CS = 011
11 0010
14 1000
⇒ A15  A14  A13  A12  A11…………….A0
0   1   1   0   0……………...0
   1 ……………...1
At t1, W = 0110
     1   0……………...0
∴ Input to ROM at t1 = 0110 = add. 6    1 ………………1
∴ at add 6, data = 1010 (w) A12 = ⇒A15A14A13A12 = 0110 = 6
At t2, W becomes 1010, ∴ A gets input 1010 and sup- ⇒ Range: 6000 – 6FFF
plies to ROM.
A12 = 1  ⇒ A15A14A13A12 = 0111 = 7 ⇒ 7000 – 7FFF
Chapter 7  Memories  |  6.77

⇒ Total range: 6000 – 6FFF and 7000 – 7FFF. P P Q Q R R


Hence, the correct option is (d).
× × ×
Two-mark Questions × × × F
1. A 2 × 2 ROM array is built with the help of diodes × × ×
as shown in the circuit below. Here WO and W1 are
signals that select the word lines and B0 and B1 are
P
signals that are output of the sense amps based on the
stored data corresponding to the bit lines during the
read operation.[2018] Q

B0 B1
Sense R
amps B0 B1
W0 The Boolean function F implemented is  [2017]
W0 D00 D01
W1 D10 D11 PQR + PQR + PQR
(A)
W1
VDD
Bits stored in the ROM array (B) (
P+Q+R P+Q+R P+Q+R )( )( )
PQR + PQR + PQR
(C)

(D) (
P+Q+R P+Q+R P+Q+R )( )( )
⎡1 0 ⎤ ⎡0 1 ⎤ 3. Consider a discrete memory less source with alphabet
⎢0 1 ⎥ (B)
(A) ⎢1 0 ⎥
⎣ ⎦ ⎣ ⎦ S = {s0, s1, s2, s3, s4, ….} and respective probabilities of
⎡1 0 ⎤ ⎡1 1 ⎤ ⎧1 1 1 1 1 ⎫
(C) ⎢ ⎥ (D)
⎢0 0 ⎥ occurrence P = ⎨ , , , , ...⎬ . The entropy of
⎣1 0 ⎦ ⎣ ⎦ ⎩ 2 4 8 16 32, ⎭
the source (in bits) is _____.  [2016]
During the real operation, the selected word line goes
high and the other word line is in a high impedance Solution:  The entropy of discrete memory less source
state. As per the implementation shown in the circuit is given as
1
diagram above, what are the bits corresponding to Dij H = P1 log2 + P2 log2 1 + ….
(where I = 0 or 1 and j = 0 or 1) stored in the ROM? P1 P2
Solution:  Consider the figure 1 1 1
H = log22+ log2 4 + log28+ ….
B0 B1 2 4 8
Sense 1 1 1 1
amps = + × 2 + .3 + π⋅4 ….
W0 2 4 8 16
1 1 1 1
= + 2 2 + 3 3 + 4 4 ….
W1 2 2 2 2
VDD α n
⎛ 1⎞
= ∑ n ⎜ ⎟ ⇒ AGP
⎝ 2⎠
n = 0
ab, (b + d)br, (a + 2d)br2
⎡1 0 ⎤ a = 1, b = 1/2, d = 1, r = 1/2
Bit stored in ROM array is ⎢ ⎥.
⎣0 1 ⎦ ab dbr
s = 1 − r + 
(1 − r ) 2
Hence, the correct option is (A).
2. A programmable logic array (PLA) is shown in the 1 1 1
1× 1× ×
­figure. 2 += 2 2 = 2.

1 ⎛ 1⎞
2
1−
2 ⎜⎝1 − ⎟⎠
2
Hence, the correct option is (2).
Chapter 8
ADC and DAC
conversion time for a 2 V input will be
One-mark Questions (a) 10 µs (b) 20 µs
(c) 40 µs (d) 50 µs [2000]
1. A digital system is required to amplify a binary-encoded Solution: (b)
audio signal. The user should be able to control the gain
Full scale V = 2.55 V
of the amplifier from a minimum to a maximum in 100
increments. The minimum number of bits required to Conversion time for 1V = 20 μs
encode, in straight binary, is Conversion time for 2V = 20 μs (same as it depends on
(a) 8 (b) 6 the number of bits only).
(c) 5 (d) 7 [2004] Hence, the correct option is (b).
Solution: (d) 5. The number of comparators in a 4-bit flash ADC is
Step size = 100 (a) 4 (b) 5
⇒ 100 different voltages can be represented (c) 15 (d) 16 [2000]
⇒ If n is the number of bits, 2n ≥ 100 Solution: (c)
64 ≥ 100 ⇒ min n = 7 Flash ADC:
Hence, the correct option is (d). For n output bits, (2n – 1) comparators required.
2. The minimum number of comparators required to build ⇒ For 4-bits, (2n – 1) = 16 – 1
an 8-bit flash ADC is        = 15 comparator required.
(a) 8 (b) 63 Hence, the correct option is (c).
(c) 255 (d) 256 [2003]
6. The resolution of a 4-bit counting ADC is 0 5 volts. For
Solution: (c) an analogue input of 6.6 volts, the digital output of the
8-bit flash type ⇒ 2n – 1 = 28 – 1 ADC will be
= 256 – 1 (a) 1011 (b) 1101
(c) 1100 (d) 1110 [1999]
255 comparators.
Solution:(d)
Hence, the correct option is (c).
Resolution = 0.5    (4-bit)
3. The number of comparators required in a 3-bit com-
parator type ADC is Vin = 6.6V
(a) 2 (b) 3 V0 of ADC = input V/Resolution (conv in binary)
(c) 7 (d) 8 [2002]
6.6
Solution: (c) = = 13.2 ~ 14 (rounding of to higher digit)
015
3-bit comparator type: (2n – 1) comparators  ⇒ 14 = 8 + 4 + 2 = (1110)2.
: 23 – 1 Hence, the correct option is (d).
: 7. 7. The advantage of using a dual slope ADC in a digital
Hence, the correct option is (c). voltmeter is that
(a) its conversion time is small
4. An 8 bit successive approximation analogue to digital
(b) its accuracy is high
converter has full scale reading of 2.55 V and its con-
(c) it gives output in BCD format
version time for an analogue input of 1 V is 20 µs. The
(d) it does not require a comparator [1998]
Chapter 8  ADC and DAC  |  6.79

Solution: (b) (ii)


Dual-slope ADC : high accuracy.
The advantage of using a dual slope ADC in a digital
voltmeter is its high accuracy.
Hence, the correct option is (b).
8. For an ADC, match the following: if
List-I
(A) Flash converter
(iii)
(B) Dual slope converter
(C) Successive Approximation Converter
List-II
(1) requires a conversion time of the order of a few
seconds
(2) requires a digital-to-analogue converter
(3) minimizes the effect of power supply interference.
(4) requires a very complex hardware
(5) is a tracking A/D converter. [1995]
(a) Both (i) and (ii) (b) Both (i) and (ii)
Solution: (c) Only (iii) (d) Only (ii) [1990]
(a) Flash → req. comparative and 0 coding circuit ⇒ Solution: (c)
(4) complex H/w
(b) Dual-slope: minimum effect of power supply (3) 3 bit R – 2R ladder – R22R are used general circuit:
(c) Successive approx: 2n turn req. DAC for A to D
conversion (2)
(a) – 4, (b) – 3, (c) – 2
9. (a) Successive approximation
(b) Dual-slope
(b) Parallel comparator i.e., with LSB, 2R is used (with gmd)
Maximum conversion time for 8 bit ADC in clock ⇒ (c)
cycles
(1) 1 (2) 2
(3) 16 (4) 256
(5) 512 [1994]
Solution: (a)
Successive approximation: Tmax = nTclk
n = number of bits = 8 Tclk
(b) Dual-slope: 2nH T = 29 T
clk clk

(c) Parallel comparator = 572 Tclk


⇒ = 1Tclk.
Hence, the correct option is (a). Two-marks Questions
10. Which of the resistance networks of figure can be used 1. In an N bit flash ADC, the analog voltage is fed simul-
as 3 bit R-2R ladder DAC. Assume V0 corresponds to taneously to 2N – 1 comparators. The output of the
LSB. comparators is then encoded to a binary format using
(i) digital circuits. Assume that the analog voltage source
Vin (whose output is being converted to digital format)
has a source resistance of 75 Ω as shown in the cir-
cuit diagram below and the input capacitance of each
comparator is 8 pF. The input must settle to an accu-
1
racy of LSB even for a full scale input change for
2
proper conversion. Assume that the time taken by the
thermometer to binary encoder is negligible. [2016]
6.80 | Digital Circuits


Vref 255
+

Thermometer
– code Digital
to binary output
Vref 2 3. The stable reading of the LED displays is
+ conversion
(a) 06 (b) 07
Vin (c) 12 (d) 13 [2008]

+ Solution: (d)
– +
Vref 1

If the flash ADC has 8 bit resolution, which one of the


following alternatives is closest to the maximum sam-
pling rate?
(A) 1 megasamples per second
(B) 6 megasamples per second 3

(C) 64 megasamples per second VDAC = ∑2


n=0
n −1
bn
(D) 256 megasamples per second
Solution:  Given that accuracy= 0.5LSB for N bit flash b3 = MSB
counter even for Full scale voltage, then using Nyquist b0 = LSB Counter is cleared initially
theorem where Fs is greater and equal to 2 Fm. So for
*Counter will count, till clk (Vin AND comp.
Vin = 2N – 1,
output) is high, i.e.,
Hence, the correct option is (A).
VDAC < V+
2. Consider a four bit D to A converter. The analog value
(V − 1) (62)
corresponding to digital signals of values 0000 and
0001 are 0 V and 0.0625 V respectively. The analog b3 b2 b1 b0 VDAC
value (in Volts) corresponding to the digital signal 1111 0 0 0 0 0
is _____. [2015] 0 0 0 1 0.5
Solution:  4-bit A/D converter 0 0 1 0 1.5
For input 0000 → output Voltage = 0 V 0 0 1 1
For input 0001 → output Voltage = 0.0625 V 0 1 0 0
V0 = K(input in decimal) 1 1 0 1 6.5
1 1 1 0 7
0.0625 = K(1) ⇒ K = 0.0625
For input 1111 = (15)10 At this point, VDAC > V+ ⇒ counter stops
∴ Corresponding stable reading is 6.5 × tc
V0 = 0.0625 × 15 = 0.9375
= 6.5 × 2
Hence, the correct Answer is (0.93 to 0.94). = 13.
Common Data for Questions 1 and 2 Hence, the correct option is (d).
In the following circuit, the comparator output is logic 4. The magnitude of the error between VDAC and Vin at
'1' if V1 > V2 and is logic ‘0’ otherwise. The D/A conver- steady state in volts is
sion is done as per the relation (a) 0.2 (b) 0.3
(c) 0.5 (d) 1.0 [2008]
3
VDAC = ∑2
n=0
n −1
bn volts, where b3 (MSB), b2, b1 and b0 Solution: (b)
Now VDAC at stable state = 6.5
(LSB) are the counter outputs. and Vin = 6.2
The counter starts from the clear state ⇒ error = 6.5 – 6.2 = 0.3.
Hence, the correct option is (b).
Chapter 8  ADC and DAC  |  6.81

Statement for Linked Answer Questions 5 and 6: i 0.5


I= = = 6.25 m A
In the digital-to-analogue converter circuit shown in 2×2×2 8
.
the figure below, VR = 10 V and R = 10 kΩ.
Hence, the correct option is (b).
6. The voltage V0 is
(a) − 0.781 V (b) − 1.562 V
(c) − 3.125 V (d) − 6.250 V
[2007]
Solution: (c)
Inverting terminal:

5. The current I is
(a) 31.25 µA (b) 62.5 µA
(c) 125 µA (d) 250 µA [2007]
Solution: (b)
VR = 10V, R = 10 kΩ V+ =
0, V− = 0 i i 5i 5 × 0.5
Inverting = + = = mA
2 8 8 8
= 3.12.5 μA
Now V0 = −R × Iinv
= − 10k × 3.125 μA
= − 3.12.5 × 104 × 10−6
V0 = − 3.125 V.
Hence, the correct option is (c).
7. A 4-bit D/A converter is connected to a free-running
3-bit UP counter, as shown in the following figure.
Which of the following waveforms will be observed at
V0?

In the figure shown above, the ground has been shown


by the symbol
(a)

VR 10
⇒ i= = = 0.5 mA
2 R 2 × 10 k
(b)
6.82 | Digital Circuits

(c) resistances and the 5 V inputs have a tolerance of ±


10%. The specification (rounded to the nearest multiple
of 5%) for the tolerance of the DAC is
(a) ±35% (b) ±20%
(c) ±10% (d) ±5% [2003]

(d) Solution: (a)

[2006]
Solution: (b)

(4 bit DAC)
5V ilp
(0 → 0)
(1 → 5V)
AK Counter DAC R
v V V V4
V0 =  1 + 2 + 3 +
8 R 
θ2 θ1 θ1 D3 D2 D1 D0
 1R 2 R 4 R
1 0 0 0 0 0 0 0 0
 V V V 
2 0 0 1 0 0 0 1 1 = − V1 + 2 + 3 + 4 
 2 4 8 
3 0 1 0 0 0 1 0 2
4 0 1 1 0 0 1 1 3 5V input, R → 10% tolerance
5 1 0 0 1 0 0 0 8 ⇒ V0 = −5 [1 + 0.5 + 0.25 + 0.125] = −9.375
6 1 0 1 1 0 0 1 9 (V0 max due to tolerance)
7 1 1 0 1 0 1 0 10 Tolerance of DAC:
8 1 1 1 1 0 1 1 11 −110  5.5 5.5 5.5 
VD = ×  5.5 + + +
90  2 4 8 
Now due to comparative: V0 will rise till 4th pulse, by
on 5th pulse, V0 goes to 8 then again rises by 1 step. −11
= × 5.5 = − 12.604
  9
T = 34.44 Ω 350/0.
Hence, the correct option is (a).
9. For the 4 bit DAC shown in the figure, the output volt-
age V0 is
8. The circuit shown in the figure is a 4-bit DAC

(a) 10 V (b) 5 V
The input bits 0 and 1 are represented by 0 and 5 (c) 4 V (d) 8 V [2000]
V, respectively. The OP AMP is ideal, but all the
Chapter 8  ADC and DAC  |  6.83

Solution: (b) At non-inv terminal:

 7k   1 1  5
1 +  = 8 ⇒ V0 =  0 + + + 0  × 8 = × 8
 1k   8 2  8

V0 = 5V.
Hence, the correct option is (b).
Unit VII
Microprocessor

Chapter 1: Basics of 8085 7.3


Chapter 2: Instructions of 8085 Microprocessor 7.5
Chapter 3: Memory Interfacing 7.9
Chapter 4: Microprocessor 8085 Interfacing 7.12
Chapter 5: Microprocessor 8085 Interrupts 7.14
Chapter 6: Microprocessor 8085 Programming 7.15
EXAM ANALYSIS
Exam Year 92 93 94 95 96 97 98 99 00 01 02 03 04 05 06 07 08 09 10 11 12 13 14-1 14-2 14-3 14-4 15 16 17 18 19
Set 1 Set 2 Set 3 Set 1 Set 2 Set 3 Set 1 Set 2
1 Marks Ques. 2 2 - 2 2 3 2 - 2 - - - - - - - - 1 1 - - 1 - - - - - 2 - 2 3 2 2 3 3 3
2 Marks Ques. - - - - - - - - 1 - 1 1 4 2 1 3 1 - 1 1 - - - 1 - 1 1 2 1 1 1 3 4 2 2 -
5 Marks Ques. - 1 1 - 1 - - - - - 1 - - - - - - - - - - - - - - - - - - - - - - - - -
Total Marks 2 7 5 2 7 3 2 - 4 - 2 2 8 4 2 6 2 1 3 2 - 1 - 2 - 2 2 6 2 4 5 8 10 7 7 3
Chapter wise marks
distribution
Basics of 8085 - 1 - 1 - - 2 - - - - - - - - - - - - - - - - - - - - - - - -
Instructions of 8085 1 - 1 1 - - - 2 - - 2 6 4 - - 2 - 2 2 - - - - - - - 2 2 1 1 -
Memory Interfacing - - - - 1 - - - - - - - - - 2 - - 1 - - - 2 2 1 - - - 1 - - -
8085 Interfacing - - - - - - - - - - - 2 - - - - - - - - - - 1 - 1 - - - - - -
8085 Interrupts - - - - - 1 - 2 - - - - - - - - - 1 - - - - - - - - - - - - -
8085 Programming - - - 1 1 - - - - - - - 2 4 - - - - - 1 1 2 - 1 - 1 - 1 1 -
Chapter 1
Basics of 8085
(c)  make the processor wait during a power
One-mark Questions shutdown.
(d) interface slow peripherals to the processor. [1993]
1. An I/O processor controls the flow of information
between Solution: (d)
(a) cache memory and I/O devices In a microcomputer, wait states are used to
(b) main memory and I/O devices interface slow peripherals to the processor.
(c) two I/O devices Hence, the correct option is (d).
(d) cache and main memories [1998]
5. In register index addressing mode, the effective address
Solution: (b) is given by
An I/O processor controls the flow of information (a) the index register value.
between main memory and I/O devices. (b)  the sum of the index register value and the
Hence, the correct option is (b). operand.
(c) the operand.
2. An instruction used to set the carry flag in a
(d) the difference of the index register value and the
computer can be classified as
operand. [1988]
(a) data transfer (b) arithmetic
(c) logical (d) program control Solution: (a)
 [1998] In register index addressing mode, the effective address
Solution: (c) is given by the register index value.
An instruction used to set the carry flag in a Hence, the correct option is (a).
computer can be classified as logical instruction.
Hence, the correct option is (c).
Two-mark Questions
3. An ‘assembler’ for a microprocessor is used for
(a) assembly of processors in a production line. 1. In an 8085 system, a PUSH operation requires more
(b)  creation of new program using different clock cycles than a POP operation. Which one of the
modules. following options is the correct reason for this?[2016]
(c)  translation of a program from assembly (A) For POP, the data transceivers remain in the same
language to machine language. direction as for instruction fetch (­memory to pro-
(d)  translation of a higher level language into cessor), whereas for PUSH their direction has to
English text. [1995] be reversed.
Solution: (c) (B) Memory write operations are slower than memory
An assembler for a microprocessor is used for the read operations in an 8085 based system.
translation of a program from assembly language to (C) The stack pointer needs to be pre decremented be-
machine language. fore writing registers in a PUSH, where as a POP
operation uses the address already in the stack
Hence, the correct option is (c).
pointer.
4. In a microcomputer, wait states are used to (D) Order of registers has to be interchanged for a
(a)  make the processor wait during a DMA PUSH operation, whereas POP uses their natural
operation. order.
(b) make the processor wait during an interrupt pro-
cessing.
7.4 | Microprocessor

Solution: 
For push operation o­ peration), resulting in (4 + 2 + 3 + 3 = 12 clock) while
in POP, operations are fetch, decoding and read. But
T1 T2 T3
here fetch + decode takes only 4 clocks resulting in (4 +
6 clock cycles 3 clock cycles 3 clock cycles 3 + 3 = 10 clock) so Push takes more clock cycles to
Fetch and execute the POP.
Write Write
Decode Hence, the correct option is (C).
2. A 16 Kb (= 16,384 bit) memory array is designed as
Total 6 + 3 + 3 = 12 clk cycles.
a square with an aspect ratio of one (number of rows
For pop operation is equal to the number of columns). The minimum
number of address lines needed for the row decoder is
T1 T2 T3 ______. [2015]
4 clock cycles 3 clock cycles 3 clock cycles Solution:  16K bit memory array is to be designed with
Fetch and an aspect ratio of one
read read
Decode 16 K bit = 24 ⋅ 210 = 214 = 27 ⋅ 27 = 27× 128
So no. of address lines = 7,
For a PUSH operation, more cycles are required
So no. of outputs of decoder = 27 = 128
because after fetching and decoding (4), data will
be written onto the stack with 3 clock each (of write no. of data lines = 128
Hence, the correct Answer is (7).
Chapter 2
Instructions of 8085
Microprocessor
3. When a CPU is interrupted, it
One-mark Questions (a) stops execution of instructions.
(b)  acknowledges interrupt and branches to a
1. In an 8085 microprocessor, the contents of the accu- sub-routine.
mulator and the carry flag are A7 (in hex) are 0, respec- (c) acknowledges interrupt and continues.
tively. If the instruction RLC is executed, then the (d) acknowledges interrupt and waits for the next in-
contents of the accumulator (in hex) and the carry flag, struction from the interrupting device. [1995]
respectively, will be [2016]
(A) 4E and 0 (B) 4E and 1 Solution:  (d)
(C) 4F and 0 (D) 4F and 1 CPU here is a microprocessor. The options are con-
Solution:  Accumulator = A7H, CY = 0 cerned with INTR only as remaining interrupts do not
require interrupt acknowledgment.
CY
Hence, the correct option is (d).
0 1 CY
4. In a microprocessor, the register that holds the address
of the next instruction to be fetched is
RLC (a) accumulator
10100 111 01001111
(b) program counter
After RLC (circular left shift without carry), (c) stack pointer
Accumulator = 4FH (d) instruction register [1993]
Solution: (b)
CY flag =1
Program counter stores the address of the next instruc-
Hence, the correct option is (D). tion to be executed.
2. The total number of memory accesses involved (inclu- Hence, the correct option is (b).
sive of the op-code fetch) when an 8085 processor exe- 5. In a microprocessor system, the stack is a used for:
cutes the instruction LDA 2003 is (a) storing the program return address whenever a sub-
(a) 1 (b) 2 routine jump instruction is executed
(c) 3 (d) 4 [1996] (b) transmitting and receiving input–output data.
Solution:  (d) (c) storing all important CPU register contents when-
Instruction LDA 2003 requires four machine cycles. ever an interrupt is to be serviced.
Machine cycle 1 → op-code fetch (LDA) (d)  storing program instructions for interrupt
service routines. [1989]
Machine cycle 2 → memory read (03)
Solution:  (a)
Machine cycle 3 → memory read (20)
Stack is a temporary set of memory locations in the
Machine cycle 4 → memory read [(2003)] to main memory. These memory locations are used to
accumulator store the binary information temporarily during the
Hence, the correct option is (d). execution of a program. Whenever a sub-routine jump
7.6 | Microprocessors

instruction (CALL) is executed, the stack is used for A = 00001110 = OE H


storing the program return address. MOV B, A B = OEH
Hence, the correct option is (a).
RLC  00001110   = 00011100
Two-marks Questions RLC  00011100   = 00111000
1. In an 8085 microprocessor, the shift register which A = 38H
stores the result of an addition and the overflow bit are,
ADD B → A + B
respectively [2015]
(A) B and F (B) A and F 00111000
(C) H and F (D) A and C 00001110
Solution:  Addition will be performed with respect to 01000110
accumulator, and result will be stored in accumulator. RRC → Rotate acc right 1 bit
ADD B,this instruction will add register B with accu- without carry
mulator and store result in accumulator.
As per the result, the flags will be affected in flag regis- A =  01000110   = 00100011
ter. So overflow bit is stored in flag register.    A = 23H
Hence, the correct option is (B). Hence, the correct option is (c).
2. In an 8085 microprocessor, which one of the following 4. For the 8085 assembly language program given below,
instructions changes the content of the accumulator? the content of the accumulator after the execution of
 [2015] the program is
(A) MOV B, M
300 MVIA, 45
(B) PCHL
3002 MOV B, A
(C) RNZ
3003 STC
(D) SBI BEH
3004 CMC
Solution:  In 8085 Microprocessor 3005 RAR
MOV B, M → move contents of Memory location to B 3006 XRA B
PCHL → copy the contents of HL to PC.
RNZ → return to main program on No Zero result (a) 00H (b) 45H
(c) 67H (d) E7H [2010]
SBI BEH → Subtract BEH from Accumulator, and store
result in Accumulator. Solution:  (c)
Hence, the correct option is (D). MVI A, 45  → A ← 45H
3. An 8085 assembly language program is given below. MOV  B, A → B ← 45H
Assume that the carry flag is initially unset. The con- TC → CY = 1
tent of the accumulator after the execution of the pro- CMC → CY = 0
gram is
RAR → Rotate accumulate 1 bit right
MVI A.07H through carry
RLC
carry
MO B, A
V 0   0100 010 1
RLC
RLC
ADD B A = 001000101
RRC XRA  B → A Ex or with B
(00100010) ⊕ (01000101)
(a) 8CH (b) 64H   = 01100111
(c) 23H (d) 15H [2011] A = 67H
Solution: (c) Hence, the correct option is (c).
MVI A, 07 H → A = 07 5. An 8085 executes the following instructions
RLC → Rotate accumulator 1 bit left 2710 LXI H, 30A0H
without carry
2713 DAD H
C
00000111 2714PCHL
Chapter 2  Instructions of 8085 Microprocessor  |  7.7

All addresses and constants are in Hex. Let PC be the 0106 MVI A, 20H
contents of the program counter and HL be the con- → 0107 → 20H
tents of the HL register pair just after executing PCHL. 0108 SUB M → A ←A – M
Which of the following statements is correct? 0109
(a) PC = 2715H HL = 30A0H M contains HL pair address data
(b) PC = 30A0H HL = 2715H H1 → 0107
(c) PC = 6140H HL = 6140H At HL 0107, operand 20H is stored thus
(d) PC = 6140H HL = 2715H [2008] A ← 20H − 20H
Solution:  (c) A ← 00H
LXI H, 30 40H → HL = 30A 0H Hence, the correct option is (c).
DAD H → HL ← HL + HL 8. It is desired to multiply the numbers OAH by OBH and
store the result in the accumulator. The numbers are
30 A0 available in registers B and C, respectively. A part of
30 A0 the 8085 program for this purpose is given below:
MVI A, 00H
  61 40H
Loop:-----------
PCHL → PC ← HL
-----------
Thus, PC = 6140 H   HL = 6140H
-----------
Hence, the correct option is (c).
HLTEND
6. If in addition following code exists from 0109H
onwards, The sequence of instructions to complete the program
would be
ORI 40 H
(a) JNZ LOOP, ADD B, DCRC
ADDM, (b) ADD B, JNZ LOOP, DCR C
what will be the result in the accumulator after the last (c) DCR C, JNZ LOOP, ADD B
instruction is executed? (d) ADD B, DCR C, JNZ LOOP [2004]
(a) 40H (b) 20H Solution:  (d)
(c) 60H (d) 42H [2005]
ADD B, DCR C, JNZ LOOP
Solution:  (c)
MVIA, 00 A = 00
At 0109, A → 00H
LOOP ADD A=A+B
    ORI 40H → A  ← 01000000
B
00000000 DCRC C=C-1
A  ← 01000000 (40H) Loop will till C = 0.
JNZLOOP So, loop will execute
    Add M
11 times (OB).
    M → HL → 20H
    A ← A + M ← 20 + 40 Common data for Questions 10 and 11.Consider an
8085 microprocessor system
    A ← 60H
Hence, the correct option is (d).
Hence, the correct option is (c).
9. Consider the sequence of 8085 instructions given
7. The following program starts at location 0100H.
below.
LXISP, 00FF
LXI H, 9258H, MOV A, M,
LXIH, 0107
Which one of the following is performed by this
MVI A, 20H sequence?
SUB M (a) Contents of location 9258 are moved to the accu-
The content of accumulator when the program counter mulator.
reaches 0109H is (b) Contents of location 9258 are compared with the
(a) 20H (b) 02H contents of the accumulator.
(c) 00H (d) FFH [2005] (c) Contents of location 9258 are complemented and
stored in location 9258.
Solution:  (c)
(d) Contents of location 5892 are complemented and
0100 LXI SP, 00FF
stored in location 589 [2004]
0103 LXI H, 0107
7.8 | Microprocessors

Solution:  (c) (a) carry flag will be set but zero flag will be reset.
LXI H, 9258 → HL ← 9258H (b) carry flag will be reset but zero flag will be set
(c) both carry flag and zero flag will be reset.
MOV A, M → A ← contents of add. 9258
(d) both carry flag and zero flag will be set. [2003]
CMA → complement accumulator
Solution:  (a)
MOV M → complement of A is stored in M
CMP B → contents of B and A are compared and the
(9258 H)
result is indicated by flag.
Hence, the correct option is (c).
A < B. ∴ CY = 1, Z = 0.
10. The number of memory cycles required to execute the
Hence, the correct option is (a).
following 8085 instructions:
(I) LDA 3000H 12. The contents of register (B) and accumulator (A) of
(II) LXI D, F0F1H 8085 microprocessor are 49 H and 3 AH, respectively.
The contents of A and the status of carry flag (CY) and
would be:
sign flag (S) after executing SUB B instructions are
(a) 2 for (I) and 2 for (II)
(a) A = F1, CY =1, S = 1
(b) 4 for (I) and 3 for (II)
(b) A = 0F, CY =1, S = 1
(c) 3 for (I) and 3 for (II)
(c) A = F0, CY = 0, S = 0
(d) 3 for (I) and 4 for (II) [2004]
(d) A = 1F, CY = 1, S = 1 [2000]
Solution: (b)
Solution:  (a)
Memory cycles
A → 3 AH → 00111010
LDA3000H → Fetch, Re ad , Re ad , Re ad
address data B → 49H → 01001001
LXI D, F0F1H → Fetch, Read, Read SUBA → 11110001
Hence, the correct option is (b). CY = 1, S = 1, A = F1
1 1. In an 8085 microprocessor, the instruction CMP B has Hence, the correct option is (a).
been executed while the content of the accumulator is
less than that of register B. As a result
Chapter 3
Memory Interfacing
(The address lines are designated as A0 to A7 for
One-mark Questions IO addresses.) The peripheral will correspond to IO
addresses in the range:
1. In the circuit shown below, the device connected to Y5
A2
can have address in the range
A3
A8 A
A9 B A4
A10 C Y5
To device A5 CHIP SELECT
chip select
A6
A7
74LS138
3-to-8 decoder (a) 60 H to 63H (b) A4H to A7H
A11 G2A
(c) 50 H to AFH (d) 70H to 73H
A12  [1997]
A13 G2B
A14 Solution:  (a)
A15
For active low chip signal, NAND gate output should
be ‘0’.
IO/M G1
NAND gate output will be ‘0’, when all the inputs are ‘1’.
So, possible combinations given in options are
Vcc GND
A7 A6 A5 A4 A3 A2 A1 A0
(a) 2000 - 20 FF (b) 2D00 - 2DFF 0 1 1 0 0 0 0 0 60 H to
(c) 2E00 - 2EFF (d) FD00 - FDFF 0 1 1 0 0 0 1 1 63 H
 [2010] 1 0 1 0 0 1 0 0 A4 H to
Solution:  (b) 1 0 1 0 0 1 1 1 A7 H
To connect Y5, input CBA should be 101. Possible 0 1 0 1 0 0 0 0 50 H to
range will be 1 0 1 0 1 1 1 1 AF H
0 1 1 1 0 0 0 0 70 H to
A15 A14 A13 A12 A11 A10 A9 A8 A7 A0
0 1 1 1 0 0 1 1 73 H
0 0 .1 0 1 1 0 1 0 ... 0
0 0 1 0 1 1 0 1 1 1 A2 and A3 should be the same. So, only options (a), (c)
and (d) satisfy this.
Therefore, the device can have address in the range Option (c) cannot be correct, because all chip select
2000 - 2DFF. signals are not the same.
Hence, the correct option is (b). Option (d) cannot be correct because it gives logic ‘1’
2. The decoding circuit shown in the below-given figure output. So, the correct option is (a).
has been used to generate the active low chip select sig- Hence, the correct option is (a).
nal for a microprocessor peripheral. 3. In an 8085 microprocessor system with memory
mapped I/O,
7.10 | Microprocessor

(a) I/O devices have 16-bit addresses. ∴ Range is F000H – F3FFH.


(b) I/O devices are accessed using IN and OUT in- Since A12 A11 A10 → 100 such that output 4 is selected.
structions. Hence, the correct option is (c).
(c) there can be a maximum of 256 input devices and
256 output devices. 5. A microprocessor with a 16-bit address bus is used in
(d) arithmetic and logic operations can be directly per- a linear memory selection configuration (i.e., address
formed with the I/O data. [1992] bus lines are directly used as chip selects of memory
Solution:  (a) and (d) chips) with 4 memory chips. The maximum address-
able memory space is
In memory mapped I/O, the I/O devices are also
treated as memory locations. Under assumption, they (a) 64 k (b) 16 k
will be given 16-bit address. Microprocessor uses (c) 8 k (d) 4 k [1988]
related instructions to communicate with I/O devices. Solution:  (b)
Arithmetic and logic operations can be directly per- Among 16 address lines, as there are 4 chips, A15 A14 A13
formed with the I/O data.
A12 are used as chip select lines. So, remaining are 12.
Hence, the correct option is (a and d).
∴212 = 4 kB; for each chip 4 × 4 kB = 16 kB.
4. An 8-bit microprocessor has 16-bit address bus A0 -
A15. The processor has a 1 kB byte memory chip as Hence, the correct option is (b).
shown. The address range for the chip is
A15 0
A0 - A9 Two-marks Questions
A14 CS 1
A13 2 1k × 8 1. An 8k byte ROM with an active low Chip Select input

A12
3
4
RAM
CS
8 ( )
CS is to be used in an 8085 microprocessor based
A11 5 Data system. The ROM should occupy the address range
6 **** 1000H to 2FFFH. The address lines are designated as
A10 7
A15 to A0, where A15 is the most significant address bit.
(a) F00FH TO F40EH Which one of the following logic expressions will gen-
(b) F100 TO F4FFH erate the correct CS signal for this ROM?
(c) F000H TO F3FFH  [2016]
(d) F700H TO FAFFH [1988]
(A)
A15 + A14 + (A13 A12 + A13 A12 )
Solution:  (c)
(B)
A15 A14 (A13 + A12)
1 kB → A9 → A0; 210 = 1 kB = 1024 B
A15 A14 A13 A12 A11 A10 A9 A8 A7 A6 A5 A4 A3 A2 A1 A0 (C) A15 A14 (A13 A12 + A13 A12 )
1 1 1 1 0 0 0 0 0 0 0 0 0 0 0 0
F 0 0 0 (D)
A15 + A14 + A13 A12
1 1 1 1 0 0 1 1 1 1 1 1 1 1 1 1 Solution:  For this ROM to generate the CS signal, the
F 3 F F
input at CS pin must be 0. The capacity of ROM is

8k Byte ROM = 8k × 8 = 213 × 8

13 Address lines and 8 data lines.


A15 A14 A13 A12 A11 A10 A9 A8 A7 A6 – – – A0
0 0 0 1 0 0 0 0 0 0 – – – 0 ⇒ 1000H
0 0 1 0 1 1 1 1 1 1 – – – 1 ⇒ 2FFFH

( )
By using higher order bits CS chip select input has 2. An 8255 chip is interfaced to an 8085 microprocessor
system as an I/O mapped I/O as shown in the figure.
to be connected.
For above range CS = 0 = A15 + A14 + (A13 A12 + The address lines A0 and A1 of the 8085 are used by
the 8255 chip to decode internally its three ports and
A13 . A12 ) the control register. The address lines A3 to A7 as well
Hence, the correct option is (A). as the IO/•M signal are used for address decoding. The
range of addresses for which the 8255 chip would get
selected is
Chapter 3  Memory Interfacing  |  7.11

2. Consider the decoder circuit shown in figure for pro-


A7 8255 viding chip select signals to an EPROM, a RAM and
A6 an I/O chip with four addressable registers from a de
A5 multiplexed 8085 address bus.
CS
A4 (i) Specify all the memory address ranges to which the
A1 A1 EPROM will respond.
A3
A0 A0 (ii) Specify all the memory address ranges to which the
IO/M
RAM will respond.
(a) F8H–FBH (b) F8H–FCH
(c) F8H–FFH (d) F0H–F7H
 [2007]
Solution:  (c)
O/P of NAND gate is 0 if A7 to A3 & IO /•M = i
A7 A6 A5 A3 A2 A1 A0
Starting 1 1 1 1 1 0 0 0 → F8H
address
Final 1 1 1 1 1 1 1 1 → FFH
address

Hence, the correct option is (c).

Five-marks Questions
1. A microprocessor has five address lines [A0- A4] and
eight data lines [D0 - D7]. An input device A, an output
device B, a ROM and a RAM are memory mapped to
the microprocessor at the addresses as shown in figure.
Devices A and B have four addressable registers each;
RAM has 8 bytes and ROM has 16 bytes. (a) Indicate
the address lines to be connected to each device and Solution:
memory. A15 A15 A A A A A A A A A A A A A A A A
Hexadecimal =0 =1 15 14 13 12 11 10 9 8 7 6 5 4 3 2 1 0
addresses
0000 8000 X X 0 0 0 0 0 0 0 0 0 0 0 0 0 0
1F
B H H
1C to to X X 0 1 1 1 1 1 1 1 1 1 1 1 1 1
ROM 1B 1FFF 9FFF
0B H H
RAM 04 4000 C000 X 1 0 0 0 0 0 0 0 0 0 0 0 0 0 0
H H
A 03 to to X 1 0 1 1 1 1 1 1 1 1 1 1 1 1 1
00 5FFF DFFF
H H
[1993]
Solution: EPROM address range: 0000H to 1FFFH
Device A has 2 add line required. Device B requires 2 or
address lines. ROM requires 3 add lines. RAM require 8000H to 9FFFH
4 address lines
SRAM address range: 4000H to 5FFFH
or
C000H to DFFFH
Chapter 4
Microprocessor 8085
Interfacing
Solution: (d)
One-mark Question To transfer, the data should be present in accumulator
1. For a microprocessor system using I/O-mapped I/O, the and proper instruction should be executed.
following statement(s) is not true. • For the proper working and gates 1 and 2 should pro-
(a) Memory space available is greater. duce output as 1.
(b) Not all data transfer instructions are available. 3 to 8 7
Input Device
(c) I/O and memory address spaces are distinct. Decoder 6
A2 C 5
(d) I/O address space is greater. [1988] 4 Digital Dt0 - DO7 Data Bus
A1 B 3 (D0 - D7 )
Solution: (d) inputs
2
A0 A 1
Number of I/O devices = 256 0
G2A G2B G1 DS1 DS2
Memory capacity = 64 kB
IO/M
Hence, the correct option is (d). RD
A3 A8
A9
A
Two-marks Questions A5 4
A
1
A10
A11
A12 2
A7 6 A13
A14
1. For the 8085 microprocessor, the interfacing circuit A15
to input 8-bit digital data (DI0 - DI7) from an external
device is shown in the figure. The instruction for cor- • Since DS1 is connected to output 0 of decoder, so
rect data transfer is input selection should be 000. Therefore, address
lines have inputs as
3 to 8 7
I/D Device
Decoder 6 A15 A14 A13 A12 A11 A10 A9 A8 A7 A6 A5
A2 C 5
4 Digital Dt0 - DI7 Dt0 - DO7 Data Bus A4 A3 A2 A1 A0
A1 B 3 (D0 - D7 )
inputs 1 1 1 1 1 0 0 0 1 1 1
2
A0 A 1
0 1 1 0 0 0
G2A G2BG1 DS1 DS2
IO/M That is, accumulator should be fed with the contents of
RD address F8F8H, i.e., LDA F8F8H.
A3 A8
A
A9 Hence, the correct option is (d).
A5 4 A10
A11 2. The 8255 programmable peripheral interface is used as
A6 A12
A7 A13 described below.
A14 (I) An A/D converter is interfaced to a microprocessor
A15
through an 8255. The conversion is initiated by a
(a) MVI A, F8H (b) IN F8H signal from the 8255 on Port C. A signal on Port C
(c) OUTF8H (d) LDAF8F8H causes data to be strorbed into Port A.
 [2014]
Chapter 4  Microprocessor 8085 Interfacing  |  7.13

(II) Two computers exchange data using a pair of 2000 START LXI SP.1000H
8255s. Port A works as a bidirectional data port LXI H, 2F37H
supported by appropriate handshaking signals.
XRA A
The appropriate modes of operation of the 8255 for (I)
MOV A, H
and (II) would be
(a) Mode 0 for (I) and Mode 1 for (II) INX H
(b) Mode 1 for (I) and Mode 0 for (II) PUSH H
(c) Mode 2 for (I) and Mode 0 for (II) CZ 20FFH i
(d) Mode 2 for (I) and Mode 1 for (II) [2004]
JMP 3000H
Solution: (c)
HLT
Options are incorrect since port A can be operated as
20FF ADD H
bidirectional port only in mode-2. (b) can be correct if
it is mode 1 for (I) and mode 2 for (II). RZ
Hence, the correct option is (c). POP B
PUSHB
Five-marks Questions RM
3000 HLT [1991]
1. Write down the sequence of Instructions which are Solution:
actually (till a HLT instruction), if the program begins 2000:LXI SP1, 1000H → SP ← 1000H
with the location 1FF5H.
2003:LXI H, 2F37H → HL ← 2F37H
Address (HEX) 8085 Instructions
2006:XRA A → A ←00H, Z21
1FF5 XRA A
2007:MOV A, H → A ← 2F
1FF6 LXI H.2000H
2008:IMX H → HL ← 2F38H
1FF9 PCHL
2009:Push H → SP ← OFFE
1FFA HLT
200A:CZ20FFH → As Z = 1, SP ← 0FFC
1FFB LXI H, 2001H
→ PC ← 20FF
1FFE ANI00H
due to call on zero
2000 LXI H, FFFFH
20FF:ADD, H → A + M → A → 2F + 2F
2003 INXH →5
2004 JZ2100H 2100:RZ → Z = 0
2007 HLT 2101:POP B → SP ← 0FFE and BC ← 200D
2100 LXI H, 1 FFFH 2103:RM → If S = 1 in previous addition
2103 MOV A, M 3000:HLT → But S = 0, so condition false
2104 INR A After execution
2105 HLT [1994] PC = 3000
2. The program given below is run on an 8085/based SP = 0FFE
microcomputer system. Determine the contents of the
B = 20
registers:
C = OD
PC, SP, B, C, H, L after a halt instruction is executed.
H = 2F
LOC
L = 38
Chapter 5
Microprocessor 8085
Interrupts
3. In the 8085 microprocessor, the RST6 instruction trans-
One-mark Questions fers the program execution to the following location:
(a) 30H (b) 24H
1. In a microprocessor, the service routine for a cer- (c) 48H (d) 60H [2000]
tain interrupt starts from a fixed location of memory Solution:  (a)
which cannot be externally set, but the interrupt can be
Hexadecimal of (6 × 8 = 48) is 30H.
delayed or rejected. Such an interrupt is
(a) non-maskable and non-vectored Hence, the correct option is (a).
(b) maskable and non-vectored 4. In an 8085 μP system, the RST instruction will cause
(c) non-maskable and vectored an interrupt
(d) maskable and vectored [2009] (a) only if an interrupt service routine is not being ex-
Solution:  (d) ecuted
(b) only if a bit in the interrupt mask is made 0
Interrupt which has a fixed address location is said to
(c) only if interrupts have been enabled by an El in-
be vectored and which can be delayed or rejected is
struction
known as maskable.
(d) None of the above [1997]
Hence, the correct option is (d).
Solution:  (c)
2. The number of hardware interrupts (which require an
RST instruction will cause an interrupt only if inter-
external signal to interrupt) present in an 8085 micro-
rupts have been enabled by an El instruction at the
processor is
beginning of program.
(a) 1 (b) 4
(c) 5 (d) 13 [2000] Hence, the correct option is (c).
Solution:  (c)
1. TRAP 2. RST 7.5
3. RST 6.5 4. RST 5.5
5. INTR
Hence, the correct option is (c).
Chapter 6
Microprocessor 8085
Programming
MVI B, 05H → B = 05H
One-mark Questions PTR: ADD B →A ←A+B
DCR B
1. The clock frequency of an 8085 microprocessor is
5 MHz. If the time required to execute an instruction is JNZ PTR Loop will execute 5 times till value of B
1.4 µs, then the number of T-states needed for execut- becomes 0
ing the instruction is [2017] A = A + B = 05 + 05 + 04 + 03 + 02 + 01 + 00
(A) 1 (B) 6   In loop due to DCR B value of B is decreased
(C) 7 (D) 8 A = 14 H
1 A01  03H A = A + 03
fmp = 5 MHz T =
Solution:  = 0.2 µsec ⋅ A = 17H.
fµ p
Hence, the correct option is (a).
1T → 0.2 msec 3. The following instructions have been executed by an
to complete an instruction it took 1.4 msec, therefore by 8085 μP:
linear relation ADDRESS INSTRUCTION
Texecution=1.4 m (HEX)
No. of T states = T/Texecution = 1.4/0.2 = 7 6010 LXI H, 8A79H
46013 MOVA,L
Hence, the correct option is (C).
6014 ADDH
2. For 8085 microprocessor, the following program is
executed. 6015 DAA
MVI A, 05H; 6016 MOVH,A
MVI B, 05H; 6017 PCHL
PTR: ADD B; From which address will the next instruction be
fetched?
DCR B;
(a) 6019
JNZ PTR; (b) 0379
ADI 03H; (c) 6979
HLT. (d) None of the above [1997]
At the end of program, accumulator contains Solution: (c)
(a) 17H (b) 20H 6010  LXI H, 8A79H →  HL ← 8A79H
(c) 23H (d) 05H [2013] 6013  MOV A, L →  A = 79
Solution: (a) 6014  ADD H →  A ← A + H A←
MVI A, 05H → A = 05H 8A + 79
7.16 | Microprocessors

10001010 At the completion of execution of the program, the


program counter of the 8085 contains______, and the
01111001 stack pointer contains_______. [1992]
1>00000011 Solution: (200A H, 0FFE H)
A = 03H  CY = 1 AC = 1 LXI  SP, 1000 → SP  ← 1000
6015  DAA  Decimal adjust after accumulator PUSH H SP ← SP-2 SP  ←  0FFEH
Since CY = 1, AC = 1 (push HL pair into stack)
A = 03 + 66 = 69H PUSH D SP ← SP-2 SP  ←  0FFCH
(push DE pair into stack)
6016  MOV  H, A → H = 69H
CALL 2050H SP ← SP-2 SP  ←  0FFAH
6017  PCHL →E  xchange contents of PC & (push PC into stack)
HL
After returning from sub-routine
  HL = 6979H
SP ← SP + 2     SP ← 0FFC
Thus, PC becomes 6979H.
POP H        SP ← SP + 2  SP ← 0FFE H
Thus, next instruction would be fetched from 6979H.
PC would contain next address of HLT
Hence, the correct option is (c).
HLT takes 1 byte; thus PC = 200A H
4. The following sequence of instructions is executed by
an 8085 microprocessor:    SP = 0FFE H
1000 LXI SP, 27FF
1000 CALL 1006 Two-marks Questions
1006 POPH
1. The following FIVE instructions were executed on an
The contents of the stack pointer (SP) and the HL reg- 8085 microprocessor.
ister pair on completion of execution of these instruc- MVI A, 33 H
tions are: MVI B, 78 H
(a) SP = 27 FF, HL = 1003 ADD B
(b) SP = 27 FD, HL = 1003 CMA
(c) SP = 27 FF, HL = 1006 ANI 32 H
(d) SP = 27 FD, HL = 1006 [1996]
The accumulator value immediately after the execution
Solution: (c) of the fifth instruction is  [2017]
LXI SP, 27 FF → SP ← 27FF H (A) 00 H (B) 10 H
1003 CALL 1006 → SP → SP-2 SP ← 27FC H (C) 11 H (D) 32 H
Solution:  Given instructions
PC gets pushed onto stack
MVI A, 33H
PC = 1006 MVI B, 78H
1006 POP H SP ← SP + 2 SP ← 27FF H ADD B
1006 gets pepped into HL CMA
ANI 32H
Thus HL = 1006H
A = 33H
SP = 27FFH + B = 78H ADD B
Hence, the correct option is (c). A = ABH ⇒ A = A + B
5. The following program is run on an 8085 A = 10101011
microprocessor: Complement A since CMA
Then A = 01010100.
Memory Address is in HEX Instruction
AND A with 32 H, i.e., 00110010
2000 LXI SP, 1000 A : 01010100
2003 PUSH H 32 H : 00110010
2004 PUSH D And Operation A = 00010000
A = 00010000
2005 CALL 2050
\ A = 10 H
2008 POP H
Hence, the correct option is (B).
2009 HLT
Chapter 6  Microprocessor 8085 Programming  |  7.17

2. Which one of the following 8085 microprocessor pro- Thus starting from 1FFEH,  A15 – A8  A7 – A0
grams correctly calculates the product of two 8-bit for fetch 1F FE
numbers stored in registers B and C? [2015]
for first memory read 1F FF
(A) MVI A, 00H
for second memory read 20 00
JNZ LOOP
CMP C for storing accumulator 12 34
LOOP DCR B Thus values of A15 – A8
HLT 1FH, 1FH, 20H, 12H
Hence, the correct option is (a).
(B) MVI A, 00H
CMP C 4. After execution of line 7 of the program, the status of
LOOP DCR B the CY and Z flags will be
JNZ LOOP (a) CY = 0, Z = 0 (b) CY = 0, Z = 1
HLT (c) CY = 1, Z = 0 (d) CY = 1, Z = 1
(C) MVI A, 00H  [2007]
LOOP ADD C Solution: (b)
DCR B ANI  9BH  → A = A.9B
JNZ LOOP = (11101010). (10011011)
HLT
A = 10001010
(D) MVI A, 00H
ADC C A = 8AH
JNZ LOOP CPI  9FH  →  8A < 9FH Accumulator compare
LOOP INR B immediate
HLT CY = 1, Z = 0
Solution:  We have to multiply register B and C, we can STA  3010  H  → Store accumulator to 3010 H
do it By adding B register C number of times, or adding
thus CY = 1, Z = 0 after line 7.
Register C, B number of times.
Hence, the correct option is (b).
Choice (C) MVI A, 00H → A = 00H reset Accumulator
Common Data for Questions 3 and 4
Loop: ADD C → Add C to Accumulator
DCR B → decrement B An 8085 assembly language program is given below.
 JNZ → till B becomes zero Add C Line1:MVI A, B5H
 HLT 2 : MVIB, OEH
Hence, the correct option is (C). 3 : XRI69H
4 : ADDB
3. An 8085 microprocessor executes `STA 1234H‘ with 5 : ANI9BH
starting address location 1FFEH (STA copies the con-
6 : CPI9FH
tents of the accumulator to the 16-bit address loca-
tion). While the instruction is fetched and executed, the 7 : STA3010H
sequence of values written at the address pins A15 – A8 8 : HLT
is: 5. The contents of the accumulator just after
(a) 1FH, 1FH.20H, 12H execution of the ADD instruction in line 4 will be
(b) 1FH.FEH, 1FH.FFH, 12H (a) C3H (b) EAH
(c) 1FH.1FH.12H.12H (c) DCH (d) 69H [2007]
(d) 1FH, 1FH, 12H.20H.12H [2014]
Solution: (b)
Solution: (a)
MVI  A, B5 H → A = 10110101
IFFE  STA 1234
MVI  B, OE H → B = 00001110
1FFEH → STA OP code stored
XRI  69 H → A = A ⊕ 69
1FFFH → ‘12’ operand stored
  A = 10110101 ⊕ 01101001
2000H → ‘34’ operand stored
  A = 11011100
STA 1234 does storing value of accumulator at address
ADD  B →A=A+B
1234 H
7.18 | Microprocessors

11011100 OUT PORT 1


00001110 HLT
NEXT: XRA B
A = 11101010

JP START
After ADD, A = EA H.
OUT PORT 2
Hence, the correct option is (b).
HLT
6. Following is the segment of an 8085 assembly language
The execution of the above program in an 8085 micro-
program:
processor will result in
LXI SP, EFFFH CALL3000H 3000 H: LXIH.3CF4H (a) an output of 87H at PORT1
PUSH PSW SPHL POP PSW RET (b) an output of 87H at PORT2
On completion of RET execution, the contents of SP is (c) infinite looping of the program execution with ac-
(a) 3CFOH (b) 3CF8H cumulator data remaining at 00H.
(c) EFFDH (d) EFFFH [2006] (d) infinite looping of the program execution with ac-
Solution: (b) cumulator data alternating between 00H and 87H
 [2002]
LXI SP, EFFF H → SP ← EFFF H
Solution: (b)
CALL  3000H   → SP ← SP-2
SP  =  EFFD H MVI  B, 87 H → B = 87H
3000H: LXI   H, 3 CFH H  → HL MOV  A, B  → A = 87H
– 3CFH  H START: JMP NEXT
 PUSH PSW    → SP NEXT: XRA B →A=A⊕B
← SP-2 SP = EFFBH = (10000111) ⊕ (10000111)
SPHL → SP ← 3CFH A = 00H
POP  PSW → SP ← SP + 2  SP = 3CF6 Sign bit = 0
RET → Pop + Jump  SP ← SP + 2 ∴Control goes back to start (JP conditional Jump only
SP = 3CF8H when sign bit = 0)
Hence, the correct option is (b). from start goes back to NEXT
7. Consider the following assembly language program. XRAB  → A ⊕ B = (00000000) ⊕ (10000111)
MVI B, 87H A = 37H
MOV A, B Sign bit = 1
START: JMP NEXT ∴ This time, it would not jump to start.
MVI B, 00H Out Port 2 → output of A = 87 H on port 2.
XRA B Hence, the correct option is (b).
Unit VIII
Communication

Chapter 1: Analog Communication Systems 8.3


Chapter 2: Random Signals and Noise 8.25
Chapter 3: Digital Communication Systems 8.46
Chapter 4: Information Theory 8.73
EXAM ANALYSIS
Exam Year 92 93 94 95 96 97 98 99 00 01 02 03 04 05 06 07 08 09 10 11 12 13 14-A 14-B 14-C 14-D 15 16 17 18 19
Set 1 Set 2 Set 3 Set 1 Set 2 Set 3 Set 1 Set 2
1 Marks Ques. - - 4 8 1 2 10 3 1 2 3 3 5 3 - 3 1 3 3 2 4 1 3 3 4 3 - 1 2 4 2 3 3 2 3 2
2 Marks Ques. 5 2 1 - 3 1 - 4 4 4 4 14 10 7 14 11 11 6 4 4 3 5 4 4 4 4 6 4 4 3 5 2 3 2 3 3
5 Marks Ques. - - - - - - - - - 1
Total Marks 2 1 - 3 1 1 2 5 5 3 3 7 4 6 3 1 4 2 4 3 2 - - - - - 12 9 10 10 12 7 9 6 9 8

Chapter wise marks


distribution
Analog Communication - 2 3 3 2 - 2 3 4 1 3 6 5 4 6 2 3 2 2 2 1 - - 3 3 1 7 4 4 2 1
Systems
Random Signals and 1 - - 1 1 1 4 - - 2 1 4 3 4 4 3 3 4 3 1 2 2 3 3 3 5 3 2 1 1 2
Noise
Digital Communication 3 - 2 4 1 2 4 4 1 3 3 7 7 2 3 9 5 2 2 3 3 3 2 - 2 1 6 9 4 2 1
Systems
Information Theory - 1 - - - - - - - - - - - - 1 - 1 1 - - 1 1 2 1 - - 1 3 1 1 -
Chapter 1
Analog Communication
Systems
One-mark Questions ∴ f max = 50 k +
10Π 2

 dm (t )
= 60 k 

= 2
max
2Π 1 ms  dt 
1. The baseband signal m(t) shown in the figure is phase-
modulated to generate the PM signal f (t) = cos(2p fct + 10Π  dm (t ) 
t min = 50 k − ⋅1 × 103 = 45 k  max = −1
km(t)). The time t on the x-axis in the figure is in mil- 2Π  dt 
liseconds. If the carrier frequency is fc = 50 kHz and t max 60 k
k = 10p, then the ratio of the minimum instantaneous = = 0.75
frequency (in kHz) is to the maximum instantaneous f min 45 k
frequency (in kHz) is _____ (rounded off to 2 decimal 2. Consider the following amplitude modulated signal:
places). [2019] S(t) = cos(2000 p t) + 4 cos(2400 p t) + cos(2800 p t).
The ratio (accurate to three decimal places) of the
m(t)
1 power of the massage signal to the power of the carrier
signal is _______. [2018]
0 1 2 3 4 5 6 7 8 9 Solution:  amplitude modulated signal is given as
t(in ms)
S(t) = cos(2000 p t) + 4 cos(2400 p t) + cos(2800 p t)
–1
μ AC
SAM (t ) = cos[ 2π ( f c − f m )t ] + AC cos( 2π f C t )
2
Solution: f (t ) = cos ( 2Πf c t + km (t )) μ AC
+ cos[ 2π ( f C − f m )t ]
qi (t ) = 2Πf c t + km (t ) 2
Ac μ
1 d S(t) = Ac ⋅ cos2p fct + ⋅ cosp (fc + fm)t
fi = qi (t ) 2
2Π dt Ac μ
1 d + ⋅ cos 2p (fc – fm)t
= qi (t ) 2
2Π dt
1 d Pt = Pc + PSB
=  2Πf c t + km (t ) 
2Π dt  From the given data
1 d Ac μ
= 2Πf c + k  m (t ) Ac = 4 and = 1
2Π  dt 2
k d
= fc + m (t ) PC =
AC
= 8
2Π dt 2
f c = 50 KHz ⎧ Ac μ ⎫ ⎛ Ac μ ⎞
2 2

k = 10Π
PSB = 2 ⎨ ⎬ =⎜ ⎟
⎩2 2 ⎭ ⎝ 2 ⎠ 
=
(1) = 1
8.4 | Communication

PSB 1 NOT gate and resistance is present; therefore, there


= = 0.125
PC 8 will be a switching threshold.

Hence, the correct answer is 0.12 to 0.13. Hence, the correct option is (C).
3. The block diagram of a frequency synthesizer con- 5. A superheterodyne receiver operates in the frequency
sisting of a phase locked loop (PLL) and divided by range of 58 MHz–68 MHz. The intermediate frequency
N counter (comprising ÷2, ÷4, ÷8, ÷16 outputs) is fIF and local oscillator frequency fLO are chosen such
sketched below. the synthesizer is excited with a 5 kHz that fIF ≤ fLO. It is required that the image frequencies
signal (input 1). The free running frequency of the PLL fall outside the 58 MHz–68 MHz band. The minimum
is set to 20 kHz. Assume that the commutator switch required fIF (in MHz) is _____. [2016]
makes contacts repeatedly in the order 1 – 2 – 3 – 4. Solution:  Frequency range of receiver is 58 MHz to 68
 [2016] MHz.
The corresponding frequencies synthesized are fLo = fs + IF
(A) 10 kHz, 20 kHz, 40 kHz, 80 kHz
(B) 20 kHz, 30 kHz, 80 kHz, 160 kHz fsi = fLo + IF
(C) 80 kHz, 40 kHz, 20 kHz, 10 kHz
fsi = fs + 2IF.
(D) 160 kHz, 80 kHz, 40 kHz, 20 kHz
Solution:  Running freq is set to be 20 kHz. Here freq As per question fsi should be outside the range.
divider is used with N counter. O/p freq becomes N fsi ≥ 68 MHz.
times the given freq so the outputs are 2 × 5 = 10 Hz,
4 × 5 = 20 Hz, 8 × 5 = 40 Hz, 16 × 5 = 80 Hz fs + 2IF ≥ 68 MHz.
Hence, the correct option is (A).
Assume
4. What is the voltage V out in the following circuit?
fs = 59 MHz.
 [2016]
+VCC 2IF ≥ 68 – 58

IF ≥ 5 MHz.
Input 1 Phase Low pass
detector filter Hence, the correct Answer is (5).
Amplifier 6. The amplitude of a sinusoidal carrier is modulated by a
single sinusoid to obtain the amplitude modulated sig-
VCO nal S(t) = 5 cos 1600πt + 20cos1800πt + 5cos2000πt.
Counter
The value of the modulation index is _____. [2016]
1 ÷2
2 ÷4 C Solution:  AM
–VCC
3 ÷8 R S(t) = Accos2pfct [1 + ka m(t)]
4 ÷16
S(t) = Accos2pfct [1 + ka Am cos2πfmt]
Synthesizer
output Ac μ
S(t) = Accos2pfct + cos2π( fc + fm) t
(A) 0 V 2
(B) (VT of PMOS + VT of NMOS)/2 Ac μ
+ cos2π ( fc – fm) t
(C) Switching threshold of inverter 2
(D) VDD
Comparing the above standard equation with given
Solution:  The given circuit is inverter connected from equation, we get
output to input.
fc = 900 Hz,
Vout
fm = 100 Hz
Ac = 20
Ac μ
= 5
An inverter is the circuit which provides the comple- 2
mented output. Here an inverter circuit consisting of
Acμ = 10
Chapter 1  Analog Communication Systems | 8.5

10 10 3 −1
μ= = µ=
Ac 20  3 +1

1 2
μ = = 0.5 . µ= = 0.5
2 4

Hence, the correct Answer is (0.5).
10. A modulated signals is y(t ) = m(t )cos(4000π t ) ,
7. For a superheterodyne receiver, the intermediate fre- where the baseband signal m(t) has frequency compo-
quency is 15 MHz and the local oscillator frequency nents less than 5 kHz only. The minimum required rate
is 3.5 GHz. If the frequency of the received signal is (in kHz) at which y(t) should be sampled to recover
greater than the local oscillator frequency, then the m(t) is _______. [2014]
image frequency (in MHz) is _____ .  [2016]
Solution:
Solution:  Intermediate frequency freq (fIF)= 15MHz ;
The minimum sampling rate is given by Nyquist rate
local oscillator frequency ( fL0) = 3.5 GHz = 3500 MHZ
fs = 2 fm
As we know that
fs = 2 × 5
( fs – fL0 = fIF) fs = 10 kHz
fs = fL0 + fIF 11. The phase response of a pass band waveform at the
receiver is given by φ (f ) = −2π a (f − f c ) − 2pβ f
= 3500 MHz + 15 MHz where fc is the centre frequency, and a and b are posi-
fs = 3515 MHz tive constants. The actual signal propagation delay
from the transmittance to receiver is
and
fsi = fs – 2fIF a−β ab
(a) (b)
a+β a+b
Up conversion (frequency of received signal is greater
than LO frequency) (c) a (d) b [2014]
Solution: (c)
fsi = 3515 – 2 × 15
Group delay is given by
= 3485 MHz
= −1 dφ = −1 [−2πα ]
Hence, the correct Answer is (3485 MHz). 2π df 2π
8. Consider the signal s(t) = m(t) cos (2π fct) + m̂ (t)
Group delay = α
sin (2π.fc.t) where m̂ (t) denotes the Hilbert transform
of m(t) and the bandwidth of m(t) is very small com- Hence, the correct option is (c).
pared to fc. The signal s(t) is a [2015] 1 2.
Consider an FM signal
(A) high-pass signal f (t ) = cos  2π f c t + β1sin2π f1t + β 2 2π f 2 t 
(B) low-pass signal

The maximum deviation of the instantaneous frequency
(C) band-pass signal
from the carrier frequency fc is
(D) double sideband suppressed carrier signal β1 f 2 + β 2 f1
(a) β1 f1 + β 2 f 2 (b)
Solution:  Signal s(t) = m(t) cos(2π fct) + m̂ (t) sin(2π fct) (c) β1 + β 2 (d) f1 + f 2  [2014]
This is the equation of SSB-SC so it will look like Solution: (a)
band-pass signal. Maximum frequency deviation is
Hence, the correct option is (C). 1 dφ
9. Consider sinusoidal modulation in an AM system. = .
2π dt
max
Assuming no overmodulation, the modulation index  
(H) when the maximum and minimum values of the ϕ = Phase deviation
envelope, respectively, are 3V and 1V, is ______. 1
 [2014] = {β1 2π f1 cos 2π f1t + β 2 2π f 2 cos 2π f 2 t}max

Solution:
Modulation index for AM modulated wave is given by 1
= ( 2πβ1 f1 + 2πβ 2 f 2 )
Amax − Amin 2π
µ=
Amax + Amin = β1f1 + β1f2

Hence, the correct option is (a).
8.6 | Communication

13. In a double side-band (DSB) full carrier AM transmis- 14. The List-I (lists the attributes) and the List-II (list of the
sion system, if the modulation index is doubled, then modulation systems) are given. Match the attribute to
the ratio of total sideband power to the carrier power the modulation system that best meets it.
increases by a factor of _____. [2014] List-I
Solution: A. Power efficient transmission of signals
Ratio of PSB to PC B. Most bandwidth efficient transmission of voice
g (t ) = e − at e + j ( wct +∆w )t signals
+
C. Simplest receiver structure
 PSB  µ 2T D. Bandwidth efficient transmission of signals with
  =
P 1 2
 significant dc component
List-II
 PSB  µ02
and  P  = 1. Conventional AM
 C 2 2
2. FM
where μ2 = 2μ
3. VSB
 PSB 
  1 4. SSB-SC
PC
Hence,  = A B C D
 PSB  4 (a) 4 2 1 3
 PC  (b) 2 4 1 3
 
(c) 3 2 1 4
Four times.
(d) 2 4 3 1 [2011]
Therefore,
Solution:(b)
AM has simple receiver and VSB is bandwidth efficient
transmission of signals with significant dc component.
SSB – SC is most bandwidth efficient transmission of
voice signal.
Hence, the correct option is (b).
The Fourier transform of output y(t) of the high pass
filter centroid at fc is 15. Suppose that the modulating signal is
m(t ) = 2cos (2π f m t ) and the carrier is
xC (t ) = AC cos (2π f c t ) . Which one of the following is
a conventional AM signal without over-modulation?
(a) x(t ) = Ac m(t ) cos (2π f c t )

Now, the output of second multiplier is y(t).w(t) = z(t) x(t ) = Ac 1 + m(t )  cos (2π f c t )
(b)
AC
x(t ) = AC cos (2π f c t )+
(c) m(t )cos (2π f c t )
4
x(t ) = AC cos (2π f m t )cos (2π f c t )
(d)  [2010]
+ AC sin (2π f m t )sin (2π f c t )

After passing through the low pass filter centred at fc the
Solution:(c)
Fourier transform of output S(t) is
Conventional AM signal is
x(t) = AC[1 + m(t)] cos (wct)
= AC cos (wct) + ACm(t) cos wct
In option b,
2
Modulation index = = 2.
1
So, it is conventional A.M signal but with over
Thus, bandwidth = (A – B) – 0
modulation.
= 100 – 40
In option c,
= 60 Hz
Chapter 1  Analog Communication Systems | 8.7

AC m(t ) Therefore, minimum value of AC should be 2.


x(t ) = AC cos( 2π f c t ) + cos( 2π f c t )
4 Hence, the correct option is (a).
Here, modulation index 19. Find the correct match between Group-1 and
2 1 Group-2.
= = Group-1
4 2
P. {1 + km (t)}A sin(ωct)
Therefore, it is conventional AM signal without over Q. km(t) A sin(ω t)
c
modulation. R. A sin {ωc t + km(t )}
Hence, the correct option is (c).  t 
S. A sin ω t ∫
+ k m(t )dt
16. Consider an angle modulated signal x(t ) = 6 cos  2π × 10 t + 2 sin(8000πt ) + 4 cos(8000π t ) V
6 c 
 −∞ 
x(t ) = 6 cos  2π × 106 t + 2 sin(8000π t ) + 4 cos(8000π t )  V Group-2
  .
W. Phase modulation
The average power of x(t) is X. Frequency modulation
(a) 10 W (b) 18 W Y. Amplitude modulation
(c) 20 W (d) 28 W [2010] Z. DSB-SC modulation
Solution:(b) (a) P-Z, Q-Y, R-X, S-W
2 (b) P-W, Q-X, R-Y, S-Z
Average power of angle modulated signal x(t) is AC
(c) P-X, Q-W, R-Z, S-Y
2 (d) P-Y, Q-Z, R-W, S-X [2005]
Here AC = 6 Solution: (d)
62
Therefore, average power = = 18W
2 Correct combination will be as followed
Hence, the correct option is (b). P. {1 + km (t)}A sin(ωct) Amplitude
modulation
17. For a message signal m(t ) = cos (2π f m t ) and carrier of Q. km(t) A sin(ω t) DSB-SC modulation
c
frequency fc, which of the following represents a single R. A sin {ω ct + km(t )} Phase modulation
side-band (SSB) signal?
(a) cos(2p f m t ) cos(2p f c t )  t 

(b) cos (2π f c t )


S. 
 ∫
A sin ωc t + k m(t )dt  Frequency

 −∞ 
  modulation
(c) co s  2π ( f c + f m )t 
(d) 1+ cos (2π f m t ) cos (2π f c t )  [2009] Hence, the correct option is (d).
20. Which of the following analogue modulation scheme
Solution:(c) requires the minimum transmitted power and minimum
cos (2π (fc + fm) (t) represents only USB of A.M – SSB/ channel band-width?
SC signal. (a) VSB (b) DSB-SC
Hence, the correct option is (c) (c) SSB (d) AM [2005]
18. Consider the amplitude modulated (AM) signal Solution: (c)
Ac coswc t + 2coswm t coswc t . For demodulating the Transmission single side band modulated signal
signal using envelope detector, the minimum value of requires same bandwidth as original message and takes
Ac should be minimum power for transmission as carrier is sup-
(a) 2 (b) 1 pressed here.
(c) 0.5 (d 0 [2008] Hence, the correct option is (c)
Solution: (a) 21. An AM signal is detected using an envelope detector.
Modulated signal The carrier frequency and modulating signal frequency
φ AM (t ) = AC cos ωc t + 2 cos ωm t + cos ωc t are 1 MHz and 2 kHz respectively. An appropriate
= (AC + 2 cos wmt) cos wct value for the time constant of the envelope detector is
(a) 500 μ sec (b) 20 μ sec
Condition for envelope detection of an Aμ signal is (c) 0.2 μ sec (d) 1 μ sec [2004]
= A + 2 cos wmt ≥ 0 Solution:(b)
AC – 2 ≥ 0 Fc = carrier frequency
AC ≥ 2 Fm = message signal frequency
8.8 | Communication

For envelop detector time RC constant is given by Frequencies Present = 1000 ± 10 (fc ± fm)
1 1 = 970,1030.
< RC ≤
fc fm Half wave symmetric wave . So, only odd harmonics
are present.
1 μ sec <RC ≤ 0.5 ms
Hence, the correct option is (c).
Hence, the correct option is (b).
22. An AM signal and a narrow-band FM signal with iden- 25. A band limited signal is sampled at the Nyquist rate.
tical carriers, modulating signals and modulation indi- The signal can be recovered by passing the samples
ces of 0.1 are added together. The resultant signal can through
be closely approximated by (a) an RC filter
(a) broadband FM (b) an envelope detector
(b) SSB with carrier (c) an PLL
(c) DSB-SC (d) an ideal low-pass filter with the appropriate band-
(d) SSB without carrier [2004] width [2001]
Solution: (b) Solution: (d)
Standard equation for narrow band FM is given by Ideal low pass filter with the appropriate BW can
Aβ A β recover a sampled band limited signal.
S (t ) = AC cos wc t + c cos( wc + wm )t − C cos 9wc − w m ) Hence, the correct option is (d).
2 2
Ac β A β 26. The amplitude modulated wave form s(t) = Ac[1 +
) = AC cos wc t + cos( wc + wm )t − C cos 9wc − wm )
2 2 Kam(t)] cosωct is fed to an ideal envelope detector. The
maximum magnitude of Ka m(t) is greater than 1.
AC ma
S (t ) AM = AC cos wc t + Which of the following could be the detector output?
2 2
(a) Acm(t) (b) Ac2 1+K a m (t )
AC ma (c) [Ac| 1 + Kam(t)|] (d) Ac|1 + Kam(t)|2
cos( wc + wm )t + cos(ωc − ωm )
2  [2000]
S(t) + S(t)AM = carrier + USB = SSB with carrier. Solution:(c)
Note: S(t) + S(t)AM = SSB + SC When the modulation index of AM wave is less than
Hence, the correct option is (b). unity the output of the envelope detector is envelope of
the AM wave but when the modulation index is greater
23. The input to a coherent detector is DSB–SC signal plus than unity then the output of the envelope detector is
noise. The noise at the detector output is not envelope but mode of the envelope of the AM wave.
(a) the in-phase component Thus, the detector output in given case would be AC[1 +
(b) the quadrature-component kam(t)].
(c) zero
(d) the envelope [2003] Hence, the correct option is (c).
Solution: (a) 27. The input to a channel is a bandpass signal. It is
The coherent detector rejects the quadrature compo- obtained by linearly modulating a sinusoidal carrier
nent of noise and therefore noise at output has in phase with a single-tone signal. The output of the channel due
component only. The in-phase component of noise and to this input is given by
output are additive at output of detector. y(t) = (1/100)cos(100t – 10-6)cos(106t – 1.56).
Hence, the correct option is (a). The group delay (tg) and the phase delay (tp) in seconds
24. A 1 MHz sinusoidal carrier is amplitude modulated by of the channel are
a symmetrical square wave of period 100 μ sec. Which (a) tg = 10-6, tp = 1.56
of the following frequencies will NOT be present in the (b) tg = 1.56, tp = 10-6
modulated signal? (c) tg = 10-8, tp = 1.56 × 10-6
(a) 990 kHz (b) 1010 kHz (d) tg = 108, tp = 1.56 [1999]
(c) 1020 kHz (d) 1030 kHz [2002] Solution: (c)
Solution: (c) Given
Given  1  −6 6
y (t ) =   cos (100t − 10 ) cos(10 t − 1.56)
1 MHz ⇒ 1000 kHz = fc (carrier frequency)  100 

100 μs ⇒10 kHz = fm (message signal frequency)
Chapter 1  Analog Communication Systems | 8.9

Now comparing with 31. A DSB-SC signal is generated using the carrier cos(ωct
y(t) = cos (t – t0) cos [wc(t – tp)] + θ) and modulating signal x(t). The envelope of the
DSB-SC signal is
tp = 1.56 × 10−6 Hz
(a) x(t)
tg = 10−8 Hz (b) |x(t)|
Hence, the correct option is (c). (c) only positive portion of x(t)
28. A modulated signal is given by s(t) = m1(t)cos (2πfct) (d) x(t) cosθ [1998]
+ m2(t) sin(2πfct) where the baseband signal m1(t) and Solution: (b)
m2(t) have bandwidths of 10 kHz and 15 kHz, respec- The 'envelope' of the DSB-SC (double side band–sup-
tively. The bandwidth of the modulated signal, in kHz, press carrier) signal depends on the |x(t)| and not on
is x(t).
(a) 10 (b) 15
Hence, the correct option is (b).
(c) 25 (d) 30 [1999]
32. The image channel rejection in a superheterodyne
Solution: (b)
receiver comes from
Given modulation technique is DSB-SC with fmax=15 (a) IF stages only
KHz, so bandwidth can be given as (b) RF stages only
Bandwidth = 2*fmax = 30 kHz (c) detector and RF stages only
Hence, the correct option is (b). (d) detector, RF and IF stages [1996]
29. A modulated signal is given by s(t) = e-atcos[(ωc + ∆ω) Solution: (b)
t]u(t), where a, ωc and ∆ω are positive constants, and Image rejection should be achieved in IF amplifier. It
ωc >> ∆ω. The complex envelope of s(t) is given by becomes impossible to remove it from wanted signal.
(a) exp (−at)exp[j(ωc + ∆ω)t]u(t) So, the image channel rejection in a super heterodyne
(b) exp (−at)exp(j∆ωt)u(t) receiver comes from RF stages only.
(c) exp (j∆ωt).u(t) Hence, the correct option is (b).
(d) exp [j(ωc + ∆ω)t] [1999]
33. The image (second) channel selectivity of a superhetero-
Solution:(d) dyne communication receiver is determined by
Complex envelope (a) antenna and preselector
g (t ) = g+ (t ) − jwct (b) the preselector and RF amplifier
(c) the preselector and IF amplifier
where g + (t) is pre envelope given as
(d) the RF and IF amplifier [1995]
g+ (t ) = g (t ) + jg (t ) Solution: (b)
− at
and g (t ) = e sin[( wc + ∆ w )t ] The image rejection should be achieved IF stage
g+ (t ) = e − at e + j ( wct +∆w )t because once it enters into IF amplifier it becomes
− at − jw t impossible to remove it from wanted signal.
Hence, g (t ) = e e c u(t )
So, the image channel selectivity depends upon pre-
g (t ) = e − at e − j ∆wt u(t ) selective and RF amplifiers only. If amplifier helps in

rejection of adjacent channel frequency and not image
Hence, the correct option is (d). frequency.
3 0. The image channel selectivity of superheterodyne Hence, the correct option is (b).
receiver depends upon
34. A PLL can be used to demodulate
(a) IF amplifier only
(a) PAM signals (b) PCM signals
(b) RF and IF amplifiers only
(c) FM signals (d) DSB–SC signals
(c) Preselector, RF and IF amplifier
 [1995]
(d) Preselector and RF amplifiers only [1998]
Solution: (c)
Solution: (d)
PLL (Phase Locked Loop) is used to demodulate the
The image rejection should be achieved before IF stage
FM signals.
because once it enters into IF amplifier it becomes
impossible to remove it from wanted signals. So, image Hence, the correct option is (c).
channel selectivity depends upon pre-selector and RF 35. A PAM signal can be detected by using
amplifiers only. The IF amplifier helps in rejection of (a) an ADC (b) an integrator
adjacent channel frequency and not image frequency. (c) a band pas filter (d) a high pass filter
Hence, the correct option is (d). [1995]
8.10 | Communication

Solution: (b) After BPF,


An integrator circuit is used in PAM receiver circuit. y(t) = a Ac cos 2πfct + 2b cos (2πfct) cos (2π fmt)
Hence, the correct option is (b).
3 6. v(t) = 5[cos(106πt) – sin(103πt) × sin(106πt)] represents 2b
=
a Ac [1 + cos (2πfmt)] cos (2π fct)
(a) DSB suppressed carrier signal a
(b) AM signal 2b
(c) SSB upper sideband signal μ=
a 
(d) Narrow band FM signal [1994]
Solution: (d) μ2 P
PSB = P = C
2 c 2 
Carrier and upper side band are in phase and lower side
band is out of phase with carrier and upper side-band. μ2 = 1, μ = 1
5 5
v(t ) = 5 cos(105 π t ) − cos(106 − 103 )π t + cos(106 + 103 ) 2b a
2 2 =1⇒ = 2
a b
5 5
v(t ) = 5 cos(105 π t ) − cos(106 − 103 )π t + cos(106 + 103 ) Hence, the correct option is (D).
2 2
So given signal is narrow band FM signal. 2. The input 4 sinC (2t) is fed to a Hilbert transformer to
obtain y (t), as shown in the figure below:
Hence, the correct option is (d).
37. A 10 MHz carrier is frequency modulated by a sinusoi- Hilbert
4sinc(2t) y(t)
dal signal of 500 Hz, the maximum frequency devia- transform
tion being 50 kHz. The bandwidth required, as given by
the Carson rule is_________ [1994] sin(π x )
Here, sinc(x) = . The value (accurate to two
Solution: πx
∆ f = frequency deviation ∞ 2
fm = message signal frequency ­decimal places) of ∫ y(t ) dt is ________.[2018]
−∞
Bandwidth is calculated by with Carson’s rule
BW = 2(∆f + fm) 4 ⎡f⎤
Solution:  4 sin c( 2t ) ⎯FT⎯→ rect ⎢ ⎥
BW = 2(50 + 0.5) 2 ⎣2⎦
BW = 101 kHz ∞
2
∫ X ( f ) df = 2( 22 ) = 8
Two-marks Questions −∞


1. Let c(t) = Ac cos ( 2π f ct ) and m(t) = cos ( 2π f mt ) . It is 2
given that f c  5 f m . The signal c(t) + m(t) is applied ∫ y(t ) dt = 8
−∞
to the input of a non-linear device, whose output no(t)
is related to the input ni(t) as no(t) = ani(t) + bno2(t), X(f )

where a and b are positive constants. The output of the 2


non-linear device is passed through an ideal ban-pass
filter with centre frequency fc and bandwidth 3fm­, to pro-
duce an amplitude modulated (AM) wave. It is desired
f(Hz)
to have the sideband power of the AM wave to be half –1 0 1
of the carrier power, then a/b is: [2018]
(A) 0.25 (B) 0.5
(C) 1 (D) 2 Hence, the correct answer is 8 to 8.
Solution:  Vi(t) = Ac cos(2 p fct) + cos(2 p fmt) 3. The unmodulated carrier power in an AM transmitter is
V0(t) = aVi(t) + b vi2 (t) 5 kW. This carrier is modulated by a sinusoidal modu-
lating signal. The maximum percentage of modula-
a Ac cos(2 p fct) + a cos(2 p fmt)
= tion is 50%. If it is reduced to 40% then the maximum
+ b[ Ac2 cos2 (2π fct) + cos2 (2π fmt) unmodulated carrier (in kW) that can be used with-
out overloading the transmitter is _______________.
+ 2Ac cos2π ( fct) cos (2π fmt)  [2017]
Chapter 1  Analog Communication Systems | 8.11

Solution:  Solution:  Y(t) = x (t) – x(t – T0)


Pc = 5 kW for µ max = 0.5 The autocorrelation of output y can be given as
 (0.5) 2  Ry(τ) = E [y(t) y(t + τ)]
Pt ( max ) = Pc 1 + 
2  = E [[x(t) – x (t – T0)] [x (t +τ) – x(t – T0 + τ]

= 5.625 kW Ry(τ) = E[x(t) x(t + τ) – x(t) x(t –T0 +τ)
µ = 0.4 – x(t – T0) x(t + τ)

 µ2  Ry(τ) = [Rx(τ) – Rx(τ – T0) – Rx(τ + T0) + Rx(τ)


Pc ( max ) = 1 + 
 2 
= Pt ( max ) Ry(τ) = 2Rx(τ) – Rx(τ – T0) – Rx(τ + T0)

⇒ Pc ( max ) = 5.208 kW Hence, the correct option is (B).


6. The complex envelope of the band pass signal
Hence, the correct answer is (5.1 to 5.2).
⎛ sin (π t 5) ⎞ ⎛ π⎞
4. A modulating signal given by x(t) = 5 sin (4p103t – 10 p x(t) =− 2 ⎜ ⎟ sin ⎜ π t − ⎟ , cantered about
cos 2 p 103 t) V is fed to a phase modulator with phase ⎝ πt 5 ⎠ ⎝ 4⎠
deviation constant kp = 5 rad/V. If the carrier frequency 1
is 20 kHz, the instantaneous frequency (in kHz) at t = f = Hz, is [2015]
0.5 ms is ___________. [2017] 2
Solution:  x(t) = 5 sin (4000pt – 10p cos 200pt) V ⎛ sin (π t 5) ⎞ j π4
(A) ⎜ e
PM signal, ⎝ π t 5 ⎟⎠
s(t) = Ac cos [ωct + Kp × (t)] ⎛ sin (π t 5) ⎞ − j π4
Instantaneous angle, (B)
⎜⎝ π t 5 ⎟⎠ e
q (t) = ωct + Kp × (t)
Instantaneous frequency is ⎛ sin (π t 5) ⎞ j π4
(C) 2 ⎜ e
d ⎝ π t 5 ⎟⎠
ωi(t) = θ (t )
dt ⎛ sin (π t 5) ⎞ − j π4
(D) 2 ⎜ e
d ⎝ π t 5 ⎟⎠
= wc + k p x (t )
dt
Solution: 
Band pass signal x(t)
ωi (t )
fi(t ) = ⎡ sin (π t 5) ⎤ ⎛ π⎞
θπ =− 2⎢ ⎥ Sin ⎜ π t − ⎟ can be as
⎣ (π t 5) ⎦ ⎝ 4⎠
At t = 0.5 ms
fi (t) = 70 kHz ⎡ sin (π t 5) ⎤ ⎡ π π ⎤
x(t) = − 2 ⎢ ⎥ ⎢sin π t cos − sin ⋅ cos π t ⎥
Hence, the correct answer is (70). ⎣ (π t 5) ⎦ ⎣ 4 4 ⎦
5. A wide sense stationary random process X(t) passes sin (π t 5)
= [cos π t − sin π t ]
through the LTI system shown in the figure. If the auto-
(π t 5)
correlation function of X(t) is Rx(τ), then the autocor-
relation function Ry(τ) of the output Y(t) is equal to π π 1
∵sin = cos =
 [2016] 4 4 2

+ So x(t) = Xc(t) cos 2 π fet – Xc(t) sin 2 π fct


X(t) Y(t)
– (low pass representation of Band pass single or L-SSB
representation)
sin (π t 5)
Delay = T0
Where Xc(t) = Xs(t) =
(A) 2Rx(τ) + Rx(τ – T0) + Rx (τ + T0) (π t 5) 
(B) 2Rx(τ) – Rx (τ – T0) – Rx(τ + T0) So complex envelope
(C) 2Rx(τ) + 2Rx(τ – 2T0) Xce(t) = Xc(t) + j Xs(t)
(D) 2Rx(τ) – 2Rx(τ – 2T0)
8.12 | Communication

sin (π t 5) Solution: Φpm(t) = A cos [ ω ct + kp f(t)]


= [1 + j ]
(π t 5) 
Where
sin (π t 5) j π4 Φpm(t) = Phase modulated signal
= 2 e
(π t 5)  m(t) or f (t) = message signal
Hence, the correct option is (C). θi(t) = ω ct + kp m(t)
7. A random binary wave y (t) is given by

here kp = 1
y (t) = ∑ X n p ( t − nT − ϕ )
So θi(t) = ω ct + m(t)
n =−∞

where p(t) = u(t) – u(t – T), u(t) is the unit step function Bandwidth = 2(∆ ω + ω m ) = 2(∆f + fm)π
and φ is an independent random variable with uniform
d θ i (t )
distribution in [0, T]. The sequence {Xn} consists of ω i (t) = = ω c + Am 2π f m cos ( 2π f m t )
dt
independent and identically distributed binary valued 
random variables with P{Xn = +1} = P{Xn = –1} = 0.5 ∆ ω = Am fm. 2π cos 2 π fmt
for each n.
The value of autocorrelation [2015] So bandwidth depends upon Am and fm.
Hence, the correct option is (C).
⎛ 3T ⎞ Δ ⎡ ⎛ 3T ⎞ ⎤
Ryy ⎜ ⎟ = E ⎢ y (t ) y ⎜ t − ⎟ ⎥ equals _______ 9. The directivity of an antenna array can be increased by
⎝ 4⎠ ⎣ ⎝ 4 ⎠⎦
adding more antenna elements, as a large number of
+∞ elements [2015]
Solution:  y(t) = ∑ X n p ( t − nT − ϕ ) (A) improves the radiation efficiency.
n=−∞ (B) increases the effective area of the antenna.
If p(t) = u(t) – u(t – T) (C) results in a better impedance matching.
(D) allows more power to be transmitted by the anten-
So amplitude (A) of p(t) = 1 na.
and we know that

⎧ 2⎛ τ ⎞ Solution:  D= Ae
⎪ A 1− ⎟⎠ , τ <T λ2
Autocorrelation RXX(τ) = ⎨ ⎜⎝ T
⎪ D ∝ Ae
⎩ 0, τ >0

So directivity increases as the effective area of antenna
⎛ 3T ⎞ increases.
⎛ 3T ⎞ 4⎟
So for Ryy ⎜ ⎟ = A2 ⎜1 − Hence, the correct option is (B).
⎝ 4⎠ ⎜⎝ T ⎟⎠
 1 0. In the system shown in Figure (a), m(t) is a low-pass
⎛ 3⎞ signal with bandwidth W Hz. The frequency response
= 1. ⎜ 1 − ⎟ of the band-pass filter H( f ) is shown in Figure (b). If it
⎝ 4⎠
 is desired that the output signal z(t) = 10x(t), the maxi-
Since A = 1 mum value of W (in Hz) should be strictly less than
1 _______. [2015]
= = 0.25
4 x(t) = m(t) y(t) = 10x (t)
cos(2400π t) + x2(t) H(f) z(t)
Hence, the correct Answer is (0.24 to 0.26). Amplifier Band-pass
filter
8. A message signal m(t) = Am sin (2πfct) is used to modu- (a)
late the phase of a carrier Ac cos(2πfct) to get the modu- H(f )
lated signal y(t) = Ac cos(2πfct + m(t)). The bandwidth
of y(t) [2015]
1
(A) depends on Am but not on fm.
(B) depends on fm but not Am.
(C) depends on both Am and fm. –1700 –700 0 700 1700 f(Hz)
(D) does not depend on Am or fm. (b)
Chapter 1  Analog Communication Systems | 8.13

Solution:  From the figure. f = 3 MHz


x(t) have frequency spectrum 3 ×108
⇒ λ= =100 m
X(f ) 3 ×106

maximum field occurs at Ψ = 0
βd cos θ + f = 0
2π π
× d cos 60° − = 0
100 2 
1 2π π
–1200–W –1200 –1200–W 1200–W 1200 1200+W f ⇒ d × × =+ 
2 100 2
d = 50 m
Hence, the correct Answer is (49 to 51).
12. In the figure, M(f) is the Fourier transform of the mes-
sage signal m(t) where A = 100 Hz and B = 40 Hz. Given
v(t ) = cos (2π f c t ) and w (t ) = cos (2π ( f + A)t ) ,
c
1200–W 1200 1200+W 2400+2W 2400 2400+2W f
where fc > A. The cut-off frequencies of both the filters
Now for Y( f ) one sided spectrum will be are fc.
Because x2(t) contains twice frequency components of
x(t) = 2 × (1200 ± W)
=
(2400 ± W)
And z(t) = 10 x(t)

So Band pass filter should completely reject the x2(t)


spectrum.
So 2400 – 2W = 1700
W = 350 Hz
The bandwidth of the signal at the output of the modu-
Hence, the correct Answer is (349 to 350). lator (in Hz) is _____. [2014]
1 1. Two half-wave dipole antennas placed as shown in the Solution:
figure are excited with sinusoidally varying currents of
frequency 3 MHz and phase shift of π/2 between them
(the element at the origin leads in phase). If the maxi-
mum radiated E-field at the point P in the x-y plane
occurs at an azimuthal angle of 60o, the distance d (in
meters) between the antennas is _______ [2015]
z
After passing through first multiplier, the output is
m(t) × v(t) = x(t)
1 3. The signal m(t) as shown is applied both to a phase
O
modulator (with kp as the phase constant) and a fre-
d y quency modulator with (kf as the frequency constant)
60° OP >> d
having the same carrier frequency

x P

Solution:  Ψ = βd cos θ + f
π
Given that f=–
2
θ = 60°
8.14 | Communication

The ratio kp/kf (in rad/Hz) for the same maximum 15. A message signal given by
phase deviation is
1 1
(a) 8π (b) 4π m(t) =   cos ω1t −   s inω2 t is amplitude-modu-
(c) 2π (d) π [2012] 2 2
Solution: (b) lated with a carrier of frequency wc to generate
For phase modulator s(t ) = 1 + m(t )  cos ωc t
What is the power efficiency achieved by this modula-
φ (t ) = 2π f c t + k p m(t ) tion scheme?
maximum phase deviation is (a) 8.33 % (b) 11.11 %
(φb ) max = k p max[m(t )] = 2k p (c) 20 % (d) 25 % [2009]

Solution: (c)
for frequency modulator ma2
Power efficacy is given as = η = × 100
2 + ma2

φ (t ) = 2π f c t + 2π k f m(t )dt
=
ma 1
Vm 1
= = ma2
VC 2
(φD′ ) max = 2π k f  m(t )dt 

  max   m = ma 2 + ma 2
a a 2

2  0.25 × 2

 ∫
(φD ) max = 2π k f  m(t )dt 

η =
2 + 0.25 × 2
× 100%
0 
= 20%
1 

(φD′ ) max = 2π k f  2. dt  Hence, the correct option is (c).
  1 6. Consider the frequency modulated signal
0 
(φD′ ) max = 8π k f 10 cos  2π × 105 t + 5 sin( 2π × 1500t ) + 7.5 sin( 2π × 1000t ) 
 
given 10 cos  2π × 10 t + 5 sin( 2π × 1500t ) + 7. 5 sin(
5
2π × 1000t )  with carrier frequency of 10 Hz. 5
 
′ The modulation index is
(φD ) max = (φ0 ) max
(a) 12.5 (b) 10
8πkf = 2kp (c) 7.5 (d) 5 [2008]
Rp Solution: (b)
= 4π
kf Modulation index
Hence, the correct option is (b). f
mf =
14. A message signal m(t ) = cos 2000 π t + 4 cos 4000 π t fm

modulates the carrier c ( t ) = cos 2π f t
c where where
fc = 1 MHz to produce an AM signal. For demodulat- f = maximum frequency deviation.
ing the generated AM signal using an envelope detec-
tor, the time constant RC of the detector circuit should fm = maximum frequency component
satisfy given that
(a) 0.5 ms < RC < 1 ms fm = 1500 Hz
(b) 1µs < < RC < 0.5 ms Deviation,
(c) RC << 1µs
(d) RC >> 0.5 ms [2011] Δθ = 5 sin (2π × 1500t) + 7.5 sin (2π × 1000t)
Solution: (b) dθ
∆w = = 5 × 2π × 1500 cos( 2π × 500t ) + 7.5 × 2π × 1000 cos( 2π × 1000t )
Time constant for envelop detector is given as dt
1 1 dθ
<< RC << ∆w = = 5 × 2π × 1500 cos( 2π × 500t ) + 7.5 × 2π × 1000 cos( 2π × 1000t )
fc fm dt

1 1 ∆wmax = 2π {7500 + 7500}
<< RC <<
1 MHz 2 kHz
∆wmax
1 μs << RC << 0.5 δ= = 1500 Hz

Hence, the correct option is (b).
Chapter 1  Analog Communication Systems | 8.15

δ 15000 (c)
mf = = = 10
fm 1500

Hence, the correct option is (b).

17. The signal coswct + 0.5 coswm tinwct is


(a) FM only
(d)
(b) AM only
(c) both AM and FM
(d) neither AM nor FM [2008]
Solution: (c)
The signal cos wct + 0.5 cos wmt cos wct is either AM or [2007]
narrow-band FM signal.
Solution: (a)
Hence, the correct option is (c).
Hillert transformer is used for SSB generation and sum
18. A Hilbert transformer is a of quadrature components given LSB.
(a) linear system Hence, the correct option is (a).
(b) non-casual system 20. The diagonal clipping in Amplitude Demodulation
(c) time-varying system (using envelope detector) can be avoided if RC time-
(d) low-pass system [2007] constant of the envelope detector satisfies the following
Solution: (a) condition (here W is message bandwidth1 and is w car-
The Hilbert transform of u(t) can be thought of as the rier frequency both in rad/sec) RC < W
convolution of u(t) with the function 1/(πt), which is a 1 1
(a) RC < (b) RC >
linear process. W W
Hence, the correct option is (a). 1
RC > 1 1
(c) RC < W (d) RC >  [2006]
19. In the following scheme, if the spectrum M(f) of m(t) is ω ω
as shown, then the spectrum Y(f) of y(t) will be Solution: (a)
To avoid diagonal clipping detector time constant
1 1
should follow the condition RC ≤ where fc
fc W
denotes carrier frequency and W represents message
signal bandwidth.
Hence, the correct option is (a).
21. A message signal with bandwidth 10 kHz is Lower-
Side Band SSB modulated with carrier frequency fc1
= 106 Hz. The resulting signal is then passed through
a Narrow-Band Frequency Modulator with carrier fre-
quency fc2 = 109 Hz.
The bandwidth of the output would be
(a) 4 × 104 Hz (b) 2 × 106 Hz
(c) 2 × 10 Hz
9
(d) 2 × 1010 Hz [2006]
(a)
Solution: (b)

(b)
8.16 | Communication

From figure Impulse response of match filter h(t) = s(T – t)


fm = 106 Hz h(t) = −s(t)
(as 10k is small in comparison to 106)
BW = 2fm = 2 × 106 Hz
Hence, the correct option is (b).
Common Data for Questions 11 and 12:
Let g(t) = p(t)* p(t), where * denotes convolution
and p(t) = u(t) – u(t – 1) with u(t) being the unit step
function.
22. The impulse response of filter matched to the signal is Hence, the correct option is (c).
given as:
2 3. An Amplitude Modulated signal is given as in the inter-
(a) s(1 – t) (b) –s(1 –t)
val . One set of possible values of the modulating signal
(c) – s(t) (d) s(t) [2006]
and modulation index would be
Solution: (c) (a) t, 0.5 (b) t, 1.0
Signal p(t) can be written as (c) t, 2.0 (d) t2, 0.5 [2006]
P(t) = μ(t) – μ(t – 1) Solution: (a)
Given signal
XAM(t) = 100 {P(t) + 0.5g (t)} cos wct

Signal g(t) can be written as


g(t) = p(t) * p(t)

s(t) = 100 {1 + m(t)} cos wct, p(t) = 1


m(t) = 0.5g(t), comparing AM equation with
i) = 0.5t
modulating signal = t {from equation of line}
modulation index = 0.5
Hence, the correct option is (a).
Then signal s(t) can be written as Common Data for Questions 24 and 25
s(t) = g(t) – g(t – 2) Consider the following Amplitude Modulated (AM)
signal, where fm < B
24. The average side-band power for the AM signal given
above is
(a) 25 (b) 12.5
(c) 6.25 (d) 3.125 [2006]
Solution: (c)
Power of signal will be given by
Chapter 1  Analog Communication Systems | 8.17

PC ma2 AC2 ma2 kHz. If the single tone frequency is increased to 2 kHz,
Ps = = × assuming that phase deviation remains unchanged, the
2 2 2 band width of the PM signal is
100 0.25 (a) 21 kHz (b) 22 kHz
Ps = × = 6.25 watt (c) 42 kHz (d) 44 kHz[2005]
2 2
Solution: (d)
Hence, the correct option is (c).
For phase modulated signal, phase deviation can be
25. The AM signal gets added to a noise with Power
given as
Spectral Density Sn(f) given in the figure below. The
ratio of average sideband power to mean noise power ∆ϕ = mf
would be: mf remains the same for both cases
∆f 10
=
2 1
Δf = 20
BW = 2(Δf + fm) = 2(20 + 2)
= 44 k
Hence, the correct option is (d).
28. A 100 MHz carrier of 1V amplitude and a 1 MHz
25 modulating signal of 1 V amplitude are fed to a bal-
(a) (b) 25
ance modulator. The output of the modulator is passed
8 N0 B 4N B 0 through an ideal high-pass filter with cut-off frequency
25 of 100MHz. The output of the filter is added with
(c) 25 (d)  [2006]
2 N0 B N 0B 100MHz signal of 1V amplitude and 900 phase shift
as shown in the figure. The envelope of the resultant
Solution: (b) signal is
Power of signal will given by
25
Ps =
4
PN = N0B
∴ SNR PS 25 (a) constant
= =
PN 4 N 0 B
A sin {ω ct + km(t )}
(b)
Hence, the correct option is (b).
26. A device with input x(t) and output y(t) is characterized
by: y(t) = x2(t).
(
(c) 5/4 - sin 2p × 106 t )
An FM signal with frequency deviation of 90 kHz and
modulating signal bandwidth of 5 kHz is applied to this
6
(d) 5/4 + cos 2p × 10 t  ( ) [2004]
device. The bandwidth of the output signal is
(a) 370 kHz (b) 190 kHz Solution:(c)
(c) 380 kHz (d) 95 kHz[2005]
Solution: (a)
fm = message signal frequency
Δf = frequency deviation
BW = 2(Δf + fm) = 2(180 + 5)
= 370 kHz
Note: When fm signal is applied to doubles frequency
derivation doubles but fm remains the same.
Hence, the correct option is (a). Given

{ } { }
27. A carrier is phase modulated (PM) with frequency 1
deviation of 10 kHz by a single tone frequency of 1 y (t ) = cos 2p × 101× 106 t + sin 2p × 100 + 106 t
  2
8.18 | Communication

1 cos( 2π × 100 × 10 )t 
6
=  
2 cos 10 t 2π − sin( 2π × 100 × 10 ) 
6 6

6 6
t sin 2π × 10 t + sin 2π × 100 × 10 t
1 1
= cos 2π × 100 × 106 t [cos 2π × 106 t ] + sin 2π × 100 × 106 t [− sin 2π × 106 + 2]
2 2
1
π × 100 × 106 t [cos 2π × 106 t ] + sin 2π × 100 × 106 t [− sin 2π × 106 + 2]
2
= A cos 2π × 106t + B sin cos 2π × 106t
The positive frequencies where Y(f) has spectral peaks
Envelope are
= A2 + B 2 (a) 1 kHz and 24 kHz
(b) 2 kHz and 24 kHz
1 1 (c) 1 kHz and 14 kHz
= cos 2 2π × 106 t + (sin 2π × 106 t − 2)
4 4 (d) 2 kHz and 14 kHz [2004]
Envelope Solution:(b)
5 Peaks of x(t) are at 1k and −1k initially output of bal-
= − sin 2π × 106 t anced modulator = fc ± 1k and fc ± −1k
4
fc ± 1k = 11k, 9k
Hence, the correct option is (c).
fc ± 1k = 9k, 11k
29. Two sinusoidal signals of same amplitude and frequen- Output of HPF with fc = 10k will be 11k frequency
cies 10 kHz and 10.1 kHz are added together. The com- component.
bined signal is given to an ideal frequency detector. The y(f) = 13k ± 11k
output of the detector is
= 24k and 2k
(a) 0.1 kHz sinusoid
(b) 20.1 kHz sinusoid Hence, the correct option is (b).
(c) a linear function of time 31. A DSB-SC signal is to be generated with a carrier fre-
(d) a constant [2004] quency fc = 1 MHz using a non-linear device with the
Solution:(a) input–output characteristic V0 = a0vi + a1vi3 where a0 and
a1 are constants. The output of the non-linear device
Let denote the output frequency as
can be filtered by an appropriate band-pass filter. Let
{ }
s(t ) = A cos 2p × 10 × 10 t + A cos {2p × 10.kt}
3
( )
Vi = Aci cos 2 f ci t + m ( t ) where m(t) is the message
signal. Then the value of f ic (in MHz) is
1
T1 = = 100 µ s (a) 1.0 (b) 0.333
10 k (c) 0.5 (d) 3.0 [2003]
1 Solution: (c)
T2 = = 99 µ s
10.1 Given
T1 V0 = a0vi + a1vi3
= rational 3

T2 = a0 [ Ac1 cos wci t ] + a0 m(t ) + a1 Acc cos 3wci t + a1m3 (t ) + 3a1m 2 (t ) Aci coos wci t + 3

∴s(t) will be periodic with period RCM of T1 and T2 = c3
= a0 [ Ac1 cos wci t ] + a0 m(t ) + a1 Ac cos i 3 2 i i i
3wc t + a1m (t ) + 3a1m (t ) Ac coos wc t + 3a1 Ac cos wc t
2 i
9900 μs, 10,000 μs
∴frequency of elector = 0.1 kHz. For DSB SC we are concerned with only last term
Hence, the correct option is (a). m(t) cos wct → fc = 1 MHz
For cos2 term 2wci = 2π × 1 MHz
30. Consider a system shown in figure. Let X(f) and Y(f)
denote the Fourier transforms of x(t) and y(t), respec- Hence, the correct option is (c).
tively. The ideal HPF has the cutoff frequency 10 kHz. Common Data for Questions 32 and 33
Let m(t) = [(4π× 103)t] be the message signal and c(t) =
5cos[(2π× 106)t] be the carrier.
Chapter 1  Analog Communication Systems | 8.19

32. c(t) and m(t) are used to generate an AM signal. The Group-1
modulation index of the generated AM signal P. Ring modulator Q. VCO
R. Foster-Seely discriminator S. Mixer
Total sideband power
is 0.5. Then the quality is Group-2
Carrier power
1. Clock recovery
2. Demodulation of FM
(a) 1/2 (b) 1/4 3. Frequency conversion
(c) 1/3 (d) 1/8 [2003] 4. Summing the two inputs
Solution: (d) 5. Generation of FM
Pc= carrier power 6. Generation of DSB-SC
ma = modulation index (a) P – 1; Q – 3; R – 2; S – 4
(b) P – 6; Q – 5; R – 2; S – 3
Ac = amplitude of Carrier signal
(c) P – 6; Q – 1; R – 3; S – 2
 m2  A2 (d) P – 5; Q – 6; R – 1; S – 3 [2003]
P1 = PC 1 + a  , PC = C
 2  2 Solution:(b)

Combination is given as followed
A2 m 2
Sideband power = C . a P. Ring modulator Generation of DSB-
2 2 SC
 AC2 ma2  Q. VCO Generation of FM
  R. Foster-Seely discriminator Demodulation of FM
PS  4  1
= =
PC  AC2  8 S. Mixer Frequency
  conversion
 2 
Hence, the correct option is (b).
Hence, the correct option is (d). 35. A superheterodyne receiver is to operate in the
33. c(t) and m(t) are used to generate an FM signal. If the frequency range 550–1650 kHz, with the intermediate
peak frequency deviation of the generated FM signal C
is three times the transmission bandwidth of the AM frequency of 450 kHz. Let R = max denote
Cmin
signal, then the coefficient of the term cos[2π(1008 ×
the required capacitance ratio of the local oscillator and
103t)] in the FM signal (in terms of the Bessel coeffi-
I denote the image frequency (in kHz) of the incoming
cients) is 5 signal. If the receiver is tuned to 700 kHz, then
(a) 5J4(3) (b) J (3)
2 8 (a) R = 4.41, I = 1600
5 (b) R = 2.10, I = 1150
(c) J (4) (d) 5J4(6) [2003]
2 8 (c) R = 3.0, I = 1600
(d) R = 9.0, I = 1150 [2003]
Solution: (d)
Solution:(a)
Given
fIF = intermediate frequency
FM modulated waveform is given by

fis = Image frequency
X FM (t ) = AC ∑ j ( β ) cos(w
n c + nwm )t C  f 
R = max =  max  = 
2
 1650 + 450 
2

n =−∞  = 4.41

Cmin  f min   550 + 450 
BW = 2wn, Δw = 3 × Bw = 6wn
1 = fs + 2fif
wc + nwn = 2π × 1008 × 103 (as given in a question) 
    = 2π × 1008 × 10 3 = 700 + 2 × 450 = 1600
∴η=4 Hence, the correct option is (a).
Bessel coefficient 5j4(6) 36. An angle-modulated signal is given by
Hence, the correct option is (d). S(t) = cos2π(2 × 106t + 30sin150t + 40cos150t).
34. Choose the correct one from among the alternative a, The maximum frequency and phase deviations of s(t)
b, c, d after matching an item in Group 1 with the most are
appropriate item in Group 2. (a) 10.5 kHz, 140π rad
(b) 6 kHz, 80π rad
8.20 | Communication

(c) 10.5 kHz, 100π rad Now passing from LPF with fc = 1 Hz
(d) 7.5 kHz, 100π rad [2002] sin 2t sin 0.5π t + cos 1.5π t
y (t ) = +
Solution:(d) 2t t

Given
38. A message m(t) band limited to the frequency fm has
s(t) = cos[2π × 106 × 2t + 2π × 30 sin 150t + 2π a power of Pm. The power of the output signal in the
× 40 cos 150t] figure is

= 2π × 2 × 106 + 2π × 4500 cos
dt
150t + 2π × 6000( − sin 150t )
1
2 2 2
∆w = w1 − wc = 2π [4500 + 6000 ]
Δw = 2π × 7.5k rad/sec P cosθ Pm
(a) m (b)
∆w 2 4
∆f = = 7.5 kHz
2π Pm cosθ
Pm sin 2θ (d)
(c)  [2000]
2 2 4 2
∆φ = 2π 30 + 40
∆φ = 100π Solution: (d)
m(t )
Hence, the correct option is (d). Output signal = cos q
2
37. In the figure m (t ) = 2 sin 2π t , s(t) = cos200πt and m(t) has power Pm
t Power of a m(t) → a2.Pm
n (t ) = sin199π t . The output y(t) will be Here a =
1
cos θ
t 2
Pm
Power of output signal = cos 2 θ
4
Hence, the correct option is (d).
39. The Hilbert transform of cosω1t + sinω2t is
(a) sinω1t − cosω2t (b) sinω1t + cosω2t
sin2p t
(a) (c) cosω1t − sinω2t (d) sinω1t + sinω2t[2000]
t
Solution: (a)
sin2p t sinp t A Hilbert transform produces a 90o phase shift
(b) + cos3p t
t t H .T
sin 2t sin 0.5π t + cos 1.5π t sin ω2 t ← → − cos ω2 t
y (t ) =
(c) +
2t t H .T
cos ω1t ← → sin ω1t
sin2p t sin0.5p t
(d) [2002] Hence, the correct option is (a).
+ cos0.75p t 
t t 4 0. In an FM system, a carrier of 100 MHz is modulated
Solution: by a sinusoidal signal of 5 kHz. The bandwidth by
Carson’s approximation is 1 MHz. If y(t) = (modulated
wave form)3, then by using Carson’s approximation, the
bandwidth of y(t) around 300 MHz and the spacing of
spectral components are, respectively.
(a) 3 MHz, 5 kHz (b) 1 MHz, 15 kHz
(c) 3 MHz, 15 kHz (d) 1 MHz, 5 kHz
Input at LPF x(t) = [m(t).s(t) + n(t)] x(t)  [2000]
x(t) = m(t).s2(t) + n(t) s(t) Solution: (a)
1 Frequency modulated waveform is given by
x(t ) = [sin 202π t + sin 199π t ]cos 200π t
t x(t) = AC cos[wct + β sin wmt]
402 2 2 398 1 399 y(t) = k cos [3wct + 3 β sin wmt]
Frequency present = , . , , ,
2 2 2 2 2 2
Chapter 1  Analog Communication Systems | 8.21

After passing from y(t) = [x(t)]3 this is the case of over modulation. So, synchronous
b' = 3b β= modulation index modulator only can detect.
∆f ′ = 3 × ∆f = ∆f >> f m Δf = frequency deviation Hence, the correct option is (c).
BW = 2∇f ′ × 3 = 3 MHz 44. A superheterodyne radio receiver with an intermediate
frequency of 455 kHz is tuned to a station operating
fm = 5 kHz
at 1200 kHz. The associated image frequency is ____
Hence, the correct option is (a). kHz. [1993]
41. An FM signal with a modulation index 9 is applied to Solution:
a frequency tripler. The modulation index in the output
IF = intermediate frequency
signal will be
(a) 0 (b) 3 fis= Image frequency
(c) 9 (d) 27 [1996] fsi = fs + 2IF
Solution: (d) Fsi = 1200 + 2(455) = 2110 KHz.
In a frequency multiplier circuit, the modulation index 45. The maximum power efficiency of an AM modulator is
is multiplied by η. So (a) 25% (b) 50%
b'FM = ηbFM βFM= modulation index (c) 33% (d) 100% [1992]
b' = 3 × 9 = 7 Solution: (b)
Hence, the correct option is (d). Power efficiency of AM modulator is given by
42. Match List-I with List-II and select the correct answer µ2
η= , μmax = 1,
using the code given below the lists: 2 + µ2
List-I List-II where is modulation index
A. SSB 1. Envelope detector 1 1
B. AM 2. Integrate and dump η max = =
2 +1 3
C. BPSK 3. Hilbert transform
D. 4. Ratio detector Hence, the correct option is (b).
5. PLL 46. A 4 GHz carrier is DSB--SC modulated by a low pass
Codes: message signal with maximum frequency of 2 MHz.
A B C The resultant signal is to be ideally sampled. The mini-
(a) 3 1 2 mum frequency of the sampling impulse train should
(b) 3 2 1 be
(c) 2 1 3 (a) 4 MHz (b) 8 MHz
(d) 1 2 3 [1994] (c) 8 GHz (d) 8.004 GHz
Solution:(a)  [1990]
Hibert transform → SSB Solution: (b)
AM → Envelope Detector Fc = carrier frequency
BPSK → Integrate and dump fm = message signal frequency
Hence, the correct option is (a). fL = lower frequency
43. Which of the following demodulator(s) can be used for fH = higher frequency
demodulating the signal fc = 4 GHz = 4000 MHz
x(t) = 5(1 + 2 cos200pt) cos20000πt Fm = 2 MHz, fH = fc + fm
(a) envelope demodulator FL = fc − fm
(b) square-law demodulator
BW = FH – FL = 2 fm
(c) synchronous demodulator
(d) none of the above [1993] BW = 2 × 2 = 4 MHz
Solution:(c) Hence, the correct option is (b).
Given that x(t) = 5(1 + 2 cos 200 πt) cos 20000 πt, 47. In commercial TV transmission in India, picture and
speech signals are modulated respectively as
standard equation for AM signal is XAM(t) = AC(1 + m
cos wmt) cos wct. (Picture) (Speech)
By comparing m = 2, (a) VSB and VSB
(b) VSB and SSB
8.22 | Communication

(c) VSB and FM Then the modulation index of the FM wave is


(d) FM and VSB [1990] (a) 4 (b) 2
(c) 4/p (d) 2/p [1989]
Solution: (c)
Solution:(a)
In commercial TV transmission in India, picture signal
is modulated using VSB (vestigial sideband) modula- Am = amplitude of message signal
tion and speech or audio signal is modulated using FM wm = carrier frequency
modulation. Kf = frequency sensitivity
Hence, the correct option is (c). k f Am ∆w
48. A carrier Ac cosωct is frequency modulated by a signal Modulation index = m f = =
wm wm
Em cosωmt. The modulation index is mf. The expression
for the resulting FM signal is k f Am 2π × 10 × 103 × 2
m1 = = =4
(a) Ac cos ωc t + m f sin ωm t  wm π × 10 4
Hence, the correct option is (a).
Ac cos ωc t + m f cos ωm t 
(b)
51. In a super heterodyne AM receiver, the image channel
(c) Ac cos ωc t + 2π m f sin ωm t  selectivity is determined by
 2π m f Em  (a) The preselector and RF stages
(d) Ac cos ωc t + cos ωm t   [1989]
ωm (b) The preselector, RF and IF stages
 
(c) The IF stages
Solution:(a)
(d) All the stages [1987]
wc = carrier frequency
Solution:(a)
wm = carrier frequency
The image rejection should be achieved before IF stage
mf  = modulation index because once it enters into IF amplifier, it becomes
Kf = frequency sensitivity impossible to remove it from wanted signal. So image
channel selectivity depends upon pre-selector and RF

X Fm (t ) = AC cos[ wc t + k f m(t )dt ] amplifiers only. The IF helps in rejection of adjacent
channel frequency and not image frequency.
X Fm (t ) = AC cos[ wc t + k f ∫E m cos wn tdt ]
Hence, the correct option is (a).
 k f Em  52. In a radar receiver the antenna is connected to the
xFm (t ) = AC cos  wc t + sin wm t  receiver through a waveguide. Placing the preamplifier
 wm 
on the antenna side of the waveguide rather than on the
Modulation index receiver side leads to
(a) a reduction in the overall noise figure.
k f Em
mf = (b) a reduction in interference.
wm (c) an improvement in selectivity characteristics

(d) an improvement in directional characteristics
XPm(t) = AC cos [wct + mf sin wmt]
[1987]
Hence, the correct option is (a).
Solution:(a)
49. Which of the following schemes suffer(s) from the
A pre-amplifier is a very large gain amplifier with low
threshold effect?
noise figure. Noise figure of cascaded amplifier can be
(a) AM detection using envelope detection
given as:
(b) AM detection using synchronous detection
(c) FM detection using a discriminator F2 − 1 F3 − 1 Fn − 1
F = F1 + + + ...
(d) SSB detection with synchronous detection [1989] G1 G1G2 G1G2G3Gn −1

Solution: (c)
Therefore, placing the pre-amplifier on the antenna
FM detection using a discriminator suffers from the side of the wave guide will result in the reduction of
threshold effect. overall noise figure of the system.
Hence, the correct option is (c). Hence, the correct option is (a).
50. A signal x(t) = 2 cos(p .10 4 t) volts is applied to an FM
modulator with the sensitivity constant of 10 kHz/volt.
Chapter 1  Analog Communication Systems | 8.23

of g(t) is shown in figure. Sketch the spectrum of the


Five-marks Questions transmitted signal and the spectrum of the received
1. A signal x(t) = exp(–2pBt) u(t) is the input to an ideal signal.
low pass filter with bandwidth B Hz. The output is
denoted by y(t). Evaluate

∫ [y(t) − x(t)]
2
dt.[1988]
−∞

Solution: g iven that, x(t) = exp(−2πBt) u(t) taking fou-


rier transform on both side,
1 1
x( f ) = = Solution:
2p B + jw 2p ( B + j f )

So, H(f) = 1 for –B ≤ f ≤ B
= 0 for otherwise
for –B ≤ t ≤ B
= 0 for otherwise
1
Y ( f ) = X ( f ).H ( f ) = for –B ≤ t ≤ B
2p ( B + j f )
 = 0 for otherwise x(t) = g(t) . cos(2π fct)
We know that, 1
x (t ) = [G ( f − f c ) + G ( f + f c )]
| y(t ) | dt 2
∞ ∞

∫ y (t )dt ∫
Received signal
1 1
−∞ −∞
∫B 2
f 2
df

y(t) = 2x + x2 = y1(t) + y2(t)
y1(t) = 2x(t) ↔ 2x(f) = y1(f)

1 g 2 (t ) g 2 (t )
| x( f ) 2 df |= y2 ( f ) = x 2 (t ) = g 2 (t ).cos 2 2p fct = + cos 4p f c t
∞ ∞
4p 2 2 2
∫ ∫
2
x (t )dt = ∞
1 1 g 2 (t ) g 2 (t )
−∞ −∞
∫B
−∞
2
+ f2
df = y2 ( f ) = x 2 (t ) = g 2 (t ).cos 2 2p fct =
4p B 2
+
2
cos 4p f c t

= y3 (t ) + y4 (t )
x(t ) y *(t )dt =
∞ ∞
1
∫ x(t ) y(t )dt = ∫ g 2 (t ) 1
B
1 1 dt = y3 (t ) = = [G ( g ) * G ( f )]
−∞ −∞ 4p ∫B
−B
2
+ t2
8p B
2 2
2
∞ x4 (t ) = g (t ) cos(2p 2 f c t ) = y3 (t ).cos(2p 2 f c t )
∫ [ y(t ) − x(t )] dt
2
1
y4 ( t ) = [ y3 ( f − 2 f c ) + y3 ( f + 2 f c )
−∞ 2


y 2 (t )dx y2(f) = y3(f) + y4(f)
∫ + ∫ x (t )dt − 2 ∫ x(t ). y(t )dt
∞ ∞
= 2 y2(f) = y1(f) + y2(f).
−∞
−∞ −∞

1 1 1 1
= + − 2. =
8p B 4p B 8p B 8p B
2. A baseband signal g(t) band limited to 100 Hz modu-
lates a carrier of frequency fc Hz. The modulated signal
g(t)cos2pfct is transmitted over a channel whose input
x and output y are related by y = 2x + x2. The spectrum
8.24 | Communication
Chapter 2
Random Signals and Noise
One-mark Questions =
2 ( 2) +
1× 1

1. Let the random variable X represent the number of times



( 2 ) (1 − 12 ) 
1− 1
2

a fair coin needs to be tossed till two consecutive heads


appear for the first time. The expectation of X is ______. (1 )
=4+ 2
 [2015]

( 14 ) 
Solution:  Given that X is a random variable represent-
ing “The number of times a fair coin needs to be tossed = 4 + 2
till two consecutive heads appear for the first time”. = 6
∴ The possible values of X are 2, 3, 4, 5, …… ∴ Substituting in (1), we have
1 1 1 1 3
P(X = 2) = P(HH) = P(H) P(H) = × = 2 E(X) = × 6 = = 1.5
2 2 2  4 2
P(X = 3) = P(THH) = P(T) P(H) P(H) Hence, the correct Answer is (1.5).
1 1 1 1 2. A zero mean white Gaussian noise having power spec-
= × × = 3 N
2 2 2 2  tral density 0 is passed through an LTI filter whose
2
P(X = 4) = P(TTHH) = P(T) P(T) P(H) P(H)
impulse response h(t) is shown in the figure. The vari-
1 1 1 1 1 ance of the filtered noise at t = 4 is  [2015]
=× × × =
2 2 2 2 24  3 3 2
∞ (A) A2 N 0 (B) A N0
∴ Expectation of X = E(x) = ∑ x P ( X = x ) 2 4
x=2  1 2
(C)
A2 N0 (D) A N0
=
2P(X = 2) + 3 P(X = 3) + 4P(X = 4) + … ∞ 2
1 1 1 h(t)
= 2× 2
+ 3 × 3 + 4 × 4 + .... ∞
2 2 2 
A
⎡ 1 1 1 1 ⎤
∴ E ( X ) = ⎢ 2 + 3 × 2 + 4 × 22 + 5 × 23 + … ∞ ⎥  (1)
⎣ 22 ⎦ t
0 1 2 3
1 1 1
Consider 2 + 3 × + 4 × 2 + 5 × 3 + .... ∞
2 2 2  –A
which is an arithmetic geometric progression with
1
a = 2, d = 1 and r = Solution:  For a zero mean white Gaussian noise
2 ­variance is infinite.
∴ The sum of infinite number of terms in AGP
So for finite variance of white Gaussian noise we
a dr should have some BIBO system.
= +
1 − r (1 − r )2 +∞
N0
 σ2 = ∫ 2
|H( f )|2 df
−∞
8.26 | Communication

By Parseval’s relation = Rx(k) + 0.5 Rx[k + 1] + 0.5 Rx(k – 1) + 0.25 Rx(k)


+∞
No Ry(k) = 1.25 Rx(k) + 0.5 Rx(k + 1) + 0.5 Rx(k – 1)
∫ h (t ) dt
2
= 
2 Rx(k) = E[x[n] x[n + k]]
−∞

No ⎡ 1 2 3

⎢ ∫ h (t ) dt + ∫ h (t ) dt + ∫ h (t ) dt ⎥
2 2 2
= 1 1
2 ⎢⎣ 0 ⎥⎦ for k = 0 E[x2[n]] = I2 + (–1)2 = 1
1 2
 2 2
For k ≠ 0
No 2 3 A2 N o
⎡ A + A2 + A2 ⎤ =
=
2 ⎣ ⎦ 2  Rx(k) = E[x[n]] ⋅ E[X(n + k)] = 0

Hence, the correct option is (A). So Ry(k) = 1.25 Rx(k) + 0.5 Rx(k – 1) + 0.5 Rx(k + 1)
For Ry(0) = 1.25 Rx(0) + 0.5 Rx(–1) + 0.5 Rx(+ 1)
3. { X n }n = −∞ is an independent and identically distributed
n =∞

= 1.25
(i.i.d) random process with Xn equally likely to be +1 or
–1. {Yn }n = −∞ is another random process obtained as Yn =
n =∞ Ry(1) = 1.25 Rx(1) + 0.5 Rx(0) + 0.5 Rx(+ 2)

Xn + 0.5 Xn-1. The autocorrelation function of {Yn }n = −∞ = 0.5


n =∞

, denoted by RY[k], is [2015] Ry(– 1) = 1.25 Rx(– 1) + 0.5 Rx(– 2) + 0.5 Rx(0)
= 0.5
(A) RY[k] 1
Ry(2) = 1.25 Rx(2) = 0.5 Rx(1) + 0.5 Rx(3)
So all values of Ry(k) are zero except at k =0, 1 and –1.
So
–3 –2 –1 0 1 2 3 k
1.25
(B) RY[k] 1.25
0.5 0.5

0.5 0.5

–1 0 +1 k

–3 –2 –1 0 1 2 3 k Hence, the correct option is (B).


(C) RY[k] 1.25 4. Let x1, x2, and x3 be independent and identically distrib-
uted random variables with the uniform distribution on
0.5 0.5 [0, 1]. The probability P{x1 is the largest} is_________
0.25 0.25  [2014]
Solution:
–2 –1 0 1 2 k Probability P(x1) = P(x2) = P(x3)
(D) RY[k] 1.25
P1 + P2 + P3 = 1 {total probability theorem}
P(x1) + P(x2) + P(x3) = 1
3p (x1) = 1
0.25 0.25
P(x1) = 1/3 = 0.3733
–3 –2 –1 0 1 2 3 k 5. Let X be a random variable which is uniformly chosen
from the set of positive odd numbers less than 100. The
Ry(k) = Ry(n, n + k)
Solution:  expectation, E[X], is [2014]
= E[Y[n] ⋅ Y[n + k)] Solution:
Y[n] = X[n] + 0.5 x[n – 1] 1 + 2 + 3 + ...99
E ( x) =
50
Ry(k) = E[(X[n] + 0.5 X[n – 1]) ⋅ (x[n + k]
+ 0.5 x[n + k – 1])] 2500
E ( n) =
= E[x[n] x[n + k] + 0.5 x[n – 1] x[n + k] 50
+ 0.5 x[n] x[n + k – 1] + 0.25 x[n – 1] x[n + k – 1] E(Y) = 50
Chapter 2  Random Signals and Noise | 8.27

6. Let X be a zero mean unit variance Gaussian random


variable. E[|X|] is equal to _____. [2014]
Solution:
For Gaussian random variables
2
µ−x 1
f x ( x) =  
σ x 2π  2σ 2 
for mean (μ) = 0 and variance (σ2X) = 1
1 2/ 2
f x ( x) = e− x


Area of shaded region
E[ x ] = ∫ ( x ) f x ( x )dx

P (max( x, y ) < 1/ 2} =
Area of entire rectangle
−∞

∞ 3 3
1 − x2 / 2
×


= ∫ x.

e dx = 2 2 =
2 × 2
9
16
−∞
0 ∞
1 2 /2 1 2 /2 Hence, the correct option is (b)


= ∫
−∞
− x.

e− x ∫
dx + x.
0

e− x dx
9. A power spectral density of a real process X(t) for posi-
tive frequencies is shown below. The values of E[X2(t)]
∞ ∞ − x2 and E  X (t ) , respectively, are
1 − x2 / 2 2

= 2 x.

e dx +
2π ∫ x.e 2 dx
0 0

2
Let − x = t
2
−2 × dx
= dt
2
∞ (a) 6000/π, 0
2
Hence
2π 0 ∫ −e −t dt = 0.8 (b) 6400/π, 0
(c) 6400/π, 20/(π/ 2 )
∴ E[|x|] = 0.8 (d) 6000/π, 20/(π 2 ) [2012]
7. If calls arrive at a telephone exchange such that the Solution: (a)
rate of arrival of any call is independent of the time of
arrival of earlier or future calls, the probability distribu-
tion function of the total number of call in a fixed time
interval will be
(a) Poisson (b) Gaussian
(c) Exponential (d) Gamma [2014]
Solution: (a)
Given system follows property of Poisson distribution.
Hence, the correct option is (a) We know
8. Two independent random variables X and Y are uni- 11×103
1

2
formly distributed in the interval [−1, 1]. The probabil- P = E[ x (t )] = 2 × S x ( w )dw
ity that max [X, Y] is less than 1/2 is 2π
9×103
(a) 3/4 (b) 9/16
(c) 1/4 (d) 2/3 [2013] 1  1 3
E[ x 2 (t )] =  400 + 2 × 6 × 2 × 10 
Solution: (b) π  

−1 ≤ x ≤ 1 and −1 ≤ y ≤ 1 is the entire rectangle, region 6400
for maximum of {x, y} is less than ½ is shaded region =
π
inside the rectangle.
8.28 | Communication

at w = 0 there is no any frequency component available. (d)


So, the value of the process will be zero.
Hence, the correct option is (a)
10. A white noise process x(t) with two-sided power spec-
tral density 1 × 10-10 W/Hz is input to a filter whose
magnitude squared response is shown below. Solution: (b)
PSD = FT{R(τ)}
Which is (sinc)2 function
Hence, the correct option is (b)
12. If E denotes expectation, the variance of a random vari-
able X is given by
(a) E[x2] – E2[x] (b) E[x2] + E2[x]
(c) E[x ] (d)
2
E2[x] [2007]
The power of the output process y(t) is given by Solution: (a)
(a) 5 × 10–7 W (b) 1 × 10–6 W
Variance can be given as
(c) 2 × 10 W
–6
(d) 1 × 10–5  [2009]
2 2 2
Solution: (b) σ x = E[ x ] − E ( x )
PSD of white noise = 1 × 10−10 w/Hz A.C. power = Total power – D.C. power
Output PSD = |H(f)|2 input PSD Hence, the correct option is (a)
G0(f) = K.|H(f)|2 13. If R(τ) is the auto-correlation function of a real, wide-
Output Noise power N0 = sense stationary random process, then which of the fol-
f0 lowing is NOT true?
(a) R(τ) = R(-τ)
∫ G ( f )dt = k × (Area under | H ( f ) |
2
cur)
0
(b) |R(τ)| ≤ (0)
− f0
(c) R(τ) = − R(−τ)
1  (d) The mean square value of the process is R(0)
= k × 2  × base × height   [2007]
2 
Solution: (c)
= kf0 × 1 Auto-correlation function is an even function.
= 1 × 10−10 × 10 × 103 = 10−6 w Hence, the correct option is (c)
Hence, the correct option is (b) 14. If S(f) is the power spectral density of a real, wide-sense
11. If the power spectral density of stationary random pro- stationary random process, then which of the following
cess is a sinc-squared function of frequency, the shape is Always true?
of its auto-correlation is [2009] (a) S(0) ≥ S(f) (b) S(f) ≥ 0
(a) ∞
S(−f) = −S(f) (d)
(c) S ( f )df = 0 ∫
−∞
 [2007]
Solution: (b)
Power Spectral Density is always positive function.
(b)
Hence, the correct option is (b)
15. The distribution function Fx(x) of a random variable X
is shown in the figure. The probability that X = 1 is

(c)
Chapter 2  Random Signals and Noise | 8.29

(a) zero (b) 0.25  x2 


1
(c) 0.55 (d) 0.30 [2004] Px ( x ) = exp  − 
2π  2 
Solution: (d)
Given We have given
P ( x ) = K .exp  − x 
2
P(x = 1) = F(x = 1+) – Fx(x = 1−)
x  
= 0.5 – 0.25  2 
= 0.30 By comparing
Hence, the correct option is (d)  1 
k = 
16. The noise at the input to an ideal frequency detector is
 2π 
white. The detector is operating above threshold. The
power spectral density of the noise at the output is Hence, the correct option is (a)
(a) raised-cosine (b) flat 19. The amplitude spectrum of a Gaussian pulse is
(c) parabolic (d) Gaussian [2003] (a) uniform
Solution: (c) (b) a sine function
(c) Gaussian
When the detector is operating above the threshold
(d) an impulse function [1998]
them PSD of the noise at the output of ideal frequency
detector is parabolic. Solution: (c)
Hence, the correct option is (c) Since Fourier transform (Time domain to frequency
domain transform) of Gaussian signal is also a Gaussian
17. The PDF of a Gaussian random variable X is given by
− ( x − 4 )2
Signal, so spectrum of amplitude will be a Gaussian
1 . The probability of the event {X also.
Px ( x ) = e 18
3 2π 2
−π t 2
= 4} is e ← F .T .
→ e −π f
1 1 Hence, the correct option is (c)
(a) (b)
2 3 2p 20. The ACF of a rectangular pulse of duration T is
(a) a rectangular pulse of duration T
1
(c) 0 (d)  [2001] (b) a rectangular pulse of duration 2T
4 (c) a triangular pulse of duration T
(d) a triangular pulse of duration 2T [1998]
Solution: (c)
Solution: (d)
For a Gaussian random variable, probability is defined
for an interval, for a point it is always zero. ACF of a rectangular pulse of duration T is

Hence, the correct option is (c)
18.
A probability density function is given by
Rx (τ ) =
−∞
∫ x(t ).x(t + τ ).dt

P (x ) = K exp( − x 2 / 2). − ∞ < x < ∞ . The value of K
should be Here by solving through this formula, auto-correlation
of rectangular pulse of duration T is a triangular pulse
1 2 of duration 2T.
(a) (b) Hence, the correct option is (d)
2p p
21. The probability density function of the envelope of nar-
1 1
(c) p (d)  [1998] row band Gaussian noise is
2 p 2 (a) Poisson (b) Gaussian
(c) Rayleigh (d) Rician [1998]
Solution: (a)
Solution: (c)
We know that this is a form of a Gaussian pulse. So, we
can compare it by a Gaussian PDF. The probability density function of the envelope of
narrow band Gaussian noise follows Rayleigh density
1  −( x − µ ) 2  function.
Px ( x ) = exp  2 
2π a 2  2a  Hence, the correct option is (c)

when σ = 1 and μ = 0 22. For a narrow band noise with Gaussian quadrature
components, the probability density function of its
envelope will be
8.30 | Communication

(a) uniform (b) Gaussian 2. A random variable X takes values –1 and +1 with proba-
(c) exponential (d) Rayleigh [1995] bilities 0.2 and 0.8, respectively. It is transmitted across
Solution: (d) a channel which adds noise N, so that the random vari-
able at the channel output is Y = X + N. The noise N is
As we have discussed in 2.2 80 Ω a narrow band noise
independent of X, and is uniformly distributed over the
with Gaussian quadrature components the probabil-
interval [–2, 2]. The receiver makes a decision
ity density function of its envelop will be a Rayleigh
function.
Hence, the correct option is (d)
X̂ = { −1, if Y ≤ θ
+1, if Y > θ

where the threshold q ∈[–1, 1] is chosen so as to mini-


Two-marks questions mize the probability of error Pr [ Xˆ ≠ X ]. The minimum
1. Let a random process Y(t) be described as Y(t) = h(t) probability of error, rounded off to 1 decimal place, is
* X(t) + Z(t), where X(t) is a white noise process with  [2019]
power spectral density Sx(  f ) = 5 W/Hz. The filter h(t) Solution: P(X = –1) = 0.2
has a magnitude response given by H ( f ) = 0.5 for –5 P(X = 1) = 0.8
≤ f ≤ 5, and zero elsewhere. Z(t) is stationary random f(x)
process, uncorrelated with X(t), with power spectral 1
density as shown in the figure. The power in Y(t), in Pdf: 4
watts, is equal to ______ W (rounded off to two deci-
mal places).  [2019]
1 Sz(f ) (W/Hz) –2 2

1 if 1 is transmitted
4

–5 5 f(Hz)
Solution:
–1 0 3

1 Sz(f ) W/Hz 1
if –1 is transmitted
4

–5 5 f(Hz) –3 0 1

P  y (t )  = P  x (t ) ⋅ h (t )  + P  z (t )  Pe = P (1) ⋅ Pe1 + P ( −1) Pe −1


Pz (t ) = Area under the curvve 1 0.4
= 2 × 0.2 × = = 0.1
1 4 4
= × 10 × 1 = 5 w
2
3. A binary source generates symbols X ∈ {–1, 1} which

are transmitted over a noisy channel. The probabil-
Px (t ) ⋅ H (t ) = H (t ) S × ( f ) df
2
∫ ity of transmitting X = 1 is 0.5. Input to the threshold
−∞
detector is R = X + N. The probability density function
H ( H ) ⋅ S x ( f ) = 0.52 × 5 = 1.25
2 fN(n) of the noise N is shown below.


fN(n)
Px (t ) ⋅ H (t ) = ∫ 1.25 dt 0.5
−∞
5
= ∫ 1.25 dt = 1.25 5 − ( −5) –2 2 n
−5

= 1.25 × 10 = 12.5 w If the detection threshold is zero, then the probability


Py (t ) = 5 + 12.5 w of error (correct to two decimal places) is ________
 [2018]
= 17.5 w
Chapter 2  Random Signals and Noise | 8.31

Solution:  If we assume that S0 & S1 be transmitted density over the interval (–1, 1). X and Z are independ-
symbols with values (–1, 1) respectively and r0 and r1 ent. If the MAP rule-based detector outputs X̂ as
be the received symbols. Then we have
⎧−0.5, Y < ∝
Xˆ = ⎨
fN(n)
⎩ 0.5, Y ≥ ∝,
0.5 1
Slope = then the value of ∝ (accurate to two decimal places) is
4
______. [2018]
Solution:  Consider the figure given below
f2(Z)

–3 –2 –1 0 1 2 3 n 1/2

P(r0/S1) P(r1/S0)

t
Probability of error can be calculated as given below –1 0 +1

Pe = P(S1) P(r0IS1) + P(S0) P(r1IS0)


1 3
P(x0) = ; P(x1) =
1 1 1 4 4
P(r0IS1) = P(r1IS0) = × 1× =
2 4 8 x0
1 >
We know that P(S0) = P(S1) = fy (Y1 | x0) P(x0) fy(Y1 | x1) P(x1)
2 <
1⎛1 1⎞ x1
⇒ Pe = + = 0.125
2 ⎜⎝ 8 8 ⎟⎠ α = –0.5
Hence, the correct answer is 0.12 to 0.14.
fy (Y1 | x0) P(x0) fy
4. Consider a white Gaussian noise process N(t) with two-
1
sides power spectral density SN( f ) = 0.5 W/Hz as input = fy (Y1 | x1) P(x1)
8
3

to a filter with impulse response 0.5e −t / 2 (where t is in =


2

8
seconds) resulting in output Y (t). The power in Y(t) in
watts is  [2018]
(A) 0.11 (B) 0.22
(C) 0.33 (D) 0.44 –1.5 –1 –1.5 0 0.5 1 1.5
Solution:  We know that power spectral density of
noise input, is expressed as Optimum threshold

SN( f ) = 0.5 W/Hz Hence, the correct answer is –0.5.


Power of output can be calculated as 6. Let X (t) be a wide sense stationary random process
∞ with the power spectral density Sx(f) as shown in Figure
2
y(t) = ∫ SN ( f ) H ( f ) df (a), where f is in Hertz (Hz). The random process X (t)
is input to an ideal lowpass filter with the frequency
−∞ 
∞ ∞ response.
2 2
0.5 ∫ H ( f ) df = 0.5 ∫ h(t ) dt
=  1
−∞ −∞  1, f ≤ 2 Hz
H( f ) = 
2 0, f > 1 Hz
1 ⎛1 2 ⎞
∞ −t 2


= ∫ ⎜ 2 ⎟⎟ dt = 0.22 W
2 −∞
⎜ e 2
⎝ ⎠ as shown in figure (b). The output of the low-pass filter
Hence, the correct option is (B) is Y(t).
5. A random variable X takes values -0.5 and 0.5 with Sx(f)
X(t) ideal lowpass Y(t)
1 3 exp (−⎜f ⎜) filter h(t)
probabilities and , respectively. The noisy observa- cutoff = 1/2 Hz
4 4
0
tion of X is Y = X + Z, where Z has uniform probability (a) (b)
8.32 | Communication

Let E be the expectation operator and consider the fol- S = 819.1 × 2


lowing statements:
819.1 × 2
I. E [X (t)] = E [Y (t)] Eb = = 31.503.
Rb
II. E [X2 (t)] = E [Y2 (t)]
Hence, the correct Answer is (31.503).
III. E[Y2(t)] = 2
8. Consider a random process x(t) = 3V(t) – 8, where V(t)
Select the correct option:  [2017] is a zero-mean stationary random process with auto-
(A) Only I is true correlation Rv(τ) = 4e–5|τ|. The power in X(t) is _____.
(B) Only II and III are true  [2016]
(C) Only I and II are true Solution:  As we know that power is the mean square
(D) Only I and III are true value
Solution:  The frequency response of the LPF is given by Given Rv(τ) = 4e–5|τ|.
E[ y(t )] = H (0) E[ x(t )] And V(t) is a zero-mean stationary random process so
E[V(t)] = 0
H ( 0) = 1
E[x2(t)] = power
⇒ E[ y(t )] = E[ x(t )]
E[(3V(t) – 8)2]
E[ y (t )] ≠ E[ x (t )]
2 2
E[9V2(t) + 64 – 48 V(t)]
As LPF does not allow 100% power from input to 9E[V2(t)] + E[64] – 48 E[V(t)]
­output
⇒ E[V(t)] = 0, E[64] = 64.

E[V2(t)] = R(0) = 4 ⋅ e–5(0) = 4.
E[ x 2 (t )] = ∫ sx ( f )df = 2 W
0 ⇒ 9 × 4 + 64.
⇒ 100.
As E[ x 2 (t )] ≠ E[ y 2 (t )] ⋅
E[ y 2 (t )] ≠ 2 Hence, the correct Answer is (100).
9. Let X be a real–valued random variable with E[X] and
Hence, statement-1 is only correct E[X2] denoting the mean values of X and X2, respec-
Hence, the correct option is (A). tively. The relation which always holds
(a) (E[x])2 > E[x2]
7. A voice-grade AWGN (additive white Gaussian noise)
(b) E[x2] ≥ (E[x])2
telephone channel has a bandwidth of 4.0 kHz and
η (c) E[x2] = (E[x])2
two sided noise power spectral density = 2.5 × (d) E[x2] > (E[x])2 [2014]
2
10  Watt per Hz. If information at the rate of 52 kbps
–5
Solution: (b)
is to be transmitted over this channel with arbitrar-
Variance σ x2 = E ( x 2 ) − [ E ( x )]2
ily small bit error rate, then the minimum bit-energy
Eb(in mJ/bit) necessary is [2016] ∴σ2xion means be negative.
η ∴E(x2) ≥ [E(x)]2
= 2.5 × 10-5 Watt
Solution:  Power spectral density Hence, the correct option is (b)
2
N = 4.5 × 10–5 Watt/Hz 10. Consider a random process X (t ) = 2 sin (2π +φ ) ,
Bandwidth B = 4 KHz where the random phase φ is uniformly distributed in
the interval [0, 2π]. The autocorrelation E[X(t1) X(t2)] is
Bit rate Rb = 52 kbps
(a) cos(2π(t1 + t2))
As we know that the capacity of channel (b) sin(2π(t1− t2))
(c) sin(2π(t1 + t2))
C = Blog2 [1 + S/N] (d) cos(2π(t1− t2)) [2014]
S Solution: (d)
Eb = = 31.503
Rb Given X (t ) = 2 sin (2π +φ ) and φ is uniformly dis-
We know that tributed random variable in interval [0, 2π]
S 2π
= 8191
N E  X (t1 ) * X (t 2 )  = ∫ 2 sin (2tπθ )* 2 sin (2π t 2 +θ )fθ (θ )dθ
0
Chapter 2  Random Signals and Noise | 8.33

=
1 ) * X (t 2 )  ∫ 2 sin (2 tπθ  
)* 2 sin (2π t 2 +θ )fθ (θ )dθ Solution:
0 2π 1 / w f ≤w
1 Sx ( f ) = 

= 2 sin (2π t1 +θ )sin (2π t 2 +θ ) dθ
2π  0 f >w
0


1 1
=
2π ∫ sin (2π (t +t ) + 2θ )dθ + 2π
0
1 2

∫ cos (2π (t
0
1 − t 2 ) )dθ

2π ∴ Rx (τ ) ↔ S x ( f )
∫ sin (2π (t1 +t 2 ) + 2θ )dθ = 0 = s sin e [2 wτ]
   0 Also
as it is integration in one time period
  1    1 
1
2π E π × (t ). x  t −   = π E  x(t ). x  t − 4 w  

∫ cos (2 x(t − t ) )dθ
0
1 2 = cos (2π (t1 − t2 ) )   4 w    

 1‘  1
So , E  X (t1 ) * X (t 2 )  = cos (2π (t1 − t2 ) ) = π Rx   = π .2 sin e  
 4w  2
Hence, the correct option is (d)
11. The input to a 1-bit quantizer is a random variable X We know that
with PDF fx(x) = 2e–2x for x ≥ 0 and fx(x) = 0 for x < sin π x
sin e( x ) =
0. For outputs to be of equal probability, the quantizer πx

threshold should be ____. [2014]
From equation (1), we get
Solution:
sin π / 2
Assume quantization voltage is Vth for outputs of quan- ∴ 2π =4
tizer to have equal probability by → π /2
P(x ≤ Vth) = P(x ≥ Vth) P2
= E[cos( 2π (t1 − t 2 )] − cos[ 2π (t1 + t 2 + 2a)]
Vth d 2


−∞
f x ( x ) ax = ∫ f x ( x )dx
=
P2
2
cos[2π (t1 − t 2 )]
Vth
Vth ∞ 1 3. A real band-limited random process X(t) has two-
∫ 2e −2 x dy = ∫ 2e
−2 x
dx sided power spectral density
0 Vth
10 −6 (3000 − f Watts/Hz ) for t ≤ 3kHz
S x (f ) = 
1
e −2 xVth =   0 otherwise
  2
where f is the frequency expressed in Hz. The signal x(t)
    Vth = 0.346
modulates a carrier cos16000πt and the resultant signal
1 2. The power spectral density of a real stationary random is passed through an ideal band-pass filter of unity gain
process X(t) is given by with centre frequency of 8 kHz and band-width of 2
1 kHz. The output power (in watts) is _______. [2014]
 , f ≤W
S x (f ) = W Solution:
0 f >W
 10 −6 (3000 − f watts/Hz for f ≤ 3 kHz
Sx ( f ) = 
The value of the expectation    0 otherwise
  1 
E π x(t ) ⋅ x  t −   is _______. [2014]
  4W  
8.34 | Communication

⇒ 

PSD
2 fx(t ) ← → Sy(t )

Then
PSD Sx ( f − fc ) + Sx ( f + fc )
x(l ) cos 2π fet ← →
4
∴PSD of x(t) cos 2A × 80 wt

= 

Now, BPF is given by

⇒ 

We know that
2 × 
S x (f ) → H (f ) → H (f ) S x (f )

Solution:
∴ PSD of BPF output
x1 + x2 ≤ x3
x1 + x2 – x3 ≤ 0
As P[x1 + x2 – x3 ≤ 0] = P[y ≤ 0]
and y is another random variable whose probability
density function is given by convolution of x1, x2 and x3.
0
y2
So, P ( y ≤ 0) = ∫
−1
2
dy
Output power = total area of output PSD
0
  y2 
 2 × 10 −3  1 10 −3  P ( y ≤ 0) =  
= 2 ( 2000)   + 2  × 1000 ×    6  −1
 4

   2 4  
P(y tt 0) = 0.16
 1
= 2 1 +  = 2.5 watts 1 5. A binary random variable X takes the value of 1 with
 4 probability 1/3. X is input to a cascade of two independ-
14. Let X1, X2 and X3 be cdistributed random variables ent identical binary symmetric channels (BSCs) each
with the uniform distribution on [0, 1]. The probability with crossover probability 1/2. The output of BSCs is
P{X1, + X2 ≤ X3} is ______. [2014] the random variables Y1 and Y2 as shown in the figure.
Chapter 2  Random Signals and Noise | 8.35

− (γ −1)2
1
fγ ( y )= e 2σ 2
H1 σ 2π

1 − (γ +1)2
The value of H(Y1) + H(Y2) in bits is _____. [2014] and fγ ( y )= e 2σ 2
H0
σ 2π
Solution: Let yopt denote an optimal threshold, then probability of
P(x = 1) = 1/3 and P(x = 0) = 2/3 error for optimum threshold will be calculated as
1
So H ( y1 ) = P ( y1 ) log fγ ( y )
P( y) H1 P( H 0 )
=
1 fγ ( y ) P ( H1 )
H ( y2 ) = P ( y2 ) log H0 y = yopt
P ( y2 )  

where P(y = 1) = P(y1 = 0) −[(γ −1) −(γ +1)]2
2σ 2
P( H 0 )
1 1 2 1 1 e =
= × + × = P ( H1 )
3 2 3 2 2
yopt

Similarly +2 yopt 2
σ P( H 0 )
P(y2 = 1) = P(y2 = 0) = y2 e =
P ( H1 )

Here H ( y1 ) + H ( y2 ) = 1 log 2 2 + 1 log, 2 + 1 log 2 2 + 1 log 2 2
2 2 2 2 So, yopt can be given as
1 1 1 1 σ 2 P ( H 0 ) −σ 2
) + H ( y2 ) = log 2 2 + log, 2 + log 2 2 + log 2 2 yopt = ln = = −0.55σ 2 , here,
2 2 2 2 2 P ( H1 ) 2
yopt is less than zero, so negative.
H(y1) + H(yn) = 2 bits
Hence, the correct option is (c)
16. Consider a communication scheme where the binary
17. Consider the Z-channel given in the figure. The input is
values signal X satisfies P{X = 1} = 0.75 and P{X =
0 or 1 with equal probability.
−1} = 0.25. The received signal Y = X + Z, where Z is a
Gaussian random variable with zero mean and variance
σ2. The received signal Y is fed to the threshold detec-
tor. The output of the threshold detector X is
 +1 Y > τ
X =
−1 Y ≤ τ
To achieve a minimum probability of error
P{X ≠ X}, the threshold τ should be If the output is 0, the probability that the input is also 0
(a) strictly positive equals ___________. [2014]
(b) zero Solution:
(c) strictly negative
(d) strictly positive, zero, or strictly negative depend-
ing on the nonzero value of σ2 [2014]
Solution: (c)
Assume hypothesis H1 and H0 are given as
H1 : X = +1; H 0 : X = −1 and their probabilities P(H1)
= 0.75 ; P(H0) = 0.25
Let received sigma γ = X + Z where Z follow normal
− z2 Given P(x) = [0.5 0.5]
distribution as N (0, - 2); with f ( Z ) = 1 2σ 2
z e y1 y2
σ 2π
 y  x 1 0 
 1+ Z ifX = 1 P  = 1 
Then received signal γ =  x x2 0.25 0.75
 
−1 + Z ifX = −1
8.36 | Communication

Then 0 ∞
1 −0.5( y −a (1+ β ))2 1 −0.5( y +a (1+ β ))2
0.5
P[ x , y ] = 
0 

= 0.5 ∫
−∞

e dy + 0.5 ∫
−0

e dy = Q (a (1 + β ))

0.125 0.375

0 ∞
and P(y1) = 0.625 = 0.5 1 −0.5( y −a (1+ β ))2 1 −0.5( y +a (1+ β ))2

P(y2) = 0.375 −∞

e ∫ dy + 0.5
−0
∫ 2π
e dy = Q (a (1 + β ))
− a2
and the conditional probability matrix When β = 0, Pe = Q( a) = 1× 10 −8 = e 2 → a = 6.07
x − a2
P   is given by Q( a) = 1× 10 −8 = e 2 → a = 6.07
 y Pe =
y1 y2 β = –0.3, Pe = Q(6.07(1 − 0.3)) = Q( 4.249)
 x  x 0.8 0  − ( 4.249 )2
P  = 1  Pe = e 2 = 1.2 × 10 −4
y x2 0.33 1 
  −4
Pe ≈ 10
x  0
Hence P  1  = P   = 0.8 Hence, the correct option is (c)
 y1  0
19. Let U and V be two independent zero mean Gaussian
18. Consider a discrete-time channel Y = X + Z, where 1 1
the additive noise Z is signal-dependent. In particu- random variables of variances and respectively.
The probability 4 9
lar, given the transmitted symbol X ∈ {-a, +a} at any
instant, the noise sample Z is chosen independently P(3 V ≥ 2U) is
from a Gaussian distribution with mean βX and unit (a) 4/9 (b) 1/2
variance. Assume a threshold detector with zero thresh- (c) 2/3 (d) 5/9 [2013]
old at the receiver. When β = 0, the BER was found to Solution: (b)
be
P(3V – 2U) = P(3V – 2U ≥ 0)
Q(a) = 1 × 10-8
= P(W ≥ 0)
 ∞ 
 Q(v ) = 1 −u2 / 2 − v 2 / 2 
 2π v ∫
e du, and for v > 1. use Q(v ) = e


Where W = 3V – 2U
U = N(0, 1/4)


↓ ↓
∞ 
−u2 / 2 −v2 / 2 
∫ e du, and for v > 1. use Q(v ) = e

Mean Variance
V = N(0, 1/a)
v 
When β = −0.3, the BER is closest to W = 3V – 2U
(a) 10-7 (b) 10-6  1 1
(c) 10 -4
(d) 10-2 [2014] W = N  0, 0 × + 4 × 
 4 9

Solution: (c)
= N(0, 2.7)
Given X ∈ [-a,-a] and P(x = –a) = P(x = a) = 0.5
Since W is Gaussian variable with 0 mean,
Let γ = X + Z be the received signal and Z follows nor-
mal distribution as N(βX, 1) as
1 2 −a + Z if X = −a
f z (Z ) = e −( Z − β X ) , y = 
2π  a+Z f X = +a

H1 : X = + a; H 0 : X = −a , and threshold =0
1 2
fγ ( y )= e −0.5(γ − a(1+ β )) (P(w ≥ 0) = 1/2 = area for w ≥ 0].
H1 2π

Hence, the correct option is (b)
1 2
and fγ ( y )= e −0.5(γ + a(1+ β )) 20. Consider two identically zero-mean random variables
H0 2π U and V. Let the cumulative distribution functions of
Bit error rate is given by U and 2 V be F(x) and G(x), respectively. Then, for all
values of x
Pe = P ( H1 ) P (e / H1 ) + P ( H 0 ) P (e / H 0 )
Chapter 2  Random Signals and Noise | 8.37

(a) F(x) – G(x) ≤ 0


(b) F(x) – G(x) ≥ 0 + d
(c) (F(x) – G(x)) ∙ x ≤ 0 X(t) + Y(t)
+ dt
(d) (F(x) – G(x)) ∙ x ≥ 0 [2013]
Solution: (d) Delay = 0.5 ms

F(x) = P(x ≤ x)
Let SY(f) be the power spectral density of Y(t), which
G(x) = P(2x ≤ x)
one of the following statements is correct?
= P(x ≤ x/2) (a) SY(f) > 0 for all f
For position values of x (b) SY(f) = 0 for |f | > 1 kHz
F(x) – G(x) > 0. (c)  SY(f) = 0 for f = nf0, f0¬ = 2 kHz, n any integer
(d) SY(f) = 0 for f = (2n + 1) f0, f0 = 1 kHz, n any integer
For negative values of x
 [2010]
F(x) – G(x) < 0
Solution: (d)
but [F(x) – G(x)], X ≥ 0 for all values of x.
y1(t) = x(t) + x(t – 0.5 × 10−3)
Hence, the correct option is (d)
y1(f ) = x(f ) [1 + e−j2π (0.5 × 10−3)
21. X(t) is a stationary random process with autocorrela-
y1 ( f ) −3
tion function Rx(τ) = exp(−πτ2). This process is passed H1 ( f ) = = 1 + e − ejπ f ×10
through the system below. The power spectral density x( f )

of the output process Y(t) is
H(f) = H1(f ) . H2(f )

= j 2π f 1 + e − j (π f ×10 ) 
−3

 
Sy(f  ) = |H(f)|2Sx(f )
(a) (4π2f2 + 1) exp(−πf2) = 4π 2 t 2  2 + 2 cos(π f × 10 −3 )  S x ( f )
(b) (4π2f2− 1) exp(−πf2)  
(c) (4π2f2 + 1) exp(−πf) For f0 = 1 kHz and f = (2n + 1) f0
(d) (4π2f2− 1) exp(−πf) [2011] πf × 10−3 is odd multiple of π
Solution: (a) ∴ Sy(f) = 0
d Hence, the correct option is (d)
y (t ) = x (t ) − x (t )
  dt Common Data Questions 23 and 24
y( f ) = jwx( f ) − x( f ) Consider a baseband binary PAM receiver shown
below. The additive channel noise n(t) is white
= [j2 πf – 1] x(f)
with power spectral density SN(f ) = N0/2 = 10-20
 y( f )  W/Hz. The low-pass filter is ideal with unity gain and
 H( f ) =  cut-off frequency 1 MHz. Let Yk represent the random
x( f ) 
 variable y(tk).
PSD = |H(f)|2 . Sx(f) Yk = Nk if transmitted bit bk = 0
=| ( j 2π f − 1) |2 .S x ( f ) Yk = a + Nk if transmitted bit b = 1

2 where Nk represents the noise sample value. The noise
S ( f ) = FT {Rx (τ )} = e −π t sample has noise probability density function, pNk (n)
x
= 0.5αe −α|n| (This has mean zero and variance 2/α2.)
Since Fourier of Gaussian is also Fourier,
Assume transmitted bits to be equiprobable and thresh-
2
S y ( f ) = [ yπ 2 f 2 + 1].e −π f old z is set to a/2 = 10-6V.

n(t)
Hence, the correct option is (a)
Y(t) Y(l) b, if y(t)> z
x(t) + r(t) LPF
22. X(t) is a station process with the power spectral density S/H F
b, if y(t)< z
SX(f ) > 0 for all f. The process is passed through a sys- Sampling Threshold
tem shown below. time z
Receiver
8.38 | Communication

23. The value of the parameter α (in V-1) is P (0 /1) + P (1/ 0)


(a) 1010 (b) 107 = = 0.5 × 1010
2
(c) 1.414 × 10-10 (d) 2 × 10-20 [2010] Hence, the correct option is (d)
Solution: (b) 25. Consider two independent random variables X and Y
SN0(f) = SN(f) |H(f)|2 with identical distributions. The variables X and Y take
SN0(f) = 10−20 |H(f)|2 1
values 0, 1 and 2 with probabilities 1 , , and
1
∞ 2 4 4
Output noise power = ∫S
−∞
N0( f )df respectively. What is the conditional probability P(X +

w Y = 2|X – Y = 0)?
= 10 −20 × 2 × 106 Hz
H2 1
(a) 0 (b)
= 2 × 10−14 w 16
1
Power = mean square value (c) (d) 1 [2009]
6
[α/α2 = 2 × 10−14)
α = 107 Solution: (c)
Hence, the correct option is (b) ( P * x + y = 2, x − y = 0)
P ( x + y = 2 / x − y = 0) =
24. The probability of bit error is  
P ( x − y = 0)
(a) 0.5 × e–3.5 (b) 0.5 × e–5 1
(c) 0.5 × e –7
(d) 0.5 × e–10 [2010] P ( x + y = 2, x − y = 0) = P ( x = 1, y = 1) =
16
Solution: (d)
P(x – y = 0) = P(x = 0, y = 0) + P(x = 1, y = 1)
When a → 1 + P(x = 2, y = 2)
rk = a + Nk = P(x = 0) .P(y = 0) + P(x = 1) P(y = 1) + P(x = 2) P(y
Threshold z = 1/2 = 10−6 = 2) = 3/8
a = 2 × 10−6 1 8 1
P ( x + y = 2 / x − y = 0) = × =
for error to occur, yk < 10−6 16 3 16
2 × 10−6 + Nk < 10−6 Hence, the correct option is (c)
Nk< −10−6 26. A discrete random variable X takes values from 1 to 5
−10 −6 with probabilities as shown in the table. A student cal-
P (0 /1) = ∫
−∞
PNK ( n)dn culates the mean X as 3.5 and her teacher calculates the
variance of X as 1.5. Which of the following statements

−10 −6
is true?
  

−∞
0.5α .eα n dn; α = 107
K 1 2 3 4 5
P(X = K) 0.1 0.2 0.4 0.2 0.1
P(0/1) = 0.5e−10
When a '0' is transmitted (a) Both the student and the teacher are right
(b) Both the student and the teacher are wrong
yk = Nk
(c) The student is wrong but the teacher is right
For error to occur (d) The student is right and but the teacher is wrong
yk > 10−6  [2009]

Solution: (b)
∫ PNK ( n)dn = 0.5 × 10 −6

P (1/ 0) =
Mean x = Σ xi P ( xi )
10 −6
= 1 × 0.1 + 2 × 0.2 + 3 × 0.4 + 4 × 0.2 + 5 × 0.
Since both bits are equiprobable,
= 3
P(0) = P(1) = 1/2
The probability of bit error σ2 variance = x 2 − x 2 = Σxi2 p( xi ) − (Σ xi P ( xi )) 2
= P(1) . P(0/1) + P(0) . P(1/0) σ2 = [(1 × 0.1) + 4 × 0.2 + 9 × 0.4 + 16 × 0.2 × 25 ×
0.1)] – (3)2
CDF
1

Chapter 2  Random Signals and Noise | 8.39


−1 0 1 X
[σ2 = 1.2] (d)
CDF
Hence, the correct option is (b)
27. The Probability Density Function (PDF) of a random 1
variable X is as shown below. [2008]
PDF
1 PDF
1 PDF
1
PDF 1
1
−1 0 X

−1 0 1 X −1
−1 0 1 X
−1 0 1 X
−1
The corresponding Cumulative
0 Distribution
1 X Fun­ction

Solution: (a)
(CDF) has the form Density function is given as
(a) fx(x) = (t + 1)u(t + 1) – 2tu(t) + (t – 1)u(t – 1)
PDF
PDF CDF function is calculated as integration of density
1 PDF
1 function.
1
PDF x
1
−∞

Fx ( x ) = f x ( x )dx

integral of increasing ramp signed is increasing parab-


ola and integral of decreasing ramp signal is decreasing
−1 0 10 X parabola.
−1 1 X
−1 0 1 X Hence, the correct option is (a)
CDF
−1
1
0 CDF
CDF
1 X 28. Px ( x ) = M exp ( −2 x ) + N exp ( −3 x ) is the prob-
(b) 1 ability density function for the real random variable X
1
CDF over the entire x-axis. M and N are both positive real
1 numbers. The equation relating M and N is
2 1
(a) M + N = 1 (b) 2M + N = 1
3 3
(c) M + N = 1 (d) M + N = 3 [2008]
−1 0 10 X Solution: (a)
−1 1 X
−1 0 1 X PDF is given as
−1 0 1 X Px(x) = m exp (−2|x|) + N exp (−3|x|)

(c)

∫ P ( x)dx = 1
−∞
x

CDF
CDF ∞

∫ {me }
1 CDF −2| x|
1 + Ne −3| x| dx = 1
1
CDF −∞
1 ∞
1
∫ (me
−2 x
+ Ne −3 x )dx =
2
0
m N 1
+ =
−1 0 10 X 2 3 2
−1 1 X
−1 0 1 X 2N
CDF 0
m+ =1
−1 CDF 1 X 3
CDF
1
1 Hence, the correct option is (a)
1 CDF

1
1
−1 0 1
−1 0 X
− X
8.40 | Communication

29. Noise with double-sided power spectral density of K (c) tuned L-C filter
over all frequencies is passed through a RC low pass (d) series R-L-C filter [2006]
filter with 3 dB cut-off frequency of fc. The noise power Solution: (a)
at the filter output is
Given
(a) K (b) Kfc
(c) Kπfc (d) ∞ [2008] 16
S (w) =
Solution: (c) 16 + w 2
Filter transfer function is given by 4
H ( s) =
1 1 4+s
H( f ) = =
1 + j 2π fRc 1 + jf/f c This is a low pass filter of RL type.
fC2 Hence, the correct option is (a)
Output PSD = |H(f  )|2 . input PSD = .K 32. The parameters of the system obtained in the above
f 2 + fC2
question would be

(a) first order R-L low pass filter would have
Output noise power = ∫ (o/p PSD).df
−∞
R=4Ω L=1H
(b) first order R-C high pass filter would have

fc 2 R = 4 Ω C = 0.25F
=k
−∞
∫ fc 2 + f 2
∂f
(c) tuned L-C filter would have L = 4 H
C=4F
= kπfc (d) series R-L-C low pass filter would have
(By substitution f = fc tan Q) R = 1Ω , L = 4H, C = 4F [2006]
Hence, the correct option is (c) Solution: (a)
30. A zero-mean white Gaussian noise is passed For given network
through an ideal low-pass filter of bandwidth L
10 kHz. The output is then uniformly sampled with
sampling period ts = 0.03 msec. The samples so
obtained would be
(a) correlated R
(b) statistically independent
(c) uncorrelated
(d) orthogonal [2006]
Solution: (b) R
H ( jw ) =
If white noise is sampled, no matter how closely in R + jwL

time the samples are taken, they are uncorrected. If
white noise is Gaussian then samples are statically So R = 4, L = 1
independent. Hence, the correct option is (a)
Hence, the correct option is (b) 33. A uniformly distributed random variable X with prob-
Common Data for Questions 31 and 32 ability density function
1
The following two questions refer to wide sense sta-
tionary stochastic processes.
f x (x ) =
10
(
u ( x + 5) − u ( x − 5) )

31. It is desired to generate a stochastic process (as voltage where u(.) in the unit step function is passed through a
process) with power spectral density transformation given in the figure below. The probabil-
16 ity density function of the transformed random variable
s(ω ) =
16 + ω 2 Y would be

By driving a linear-time-invariant system by zero y
mean white noise (as voltage process) with power spec-
tral density being constant equal to 1, the system which
can perform the desired task could be 1
(a) first order low pass R-L filter
(b) first order high pass R-C filter
x
2.5 2.5
Chapter 2  Random Signals and Noise | 8.41

1 Area = 1
(a) f y (y ) =
5
(
u ( y + 2.5 ) − u ( y − 2.5 ) ) k×4
=1
f y (y ) = 0.5δ(y ) + 0.5δ(y − 1) 2
(b)
f y (y ) = 0.25δ(y + 2.5) + 0.25δ k = 1/2
(c) 4
(y − 2.5) + 0.5δ( y )

E[ x ] = x 2 f x ( x )dx
2

f y (y ) = 0.25d (y + 2.5) + 0.25d (y − 2.5)


0
(d)
1
+ ( )
u ( y + 2.5 ) − u ( y − 2.5 ) 4
x x4
4
10

2
= x   dx =
[2006] 8 4×8
0
0
Solution: (c)
E[x2] = 8
Sample space of random variable x = (-∞, ∞)after
transformation sample space of random variable Hence, the correct option is (c)
y = (0, 1) Common Data for Questions 36 and 37.
Hence fx(y) = Aδ(y) + p[δ(y – 1)] A symmetric three-level midtread quantizer is to be
Hence, the correct option is (c) designed assuming equiprobable occurrence of all
quantization levels.
34. Noise with uniform power spectral density of N0 W/
Hz is passed through a filter H(ω) = 2exp(-jωtd) 36. If the probability density function is divided into three
followed by an ideal low pass filter of bandwidth regions as shown in the figure, the value of a in the
B Hz. The output noise power in watts is figure is
(a) 2 N0B (b) 4 N0B 1
(c) 8 N0B (d) 16 N0B [2005] 4
Region1 1
Solution: (b) 8
Region3
Output noise power is calculated by calculating output Region2
x
power spectral density as −3 −1 −a +a +1 +3
Sy = |H(w)|2Sx
SNout = |H(w)|2 SNinput (a) 1/3 (b) 2/3
= 4N0 (c) 1/2 (d) 1/4 [2005]
PN = B.W. × SNout = 4N0B. Solution: (b)
Hence, the correct option is (b) Probability of occurrence of each region is
a
35. An output of a communication channel is a random
variable v with the probability density function as ∫ f ( x)dx = 1/3
−1
x
shown in the figure. The mean square value of v is
a
1 1 2a 1
4 ∫ xdx = 3 ⇒
−a
=
4 3

[a = 2/3]
3 7. The quantization noise power for the quantization
region between –a and +a in the figure is
4 1
(a) (b)
81 9
(a) 4 (b) 6 5 2
(c) 8 (d) 9 [2005] (c) (d)  [2005]
81 81
Solution: (c) Solution: (a)
Probability density function can be written as Noise power =
fx(x) = y = mx + c = x/8 a a
1
Total probability = 1 E[ x 2 ] = ∫
−a

x 2 f x ( x )dx = 2 x 2 . .dx
4
0
8.42 | Communication

2/3 2/3 40. Let X and Y be two statistically independent random


1 2 1  x3 
∫ variables uniformly distributed in the ranges (–1, 1)
2
E[ x ] = 2 x dx =  
0
4 2  3 
0
and (–2, 1) respectively. Let Z = X + Y. Then the prob-
ability that (Z ≤ -2) is
1 8 4
E[ x 2 ] = × = (a) zero (b)
1
6 27 81
6
38. A 1 mW video signal having a bandwidth of
100 MHz is transmitted to a receiver through a cable that 1 1
(c) (d)  [2003]
has 40 dB loss. If the effective one-sided noise spectral 3 12
density at the receiver is 10-20 watt/Hz, then signal-to- Solution: (d)
noise ratio of the receiver is
(a) 50 dB (b) 30 dB
(c) 40 dB (d) 60 dB [2004]
Solution: (a)
PS 10 −3
SNR = = −20 = 109
N 0 B 10 × 100 × 106 Area under curve fx(x) = 1

(SNR)dB = 10 log10 (SNR) = 10 log10 (109) = 90 dB 2 × k = 1 = k = 1/2
Cable loss = 40 dB
SNR at x = (90 – 40)dB = 50 dB
Hence, the correct option is (a)
39. A random variable X with uniform density in the inter-
val 0 to 1 is quantized as follows:
3 × k’ = 1 ,k’ = 1/3
If 0 ≤ X ≤ 0.3, xq = 0
If 0 .3 < X ≤ 1, xq = 0.7
Where xq is the quantized value of X. The root-mean
square value of the quantized noise is
(a) 0.573 (b) 0.198
(c) 2.205 (d) 0.266 [2004] k” = 1/6
Solution: (b) Since it lies in mid
P{2 ≤ −2] = area of graph [2 ≤ −2]
1 1 1
= × ×1 =
2 6 12
Common Data for Questions 2.32 and 2.33.
Quantization error = Sampled Value – Quantized value Let X be the Gaussian variable obtained by sampling
= x – xq the process at t = ti and let
Mean square value of quantization Noise ∞ x2
1
= E[(x – xq)2]
1

Q(– ) = −
2
e2 dy


= ( x − xq ) 2 f x ( x )dx
0
mean = 0
( )
Auto correlation function R xx (t ) = 4 e −0.2 t + 1 and
1 0.3 1


= ( x − xq ) 2 /dx = ∫ ( x − 0) dx + ∫ ( x − 0.7)
2 2 Hence, the correct option is (d)
0 0 0.3 41. The probability that [x ≤ 1] is
(a) 1 – Q(0.5) (b) Q(0.5)
dx = 0.03
Root mean square value of quantization noise  1   1 
(c)
Q 1− Q
 (d)   [2003]
=
=0.039 0.198  2 2  2 2
Hence, the correct option is (b)
Chapter 2  Random Signals and Noise | 8.43

Solution: (d) (a) a2Sg(ω) and a2Pg (b)


a2Sg(ω) and aPg
Probability function is given by (c) aSg(ω) and a2Pg (d) aSg(ω) and aPg
 [2001]
 x−µ 
P(x ≤ 1) = Fx(1) = 1 − Q   Solution: (a)
 a  x =1
Here μ = 0 We know that
Rx(0) = σ2 S ( w ) --- H ( w ) --- S y ( w )
x
σ2 = 8
Sy(w) = |H(w)|2Sx(w)
⇒ σ =2 2
Hence, the correct option is (a)
 1 
∴P(x ≤ 1) = 1 − Q   45. The power spectral density of a deterministic signal is
2 2 2
given by sin(f )/(f )  , where ‘f’ is frequency. The auto-
Hence, the correct option is (d)
correlation function of this signal in the time domain is
42. Let Y and Z be the random variables obtained by sam- (a) a rectangular pulse
pling X(t) at t = 2 and t =4, respectively. Let W = Y − Z. (b) a delta function
The variance of W is (c) a sine pulse
(a) 13.36 (b) 9.36 (d) a triangular pulse [1997]
(c) 2.64 (d) 8.00 [2003]
Solution: (d)
Solution: (c)
Auto-correlation function and power spectral density
Variance can be given by make the Fourier transform pair
w
σ 2 = E ( y − z )2 ] .T .
Rx (τ ) F → Gx ( w )

σ2w=E(y2)+E(z2)-2E(y.z)
  f 
σ2w=σ2y+a2z-2Rxx(2) Rx (τ ) == F −1 sin C 2   
  π 
+8 + 8 – 2[ye−0.2121 + 1]
σ2w=2.64 Since we know that Fourier transform of triangular
signal is a square of sin C function, so inverse Fourier
Hence, the correct option is (c)
transform will be a triangular signal.
43. If the variance σ2x of d(n) = x(n)–x(n – 1) is one-tenth
Hence, the correct option is (d)
the variance and σ2x of a stationary zero-mean discrete-
time signal x(n), then the normalized autocorrelation 46. The auto-correlation function of an energy signal has
function Rxx ( k ) / σ x2 at k = 1 is (a) no symmetry
(a) 0.95 (b) 0.90 (b) conjugate symmetry
(c) 0.10 (d) 0.05 [2002] (c) odd symmetry
(d) even symmetry [1996]
Solution: (a)
Solution: (b, d)
The variance for the signal is given by
Auto-correlation function has property of conjugate
= σ x2 E{[x(n) – x(n – 1)]2} symmetric and even symmetric for energy signal.
= E[x2(n)] + E[x2(n – 1)] – 2E[x(n) η(n – 1)] [Rx(τ) = Rx(−τ)] even symmetry
ax 2 [Rx(τ) = Rx*(−τ)] conjugate symmetry
= = ax 2 + ax 2 − 2 Rxx (1)
10 Hence, the correct option is (b, d)
19 2 47. For a random variable ‘X’ following the probability
2 Rxx (1) = σx density function, p(x), shown in figure, the mean and
10
the variance are, respectively
 Rxxx (1) 19 
 2
= = 0.95
 σx
20 
Hence, the correct option is (a)
44. The PSD and the power of a signal g(t) are, respec-
tively, Sg(ω) and Pg. The PSD and the power of the sig-
nal ag(t) are, respectively,
8.44 | Communication

(a) 1/2 and 2/3 (b) 1 and 4/3 y


(c) 1 and 2/3 (d) 2 and 4/3 [1992] Let that =t
N
Solution: (b)
dy = N dt
We know that mean is given as
∞ ∞ −t 2


1
µ x = E ( x) = x Px ( x ) dx
∫ Nt
2 2
E[ y ] = e 2 N dt
−∞ 2π N
0
3
1 ∞ −t 2
= ∫ x
4
dx
=
N
∫t
2
e 2 dt
−1 2π
  0
3
1 x 2  N
=   E[ y 2 ] =
4  2  −1 2

Hence, the correct option is (b)
[μx = 1]
49. Events A and B are mutually exclusive and have
Variance is given as
a nonzero probability. Which of the following

statement(s) are true?
σ x 2 = E[( x − µ ) 2 ] = ∫ (x − µ )
2
n Px ( x )dx P (A ∪ B) = P (A) + P ( B )
(a)
−∞
P (BC ) > P ( A)
3 3 (b)
1 1  x2 2x2  P (A ∩ B) = P ( A) P ( B)

2 2 (c)
sx = ( x + 1 − 2 x )dx =  + x − 
4 4  3 2  P (BC ) < P ( A)
−2 −1
(d)  [1988]
 2 4 Solution: (a)
σ x = 3  Since events P ∩ B) = Pexclusive,
( A) + P ( Bso
) + P ( A ∩ B)
  are
( Amutually
P ( A ∩ B ) = P ( A ) + P ( B ) + P ( A ∩ B )
Hence, the correct option is (b)
48. Zero mean Gaussian noise of variance N is applied to a [ P ( A ∩ B ) = P ( A) + P ( B)]
half wave rectifier. The mean squared value of the recti-
Hence, the correct option is (a)
fier output will be
(a) Zero (b) N/2 50. The variance of a random variable X is σ2X. Then the
(c) (d) N [1989] variance of –kx (where k is a positive constant) is
N z
(a) σ2X (b) –kσ2X
Solution: (b)
(c) kσ X (d)
2
k2σ2X  [1987]
Half wave rectifier is given as
Solution: (d)
y = x for x ≥ 0
Given that variance of =var (x) = σ2X
= 0 for x< 0
Since var (x) = E(x2)
So f ( y ) =
1
d ( y) +
1
e
( − y2/ 2 N ) So var(−kx) = E((−kx)2)
2 2p N a2 = E[k2x2]

a2 = k2E[x2]
E( y2 ) = ∫y
2
+ ( y )dy
a2 = k2σ2x
0
Hence, the correct option is (d)
∞  − y2  51. White Gaussian noise is passed through a linear narrow
 1 1  dy

2
= y δ ( y) + e 2N band filter. The probability density function of the enve-
2 2π N 
0   lope of the noise at the filter output is
(a) Uniform (b) Poisson
∞ − y2 (c) Gaussian (d) Rayleigh [1987]
y2
= 0+ ∫ 2π N
e 2 N dy Solution: (d)
We know that narrow band re-presentation of Noise is
0
n(t) = nc(t) cos wct – ns(t) sin wct
envelope for the given signal will be
Chapter 2  Random Signals and Noise | 8.45

The cross correlation between input process x(t) and


R(t ) = nc2 (t ) + ns2 (t ) output process y(t) is

Where nc(t) and ns(t) are two independent zero mean Rxy (t ) = h(t ) * Rxx (t )
Gaussian processes with same variance, resulting enve-
S xy (w ) = H (w ), S xx (w )
lope will be Rayleigh variable.
Hence, the correct option is (d) S (w ) = S xy ( −w ) = S xy ( −w ).H ( −w )
yx
S (w ) = H * (w )S xx (w )
yx
Five-marks Questions
R (t ) = h( −t ) * Rxx (t )
yx
1. White Gaussian noise with zero mean and double-sided Autocorrelation function of x(t) i.e. white noise is an
power spectral density h/2 is the input x(t) to a linear Impulse function.
system with impulse response h(t) = exp(-t/RC)u(t).
h
The output is y(t). Evaluate E[x(t + t) y(t)] for t > 0. Rxx (t ) = d (t )
(Note: u(t) is the unit step function) [1988] 2
Solution:  t 
h( −t ) = exp   .u( −t )
 RC ) 

h( −t ) is defined for t = ( −∞, 0)
h  t 
Rxx (t ) * h( −t ) = exp   .u( −t )
E[ x(t − t ). y(t )] = Rxy (t ) 2  RC 


Rxx (t ) = 0 for τ > 0
Chapter 3
Digital Communication P(f)
P(f)

Systems
1
1
f (kHz)
f (kHz) −1.2 −0.8 0 0.8 1.2
−1.2 0 1.2

P(f) (D) P(f)


One-mark Questions
1 1

1. Which one of the following statements about differen-


f (kHz) f (kHz)
−1.2 −1 −1.2 0 1.2
tial pulse code modulation (DPCM) is0true? 1 1.2
 [2017] Solution:  For a pulse, which is free from inter symbol
(A) The sum of message signal sample with its predic- interference (ISI), If P(t) is having spectrum P(f)
tion is quantized

(B) The message signal sample is directly quantized,
and its prediction is not used
Then ∑
K =−∞
p( f − kRs ) = constant
(C) The difference of message signal sample and a
random signal is quantized Rs = 2 KSpa
(D) The difference of message signal sample with its This condition is met by pulse given in option B.
prediction is quantized Hence, the correct option is (B).
Solutions:  Unlike in PCM, to conserve the channel 3. Which one of the following statements about dif-
bandwidth, the difference of message signal sample ferential pulse code modulation (DPCM) is true?
with its prediction is quantized in DPCM.  [2017]
Hence, the correct option is (D). (A) The sum of message signal sample with its predic-
2. In a digital communication system, the overall pulse tion is quantized.
shape p(t) at the receiver before the sampler has the (B) The message signal sample is directly quantized,
Fourier transform P(f). If the symbols are transmitted and its prediction is not used.
at the rate of 2000 symbols per second, for which of (C) The difference of message signal sample and a
the following cases is the inter symbol interference 0? random signal is quantized.
 [2017] (D) The difference of message signal sample with its
P(f)
(A) P(f) prediction is quantized.
1
1 Solution:  Unlike in PCM, to conserve the channel
P(f) bandwidth, the difference of message signal sample
P(f) f (kHz)
f (kHz) −1.2 −0.8 10 0.8 1.2
−1.2 10 1.2 with its prediction is quantized in DPCM.
(B) P(f) f (kHz)
Hence, the correct option is (D).
P(f) f (kHz) −1.2 −0.8 0 0.8 1.2
−1.2 01 1.2 4. Consider a wireless communication link between a
1
P(f) P(f) transmitter and a receiver located in free space, with
f (kHz) 1
f (kHz) −1.2 −0.81 0 0.8 1.2 finite and strictly positive capacity. If the effective areas
−1.2 0 1.2
P(f) of the transmitter and the receiver antennas, and the
(C) P(f) f (kHz)
−1.2 −1 10
f (kHz) −1.2 10 distance
1.2 between them are all doubled, and everything
1 1.2
else remains unchanged, the maximum capacity of the
P(f) P(f) f (kHz) wireless
f (kHz) link  [2017]
−1.2 −1 −1.2 0 1.2
0 111.2
1

f (kHz) f (kHz)
−1.2 −1 −1.2 0 1.2
0 1 1.2
Chapter 3  Digital Communication Systems | 8.47

(A) increases by a factor of 2. 7. A discrete memory-less source has an alphabet {a1, a2,
(B) decreases by a factor of 2. ⎧ 1 1 1 1⎫
(C) remains unchanged. a3, a4} with corresponding probabilities ⎨ , , , ⎬.
(D) decreases by a factor of 2. ⎩ 2 4 8 8⎭
The minimum required average code word length in
2
P  λ  bits to represent this source for error-free reconstruc-
Solution: R =  DD
PT  4π R  tion is ____. [2016]
PR → Received power Solution:  The discrete memory-less source has
PT → Transmitted power ­following probabilities:
Dt → Transmitting antenna directivity Pa1 = 1/2, Pa2 = 1/4, Pa3 = 1/8, Pa4 = 1/8
Dr → Receiving antenna directivity
l → wave length The entropy can be expressed as
4K 1
d= Ae , H = Pa1 log2 + Pa2 log2 1
λ2 pa1 pa2

where Ae → effective area
1 1
P + Pa3 log2 + Pa4 log2 .
1 pa3 pa4
∴ R = 2 2 Aet Aer
PT λ R
P 1 1 1 1
Given that Aet, Aer and R each is doubled. Then, R = log 2 + log24 + log28 + log28
remains constant PT 2 2 4 8 8

Hence, the correct option is (C). H = 1.75 bits per word


5. A sinusoidal message signal is converted to a PCM Hence, the correct Answer is (1.75).
signal using a uniform quantizer. The required signal-
to-quantization noise ratio (SQNR) at the output of 8. A speech signal is sampled at 8 kHz and encoded
the quantizer is 40 dB. The minimum number of bits into PCM format using 8 bits/sample. The PCM data
per sample needed to achieve the desired SQNR is is transmitted through a baseband channel via 4-level
_______. [2017] PAM. The minimum bandwidth (in kHz) required for
transmission is _____. [2016]
Solution:  For the sinusoidal input
Solution:  Speech signal frequency Fs = 8khz,
SQNR = 6n + 1.8
No. of levels M = 4
(n : no. of bits/ sample)
No. of bits = n = 8
40 ≤ 6n + 1.8 M = 2n then n = Log2M
38.2 and Bit rate rb = n ⋅ Fs
n≤ =7
6 Bit rate = sampling rate × no. bits per sample
Hence, the correct answer is (7). = 8k samples/sec × 8 bits/sample
6. Consider binary data transmission at a rate of 56 kbps rb = 64 kbps
using base band binary pulse amplitude modulation
(PAM) that is designed to have a raised cosine spec- rb
(BW)min =
trum. The transmission bandwidth (in kHz) required 2 log 2 M

for a roll off factor of 0.25 is _____. [2016]
M = no of level PAM.
Solution:  Data transmission rate Rb = 56 kpbs,
64
Roll off factor α = 0.25. (BW)min =
2 log 2 4
The transmission bandwidth (in kHz) 
Rb (BW)min = 16 kHz
BW = [1 + α]
2 Hence, the correct Answer is (16).
56 9. An analog voltage in the range 0 to 8 V is divided in
= [1 + 0.25] kHz
2 16 equal intervals for conversion to 4-bit digital out-
put. The maximum quantization error (in V) is ______.
=
35 kHz.
 [2014]
Hence, the correct Answer is (35).
8.48 | Communication

Solution: independent and equally probable. If we transmit two


The maximum quantization error quantized samples per second, the information rate is
(a) 1 bit/sec (b) 2 bit/sec
Q =∆ (c) 3 bit/sec (d) 4 bit/sec [2011]
e
2
Solution: (d)
A = Dynamic range
2 n or L Quantization levels are equiprobable hence

H = log24 = 2 bits/sample
∆= 8
24 r = 2 sample/sec

Hence information rate = rH
8
∆= = 0.5 R=2×2
16
= 4 bits/sec
0.5
Qe = Hence, the correct option is (d)
2
= 0.25 13. Consider the pulse shape s(t) as shown. The impulse
response h(t) of the filter matched to this pulse is
10. The bit rate of digital communication system is R
 [2010]
Kbits/s. The modulation used is 32-QAM. The mini-
mum bandwidth required for |s| free transmission is
(a) R/10 Hz (b) R/10 kHz
(c) R/5 Hz (d) R/5 kHz  [2013]
Solution: (b)
2n = m
n = log2 m = log232 = 5
Rb R
Rb′ = = kbps
n 5
R ′ R (a)
( BW ) min = b = kHz
2 10
Hence, the correct option is (b)
11. In a baseband communications link, frequencies up to
3500 Hz are used for signalling. Using a raised cosine
pulse with 75% excess bandwidth and for no inter-
symbol interference, the maximum possible signalling
rate in symbols per second is
(a) 1750 (b) 2625
(c) 4000 (d) 5250 [2012]
Solution: (d) (b)
Frequency used for signalling is f = 3500 Hz
Excess BW used in B = 0.75 × f = 2625
We know that if B is the BW available, then
Rb
≤B
2 Rb
That is, if PD is data rate then 2 is the minimum BW (c)
required for transmission.
Rb ≤ 2B
Rb ≤ 5250
(Rb)max = 5250 symbol/sec
Hence, the correct option is (d)
12. An analog signal is band-limited to 4kHz, sampled
at the Nyquist rate and the samples are quantized
into 4 levels. The quantized levels are assumed to be
Chapter 3  Digital Communication Systems | 8.49

(d) Solution: (c)


SNRQ α 22h
α 4n
as n = 2
So SNRQ increases by 16.
Hence, the correct option is (c)
16. In the output of a DM speech encoder, the consecutive
pulses are of opposite polarity during time interval t1 ≤
t ≤ t2. This indicates that during this interval
Solution: (c)
(a) the input to the modulator is essentially constant
(b) the modulator is going through slope overload
(c) the accumulator is in saturation
(d) the speech signal is being sampled at the Nyquist
rate [2004]
Solution: (a)
In between two adjacent sampled value, if the base
band signal changes an amount less than step size the
output of the DM is a sequence of alternative positive
and negative pulses. This small change in base band
signal indicated that the base band is almost constant.
Impulse response of matched filter Hence, the correct option is (a)
17. At a given probability of error, binary coherent FSK is
inferior to binary coherent PSK by
(a) 6 dB (b) 3 dB
(c) 2 dB (d) 0 dB [2003]
Solution: (b)
For given probability of error, binary coherent FSK is
Hence, the correct option is (c) inferior to binary coherent PSK by 3dB.
14. Refractive index of glass is 1.5. Find the wavelength Hence, the correct option is (b)
of a beam of light with frequency of 1014 Hz in glass. 18. Consider a sample signal
Assume velocity of light is 3 × 108 m/s in vacuum

+∞
y (t ) = 5 × 10−6 x (t ) δ(t − nTs ) where
(a) 3 mm (b) 3 mm n = −∞
(c) 2 mm (d) 1 mm [2005] x(t) = 10 cos (8π × 103) t and Ts =100µsec
Solution: (c) When y(t) is passed through an ideal low-pass filter
In vacuum C = fλ with a cutoff frequency of 5 kHz, the output of the filter
C = 3 × 108 is
f = 1014 (a) 5 × 10-6 cos(8π × 103)t
3 × 108 (b) 5 × 10-5cos(8π × 103)t
λ= (c) 5 × 10-1cos(8π × 103)t
1014
(d) 10cos(8π × 103) [2002]
λ = 3 × 10−6m
Solution: (c)
λ 3 × 10−6
In class λg = = ∞


h
= 2μm
1.5
y (t ) = 5 × 10−6 x (t ) ∑ δ (t − nT )
s
n =−∞
Hence, the correct option is (c)
15. In a PCM system, if the code word length is increased  1 2 
= 5 × 10 −6 x(t )  + (cos ws t + cos 2ws t + .....) 
from 6 to 8 bits, the signal to quantization noise ratio
 Ts Ts 
improves by the factor
(a) 8/6 (b) 12  x(t ) 2 
= 5 × 10 −6  + ( x(t ) cos ws t + x(t ) cos 2ws t + ...) 
(c) 16 (d) 8 [2004]  Ts Ts 

8.50 | Communication

x (t ) 2  Solution: (a)
+ ( x(t ) cos ws t + x(t ) cos 2ws t + ...)  Flat top sampling of low pass signals gives rise to aper-
Ts Ts  ture effect.
Given
Hence, the correct option is (a)
cut off frequency of low pass fitter = 5 kHz
22. Quadrature multiplexing is
Ts = 100 ×10-6
(a) the same as FDM
1 (b) the same as TDM
fs = × 106 = 10 kHz
100 (c) a combination of FDM and TDM
θπ × 10 (d) quite different from FDM and TDM
fm = = 4 Hz
2π [1998]
Only first component gets passed, other will be sup- Solution: (d)
pressed because = fs – fm1 fs + fm12fs – fm, 2fs + fm> 5 Hz
Quadrature carrier multiplexing utilizes carrier phase
5 × 10 −6 × (t ) 5 × 10 −6 × 10 cos(8π × 103 ) shifting and synchronous detection to permit two DSB
y (t ) = = signals to occupy the same frequency band. It is the
Ts 100 × 10 −6
scheme where same carrier frequency is used for two
= 5 × 10−1 cos (2π × 103). different DSB signals. It is also known as QAM. So,
Hence, the correct option is (c) quadrature multiflexing is quite different from FDM
19. For a bit-rate of 8 Kbps, the best possible values of the and TDM.
transmitted frequencies in a coherent binary FSK sys- Hence, the correct option is (d)
tem are 23. Compression in PCM refers to relative compression of
(a) 16 kHz and 20 kHz (a) higher signal amplitudes
(b) 20 kHz and 32 kHz (b) lower signal amplitudes
(c) 20 kHz and 40 kHz (c) lower signal frequencies
(d) 32 kHz and 40 kHz [2002] (d) higher signal frequencies [1998]
Solution: (d)
Solution: (a)
For fsk
Compression is PCM refer to relative compression of
f1 = nfb higher signal frequencies.
f2 = mfb Hence, the correct option is (a)
So f1 = 4 × 8 = 32 kHz 24. The line code that has zero DC component for pulse
f2 = 5 × 8 = 40 kHz transmission of random binary data is
Hence, the correct option is (d) (a) more-return to zero (NRZ)
20. In a PCM system with uniform quantization, increasing (b) return to zero (RZ)
the number of bits from 8 to 9 will reduce the quantiza- (c) alternate mark inversion (AMI)
tion noise power by a factor of (d) none of the above [1997]
(a) 9 (b) 8 Solution: (c)
(c) 4 (d) 2 [1998] Alternate mark inversion (AMI) code has zero DC
Solution: (c) component for pulse transmission of random binary
n data.
SNRQ α 22 Hence, the correct option is (c)

SNRQ1 22×8 25. A deterministic signal has the power spectrum given
= in figure. The minimum sampling rate needed to com-
SNRQ1 22×9
pletely represent signal is
(SNRQ)2 = 4(SNRQ)1
So, increases by a factor of 4.
Hence, the correct option is (c)
21. Flat top sampling of low pass signals
(a) gives rise to aperture effect
(b) implies oversampling
(c) leads to aliasing
(d) introducing delay distortion [1998]
Chapter 3  Digital Communication Systems | 8.51

(a) 1 kHz (b) 2 kHz (a) depends upon the sampling frequency employed
(c) 3 kHz (d) none of these [1997] (b) is independent of the value of ‘n’
Solution: (b) (c) increases with increasing value of ‘n’
(d) decreases with the increasing value of ‘n’ [1995]
In sine function maximum power contain in main lobe.
So, we consider only main lobe Solution: (c)
BW = 2 kHz We know that
fs = 2 × BW 3 n
SNRQ = 22
2
fs = 4 kHz
So SNRQ depends on n
BW = 2(600 + 150)
Hence SNRQ increases as increasing value of n.
= 2 × 750
Hence, the correct option is (c)
= 1500 Hz
29. For a given data rate, the bandwidth Bp of a BPSK
Hence, the correct option is (b)
signal and the bandwidth B0 of the OOK signal are
26. If the number of bits per sample in a PCM system is related as
increased from a n to n + 1, the improvement in signal (a) BP = B0/4
to quantization nose ratio will be (b) BP = B0/2
(a) 3 dB (b) 6 dB (c) BP = B0
(c) 2 n dB (d) n dB [1995] (d) BPv = 2B0 [1995]
Solution: (b) Solution: (c)
We know that 2 n
(SNRQ)dB = 1.8 + 6n SNRQ α 2
for n bit n n
(SNRQ)1 22 22
(SNRQ)dB = 1.8 + 6n = 2( n +1) =
(SNRQ) 2 2 n
2 2 .2 2
for n + 1 bit
(SNRQ2) = (SNRQ) × 4
(SNRQ)′dB = 1.8 + 6( n + 1)
So SNRQ increases by factor = 4
= 1.8 + 6n + 6
So this is independent of n.
So, SNR increases by 6 dB
Hence, the correct option is (c)
Hence, the correct option is (b)
30. Increased pulse width in the flat top sampling leads to
27. A 1.0 kHz signal is flat top sampled at the rate of 1800
(a) attenuation of high frequencies in reproduction
samples/sec and the samples are applied to an ideal rec-
(b) attenuation of low frequencies in reproduction
tangular LPF with cut-off frequency of 1100Hz, then
(c) greater aliasing errors in reproduction
the output of the filter contains
(d) no harmful effects in reproduction [1994]
(a) only 800 Hz component
(b) 800 Hz and 900 Hz components Solution: (a)
(c) 800 Hz and 1000 Hz components Increased pulse width in the flat-top sampling leads
(d) 800 Hz, 900 Hz and 100 Hz components [1995] to greater attenuation of high frequencies in reproduc-
Solution: (c) tion. This effect is known as aperture effect.
M (t ) 2 Hence, the correct option is (a)
f out = T + T 31. The bandwidth required for the transmission of a
s s
(m(t) cos ws t + m(t) cos 2 ws t ) PCM signal increases by a factor of _____ when the
fm = 1 kHz number of quantization levels is increased from 4 to
64. [1994]
fs = 1.8 kHz
Solution:
fcut off = 1.1 kHz η fs
BW of PCM system =
So, at output frequencies present = fm, fs − fm 2
BW α n
= 1000 Hz and 800 Hz components
L1 = 4, L2 = 64
Hence, the correct option is (c)
η1 = log24, η2 = log264
28. The signal to quantization noise ratio in an
η1 = 2, η2 = 6
n-bit system
8.52 | Communication

BW1 η1 2 Where T is the bit-duration interval and t is in seconds.


BW = η = 6 3 Both u0(t) and u1(t) are zero outside the interval 0 ≤ t ≤
2 2
T. With a matched filter (correlator) based receiver, the
BW2 = 3 × BW1 smallest positive value of T (in milliseconds) required
Increases by a factor of 3. to have u0 (t) and u1 (t) uncorrelated is  [2017]
(A) 0.25 ms (B) 0.5 ms
(C) 0.75 ms (D) 1.0 ms
Two-marks Questions u0 (t ) = 5 cos 20000π t
Solution: 
1. A single bit, equally likely to be 0 and 1, is to be sent
u1 (t ) = 5 cos 22000π t
across an additive white Gaussian noise (AWGN) chan-
nel with power spectral density N0/2 Binary signaling FSK gets uncorrelated when
with 0 → p(t) and 1 → q(t), is used for the transmis-
R
sion, along with an optimal receiver that minimizes the f1 − ff 2 −= fn =b n; Rb n; = 1n, 2=, 31,....
2, 3....
1 2 2
bit-error probability. 2
 f − f  2000
  Let f1(t), f2(t) from an orthonormal signal set. Rb =R2 = 12  f12 − =f 2  = 2000 bits/sbits/s
  If we choose p(t) = f1(t) and p(t) = –f1(t) we would
b
 n n  n n
obtain a certain bit-error probability Pb. Rb ( max ) = 200 bits/see
  If we keep p(t) = f1(t), but take q(t) = E f2(t), for
1
what value of E would we obtain the same bity-error Tb ( min ) = = 0.5 ms
probability Pb? [2019] Rb ( max )
(A) 0 (B) 3
(C) 1 (D) 2 Hence, the correct option is (B).
1 3. A digital communication system uses a repetition code
Solution: P ( 0) = P (1) =
2 for channel encoding/decoding. During transmission,
each bit is repeated three times (0 is transmitted as
000, and 1 is transmitted as 111). It is assumed that the
source puts out symbols independently and with equal
probability. The decoder operates as follows: In a block
p(t) 1 q(t) of three received bits, if the number of zeros exceeds
1
f 1(t) the number of ones, the decoder decides in favour of a
dmin = 2 1. Assuming a binary symmetric channel with crosso-
ver probability p = 0.1, the average probability of error
f 2(t)
is _____. [2016]
dmin = √E + 1
q(t)
Solution:  Using the error probability for digital s­ ystem
√E √E + 1 when each bit is repeated 3 times. All the symbols are
independent and have equal probabilities.
1 f 1(t) Pe = P3 + 3P2 (1 – P)
P(t)
P = 0.1
Pe = 0.028.
For probability of error to be same Hence, the correct option is (0.028).
d1 = d2 4. An analog pulse s(t) is transmitted over an additive
white Gaussian noise (AWGN) channel. The received
2 = E +1 signal is r(t) = s(t) + n(t), where n(t) is additive white
4 = E +1 N
Gaussian noise with power spectral density 0 . The
E =3 2
received signal is passed through a filter with impulse
Hence, the correct option is (B)
response h(t). Let Es and Eh denote the energies of the
2. In binary frequency shift keying (FSK) the given signal pulse s(t) and the filter h(t) respectively. When the sig-
waveforms are nal to noise ratio (SNR) is maximized at the output
u0(t) = 5 cos (20000pt); 0 ≤ t ≤ T, and of the filter (SNRmax) which of the following holds?
u1(t) = 5 cos (22000 pt); 0 ≤ t ≤ T,  [2016]
Chapter 3  Digital Communication Systems | 8.53

2 Es 6. An ideal band pass channel 500 Hz–2000 Hz is deployed


Es = Eh; SNRmax =
(A) for communication. A modem is designed to transmit
N0
bits at the rate of 4800 bits/s using 16 – QAM. The
Es roll off factor of a pulse with a raised cosine spectrum
Es = Eh; SNRmax =
(B)
2N 0 that utilizes the entire frequency band is __________.
2 Es  [2016]
Es > Eh; SNRmax >
(C) Solution:  Bit rate Rb= 4800 bits/s
N0
2 Eh Bandwidth BW = 2000Hz – 500Hz
Es < Eh; SNRmax =
(D)
N0 =
1500 Hz

2Eb Number of Levels


SNRmax =
Solution:  [∵ Eb = Es]
No m = 16, m = 2n
Hence, the correct option is (A). where n is the number of bits and
5. An information source generates a binary sequence
n = log2 m
{αn}. αn can take one of the two possible values –1
and +1 with equal probability and are statistically inde- Let the roll-off factor of a pulse be α = ?
pendent and identically distributed. This sequence is
Now using the relation
precoded to obtain another sequence {βn}, as βn = αn +
kαn – 3. The sequence {βn} is used to modulate a pulse R (1 + α )
BW = b
g(t) to generate the base band signal [2016] log 2 m


X(t) = ∑ n= −∞ βn g (t − nT ), 1500 =
Rb (1 + α )
log 2 16
where 
⎧1, 0≤t ≤T α = 0.25 .
g(t) = ⎨ .
⎩0, otherwise
Hence, the correct Answer is (0.25).
1 7. A binary communication system makes use of the
If there is a null at f = in the power spectral density
of X(t), then k is _____. 3T symbols ‘zero’ and ‘one’. There are channel errors.
Consider the following events:
Solution:
G( f ) ⎡ ∞
2
⎤ x0: a ‘zero’ is transmitted
⎢ ∑ Rb (τ ) e
j 2π f τ T
Sx ( f ) = ⎥ x1: a ‘one’ is transmitted
T ⎢⎣ m = −∞ ⎥⎦  y0: a ‘zero’ is received
Rb (τ) = E[βn βn – τ] y1: a ‘one’ is received
1
=
E [ (αn + kαn – 3) (αn – τ + k αn – τ – 3)] The following probabilities are given: P(x0) = P(y0/x0)
3 1 2
= and P(y0/x1) = . The information in bits that you
=
(1 + k2) R(τ) + KR(τ + 3) + KR(τ – 3) 4 2
obtain when you learn which symbol has been received
⎡1 + k 2 τ=0 ⎤ (while you know that a ‘zero’ has been transmitted) is
⎢ ⎥
Rb (τ ) = ⎢ k τ = ±3 ⎥
_____. [2016]
⎢ 0 otherwise ⎥
⎣ ⎦ Solution: 
It is given that,
PSD, Sb ( f ) = 1 + k + 2 K cos (2π f 3T)
2 3
4
X0 Y0
1
Null occurs at f = 1
3T 1
2
4
1 ⎛ 1⎞
⇒ f = , Sb = 1 + K2 + 2Kcos 2π ⎜ ⎟ × 3T
3T ⎝ 3T ⎠ X1 Y1
1
i.e., 1 + k2 + 2k = 0 2

⇒ k = –1 P(x0) = 1/2


Hence, the correct Answer is (-1). P(y0/x0) = 3/4   P(y0).P(x0) = P(x0, y0)
8.54 | Communication

3 p(t) p(t)
P(y1/x0) = 1/4   × 1/2 = P(x0, y0)
4 1 1
3 √Ts ∗ √Ts
= P(x0, y0)
8
Ts Ts
P(x0, y1) = P(x0) P(y1)
y(t)
=
1/2 × 1/4 = 1/8. 1

(Using the expression of entropy for conditional


probability)
1 t
H(y/x0) = P(y0/x0) log2 + P(y1/x0) 0 Ts 2Ts
P(y 0 /x 0 )
1 Since, the convolution of two rect pulses is always tri-
log2 angular with width(= 2Ts) equal to the sum of the dura-
P(y1 /x 0 )
 tion of two rect pulses. So option (C).
−3 3 1 1 Hence, the correct option is (C).
= log2 – log2
4 4 4 4 9. Consider a four-point moving average filter defined
by the equation y[n] = ∑ i = 0 α1 x [ n − i ] . The condition
3
H (y/x0) = 0.81 bits per symbol.

Hence, the correct Answer is (0.81). on the filter coefficients that results in a null at zero
­frequency is [2015]
8. A binary baseband digital communication system
(A) α1 = α2 = 0; α0 = -α3
employs the signal
(B) α1 = α2 = 1; α0 = -α3
⎧ 1 (C) α0 = α3 = 0; α1 = α2
⎪ , 0 ≤ t ≤ Ts
p(t) = ⎨ Ts (D) α1 = α2 = 0; α0 = α3
⎪0, otherwise
3
⎩ Solution:  Given Y[n] = ∑ α i x[n − i]
for transmission of bits. The graphical representation i=0
of the matched filter output y(t) for this signal will be  α 0 x[n] + α1 x [n − 1] + α 2 x [n − 2] + α 3 x [n − 3]
=
 [2016]
(A) y(t) Given filter represents high pass filter because null is at
1/Ts zero frequency.
From options, let us consider α = α 2 = 0; α 0 − α 3
t Then Y[n] = – α 3 ⎡⎣ x[n] − x [ n − 3]⎤⎦
0 Ts 2Ts 
(B) y(t)
H[z] = – α 3 [1 − z −3 ] 
0.5
H[e jn] = – α 3 [1 − e − j 3n ]

t
0 Ts 2Ts At Ω = 0 then H(e0) = – α 3 [1–1]
(C) y(t) =
0
1
So option A is satisfied.
Hence, the correct option is (A).
t
0 Ts 2Ts 10. The modulation scheme commonly used for transmis-
(D) y(t) sion from GSM mobile terminals is [2015]
1 (A) 4-QAM
(B) 16-PSK
t (C) Walsh–Hadamard orthogonal codes
0 Ts/2 Ts (D) Gaussian minimum shift keying (GMSK)
Solution:  Convolution of 2 pulse with equal width = Solution:  The modulation scheme commonly used for
triangular. transmission from GSM mobile terminals is Gaussian
minimum shift keying (GMSK).
Hence, the correct option is (D).
Chapter 3  Digital Communication Systems | 8.55

11. The input X to the Binary Symmetric Channel (BSC) (A)


g(t)
shown in the figure is ‘1’ with probability 0.8. The cross-
over probability is 1/7. If the received bit Y = 0, the con- 1
ditional probability that ‘1’ was transmitted is ______.
 [2015]
X 6/7 Y 0 1 t
P[X = 0] = 0.2
0 0
(B)
g(t)
1/7 1/7

1
P[X = 1] = 0.8 1 1
6/7

Solution:  P[x] = [0.2 0.8]


0 1 t
Y1 Y2
(C)
g(t)
⎡ Y ⎤ x ⎡6 7 1 7 ⎤
P⎢ ⎥= 1 ⎢
x 1 7 6 7⎥⎦
⎣ ⎦ 2 ⎣
X
 1

Y1 ⎡ 6 Y2 ⎤
X 1 ⎢ × 0.2 1 7 × 0.2⎥
 Joint probability P[X, Y] = 7
X2 ⎢ 1 ⎥ 0 1 t
⎢ × 0.8 6 × 0.8⎥
⎣ 7 7 ⎦ (D)
g(t)
Y1 Y2
1
X 1 ⎡ 0.171 0.0285⎤
=
X 2 ⎢⎣0.114 0.685 ⎥⎦

P(Y1) = 0.171 + 0.114 = 0.285 0 1 t

P[Y2] = 0.0285 + 0.685 = 0.7130 Solution:  For a matched filter receiver the probability
of error
Y2
X 1 ⎡0.6 0Y.10399 ⎛ 2E ⎞

P[X/Y] = =Q⎜ ⎟
X 2 ⎢⎣0.4 0.960 ⎥⎦ ⎝ No ⎠

By dividing columns of P(X, Y) by P(Y1) and P(Y2) The probability of error will be minimum for which
energy is maximum
⎛X ⎞ 1
So P ⎜ 2 ⎟ = 0.4 So for option (A) energy E = ∫ (1) dt = 1
2
⎝ Y1 ⎠
 0
Hence, the correct Answer is (0.39 to 0.41). 1
2

For option (B) energy E = 2 ∫ ( 2t ) dt


2
12. The transmitted signal in a GSM system is of 200 kHz
bandwidth and 8 users share a common bandwidth 0
1
using TDMA. If at a given time 12 users are talking in ⎡ 4t 3 ⎤ 2
a cell, the total bandwidth of the signal received by the =2⎢ ⎥
⎣ 3 ⎦0
base station of the cell will be at least (in kHz) ______. 
 [2015] 1
=
Solution:  8 users share a channel of bandwidth 200 kHz. 3 
1
So for 12 users two channels will be used so bandwidth For option (C) and (D) energy E = ∫ (t ) dt
2

will be 2 × 200 = 400 kHz. 0


Hence, the correct Answer is (400). 1
⎡t3 ⎤ 1
13. Consider a binary, digital communication system =⎢ ⎥ =
3 3
which uses pulses g(t) and –g(t) for transmitting bits ⎣ ⎦ 0 
over an AWGN channel. If the receiver uses a matched Hence, the correct option is (A).
filter, which one of the following pulses will give the 14. A sinusoidal signal of amplitude A is quantized by a
minimum probability of bit error?  [2015] uniform quantizer. Assume that the signal utilizes all
8.56 | Communication

the representation levels of the quantizer. If the signal α = 4 × 10−3 V


to quantization noise ratio is 31.8 dB, the number of 1
T = = 2 × 10 −6 sec
levels in the quantizer is _________.  [2015] Then b Rb
Solution:  (SNR)dB = 6N + 1.8
2 × 10−6 × 16 × 10−6
Where  N = Number of bits used to encode Eb =
2
31.8 = 6N + 1.8 = 16 × 10−12
6N = 30 1  16 × 10 −12 

Pe = erfc  
N = 5 2  2 × 10 −12 
 
So no. of levels = 25 = 32. 1

= erfc 8
Hence, the correct Answer is (32). 2
1  8× 2 
Pe = erfc  
15. Let Q ( γ ) be the BER of a BPSK system over an 2  2 


AWGN channel with two-sided noise power spectral
density N0/2. The parameter γ is a function of bit energy 1  16 
= erfc 
and noise power spectral density. A system with two 2  2 
independent and identical AWGN channels with noise
 
power spectral density N0/2 is shown in figure. The 1  4 
BPSK demodulator receives the sum of outputs of both = erfc  
2  2
the channels.
 
Pe = θ(y) ∵ 1 erfc  x  = Q( x ) 
 2  2 
 y=0 1 1 1
P
x = 1  = 4 × 4 × 1 = 16
 

If the BER of this system is Q (b γ ) then the value of 1 1 1


Pe = ×0 + ×
b is _____. [2014] 2 2 8
Solution: 1
=
BER for the given system is Q 2r 16
b= 2 Hence, the correct option is (c)
= 1.414. 17. In a PCM system, the signal m(t) =
1 6. Coherent orthogonal binary FSK modula- {sin (100πt + cos(100πt ))} is sampled at the Nyquist
tion is used to transmit two equiprobable symbol rate. The samples are processed by a uniform quantizer
waveforms s1(t) = 2cos2πf1t and s2(t) = 2cos2πf2t, where with step size 0.75 V. The minimum data rate of the
α= 4 mV. Assume an AWGN channel with two-sided PCM system in bits per second is [2014]
Solution:
noise power spectral density N 0 = 0.5 × 10 −12 W/Hz. m(t) = sin (100 πt) + cos (100 πt)
2
Using an optimal receiver and relation the bit error Maximum peak of m(t) is 2
probability for a data rate of 5000 kbps is Vmax − Vmin
∆=
(a) Q(2) (b) Q(2 2 ) 2n
(c) Q(4) (d) Q 4 2  [2014] ( ) 0.75 =
2 2
Solution: (c) 2n
1  Eb 
Pe = erfc   2 2
2  2N 0  0.75 =
  2n
Rb = 500 × 103 bps n=2
N0 Rb = nfs
= 0.5 × 10 −12
2 = 200 bits/sec
Chapter 3  Digital Communication Systems | 8.57

18. An M-level PSK modulation scheme is used to trans-


f   = ; 0 < 2 < 2
2 2
mit independent binary digits over a band-pass channel
with bandwidth 100 kHz. The bit rate is 200 kbps and 1 4
the system characteristic is a raised-cosine spectrum The point of intersection which decides optimum
with 100% excess bandwidth. The minimum value of threshold
M is _______. 2
1− 2 =
[2014] 4
Solution: 2
1= 2+
Rb 4
B= (1 + α )
log 2 m 4
2=
200 × 2 5
100 =
log 2 m 21. A binary symmetric channel (BSC) has a transition

probability of 1/8. If the binary transmit symbol X is
log2m = 4 such that P(x = 0) = 9/10, then the probability of error
m = 16 for an optimum receiver will be
19. If the detection threshold is 1, the BER will be (a) 7/80 (b) 63/80
1 1 (c) 9/10 (d) 1/10 [2012]
(a) (b) Solution:
2 4
1 (d) 1 9
(c) P (=
x 0=
)
8 16 10
[2013]
Solution: (d) 9
P ( x = 1) = 1 −
10
BER is given as
 1  0 = 1
Pe = P ( 0 ) P   + P (1)P   10
 0  1 1 0 1
Transition Probability = P   = P   =
If detection threshold = 1 then 0 1 8

1 1 0
P=
(0) P=
(1) Pe = P (0) P   + P (1) P  
2 0 1

 y =1  +9 1 1 1
P =0 = × + ×
x = 0  10 8 10 8

1
=
8
22. A BPSK scheme operating over an AWGN chan-
nel with noise power spectral density of N0/2, uses
equiprobable signals s (t ) = 2E sin(ω t ) and
1 o
2E T
Hence, the correct option is (d) s 2 (t ) = − sin(ω ot ) over the symbol internal (0,
T
20. The optimum threshold to achieve minimum bit error T). If the local oscillator in a coherent receiver is ahead
rate (BER) is in phase 45° with respect to the received signals, the
1 4 probability of error in the resulting system is
(a) (b)
2 5
 2E   E 
(c) 1 (d) 3  [2013] (a)
Q  (b)
Q 
 No   No 
Solution: (b) 2
 E  
Optimum threshold is given by the point of intersection (c) E 
of two PDF curves
Q  (d)
Q 
 2N o   4N o 
2
f   = 1 − 2 |2| ≤ 1  [2012]
0
8.58 | Communication

Solution: (b)
BPSK receiver is shown as

 E 
2

2E 0  x +   0  x +
Given BPSK signals are S1 (t ) = sin(ωc t ) and  E 1   2  1
2E
T P N <
 ∫ 2 
=
N
exp 
N
2 0
 dx = ∫
πN0
exp
N0
S2 (t ) = − sin(ωc t ) for the interval of 0 ≤ t ≤ t dura- 2π 0 −∞   −∞
T 2  2 

tion. Let φ (t) represent the local oscillator generated
signal which is 45° leading with respect to the  receivedE 
2
 E
2

0  x +   0  x + 2 
 E 2 1 )  0 ≤ t ≤ T2   dx = 1
signal. N < φ (t ) = = T sin(ωc t + 45
P Then ∫ exp  ∫  exp dx
 2 N0 N0 πN0 N0
assuming φ(t) is of Unit energy. −∞ Let2π R (t) and 2
 R (t) be  −∞
21  2 2 

2
the received signal output, given as  E
 x +
2 
T T
2E 2 E  N0
∫ 0

= R1 (t ) = S1 (t )φ (t ) dt =
0
T
sin(ω ct )
T
sin(ω ct + 45°) dt
= Let
2 N0
= Z . Then dx =
2
dz ,


T T  E 1  Z2
∫ ∫
= R1 (t ) = S1 (t )φ (t ) dt =
2E
T
sin(ω ct )
T
2
sin(ω ct + 45°) dt =
E
2
P 

N < ∫
2 

=
E

exp  − 2  dz
 
0 0 So, N 0
Similarly,
 E 
E =Q 
R (t ) = − .  N0 
2
2
E As symbols are equiprobable
Let decision threshold be given as γ = + N . Then
1
2 so P (e) = ( P ( S2 / S1 ) + P ( S1 / S2 ) )
probability of error while transmitting S1 and receiving 2
it as s2 is
so
 E 
P (S 2 / S1 ) = P (γ < 0) = P  + N < 0 1  E  1  E   E 
 2  P (e ) = Q   + Q  =Q 
2  N0  2  N0   N0 
 E
= P N < − Hence, the correct option is (b)
 2 
Common Data Questions 23 & 24
And
A four-phase and an eight-phase signal constellation
 E  are shown in the figure below.
P (S 2 / S1 ) = P (γ < 0) = P  + N < 0
 2 
 E
= P N < −
 2 

23. For the constraint that the minimum distance between


pairs of signals points be d for both constellations, the
radii r1 and r2 of the circles are
Chapter 3  Digital Communication Systems | 8.59

(a)
r1 = 0.707 d, r2 = 2.782 d To active same error 2nd must have 3.42 time than 1st.
(b)
r1 = 0.707 d, r2 = 1.932 d The value in dB = 10 log (3.42)
(c)
r1 = 0.707 d, r2 = 1.545 d = 5.33 dB
(d)
r1 = 0.707 d, r2 = 1.307 d  [2011]
Hence, the correct option is (d)
Solution: (d)
25. The Nyquist sampling rate for the signal
For m c
sin(500 t ) sin(700 t )
s (t ) = ×
π  t t
d = 2sin   Es
m is given by
Distance of any point from origin is Es (a) 400 Hz (b) 600 Hz
(c) 1200 Hz (d) 1400 Hz
For 4-ary, r1 = E s1 [2010]
Solution: (c)
8-ary, r2 = Es2
1  2 sin(500π t ) sin(700π t ) 
s( t ) =
π  2  π 2t 2


d1 = 2 sin   r1
For  4 ary m = 4 4
1
= cos(700π t − 500π t ) − cos(700π t + 500π t ) 
π  2π 2 t 2
8 ray m = 8 d2 = 2 sin   r2
4 1
if d1 = d2 = d then =2
[cos 200π t − cos 1200π t ]
2π 4
π 
2 sin   r1 = d Maximum frequency component
4
1200 π
fm = = 600
d 2π
r1 = = 0.707d
2 Nyquist sampling rate = 2fm = 1200 Hz

Hence, the correct option is (c)
π 
2 sin   r2 = d Common data for Questions 26 and 27
8
The amplitude of a random signal is uniformly distrib-
d uted between –5V and 5V.
r2 =
π 
2 sin   26. If the signal to quantization noise ratio required in uni-
6 formly quantizing the signal is 43.5 dB, the step size of
r2 = 1.307d
the quantization is approximately
(a) 0.0333 V (b) 0.05 V
Hence, the correct option is (d) (c) 0.0667 V (d) 0.10 V  [2009]
24. Assuming high SNR and that all signals are equally Solution: (c)
probable, the additional average transmitted signal
SNR = 43.5 dB
energy required by the 8-PSK signals to achieve the
same error probability as the 4-PSK signals is 1.76 + 6.02n = 43.5
(a) 11.90 dB (b) 8.73 dB n = 6.94
(c) 6.79 dB (d) 5.33 dB [2011] ≈7
Solution: (d) VH − VL
Step size ∆ =
Pe α E s 2n

5 − ( −5)
ES1 r1 0.707 d =
= = 27
E S2 r2 1.307 d = 0.07

2
≈ 0.0667
ES2  1.307 
= = 3.42 Hence, the correct option is (c)
ES1  0.707 
8.60 | Communication

27. If the positive values of the signals are uniformly fs3 = 2 × 2w = 4w


quantized with a step size of 0.05 V, and the negative fs4 = 2 × 3w = 6w
values are uniformly quantized with a step size of 0.1
fs = 2w + 2w + 4w + 6w
V, the resulting signal to quantization noise ratio is
approximately = 14w
(a) 46 dB (b) 43.8 dB For minimum bandwidth n = 1
(c) 42 dB (d) 40 dB [2009] Rb = nfs
Solution: (b) = 1 × 14
For the samples = 14w
S = 0.05 R
( BW ) min = b
Q = 2n 2
5 (BW)min = 7W.
=
Q = 100

0.5 Hence, the correct option is (d)
Q = 2n Common Data for Questions 30, 31 and 32.
n=7 A speech signal, band width limited to 4 kHz and peak
SNR = 1.76 + 6n voltage varying between +5 V and –5 V, is sampled at
the Nyquist rate. Each sample is quantized and repre-
= 43.76 dB
sented by 8 bits.
for one samples
30. If the bits 0 and 1 are transmitted using bipolar pulses,
5
s = 0.1, θ = = 10 the minimum bandwidth required for distortion free
0.1 transmission is
ϑ = 2n, n = 4 (a) 64 kHz (b) 32 kHz
SNR = 1.76 + 6n = 25.76 dB (c) 8 kHz (d) 4 kHz [2008]
Best SNR = 43.8 dB. Solution: (b)
Hence, the correct option is (b) While using the bipolar pulses to transmit the bits 0 and
28. Consider a Binary Symmetric Channel (BSC) with 1 the minimum bandwidth required for d­ istortion-free
probability of error being p. To transmit a bit, say 1, transmission is four times the theoretical bandwidth
we transmit a sequence of three 1s. The receiver will (Nyquist bandwidth)
interpret the received sequence to represent 1 if at least fm = 4 kHz
two bits are 1. The probability that the transmitted bit Nyquist bandwidth
will be received in errors is fs min = 2fm = 8 kHz
(a) p 3 + 3 p 2(1 – p) (b) p3
(c) (1 – p) (d)
3
p 3 + p 2(1 – p) Minimum bandwidth of bipolar signalling is
[2008] 4fsmin = Pb/2
Solution: (a) =4×8
Pe = if two of received bit 0 or 3 of 3 received bit 0 = 32 kHz
3 3 Hence, the correct option is (b)
= C 2 P .P (1 − P ) + C 3P .P .P
31. Assuming the signals to be uniformly distributed
= 3P2 (1 – P) + P3 between its peak to peak value, the signal to noise ratio
Hence, the correct option is (a) at the quantizer output is
29. Four message band limited to W, W, 2W and 3W, (a) 16 dB (b) 32 dB
respectively, are to be multiplexed using Time Division (c) 48 dB (d) 64 dB [2008]
Multiplexing (TDM). The minimum bandwidth Solution: (c)
required for transmission of this TDM signal is SNR = 1.76 + 6.02 n
(a) W (b) 3W
≈ 6n
(c) 6W (d) 7W [2008]
≈ 6 × 8
Solution: (d)
= 48 dB
fs1 = 2 × w = 2w
Hence, the correct option is (c)
fs2 = 2 × w = 2w
Chapter 3  Digital Communication Systems | 8.61

32. The number of quantization levels required to reduce Solution: (c)


the quantization noise by a factor of 4 would be sin 4 wt
(a) 1024 (b) 512 p (t ) = .
4 wt(1 − 16w 2t 2 )
(c) 256 (d) 64 [2008]
Solution: (b) 1
sin 4π w ×
2  1  4w
Quantization noise power P = ∆ p =
N  4w  1  16w 2 
12 4π w × 1 − 
PN 4w  16w 2 
PN′ =
4 At it is in percentage form
 1 cos 4π wt
∆2 ∆2 p  =
=  2  1 − 48w 2t 2
12 12 × 4
by L hospital rule
1
s′ = cos π
s2 p (t ) =
1− 3
VPP = 0.5
s=
2n Hence, the correct option is (c)
VPP VPP V pp 35. During transmission over a certain binary communica-
= n
= n
2 2.2 2.28 tion channel, bit errors occur independently with prob-
2 = 2 = 512.
n 9 ability p. The probability of AT MOST one bit in error
in a block of n bits is given by
Therefore, number of quantization levels required to
(a) pn (b) 1 – pn
reduce the quantization noise by a factor 4 would be
(c) np(1 – p) +(1 – p) (d) 1 – (1 – p)n
n–1 n
512.
[2007]
Hence, the correct option is (b)
Solution: (c)
33. In delta modulation, the slope overload distortion can Out of n bit at most one error of probability is
be reduced by n 0 n n 1 n −1
(a) decreasing the step size C0 ( P ) (1 − P ) + C1 ( P ) (1 − P )
(b) decreasing the granular noise = (1 – P)n + nP (1 – P)–1
(c) decreasing the sampling rate
(d) increasing the step size [2007] 36. In a GSM system, eight channels can co-exist in 200
kHz bandwidth using TDMA. A GSM based cellular
Solution: (d) operator is allocated 5 MHz bandwidth. Assuming a
For no slope overload frequency reuse factor of 1 5, i.e. a five-cell repeat
pattern, the maximum number of simultaneous chan-
d
∆f s ≥ m(t ) nels that can exists in one cell is
dt max (a) 200 (b) 40

(c) 25 (d) 5 [2007]
So by increasing step size slope, overload is reduced.
Solution: (b)
Hence, the correct option is (d)
Allocated BW = 5 mHz
34. The raised cosine pulse p(t) is used for zero |S| in digi- Frequency reduced factor = 1/5
tal communications. The expression for p(t) with unity BW allocated = 5 × 1/5
roll-off factor is given by
= 1 mHz
sin 4π wt 1 mHz
p (t ) = Number of simultaneous channel = ×8
4π wt (1 − 16w 2t 2 ) 200 mHz

The value of p(t) = 40
1 Hence, the correct option is (b)
at t = is
4w
37. In a Direct Sequence CDMA system, the chip rate is
(a) −0.5 (b) 0
1.2288 × 106 chips per second. If the processing gain is
(c) 0.5 (d) ∞ [2007]
desired to be AT LEAST 100, the data rate
8.62 | Communication

(a) must be less than or equal to 12.288 × 103 bits per 39. If the constellation are used for digital communication
sec over an A WGN channel, then which of the following
(b) must be greater than 12.288 × 103 bits per sec statements is true?
(c) must be exactly equal to 12.288 × 103 bits (a) Probability of symbol error for Constellation 1 is
per sec lower
(d) can take any values than 122.88 × 103 bits per sec (b) Probability of symbol error for Constellation 1 is
 [2007] higher
Solution: (a) (c) Probability of symbol error is equal for both the
constellations
RC
Processing gain = (d) The value of N0 will determine which of the two
Rb constellations has a lower probability of symbol
RC error [2007]
≥ 100
Rb Solution: (a)

The probability of error decreases with increase in
RC average energy. As constellation 1 has more energy
Rb ≤
100 than that of constellation 2, so the probability of sym-
Rb ≤ 12.288 × 103 bits per second. bol error for constellation 1 is lower.
Hence, the correct option is (a) Hence, the correct option is (a)
Common Data for Question 38 & 39 Common Data for Questions 40 and 41
Two 4-ray signal constellations are shown. It is given An input to a 6-level quantizer has the probability
that φ1 and φ2 constitute an orthonormal basis for the density function f(x) as shown in the figure. Decision
two constellations. Assume that the four symbols in boundaries of the quantizer are chosen so as to maxi-
both the constellations are equiprobable. Let N0/2 mize the entropy of the quantizer output. It is given that
denote the power spectral density of white Gaussian three consecutive decision boundaries are ‘–1’, ‘0’ and
noise. ‘1’.

38. The ratio of the average energy of Constellation 1 to the 40. The values of a and b are
average energy of Constellation 2 is (a) a = 1/6 and b = 1/12
(a) 4a2 (b) 4 (b) a = 1/5 and b = 3/40
(c) 2 (d) 8 [2007] (c) a = 1/4 and b = 1/16
(d) a = 1/3 and b = 1/24 [2007]
Solution: (b)
Solution: (a)
Average energy of constellation 1 is
To maximize the entropy, probability must be equal for
0 + 4 a 2 + 4 a 2 + 8a 2 each symbol.
E1 = = 4a2
4 5
1
2
a +a +a +a 4a2 2 2 2

∫ bdx = 3

E2 = = 1
4 4 b × 4 = 1/3
= a2
1
E1 b=
=4 12

E2

0 1  1
Hence, the correct option is (b) 
 ∫
a dx + dx  =
 3 ∫
 −1 0 
Chapter 3  Digital Communication Systems | 8.63

1 fs = (2fm) × 3
2a =
3 = (2 × 1000) × 3

= 6 × 103 Hz
1
a= Hence, the correct option is (c)
6
43. The minimum step-size required for a Delta-
Hence, the correct option is (a) Modulator operating at 32K samples/sec to track
41. Assuming that the reconstruction levels of the quan- the signal (here u(t) is the unit step function) x(t)
tizer are the mid-points of the decision boundaries, the = 125t(u(t) – u(t – 1)) + (250 – 125t)(u(t – 1) –
ratio of signal power to quantization noise power is u(t – 2)) so that slope-overload is avoided would be
64 (a) 2−10 (b) 2−8
152 (b)
(a) (c) 2−6 (d) 2−4 [2006]
2 3
Solution: (b)
76
(c) (d) 28 [2007]
3 d
∆f s ≥ m(t )
dt max
Solution: (d)
−1 1 5
1 1 1
Ps = ∫
−5
x2 .
12
dx + ∫
−1
6 ∫
x 2 × dx + x 2 x dx
12
1

−1 1 5
1  x3  1  x3  1  x3 
=   +   +  
12  3 −5 6  3 −1 12  3 1

1 1 1
= ( −1 + 125) + (1 + 1) + (124)
36 16 36
124 1 124
= + ×2+
36 18 36
= 7 volt2
∆ 2 (5 − ( −5)) 2
PN = =
12 12 × 6 2


≈ 0.25
7
=
SNRQ = 28
0.25
Hence, the correct option is (d) ( m(t ) )max 125
∆ min = =
4 2. The minimum sampling frequency (in sam- fs 32 × 1000
8
ples/sec) required to reconstruct the follow-
ing signal from its samples without distortion =2 −8

 sin 2p 1000t 
3
 sin 2p 1000t 
2 Hence, the correct option is (b)
x (t ) = 5   + 7  4 4. In the following figure the minimum value of the con-
 p t   pt 
stant `C‘, which is to be added to y1(t) such that y1(t)
would be and y2(t) are different, is
(a) 2 × 103
(b) 4 × 103
(c) 6 × 103
(d) 8 × 103 [2006]
Solution: (c)
3 2
 2π × 103 t   sin 2π × 103 t 
x(t ) = 5 sin   + 7   −v v 

 πt 


 πt   2 , 2
 
8.64 | Communication

(a) ∆ (b) ∆/2 (d)


(c) ∆2/12 (d) ∆/L [2006]
Solution: (b)

Solution: (c)
y(t) = x(t) × x(T – t)

For y1(t) and y2(t) to be different minimum step size of


A/2 is needed, else they will be same.
Hence, the correct option is (b)
45. A signal as shown in the figure is applied to a matched
filter. Which of the following does represent the output
of this matched filter? [2005]

(a)

(b)

Hence, the correct option is (c)


46. A source produces binary data at the rate of 10 kbps. The
binary symbols are represented as shown in the figure

(a)
(c)
Chapter 3  Digital Communication Systems | 8.65

0.2
(b) 0
Pe   =
1 ∫ 1.dx = 0.2
0

0.1 + 0.2
( Pe )ay =
2
= 0.15
Hence, the correct option is (a)

48. Choose the correct one from among the alternatives a,


The source output is transmitted using two modulation
b, c, d after matching an item from Group 1 with the
schemes, namely binary PSK (BPSK) and quadrature
most appropriate item in Group 2.
PSK (QPSK). Let B1 and B2 be the bandwidth require-
ments of BPSK and QPSK, respectively. Assuming Group-1 Group-2.
that the bandwidth of the above rectangular pulses is 1. FM P. Slope overload
10 kHz, B1 and B2 are 2. DM Q.µ -law
(a) B1 = 20 kHz, B2 = 20 kHz
3. PSK R. Envelope detector
(b) B1 = 10 kHz, B2 = 20 kHz
(c) B1 = 20 kHz, B2 = 10 kHz 4. PCM S. Capture effect
(d) B1 = 10 kHz, B2 = 10 kHz [2004] T. Hilbert transform
Solution: (c) U. Matched filter
Bit rate = Rb = 10k (a) 1 – T, 2 – P, 3 – U, 4 – S
Bt = 2Rb = 20k (b) 1 – S, 2 – U, 3 – P, 4 – T
(c) 1 – S, 2 – P, 3 – U, 4 – Q
B2 = Rb = 10k
(d) 1 – U, 2 – R, 3 – S, 4 – Q  [2004]
Hence, the correct option is (c)
Solution: (c)
47. Consider a binary digital communication system with 1. FM- capture effect
equally likely 0’s and 1’s. When binary 0 is transmit- 2. DM- slope overload
ted the detector input can lie between the levels –0.25V 3. PSK- matched filter
and +0.25V with equally probability: when binary 1 4. PCM-.µ-law
is transmitted, the voltage at the detector can have any
Hence, the correct option is (c)
value between 0 and 1V with equal probability. If the
detector has a threshold of 0.2 V (i.e. if the received sig- 49. Three analog signals, having bandwidths 1200 Hz, 600
nal is greater than 0.2 V, the bit is taken as 1), the average Hz and 600 Hz, are sampled at their respective Nyquist
bit error probability is rates, encoded with 12 bit words, and time division
(a) 0.15 (b) 0.2 multiplexed. The bit rate for the multiplexed signal is
(c) 0.05 (d) 0.5 [2004] (a) 115.2 kbps (b) 28.8 kbps
Solution: (a) (c) 57.6 kbps (d) 38.4 kbps [2004]
Solution: (c)
Sampling frequency
fs = 2(1200 + 600 + 600)
= 4800 Hz
Rb = nfs
= 12 × 4.8
= 57.6 kpbs
k × 1/2 = 1 Hence, the correct option is (c)
0.25
1
Pe   =
0 ∫2 50. Consider the signal x(t) shown in the figure. Let h(t)
denote the impulse response of the filter matched to
0.2
x(t), with h(t) being non-zero only in the interval 0 to 4
= 2 × (0.25 – 0.2) sec. The slope of h(t) in the interval 3 < t < 4 sec is
= 2 × 0.05
= 0.1
8.66 | Communication

(a) 2.7, 3.4 (b) 3.3, 3.6


(c) 2.6, 2.7, 3.3, 3.4, 3.6 (d) 2.7, 3.3 [2003]
Solution: (d)
Exponential Fourier series coefficient is given as
T0 / 6
1
Cn = Ae − jnw0t
T0 −T0 / 6

1  nπ 
(a) sec -1 (b) –1 sec–1
=
A
sin 
2 π n  3 

1
- sec -1
(c) (d) 1 sec–1 [2004] from cn is clear that 1, 2, 4, 5, 7 …. Harmonics are
2 present.
Solution: (b) Frequency components are = 103.2 × 103.4 × 103.
x(t ) → h(t ) → y(t ) x(t) has frequency components 0.7t, 0.4k

(PH) ± (t) gives (1 ± 0.7)k, (2 ± 0.7)k, (4 ± 0.7)k,
H(t) = x(T – t) (1 ± 0.4)k, (2 ± 0.4)
Frequencies present in range of 2.5k to 3.5k are 2.7, 3.3
Hence, the correct option is (d)
52. A sinusoidal signal with peak-to-peak amplitude of
1536 V is quantized into 128 levels using a mid-rise
uniform quantizer. The quantization-noise power is
(a) 0.768 V (b) 48 × 10-6 V 2
(c) 12 × 10−6 V 2 (d) 3.072 V [2003]
Solution: (c)
VP – P = 1.5366
  L = 128
  n = log2 12.8 = 7
2
 Vp− p 
 
∆ 2  L 
PN = =
12 12
2
 1.5366 
 128 
= 
12
= 12 × 10−6V2
Hence slope between 3 < t < 4 = −1. Hence, the correct option is (c)
Hence, the correct option is (b) 53. If Eb, the energy per bit of a binary digital signal, is 10–5
5 1. Let x (t ) = 2cos(800 t ) + cos(1400 t ).x (t ) be sam- watt-sec and the one-sided power spectral density of
pled with the rectangular pulse train shown in the fig- the white noise, N0 = 10–6 W/Hz, then the output SNR
ure. The only spectral components (in kHz) present in of the matched filter is
the sampled signal in the frequency range 2.5 kHz to (a) 26 dB (b) 10 dB
3.5 kHz are (c) 20 dB (d) 13 dB  [2003]
Solution: (d)
Signal to noise ratio is given as
2EB
SNR =
N0

2 × 10 −5
= = 20
10 −6
Chapter 3  Digital Communication Systems | 8.67

(SNR)dB = 10 l0g10 20 (a) R = 32 kbps, SNRq = 25.8 dB


= 13 dB (b) R = 64 kbps, SNRq = 49.8 dB
(c) R = 64 kbps, SNRq = 55.8 dB
Hence, the correct option is (d)
(d) R = 32 kbps, SNRq = 49.8 dB [2003]
54. The input to a linear delta modulator having a step-size
Solution: (b)
∆ = 0.628 is a sine wave with frequency fm and peak
Bit rate = nfs
amplitude Em. If the sampling frequency fs = 40 kHz,
the combination of the sine-wave frequency and the = 2 × 8k = 64 kbps
peak amplitude, where slope overload will take place, SNR = 1.76 + 6n
is = 1.76 + 6 × 8
≥ fm pm = 49.8
(a) 0.3 V 8 kHz Hence, the correct option is (b)
(b) 1.5 V 4 kHz 57. A signal x(t) = 100 cos(24π × 103 )t is ideally sampled
(c) 1.5 V 2 kHz with a sampling period of 50 µsec and then passed
(d) 3.0 V 1 kHz [2003] through an ideal low pass filter with cutoff frequency
Solution: (b) of 15 kHz. Which of the following frequencies is/are
For no slop over load to occur present at the filter output?
(a) 12 kHz only (b) 8 kHz only
d (c) 12 kHz and 9 kHz (d) 12 kHz and 8 kHz
∆f s ≥ m(t )
dt max  [2002]

Solution: (d)
for slope over load to occur
x(t) = 100 cos (24π × 103)t
d
∆f s < m(t ) fm = k kHz
dt max
1
fs = × 106 = 20 kHz
∆fs = 0.625 × 40 × 103 50
= 25.12 k fcut off = 15 Hz
Solving by options y (t ) 2
y (t ) = + ( x(t ) cos ws t + x(t ) cos 2ws t + ....)
2π × 1.5 × 4 = 37.7 Ts Ts
which is greater than ∆fs Passed component = fm1, fs − fm
5 5. If S represents the carrier synchronization at the = 12 Hz, 8 Hz
receiver and ρ represents the bandwidth efficiency, then
the correct statement for the coherent binary PSK is it will suppress bear other components
(a) ρ = 0.5, S is required having frequency > 15 Hz.
(b) ρ = 1.0, S is required Hence, the correct option is (d)
(c) ρ = 0.5, S is not required 58. A video transmission system transmits 625 picture
(d) ρ = 1.0, S is not required [2003] frames per second. Each frame consists of a 400 × 400
Solution: (a) pixel grid with 64 intensity levels per pixel. The data
For BPSK rate of the system is
(a) 16 Mbps (b) 100 Mbps
bit rate (c) 600 Mbps (d) 6.4 Gbps [2001]
bandwidth efficiency =
BW Solution: (c)
fb
= Data rate = 625 × 400 × 400 × 6
2 fb
= 600 Mbps
= 0.5 Hence, the correct option is (c)
For coherent BPSK synchronization is required at the 59. The Nyquist sampling interval for the signal sin c(700t)
receiver. + sin c(500t) is
Hence, the correct option is (a)
(a) 1 sec (b) sec
56. A signal is sampled at 8 kHz and is quantized using 350 350
8-bit uniform quantizer. Assuming SNRq for a sinusoi-
dal signal, the correct statement for PCM signal with a 1
(c) sec (d) sec  [2001]
bit rate of R is 700 175
8.68 | Communication

Solution: (c) Solution: (c)


f(t) = sin c(700 t) + sin c(500 t) Given
sin 700π t sin 500π t s(t) = 6 × 104 sin c3 (400 t) × 106 sin c3 (100 t)
= +
700π t 500π t fm1 = 400 × 3 = 1200 H2

Total P = Pm1 + Pm2 + Pm3 = 150
700 π
fs = ×2 fm2 = 100 × 3 = 300Hz 3 = 150Hz

Hence, the correct option is (c)
1 63. The peak-to-peak input to an 8-bit PCM coder is 2
TS =
fs volts. The signal power-to-quantization noise power

ratio (in dB) for an input of 0.5cos(ωmt) is
1 (a) 47.8 (b) 43.8
TS = sec
700 (c) 95.6 (d) 99.6 [1999]
Hence, the correct option is (c) Solution: (b)
6 0. During transmission over a communication channel, bit VM2 (0.5) 2 0.25
=
Signal power = =
error occur independently with probability p. If a block 2 2 2
of n bits is transmitted, the probability of at most one
= 0.125
bit error is equal to
(a) 1 – (1 – p)n Vmax − Vmin
∆=
(b) p + (n – 1)(1 c p) L
(c) np (1 – p)n-1 VPP
(d) (1 – p)n + np(1 – p)n–1 [2001] =
L
Solution: (d)
2 1
Probability of error = P = = 8
2 27
Probability no error = 1 – P ∆2
Noise power PN =
Out of n bit probability of at most one bit error = either 12
there is no error or 1 error  1  1
η 0 n n 1 n −1
=  14  ×
= C0 ( P ) (1 − P ) + C1 ( P ) (1 − P )
12
2 
= (1 – P)n + np(1 – P)n 0.125 14
SNRQ = × 2 × 12
Hence, the correct option is (d) 1
6 1. In a digital communication system employing (SNRQ)dB = 10 log 0.125 × 12 × 214
Frequency Shift Keying (FSK), the 0 and 1 bit are rep- = 43.8 dB
resented by sine waves of 10 kHz and 25 kHz, respec-
tively. These waveforms will be orthogonal for a bit Hence, the correct option is (b)
interval of 64. The input to a matched filter is given by
(a) 45 µsec (b) 200 µsec
10 sin( 2 × 106 ) 0 < t < 10-4 sec
(c) 50 µsec (d) 250 µsec [2000] s(t ) = 
0 Otherwise
Solution: (b)
The peak amplitude of the filter output is
fm0 = 10 kHz = 2 × 5 kHz
(a) 10 volts (b) 5 volts
fm1 = 25 kHz = 5 × 5 kHz (c) 10 millivolts (d) 5 millivolts
1 1000 [1999]
Tb = ×
5 kHz 1000 Solution: (d)
Output of matched filter = energy of signal
Tb = 200 µ s

Hence, the correct option is (b)
62. The Nyquist sampling frequency (in Hz) of a signal
=

−∞
x(t ) dt

given by 6 × 104 sinc3(400)*106 sinc3(100t) is


10 −4
(a) 200 (b) 300
∫ 100 sin (2π ×10 )t dt
2 6
(c) 1500 (d) 1000 [1999] =
0
Chapter 3  Digital Communication Systems | 8.69

10 −4 Solution: (c)
100

6
= (1 − cos 2 × 2π × 10 +) Given that
2
0 reference bit = 1
10 −4 logic 0 = π
= (50T )0 +0
logic 1 = 0°
= 50 × 10−4
= 5 mill V0H
Hence, the correct option is (d)
6 5. Four independent messages have bandwidths of 100Hz,
100Hz, 200Hz, and 400Hz, respectively. Each is sam-
pled at the Nyquist rate, and the samples are Time Hence, the correct option is (c)
Division Multiplexed (TDM) and transmitted. The 6 8. Coherent demodulation of FSK signal can be detected
transmitted sample rate (in Hz) is using
(a) 1600 (b) 800 (a) correlation receiver
(c) 400 (d) 200 [1999] (b) bandpass filter and envelope detectors
Solution: (a) (c) matched filter
fm1 = 100 Hz (d) discriminator detection [1992]
fm2 = 100 Hz Solution: (a)
fm3 = 200 Hz Coherent demodulation of FSK signal can be detected
using correlation receiver.
fm4 = 400 Hz
Hence, the correct option is (a)
sample rate = 2(fm1 + fm2 + fm3 + fm4)
69. Source encoding in a data communication system is
= 2(100 + 100 + 200 + 400)
done in order to
= 2 × 800 (a) enhance the information transmission
= 1600 Hz (b) bandpass filters and envelope rate detectors
Hence, the correct option is (a) (c) conserve the transmitted power
(d) discriminator detection [1992]
66. The number of bits in a binary PCM system is increased
from n to n + 1. As a result, signal to quantization noise Solution: (a)
ratio will improve by a factor The purpose of source encoding in a data communica-
(a) (n + 1)/n tion system is to increase the information transmission
(b) 2(n+1) rate and the purpose of channel encoding is to decrease
(c) 2(n+1) the probability of error. The channel encoding helps in
(d) which is independent of n [1996] detection and correction of errors.
Solution: (d) Hence, the correct option is (a)
Signal to quantization ratio is given as 70. For the signal constellation shown in the figure, the
3 2n type of modulation is [1991]
( SNRQ )1 = 2 cos 2π(n/T)t
2
SNR with n = (n + 1) S2 S2
3
( SNRQ ) 2 = 22( n +1) = 4( SNRQ )1 sin 2π(n/T)t
2 T = symbol duration
S2 SA
Hence, the correct option is (d)
67. The bit stream 01001 is differentially encoded using
‘Delay and Ex OR’ scheme for DPSK transmission. Solution:
Assuming the reference bit as a ‘1’ and assuming For given signal phase shift in each symbol is 90°.
phases of ‘0’ and ‘π’ for 1’s and 0’s respectively, in the So modulation type is QPSK.
encoded sequence, the transmitted phase sequence
71. A signal has frequency components from 300 Hz to 1.8
becomes
kHz. The minimum possible rate at which the signal
(a) π,0, π, π, 0 (b) 0, π, π, 0, 0
has to be sampled is _______. [1991]
(c) 0, π, π,π, 0 (d) π, π, 0, π, π
[1992]
8.70 | Communication

Solution: Solution:
Given Given
fH = 1800 Hz fH = 1800 Hz
fL = 300 Hz fL = 300 Hz
BW = 1800-300 BW = 1800 – 300
= 1500 Hz = 1500 Hz
fH 1800 fH 1800
= k =
=k =
BW 1500 BW 1500
k = 1 k = 1
2 fH 2 fH
( f s )min = k ( f s )min =
k

2 × 1800 2 × 1800
= =
1 1
= 3600 samples/sec = 3600 samples/sec
72. A 4 GHz carrier is DSB-SC modulated by a low 75. In binary data transmission DPSK is preferred to PSK
pass message signal with maximum frequency of because
2 MHz. The resultant signal is to be ideally sampled. (a) a coherent carrier is not required to be generated at
The minimum frequency of the sampling impulse train the receiver
should be (b) for a given energy per bit, the probability of error is
(a) 4 MHz (b) 8 MHz less
(c) 8 GHz (d) 8.004 GHz [1990] (c) the 180° phase shifts of the carrier are unimportant
(d) more protection is provided against impulse noise
Solution: (b)
 [1989]
Given that
Solution: (a)
fc = 4 GHz
A coherent carrier is required in PSR but in DPSK
fm = 2 mHz coherent carrier is not required. So DPSW is preferred
fmax = fc + fm = 4000 + 2 = 2002 over PSK.
fmin = fc – fm = 4000 – 2 = 3998 Hence, the correct option is (a)
B.W. = (fmax – fmin) 76. The transfer function of a zero-order hold is
= (4002 – 3998) 1 - exp( -Ts)
(a) (b) 1/s
= 4 mHz s 1
fs = 2 × 4 = 8 mHz (c) 1 (d) [1 - exp( -Ts )]
Hence, the correct option is (b)
 [1988]
73. In a BPSK signal detector, the local oscillator has a
Solution: (a)
fixed phase error of 20°.This phase error deteriorates
the SNR at the output by a factor of A zero order hold systems holds the input signal value
(a) cos 20° (b) cos2 20° for a period of T, i.e., for an input of short duration (∆)
(c) cos70° (d) cos270° [1990] pulse, it produces an output pulse of duration T, the
sampling period.
Solution: (b)
Output power charges by cos 2 φ.
So φ = 20°
Output power changes by = cos2 30.
Hence, the correct option is (b)
74. A signal has frequency components from 300 Hz to 1.8
kHz. The minimum possible rate at which the signal
has to be sampled is [1991]
Chapter 3  Digital Communication Systems | 8.71

input x(t) = δ(t) Signal power


x(s) = 1 V
1
∫x
2
y(t) = 4(t) – 4(t – T) = × dx
2V
−V
1 e −Ts
y ( s) = − V
s s 1  x3 
=  
1 − e −Ts 2V  3 −V
y ( s) =
s
y ( s)
=
1
6V
(
V 3 +V 3 )
= H ( s)
x ( s)
V2
=
1 − e −Ts 3
H ( s) =
s V2
Hence, the correct option is (a) 3
SNRQ = × 12 4
77. A signal having uniformly distributed amplitude in the  RV 
2

interval (–V to +V) is to be encoded using PCM with  n 


uniform quantization. The signal to quantizing noise  2 
2 n
ratio is determined by the SNRQ = 2
(a) dynamic range of the signal Hence signal to quantization noise ratio is determined
(b) sampling rate by the number of quantization levels.
(c) number of quantizing levels
(d) power spectrum of signal [1988] Hence, the correct option is (c)
Solution: (c) 78. The message bit sequence to a DPSK modulator is 1, 1,
0, 0, 1, 1. The carrier phase during the reception of the
first two message bits is π, π. The carrier phase for the
remaining four message bits is
(a) π, π, 0, π (b) 20, 0, 0, π, π
(c) 0, π, π, π (d) π, π, 0, 0 [1988]
Solution: (c)

Area under the curve = 1


k × 2V = 1
1
k=
2V
Logic 1 = 0°
SNRQ = Signal power Logic 0 = 180° = π
noise power
Let reference bit = 0
We know that
2
noise power = ∆
12
Vmax − Vmin
∆= L
0, π, π, π
L = 2n
Hence, the correct option is (c)
V − ( −V ) 2V
∆= n
= 7 9. In a digital communication system, transmission of
2 2n
successive bits through a noisy channel is assumed to
2 be independent events with error probability P. The
P =  2V  × 1 (1)
N  n  12 probability of at most one error in the trans mission of
2  an 8-bit sequence is
8.72 | Communication

This improvement is for


(a) 7(1 − p ) + 7(1 − p ) + p
8 8 8 (a) lower frequency components only
(b) higher frequency components only
(b) (1 – p)8 + 8p(1 – p)7 (c) lower amplitudes only
(c) (1 – p)8 + p(1 – p)7 (d) higher amplitudes only [1987]
(d) (1 – p)8 + p(1 – p)7. [1988]
Solution: (c)
Solution: (b)
Companding results in making SNR uniform, through-
Probability of error = P out the signal irrespective of amplitude levels, since in
If there is no error then probability = 1 – P, uniform quantization, step size is same the quantization
Probability of at most one error in the transmission of noise power is uniform, throughout the signal.
an 8 bit sequence is = No error + 1 error Thus, higher amplitude of signal will have better SNR
8 0 8 8 1 7 than the lower amplitudes.
= C0 ( P ) (1 − P ) + C1 P (1 − P )
Hence companding is used for improving SNR at lower
= (1 − P )8 + 8 P (1 − P )7 amplitudes.
Hence, the correct option is (b) Hence, the correct option is (c)
8 0. Companding in PCM system lead to improved signal to
quantization noise ratio.
Chapter 4
Information Theory
(C)
One-mark Questions

capacity
1

1. A linear Hamming code is used to map 4-bit messages


to 7-bit codewords. The encoder mapping is linear. If
the massage 0001 is mapped to the codeword 0000111, P
and the massage 0011 is mapped to the codeword 0 1
1100110, then the massage 0010 is mapped to [2019]
(A) 0010011 (B) 1100001 (D)

capacity
1
(C) 1111000 (D) 1111111
Solution:
C1: 0 0 0 1 0 0 0 0 1 1 1
C2: ≈ 0 0 1 1 1 1 0 0 1 1 0 P

0 0 1 0 1 1 0 0 0 0 1 0 1

Hence, the correct option is (B). Solution:  The capacity of memoryless BSC is ex-
2. Let (X1, X2) be independent random variables. X1 has pressed as
mean 0 and variance 1, while X2 has mean 1 and vari-
C = 1 + p log 2 p + (1 − p) log 2 (1 − p)
ance 4. The mutual information I (X1; X2) between X1,
and X2 in bits is __________. [2017] Hence, the correct option is (C).
Solution:  For two independent random variables, 4. In a code-division multiple access (CDMA) sys-
I (x : y) = H(x) = H (x/y) tem with N = 8 chips, the maximum number of users
H (x/y) = H (x) for independent X and y ⇒ I(x : y) = 0 who can be assigned mutually orthogonal signature
Hence, the correct answer is (0). sequences is ______. [2014]
3. Which one of the following graphs shows the shannon Solution:
capacity (channel capacity) in bits of a memoryless Maximum number of users who can be assigned mutu-
binary symmetric channel with crossover probability ally orthogonal signature sequences is = 8.
P? [2017] 5. The capacity of a Binary Symmetric Channel (BSC)
(A) with cross-over probability 0.5 is _____.
capacity

1
[2014]
Solution:
Channel capacity C = P log2 P + (1 – P)log2
P (1 – P) + 1
0 1    C = 0.
(B)
It is the case of channel with independent
capacity

1
input and output, hence C = 0.
6. The capacity of a band-limited additive white
Gaussian noise (AWGN) channel is given by
P
0 1
8.74 | Communication

 P  A B C D E
C = W log 2 1 + 2  bits per second (bps), where
 σ W 0 1 1 1 1
W is the channel bandwidth, P is the average power
1 0 0 0 0
received and σ2 is the one-side power spectral density
of the AWGN. 1 0 0 1 1
P 1 0 1 0 0
For a fixed = 1000, the channel capacity (in kbps)
σ2 1 0 1 1 1
with infinite bandwidth (W → ∞) is approximately 1 1 0 0 0
(a) 1.44 (b) 1.08
1 1 0 1 1
(c) 0.72 (d) 0.36 [2014]
1 1 1 0 0
Solution: (a)
Channel capacity is given by 1 1 1 1 1

 P  The maximum possible code words are 16.


C = W log 2 1 + 2 
 σ W Hence, the correct answer is 16.

2. Consider a binary memoryless channel characterized
Hence, the correct option is (a)
by the transition probability diagram shown in the fig-
7. A source alphabet consists of N symbols with the prob- ure. [2017]
ability of the first two symbols being the same. A source
0.25
encoder increases the probability of the first symbol by 0 0
a small amount e and decreases that of the second by e
0.25
. After encoding, the entropy of the source
(a) increases
(b) remains the same 0.75
(c) increases only if N = 2
1 1
(d) decreases [2012] 0.75
Solution: (d)
We know that Entropy is maximum when symbols are The channel is
equal probable, so if probability will change from equal (A) lossless (B) noiseless
to non–equal, entropy will decrease. (C) useless (D) deterministic
Hence, the correct option is (d)  y    0.25 0.75 
Solution: 
P    =  
 x    0.25 0.75 
Two-mark Questions If mutual information I(X;Y) = 0 for every possible
1. Consider a binary channel code in which each code- input distribution, then the channel is called as useless
word has a fixed length of 5 bits. The Hamming dis- (or) zero-capacity channel.
tance between any pair of distinct code words in this
code is at least 2. The maximum number of code words Let [ P ( x )] = [α (1 − α )]
such a code can contain is ______.  [2018]  H ( x ) = −α log 2 α − (1 − α ) log 2 (1 − α ) bits/symbol
Solution:  Possible number of coder words in which   y 
[ P ( y )] = [ P ( x )]  P    = [ 0.25 0.75]
the Hamming distance is atleast 2 are as follows   x 
A B C D E  P ( X , Y )  =  
0 0 0 0 0  α 3α 
 4 4 
0 0 0 1 1  
 (1 − α ) 3 (1 − α ) 
0 0 1 0 0  4 4 
0 0 1 1 1
  X   P ( X , Y )   
0 1 0 0 0  P  Y  = = α α 
    P (Y )  d 1 − α 1 − α 
0 1 0 1 1  
0 1 1 0 0
Chapter 4  Information Theory  |  8.75

X  1
 = −∑ ∑ P ( X iYi )
H 4. A source emits bit 0 with probability and bit 1 with
Y  1 i 2 3
probability . The emitted bits are communicated to
X  3
Log P  i  bits/symbol the receiver. The receiver decides for either 0 or 1 based
 Yi  on the received value R. It is given that the conditional
= −α log 2 α − (1 − α ) log 2 (1 − α ) bits/symbol density functions of R are as [2015]
X ⎧1
I = ( X ;Y ) = H ( X ) − H   = 0 ⎪ , −3 ≤ x ≤ 1
Y  f R|0 ( r ) = ⎨ 4
Hence, the given binary memoryless channel is a use- ⎪⎩ 0, otherwise

less channel. and
Hence, the correct option is (C). ⎧1
⎪ , −1 ≤ x ≤ 5
3. If the vectors e1 = (1, 0, 2) e2 = (0, 1, 0) and e3 = (–2, f R/ 2 ( r ) = ⎨ 6
0, 1) form an orthogonal basis of the three dimensional ⎪⎩ 0, otherwise
real space R3, then the vector u = (4, 3, –3) ∈ R3 can be 
expressed as [2016] The minimum decision error probability is [2015]
2 11 (A) 0 (B) 1/12
(A) u = – e1 – 3e2 – e (C) 1/9 (D) 1/6
5 5 3
Solution:  The conditional density function for
2 11
u=–
(B) e1 – 3e2 + e ⎧1
5 5 3 ⎪ −3 ≤ x ≤ 1
fR/0 (r) = ⎨ 4
2 11 ⎪⎩ 0
u = – e1 + 3e2 +
(C) e otherwise

5 5 3
2 11 ⎧1
u = – e1 + 3e2 –
(D) e ⎪ , −1 ≤ x ≤ 5
5 5 3 fR/1 (r) = ⎨ 6
⎪⎩ 0, otherwise
Solution:  Given vectors are e1 = (1, 0, 2), e2 = (0, 1, 0) 
and e3 = (−2 , 0, 1) So error region will be
−1
Given u = (4, 3, −3) 1 1
PR 0 ( r1 ) = ∫ dx =
Let, u = ae1 + be2 + ce3(1) 4 2
−3 
for function 1
∴ (4, 3, −3) = a(1, 0, 2) + b(0, 1, 0) + c(−2, 0, 1) 1
1 1
PR 1 ( r1 ) = ∫ dx =    
= (a – 2c, b, 2a + c) −1
4 2
∴ (4, 3, −3) = (a – 2c, b, 2a + c) 5
1 2
and PR 1 ( r2 ) = ∫ dx =
Comparing the corresponding values on both sides, 6 3
1 
4 = a – 2c  ⇒  a – 2c = 4 (2) 1 for function 2
1 1
PR 0 ( r2 ) = ∫ dx =    
3 = b  ⇒  b = 3 −1
6 3
So minimum decision error probability
–3 = 2a + c  ⇒ 2a + c = –3 (3)
1 1 1
Solving (2) and (3), we get = PR/1(r1) ⋅ PR/0 (r2) = . =
2 3 6
−2
a= Hence, the correct option is (D).
5
and 5. A fair coin is tossed repeatedly until a ‘head’ appears
−11 for the first time. Let L be the number of tosses to get
c=
5 this first ‘head’. The entropy H(L) in bits is ______.
substituting the values of a, b and c in (1), we get  [2014]
−2 11 Solution:
u= e1 + 3e2 − e3 1
5 5 P1 = (probability of getting first head).
2
Hence, the correct option is (D).
8.76 | Communication

1 1 1 (c) increases as n
=
Probability to get head in 2nd time =. (d) increases as n log n [2008]
2 2 4
1 1 1 1 Solution: (a)
= P3 =. .
2 2 2 8 n
N
1 Entropy H ( m) = − ∑ P log(a) bits
i
Entropy H = ∑ P log
i =1
i 2   .
 Pi 
If i =1

H(m) = log η
H ≈ 2
n
1
6. Let U and V be two independent and identically distrib-
uted random variables such that P(U = + 1) = P(U = -1)
∴ ∑ n =1
i =1
= 1 . The entropy H(U + V) in bits is Hence, the correct option is (a)
2 9. A source generates three symbols with probabilities
(a) 3/4 (b) 1 0.25, 0.25, 0.50 at a rate of 3000 symbols per second.
(c) 3/2 (d) log2 3 [2013] Assuming independent generation of symbols, the
Solution: (c) most efficient source encoder would have average bit
U   V U + V rate of
(a) 6000 bits/sec (b) 4500 bits/sec
1   1  2 1
P (U + V = 2) = (c) 3000 bits/sec (d) 1500 bits/sec
4  [2006]
+1  −1  0 Solution: (b)
1 1 1
P (U + V = 0) = + = Bit rate = H.Rb
4 4 2
−1  +1  0
1 1
PCV + V = − 2 = 4 H = ΣPi log 2   = 1.5
−1 −1  −2
 Pi 
3 Bitrate = 1.5 × 3000
H (U + V ) = = 4500 bps
    2
Hence, the correct option is (c) Hence, the correct option is (b)
7. A communication channel with AWGN operating at a 10. A binary source has symbol probabilities 0.8 and 0.2.
signal to noise ratio SNR >> 1 and bandwidth B has IF extension coding (blocks of 4 symbols) is used, the
capacity C1. If the SNR is doubled keeping B constant, lower and upper bounds on the average code word
the resulting capacity C2 is given by length are
(a) C2 ≈ 2C1 (b) C2 ≈ C1 + B (a) lower _____. (b) higher _____.[1991]
(c) C2 ≈ C1 + 2B (d) C2 ≈ C1 + 0.3B Solution:
 [2009] n
Solution: (b) Entropy = H = − ∑ P log (P )
i =1
i 2 i

Given
H = [0.8 log2 (0.8) + 0.2 log2 (0.2)]
SNR >> 1
H = 0.72 bits/symbol
 S  S  For the extended coding scheme, using blocks of 4
C1 = B log 2 1 +  = B log 2  
 N  N symbols, the entropy is given by 4 × 0.72 = 2.8876
when SNR is doubled bits/4 symbols
The bounds on the average code
 2S 
C1 = B log 2   = C1 + B word length are
N 
H (x) <•L H (x) + 1
C1 = C1 + B
Hence, the correct option is (b) (a) Lower bound = 2.8876 bits
(b) Upper bound = 3.8876 bits
8. A memoryless source emits n symbols each with a
probability p. The entropy of the source as a function 11. An image uses 512 × 512 picture elements. Each of the
of n picture elements can take any of the 8 distinguishable
(a) increases as log n intensity levels. The maximum entropy in the above
(b) decreases as log (1/n) image will be
Chapter 4  Information Theory  |  8.77

(a) 2097152 bits (b) 786432 bits n


(c) 648 bits (d) 144 bits ∑
[1990] Entropy = H = − Pi log 2 ( P)
i =1
Solution: (b)
Number of symbols that one image pixel can have = M 1 1 1 1 1 1 1  1 
= 8. H = −  log 2   + log 2   + log 2   + log 2   
2 2 4 4 8 8 8  8 
Maximum Entropy associated with one image pixel
Hmax = log2 M= log28 = 3 1 1 1 1 1 1 1  1 
H = −  log 2   + log 2   + log 2   + log 2   
2
Maximum entropy associated with image= 512 × 512 2  4 4 8 8 8  8 
   
× Hmax
1 1 1 1
= 512 × 512 × 3 H = + ×2 + ×3+ ×3
2 4 8 8
= 786432
3 7
Hence, the correct option is (b) H = 1+ =
4 4
12. A source produces 4 symbols with probability
1 1 1 1 . For this source, a practical coding Code efficiency = η = H ×100%
, , and L
2 4 8 8 7
scheme has an average codeword length of 2 bits/sym-
bols. The efficiency of the code is η = 4 × 100%
(a) 1 (b) 7/8 2
(c) 1/2 (d) 1/4 [1989] 7
η=
Solution: (b) 8
Code efficiency = Hence, the correct option is (b)
(Entropy)H
η= ×100%
(Average code length)L
Unit IX
Electromagnetic Theory

Chapter 1: Basics 9.3


Chapter 2: Uniform Plane Waves 9.20
Chapter 3: Transmission Lines 9.39
Chapter 4: Waveguide 9.56
Chapter 5: Antenna 9.67
EXAM ANALYSIS
Exam Year 92 93 94 95 96 97 98 99 00 01 02 03 04 05 06 07 08 09 10 11 12 13 14-1 14-2 14-3 14-4 15 16 17 18 19
Set 1 Set 2 Set 3 Set 1 Set 2 Set 3 Set 1 Set 2
1 Marks Ques. - - 5 5 3 2 12 3 4 4 4 2 2 1 2 2 2 2 2 4 4 1 2 2 2 4 1 2 1 2 4 2 2 2 3 3
2 Marks Ques. 2 6 2 - 4 1 - 4 4 4 4 7 6 6 8 7 5 3 2 3 5 2 3 4 3 3 3 2 1 7 4 4 3 3 3 3
5 Marks Ques. - 2 4 3 3 3 5 3 2 3 2 - - - - - - - - - - - - - - - - - - - - - - - - -
Total Marks 6 22 29 20 26 19 37 26 22 27 22 16 14 13 18 16 12 8 6 10 14 5 8 10 8 10 7 6 3 16 12 10 8 8 9 9

Chapter wise marks


distribution
Basics - 6 3 2 1 - 1 1 - - 2 3 2 - - 1 1 5 - 3 6 - 1 2 4 8 3 11 5 1 3
Uniform Plane Waves - 4 1 2 4 - 6 2 4 3 4 7 2 1 7 5 2 - 3 3 3 4 - 2 2 - 5 3 2 1 -
Transmission Lines 2 2 3 - 2 1 1 4 1 3 2 2 8 8 4 4 4 2 3 3 2 1 4 3 1 - 4 3 1 2 -
Waveguide - - 1 - - 1 - 1 3 2 1 2 1 2 4 4 2 1 - 1 2 - 1 3 1 - 1 2 1 - 2
Antenna 4 - 1 1 4 2 4 2 3 4 3 2 1 2 3 2 3 - - - 1 - 2 - - 2 - 4 1 1 1
Chapter 1
Basics
ε0
One-mark Questions
1. In the table shown. List-I and II. Respectively, contain
terms appearing on the left-hand side and the right- H Q
hand side of Maxwell’s equations (in their standard
form). Match the left-hand side with the corresponding
right-hand side. [2019]

List-I List-II 4H H ε0
(A) (B)
1.  ∇ ⋅ D P. 0 Qε 0 4Q
2.  ∇ × E Q.  r
HQ HQ
  ∂B (C) (D)
3.  ∇ ⋅ B R. − ε0 4ε 0
∂t
∂D Solution: The electric flux leaving the cylinder of
4.  ∇ × H S.  J  +
∂t height, H
Q  2ΠpH 
(A) 1 – R, 2 – Q, 3 – S, 4 – P = ∫ E .da =
(B) 1 – Q, 2 – S, 3 – P, 4 - R 2ΠEo p  4 
(C) 1 – P, 2 – R, 3 – Q, 4 – S QH
= ∫ E .da =
(D) 1 – Q, 2 – R, 3 – P, 4 – S 4 Eo

Solution: The differential four of the 4 Maxwell’s Hence, the correct option is (D).
equations are: 3. Two conducting spheres S1 and S2 of radii a and b(b>a)
respectively, are placed far apart and connected by a
∇⋅ D = P
long, thin conducting wire, as shown in the figure.
∇⋅ B = O
S2
−∂B
∇×E =
∂t Wire
S1
∂D
∇×H = J +
∂T Radius a

Radius b
Hence the correct option is (D)
For some charge placed on this structure, the potential
( )
2. What is the electric flux ∫E ⋅ d â through a quarter- and surface electric field on S1 are Va and Ea and that
on S2 Vb and Eb, respectively. Then, which of the fol-
cylinder of height H (as shown in the figure) due to an
lowing is CORRECT? [2017]
infinitely long line charge along the axis of the cylinder (A) Va = Vb and Ea< Eb
with a charge density of Q? [2019] (B) Va> Vb and Ea> Eb
9.4 | Electromagnetic Theory

(C) Va = Vb and Ea> Eb the azimuthal component of magnetic field outside the
(D) Va> Vb and Ea = Eb conductor and r is the radial distance from the conduc-
4. Concentric spherical shells of radii 2 m, 3 m and 8 m tor? [2015]

(A)
carry uniform surface charge densities of 20 nC/m2,
–4 nC/m2 and ρs, respectively. The value of ρ (nC/m )
2

required to ensure that the electric flux density D = 0


at radius 10 m is _____. [2016]
Solution:  The electric flux density D = 0 at r = 10 m r
when the total charge enclosed by the sphere of radius
(B)

10 m, is equal to zero, then
20 × 10–9 × 16p – 4 × 10–9 × 64p + ρs × 256p = 0

⇒ ρs = –0.25 nC/m2
r
Hence, the correct Answer is (–0.25 nC/m2).

(C)
5. A uniform and constant magnetic field B = ẐB exists
in the Ẑ direction in vacuum. A particle of mass m
with a small charge q is introduced in to this region
with an initial velocity V = xv ˆ x + zv
ˆ z . Given that B, m,
q, vx and vz are all non-zero, which one of the following r
describes the eventual trajectory of the particle?[2016]

(D)
(A) Helical motion in the ẑ-direction
(B) Circular motion in the xy plane
(C) Linear motion in the ẑ-direction
(D) Linear motion in the x̂-direction
Solution:  When the field is directed in the ‘z’ direction r
according to the right hand thumb rule, the motion of Solution: 
Given that conductor is lying on z-axis
the particle is in helical trajectory in the z-direction. → →
At a fixed point in the z-direction the, the particle moves l dl × r
Hφ = ∫
in the x–y plane. L
→3
4π r
Hence, the correct option is (A).

6. Faraday’s law of electromagnetic induction is mathe- 1
matically described by which one of the following equa- Hφ ∝ 
r2
tions? [2016]
 
 Hϕ
(A) ∇⋅ B = 0 (B) ∇⋅ D = ρv
 

 ∂B   ∂ D
(C) ∇× E = − (D) ∇× H = σE +
∂t ∂t
Solution:  From the basic four equations, faraday’s law
of EMI can be best represented by (C) showing that
the time varying electric field produces an orthogonal r
space varying magnetic field and vice versa Hence, the correct option is (C).
∂B → →


∇xE = − 8. A vector P = x y a x - x y a y - x2yz a z . Which one of
3 2 2
∂t
One of the Maxwell’s equations based on Faraday’s law the following statements is TRUE? [2015]

of electro-magnetic induction. (A) P is solenoidal, but not irrotational
Hence, the correct option is (C). →
(B) P is irrotational, but not solenoidal
7. Consider a straight, infinitely long, current carrying →
conductor lying on the z-axis. Which one of the follow- (C) P is neither solenoidal nor irrotational
ing plots (in linear scale) qualitatively represents the →
dependence of HΦ on r, where HΦ is the magnitude of (D) P is both solenoidal and irrotational
Chapter 1  Basics  |  9.5

Given P = x3y ax – x2y2 a y –x2yz az


Solution:  Solution: (2.121)
∂ ∂ ∂ xy
Div P = ∇.P =
∂x
( )
x3 y +
∂ y
(
− x2 y2 +
∂z
− x 2 yz ) ( )
F ( x, y ) =
2
( x + y)

=
3x2y – 2x2y – x2y ∂F ∂F
∇ F=
 axˆ + ayˆ
=
0 ∂x ∂y

∴P is solenoidal (1) (2 xy + y 2 ) (2 xy + x 2 )
= axˆ + ayˆ
i j k 2 2
∂ ∂ ∂ 3 3
Curl P = ∇ × P = ∆F (1,1) = axˆ + ayˆ
∂x ∂y ∂z
2 2
x3 y − x2 y2 − x 2 yz
 1 1. Consider a closed surface S surrounding a volume V. If

⎛ ∂ ∂ ⎞ ⎛ ∂ ⎞ ∫∫

5r ⋅ nˆ the
is the position vector of a point inside S, with dS unit
=i⎜
⎝∂y
(
− x 2 yz −
∂z
) ⎠
(
− x2 y2 ⎟ − j ⎜
⎝∂x
)
− x 2 yz ⎟

( ) normal on S, the value of the integral
S 
∫∫

5r ⋅ nˆ dS is

S
∂ 3 ⎛ ∂ ∂ 3 ⎞

∂z
( )
x y +k⎜
⎝∂x
− x2 y2 − (
∂y
x y⎟

) ( ) (a) 3 V (b) 5 V
 (c) 10 V (d) 15V [2011]


( ) (
= − x 2 z + 0 i + 2 xyz j + −2 xy 2 − x 3 k

) Solution: (d)
 
⇒ curl P = –x 2 i + 2 xyz j − x + 2 xy
2
( 3 2
)k

∫ ∫
5r .nˆ ds =
S
∫∫∫ ∇.5r dv [applying divergence
v
theorem]
∴ curl P ≠ 0 

∴ P is NOT irrotational (2)
=5 ∫ ∫ ∫ ∇.r dv
v

From (1) and (2), =5×3


= 15 V
P issolenoidal, but not irrotational.
Hence, the correct option is (d).
Hence, the correct option is (A). 12. Two infinitely long wires carrying current are as shown
9. The magnitude of the gradient for the function f(x, y, in the figure below. One wire is in the y-z plane and par-
z) = x2 + 3y2 + z3 at the point (1, 1, 1) is_________. allel to the y-axis. The other wire is in the x-y plane and
 [2014] parallel to the x-axis. Which components of the result-
Solution: (7) ing magnetic field are non-zero at the origin?
F(x, y, z) = x2 + 3y2 + z3
∂F ∂F ∂F
∇F = ax + ay + az
∂x ∂y ∂z


= 2 xax + 6 yay + 3 z 2 az

∇F (1, 1, 1) = 2ax + 6 ay + 3az


∇F(1, 1, 1) = 22 + 6 2 + 32

(a) x, y, z components (b) x, y components


     = 4 + 36 + 9 (c) y, z components (d) x, z components
 [2009]
∇F (1, 1, 1) = 7
Solution: (d)
xy
10. The directional derivative of f ( x, y ) = ( x + y ) at H(1) = ay – az [at origin]
2
p = −ax
(1, 1) in the direction of the unit vector an angle of
4 H(2) = ax × −ay
with y-axis, is given by____ [2014] = −az
9.6 | Electromagnetic Theory

∴ x, z components will be present. 15. The Unit of ∇ × H is


(a) Ampere (b) Ampere/meter
(c) Ampere/meter2 (d) Ampere-meter
 [2003]
Solution: (c)
∂ ∂ ∂
∇= ax + ay + az
∂x ∂y ∂z

unit of ∇ is m−1 (per meter)
unit of H = Ampere/meter
Hence, the correct option is (d). 1 Ampere
∇× H = ×
1 3. For static electric and magnetic fields in an inhomoge- meter meter
neous source free medium, which or the following rep- So, unit of ∇ × H is Ampere/(meter)2
resents the correct form of two of Maxwell’s equations?
Hence, the correct option is (c).
(a) ∇.E=0
∇ × B = 0 16. An electric field on a plane is described by its potential
(b) ∇.E=0 V = 20(r-1 + r-2) where r is the distance from the source.
∇ . B = 0 The field is due to
(c) ∇×E=0 (a) a monopole
∇ × B = 0 (b) a dipole
(b) ∇×E=0 (c) both a monopole and a dipole
∇ × B = 0 [2008] (d) a quadrupole [1999]
Solution: (d) Solution: (c)
Maxwell’s equations are for static field, V = 20(r−1 + r−2)
20 20
∇ ⋅ D = ρ v ∇⋅E = 0 = +
r r2
∇ ⋅ B = 0 ∇⋅H = J
θ
Hence, the correct option is (d). V [monopole] =
4π Er
1 4. If C is a closed curve enclosing a surface S, then
 the

 θ d cos θ
magnetic field intensity H , the current density J and V [dipole] =

 4π Er 2
the electric flux density D are related by
 Therefore, the field is due to both a monopole and a
    ∂ D  
∫∫
(a) H ids =  J + ∫ 
C 
∂t 
idl dipole.
s Hence, the correct option is (c).

 

    ∂ D     ∂ D

(b) H idl =
C
∫∫
 S
 J +
 ∂t 
ids 17. The Maxwell’s equation, ∇ × H = J+
∂t
is based on

  (a) Ampere’s law (b) Gauss’s law


  ∂D  
S
∫∫

(c) H ids =  J +
C
 ∫ ∂t 
idl

(c) Faraday’s law (d) Coulomb’s law
[1998]
 Solution: (a) Ampere’s Law
   ∂ D 
∫
(d) H idl = J + ∫∫ ∂t
ids  [2007]
  dD

C s Maxwell 4 equation, ∇ × H = j +
th
is based upon
dt
Solution: (d)
modified ampere’s Law,
Maxwell Fourth equation i.e.
 Hence, the correct option is (a)
    dD  
∫
H .dl = ∫∫  J +

 .ds
dt 
18. A metal sphere with 1 m radius and a surface charge
density of 10 Coulombs/m2 is enclosed in a cube of 10
C S
m side. The total outward electric displacement normal
Hence, the correct option is (d). to the surface of the cube is
Chapter 1  Basics  |  9.7

(a) 40 π Coulombs
(b) 10 π Coulombs
(c) 5 π Coulombs
(d) None of the above [1996]
Solution: (a)

Using Gauss Law, ∫ D.ds = Q enclosed

Qenclosed = ρ s × surface area


= 10 c/m2 × 4πr2 m2 Hence, the correct option is (b).
= 40 π Coulombs   
Hence, the correct option is (a).
21.
∫ A.dl = ∫ ___ ds 
C S
[1994]

19. In the infinite plane, y = 6 m, there exists a uniform sur- 


face charge density of (1/6000) μC/m2. The associated Solution: (∇ × A)
electric field strength is   
(a) 30 î V/m (b) 3 V/m
By using stoke’s theorem, ∫
C

A.dl = (∇ × A).ds
S
(c) 30 k V/n (d) 60 V/m [1995]
Solution: (b) 22. An electrostatic field is said to be conservative when….
 
Sheet charges and uniform fields.
 ρ   ρ 
∫
E ⋅ dl = 0 (1)
D = s aN and E = s aN i.e. the closed line integral of the field is zero because
2 2E  

for the open line integral, the field will be,
[i.e. voltage across the line]
E .dl = V ∫
Now, apply stokes theorem on equation (1)
s
    
i.e.,
∫ E.dl =∫ (∇ × E )ds thus, ∇ × E = 0 
C s
[1994]

Solution: (b)
the curl of the field is equal to zero. An electrostatic
  1 field is said to be conservative when the closed line
Here, aN = ay and ρ s = µ c/m 2
6000 integral of the field is zero.
 


∫
 −6
1× 10 E . dl = 0
E= ay
6000 × 8.854 × 10 −12 × 2 Stoke’s theorem

  
   
ˆ
E = 3 j v/m ∫
c

E ⋅ dl = (∇ × E ) ⋅ ds
s
Hence, the correct option is (b). 
2 0. The electric field strength at distant point. P, due to a So, ∇ × E = 0
point charge, + q, located at the origin is 100 μV/m If Hence, the correct option is (b).
the point charge is now enclosed by a perfectly con-
ducting metal sheet sphere whose center is at the origin,
then the electric field strength at the point, P, outside Two-marks Questions
the sphere, becomes
1. Two identical copper wires W1 and W2, placed in par-
(a) zero (b) 100 μV/m
allel as shown in the figure. Carry currents I and 2I,
(c) –100 μV/m (d) 50 μV/sm [1995]
respectively, in opposite direction. If the two wires are
Solution: (b) separated by a distance of 4r, then the magnitude of
Since, in conductor charge induces equal and opposite 
the magnetic field B between the wires at a distance r
charge on surface then field remain same. from W1 is [2019]
i.e. 100 μV/m
9.8 | Electromagnetic Theory

W1 \ The field represents an elliptically polarized plane


r wave propagating along the x – y plane.
Hence, the correct option is (C).

 3. If the vector function F = aˆ x (3 y − k1 z ) + aˆ y ( k2 x − 2 z ) − aˆ z ( k3 y + z )
W2
F = aˆ x (3 y − k1 z ) + aˆ y ( k2 x − 2 z ) − aˆ z ( k3 y + z ) is irrotational, then the values of the con-
stants k1, k2 and k3, respectively, are  [2017]
μ02 I 2 6 μ0 I
(A) 0.3, -2.5, 0.5 (B) 0.0, 3.0, 2.0
(A) (B)
2π r 2 5π r (C) 0.3, 0.33, 0.5 (D) 4.0, 3.0, 2.0
μ0 I 5μ0 I Solution:  Given F = aˆ x (3y – k1z) +â y(k3y + z) is
(C) (D)
6π r 6π r ­irrotational
⇒ curl F = 0
Solution:
i j k
W1
∂ ∂ ∂
r B1 ⇒ =0
∂x ∂y ∂z
3r 3 y − k1 z k2 x − 2z − k 3 y − z
B2
W2
⇒ i (−k3 + 2) − j (0 + k1) + k (k2 − 3) = 0
B = B1 + B2 ⇒ −k3 + 2 = 0, k1 = 0 and k2 − 3 = 0
m0 I ⇒ k3 = 2; k1 = 0 and k2 = 3.
B1 =
2Π r
Hence, the correct option is (B).
m 2I
B2 = 0
2Π 3r 4. The current density in a medium is given by
 400 sin θ
B = B1 + B2 J = âr Am–2.
m0 I  2  2π r 2 + 4 ( )
= 1+
2Π r  3  The total current and the average current density flow-
5m I ing through the portion of a spherical surface r = 0.8 m,
= 0 π π
6Πr ≤θ≤ , 0 ≤ f ≤ 2π are given respectively, by
12 4
Hence, the correct option is (D).  [2016]
(A) 15.09 A, 12.86 Am–2
2. The expression for an electric field in free space is
E = E0 ( xˆ + yˆ + j 2 zˆ )e − j (wt − kx + ky ) , where x, y, z represent (B) 8.73 A, 13.65 Am–2
the spatial coordinates, t represents time, the w, k are (C) 12.86 A, 9.23 Am–2
constants. This electric field. [2017] (D) 10.28 A, 7.56 Am–2
(A) Does not represent a plane wave.
Solution:  Radius of spherical surface r = 0.8 m.
(B) represents a circularly polarized plane wave prop-
agating normal to the z-axis. Current density is given as
(C) represents an elliptically polarized plane propagat- 400 sin θ
ing along the x-y plane. aˆ J = aˆr Am −2
2π ( r + 4)
2
(D) represents a linearly polarized plane wave.
Total current flowing through the given spherical
Solution:  Direction of propatation between +x and +y ­surface, can be calculated using
e − jωt e − jk x e − jky
I = ∫∫ J d s
The magnitudes of electric field z-component and x - y
components are not equal s 
2π π /4
400 sin θ
E zo = 2 E0 = ∫ ∫π 2π ( r + 4)
2
ar r 2 sin θ dθ dϕ ar
Q =0 θ =
E xo = 2 E0 2 
r = 0.8
and the phase difference between the two components
= 90° =
15.09 A.
Chapter 1  Basics  |  9.9

Average current density through the surface, will be 6. A positive charge q is placed at x = 0 between two infi-
nite metal plates placed at x = –d and at x = +d respec-
I 15.09
J av = = 2π π /4
tively. The metal plates lie in the yz-plane.
∫∫ ds ∫ ∫ r 2 sin θ dθ dϕ
s
Q = 0 θ =π / 2

=
12.86 A/m2.
Hence, the correct option is (A).

at x = +d
at x = –d
+q
5. The parallel plate capacitor shown in the figure has x=0
movable plates. The capacitor is charged so that the
energy stored in it is E, when the plate separation is d,
the capacitor is then isolated electrically and the plates
are moved such that the plate separation becomes 2d.
d
The charge is at rest at t = 0, when a voltage +V is
applied to the plate at –d and voltage –V is applied
to the plate at x = +d. Assume that the quantity of the
charge q is small enough that it does not perturb the
field set up by the metal plates. The time that the charge
At this new plate separation, what is the energy stored q takes to reach the right plate is proportional to[2016]
in the capacitor, neglecting fringing effects? [2016] d d
(A) (B)
(A) 2E (B)
2E V V
d d
E (C) (D)
(C) E (D) V V
2
Solution:  The capacitor is isolated and fringing effects Solution: 
Charge transit time
are neglected then the energy of capacitor is d
t=
1 V
E= CV 2
2  Where the velocity
1 Q2 v∝ v
E=
2 C  d
∴ t∝
Q = constant v 
1 Hence, the correct option is (C).
E∝  (1)
C 7. Consider the charge profile shown in the figure. The
εA resultant potential distribution is best described by
C=  [2016]
d 
ε, A are constant p(x)

1 p1
C ∝ (2)
d
From (1) and (2) b 0
a x
E ∝ d
E1 d p2
= 1
E2 d2

E d (A)  V(x)   (B)  V(x)
=
E2 2d

E2 = 2E b 0 b 0
a x a x
Hence, the correct option is (A).
9.10 | Electromagnetic Theory

(C)  V(x)   (D)  V(x) Solution: (3)



r = xax + ya y + zaz (1)
b 0 b 0 
r =r
a x a x
2
div( r ∇(ln r ))
Solution:  The resultant potential distribution for the  1  ∂r ∂x ∂r  
given charge profile can be better described by (d) be- = div r 2 . 2  axˆ + ayˆ + azˆ  
 r  ∂x ∂y ∂z  
cause the input is having unit step change with a dis-
continuity outside the given interval from b to a, thus  dr ∂r ∂r 
its potential distribution with a linear change from b to = div r1 axˆ + r ayˆ + r azˆ 
a and constant values at the remaining values.  ∂x ∂y ∂z 
(2)
Hence, the correct option is (D). Put equation (1) in equation (2)
8. In a source free region in vacuum, if the electrostatic So, div( r 2 ∇(ln r )) = div[r ( axˆ + ayˆ + azˆ )]
potential f = 2x2 + y2 + cz2, the value of constant c must
be _______. [2015] ∂r ∂r ∂r
Solution:  In source free region ∇.E = 0 = + +
∂x ∂y ∂z

Where E = –∇.f
div r 2 1n = 3
⎡ ∂ ∂
( 2 2 2
) 2 2 2
( ⎤
⎢ ∂ x 2 x + y + cz aˆ x + ∂ y 2 x + y + cz aˆ y + ⎥ ) 11. Given the vector
 E  =   –⎢ ⎥ 
⎢ ∂ ⎥

( 2 2 2
⎢ ∂ z 2 x + y + cz aˆ z ) ⎥

A = (cos x )(sin y )aˆ x + (sin x )(cos y )aˆ y where

aˆ x , aˆ y denote unit vectors along x, y directions, respec-



E = − ⎡⎣ 4 xaˆ x + 2 yaˆ y + 2cz aˆ z ⎤⎦ tively. The magnitude of curl of A is______ [2014]
 Solution: (0)
Source free region ∇.E = 0
axˆ ayˆ azˆ
⇒ – [4 + 2 + 2c] = 0  ∂ ∂ ∂
∇× A =
2c = –6 ∂x ∂y ∂z
c = –3 Ax Ay Az


A = (cos x )(sin y )ax + (sin x )(cos y )ay
Hence, the correct Answer is (–3.1 to –2.9). ˆ ˆ
9. The force on a point charge +q kept at a distance d Ax = (cos x) (sin y)
from the surface of an infinite grounded metal plate in Ay = (sin x) (cos y)
a medium of permittivity ∈ is Az = 0
(a) 0
axˆ ayˆ azˆ
q2
(b) away from the plate  ∂ ∂ ∂
16π ∈ d 2 ∇× A =
∂x ∂y ∂z
q2 cos x sin y sin x cos y 0
(c) towards the plate
16π ∈ d 2
2 = az1 (cos x cos y − cos x cos y ) = 0
(d) q towards the plate [2014] 
4π ∈ d 2 ∴ ∇× A = 0

Solution: (c)
1 2. A region shown below contains a perfect conducting
Direction if force will be +q to –q therefore, towards to  2 sur-
half-space and air. The surface current Ks on
= xthe

q2 2
face of the perfect conductor is Ks = x amperes per
the plate, F =
 16π Ed 2 meter.

10. If r = xax + ya y + zaz and |r| = r then div(r2 The tangential H field in the air just above the perfect

(lnr))_____. [2014] conductor is
Chapter 1  Basics  |  9.11

 4 × 1011 × (5 × 10 −6 ) 2 
= −  20 × 105 × (5 × 10 −6 ) + 
 2 

ψ B −ψ A = −[10 + 5]

ψ B −ψ A = −15V
(a) –(x + z) amperes per meter 
(b) ( xˆ + zˆ ) amperes per meter Given F = zaˆ x + xaˆ y + yaˆ z . If S represents the portion
14. 
 

(c)
− ẑ 2 amperes per meter
of the sphere x2 + y2 + z2 = 1 forz ≥ 0, then ∇ × F ⋅ ds ∫
s
_____. [2014]
ẑ 2 amperes per meter
(d) [2014]
Solution: (d)
Solution: (3.14)
Properly of magnetic boundary conditions states 

 F = zaxˆ + xayˆ + yazˆ
H1t − H 2t = ks
 axˆ ayˆ azˆ
2
Where, Ks =is xsurface current.  ∂ ∂ ∂
∇× F =
Here, H2t = 0 ∂x ∂y ∂z
H1t = 2 xˆ × yˆ z x y

 
H1t = 2 z A/m
ˆ
A = ∇ × F = ax + ay + az
ˆ ˆ ˆ
Hence, the correct option is (d).  
13. The electric field (assumed to be one-dimensional) ∫
(∇ × F ).ds =
S
∫∫
A. dydz ax

between two points A and B is shown Let ΨAand ΨBbe  


the electrostatic potentials at A and B, respectively. The

+ ∫ ∫ A. dxdz ay + ∫ ∫ A. dxdy az
value ofΨAΨB in Volts is_______
For, z ≥ 0

∫∫
(∇ × A).ds = ∫∫ (axˆ + ayˆ + azˆ).(azˆ dxdy)
S


= dxdy
S

∫∫ (∇ × A). ds = 3.14
S
[2014]
Solution: (–15)

dv 15. If E = −(2 y 3 − 3 yz 2 ) is the electric
E=−
dx 2 2
xˆ − (6 xy − 3 xz ) yˆ + (6 xyz ) zˆ
20 × 105 field in a source free region, a valid expression for the
= 20 × 105 r/m + x electrostatic potential is
5 × 10 −6 (a) xy3 – yz2 (b) 2xy3 – xyz2
E = ( 20 × 105 + 4 × 1011 x )v/m (c) y + xyz
3 2
(d) 2xy3 - 3xyz3
 [2014]
ψB 5 µm
Solution: (d)

ψA

dv = − E .dx
0
 3 2 2 2
E = −(2y − 3yz )xˆ − (6xy − 3xz )yˆ + (6 xyz )zˆ

E = −∇V
5×10 −6
 4 × 1011 r 2  For, V = 2xy3 – 3xyz2
ψ B −ψ A = −  20 × 105 x +  ∂v
 2  0 = 2 y 3 − 3 yz 2
∂x

9.12 | Electromagnetic Theory

∂v For, r < a, I = Jπr2


= 6 xy 2 − 3 xz 2
∂y µ0 J r
So, B =
2
∂v
= −6 xyz So, B ∝ r
∂z
For r > aI = Jπa2
which satisfy given E.
µ J π a2
Hence, the correct option is (d). So, B = 0
2π r
1 6. The direction of vector A is radially outward from the µ0 Ja 2
origin, with |A| =krn where r2 = x2 + y2 + z2 and k is a =
πr
constant. The value of n for which .A = 0 is

(a) –2 (b) 2 1
B ∝
(c) 1 (d) 0 [2012] r
Solution: (a) Hence, the correct option is (c).
 1 ∂ 2 1 ∂ 1 ∂
∇. A = 2 ( r Ar ) + ( Aθ sin θ ) + Aφ 18. A hole of radius b(b < a) is now drilled along the length
r ∂r r sin θ ∂θ r sin θ ∂φ of the wire at a distance d from the center of the wire as
shown below.
1 ∂ 1 ∂
+ ( Aq sin q ) + Af
r sin q ∂q r sin q ∂f
(Spherical coordinates)
Here, |A| = krn

A = kr n arˆ [radically outward]
 1 ∂ 2 n
So, ∇. A = 2 ( r kr ) + 0 + 0
r ∂r The magnetic field inside the hole is
(a) uniform and depends only on d
 k ∂ n+ 2
∇. A ⇒ 2 (r ) (b) uniform and depends only on b
r ∂r
(c) uniform and depends on both b and d
 (d) non uniform [2012]
∇. A = 0 only if rn + 2 = constant
Solution: (c)
∂ The magnetic field inside the hole will be uniform and
So, (constant) = 0
∂r depend upon both b and d.
rn + 2 = 1 at n + 2 = 0 i.e. n = −2
Hence, the correct option is (a).
Common Data for Questions 17 and 18
An infinitely long uniform solid wire of radius a carries

a uniform dc current of density j .

17. The magnetic field at a distance r from the center of the
Hence, the correct option is (c).
wire is proportional to
1 9. The electric and magnetic fields for a TEM wave of fre-
(a) r for r < a and 1/r2 for r > a
quency 14 GHz in a homogeneous medium of relative
(b) 0 for r < a and 1/r for r > a permittivity er and relative permeability μr = 1are given
(c) r for r < a and 1/r for r > a by
(d) 0 for r < a and 1/r2 for r > a [2012] j ( wt − b y )
E z = Ee 4 zˆ
Solution: (c)
j(ω t − 280π y)
H = 3e u z A/m
For infinitely long uniform wire,
|B| is given as, Assuming the speed of light in free space to be 3 ×104
µ0 I m/s, intrinsic impedance of free space to be 120p the
B = relative permittivity er of the medium and the electric
2π r
field amplitude Ep are
Chapter 1  Basics  |  9.13

(a) er = 3 Ep = 120p (b) er = 3 Ep = 360p Solution: (b)


(c) er = 9 Ep = 360p (d) er = 9 Ep = 120p  
V = ∇ × A (given)
 [2011]  
Solution: (d) ∫A.dl = ∫∫ (∇ × A)ds [using stoke’s theorem]
 ( wt − 280p y ) C S
E = E pe j 4 zˆ v/m    

 j ( wt − 280π y )
 ∫C
∇ × A ⋅ dl = ∫∫ V ⋅ dS
H = 3e 4 x A/m SC

V = 3 × 108 m/s [Free space] Hence, the correct option is (b).
η [Free space] = 120π 2 1. A magnetic field in air is measured to be
General equation is given as,  x y 
j ( wt − β y ) B = B0  2 yˆ − 2 xˆ 
x +y x + y 
E z = Ee 4z 2 2

j ( wt − β y )
H x = He 4x What current distribution leads to this field?[Hint: the
algebra is trivial in cylindrical coordinates]:
E Ep 120π
η= = =  B zˆ  1 
H 3 ∈ r J =− 0  2
(a)  , r ≠ 0
m 0  x + y 2 
2π 1
β= = 280π ⇒ meter
λ 140  B zˆ  2 
J =− 0  2
(b)  , r ≠ 0
v m 0  x + y 2 
λ= 
f J = 0, r ≠ 0
(c)

V = λ.f  B zˆ  1 
1 J =− 0  2
(d)  , r ≠ 0  [2009]
= × 14 × 109 m 0  x + y 2 
140
Solution: (c)
V = 1 × 108 m/sec
  x y 
C 3 × 108 B = B0  2 yˆ − 2 xˆ 
V= ⇒ = 1× 108 2
x + y2
∈ r ∈ r  x + y 
(1)

∈ r=9 Convert rectangular coordinates to cylindrical
coordinates.
3 × 120π
So, E p = i.e., x = ρ cos φ
∈ r   
E = 120π ax = cos φ a ρ − sin φ aφ
p
Hence, the correct option is (d). y = ρ sin φ
      
20. If a vector field A ⋅ dl = ∫
V ⋅ dS is related to
C ∫∫
SC
ay = sin φ a ρ + cos φ aφ
    z=z
another vector field A through V = V × A , which of 
  
the following is true? Note C and SC refer to any closed az = az
contour and any surface whose boundary is C put these values in equation  (1)
     
∫
(a) V ⋅ dl = C
A ⋅ dS ∫∫
SC
B = B0 aφ

    B 
H = B aφ = constant i.e. 0 ap
(b) A.dl =∫
C
∫∫V .ds
S
µ m
       B  
∫
(c) ∇ × V ⋅ dl =
C ∫ ∫ ∇ × A ⋅ dS So, J = ∇ × H = ∇ ×  0 aφ 
SC   µ 
    J =0
(d) C ∫ A ⋅ dl = ∫∫
SC
V ⋅ dS
 [2009] Hence, the correct option is (c).
9.14 | Electromagnetic Theory

22. A parallel plate air-filled capacitor has plate area of 10-4 ∈=80 . ∈0 The surface charge density on the conductor
m2 and plate separation of 10–3 m. It is connected to a 0.5 is
V, 3.6 GHz source. The magnitude of the displacement (a) 0 C/m2
current is (b) 2 C/m?
(e0 = 1/36p×10–9F/m) (c) 1.8 × 10–11 C/m2
(a) 10 mA (b) 100 mA (d) 1.41 ×10–9C/m2
(c) 10A (d) 1.59 mA [2004] (∈= 10-9)/(36 π)F/m)s [2002]
Solution: (a) Solution: (d)
Displacement current is equal to, D = ∈E
Id = AJd also, D = ρs [Gauss Law]
A = area of plate ∈E = ρs = ∈r∈oρs
Id = current density 1 10 −9
ρ s = 2 × 80 × ×
dD ∈ dE 36 π
Jd = =
dt dt −4 2
ρ s = 1.41× 10 c/m
E = v/d
Hence, the correct option is (d).
25. Match List-I with List-II and select the correct answer
using the code given below the Lists:
List-I
 
A. ∇.H = J
  

B. E.dI = − B.ds
C

S

 ∂ρ
C. ∇⋅J =
∈ ×V ∂t
I d = A × 2π × F ×
d List-ll
1. Continuity equation
1 0.5
= 10 −4 × 2π × 3.6 × 109 × × 10 −9 × −3 2. Faraday’s Law
36π 10 3. Ampere’s Law
4. Gauss’s Law
I = 10 mA
d 5. Biot-Savart Law
Hence, the correct option is (a). Codes:
23. If the electric field intensity is given by E = (xux + yuy + A B C
zuz)volt/m the potential difference between X (2, 0, 0)
and Y (1, 2, 3) is (a) 3 2 1
(a) +1 volt (b) -1 volt (b) 2 1 3
(c) +5 volt (d) +6 volt [2003] (c) 2 3 1
Solution: (c) (d) 1 2 3
 [1994]
E = (xVx + yVy + zVz) v/m, x(2, 0 ,0) and y(1, 2, 3)
Solution: (a)




V = − E .dl (A) ∇.H = J → Ampere Law

 

 2  0  0  
 ∫
(B) E .dl = − ∫ B.ds

 1
∫ ∫
= −  x dxVx + y.dyVy + z.dz Vz 
 
C S
2 3 dB
= 5v ∇ × E = − → Faraday Law
dt
Hence, the correct option is (c).  ∂ρ
24. The electric field on the surface of a perfect conduc- ∇⋅J =
(C) continuity equation.
∂t
tor is 2 V/m. The conductor is immersed in water with
Chapter 1  Basics  |  9.15

Hence, A – 3, B – 2, C – 1 −1 1

∫∫x e
2 z
Hence, the correct option is (a). B1 = dxdy
0 0
26. Given V = x cos 2 yiˆ + x 2 e 2 ˆj + z sin 2 ykˆ and S the
1
surface of unit cube with one corner at the origin and B1 = [e − 1]
edges parallel to the coordinate axis, the value of the 3

∫∫
−1 1
integral ˆ is_____.
V .ndS [1993]
∫∫x e
2 z
c at, y = 1, B2 = − dxdy
 1 0 0
Solution: V =
∫∫ V .n ds = sin( 2)
2 1
B2 = − [e − 1]
3
We have given, V = x cos 2 yi + x 2 e z j + z sin 2 yk can
be written as, B = B1 + B2 = 0 (2)
  

V = x cos 2 ax + x 2 e z ay + z sin 2 az y2 x2
 

 C= ∫∫ Vz dxdy az
Now, ∫∫
V .n ds will be equal to,
C
y1 x1

1 1

∫ ∫ 0.dxdy = 0
z2 y2
  z2 x2   y2 x2   at, z = 0, C1 =
∫∫ Vx dydz ax + ∫∫
Vy dxdz ay + Vz dxdy az ∫∫ 0 0
z1 y1 z1 x1 y1 x1
1 1

∫ ∫ sin
2
Here, Vx = x cos 2 y at, zF – 1, C2 = − y dxdy
 2 0 0
Vy = x ez
1  sin( 2) 
Y C2 = − 1 −
2 2 

M R
1  sin( 2)  (3)
Q C = C1 + C2 = − 1 −
2 
N
2
T S
Added up equations (1), (2), (3)
X V=A+B+C
O P
1  sin( 2)  1  sin( 2) 
= 1+  + 0 − 1 −
2 2  2 2 
Z
 1
z2 y2
  V= ∫ ∫V .n ds = 2 sin(2)
A=
z1 y1
∫∫ Vx dydz ax
27. For a uniformly charged sphere of radius R and charge

density ρ, the ratio of magnitude of electric fields at
at, x = 0, A1 = 0. distances R/2 and 2R from the centre, i.e.,
−1 1
E ( r = R/2)
∫ ∫ cos
2 is _____ [1993]
At x = 1, A2 = y dydz
E (r = 2R)
0 0
Solution: (2)
1  1 + sin( 2) 
A2 = +   [For r < R]
2 2 

1 1 + sin( 2)  ∫
D.ds = 2
So, A =   (1)
2 2  D.S=2
4
z2 x2
  ρv × π r 3
3 ρ .r
B= ∫∫
z1 x1
Vy dxdz ay aty = 0,

D=
4π r 2
= v
3
9.16 | Electromagnetic Theory

Solution: (c)
I(t) = I0 cos (wt)
We know, H = NI
So, H = NI0cos(wt)
B = H0H
B = μ0NI0 cos (wt)
According to maxwell second equation,

 dB 
E=
D ρ v .r
=
∫
E .dl = −
dt∫.ds

t 3E
2
R So, E .2π R = µ0 NI 0 d [cos( wt )] × π R
Here, r = 2 dt y
2
ρv.R µ0 NI 0 wR sin w (t )
E= E=
6E y

4 Hence, the correct option is (c).
ρ v × π R3
(For r>R] 3 ρ v R3
D= = 29. The electric field strength at a far-off point P due to
4π R 2 3r 2 a point charge, +q located at the origin, O is 100 mil-
ρ v R3 livolts/meter. The point charge is now enclosed by a
E= perfectly conducting hollow metal sphere with its cen-
3Er 2
tre and the origin, O. The electric field strength at the
Here, r = 2R point, P
ρv R (a) remains unchanged in its magnitude and direction.
E= (b) remains unchanged in its magnitude but reverse in
12t
direction.
(c) would be that due to a dipole formed by the charge,
+q, at O and -q induced.
(d) would be zero. [1989]

Solution: (d)
According to Gauss Law:—The total displacement or
electric flux through any closed surface surrounding
charges is equal to the amount of charge enclosed.

E[r = R/2] ∫
D.ds = 2
So, =2 Here, total enclosed charge is, −q + q = 0 thus, D = 0
E [ r = 2 R]
D
also, =
E = 0
28. A long solenoid of radius R, and having N turns power E
unit length carries a time dependent current I(t) = I0 Hence, the correct option is (d).
cos (wt). The magnitude of induced electric field at a
distance R/2 radially from the axis of the solenoid is 30. Which of the following field equations indicate that the
free magnetic charge do not exist
R
(a) m 0 NI 0w sin (wt) 1
2 (a)
H= ∇× A
µ
(b) R
µ0 NI 0ω cos (ω t) 1dI × R
4 (b)
H=
4p R 2
∫
(c) R
µ0 NI 0ω cos(ω t )
4 ∇.H = 0
(c)
Rμ0NI0wsin (wt)
(d) [1993] ∇ × H = J
(d) [1990]
Chapter 1  Basics  |  9.17

Solution: (c)  1 ∂ 2 1 ∂
PV = ∇.D = 2 ( r Dr ) + .
As we know, ∫ B.ds = 0 or, ∇.(Bµ=H0) [Solenoidal
=0 r ∂r r sin θ ∂θ
1 ∂Dφ
in nature] B = μH thus, ∇.( µ H ) = 0 ( Dθ sin θ ) +
r sin θ ∂φ
∇.H = 0
1 2
Hence, the correct option is (c). = 2 4r 3 + 2 sin θ .cos θ
r r sin θ
31. Vector potential is a vector
4
(a) whose curl is equal to the magnetic flux density = 4 r + cos θ
(b) whose curl is equal to the electric field intensity r
(c) whose divergence is equal to the electric potential  1 
= 4  r + cos θ 
(d) which is equal to the vector product E × H [1988]  r 

Solution: (a)  
The term vector potential physical stands for work ∫
2. Evaluate the integral, r ⋅ dr , where C is the helical
path described by, x = cost,
c y = sint, z = t, joining the
done per basic cause (i.e. current element in magnetic
 w points given by t = 0 and t = p/2 [1994]
field). Hence, A =   Solution: In Cartesian coordinates,
I .dl 
r = xaˆ x + yaˆ y + zaˆ z
is vector so, potential is a vector. Whose curl will be

equal to, ∇ × A = B also, 
dr = dx aˆ x + dy aˆ y + dz aˆ z
 
∇× A = µH 1 Q

Hence, the correct option is (a). ∫ ∫


I = r .dr = ( xdx + ydx + zdz ) =  x 2 + y 2 + z 2 
2   P
C C
3 2. On either side of a charge-free interface between two
with, t = OP (y1y1z) = (1, 0, 0)
media
(a) the normal components of the electric field are π
with, t = Q( x, y, z ) = (0,1, π / 2)
equal 2
(b) the tangential component of the electric field are 1  2  a 1  2 1 1  2 π / 2 π 2
equal I= x + y + z =
2  1 2   0 2   0 8
(c) the normal components of the electric flux density
are equal 3. Three electrostatic point charges are located in the xy-
(d) the tangential components of the electric flux den- plane as given below:
sity are equal [1988] +Q at (-a/2, 0), + Q at (a/2, 0) and
Solution: (b) and (c) - 02Q at (0, > /3/2)
Conditions at Boundary surfaces. Calculate the coordinate of the point, P, on the y-axis,
(a) Et1 = Et2 (b) 
Ht1 = Ht2 where the potential due to these charges is zero. Also,
(c) Bn1 = Bn2 (d) 
Dn1 = Dn2 calculate the magnitude of the electric field strength at

Hence, the correct option is (b) and (c). P. At the point, P what is the angle between the equipo-
tential passing through P and the y-axis? [1995]
Solution:
Five-marks Questions
2
1. Given that D = r ar + 2Sinθ aθ in spherical coordinate
system, where D is the electric flux density, find the
charge density p? [1987]
 2
Solution: Given, D = r ar + 2sin θ aθ and
  
  D = Dr a −1r + Daθ + Dφ aφ
So,
Dr = r2
Dϑ = 2 sin ϑ
Dø = 0
and,
9.18 | Electromagnetic Theory

 −9  9   a 
point, charge location +QA  , 0  , +QB  , 0  At P(a, y) = P  o, 
  2  5   2 3
3 
and −2Q C  0,
 2 
a −3 / 2 
  2 2 
   −a  a + a  
(i) Let, P(a, y) be the point on y-axis, where V = V1 +   2 3  4 12  
−Q  
V2 + V3 due to these change is zero, V = V1 + V2 + E=  a
2π E0   −2  x
V3 = 0 
+  a − 3 a  
Q  1 1 2    2 3 2  

4π E0  AP + BP − CP  = 0
  3Q
E = Ey =
But AB = BP, so, 4π E0 a 2
1 1 2 
+ − or AP = CP (iii) The direction of E at P is the direction of the
AP AP CP normal to the equipotential surface (V = 0) at that
points in the direction
 of the decreasing values of V.
a2 3 ⇒y= a
+ y2 = y − a, The direction of E is in the –ve y direction, the
4 2 2 3 angle between the equipotential surface and y-axis
 a  is zero.
P ( a, y ) = P  0, 
 2 3 4. Given an irrotational vector field
 
(ii)
V at point P(0, y) F = (k1 xy + k2 z 3 )a x +
 
Q  1 1 2  (3 x 2
− k z ) a y + (3 xz 2
− y ) a z
V=  AP + BP − CP  3
4π E0   
Find ∇ F at (1, 1, – 2). [1998]
Q  1 1 
= − Solution: Given
2π E0  AP CP  
F = ( k1 xy + k2 z 3 )aˆ x + (3 x 2 − k3 z )aˆ y + (3 yz 2 − y )aˆa
 
  ∂ ∂ ∂
Q  1 1  ∇.F = Fx + Fy + Fz
V= − ∂y ∂y ∂z
2π E0  a 2 3 

 + y2 y− a
 4 2  = k1y + 0 + 6xz = k1y + 6xz

 at (1, 1, −2)
∂ ∂ ∂  
E = −∇V = −  Vaˆ x + Vaˆ y + Vaˆ z 
 ∂x ∂y ∂z  ∇.F = k1 (1) + 6(1)( −2) = k1 − 12.
∂V 5. Given E= 10e-j(4x - kt) - V/m in free space.
=0 (a)  Write all the four Maxwell’s equations in free
∂x
space.
∂V (b) Find D × E.
=0
∂z (c) Find H. [2000]
Solving Solution: Given, E = 10e −4 j ( 4 x − kt ) a yV/m
 −3 / 2  represents uniform plane wave travelling in x direction
 2 
− 1  a + y 2   with velocity v – k/4 and Ex = 0, E2 = 0
  
∂v Q  2 4  So,
=  
∂y 2π E0  
−2
  ∂
3 E y = − j 40e − j ( 4 x − kt )

( 2 y ) +  y − a 
  2   ∂y
  

  2 −3/ 2  ax ay az ax ay az

− y  a + y 2    ∂ ∂ ∂ ∂
   4   ∇× E = = 0 0
−Q   ∂x ∂y ∂z ∂y
E=  
2p E0   
−2  Ex Ey Ez 0 Ey 0
 +  y − 3 a  a y 
  2  
Chapter 1  Basics  |  9.19

  ∂ 6. A system of three electric charges lying in a straight


∇ × E = ax (0) − a y (0) + a2 . E y line is in equilibrium. Two of the charges are posi-
∂y
tive with magnitudes Q and 2Q, and are 50 cm apart.
= j 40e − j ( 4 x − kt ) = 40e j ( 4 x − kt + 90 ) Determine the sign, magnitude and position of the third
E charge. [2001]
(c) For uniform plane wave, η = , E = E y aˆ y
H Solution:
   Since, wave  is travelling in x direction so, the direc-
tion of H is in z-direction. 1 2 3

yˆ + zˆ = xˆ
Ey Let Q3 be the third change located at the distance x.
η = η0 = 120π =
Az From Q
Ey 10 − j ( 4 x − kt ) 2Q Q3 (1)
H2 = = e V1 = +
120π 120π 4π E (50) 4π E ( x )
  1 − j ( 4 x − kt )  Q Q3
H = H 2 az = e a2 V2 = + (2)
12π 4π E (50) 4π E ( x − 50)
Chapter 2
Uniform Plane Waves
Solution:  To find out the power through z = 0 plane,

One-mark Questions ⎛ μ⎞
the plane area, medium intrinsic impedance η ⎜ =
1. The permittivity of water at optical frequencies is ⎝ ε ⎟⎠
1.75  e0. It is found that an isotropic light source at a are required.
distance d under water forms an illuminated circular Hence, the correct option is (D).
area of radius 5 m, as shown in the figure. The critical
angle is qc. 3. If a right-handed circularly polarized wave is inci-
The value of d (in meter) is _______________. dent normally on a plane perfect conductor, then the
 [2017] reflected wave will be [2016]
(A) right-handed circularly polarized
(B) left-handed circularly polarized
5m (C) elliptically polarized with a tilt angle of 45°
Air
(D) horizontally polarized
Solution:  When a right hand circularly polarized wave
θC d is incident normally on a plane perfect conductor,
Water
i.e., σ = ∞ then orientation changes by 180° at any of
the two fields present. Therefore, option (B) is correct
Light Hence, the correct option is (B).
Source
4. In the electric field component of a plane wave travel-
Solution:  →
ling in a lossless dielectric medium is given by E (z, t)
n   ε  ∧ ⎛ z ⎞
θC = sin −1  2  = sin −1  r 2  = 49.1° = a y 2cos ⎜108 t − ⎟ V/m. The wavelength (in m) for
 n1   ε r1  ⎝ 2⎠
5 the wave is _______ [2015]
tan 49.1 = = 1.5447
d Solution:  Given equation
⇒ d = 4.33 m → ⎛ z ⎞
E (z, t) = â y 2cos ⎜108 t − ⎟ V m
Hence, the correct answer is (4.30 to 4.36). ⎝ 2⎠ 
1
2. Let the electric field vector of a plane electromag- β=
2
netic wave propagating in a homogeneous medium be
expressed as E = xEˆ x e − j (ωt − β z ) where the propagation ω = 108
constant β is a function of the angular frequency ω. 2π 1
Assume that β(ω) and Ex are known and are real. From β= =
λ 2
the information available, which one of the following
CANNOT be determined?  [2016] ⇒ λ = 2 2π 
(A) The type of polarization of the wave
(B) The group velocity of the wave =
8.88m
(C) The phase velocity of the wave
(D) The power flux through the z = 0 plane Hence, the correct Answer is (8.85 to 8.92).
Chapter 2  Uniform Plane Waves  |  9.21

5. The electric field of a uniform plane electromagnetic ( − j 25 x +


p
)
wave is E z = 10e 2 azˆ


⎛→ →
⎞ Both component are of equal magnitude with phase
E = ⎜ a x + j 4 a y ⎟ exp [j(2π × 107t – 0.2z)]
⎝ ⎠ difference 90°.
The polarization of the wave is [2015]
(A) right handed circular
(B) right handed elliptical
(C) left handed circular
(D) left handed elliptical
Solution:  From the given equation
|Ex| ≠ |Ey|

So it is elliptically polarized wave So, polarization of the wave is left circular.


Generalized expression for left elliptically polarized 2π
electric field is β= = 25
λ
( )
E = E x aˆ x + E y aˆ y e j (ωt − β z )
λ=

25
Hence, the correct option is (D).
6. A coaxial-cable with an inner diameter of 1 mm and C 3 × 108 × 25
f = =
outer diameter of 2.4 mm is filled with a dielectric of λ 2π

relative permittivity 10.89. Given µ0 = 4π 10-7H/m, f = 1.2 GHz


10 −9 Hence, the correct option is (a)
ε0 = F/m the characteristic impedance of the 8. A plane wave propagating in air with
36π
cable as ( −8aˆ x + 6 aˆ y − 5aˆ z )e j (wt − 3 x − 4 y ) V / m is incident on
(a) 330 Ω (b) 16 W ≤ 0 the field
a perfectly conducting slab positioned at x
(c) 143.3 W (d) 43.4 W [2012] of the reflected waves is
Solution: (b)
( −8aˆ x − 6 aˆ y − 5aˆ z )e j (wt + 3 x + 4 y ) V / m
(a)
136 D
z0 = log
∈r d ( −8aˆ x − 6 aˆ y − 5aˆ z )e j (wt + 3 x + 4 y ) V / m
(b)

∈r = 10.89, D = 2.4 mm, d = 1 mm ( −8aˆ x − 6 aˆ y − 5aˆ z )e j (wt + 3 x + 4 y ) V / m
(c)

188  2.4  ( −8aˆ x − 6 aˆ y − 5aˆ z )e j (wt + 3 x + 4 y ) V / m 


(d) [2012]
z0 = log  
10.89  1  Solution: (c)

Z0 = 15.89 Ω Ei = (8axˆ + 6 ayˆ + 5azˆ )e j ( wwt + 3 x + 4 y ) v /m

Hence, the correct option is (b) For perfectly conducting slab, Et = 0
7. The electric field of a uniform plane electromagnetic Ei = −Er
wave in free space, along the positive x direction, is
 Γ x = −1
given by E = 10( a y + jaz )e − j 25 x . The frequency and

Direction of propagation of reflected wave for normal


polarization of the wave, respectively, are components reversed.
(a) 1.2 GHz and left circular 
En = − x direction
(b) 4 Hz and left circular 
r

(c) 1.2 GHz and right circular So, Er = − Ei [with – x direction]
(d) 4 GHz and right circular [2012] 
Ei = (8axˆ + 6 ayˆ + 5azˆ )e j ( wt + 3 x + 4 y ) v /m
Solution: (a) Hence, the correct option is (c)

E = 10e − j 25 x ayˆ + j10e − j 25 x azˆ 9. Consider the following  statements regarding the com-

plex Poynting vector P for the power radiated by a point
E y = 10e − j 25 x ayˆ source in an infinite homogenous and lossless medium.

9.22 | Electromagnetic Theory

  
Re ( P ) denotes the real part of, S denotes a spherical i  wt − x −
p 3p
z
  l l 
surface whose centre is at the point source, and n̂ ˆ 0e 
E = yE
(a) 
denotes the unit surface normal on S. Which of the fol-
 p 3p 
lowing statements

is TRUE?  i  wt − x + z
 l l 
(a) Re ( P ) remains constant at any radial distance (b) ˆ 0e 
E = yE 
from the source
 3p p 
(b) Re ( P ). increases with increasing radial distance  i p t +

x+ z 
l l 
from the source (c) ˆ 0e 
E = yE

∫∫
 p 

(c)  Re( P ). nˆ dS remains constant at any radial dis-
s

(d)
i  wt −
ˆ 0e 
E = yE

3p
l
x− z
l 
 [2007]
tance from the source
Solution: (a)

(d)  ∫∫
 Re ( P ). nˆ dS decreases with increasing
tan 30° =
βz
=
1
s
βx 3
radial distance from the source [2011]
Solution: (c)

β x = 3β z

∫∫
 s
Re ( p). nˆ ds remains constant at any radial
2π  3π
2
  π 2
β= =   +  
distance from the source.  λ

w   λ
Hence, the correct option is (c)

10. The electric field component of a time harmonic = ( Bx ) 2 + ( Bz ) 2



plane EM wave travelling in a nonmagnetic lossless − j b x x − j b z . z jwt
E x z t = E0 e
dielectric medium has amplitude of 1 V/m. If the rela- ( , , ) e e yˆ
tive has permittivity of the medium is 4, the ≤≤ mag-
nitude ofthe time average power density vector (in W/  3p p 
ˆ 0 e j  wt −
E ( x, z , t ) = yE x− z
m2) is  l l 
 

1 1
(a) (b)
30p 60p
1 1
(c) (d)  [2010]
120p 240p

Solution: (c)
E =V m

∈r = 4
Hence, the correct option is (a)
E 2 1 2. The electric field of an electromagnetic wave propagat-
Pav = ing in the positive z-direction is given by

E = aˆ x sin(wt − b z) + aˆ y sin(wt − b z + p / 2)
120π
η= = 60π Ω The wave is
∈r
(a) linearly polarized in the z-direction
(b) elliptically polarized
(1) 2 1
Pavg = = w/m 2 (c) left-hand circularly polarized
2 × 60π 120π (d) right-hand circularly polarized [2006]
Hence, the correct option is (c) Solution: (c)
11. A plane wave of wavelength λ is travelling in a direc- E = axˆ sin( wt − b z ) + ayˆ sin( wt − b z + p /2)

tion making an angle 30° with positive x-axis and 90° E x = axˆ sin( wt − b z )
with positive y-axis The E field of the plane wave can
be represented as (E0: s a constant) E y = ayˆ sin( wt − b z + p /2)

Chapter 2  Uniform Plane Waves  |  9.23

Ex and Ey both are out of phase by 90° with equal mag- Solution:(b)
nitude and, axˆ + ayˆ = azˆ
∂ 2 Ex ∂ 2 Ex
So, left circularly polarized = C2 ,
For the wave equation ∂z 2 ∂t 2
the wave will propagates in the z-direction.
Hence, the correct option is (b)
16. Identify which one of the following will NOT satisfy
the wave equation.
(a) 50ei(wt – 3z)
(b) sin[w (10z + 5t)]
(c) cos(y2 + 5t)
Hence, the correct option is (c) (d) sin(x) cos(t) [1999]
13. The magnetic field intensity vector of a plane wave is Solution: (c)
given by cos (y2 + 5t) does not satisfy wave equation.
R(x, t) = 10 sin (50000t + 0 004x + 30) aˆ y , where aˆ y Hence, the correct option is (c)
17. The intrinsic impedance of copper at high frequency is
denotes the unit vector in y direction'. The wave is
(a) purely resistive
propagating with a phase velocity
(b) purely inductive
(a) 5 × 104 m/s (b) 3 × 103 m/s (c) complex with a capacitive component
(c) 1.25 × 107 m/s (d) 3 × 106 m/s [2005] (d) complex with an inductive component
Solution: (c) [1998]
 Solution: (d)
H ( x, t ) = 10 sin(50000t + 0.004 x + 30)ayˆ
jw µ
w = 50,000 Intrinsic impedance, η =
σ + jw ∈
β = −0.004
For copper, σ >> wt
Vp = −1.25 × 107 m/s
Hence, the correct option is (d)
w
Vp = = phase velocity 1 8. The wavelength of a wave with propagation constant
β (0.1π + j0. 2π)m-1is
2
5 × 10 4 (a) m (b 10 m
= −3 0.05
−4 × 10
(c) 20 m (d) 30 m [1998]
VP = -125 × 107m/s
Solution: (b)
Hence, the correct option is (c)
Propagation constant of a wave is,
14. The depth of penetration of electromagnetic. wave in a χ = α + jβ
medium having conductivity σ at a frequency of 1 MHz
is 25 cm. The depth of penetration at a frequency of 4 given, χ = 0.1π + j 0.2π
MHz will be 2π
β= = 0.2π
(a) 6.25 cm (b) 12 50 cm λ
(c) 50.00 cm (d) 100.00 cm [2003]
Solution: (b) λ = 10 m
Hence, the correct option is (b) Hence, the correct option is (b)
1 9. The depth of penetration of a wave in a lossy dielectric
15. If a plane electromagnetic wave satisfies the equation increases with increasing
∂2 E ∂2 E (a) conductivity (b) permeability
2
= σ 2 2 the wave propagates in the
∂Z ∂t (c) wavelength (d) permittivity [1998]
(a) x-direction Solution: (c)
(b) z-direction Depth of penetration or skin depth for lossy dielectric
(c) y-direction is,
(d) xy plane at an angle of 45° between the x and z  f ( wt - bz )
E = E0 e
( axˆ + ayˆ )
directions [2001]
9.24 | Electromagnetic Theory

1 Solution: (d)
δ=
f The voltage in the loop is due to two reactions.

dB 
δ∝ λ
(i) Time varying magnetic field is VB = −
C
dt ∫. ds
Hence, the correct option is (c) (ii) Voltage induced in a loop moving with velocity ‘v’
20. The polarization of a wave with electric field in steady magnetic field.
     
vector E = E0 e j ( t − †z ) ( ax + a y ) is
∫
Vm = (V × B)dl
(a) linear L
(b) elliptical
(c) left hand circular
Vtotal = Vm + VB

(d) right hand circular [1998] Hence, the correct option is (d)
Solution: (a) 2 3. The intrinsic impedance of a lossy dielectric medium is
 f ( wt − β z ) given by
E = E0 e ( ax + ay )
Icon
the phase difference between Ex and Ey component is jωµ jω ∈
(a) (b)
zero σ µ
∴ the polarization of a wave is linear
jωµ µ
Hence, the correct option is (a) (c) (d)  [1995]
(σ + ω ∈ ) ∈
21. The time
 averaged Poynting vector, in W/m ,for a wave
2

with E = 24e j (ωt − β z ) a y V/m in free space is Solution: (c)
2.4 
The intrinsic impedance of a lossy dielectric medium is
2.4 

(a) a (b) az given as,
π p
4.8  jw µ
4.8 
− az  η=
(c) az (d) [1998] σ + jw ∈
p π
Solution: (a) Hence, the correct option is (c)

 2 4. Copper behaves as a
1 2
Pavg = E = −az (a) conductor always
2η (b) conductor or dielectric depending on the applied


Direction of power = -az since, wave is travelling in electric field strength


-az direction (c) conductor or dielectric depending on the frequency
 (d) conductor or dielectric depending on the elec
E = 24e j ( wt + β z ) ay v/m tric current density [1995]
|E| = 24 Solution: (a)
η = 120π ohm σ
Loss tangent factor for copper is >> 1 as, σ for cop-
( 24) 2 
 per = 5.8 × 107 mho/m wt
Pavg = ( −az )
2 × 120π ∈ = 8.85 × 10−12 F/m
2.4 
 jw µ w µ jπ / 4 wπ wµ
Pavg = − az w /m 2 η= = e = +j
π σ σ 2σ 2σ

Hence, the correct option is (a) Thus, copper behaves as a conductor always.
Hence, the correct option is (a)
22. A loop is rotating about the y-axis in a magnetic field
B = B0 cos(wt + f )aˆ T . The voltage in the loop is 2 5. A plane electromagnetic wave traveling along the +zdi-
rection, has its electric field given by Ex = 2 cos (wt) and
(a) zero Ey = 2cos(wt + 90)° the wave is
(b) due to rotation only (a) linearly polarized
(c) due to transformer action only (b) right circularly polarized
(d) due to both rotation and transformer action (c) left circularly polarized
[1998] (d) elliptically polarized [1994]
Chapter 2  Uniform Plane Waves  |  9.25

Solution: (a) z
Ex = 2 cos wt
Ey = 2 cos (wt + 90°)
Since E.M wave has two planar components of E-field
θi
both out of phase by 90° with equal amplitude so, wave x
is circularly polarized.
t
en
c id tor
In ec
v

∵ n1 = 1
βz
θi = tan–1
βx

=
tan–1 ( 2 ) 

⎛n ⎞
⇒ tan–1 ⎜ 2 ⎟ = tan–1 ( 2 ) 
⎝ n1 ⎠
n2 = 2
Also from figure, we can say wave is left circularly
polarized. ε r2 = 2
Hence, the correct option is (a)
ε r2 = 2

Two-marks Questions ∵ n1 = 1
1. A uniform plane wave traveling in free space and hav-
Hence, the correct answer is 1.9 to 2.1.
ing the electric field
2. The distance (in meters) a wave has to propagate in a

iˆ E = ( 2aˆ x − aˆ2 ) cos[6 3π × 108 t − 2π ( x − 2 z )]V/m medium having a skin depth of 0.1 m so that the ampli-
tude of the wave attenuates by 20 dB, is  [2018]
Is incident on a dielectric medium (relative permittivity (A) 0.12 (B) 0.23
> 1, relative permeability = 1) as shown in the figure (C) 0.46 (D) 2.3
and there is no reflected wave. Solution:  We know that the amplitude of e–field in the
z direction of propagation can be expressed as
Free space
Dielectric Ez = E0 −α Z
medium (ε r > 1) e
here E0 represents the maximum, amplitude that occurs
x at Z = 0.
Now we have

∴ 20 log10 ⎡ E0 ⎤ = 20
⎢ − z /8 ⎥
The relative permittivity (correct to two decimal places) ⎣ E0 e ⎦ 
of the dielectric medium is _______.  [2018] 1
Solution:  We know that uniform plane wave is travel- ∵ δ= = 0.1 m
α
ling with electric field vector being parallel to the plane
of incidence. Therefore it is parallel polarization. We log10 ⎡e10 z ⎤ = 1
⎣ ⎦ 
know that wave is not reflected otherwise it represents
total transmission. Angle of incidence, θi = Brewster ⇒ z = 0.23 m
angle θ13
Hence, the correct option is (B)
⎛ n2 ⎞
= tan–1 ⎜ n ⎟ 3. The electric field of a uniform plane wave travelling
⎝ 1⎠ along the negative z direction is given by the following
equation
9.26 | Electromagnetic Theory

(
EWi = aˆ x + jaˆ y E0e jkz. ) Wave is propagating along x-axis so field is in y-z plane
in 10 cm × 10 cm square
This wave is incident upon a receiving antenna placed P = ∫ Pavg .ds
at the origin and whose radiated electric field towards 
the incident wave is given by the following equation ⎛ x + aˆ y ⎞
ˆ
a
aˆ = ∫ 7.53aˆ x ⋅ ⎜
( 1
Ea = aˆ x + 2aˆ y EI e–jkr
r
)
⎝ 2 ⎠
⎟ ds

aˆ x + aˆ y
The polarization of the incident wave, the polarization Where is unit vector
of the antenna and losses due to the polarization mis- 2
match are, respectively, [2016] 10 −1 10 −1
7.53


(A) Linear, Circular (Clockwise) – 5dB
(B) Circular (clockwise), Linear –5dB
=
2
∫ ∫ dy dz
0 0 
(C) Circular (clockwise), Linear, –3dB
(D) Circular (anti clockwise), Linear, –3dB =
5.32 × 10–2 W
Solution:  Polarization of incident wave, =53.2 mW
Ewi = ( ax + ja y ) E0 e jkz Hence, the correct Answer is (53 to 54).
Direction of propagation ‘–z’ 5. Consider a uniform plane wave with amplitude (E0) of
10 V/m and 1.1 GHz frequency travelling in air, and
E xo = E yo incident normally on a dielectric medium with complex
relative permittivity (εr) and permeability (µr) as shown
Ey leads Ex by 90° in the figure.
Right (clock wise) circular polarization.
Air Dielectric
Polarization of antenna, η = 120π Ω μr = 1 – j2
ε r = 1 – j2
1 − jkr
Ea = ( ax + 2ay ) E I e
r  E = ?
10 cm
E XO = E I
 E0 = 10 V/m
Freq = 1.1 GHz
EYO = 3E

The magnitude of the transmitted electric field compo-
Ex and Ey are in phase
nent (in V/m) after it has travelled a distance of 10 cm
Linear polarization: The linearly polarized antenna sim- inside the dielectric region is ________. [2015]
ply picks up the in phase component of circularly polar-
Solution: 
ized wave which has two orthogonal linearly polarized
waves 90° out of phase. As a result LP antenna has a Air Dielectric
polarization mismatch loss of 0.5 (–3 dB) PLF (linear η1 = 120 π Ω µr = 1 – j2
to circular) = 0.5 = –3 dB. Eo = 10 V/m εr = 1 – j2
Hence, the correct option is (C). f = 1.1 GHz η2 = 120 π Ω
(From the given eqn, we see that both Ex and Ey both
are in phase which shows that there is linear polariza- E = Eo e– γ z for decaying electric field
tion in it with mismatch loss of –3 dB option (c))
γ= jωμ (σ + jωε ) [ σ <<j ωε ]
4. The electric field intensity of a plane wave travelling in
free space is given by the following expression E(x, t) =
= jωμ ( jωε )
ay 24 πcos(ωt – k0x) (V/m) 
In this field, consider a square area 10 cm × 10 cm on a = jω μ r ε r μo ε o
plane x + y = 1. The total time-averaged power (in mW) 
passing through the square area is ________. [2015] α + jβ = jω μo ε o (1 – j2)
2
Eo
Pavg =
Solution:  ân From above equation α = 2 ω μo ε o

2 × 2π × 1.1 × 109
=
(24π )2 âx
=
2 × 120π 3 × 1010 

=
0.4607 cm
=
7.53 âx 
Chapter 2  Uniform Plane Waves  |  9.27

Electric field strength after 10cm


E = 10e–10 × 0.4602
=
0.099 V/m

Hence, the correct Answer is (0.08 to 0.12).

6. The electric field of a plane wave propagating in a loss-


less non-magnetic medium is given by the following
expression [2015]
So, given wave is left elliptical polarized.
E(z, t) = ax 5 cos(2π × 109t + βz) Hence, the correct option is (a)
π 8. Assume that a plane wave in air with an electric field
+ ay 3 cos(2π × 109t + βz – )
2 
E = 10 cos(wt − 3 x − 3 z )aˆ y V/mis incident on a non-
The type of the polarization is
magnetic dielectric slab of relative permittivity 3 which
(A) Right-hand Circular
covers the region z > 0. The angle of transmission in the
(B) Left-hand Elliptical dielectric slab is_______degrees. [2014]
(C) Right-hand Elliptical
Solution: 30°
(D) Linear  
E = 10 cos( wt − 3 x − 3 z )ay
Solution:  From given wave equation
|EX| ≠ |Ey| so it is elliptically polarized wave at 3
tan θ1 = = 3
3 π
ωt + βz = 0 3aˆ y at β z =
2
then E(z, t) = âx 5    θ1 = 60°
Using snell’s law,
π
ωt + βz =
2 η1 sin θ1 = η2 sin θ 2
5aˆ x at β z = 0
∈1 sin 60° = ∈2 sin θ 2
E(z, t) = â y 3
t1 = 1, t2 = 3
So it is left-hand elliptically polarized.
3 1 1
Hence, the correct option is (B). sin θ 2 = × =
2 3 2

7. If the electric field of a plane wave is
 θ 2 = 30°
E ( z , t ) = xˆ 3cos(wt − kz + 30 )
− yˆ 4 sin(w − kz + 45 )( mV / m)

the polarization state of the plane wave is
(a) left elliptical (b) left circular
(c) right elliptical (d) right circular
[2014]
Solution: (a)

E ( z , t ) = xˆ 3cos( wt − kz + 30°) 9. The angle of incidence and the expression for are
− yˆ 4 sin( wt − kz + 45°)(mv/m) −j
p ×10 4 ( x + z )
  Eo
60 and
(a) ( aˆ x − aˆ z )e 3 2 V /m
E x = 3cos( wt − kz + 30°) 2

 p ×10 4 z
E y = −4 sin( wt − kz + 45°)  Eo −j
(b)
60 and ( aˆ x + aˆ z )e 3 V /m
 2
E y = −4 cos( wt − kz + 35°)
p ×10 4 ( x + z )
Eo −j
Phase difference between two components is 105° with 
45 and
(c) ( aˆ x − aˆ z )e 3 2 V /m
unequal magnitude. 2
9.28 | Electromagnetic Theory

π ×10 4 z magnetic field in Region-2 at x = 0 + is


Eo −j

(d)
60 and ( ax − az )e 3 V m  [2013]
2
Solution: (c)
∈r1 sin θ1 = ∈r2 sin θt 
(a) H 2 = 1.5ux + 30u y − 10uz A / m
θt = 19.2° (b) H 2 = 3uˆ x + 30uˆ y − 10uˆ z A / m

sin θi = 4.5 sin(19.2°) (c) H 2 = 1.5uˆ x + 40uˆ y A / m

sin θi = 2.12 × 0.3289
(d) H 2 = 3uˆ x + 30uˆ y + 10uˆ z A / m  [2011]
= 44.2° ~ 45° Solution: (a)

From property of magnetic field boundary relation.
2π 2π π × 10 4
β= = = Bx 2 + By 2 B = Bn2
λ 600 × 10 −6 3 n1

B × 1 = B × 2 [normal component]
and direction of Ei component is ax and –az so,
µ H = µ2 H x2
−j
π ×10 4
( x+ z) 1 x1
E0 
Thus, 45° and ( ax − az )e 3 2
v/m H1 = 3uxˆ + 3ouyˆ A/m
2
Hence, the correct option is (c) 3 ×1
10. The expression for H x2 = 4 x = 1.54 x
2
p ×10 4 z  
Eo −j

60 and ( aˆ x − aˆ z )e 3 V / m is H t1 − H t2 = − J s × an
2
− jp ×10 4 ( x − z )
0.23E0 30 uy − H t2 = − 10uyˆ × 4 xˆ
(a) Er = ( axˆ + azˆ )e 3 2 v/m
2
H = 30uy − 10uz
p ×10 4 z t2
E j   
− 0 ( aˆ x + aˆ z )e
(b) 3 V /m So, H 2 = H n1 + H t2
2 
p ×10 4 ( x − z ) H 2 = 1.5uxˆ + 30uy − 10uzˆ
E0 −j
0.44
(c) ( aˆ x + aˆ z )e 3 V /m
2 Common Data for Questions 12 and 13
p ×10 4 ( x + z ) A monochromatic plane wave of wavelength λ = 600
E −j
(d) 0 ( aˆ x + aˆ z )e 3 V /m [2013] µrn is propagating
  in the direction
 as shown in the fig-
2 ure below. Ei , Er , and Et denote incident, reflected,
Solution: (a) and transmitted electric field vectors associated with
the wave.
Er ∈1 − ∈2 1 − 4.5 −1.12
= = =
Ei ∈2 + ∈1 1 + 4.5 3.12

Er
= −0.359
Ei

  |Er| = 0.23
− jπ ×10 4 ( x − z )
0.23E0
Er = ( ax + az )e 3 2
v/m
  2
Hence, the correct option is (a)

11. A current sheet H 2 = 1.5uˆ x + 30uˆ y − 10uˆ z A / m lies Hence, the correct option is (a)
on the dielectric interface x = 0 between two dielectric 1 2. A plane wave having the electric field component
media with ε r1 = 1 , µr = 1 in Region-1 (x < 0) and Ei = 24 cos(3 × 108 t − b y )aˆ z V/m and travelling in free
1
ε r2 = 2 , µr2 = 2 in Region-2 (x > 0) If the magnetic space is incident normally on a lossless medium with µ
= µ0 and ∈ = 9 ∈0 which occupies the region y ≥ 0. The
field in Region-1 at x = 0 is H1 = 3uˆ x + 30uˆ y A/m, the
reflected magnetic field component is given by
Chapter 2  Uniform Plane Waves  |  9.29

1 Solution: (d)
8
(a) cos(3 × 10 t + ²y)aˆ x A/m
10p E2 η H2
For free space, P = or, 0
2η0 2
1 5 3 5  p
(b) cos(3 × 108 t + ²y)aˆ x A/m H= cos( wt − b z ) xˆ + sin  wt − pz +  yˆ
20p h0 h0 2

1 H = Hx 2 + Hy 2
(c) − cos(3 × 108 t + b y)aˆ x A/m
20p
1 5
(d) − cos(3 × 108 t + β y )ax A/m  [2010] = ( 3 )2 + 1
10π η0

Solution: (a)
 10
8 H=
E i = 24 cos(3 × 10 t − b y ) az ˆ v/m η0

azˆ × aˆ[ Hi ] = ayˆ ⇒ azˆ + axˆ = ayˆ η0 × 100

P= = 50 watt
1  1 2η02
Hi = Ei axˆ = cos(3 × 108 t − py )axˆ
h 5p Hence, the correct option is (d)
∈2 − ∈1 1 5. A Right Circularly Polarized (RCP) plane wave is inci-
H r η1 − η2 9− 1 1
= = = = dent at an angle of 60° to the normal, on an air-dielec-
H i η1 + η2 ∈2 + ∈1 9+ 1 2 tric interface. If the reflected wave is linearly polarized,

the relative dielectric constant ∈r2 is
1
Hr = Hi
2
1
Hr = cos(3 × 108 t + β y )ax
10 π
Hence, the correct option is (a)
13. A uniform plane wave in the free space is normally
incident on an infinitely thick dielectric slab (dielectric
constants εr = 9) The magnitude of the reflection coef-
ficient is
(a) 0 (b) 0.3
(c) 0.5 (d) 0.8 [2008]
Solution: (c)
(a) 2 (b) 3
εr1= 9 tr2= 9 (c) 2 (d) 3 [2007]
∈r1 − ∈r2 1− 3 Solution: (d)
Γ= =
∈r1 + ∈r2 1+ 3 Brewster’s angle of incidence is given as,

∈2
Γ = 0.5 tan θ B =
∈1

Hence, the correct option is (a)
 ∈2
1 4. The H field (in A/m) of a plane wave propagating in tan 60° = = 3
free space is given by 1
 5 3 5  π t2 = 3
H=x cos(ω − β ) + y sin  ω − β z +  Hence, the correct option is (d)
η0 η0  2

16. A medium of relative permittivity ∈r2 = 2 forms an
The time average power flow density in watts is interface with free-space. A point source of electromag-
η0 100 netic energy is located in the medium at a depth of 1
(a) 100 (b) η0 meter from the interface. Due to the total internal reflec-
tion, the transmitted beam has a circular cross-section
50
(c) 50h02 (d)  [2007] over the interface. The area of the beam cross-section at
η0 the interface is given by
9.30 | Electromagnetic Theory

(a) 2πm2 (b) π2m2  


t1 En1 = t 2 En2
(c) π/2 m2 (d)
πm2 [2006]
Solution: (d)  3  
En2 = × 4σ x = 3ax
∈ r2 sin θ = 1 4
  
E2 = En2 + Et2
1
sin θ =     
2 E2 = 3ax + 3ay + 5az

θ = 45°
Oθ = 1 m
Pθ = 1 m

Hence, the correct option is (c)


1 8. When a plane wave travelling in free space is incident
normally on a medium having ∈r = 4.0, the fraction of
power transmitted into the medium is given by
(a) 8/9 (b) 1/2
Area of circle at interface is, π(pθ)2 (c) 1/3 (d) 5/6 [2006]
A = π m2 Solution: (a)
Hence, the correct option is (d)
Pi = Pt + Pr
1 7. A medium is divided into regions I and II about x = 2
0 plane, as shown in the figure below. An electromag- Pr = Γ1 Pi
netic wave with electric field E1 = 4aˆ x + 3aˆ y + 5aˆ z is Pt = (1 − Γ 2 ) Pi

incident normally
 on the interface from region I. The
electric field E2 in region-ll at the interface is η2 − η1 ∈1 − ∈2
Γ= =
η2 + η1 ∈1 + ∈2

∈1 = 1,∈2 = 4
1
Γ=−
3
2
Pt 1 8
= 1−   =

Pi 3 9
Hence, the correct option is (a)
E2 = E1
(a)

4 aˆ x + 0.75aˆ y − 1.25aˆ z
(b) 19. If H =  k  ( ayˆ + jaxˆ )e j *kz − wt ) and
3aˆ x + 3aˆ y + 5aˆ z
(c)  wm 
−3ax + 3aˆ y + 5aˆ z    k 
(d) ˆ [2006]
H= ( )
 aˆ y + jaˆ x e
jkz − jwt
the time averaged
Solution: (c)  wm 
    
E1 = 4 ax + 3ay + 5az Poynting vector is
 k 
  (a) null vector (b) 
Et1 = Et2  â z
 wm 
     k 
Et1 = 3ay + 5az = Et2  2k 
(c)
 
 â z (d)  az
 wm   2ωµ 
Dn1 = Dn2
 [2004]
Chapter 2  Uniform Plane Waves  |  9.31

Solution: (a)

E = ( axˆ + jayˆ )e j ( kz − wt )

  k  j *kz − wt )
H =  ( ayˆ + jaxˆ )e
 wm 
1  
Pavg = Re [ E × H ]
2
1   k  j ( kz − wt ) 
= Re ( ax + jay )e j ( kz − wt ) ×   ( ay − jax )e 

2   wµ   Hence, the correct option is (c)
1  21. A uniform plane wave travelling in air is incident on
 k  j ( kz − wt ) 
= Re ( ax + jay )e j ( kz − wt ) ×   ( ay − jax )e  the plane boundary between air and another dielectric
2 
 wµ   medium with εr = 4. The reflection coefficient for the
1 k  normal incidence, is
=   ( axˆ + ayˆ ) + j ( axˆ + axˆ ) + j ( ayˆ + ayˆ ) + ( ayˆ + axˆ ) (a) Zero (b) 0.5 ∠ 180°
2  wm 
(c) 0.333 ∠ 0° (d) 0.333∠180° [2003]
1 k  Solution: (d)
=   ( axˆ + ayˆ ) + j ( axˆ + axˆ ) + j ( ayˆ + ayˆ ) + ( ayˆ + axˆ ) 
2  wm  1 1

η − η1 ∈2 ∈1
1 k  Γ= 2 =
=   [azˆ + 0 + 0 − azˆ ] η2 + η1 1 1
2  wm  +
∈2 ∈1

Pavg = 0
Hence, the correct option is (a) ∈2 = 4, ∈1 = 1
20. Medium 1 has the electrical permittivity ε1= 1.5ε0 1
−1
farad/m and occupies the region to the left of x = 0 1
Γ= 2 = − = 0.333 < 180°
plane Medium 2 has the electrical permittivity ε2 = 1 3
+1
2.5ε0 farad/m and occupies the region to the right of 2
x = 0 plane. If E1 in medium 1 is E1 = (2ux - 3uy + 1uz) Hence, the correct option is (d)
volt/m, then E2 in medium 2 is
22. If the electric field intensity associated with a uniform
(a) (2.0ux-7.5uy + 2.5uz)volt/m plane electromagnetic wave travelling in a perfect
(b) (2-0ux -2.0uy+ 0.6uz)volt/m dielectric medium is given by E(z, t) = 10cos(2π107t −
(c) (1.2ux-3.0uy + 1.0uz)volt/m 0.1πz). volt/m, the velocity of the travelling wave is
(d) (1.2ux-2.0uy+ 0 6uz)volt/m [2003] (a) 3.00 × 108 m/sec (b) 2.00 × 108 m/sec
(c) 6.28 × 10 m/sec
7
(d) 2.00 × 107 m/sec
Solution: (c)  [2003]
Et1 = Et2 Solution: (b)

Et1 = −3uy + uz = Et2 E(z, t) = 10 cos (2π × 107t – 0.1πz) v/m
w = 2π × 107
∈1 En1 = ∈2 En2 β = 0.1π

1.5 ∈0 . 2ux = 2.5 ∈0 En2 w 2π × 107
V= =
β 0.1π
En2 = 1.2 ux
V = 2 × 108 m/s
So, E2 = 1.2ux – 3uy + uz Hence, the correct option is (b)
9.32 | Electromagnetic Theory

23. A plane wave is characterized by η1 sin θi = η2 sin θt



E = (0.5 xˆ + yeˆ jwt − jkz )e jwt − jkz . This wave is
1 sin 30°
(a) linearly polarized =
(b) circularly polarized ∈2 sin 45°

(c) elliptically polarized
d
(d) unpolarized [2002] ∈2 =
2
Solution: (c)
 ∈2 = 2
ˆ jp /2 )e( wt − kz )
E = 10.5 xˆ + ye

Here, E x = 0.5e j ( wt − kt ) xˆ
E y = ej ( wt − kz + p /2) yˆ
Ex and Ey are out of phase by 90° with unequal magni-
tude so, this wave is elliptically polarized
Hence, the correct option is (c)
24. Distilled water at 25°C is characterized byσ = 1.7 ×
10−4 mho/m and ∈ = 78 ∈0 at a frequency of 3 GHz. Its
loss tangent tan δis
(a) 1.3 × 10-5 (b) 1.3 × 10-3
(c) 1.7 × 10 /78 -4
(d) 1.7 × 10"4/(78∈0) Hence, the correct option is (c)
(∈=10−9/(36π)F/m) [2002] 2 7. Two coaxial cables 1 and 2 are filled with different die-
/ ∈r2∈r1 /respectively.
lectric constants ∈r1and ∈r2 The ratio of
Solution: (a)
the wavelengths in the two cables, (λ1/λ) is
σ
Loss tangent,
w∈ (a) ∈r1 / ∈r2 (b)
∈r2 / ∈r1
−4
1.7 × 10 × 36π
= ∈r1 / ∈r2 (d)
∈r2 / ∈r1 
9 9 (c) [2000]
2 × λ × 3 × 10 × 78 × 10
Solution: (b)
= 1.3 × 10-5
Hence, the correct option is (a) 1
Velocity, V ∝
2 5. A material has conductivity of 10-2 mho/m and a relative t
[V = fλ]
permittivity of 4. The frequency at which the conduc-
tion current in the medium is equal to the displacement So, λ1 = ∈2
current is λ2 ∈1
(a) 45 MHz (b) 90 MHz Hence, the correct option is (b)
(c) 450 MHz (d) 900 MHz 28. A plane wave propagating through a medium ∈r. = 8, µr
[2001] = 2, and σ = 0 has its electric field given by

Solution: (a) E = 0.5 sin(108t βz)V/m. The wave impedance, in ohms
Conduction current = |σE| is
(a) 377 (b) 198.5 ∠ 180°
displacement current = |jw∈E|
(c) 182.9 ∠ 14° (d) 188.5 [1999]
2πf∈E = σE
Solution: (d)
σ 9 × 109 × 2 × 10 −2 Er = B, μr = 2, σ = 0
f = =
2π × ∈0∈ r 4  8

E = 0.5 sin(10 t − β z )
f = 45 mHz
Hence, the correct option is (a) jw µ
η=
26. A uniform plane wave in air impinges at 45° angle on σ + jw ∈

a lossless dielectric material with dielectric constant ∈r.
w = 108
The transmitted wave propagates in a 30° direction with
respect to the normal. The value of ∈r .is µ r µ0 µr
η= = 120π
(a) 1.5 (b) 1.5 ∈ r t0 ∈r
(c) 2 (d) 2  [2000]
Solution: (c)
Chapter 2  Uniform Plane Waves  |  9.33

2  1 
η = 120π −9
8 ∈0 = 36π × 10 F/m 

 
η = 60π = 188.5Ω (a) a good conductor

(b) a good dielectric
Hence, the correct option is (d) (c) neither a good conductor, nor a good dielectric
2 9. A uniform plane wave in air is normally incident on an (d) a good magnetic material [1993]
infinitely thick slab If the refractive index of the glass Solution: (a)
slab is 1.5, then the percentage of the incident power σ
Loss tangent for given problem is,
that is reflected from the air-glass interface is wt
(a) 0% (b) 4% σ 106
=
(c) 20% (d) 100% [1996] wt 10
2π × 10 × 109 × × 10 −9
Solution: (b) 36π
η2 − η1 σ
Reflection coefficient, Γ1 = = 1.798 × 105
η2 + η1 wt

For given problem, η1 (air) = 1 which is very large value as compare to 1


η2(dielectric slab) = 1.5. So, the material at this frequency is a good conductor
Hence, the correct option is (a)
1.5 − 1 1
Γ = 1.5 + 1 = 5 32. A plane wave is incident normally on a perfect con- 
ductor as shown in figure. Here, E iX , H iy and P i
1
Γ = are electric field, magnetic field and Poynting vector,
5 respectively, for the incident wave. The reflected waver
Pr 2 1 should have
= Γ =
Pi 25

1
% Pr = × Pi × 100 = 4% of Pi
25
Hence, the correct option is (b)
3 0. Some unknown material has a conductivity of 106
mho/m and a permeability of 4π 10–7 H/m.The skin
depth for the material at 1 GHz is
(a) 15.9 µm (b) 20 9 µrn
(c) 25.9 µm (d) 30.9 µm [1996]
Solution: (a) (a) E xr = E xi (b) H yr = − H iy
r i
σ = 106 mho/m (c) P = − P (d) E xr = E xi  [1993]
H = 4π × 10−7 H/m Solution: (a) and (c)
f = 1 GHz For a perfect conductor, E is zero with in a perfect
conductor.
2 1
Skin depth, δ = = i r t
w µσ π f µσ Ex + Ex = Ex
E xi + E xr = E xt =0 For perfect conductor.
1
= i r
3.14 × 1× 10 × 4π × 10 −7 × 106
9 Ex = − Ex
δ = 15.9 μm and, H xr = H xi
Hence, the correct option is (a) So, Pr = −Pc
31. A material is described by the following electrical Hence, the correct option is (a) and (c)
parameters at a frequency of 10 GHz: σ = 106 mho/m,µ 33. The electric field component of a uniform plane elec-
= µ0 and ∈/∈0 = 10. The material at this frequency is tromagnetic wave propagating in the Y-direction in a
considered to be lossless medium will satisfy the equation
9.34 | Electromagnetic Theory

∂2E ∂2E (b) Et and Ht are out of phase


(a) 2x = µ ∈ 2 x (c) Ht leads E, by 90°
∂y ∂t (d) Et leads H, by 45° [1988]
2 2
∂ Ey ∂ Ey Solution: (d)
(b) 2 = µ ∈ 2
∂x ∂t η = intrinsic impedance

∂2 E ∂ 2 Ex = Et = jωµ 2
(c) x = µ E
∂y 2 ∂t 2 Ht σ + jω ∈
E 2 + EZ2 For a good conductor
(d) 2x = µ / ∈ [1991]
H x + H z2 σ>>w∈
Et jωµ ωµ jπ / 4
Solution: (d) = = e
Ht σ σ
Since E.M wave is propagating in y-direction than Ey
and Hy component must be zero. So, Et leads Ht by an angle of 45°.
Ey and Hy = 0 Hence, the correct option is (d)
E.M wave equation given as, 36. The skin depth of copper at a frequency of 3 GHz is
1 micron (10-6 meter). At 12 GHz, for a non-magnetic
∂ 2 Ex ∂ 2 Ex
= µ ∈ conductor whose conductivity is 1/9 times that of cop-
∂y 2 ∂t 2 per, the skin depth would be
Ex
= −η and E z = η (a) 9 × 4 microns (b) 9 × 4 microns
Hz Hx
(c) 4 / 9 microns (d) 1 / 9 × 4 microns
E E x + E z2
2
µ [1989]
= 2 =η =
H Hx + Hz 2 ∈ Solution: (b)
Hence, the correct option is (d) We know that
34. The incoming solar radiation at a place on the surface For a good conductor
of the earth is 1.2 KW/m2. The amplitude of the elec-
1
tric field corresponding to this incident power is nearly Skin depth = δ =
equal to π f µσ
1
(a) 80mV/m (b) 2.5 V/m δ∝
(c) 30 V/m fσ

(d) 950 V/m [1990]
δ2 f1σ 1
Solution: (d) =
δ1 f 2σ 2
E2
Power =
2η Given that,

δ1 = 1 micron
E = Electric field intensity
f1 = 3 GHz
η = Intrinsic impedance
f2 = 12 GHz
E = 2η p
σ2 1
=
η = 120π σ1 9

P = 1.2 kW/m2
δ2 3 9 9
E = 2 × 120π × 1.2 × 103 = × =
1 12 1 4
E = 950 v / m
9
δ2 = microns
Hence, the correct option is (d) 4
35. In a good conductor the phase relation between the tan- Hence, the correct option is (b)
gential components of electric field Et and the magnetic 3 7. For an electromagnetic wave incident from one medium
field Ht is as follows to a second medium, total reflection takes place when
(a) Et and Ht are in phase (a) The angle of incidence is equal to the Brewster
Chapter 2  Uniform Plane Waves  |  9.35

angle with E field perpendicular to the plane of in- Solution:


cidence  8a x + 6 a y + 5a z
(b) The angle of incidence is equal to the Brewster an- E = E0J (ωt + 3 x − 4 y ) V/m
gle with E field parallel to the plane of incidence 125
(c) The angle of incidence is equal to the critical angle f = 10 × 109 Hz
with the wave moving from the denser medium to a 2
rarer medium E1
Pavg1 =
(a) = 0.997 w/m 2
(d) The angle of incidence is equal to the critical angle 2η0
with the wave moving from a rarer medium to a
ε = ε ′ − jε ′′ ε ′ = 9ε 0
(b)
denser medium [1987]
′′ = = −3 ′
Solution: (c) ε 0.09ε 0 10 ε .
Total internal inflection takes place when incident ω 40
α = E ′′ = 6.29 × 10 −3 n/m
angle is greater than critical angel θc. 2 E ′
n  ∈r2 Skin depth = α = 1/α = 159 m
θ c = sin −1  2  = sin
−1

 n1  ∈r1
(c) E2 = E1e −α x
For total internal reflection x = 5f = 5/α
θi ≥ θc E2 = E1e −α 5 / α = E1e −5

 ξr 
θi ≥ sin −1  2  µ0 µ0 1
 ξ r1  η2 = 1
= = × 120π = 40π
  E 9 E0 3

For total internal reflection to take place, the wave E22 339.24 −10
should move from a denser medium to a rarer medium P2 = = e
2η2 80π
and the angle of incidence should be greater than or
equal to the critical angle. P2 = 3e−10 w/m2
Hence, the correct option is (c) (i) βx = −3, βy = +4, βz = 0
8aˆ x 6 aˆ y 5aˆ z
hˆ = + +
, which conclude that
Five-marks Questions 125 125 125
1. A medium has a breakdown strength of 16 kV/m rms. the given field vector represents a plane wave in
Its relative permeability is 1.0 and relative permittiv- direction of xn.
ity is 4.0. A plane electromagnetic wave is transmitted β x2 + β y2 + β z2 = β 2 (cos 2 A + cos 2 B + cos 2 C )
(ii)
through the medium. Calculate the maximum possible
power flor density and the associated magnetic field.  64 36  4 2
= β2  + = β
 [2001]  125 125  5

Solution: μr = 1, Er = 4. 4 2
given, E = Ebrms = 16 kv/m
β = ( −3) 2 + ( 4) 2 = 252 , ⇒ β = 5.59
5
µr 2
η= 120π = 60π , P = Ebrms γ = α + jβ = 0 + j5.59
Er η
  = 5.39 j
3 2
So, Pmax = (16 × 10 ) = 1.36 × 106 w/m 2
(iii) Phase velocity in the y-direction.
60π ω ω 2π × 1010
Vy = = = = 1.57 × 1010 m/sec
E β y β cos β 4
H brms = brms = 84.88 A/m
η 3. A plane wave with
2. The electric field vector of a wave is given as jw µ w µ jπ / 4 wπ wµ
   η= = e = +j
 j(ω t+3x-4y) 8a x +6a y +5a z σ σ 2σ 2σ is inci-
E=E0 e V/m dent normally on a thick plane conductor lying in the
125
X-Y plane. Its conductivity is 6 × 106 S/m and surface
Its frequency is 10 GHz
impedance is 5 × 10-4 ∠45°W. Determine the propa-
(i) Investigate if this wave is a plane wave,
gation • constant and the skin depth in the conductor.
(ii) Determine its propagation constant, and
 [1998]
(iii) Calculate the phase velocity in y-direction [1998]
9.36 | Electromagnetic Theory

Solution:  Given, E y = 10e j (ωt − β z ) for good conduc-


tor, σ >> ωE

jωµ jωµ
Zs = ≅ = Rs + jxs
v + jω E σ

Rs = 5 × 10 −4 ∠45°

ω /4 5 × 10 −4
Rs = xs = =
2σ 2
Propagation constant, (iii)

jωσ 5 × 10 −4
α= β = ( Rs )σ = × 6 × 106
2 2
⇒ α = β = 2|2|
γ = α + Jβ = 2|2| + j2|2|
1 1
Skin depth, d = = = 0.471 mm,
α 2|2| (iv)

4. Match the following descriptions with each of the dia-


grams given in figure. Fields are near the interface, but
on opposite sides of the boundary. Vectors are drawn to
scale.
(a) Medium 1 and medium 2 are dielectrics with e1 >
e1.
(b) Medium 1 and medium 2 are dielectrics with e., e1
< e1.
(c) Medium 2 is a perfect conductor
(d) Impossible (v)
(e) Medium 1 is a perfect conductor. [1993]
Solution: Fig.1 DN1 – DN2 = PS
So DN1 = PS. ⇒ EN1 = PS/E
Et1 = Et2 = 0
E1 = Et1 + EN1 = 0 + PS/E = PS/E
Fig.2 Dt2 > Dt1
As, Et1 = Et2
E2 > E1
(i)
Fig.3 e1 > E1
Fig.4 H1 = 0, J = H2 A/m
Fig 5. EN2 > EN1, Et1 ≠ Et2.
which is not a possible, boundary condition.
Q 2Q
V3 = 4π E + 4π E ( x − 50) (3)
x

at equilibrium P.E. = QV1 + 2QV2 + Q3V3 = 0


from equation (1) (2) and (3)
(ii)
Chapter 2  Uniform Plane Waves  |  9.37

 2Q Q3   Q Q3  (2Qωt) = −4/9 2Q
 tan 
A +  + 2 Q  + + Q
 3 ⇒ ωt = −11.98°+ =0
 4π E ( 50 ) 4 π E ( x )   4 π E ( 50 ) 4π E ( x − 50 )   4π E ( x ) 4π E ( x − 50 ) 
So, Ex = 3 cos (11.98) = 2.934
 2Q Q3   Q Q3   Q 2Q E  = 2 cos (−11.98 + 45) = 1.676
A +  + 2Q  +  + Q3  +  y= 0
 4π E (50) 4π E ( x )   4π E (50) 4π E ( x − 50)   4π E ( x ) 4π E ( x − 50) 
E = E x2 + E y2 = 3.378

 Q Q3   Q 2Q  (4) at, ωt = 78.02
Q +  + Q3  + =0
 4π E ( 50 ) 4π E ( x − 50 )   4π E ( x ) 4 π E ( x − 50 ) 
Ey = 0.6227 and Ey = −1.08
Solving equation (4), Q = −50Q3
E = E x2 + E y2 = 1.246
−1
or Q3 = 50 Q
So, Emax = 3.3378
so, x(x – 50) = (x + 25) ⇒ x2 – 50x – x + 25 = 0  (5)
Emin = 1.246
−51 ± ( −51) 2 − 4(1)( 25) Emax 3.378
Solving (5) x = Axial = ratio = = 2.71
2 Emin 1.246
= 30.5 of 0.5 cm
Ey
x = 0.5 cm as (0.5 << 50.5) (b) tan θ = at Emax
Ex
So, x = 50.5 cm.
tan θ = 1.676 ⇒ ϑ = 29.73°
5. A wave travelling in the + z -direction is the resultant 2.934
of two linearly polarized components, Ex = 3coswt, and
Ey = 2cos(wt + 45°). Determine 6. A uniform plane wave having parallel polarization
(a) the axial ratio, and is obliquely incident on an air-dielectric interface as
(b) the angle between the major axis of the polarization shown in figure. If the wave has an electric field E = 10
ellipse and the +x axis. [1994] V/m. Find
Solution:  Wave is travelling in +z direction. (i) the angle of incidence 9, for which there is no re-
flection of the wave, and
(ii) the surface charge density at the interface.

Ex = 3 cos ωt
Ey = 2 cos (ωt + 45°)

t varies from 0 to T = θB Brewster angle,
Solution: 
ω
OAmax = OP = semi major axis of ellipse. Er2
OAmin = OQ tan(θ b ) = =2
Er1

OA2 = E 2 = E x2 + E y2
θb = 6.3.4°
E 2 = (3 cos ωt ) 2 + ( 2 cos(ωt + 45°)) 2 (ii) Et = Ei

(OA)2 = E2 = 6.5 + 4.5 cos (2ωt) −2sin 2 ωt  (1) Dt Di E2 E0 E r
= =
⇒ Dt = Di = 4 Di
E2 E1 E1 E0
dE
OA is max, when, = 0, and Solving (1)
dt Ps = Pt – Di = 3Di = 3E0Ei = 0.26 nc/m2
9.38 | Electromagnetic Theory

7. A uniform plane wave is normally incident from air on 


8. A plane wave in free space with E = ( π )
an infinitely thick magnetic material with relative per-
meability 100 and relative permittivity 4 (Figure). The (10.0 xˆ + 11.8 yˆ ) exp[ j (4p × 108 t - kz )].

wave has an electric field of 1 V/meter (rms). Find the
where x̂ and y are unit vectors in the x and y-direc-
average Poynting vector inside the material. [1997]
tions, respectively, is incident normally on a semi-infi-
nite block of ice as shown in Figure. For ice, μ = μ0, σ
= 0 and e = 9e0(1 – j0.001).
(a)  Calculate the average power density associated
with the incident wave.
(b) Calculate the skin depth in ice.
(c) Estimate the average power density at a distance
5 times the skin depth in the ice block, measured
from the interface.

Solution:
µr1
η1 = (120π ) = 120π
Er1

µr2
η2 = (120π ) = 600π
Er2

Reflection coefficient. Solution: from given data,
 
η2 − η1 600π − 120π 2 E = 24e j (ωt − β z ) a y
Γ= = = ω = 4π × 108
η2 + η1 600π + 120π 3
  
E y = 11.8 π β = k = ω Hω
E2 1
Parg i = i =
η1 120π |E|2 = |Ex|2 + |Ey|2 = 751.6, η = η0 = 120π

2
( Pavg )i 2 3
= 1 − (Γ) 2 = 1 −   =
( P )i 3 9
avg
5 1  
( Pavg )t = × a2 = 1.47a2 mxy /m 2
9 120π
Chapter 3
Transmission Lines
j0)Ω at ω = 106 rad s–1. The values of the line constants
One-mark Questions L, C, R, G are respectively [2016]
(A) L = 200 µH/m, C = 0.1 µF/m, R = 50 Ω/m,
1. The voltage of an electromagnetic wave propagating in G = 0.02 S/m
a coaxial cable with uniform characteristic impedance (B) L = 250 µH/m, C = 0.1 µF/m, R = 100 Ω/m, G =
is V(  ) = e -g  + jwt volts. Where  is the distance 0.04 S/m
along with length of the cable in metres, γ  = (0.1 + (C) L = 200 µH/m, C = 0.2 µF/m, R = 100 Ω/m, G =
j40) m-1 is the complex propagation constant, and w 0.02 S/m
= 2p × 109 rad/s is the angular frequency. The abso- (D) L = 250 µH/m, C = 0.2 µF/m, R = 50 Ω/m,
lute value of the attenuation in the cable in dB/metre is G = 0.04 S/m
______________. [2017] Solution:  Propagation constant of a loose transmission
γ = 0.1 + j 40 −1 m −1
Solution:  line is (2 + j5) m–1.
Characteristic impedance is (50 + j0)Ω.
 γ = α + j β m-1
Angular frequency ω = 106 rad s–1.
As Z is real, line is a lossless line,
α 
→ attenuation constant (Np/m or dB/m)
β 
→ Phase constant (rad/m or degree/M) L
⇒ Z= = 50
C 
∴ α = 0.1 Np/m


β = ω LC 
1 Np = 8.686 dB
\ a = 0.8686 dB/m 5 = 106 × LC 
Hence, the correct answer is (0.85 to 0.88).
⇒ LC = 5 × 10 −6 
2. A two wire transmission line terminates in television
set. The VSWR measured on the line is 5.8. The per- L
centage of power that is reflected from the television set LC × = L = 250 μ H/m
C 
is __________. [2017]
Solution:  VSWR = 5.8 ⇒ LC = 5 × 10 −6 
VSWR − 1
Reflection Coefficient, K = 25 × 10 −12
VSWR + 1 ⇒ C=
25 × 10 −5 
5.8 − 1 4.8
= = = 0.7 =
0.1 µF/m.
  5.8 + 1 6.8
Percentage of power that is reflected Assuming the line to be a low loss distortion less line.
2
k × 100 = (0.7) 2 × 100 = 49% α = RG = 2 

Hence, the correct answer is (48.5 to 49.5). RG = 4
3. The propagation constant of a loose transmission line R G
= 
is (2 + j5) m–1 and its characteristic impedance is (50 + L C
9.40 | Electromagnetic Theory

R L (B) RΔz/2 LΔz/2 RΔz/2 LΔz/2


or = = 2500
G C 
R
⇒ RG × = R 2 = 10 4 CΔz
G 
R = 100 Ω 
RG = 4 Δz

G = 4/100 = 0.04 S/m. (C) LΔz/2 LΔz/2

Hence, the correct option is (B).

4. Light from free space is incident at an angle θi to the GΔz CΔz


normal of the facet of a step index large core opti-
cal fibre. The core and cladding refractive indices are
n1 = 1.5 and n2 = 1.4 respectively Δz

(D) RΔz LΔz


Free n2 (Cladding)
space
n1 (Core)
θi

Light GΔz CΔz

The maximum value of θI (in degrees) for which the


incident light will be guided in the core of the fibre is Δz
____. [2016]
σ
Solution:  Given that loss tangent tan δ = 0 =
Solution:  The refractive index of core n1= 1.5 σ ωε
Loss tangent tan δ = 0 =
Refractive index of core n2= 1.4 ωε
Using the law of optical refraction, we get ⇒ σ = 0
So, conductance G = 0.
sinθi = n12 − n22 Hence, the correct option is (B).


{( }
6. To maximize power transfer, a lossless transmission
1.5) − (1.4 )
2 2
θi = sin–1 line is to be matched to a resistive load impedance via
 a λ/4transformer as shown,
= sin {0.538} = 32.58°
–1
Lossless transmission line
λ/4 transformer
Hence, the correct Answer is (32.58°).
ZL = 50 Ω ZL = 100 Ω
5. A coaxial cable is made of two brass conductors. The
spacing between the conductors is filled with Teflon
(εr = 2.1, tan δ = 0). Which one of the following circuits
can represent the lumped element model of a small
piece of this cable having length ∆z? [2015] The characteristic impedance (in W)of the λ/4 trans-
(A) RΔz/2 LΔz/2
former is____. [2014]
RΔz/2 LΔz/2
Solution:  (70.72)
For λ/4 transformer,

GΔz CΔz z02


zin =
zL

Here, zin = 50 Ω and zL = 100 Ω
Δz
z0 = zin . z L

Chapter 3  Transmission Lines  |  9.41

1+ Γ
= 50 × 100 VSWR = =
1.1
1− Γ 0.9
z0 = 70.72 Ω
7. In the following figure, the transmitter Transmission VSWR = 1.22
sends a wideband modulated RF signal via a coaxial Hence, the correct option is (a).
cable to the receiver Rx The output impedance ZT of 9. A transmission line of characteristic impedance 50 W
Transmission, the characteristic impedance Z0, of is terminated by a 50 W load. When excited by a sinu-
the cable and the input impedance ZR of Rx. are all soidal voltage source at 10 GHz, the phase difference-
real. between two points spaced 2 mm apart on the line is
found to be π/4 radians. The phase velocity of the wave
along the line is
(a) 0.8 × 108 m/s (b) 1.2 ×108 m/s
(c) 1.6 × 10 m/s
8
(d) 3 ×108 m/s
 [2011]
Which one of the following statements is TRUE about Solution: (c)
the distortion of the received signal due to impedance π
β .l = [for l = 2 mm]
mismatch? 4
π
(a) The signal gets distorted if ZR ≠ Z0, irrespective of β=
the value of ZT. 4 × 2 ×10 −3
(b) The signal gets distorted if ZT ≠ Z0, irrespective of 1000π
the value of ZR. β= rad/m
8
(c) Signal distortion implies impedance mismatch at
both ends: ZT ≠ Z0 and ZR≠ Z0. w = 2π × 10 × 109 rad/sec
(d) Impedance mismatches do NOT result in signal w 2π × 10 × 109
distortion but reduce power transfer efficiency. Velocity, V = β = 1000π ×8
 [2014]    V = 1.6 × 108 m/s
Solution: (c) Hence, the correct option is (c).
For given problem, following statements (a) and 10. A transmission line has a characteristic impedance of
(b) cannot be true because transmission line can be 50 W and a resistance of 0 1 W/m. If the line is distor-
matched for zR ≠ z0 and zT ≠ z0 by using quarter wave- tionless, the attenuation constant (in Np/m) is
length transformer. (a) 500 (b) 5
Statement (d) also not true because signal distortion (c) 0.014 (d) 0.002 [2010]
occurs due to mismatching at both ends i.e. zT ≠ z0 and Solution: (d)
zR ≠ z0 For distortionless transmission line,
Therefore (c) is true about the distortion of the received L C
signal. =
R G
Hence, the correct option is (c).
L R
8. The return loss of a device is found to be 20 dB. The =
z0 =
C G
voltage standing wave ratio (VSWR) and magnitude of
reflection coefficient are respectively R
(a) 1.22 and 0.1 (b) 0.81 and 0.1 attenuation constant, ∝= RG = R .
Z0
(c) –1 22 and 0.1 (d) 2.44 and 0.2
[2013] R 0.1
∝= =
Solution: (a) Z0 50

Return Loss = −20 log10|Γ+ dB
∝= 0.002 Np/m
−20 log10|Γ| = 20 dB
log10|Γ| = −1 Hence, the correct option is (d).
Γ = 10 −1 = 0.1 1 1. The VSWR can have any value between
(a) 0 and 1 (b) – 1 and + 1
(c) 0 and ∞ (d) 1 and ∞  [2002]
9.42 | Electromagnetic Theory

Solution: (d) Solution: (b)


1+ Γ zsc = 25 Ω
VSWR =
1− Γ Zoc = 100 Ω

z0 = zsc . zoc
For, |Γ| = 0, VSWR = 1
For, |Γ| = 1, VSWR = ∞
  = 25 ×100
So, VSWR can have any value between (1 and ∞)
z = 50Ω
Hence, the correct option is (d). 0
1 2. In an impedance Smith chart, a clockwise movement Hence, the correct option is (b).
along a constant resistance circle gives rise to 15. All transmission line section in Figure, have a char-
(a) a decrease in the value of reactance acteristic impedance R0 + j0. The input impedance Zin
(b) an increase in the value of reactance equals
(c) no change in the reactance value
(d) no change in the impedance value [2002]
Solution: (b)
In clockwise direction along constant resistance circle
gives rise to an increase in the value of reactance.

2
(a) R0 (b)
R0
3
3
(c) R0 (d) 2R0 [1998]
2
Solution: (b)
 λ
R1 l =  = 2 R0
 2
Hence, the correct option is (b).
1 3. A transmission line is distortion less if 2
 λ z
1 R2 l =  = 0
(a) RL = (b) RL = GC  4  zL

GC
(c) LG = RC (d) RG -= LC [2001] R02
=
Solution: (c) R0 / 2

Condition for distortion less is, R2 = zR0
R G 2 R0 × 2 R0
= z L = ( R1 || R2 ) = = R0
L C 4 R0

Rise time constant = Fall time constant
 z + jz0 
RC = LG zin = [l = λ /8] = z0  L 
Hence, the correct option is (c).
 z0 + jz L 
14. The magnitudes of the open-circuit and short-circuit [ R0 + jR0 ]
zin = R0
input impedances of a transmission line are 100 W and ( R0 + jR0 )

25 Wrespectively. The characteristic impedance of the
zin = R0
line is,
(a) 25 W (b) 50 W
(c) 75W (d) 100W [2000]
Chapter 3  Transmission Lines  |  9.43

Solution: (b)
L V= 1
z0 = ,
C LC
L = z0 C

So,
1
V=
z0 C

Hence, the correct option is (b). Hence, the correct option is (b).
1 9. A load impedance, (200 + j0) W is to be matched to a
16. A transmission line of 50 W characteristic impedance 50 W lossless transmission line by using a quarter wave
is terminated with a 100 W resistance The minimum linetransformer (Q W T). The characteristic impedance
impedance measured on the line is equal to of the QWT required is......... [1994]
(a) 0W (b) 25W
Solution: (100)
(c) 50 W (d) 100W [1997]
zL = (200 + j0)
Solution: (b)
zin = 50 W
z0 = 50 Ω
For quarter wave transformer,
zL = 100 Ω
z02
z 2 50 × 50 zin =
zin (min) = 0 = zL

zL 100
2
zin(min) = 25 Ω z0 = 200 × 50
Hence, the correct option is (b). z0 = 100Ω

17. A lossless transmission line having 50 W characteristic
impedance and length λ/4 is short circuited at one end Two-marks Questions
and connected to an ideal voltage source of 1 V at the
other end. The current drawn from the voltage sources 1. There are two photolithography systems : one with
is light source of wavelength l1 = 156nm (system 1) and
(a) 0 (b) 0.02 A another with light source of wavelength l2 = 325nm
(c) ∞ (d) none of the these (system 2). Both photolithography systems are other-
wise identical. If the minimum feature sizes that can be
[1996]
realized using System 1 and System 2 are Lmin1 and Lmin2
Solution: (a) respectively, the ratio Lmin1/Lmin2 (correct to two decimal
z02 places) is ______. [2018]
For Quarter wave transformer, zin = Solution:  We know that
zL
zL = 0 Ω [short circuited] L∝ λ
zin = ∞ = open circuit
Therefore
V V Lmin1 λ1
I s = s = s = 0 Amp =
zin ∞ Lmin 2 λ2

Hence, the correct option is (a). 156 nm
= 0.48
18. The capacitance per unit length and the characteristic 325 nm
impedance of a lossless transmission line are C and Z0 Hence, the correct answer is 0.47 to 0.51.
respectively. The velocity of a travelling wave on the 2. A lossy transmission line has resistance per unit length
transmission line is R = 0.05 W/m. The line is distortionless and has char-
(a) Z0C (b) 1/(Z0C) acteristic impedance of 50 W. The attenuation constant
(c) Z0/C (d) C/Z0 [1996] (in Np/m, correct to three decimal places) of the line is
______. [2018]
9.44 | Electromagnetic Theory

Solution:  resistance per unit length R = 0.05 Ω/m 4. A lossless micro strip transmission line consists of a
R G trace of width w. It is drawn over a practically infinite
Distortionless line → = ground plane and is separated by a dielectric slab of
L C
thickness t and relative permittivity εr > 1. The induct-
characteristic impedance Z0 = 50 Ω
ance per unit length and the characteristic impedance
We know that of this line are L and z0, respectively. [2016]
Z0 = L = 50 w ε = ε0
C
L ε = ε0 εr ; εr > 1 t
⇒ = 2500
C
L R
= = 2500 Which one of the following inequalities is always
C G satisfied?
0.05
⇒ G= = 2 × 10–5 /m Lt Lt
2500 Z0 >
(A) Z0 <
(B)
 ε0εr w ε0εr w
Alternation constant can be calculated as
LW LW
α = RG Z0 >
(C) Z0 <
(D)
ε0εr t ε0εr t
= 0.05 × 2 × 10 −5 = 0.001 Np/m Solution:  From the given fig, where relative permittiv-
= 1 mNp/m ity are varying, the expression for characteristics im-
Hence, the correct answer is 0.001. pedance can be better inferred from option (B).
3. An optical fiber is kept along the ẑ direction. The Hence, the correct option is (B).
refractive indices for the electric fields along x̂ and ŷ 5. A microwave circuit consisting of lossless transmission
directions in the fiber are nx = 1.5000 and ny = 1.5001, lines T1 and T2 is shown in the figure. The plot shows
respectively (nx ≠ ny due to the imperfection in the fiber the magnitude of the input reflection coefficient TL as a
cross-section). The free space wavelength of a light function of frequency f. The phase velocity of the sig-
wave propagating in the fiber is 1.5 μm. If the light- nal in the transmission lines is 2 × 108 m/s. [2016]
wave is circularly polarized at the input of the fiber, the
T1 Length = 1 m T2 Length = L
minimum propagation distance after which it becomes Input
Z0 = 50 Ω Z0 = 50 Ω
linerly polarized, in centimeters, is ______________. Open
 [2017] Γ 50 Ω
Solution:  The phase difference between the electric
π
field x and y components is ⋅(circular polarization).
2
Γ (Reflection coefficient)

The wave polarization changes to linear, when the wave 1


travels a minimum distance such that the phase differ-
0.8
π
ence between the Ex and Ey gets chaged from ⋅to p. 0.6
2
π 0.4
Z min k x − Z min K y =
2 0.2
 ω ω  π
Z min  − = 0 0.5 1 1.5 2 2.5 f (in GHz)
 µ px µ py  2
The length L (inn meters) of T2 is _____.
∴ ω = 2π f
Solution:  We know that
c
VP = Zoc = jZ0 cot(βL)
n
n = εr Effective Load impedance for 1 m long line is
nx = 1.5 µm ZL = 50 || [–jZ0 cotβL]
my = 1.5001 µm for γ = 0, ZL = 50
Z min = 0.375 cm
i.e. –jZ0 cot βL = ∞
Hence, the correct answer is (0.36 to 0.38).
Chapter 3  Transmission Lines  |  9.45

βL = π The transient response of the circuit at the input of the


line (z = 0) is also drawn in the figure. The value of RL

ω LC × L = π  (in Ω) is ______. [2015]
1
2π × 109 × × L =π
2 ×108 
Rs = 150 Ω R0 = 50 Ω RL
1 εr,eff = 2.25
⇒ L = = 0.1 m Vs = 400 V
10 
Hence, the correct Answer is (0.1). 200 m
6. Consider the 3 m long lossless air-filled transmission z=0 z=L
line shown in the figure. It has a characteristic imped-
ance of 120 π Ω, is terminated by a short circuit, and V(0, t)
is excited with a frequency of 37.5 MHz. What is the
nature of the input impedance (Zin)?  [2015]
100 V

ZL = 0 62.5 V

Zin 3m 2.0 t(μ s)

(A) Open (B) Short Solution: ⎡S ⎡R


(C) Inductive (D) Capacitive
ℓ=3m
Solution:  0
100
η = 120 π Ω 
⎡R.100 1
f = 37.5 MHz
2 ⎡R. ⎡S.100
ZL = 0
62.5
3
Zin = Zo ⎡ Z L + jZo tan β l ⎤ Ω
⎢ ⎥
⎣ Zo + jZ L tan β l ⎦  Z=0 Z=1

2π Given V(t = 2 μ s , Z = 0) = 62.5


β=
λ  62.5 = V(t = 0, z = 0) + V(t = 1, z = 0)
C 3 × 108 + V (t = 2, z = 0)
λ= = =8 m
f 37.5 × 106
 62.5 = 100 + ୮R . 100 + ୮R . ୮S. 100

βl = × 3 = 3π RL − 50
8 4 ୮R =
RL + 50
Zin = –jZo tan β ℓ 
and
3π 150 − 50 1
=
–jZo tan ୮S = = .
4  150 + 50 2
=
–jZo 0.625 = 1 + ୮R + 0.5 ୮R
So Zin represents capacitive. 1.5 ୮R = 0.625 – 1
Hence, the correct option is (D). −1
7. A 200 m long transmission line having parameters ୮R = .
4
shown in the figure is terminated into a load RL. The
∴ RL = 30 Ω .
line is connected to a 400 V source having source
resistance RS through a switch, which is closed at t = 0. Hence, the correct Answer is (29 to 31).
9.46 | Electromagnetic Theory

8. A coaxial capacitor of inner radius 1 mm and outer Solution: (b)


radius 5 mm has a capacitance per unit length of For parallel plate,
172 pF/m. If the ratio of outer radius to inner radius
πl∈
is doubled, the capacitance per unit length (in pF/m) is C=
_____. [2015] ln( D /d )

µ D
2πε L= l .ln  
Co-axial capacitor C =
Solution:  F m λ
( )
ln b
a
d
1 µ D D


( a) 
ln b
η=
L
=
ln
π  d 
ln  
=  
d µ
C π∈D π ∈
172 ln (10 ) ln  d 
⇒ =
C2 ln (5)

1 1
⇒ C2 = 120.22 pF/m V= =
LC µt
Hence, the correct Answer is (120 to 120.4).
Now, reduction in D results no change in V but having
λ of z.
9. The input impedance of a section of a lossless
8
transmission line of characteristic impedance 50W is
found to be real when the other end is terminated by a
load ZL = (R + jX) W. If X is 30W the value of R (in W)
is______. [2014]
Solution: (40)
Hence, the correct option is (b).
For l = λ/8
1 1. In the transmission line shown, the impedance Zin (in
 z + jz0  ohms) between node A and the ground is
zin = z0  1 
 z0 + jz L 
z0 = 50 Ω, zL = R + jx = R + j30
zin = real

 R + j 30 + j 50   R + j80 
zin = 50   = 50  
 50 + jR − 30   20 + jR 

Solution: (33.33)
[ R + j80][20 − jR]
zin = 50 zin = (z1 || z2)
( 20 2 + R 2 )
z2 = 100 Ω
50[20 R + 80 R] j 50[1600 − R 2 ] For, l = λ/2, βl = π
= +
( 20 2 + R 2 ) ( 20 2 + R 2 ) zin1 = zL

∴z1 = 50 l
Since, zin is real so, its imaginary part must be zero.
100 × 50
1600 – R2 = 0 zin = = 33.3Ω
100 + 50
R = 40Ω

10. For a parallel plate transmission line, let v be the speed


of propagation and Z be the characteristic impedance.
Neglecting fringe effects, a reduction of the spacing
between the plates by a factor of two results in 12. A transmission line with a characteristic impedance
(a) halving of v and no change in Z of 100 W is used to match a 50 W section to a 200
(b) no changes in v and halving of Z Wsection. If the matching is to be done both at 429
(c) no change in both v and z MHz and 1 GHz, the length of the transmission line
(d) halving of both v and z [2014] can be approximately
Chapter 3  Transmission Lines  |  9.47

(a) 82.5 cm (b) 1.05 m z1 = jz0 = j 30 Ω


(c) 1.58 m (d) 1.75 m [2012] For l = λ/y, zL = 30 Ω
Here, zi = 50 Ω, zL = 200 Ω
z02 30 × 30 × 2
z0 = 50 × 200 = 100Ω zin = = = 60Ω
zL 30

For making, quarter wavelength transformer
c
For given, frequency, λ = and, l will be, λ/4
f
13. A transmission line of characteristic impedance 50 W
is terminated in a load impedance ZL. The VSWR of
the line is measured as 5 and the first of the voltage
maxima in the line is observed at a distance of λ/4 from
the load. The value of ZL is
(a) 10 W
(b) 250 W
(c) (19.23 + j46.15)W
(d) (19.23 – j46.15)W [2011]
Solution: (a) Z1L = Z1 + Z2
The distance between maxima and minima is λ/4.   = 60 + j30
Thus minima at load and reflection coefficient are z1L − z0 60 + j 30 − 60
negative. Γ= =
z1L + z0 60 + j 30 + 60
1+ Γ 1+ Γ
VSWR = =5= j 30 j
1− Γ 1− Γ Γ= =
120 + j 30 4 + j

2
=− 1
3 Γ =
17
zl − zo
=
zl + zo 1+ Γ 17 + 1
VSWR = =
2 zl − 50 1− Γ 17 − 1
− =
3 zl + 50
VSWR = 1.64
zl = 10
Hence, the correct option is (b).
Hence, the correct option is (a).
15. A transmission line terminates in two branches each of
14. In the circuit shown, all the transmission line sections length λ/4, as shown. The branches are terminated by
are lossless. The Voltage Standing Wave Ratio (VSWR) 50 Ω loads. The lines are lossless and have the charac-
on the 60 W line is teristic impedances shown. Determine the impedance
Z0 as seen by the source.

(a) 1.00 (b) 1.64


(a) 200W (b) 100W
(c) 2.50 (d) 3.00 [2010]
(c) 50 W (d) 25 W [2009]
Solution: (b)
Solution: (d)
For l = λ/8, zL = 0 Ω
R = 200 Ω
[ z L + j z0 ]
zin = z0 zi = 25 Ω
[ z0 + jz L ]

9.48 | Electromagnetic Theory

2-port scattering parameter matrix (S-matrix) of the


shunt element is

Hence, the correct option is (d).


 1 1 
16. One end of a loss-less transmission line having the − 2  (b)
0 1 
characteristic impedance of 75Ω and length of 1 cm (a)  2   
is short-circuited. At 3 GHz, the input impedance at the  1 1 1 0 
 2 − 
other end of the transmission line is 2
(a) 0 (b) Resistive  1 2   1 3
(c) Capacitive (d) Inductive [2008] − 3 3   4 − 4
(c)   (d)  
Solution: (d) − 3 1 
 2 − 1
For short-circuit load, zL = 0  3 3   4 4 
zin = j z0 tan (βl)  [2007]
8
2π and λ = c = 3 × 10 = 10 cm Solution: (b)
β=
λ f 3 × 109 Since, zL = z0, so impedance are matched thus no reflec-

tion occurs at load and source.
2π π
β= = rad/cm S11 = S22 = 0
10 5
S = [0 1]
l = 1 cm [1 0]
π Hence, the correct option is (b).
β .l = = 36°
5 19. The parallel branches of a 2-wire transmission line are
Zin = J75 tan 36° terminated in 100 Ω and 200 Ω resistors as shown in
the figure. The characteristic impedance of the line is
zin = j 54.49Ω Z0 = 50 Ω and each section has a length of λ/4. The
Input impedance is inductive voltage reflection coefficient Γat the input is
Hence, the correct option is (d).
1 7. In the design of a single mode step index optical fiber
close to upper cut-off, the single-mode operation is
NOT preserved if .
(a) radius as well as operating wavelength are halved
(b) radius as well as operating wavelength are doubled
(c) radius is halved and operating wavelength is dou-
bled
(d)  radius is doubled and operating wavelength is
halved [2008]
Solution: (d) 7 -5
-i (b)
(a)
2π a 5 7
Cut-off frequency, Fc = sin θ
λ 5 5
a = radius of core, λ = wave length (c) j (d) [2007]
7 7
For single mode step index, Fc must lie between 0 <Fc<
2.405. Solution: (d)
Out of four options, (d) radius doubled and λ halved λ
For, l =
lead to Fc> 2.405. 4
Hence, the correct option is (d).
18. A load of 50 Wis connected in shunt in a 2-wire trans- z02
zin =
mission line of Z0 = 50Ω as shown in the figure. The zL
Chapter 3  Transmission Lines  |  9.49

50 × 50 3 × (zL – 50) = zL + 50
R1 = = 25Ω
100 3zL – 150 = zL + 50

zL = 100 Ω
50 × 50 25
R2 = = Ω
200 2
25
25 ×
R3 = ( R1 || R2 ) = 2 = 25 Ω
25 3
25 ×
2
50 × 50
zin = = 300Ω
25/ 3
zin − z0 300 − 50 Hence, the correct option is (d).
Γ= =
zin + z0 300 + 50 2 1. The steady-state current through the load resistance is
(a) 1.2 Amps (b) 0.3 Amps
250 5 (c) 0.6 Amps (d) 0.4 Amps [2006]
Γ= =
350 7 Solution: (b)
VL 30
=
IL =
Z L 100

IL = 0.3 Amps
Hence, the correct option is (b).
22. Characteristic impedance of a transmission line is
50W. Input impedance of the open-circuited line is
Zoc. = 100 + j 150 W. When the transmission line is
short-circuited, then value of the input impedance will
be
(a) 50 W (b) 100 + j150Q.W
Hence, the correct option is. (d)
(c) 7.69 + j11.54 W (d) 7.69 – j 11.54 W
Common data for Questions 20 & 21:  [2005]
A 30-Volts battery with zero source resistance is con- Solution: (d)
nected to a coaxial line of characteristic impedance of
z0 = 50 Ω
50 Ohms at t = 0 second and terminated in an unknown
resistive load. The line length is such that it takes 400 zoc = 100 + j 150 Ω
us for an electromagnetic wave to travel from source z0 = zoc . zsc
end to load end and vice-versa. At t = 400 µs, the volt-
age at the load end is found to be 40 Volts. z02 50 × 50
zsc = =
20. The load resistance is zoc 100 + j150

(a) 25 Ohms (b) 50 Ohms
(c) 75 Ohms (d) 100 Ohms 50 50( 2 − j 3)
zsc = =
 [2006] 2 + j3 13

Solution: (d)
zsc = 7.69 − j11.54 Ω
vt = vr + vi
vi = 30 v, vt = 40 v Common Data for Questions 23 and 24
vr = 10 v Voltage standing wave pattern in a lossless transmis-
V 10 1 sion line with characteristic impedance 50 ohms and a
Γ= r = = resistive load is shown in the figure.
Vi 30 3

z −z 1
Γ[at load ] = L 0 =
z L + z0 3

9.50 | Electromagnetic Theory

25. Many circles are drawn in a Smith chart used for trans-
mission line calculations. The circles shown in the fig-
ure represent

Hence, the correct option is (d).


23. The value of the load resistance is
(a) 50 W (b) 200 W (a) unit circles
(c) 12.5 W (d) 0W [2005] (b) constant resistance circles
(c) constant reactance circles
Solution: (c)
(d) constant reflection coefficient circles [2005]
Vmax 4
=
VSWR = =4 Solution: (b)
Vmin 1
The circles shown in the figure represent constant
1+ Γ resistance circles.
VSWR = =4
1− Γ
1 + Γ = 4 − 4Γ
3
Γ =
5 r

Since, Vmin at load then, zL < z0
z0 50
VSWR = = 4 ⇒ zL = = 12.5Ω
zL 4

Hence, the correct option is (b).


26. Consider a 300 W, quarter-wave long (at 1 GHz) trans-
mission line as shown in the figure It is connected to a
10V, 50 Wsource at one end and is left open circuited at
the other end. The magnitude of the voltage at the open
circuit end of the line is

Hence, the correct option is (c).


24. The reflection coefficient is given by (a) 10 V (b) 5V
(a) –0.6 (b) –1 (c) 80 V (d) 60/7 V [2004]
(c) 0.6 (d) 0 [2005] Solution: (c)
Solution: (a)
10V 1
(1 + Γ) z0 1 I= = Amp
= = 50Ω 5
(1 − Γ) z L 4
vL = z0I
4 + 4Γ = 1 – Γ 300 × 1
=
−3 5
Γ= = −0.6
5
vL = 60 v
Hence, the correct option is (a). Hence, the correct option is (c).
Chapter 3  Transmission Lines  |  9.51

27. Consider an impedance Z = R + jX marked with point 1 − ∈2 −2


P in an impedance Smith chart as shown in the figure. =
The movement from point P along a constant resist- 1 + ∈2 3

ance circle in the clockwise direction by an angle 45° is
[η2 < tη1]
equivalent to
∈2 = 25
120π 120π
η2 = = Ω
∈2 5

η2 = 24π Ω

Hence, the correct option is (d).
29. A lossless transmission line is terminated in a load
which reflects a part of the incident power. The meas-
ured VSWR is 2. The percentage of the power that is
reflected back is
(a) 57.73 (b) 33.33
(c) 0.11 (d) 11.11 [2004]
(a) adding an inductance in series with Z Solution: (d)
(b) adding a capacitance in series with Z
(c) adding an inductance in shunt across Z 1+ Γ
VSWR = =2
(d) adding a capacitance in shunt across Z [2004] 1− Γ

Solution: (a)
1+ Γ = 2 − 2 Γ
On movement in clockwise direction by an angle 45°
on constant resistance circle is equivalent to increase in 1
reactance which is addition of inductance in series with Γ =
3
z.
Pref = |Γ|2 Pinc
Z = R0
2
Z = R0 + jX [in clockwise direction] Γ Pinc × 100
%Pref =
Hence, the correct option is (a). Pinc

28. A plane electromagnetic wave propagating in free space
1
is incident normally on a large slab of loss-less, non- = × 100%
magnetic, dielectric material with ∈ > ∈0. Maxima and 9
minima are observed when the electric field is meas- = 11.11%
ured in front of the slab. The maximum electric field Hence, the correct option is (d).
is found to be 5 times the minimum field. The intrinsic
impedance of the medium should be 3 0. A short-circuited stub is shunt connected to a trans-
(a) 120 πW (b) 60 πW mission line as shown in the figure. If Z0=50 W, the
(c) 600 πW (d) 24 πW [2004] admittance Y seen at the junction of the stub and the
transmission line is
Solution: (d)
Emax = 5 Emin
Emax
=
VSWR = 5
Emin

1+ Γ 2
=5⇒ Γ =
1− Γ 3

η2 − η1 ∈1 − ∈2
Γ = =
η2 + η1 ∈1 + ∈2

∈1 = 1, ∈2= ? (a) (0.01 – j0 02)mho


(b) (0.02 – j0.01)mho
9.52 | Electromagnetic Theory

(c) (0.04 + j0 02)mho 1


(d) (0 02 + j0) mho [2003] Γ = = 0.5
2
Solution: (a)
2
z0 = 50 Ω Γ Pi
% Pr = × 100 = (0.5) 2 × 100
ys = admittance at junction of stub Pi

β.ls = λ/8 % Pr = 25%
2p l p
b .ls = . = Hence, the correct option is (b).
l 8 4
32. In a twin-wire transmission line in air, the adjacent
 π π 
 z L cos 4 + jz0 sin 4  voltage maxima are at 12.5 cm and 27.5 cm. The oper-
zins = z0   ating frequency is
 z0 cos π + jz L sin π  (a) 300 MHz (b) 1 GHz
 4 4  (c) 2GHz (d) 6.28 GHz [1999]
Here, zL = 0 [short circuit stub] Solution: (b)
π λ/2 = (27.5 – 12.5) cm
zins = z0 j tan
4 1
Γ = =15CM
zins = j z0 = j 50 Ω 2
1 λ = 30 cm = 0.3 m
ys = = − j 0.02 mho
zins
c 3 × 108 m/s
f = =
yd = admittance at junction of transmission line λ 0.3 m
lt = λ/2
f = 1 GHz
2p l
b .lt = . =p
l 2

zint = zL = 100 Ω
1
=
yd = 0.01 mho
zint

Hence, the correct option is (b).


3 3. In air, a lossless transmission line of length 50 cm with
L = 10uH/mC = 40pF/m is operated at 25 MHz. Its
electrical path length is
(a) 0.5 meters (b) λ meters
(c) π /2 radians (d) 180 degrees
Hence, the correct option is (a).
[1999]
3 1. A uniform plane electromagnetic wave incident nor-
mally on a plane surface of a dielectric material is Solution: (c)
reflected with a VSWR of 3. What is the percentage of 1 1
V= =
incident power that is reflected? LC 10 × 10 −6
× 40 × 10 − 12
(a) 10% (b) 25%
(c) 50% (d) 75% [2001] V = 0.5 × 108 m/s
Solution: (b) v 0.5 × 108
l= = = 2m
1+ Γ f 25 × 106
VSWR =
1− Γ

β= =π
1+|Γ1| = 3[1–|Γ1| λ
4|Γ| = 2
Chapter 3  Transmission Lines  |  9.53

π × 50 z L − z0 j 50 − 50 j − 1
βl = Γ= = =
Electrical path length is 100 z L + z0 j 50 + 50 j + 1

π |Γ| = 1
β l = radians
2
1 + Γ1
Hence, the correct option is (c). VSWR = =∞
1 − Γ1
34. If a pure resistance load, when connected to a lossless
75 ohm line, produces a VSWR of 3 on the line, then the Hence, the correct option is (c).
load impedance can only be 25 ohms. True/False (Give 36. A transmission line whose characteristic impedance is
Reason) [1994] a pure resistance
Solution: (FALSE) (a) must be a lossless line
For pure resistance load, say RL (b) must be a distortionless line
Lossless line, R0 = 75 Ω (c) may not be a lossless line
(d) may not be a distortionless line [1992]
R
VSWR = L Solution: (b) and (c)
R0
For a distortion less line
(If, RL = R0) RC = LG
R
or,VSWR = 0 (If, R0 > RL) α − RG
RL
For, VSWR = 3 β = ω LC
So, RL = 3R0 = 225Ω [RL > R0] L
Z=
o R=
o
or R = R0 = 25Ω [RL < R0] C
L
3 R=0
1+ Γ G=0
=3
1− Γ a = 0
Now,
1+|Γ|= 3-3|Γ|| β = ω LC
4|Γ| = 2
L
1 Z=
o R=
o
Γ = C
2
A lossless line is always a distortion less line but a dis-
1 1 tortion less line may or may not be lossless line.
Now, Γ = or -
2 2 Hence, the correct option is (b) and (c).
For, Γ = 1 , zL = 225 Ω 37. The input impedance of a short circuited lossless trans-
2 mission line quarter wave long is [1991]
−1 Solution: (c)
and for Γ = , zL = 25 Ω
2
l = λ/y [quarter wave transformer]
Therefore, statement the load impedance can only be
25 Ω is false. 2π λ π
β ⋅l = . =
35. Consider a transmission line of characteristic imped- λ y 2
ance 50 ohm. Let it be terminated at one end by (+j 50)
ohm. The VSWR produced by it in the transmission line [ z L cos π / 2 + jz0 sin π / 2] z02
zin = z0 =
will be [ z0 cos π / 2 + jz L sin π / 2] z L

(a) +1 (b) 0
zL = 0 [short-circuited]
(c) ∞ (d) + j [1993]
zin= ∞
Solution: (c)
Hence, the correct option is (c).
z0 = 50 ohm
38. A 50 ohm lossless transmission line has a pure reac-
ZL = j50 ohm
tance of (j 100) ohms as its load. The VSWR in the line
is
9.54 | Electromagnetic Theory

(a) 1/2 (Half) (b) 2 (Two)


(c) 4 (Four) (d) ∞ (Infinity) Five-marks Questions
 [1989]
Solution: (d) λ
1. A - section of a 600 W transmission line, short cir-
Z − Z0 2
Reflection coefficient = Γ = L
Z L + Z0 cuited at one end and open circuited at the other end, is
j100 − 50 shown in figure. A 100 V/ 75 W. generator is connected
=
j100 + 50 at the mid point of the section as shown in the figure.

Find the voltage at the open-circuited end of the line.
(100) 2 + (50)
Γ = =1
(100) 2 + (50)

1+ Γ 1+1 2
VSWR = = = =∞
1− Γ 1−1 0

Hence, the correct option is (d).
3 9. A Two wire transmission line of characteristic
impedance Z0 is connected to a load of impedance ZL(ZL
≠ Z0) Impedance matching cannot be achieved with
[1997]
(a) a quarter-wavelength transformer
(b) a half-wavelength transformer Solution: 
The current through generator,
(c) an open-circuited parallel stub 100 4 2π λ
(d) a short-circuited parallel stub [1988] =
tg = amp, β = ;l=
75 3 λ 4
Solution: (b)
2π λ π
For z0 ≠ zL βl = × =
λ 4 2
a half-wavelength transform (λ/2) can not be used for
impedance matching because for, l = λ/2 V2 at a distance l is given by,
2p l Nl = Vg cos βl + jIg R0 sin βl
b .l = . =p   = jIg R0 = j800 = 800 ∠900
l 2
2. The three regions shown in figure all lossless and non-
( z L cos π + z0 sin π )
zin = z0 magnetic. Find
( z0 cos π + z L sin π ) (a) Wave impedance in medium 2 and 3.

(b) d such that medium 2 acts as a quarter wave (λ/4)
z =z
in L transformer.
Hence, the correct option is (b). (c)  Reflection coefficient (Γ) and voltage standing
wave ratio (VSWR) at the interface of the mediums
4 0. A transmission line of pure resistive characteristic 1 and 2, when d = λ/4
impedance is terminated with an unknown load. The
measured value of VSWR on the line is equal to 2 and a
voltage minimum point is found to be at the load. The
load impedance is then
(a) Complex
(b) Purely capacitive
(c) Purely resistive
(d) Purely inductive [1987]
Solution: (c)
Given, voltage minimum (Vmin) point is atload. [2000]
If Vmin or Vmax occurs at the load for a lossless transmis- For a lossless line, α = 0, and μ = μ0
Solution: 

sion line then load impedance ZL is purely resistive. = 4π × 10−7
Hence, the correct option is (c). µ0 1 120π
(a) η2 = × = = 60π Ω
E0 Er 4
2
Chapter 3  Transmission Lines  |  9.55

µ0 1 120π Z02 η22 (60π ) 2


η3 = × = = 40πΩ Zin = = = = 90πΩ
E0 Er 3 9 Z L η3 ( 40π )


Z L − Z0
1 C    Γ = , ZL = Zin = 90π
(b) V2 = = = Z1 + Z0
m 2 E2 Er2

3 × 108 µ0 1
= 1.5 × 108 m/sec Z0 = η1 = × = 120π
4 E0 Er 1
  
V
V2 = fλ, λ = 2 = 0.15 m. 90π − 120π
f Γ= = −1/ 7
λ     90π + 120π
d = = 3.75 cm
4 1+ | Γ | 4
  and VSWR = =
(c) z 0 = η 2 , z1 = η3 1− | Γ | 3
Chapter 4
Waveguide
ZL = 0
One-mark Questions Γ2 = −1
1. A two port network has scattering parameters given by V1− S12 S21
So, = S11 −
s s  V1+ 1 + S22
[S ] =  11 12  . If theport-2 of the two-port is short
s s
 21 22 
V1− S11 + S11S22 − S12 S21
circuited, thes11, parameter for the resultant one-port =
network is V1+ 1 + S22

s11 − s11s22 + s12 s21 Hence, the correct option is (b)
(a)
1 + s22 2. Which one of the following field patterns represents a
s11 + s11s22 − s12 s21 TEM wave travelling in the positive x direction?
(b)
1 + s22 (a) E = +8 yˆ , H = −4 zˆ
s11 + s11s22 + s12 s21 E = −2 yˆ , H = −3 zˆ
(b)
(c)
1 − s22 E = +2 zˆ, H = +2 yˆ
(c)
s11 − s11s22 + s12 s21 E = −3 yˆ , H = +4 zˆ 
(d) [2014]
(d)  [2014]
1 − s22
Solution:(b)
Solution: (b)
For, Ey + Hz = TEMx (+ve)
V1− = S11V1+ + S12V2+ E−z × Hy = TEMx (+ve)

V2− = S21V + + S22V2+ E−y × H−z = TEMx (+ve × direction)

Hence, the correct option is (b)
ΓL = at load on port 2
3. Consider an air filled rectangular waveguide with a
V2+ cross-section of 5 cm × 3 cm. For this waveguide, the
Γ2 =
V2− cut-off frequency (in MHz) of TE21 mode is________.

 [2014]
− + −
V1 = S11V1 + S12 Γ 2V2 Solution:  (7810)
− + − a × b = 5 cm × 3 cm
V2 = S21V1 + S22 Γ 2V2
2 2
V2− (1 − S22 Γ 2 ) = S21V1+ V 2 1
fc = + × 100
2  5   3 

Γ 2 S21V1+
V1− = S11V1+ + S12 For TE21,
1 − S22 Γ 2
V = 3 × 108 m/s
V1− S12 S21Γ 2 2 2
= S11 + 3 ×108  2  1
V1+ 1 − S22 Γ 2 fc =  5  +  3  × 100
2    

z L − z0
Γ2 = fc = 7810 MHz
z L + z0

Chapter 4  Waveguide  |  9.57

4. The modes in a rectangular waveguide are denoted by Solution: (c)


TEmn/TMmn where m and n are the eigen numbers along c
the larger and smaller dimensions of the waveguide Vp =
2
respectively. Which one of the following statements is  f 
1−  c 
TRUE?  f 
(a) The TM10 mode of the waveguide does not exist
(b) The TE10 mode of the waveguide does not exist Vp > C
(c) The TM10 and the TE10 modes both exist and have Hence, the correct option is (c)
the same cut-off frequencies 8. The phase velocity of waves propagating in a hollow
(d) The TM10 and the TM01 modes both exist and have metal waveguide is
the same cut-off frequencies [2011] (a) greater than the velocity of light in free space.
Solution: (a) (b) less than the velocity of light in free space
For TM mode, TM00, TM01, TM10 does not exist (c) equal to the velocity of light in free space
For TE mode (d) equal to the group velocity [2001]
TE00 does not exist but TE01 and TE10 exists Solution: (a)
Hence, the correct option is (a ) c
Vp =
2
5. Which of the following statements is true regarding the  f 
fundamental mode of the metallic waveguides shown? 1−  c 
 t 
For wave propagation, f > Fc
VP > C
(a) Only P has no cutoff-frequency
Hence, the correct option is (a)
(b) Only Q has no cutoff-frequency
(c) Only R has no cutoff-frequency 9. The dominant mode in a rectangular waveguide is TE10,
(d) All three have cutoff-frequencies [2009] because this mode has
(a) no attenuation
Solution: (a)
(b) no cut-off
For the three given metallic waveguides P, Q, R only P, (c) no magnetic field component
i.e., coaxial cable support TEM mode which means P (d) the highest cut-off wavelength [2001]
has no cut off frequency while d and R both have cut off
Solution: (d)
frequency.
For TE10
Hence, the correct option is (a)
λc = 2a
6. The phase velocity of an electromagnetic wave propa-
gating in a hollow metallic rectangular waveguide in which is highest cut-off wavelength.
the TE10 mode is Hence, the correct option is (d)
(a) equal to its group velocity 10. A TEM wave is incident normally upon a perfect con-
(b) less than the velocity of light in free space ductor. The E and H fields at the boundary will be,
(c) equal to the velocity of light in free space respectively,
(d) greater than the velocity of light in free space (a) minimum and minimum
[2004] (b) maximum and maximum
Solution: (d) (c) minimum and maximum
(d) maximum and minimum [2000]
Inside the waveguide phase velocity is greater and
group velocity is smaller than c Solution: (a)
−α z
Hence, the correct option is (d) Electric field equation, E = E0 e cos( wt − β z )ax
7. The phase velocity for the TE10-mode in an air-filled For good conductor, γ = α + j β
rectangular waveguide is Attenuation constant, α = π f µσ
(a) less than c For good conductor, σ >> 1
(b) equal to c
Which lead exponential damping of the wave (e−αz
(c) greater than c
factor).
(d) none of the above
Therefore, E and H field at the boundary of perfect con-
Note: (c is the velocity of plane waves in free space)
ductor will be minimum
 [2002]
Hence, the correct option is (a)
9.58 | Electromagnetic Theory

11. Assuming perfect conductors of a transmission line, 1.5 ×108


pure TEM propagation is NOT possible in λc = = 8 cm
(a) coaxial cable 1.872 ×109
(b) air-filled cylindrical waveguide ∴ cut off wavelength, λc = 8 cm
(c) parallel twin-wire line in air
(d) semi-infinite parallel plate waveguide 1999]
Solution: (b)
Two-marks Questions
Pure TEM propagation is not possible in waveguide 1. A rectangular waveguide of width w and height h has
like rectangular or cylindrical waveguide since, for a cut-off frequencies for TE10 and TE11 modes in the ratio
TEM wave, 1 : 2. The aspect ratio w/h, rounded off to two decimal
places, is _____. [2019]
Ez = 0 and Hz = 0
Solution:
This makes Ex, Ey, Hx, Hy vanish and hence a TEM wave
cannot exist inside a waveguide.  m  n C
2 2

Hence, the correct option is (b) fC =   +   ⋅


 a  b 2

12. A rectangular air filled waveguide has a cross section
of 4 cm × 10 cm. The minimum frequency which can fCTE10 1
=
propagate in the waveguide is fCTE 2
(a) 1.5 GHz (b) 2 0 GHz C 1
(c) 2.5 GHz (d) 3.0 GHz [1997] ⋅
2 w 1
=
Solution:  (none of the above) 2 2 2
C  1  1
b × a = 4 cm × 10 cm   + 
2  w   h
c 3 ×108 1
f c min = =
2a 2 × 10 × 10 −2 w 1
=
2 2 2
  fc min = 1.5 GHz  1  1
 w  +  h 
The minimum frequency which can propagate will be f
≥ fc min 2 2 2
 1  1  1
i.e. 1.5 GHz  w  +  h  = 4  w 
20 20 2 2
13. The interior of a cm × cm rectangular wave-  1  1
3 4  h  = 3  w 
guide is completely filled with a dielectric of ∈r =
w
4.Waves of free space wave lengths shorter than .....can = 3 = 1.732
h
be propagated in the TE11 mode.
[1994] 2. The dispersion equation of a waveguide. Which
relates the wave number k to the frequency w is k(w) =
Solution: (8)
⎛ 1⎞ 2 2
m = 1 and n = 1 ⎜⎝ ⎟⎠ ω − ω 0
c
v
λc = Where the speed of light c = 3 × 108 m/s, and w0 is a
fc constant. If the group velocity is 2 × 108 m/s, then the

phase velocity is [2019]
c 3 ×108 (A) 2 × 108 m/s (B) 3 × 108 m/s
V= = = 1.5 ×108 m/s
∈r 4 (C) 1.5 × 108 m/s (D) 4.5 × 108 m/s

2 2 Solution:
V m n
fc = +   1
2  a  b K ( w ) =   w 2 − w02
C
20 20 C = VgV p
a= cm, b = cm
3 4
( )
2
fc = 1.875 × 109 Hz C2 3 × 108
Vp = = = 4.5 × 108 m/s
Vg 2 × 108
Hence, the correct option is (D).
Chapter 4  Waveguide  |  9.59

3. The cutoff frequency of TE01 mode of an air filled rec- (C) 6.55 GHz ≤ f ≤ 13.1 GHz
tangular waveguide having inner dimensions a cm × b (D) 1.64 GHz ≤ f ≤ 10.24 GHz
cm (a > b) is twice that of the dominant TE10 mode. Solution:  Range of operating frequency:
When the waveguide is operated at a frequency which
is 25% higher than the cutoff frequency of the domi- 11..25 FCTE10 ≤ f ≤ 0.95 FCTE 01
25 F CTE 10 ≤ f ≤ 0.95 FCTE 01
nant mode, the guide wavelength is found to be 4 cm.
The value of b (in cm, correct to two decimal places) is F = cc = 33 × 1010
× 1010
= 66..55
55 GHz
GHz
FCTE10 = 2a = 2 × 2.29 =
CTE 10

______  [2018] 2a 2 × 2.29


C
C = 13.1 GHz
Solution:  F 20 =
CTE 20 = a = 13.1 GHz
FCTE
a
If Kc is the cut off phase cause and c is speed of light. ∴ GHz ≤
Then the frequency can be expressed as ∴ 88..1875
1875 GHz ≤F F≤≤ 12
12..445 GHz
445 GHz
KCTE01 = 2 KCTE10 Hence, the correct option is (B).
5. Consider an air filled rectangular waveguide with
π 2π   ∵ K
= c dimensions a = 2.286 cm and b = 1.016 cm. At 10 GHz
b a operating frequency, the value of the propagation con-
a stant (per meter) of the corresponding propagating
⇒ b=
2 mode is __________. [2016]
Solution:  The two dimensions of waveguide are a =
λ
lg = 2.286 cm and b = 1.016 cm.
2
⎛ f ⎞
Frequency f = 10GHz
1− ⎜ c ⎟
⎜ f ⎟
⎝ ⎠  So, λ= 0.03 m
(3 × 1010 / f ) Guide propagation constant,
4 =
2
⎛ 1 ⎞ γ g2 = γ 2 + kc2
1− ⎜ ⎟ 
⎝ 1.25 ⎠ 
Where, γ = jω μ ∈ , intrinsic propagation constant
3 × 1010 2π
f = = 12.5 GHz kc = , cut off phase constant
2.4 λc
1.25 fc = 12.5 GHz ⎛ 2 2 ⎞
⎜ λ c = ⎛ mπ ⎞ + ⎛ nπ ⎞ . c ⎟
⎜ ⎜ ⎟ ⎜ ⎟
⎝ a ⎠ ⎝ b ⎠ 2⎟
⇒ fc = 10 GHz (TE10) ⎝ ⎠
So, λc is obtained from above expression
C
= 10 × 109 ⇒ γ g = γ 2 + kc2
2a 
3 × 1010 =
157 m–1
= 1010
2 a Hence, the correct Answer is (57 m–1).
a = 1.5 cm
6. Consider an air filled rectangular waveguide with
a dimensions a = 2.286 and b = 1.016 cm. The increasing
b = = 0.75 cm
2 order of the cut off frequencies for different modes is
Hence, the correct answer is 0.7 to 0.8. (A) TE01 < TE10 < TE11 < TE20
(B) TE20 < TE11 < TE10 < TE01
4. Standard air-filled rectangular waveguides of dimen-
(C) TE10 < TE20 < TE01 < TE11
sions a = 2.29 cm and b = 1.02 cm are designed for (D) TE10 ,< TE11 < TE20 < TE01
radar applications. It is desired that these waveguides
operate only in the dominant TE10 mode with the oper-
Solution:  Cut off frequency, can be expressed as
ating frequency at least 25% above the cutoff frequency kc c
fc =
of the TE10 mode but not higher than 95% of the next 2π
higher cutoff frequency. The range of the allowable
Where, kc = cut off phase constant
operating frequency f is  [2017]
(A) 8.19 GHz ≤ f ≤ 13.1 GHz c = speed of light
(B) 8.19 GHz ≤ f ≤ 12.45 GHz
9.60 | Electromagnetic Theory

π π
TE10 → kc = = (m = 1, n = 0))
a 2 . 286 3 × 1010
2π π fC = 6 GHz =
Solution:  = TE10
TE20 → kc = = (m = 2, n = 0) 2a
a 1.143 ⇒ a = 2.5cm
π π For the same waveguide fC for TM11 is 15 GHz
TE01 → kc = = (m = 0, n = 1)
b 1.016
2ab 3 × 1010
2 2 λC = = =2
⎛π⎞ ⎛π⎞ π a2 + b2 15 × 109
TE11 → kc = ⎜ ⎟ + ⎜ ⎟ = (m = n = 1) 
⎝ a⎠ ⎝ b⎠ 0.862

b = 1.09 cm
∴ fcTE10 < fcTE20 < fcTE01 < fcTE11
Hence, the correct option is (C). 3 × 1010
then cut-off frequency of TE01 =
λC

7. The longitudinal component of the magnetic field λC = 2b


inside an air-filled rectangular waveguide made of a
perfect electric conductor is given by the following =
2.18cm
expression
3 × 1010
Hz(x, y, z, t) = 0.1 cos(25 πx) cos(30.3 πy) cos(12 π × fC of TE 01 = = 13.7 GHz
2.18
109t – βz) (A/m)
Hence, the correct Answer is (13.5 to 13.8).
The cross-sectional dimensions of the waveguide are
given as a = 0.08m and b = 0.033 m. the mode of propa-
9. For a rectangular waveguide of internal dimensions a
gation inside the waveguide is [2015]
(A) TM12 (B) TM21 × b (a > b), the cut-off frequency for the TE11 mode is
(C) TE21 (D) TE1 the arithmetic mean of the cut-off frequencies for TE10
mode and TE20 mode. If a = 5 cm, the value of b (in
Solution:  General magnetic field expression is given as
cm) is_____. [2014]
Hz(x, y, z, t)
Solution: (2)
⎛ mπ ⎞ ⎛ nπ ⎞
= Hocos ⎜ x cos ⎜ y cos(ωt – βz)A/m
⎝ a ⎟⎠ ⎝ b ⎟⎠ f c TE11 + fc TE
20
Given f c TE11 =  (1)
So by comparing given equation with generalized 2
V
equation f c TE10 =
mπ 2 a
= 25π
a  V
f c TE20 =
⇒ m = 25 × 0.08 = 2 a

= 30.3π V 1 1
2 2
b  f c TE11 = +
n = 30.3 × 0.033 2  a   b 

=
0.99 h 1 Put all above values in equation (1)
V V
Hz field is present only for transverse electric field only. 2 2 +
V 1 1 2a a
+ =
2  a   b 
So mode is TE 21
2
Hence, the correct option is (C).
8. An air-filled rectangular waveguide of internal dimen- a = 5 cm

sions a cm × b cm (a > b) has a cutoff frequency of
6 GHz for the dominant TE10 mode. For the same 2a
b=
waveguide, if the cutoff frequency of the TM11 mode is 5
15 GHz, the cutoff frequency of the TE01 mode in GHz
b = 2 cm
is _______. [2015]
Chapter 4  Waveguide  |  9.61

10. The magnetic field along the propagation direction c 3 ×108


inside a rectangular waveguide with the cross-section fc = = = 10 GHz
shown in figure is Hz = 3 cos (2.094 × 102 × ) cos (2.618 λc 3 ×10 −12

× 102 y) cos (6.283 × 1010 t-z) η0
ηTE =
2
 f 
1−  c 
 f 
η0 = 377 Ω, f = 30 GHz
377
ηTE20 =
2
The phase velocity vp of the wave inside the waveguide  10 
1−  
satisfies  30 

(a) v p> c
(b) vp = c η = 400Ω
TE20
(c) 0 < vp < c
(d) vp = 0 [2012] Hence, the correct option is (c)

Solution: (a) 1 3. The E field in a rectangular waveguide of inner dimen-
sions a × b is given by
c
Vp =  wm  p   2p r 
Since, 2 E = 2   H 0 sin  sin (wt − b z ) yˆ
 f 
1−  c  h  a   a 

 f 
where H0 is a constant, and a and b are thedimensions
So, Vp > C always along the x-axis and the y-axisrespectively The mode of
propagation inthe waveguide is
Hence, the correct option is (a)
(a) TE20 (b) TM11
11. A rectangular waveguide of internal dimensions (a = 4 (c) TM20 (d) TE10 [2007]
cm and b = 3 cm) is to be operated in TE11 mode. The
Solution: (a)
minimum operating frequency is
(a) 6.25 GHz (b) 6.0 GHz  wm  p   2p x 
(c) 5.0 GHz (d) 3.75 GHz [2008] E = 2  a  H 0 sin  a  sin ( wt − b z ) yˆ  (1)
h    
Solution: (a) 
E y in general is,
a × b = 4 cm × 3 cm
 − x  h p   mp   np  j ( wt − xz )
c 1 1
2 2 E = 2 c  sin   x. cos   ye (2)
For TE11 mode, f c = + h  b   a   b 
2  a   b 
Compare equation (1) and (2)
2 2 Here, m = 2 and n = 0
3 × 108  100   100 
fc =  4  + 3  So, mode is TE20
2    
Hence, the correct option is (a)
fc = 6.25 GHz 14. A rectangular waveguide having TF10 mode as domi-
Hence, the correct option is (a) nant mode is having a cutoff frequency of 18-GHz for
1 2. An air-filled rectangular waveguide has inner dimen- the TE30 mode. The inner broad-wall dimension of the
sions of 3 cm × 2 cm. The wave impedance of the TE20 rectangular waveguide is
mode of propagation in the waveguide at a frequency of (a) 5/3 cms (b) 5 cms
30 GHz is (free space impedance η0= 377 W) (c) 5/2 cms (d) 10 cms [2006]
(a) 308 W (b) 355W Solution: (c)
(c) 400 W (d) 461 W [2007] 2 2
c m n
Solution: (c) Fc = + For TEmn
2  a   b 
za
For TE20, λc = m = a cm
For TEmo f c =
a = 3 cm 2a
λc = 3 cm
9.62 | Electromagnetic Theory

c 17. A rectangular waveguide has dimensions 1 cm ×0.5


For TE10, f c = cm. Its cut-off frequency is
2a
(a) 5 GHz (b) 10 GHz
C 2a (c) 15 GHz (d) 12 GHz [2000]
λ Cm = =

Fc m
Solution:(c)
λC1 = λC3 Cut-off frequency for TE10 [dominant mode] is,

c
c 3 ×108 1 fc =
λC1 = = 9
= 2a
Fc 18 ×10 60

a = 1 cm
2a 1
λC3 = = 3 ×108
3 60 fc =
2 ×10 −2
1
a= m fc = 15 GHz
40
Hence, the correct option is (c).
For TE10, a = 2.5 cm
Hence, the correct option is (c). 18. Choose the correct statements
15. In a microwave test bench, why is the microwave signal For a wave propagating in an air filled rectangular wave
amplitude modulated at 1 kHz? guide
(a) To increase the sensitivity of measurement (a) Guided wavelength is never less than the free space
(b) To transmit the signal to a far-off place wavelength.
(c) To study amplitude modulation (b) Wave impedance is never less than the free space
(d) Because crystal detector fails at microwave fre- impedance.
quencies [2006] (c) Phase velocity is never less than the free space ve-
Solution:(d) locity.
Since crystal detector fails at microwave frequencies, (d) TEM mode is possible if the dimensions of the
microwave signal amplitude modulated at 1 KHz. wave guide are properly chosen. [1990]
Hence, the correct option is (d) Solution: (a) and (b)
16. A rectangular metal wave guide filled with a dielectric λ
material of relative permittivity ∈r = 4 has the inside (i) λg [guided wavelength] =
2
dimensions 3.0 cm x 1.2 cm. The cut-off frequency for  f 
1−  c 
the dominant mode is lg>λ  f 
(a) 2.5 GHz (b) 5.0 GHz
2
(c) 10.0 GHz (d) 12.5 GHz [2003]  f 
(ii) ηTM [Impedance for TM mode] = η0 1−  c 
Solution:(a)  f 
where, η0 = 120π
2 2
v m n η + m<η0
fc =   + 
2 a b η0

2
V ηTE [TE mode] =  f 
fc = 1−  c 
2a  f 
c 3 × 108 ηTE> η0
V= = (iii) 
tr 4 c
Vp =
2
For dominant node TE10, m = 1, n = 0  f 
  Vp>c 1−  c 
V = 1.5 × 108 m/s  f 
a = 3 cm
Hence, the correct option is (a and b)
1.5 × 108 1 9. For a normal mode EM wave propagating in a hollow
fc = = 2.5 GHz
2×3 rectangular wave guide
Hence, the correct option is (a).
Chapter 4  Waveguide  |  9.63

(a) the phase velocity is greater than the group velocity (a)
(b) the phase velocity is greater than velocity of light
in free space
(c) the phase velocity is less than the velocity of light
in free space
(d) the phase velocity may be either greater than or less
than the group velocity [1988]
Solution: (a) and (b)
c
Phase velocity, V p = (b)
2
 f 
1−  c 
 f 
2
 f 
Group velocity, Vg = c 1 −  c 
 f 
For normal mode, f > fc (c)
So, Vp > c and Vg < c
Vp > c > Vg
20. The cut off frequency of a waveguide depends upon
(a) The dimensions of waveguide
(b) The dielectric property of the medium in the wave-
guide
(c) The characteristic imoedance of the waveguide
(d) The transverse and axial components of the fields
 [1987] (d)
Solution:  (a) adn (b)
2 2
Vo  mπ   nπ 
fc =  a  +  b 
2π    

c
where, vo =
µr ∈r

2 2 Solution: (d)
c  mπ   nπ 
fc =  a  +  b  For the given figures.
µr ∈r × 2    
(a) TE10 (b) TE01
So, fc depends on ‘a’ and ‘b’ (dimensions of waveguide) (c) TE20 (d) TE02
1 Hence, the correct option is (d)
fc ∝
µr ∈r

Five-marks Questions
So, fc depends upon the dielectric property of the
medium in the waveguide.
1. Consider a parallel plate waveguide with plate separa-
Hence, the correct option is (a and b) tion d as shown in figure. The electric and magnetic
21. Which one of the following does represent the electric fields for the TEM mode are given by
field lines for the TE02 mode in the cross-section of a E0 - jkz + jwt
hollow rectangular metallic waveguide? E x = E0 e- jkz + jwt , H y = e
h
9.64 | Electromagnetic Theory

Where k = T < one. (a)  Calculate the cut-off wavelength and the guide
(a) Determine the surface charge densities ρs on the wavelength for this mode.
plates at x = 0 and x = d.  (b) What are the other (TE or TM) modes that can
(b) Determine the surface current densities J s on the propagate through the waveguide?
same plates. 
(c) If a = b = 2.29 cm, what are the modes which can
(c) Prove that ρs and J s satisfy the current continuity propagate through the waveguide? [2001]
condition. Solution:  a = 2.29, b = 1.02 cm, c = 10 GHz
(a) λ = 20 = 4.58 cm
c 3 × 108
l= = = 3 cm
f 10 × 109

V
f c = 0 = 6.55 GHz
lC

l
Solution: Given that wave is propagating in +2 direc- l = = 3.97 cm,
2
tion.  f 
1−  c 
E x = E0 e + j (wt − kz )  f 

E0 + j (wt − kz ) V0
Hy = e (b) for TE10, f c1 = = 6.55 GHz
h 2a
V0
 P PS for TE01, f c2 = = 14.7 GHz,
E = S ax or E x =
(a) 2b
E E fc2 > f so only TE10 mode can propagate through the
given rectangular waveguide.
PS =∈ E x =∈ E0 cos (ωt − kz )
  (c) a = b = 2.29 cm
(b) J S = h × H = for surface x = 0, f = 10 GHz
   E
J S = ax × H = H y aˆ z = 0 cos(wt − kz )aˆ2 V0 3 × 1010
h for TE10, f c10 = = = 6.55 GHz
2a 2 × 2.29
k = hwE V0 3 × 108
for TE01 f c01 = = = 6.55 GHz
1 wE 2b 2 × 2.29
⇒ = for TE11 or TM11
h K
 E 
J S = w ∈ 0 cos(wt − kz )a2 V0 2 2 V
K fC11 = 1 + 1 = 0 = 9.226 GHz
2a 2a

 E
at, x = d, J S = − 0 cos(wt − kz )a2 for TE20 and TE02
h
V0 3 × 1010
(c) From current continuity egn. for time varying fields fC20 = fC02 = = = 13.1 GHz
given by
a 2.29
 −∂ d −∂ So, only TE10, TE01, TE11 and TM11 are the possible
∇.J = P ⇒ JSJ = P. mode that propagates through the rectangle wave guide.
∂t dz ∂t
3. A 100 m section of an air-filled rectangular waveguide
d  ∈ w E0 
cos(wt − kz )  = w ∈ E0 sin(wt − kz ) operating in the TE10 mode has a cross-sectional dimen-
dz  k  sion of 1.071 cm x 0.5 cm. Two pulsed carriers of 21

−∂ ∂ GHz and 28 GHz are simultaneously launched at one
⇒ P = (∈ E0 cos(wt − kz ) end of the waveguide section. What is the time delay
∂t ∂t
difference between the two pulses at the other end of
= w ∈ E0 sin(wt − kz ),
the waveguide? [1999]
d −∂
So, dz JSz = ∂t P . Solution: Time delay = T = b z
Thus current continuity equation verified. w
2. A rectangular hollow metal waveguide has dimensions V0
fc = = 14 GHz
a = 2.29 cm and b = 1.02 cm. Microwave power at 10 2a
GHz is transmitted through the waveguide in the TE10
mode.
Chapter 4  Waveguide  |  9.65

2
 f 
b = b 1 −  c  = 2p m E f 2 − fC2
 f 
for f = f1 = 21 GHz
b = 2p me [(21) 2 − (14) 2 ] × 1018
= 2p me 15.65 × 109

b 13.65 × 1011
T1 = × 100 = 2p me ×
w 2p × 21× 109
= 0.745 me × 100

[1998]
for f = f2 = 28 GHz =
Solution:  Given, a = 5 cm
2 2 18
2p m E [(28) − (14) ] × 10 TE10, f = 30 GHz
b 2 = 2p m E 24.2 × 109 If medium – 1

V0
fc = = 3 GHz
T = × 100 = 0.866 F × 100 2a
h0 120p
Time delay difference, h1 = 2
= 1/ 2
= 378.89Ω
 fc    3 2 
Td = T2 − T1 = m E (0.866 − 0.745) × 100 1 −  1 −   
 f   30 
   
1
= × 0.121× 100 = 0.04 m sec Medium – 2, μ = μ0, E = 4E0
˙× 8
4. A rectangular waveguide with inner dimensions 6 cm E 3 × 108
VP = = = 1.5 × 108 m/sec
x3 has been designed for a single mode operation. Find Er 4

the possible frequency range of operation such that the
lowest frequency is 5% above the cut-off and the high- V0
fc = = 1.5 GHz
est frequency is 5% below the cutoff of the next higher
2a
mode. [1998]
m 120p
Solution:  Given, b = 3 cm, a = 6 cm, a = 2b, h0 = = = 60p
E 4
Lowest TE10

h0
V0 h2 = = 188.73Ω,
f c1 = = 2.5 GHz 2
2a  f 

1−  c 
Next higher mode TE01 or TE20

 f 
flowest = 1.05 f c1 = 2.625 GHz, h −h 188.73 − 378.89
Reflection coeff. Γ = 2 1 =
V0 h 2 + h 1 188.73 + 378..89
f c=
2
= 5 GHz |Γ| = 0.335
a
1+ | Γ |
fhighest = 0.95 f c2 = 4.75 GHz, VSWR = =2
1− | Γ |
5. The region between a pair of parallel perfectly conduct-
ing planes of infinite extent in the y and z direction is 6. In an air-filled rectangular waveguide, the vector elec-
partially filled with a dielectric as shown in figure. A 30 tric field is given by
GHz TE10 wave is incident on the air dielectric interface    40π  
as shown. Find VSWR at the interface. E = cos( 2π y ) exp - j   z + jωt  i xV / m
  3  
Find vector magnetic field and the phase velocity of the
wave inside the waveguide. [1996]
9.66 | Electromagnetic Theory

Solution: λC = 2a = 3.2 2, ⇒ a = 2.26,


iˆx
   40p   Since ba = 2b
E = cos(2 xy ) exp  − j   2 + jwt 
  3   V/m V
f c = 0 m 2 + ( 4 n) 2
2a
 40π
β= V0
3 For TE10 and TE20, =
fc = 13.257 GHz
9
ωµ ω × 4π × 10 −7
η= = = 3ω × 10 −8 8. A rectangular hollow metal wave guide of internal
β 40π / 3 cross-section of 7.366 cm × 3.556 cm carries a 3 GHz

signal in the TE10-mode. Calculate the maximum power
Ex 1    40p  
Hy = = cos(2 xy )e − J   z + jwt  handling capability of the waveguide assuming the
h 3w × 10 −8    3   maximum permissible electric field inside the wave-
 guide to be 30 kV/vm. [1994]
H = H x jy
Solution:
7. A rectangular hollow metal waveguide is required to
be so designed to propagate a 9375 MHz signal in its V 3 × 108 × 10 2
fc = = = 2.036 Ghz
TE10-mode that the guide-wavelength equals the cut-off 2a 2 × 7.366
wavelength. Calculate the value of 'a' (breadth or the
wider dimension of the waveguide). Take b = a/2. Also, 2.036
= fc = 0.679
calculate the cut-off frequency of the next higher order 3
mode. [1995] 2
 fc 
Solution:  For rectangular waveguide.   = 0.46,
TE10 mode. f = 9375 MHz  f 
b = 9/2 η0 120π
η= = = 513.42
For, follow waveguide,  f 
2 1 − 0.46
v0 = 3 × 108 m/sec 1−  c 
 f 
V0
λ0 = = 3.2 cm 2
f Emax
Pmax d = = 8.766 × 109 w/m 2

guide wavelength, λ = λC = cutoff wavelength
Pmax = Pmaxd X Area = 23 mw
λλc λC2 λC
λ0 = = = = 3.2 2 .
λ 2
+ λC2 2λC 2

Chapter 5
Antenna
2. For an antenna radiating in free space, the electric field
One-mark Questions at a distance of 1 km is found to be 12mV/m. Given
that intrinsic impedance of the free space is 120 pW, the
1. Radiation resistance of a small dipole current element magnitude of average power density due to this antenna
of length  at a frequency of 3 GHZ is 3 ohms. If the at a distance of 2 km from the antenna (in nW/m2 )
length is changed by 1%, then the percentage change is____. [2014]
in the radiation resistance, rounded off to two decimal Solution: (47.7)
places, is _____ %. [2019]
E at a distance of 1 km = 12 mv/m
Solution:
E at distance of 2 km, = 6 mv/m
Radiation resistance of a small de’pole is given by
1 E2
Pavg =
l
2
2 η
Rr = 80Π 2  
 l 1 (6 × 10 −3 ) 2
= ×
length is changed by 1% 2 377
= 47.7 nw/m2
∴ l2 = l1 + 1% l1
3. Match Column-A with Column-B.
= l1 + 0.01 l1
Column-A
= l1 (1 + 0.01) 1. Point electromagnetic source
l2 = 1.01 l1 2. Dish antenna
3. Yagi-Uda antenna
2 2
l  l  Column-B
Rr = 80Π 2  1  ; Rr1 = 80Π 2  2  P. Highly directional Q. End fire
 l  l
R. Isotropic
Taking ratio P Q R
2 (a) 1 2 3
2l 
80Π  2  2 2 (b) 2 3 1
Rr2  l l   1.01 l 1 
= 2
= 2 =  (c) 2 1 3
Rr1 2 l   l1   l1  (d) 3 2 1 [2014]
80Π  1 
 l Solution: (b)
= 1.0201 1. Point electromagnetic source.

R = 1.0201 Rr1
r2
Percentage change in radiation resistance:
Rr2 − Rr1
= × 100
Rr1
1.0201 Rr1 − 1 Rr1
= × 100
Rr1

= 0.0201 × 100 = 2.01%


9.68 | Electromagnetic Theory

2. Dish antenna HPBW = 90°


Hence, the correct option is (c).
6. A transmission line is feeding 1 Watt of power to a horn
antenna having a gain of 10 dB. The antenna is matched
to the transmission line. The total power radiated
(a) 10 Watts (b) 1 Watt
(c) 0.1 Watt (d) 0.01 Watt [2006]
Solution: (b)
Gain of amplifier = 10 dB
But gain of antenna = Directive gain so, radiated power
will be 1 watt
Hence, the correct option is (b).
7. Consider a lossless antenna with a directive gain of +
So, Ans → P – 2; Q – 3; R – 1 6dB. If 1 mW of power is fed to it the total power radi-
ated by the antenna will be
Hence, the correct option is (b).
(a) 4 mW (b) 1 mW
4. The radiation pattern of an antenna in spherical co- (c) 7 mW (d) 1/4 mW [2004]
ordinates is given by
Solution: (b)
F(θ) = cos4θ, 0 ≤ θ ≤ π/2
For a lossless antenna, its efficiency is 100%. So, total
The directivity of the antenna is power radiated will be 1 mw
(a) 10 dB (b) 12.6 dB
Hence, the correct option is (b).
(c) 11.5 dB (d) 18 dB [2012]
8. The line-of-sight communication requires the transmit
Solution: (a)
and receive antennas to face each other. If the transmit
4π U max antenna is vertically polarized, for best reception the
D=
π rad receiver antenna should be

(a) horizontally polarized
f (θ ) = cos 4 θ , 0 ≤ π/2 (b) vertically polarized
2π π / 2 (c) at 45° with respect to horizontal polarization
π rad = ∫∫ f (θ ) sin θ dφ (d) at 45° with respect to vertical polarization [2002]
0 0 Solution: (b)
2π π / 2 For best reception receiver antenna must have same
∫ ∫ cos
4
π rad = θ sin θ dφ dθ polarization as transmit antenna
0 0 Hence, the correct option is (b).
2π 9. The frequency range for satellite communication is
π rad =
5 (a) 1 kHz to 100 kHz

(b) 100 kHz to 10 kHz
4π U max (c) 10 MHz to 30 MHz
D= = 10U max
 2π  (d) 1 GHz to 30 GHz [2000]
 5 
  Solution: (d)
Frequency range for satellite communication is, (1GHz
D = 10[ f (θ )]max = 10 to 30 GHz)
D(dB) = 10 log10(10) Hence, the correct option is (d).
= 10 dB 10. If the diameter of a λ/2 dipole antenna is increased
Hence, the correct option is (a). λ λ
5. For a Hertz dipole antenna, the Half Power Beam Width from to , then its
100 50
(HPBW) in the E-plane is
(a) bandwidth increases
(a) 360° (b) 180°
(b) bandwidth decreases
(c) 90° (d) 45° [2008]
(c) gain increases
Solution: (c) (d) gain decreases [2000]
For a Hertz dipole antenna,
Chapter 5  Antenna  |  9.69

Solution: (b) Pr = 2 × 10−6w


1 = 1.061 × 10−6
Bandwidth ∝
diameter 2 ×10 −6
Ae =
Hence, the correct option is (b).
1.061×10 −6


1 1. The vector
 × H = Jthe
in C +far
J Dfield of an antenna satisfies Ae = 1.885 m2
(a) ∇ ⋅ H = 0 and ∇ × H = 0 Hence, the correct option is (c).
 
(b) ∇ ⋅ H ≠ 0 and ∇ × H ≠ 0 1 4. The far field of an antenna varies with distance r as
 
(c) ∇ ⋅ H = 0 and ∇ × H ≠ 0 (a) 1/r (b) 1/r2
  (c) 1/r3 (d) 1/ r  [1998]
(d) ∇ ⋅ H = 0 and ∇ × H = 0  [1998]
Solution: (a)
Solution: (c)
1
We know, E∝
 r
∇ × H = JC + J D
Hence, the correct option is (a).
JC = Conventional current density
1 5. An antenna, when radiating, has a highly directional
Jd = Displacement current density
 radiation pattern. When the antenna is receiving, its
So, ∇ × H ≠ 0 radiation pattern

∇. B = 0 (a) is more directive
 (b) is less directive
So, ∇.H = 0 (c) is the same
Hence, the correct option is (c). (d) exhibits no directivity at all [1995]
1 2. The radiation resistance of a circular loop of one turn is Solution: (c)
0.01 W. The radiation resistance of five turns of such a An antenna is a reciprocal device, whose characteris-
loop will be tics remain same when it is transmitting or receiving.
(a) 0.002 W (b) 0.01 W
(c) 0.05 W (d) 0.25 W [1998] Hence, the correct option is (c).
Solution: (d) 16. For a dipole antenna.
2
(a) the radiation intensity is maximum along the nor-
 ndl  mal to the dipole axis
Rr = 80π  
 λ  (b) the current distribution along its length is uniform
irrespective of the length .
Rr ∝ n2 (c) the effective length equals its physical length

(d) the input impedance is independent of the location
Rr1 η12 of the feed-point [1994]
=
Rr2 η22 Solution: (a)

0.01 1 The radiation intensity is maximum along the normal
= to the dipole axis.
Rr2 (5) 2

Rr2 = 0.25Ω Main lobe

Hence, the correct option is (d).
1 3. An antenna in free space receives 2 μW of power when
the incident electric field is 20 m V/m rms. The effec- dipole axis
tive aperture of the antenna is
(a) 0.005 m2 (b) 0.05 m2
(c) 1.885 m
2
(d) 3.77 m2 [1998] λ/2
Solution: (c)
Pr
Ae =
Pd

Hence, the correct option is (a).
9.70 | Electromagnetic Theory

respectively. Assuming that the input power in the loss-


Two-marks Questions less transmission line connected to the antenna is 2 W
1. A half wavelength dipole is kept in the x-y plane and and that the antennas are perfectly aligned and polari-
oriented along 45o from the x-axis. Determine the zation matched, the power (in mw) delivered to the load
direction of null in the radiation for q ≤f ≤ p. Here the the receiver is _____. [2016]
angle f(0 ≤ f ≤ 2p) is measured from the z-axis and Solution: 
the angle is measured from the x-axis in the x-y plane.
R = 200 λ Dot = 63.1
 [2017]
(A) q = 90°, f = 45° (B) q = 45°, f = 90° Γ t = 0.15 D0r = 158.5
(C) q = 90°, f = 135° (D) q = 45°, f = 135° Γ r = 0.18
Solution:  Assuming the dipole is centrefed, the max
Pi = 2 W ∴ ⎡⎣ D ( dB ) = 10 log10 D ⎤⎦
radiation goes at right angles to its axis and no radiation
(null) along its axis.
Pt = Pi ⎡1 − Γ t ⎤ = 1.955 W
2

\ for 0 ≤ q ≤ p, the null exists for f =45o and f = 90o ⎣ ⎦


Hence, the correct option is (A). Friis transmission formula
2
2. An antenna pointing in a certain direction has a noise ⎛ λ ⎞
Pr = Pt ⎜ D D
temperature of 50 K. The ambient temperature is 290 K. ⎝ 4π R ⎟⎠ Ot Or
The antenna is connected to a pre amplifier that has a
noise figure of 2 dB and an available gain of 40 dB over =
3.1 mW
an effective band width of 12 MHz. The effective input
noise temperature Te for the amplifier and the noise Power delivered to load,
power Pa0 at the output of the preamplifier, respectively, PL = Pr ⎡⎣1 − Γ 2r ⎤⎦ ≈ 3 mW
are [2016]
(A) Te = 169.36 K and Pa0 = 3.73 × 10–10 W Hence, the correct Answer is (3 mW).
(B) Te = 170.8 K and Pa0 = 4.56 × 10–10 W 4. The far zone power density radiated by a helical antenna
(C) Te = 182.5 K and Pa0 = 3.85 × 10–10 W is approximated as
(D) Te = 160.62 K and Pa0 = 4.6 × 10–10 W
Solution:  F = 2dB = 102   1
(Bandwidth) BW=12 MHz, and Gain is 40 dB = 104 Wrad = Waverage = ≈ âr C0 2 cos4θ.
r
Effective input noise temperature, The radiated power density is symmetrical with respect
Te = To [F – 1] to f and exists only in the upper hemisphere; 0 ≤ θ ≤
π
; 0 ≤ f ≤ 2π; C0, is a constant. The power radiated by
Where F is noise figure, To = ambient temp = 290 K 2
the antenna (in watts) and the maximum directivity of
=
290 [1.58 – 1] the antenna, respectively are [2016]
=
290 × 0.58 (A) 1.5 C0, 10 dB
(B) 1.256 C0, 10dB
=
169.36° K (C) 1.256 C0, 12 dB
(D) 1.5 C0, 12 dB
Output noise power,
Solution:  The radiated power can be expressed as
Pao = KATeB
prad = ∫∫ w .d s
rad 
Pao = (50 + 169.36) × 1.38 × 10 –23
× 10 × 12 × 10
4 6
2π π / 2
1
=
3.63 × 10–10 W c0 ∫ ∫ ar
= 2
cos 4 θ .ar r 2 sin θ dθ dϕ
ϕ =0 θ =0 r

Hence, the correct option is (A).
=
1.256 Co.
3. Two lossless X band horn antennas are separated by
a distance of 200 λ The amplitude reflection coeffi- The radiated power density is also symmetrical with
cients at the terminals of the transmitting and receiving respect to f and present only in upper hemis so
antennas are 0.15 and 0.18, respectively. The maximum
U max
directivities of the transmitting and receiving anten- D0 = 4π
nas (over the isotropic antenna) are 18 dB and 22 dB Prad

Chapter 5  Antenna  |  9.71

C0 2π π / 2
4π ⎡∵ U = r 2W , U max = C0 ⎤ 10000

1.256C0 ⎣ ⎦

= ∫∫
0 0
2η r 2
.r 2 sin 3 θ dθ dφ
Do = 10
π /2
10000 3
Do(dB) = 10 dB. = 2π
0
∫ 2η
sin θ dθ
Hence, the correct option is (B).
5. A radar operating at 5 GHz uses a common antenna = 55.5
for transmission and reception. The antenna has a gain 7. At 20 GHz, the gain of a parabolic dish antenna of 1
of 150 and is aligned for maximum directional radia- meter diameter and 70% efficiency is
tion and reception to a target 1 km away having radar (a) 15 dB (b) 25dB
cross section of 3 m2. If it transmits 100 kW, then the (c) 35 dB (d) 45 dB [2008]
received power (in µW) is__________. [2016] Solution: (d)
Solution:  Gain of antenna G = 150 f = 20 GHz
Frequency of radar f = 5 GHz; Pt = 100 kW 3 × 108
Distance of target r = 1 km; λ= = 1.5 cm
20 × 109
Radar cross sectional area A = 3 m2
η = 70%, D = 1 m
λ = c/f = (3 × 108)/(5 × 109) 2
D
=
0.06 m G = ηπ 2  
λ
Received power, 2
2 1× 20 ×109
pt G λ S
2 2
G = 0.7π
pr = 3 ×108
(4π ) 3
r 4
G = 30705.43
=
(100 × 10 × 150 × 0.06 × 3)/(4 × 3.14)3(1000)4
3 2 2

G(dB) = 10 log10 (30705.43)


=
0.012 µW
= 45 dB
6. In spherical coordinates, let aθ , aφ denote unit vectors Hence, the correct option is (d).
along the θ ϕ, directions. λ λ0
100 8. A dipole is kept horizontally at a height of
2 2
E= sin q cos (w − b ) aˆq and above a perfectly conducting infinite ground plane.
r
The
 radiation pattern in the plane of the dipole
0.265 ( E plane) looks approximately as
H= sin q cos (wt − b r )aˆf A / m
r
(a)
represent the electric and magnetic field components of
the EM wave at large distances r from a dipole antenna,
in free space The average power (W) crossing the hemi-
spherical shell located at r = 1 km, 0 ≤ θ ≤ ϕ/2 is____
 [2014]
Solution: (55.5)
E02 (b)
Pavg =

2π π / 2
E02 2
Power radiated, =
0 0
2η ∫∫
.r sin θ dθ dφ
Here, η = 377 Ω
r = 1 km
100
E0 = sin θ
r
Putting this in the integral, we get
9.72 | Electromagnetic Theory

(c) with equal currents but the right most antenna has a
phase shift of +90°. The radiation pattern is given as
(a)

(d)

(b)

[2007]
Solution: (b)
For, (a) →d = λ/4
(b) →d = λ/2
(c) →d = λ
(c)
Hence, the correct option is (b).
9. A mast antenna consisting of a 50 meter long vertical
conductor operates over a perfectly conducting ground
plane. It is base-fed at a frequency of 600 kHz The radi-
ation resistance of the antenna in Ohms is
2p 2 p2
(a) (b)
5 5
2
4p
(c) (d) 20 p2 [2006] (d)
5
Solution: (a)
2
 dl 
Rrad = 80π 2   Ω
λ
2
2  dl 
For conducting ground, Rrad = 40π   Ω
λ
f = 600 kHz [2005]
3 × 10 8 Solution: (a)
λ= = 1.5 cm
20 × 109

λ = 500 m, l = 50 m
40π 2
Rrad = 2
 50 
 500 
 

2π 2
Rrad =
5
Hence, the correct option is (a).
Hence, the correct option is (a).
1 0. Two identical and parallel dipole antennas are kept
apart by a distance of λ/4 in the H-plane. They are fed 1 1. Two identical antennas are placed in the ϕ = p/2 plans as
shown in the figure The elements have equal amplitude
Chapter 5  Antenna  |  9.73

excitation with 180° polarity difference, operating a: (a) 942 m (b) 2070 m
wavelength λ. The correct value of the magnitude of (c) 4978 m (d) 5320 m [2002]
the far-zone resultant electric field strength normalized Solution: (b)
with that of a single element, both computed for ϕ = 0,
P
is Signal strength at distance R, P1 =
4π R12
R = 5000 m
2
P1  λ  P
=  = 1
P 5000  P1/ 2
2 

λ = 5000 2
Extra distance = (5000√2 – 5000) m = 2071 m
Hence, the correct option is (b).
1 3. In a uniform linear array, four isotropic radiating ele-
ments are spaced λ/4apart. The progressive phase shift
between the elements required for forming the main
 2π s  πs  beam at 60° off the end-fire is:
2 cos 
(a)  = 2sin  
(b)
 λ   λ  (a) -pradians (b) -p/2 radians
(c) -p/4 radians (d) -p/8 radians [2001]
πs  πs 
(c) 2 cos   (d) 2sin    [2003] Solution:  (c)
 λ   λ 
Phase difference, ψ = β d cos θ + α
Solution: (d) For an array to be end fire, ψ = 0

ψ = β d sin θ cos φ + δ So, β d cos θ + α = 0
θ = 90°, d = 2 s a = − b d cos q
ϕ = 45°, δ = 180° α = − β d [maximum]
2π So, ψ = β d cos θ − β d
ψ = 2 cos 45° + 180°
λ
  = β d(cos θ − 1)
ψ λ
Normalized array factor = 2 cos d= and θ = 60°
2 4
 π  2π λ
= 2 cos  2 s cos 45° + 90°   ψ = × [cos 60 − 1]
  λ  λ 4

 2π s  π
2 sin   ψ =− radians
 λ 
4

Hence, the correct option is (c).
14. A medium wave radio transmitter operating at a wave-
length of 492 m has a tower antenna of height 124 m.
What is the radiation resistance of the antenna?
(a) 25W (b) 36.5 W
(c) 50W (d) 73W [2001]
Solution: (b)
Here, L = 124 m i.e. nearly about λ/4
492
L~ = 123 m
Hence, the correct option is (d). 4
1 2. A person with a receiver is 5 km away from the trans- So, given antenna is quarter wave monopole antenna,
mitter. What is the distance that this person must move whose Ra = 36.5 Ω
further to detect a 3 dB decrease in signal strength? Hence, the correct option is (b).
9.74 | Electromagnetic Theory

15. For an 8 feet (2.4m) parabolic dish antenna operating 18. A transverse electromagnetic wave with circular polari-
at 4 GHz, the minimum distance required for far field zation is received by a dipole antenna. Due to polariza-
measurement is closest to tion mismatch, the power transfer efficiency from the
(a) 7.5 cm (b) 15 cm wave to the antenna is reduced to about
(c) 15 m (d) 150 m [2000] (a) 50% (b) 35.3%
Solution: (d) (c) 25% (d) 0% [1996]
F = 4 GHz Solution: (d)
The power transfer efficiency from the wave to antenna
3 ×108
λ= 9
is reduced to 0%.
4 ×10
Hence, the correct option is (c).
λ = 7.5 cm 19. A 1 km long microwave link uses two antennas each
l = 2.4 m having 30 dB gain If the power transmitted by one
2l 2 antenna is 1 W at 3 GHz the power received by the other
(distance), d =
λ antenna is approximately
d = 1.536 m 150 cm (a) 98.6 μW (b) 76.8 μW
(c) 63.4 μW (d) 55.2 μW [1996]
Hence, the correct option is (d). Solution: (c)
1 6. A transmitting antenna radiates 251 W isotropically. A PG
receiving antenna, located 100 m away from the trans- Pd (at d = 1 km) = t t
mitting antenna has an effective aperture of 500 cm2 4π R 2
1
The total power received by the antenna is Pd =
4000π
(a) 10 μW (b) 1 μW
(c) 20 μW (d) 100 μW [1999] Pt = 1 w, Gt = 1000
Solution: (d) R = 1 km = 1000 cm
Intensity [at d = 100 m], Gr λ 2
( Ae ) r =
251 4π
I= w/m 2
4π (100) 2 Gr λ 2 1

Pr = .
Power = I × Ae 4π 4000π
Ae = 500 cm2 Pr = (Ae)rPd
251
power = 2
× 500 ×10 −4 watt Gr = 1000
4π (100)
c 3 × 108
λ= = = 10 cm
P = 100 µ w f 3 × 109

Hence, the correct option is (d). Pr = 63.3µ w
17. A parabolic dish antenna has conical beam 2° wide. Hence, the correct option is (c).
The directivity of the antenna is approximately
(a) 20 dB (b) 30 dB 2 0. Two dissimilar antennas having their maximum direc-
(c) 40 dB (d) 50 dB [1997] tivities equal,
(a) must have their beam widths also equal
Solution: (c) (b) cannot have their beam widths equal because they
41253 are dissimilar antenna
Directivity =
BW (θ ) BW (φ ) (c) may not necessarily have their maximum power
BW (θ ) = BW (f ) =2° gains equal
(d) must have their effective aperture areas (capture ar-
41253 eas) also equal [1992]
= D = 10, 000
( 2) 2 Solution: (a)

D dB = 10 log(10)4 Directivity, D = 4π [Beam width]
Ω2
D = 40 dB So, If D1 = D2
Hence, the correct option is (c). then, Ω1 = Ω2
Chapter 5  Antenna  |  9.75

Therefore, Beam width also equal. Say E0 = 1


Hence, the correct option is (a). ψ  4π cos θ 
21. The beamwidth between firs nulls of a uniform lin-
2 cos
2
= 2 cos 
2  = 2 cos( 2π cos θ )
 
ear array of N equally spaced (element spacing = d),
equally excited antennas, is determine by For θ: 0 to 2π
p p 2p 4p 3p 5p
(a) N alone and not by d |ET| is maximum at θ = 0, , , ,p , , ,
3 2 3 3 2 3
(b) d alone and not by N
(c) the ratio, (N/d) So, total number of lobes are 8.
(d) the product, (Nd) [1992] Hence, the correct option is (d).
Solution: (d) 24. The electric field E and the magnetic field H of a short
BWFN of a uniform linear array of N equally spaced is, dipole antenna satisfy the condition
(a) the r component of E is equal to zero
4 2λ (b) both r and θ components of H are equal to zero
For end fire array, BWFN =
3 Nd (c) the θ component of E dominates the r component
2π in the far-field region
For Broadside array, BWFN =
Nd (d) the θ and ϕ components of H are of the same order
So, we need the product (Nd) to determine BWFN. of magnitude in the near field region [1988]
Hence, the correct option is (d). Solution: (b) and (c)
22. In a board side array of 20 isotropic radiators, equally For short dipole antenna, components of E and H are
spaced at distance of λ/2, the beam width between first Er, Eθ and Hϕ
nulls is
(a) 51.3 degrees (b) 11.46 degrees Hence, the correct option is (b) and (c).
(c) 22.9 degrees (d) 102.6 degrees
 [1991]
Solution: (b) Five-marks Questions
2λ 1. A thin-dipole antenna is (λ/15) long, where X is the
BWFN = radian
ηd operating wavelength. If the dipole has a loss resistance
λ of 1.5 ohm determine its
Here, η = 20 and d = (i) radiation resistance, and
2
2λ 1 (ii) terminal resistance [1987]
BWFN = = radian
20.λ / 2 5 l l
Solution:(i) y= <
180 1 15 10
= × degrees So, the given dipole antenna can be considered as a
    π 5 short antenna, so the current distribution is uniform
BWFN = 11.46 degrees with average equal to lm/2.
2 2
Hence, the correct option is (b). l   I  ( I )2
Pr = 80p 2    m  = m 2 Rx
23. Two isotropic antennas are separated by a distance of l  2 2  ( 2)
two wavelengths. If both the antennas are fed with
2
currents of equal phase and magnitude, the number of 2 l 
lobes in the radiation pattern in the horizontal plane are So, Rx = 20p  
l 
(a) 2 (b) 4
(c) 6 (d) 8 [1990] (ii) Terminal resistance of the Antenna
2
Solution: (d) l 
= R10ss + Rrad = 1.5 + 20p 2  
Phase difference, ψ = α + βd cos θ l 
2. Consider an array of two non-directional radiators with
α = current phase difference = 0
spacing d = 0.5λ. Determine the directions of maxi-
d = 2λ mum radiation when the radiators are excited as shown
2π in figure. Calculate the phase shift required for turning
ψ = . 2λ cos θ = 4π cos θ
∴ λ the direction of the maximum radiation by 90°, keeping
the separation, d, unchanged.
ψ
ET = 2 E0 cos
2
9.76 | Electromagnetic Theory

operating at a frequency of 2 GHz with an aperture effi-


ciency of 64%. If the spacecraft A's receiver requires 1
pW for a 20 dB signal-to-noise ratio, what transmitter
power is required on the spacecraft B to achieve this
signal-to-noise ratio? [1994]
Solution: R = 3000 km, D = 0.85 m
eA = eB = 64%
f = 2 × 109 Hz
c 3 × 108
l= = = 15 cm = 0.15cm
[1993] f 2 × 109

Solution: l2 = kl1 < a 20 dB (SNR) = 10 log10 (SNR)
Given, I 2 = I1 = I ∠0° SNR = 100
So, a = 0 PA = 1PW =10−12 W
k=1 if gain of 2nd Antenna
d = 0.5λ
4p p D 2 p 2 D 2
GB = . = , power density
l2 4 l2
of 2nd Antenna

W = eB .PB .GB , So power transmitted by 2nd Antenna,


2
W4p
× 4Rp R 2 l 2 4WR 2 l 2
PB = =
eB p eB D 2 p eB D 2

So, power received at transmitting Antenna,
p D 2 W = PA × 4
PA = W × w A × , e Ap D 2
for n element with k = 1 4
 y So,
sin h 
ET  2
= 4 R2l 2 4 PA 16 P R 2l 2
E1 y  PB = 2
× 2
= 2 A = 1.5335 W
sin   pD e Ap D p e Ae B D 4
2

4. Two dipoles are so fed and oriented in free space that
2p l they produce the following electromagnetic waves:
y = b d cos f = a = . cos f + 0 = p cos f
l 2 Ex = 10ei(wt - zop/3)volts/meter
for n = 2 Ey = j10e i(wt - zop/3) volts/meter
ET y  p  (a) Write down the expression for the corresponding
= 2 cos   = 2 cos  cos f  magnetic field strength vector.
2
E 2 2  (b) Calculate the frequency of the wave.
for maximum radiation, (c) Give the complete description of the polarization
of the wave. [1995]
p
cos f = 0, ⇒ f = ±p /2 Solution: given, E x 10e J (ωt −π / 3 z )
2
for turning the direction of the maximum radiation by E y = J 10e J (wt − 2p / 3) = 10e J (wt − 2p / 3+p / 2)
90°
em wave is travelling in the positive z-direction β = π/3.
l l
y= < for free space, h 0 = 120pΩ
15 10
p p Ey
f ==±0 and
+ �π/2
� , (a) h =− ⇒
2 2 Hx
3. Two spacecrafts are separated by 3000 km. Each has
a paraboloidal reflector antenna of 0.85 m diameter
Chapter 5  Antenna  |  9.77

Ey 1 J (wt − 2p / 3−p l 2) y y
Hx = − =− e 2sin .cos
h 12p ET
= 2 2 = 2cos y
EA y 2
Ex E y 10e J (wt − 2p / 3) sin
h = ⇒ Hy = = 2
Hy h 120p  3p a 
H2 = 0, = 2cos  cos f + 
 4 2 (1)
2p 2p
l=
(b) = = 6m ϕ = 30°
b p /3
f = 90 − q = 60°
C 3 × 108
f = = = 50 MHz 3p 1 a p
l 6 from (1) × × =
4 2 2 2
or, α = π/4
(c) let, Z = 0, Ex = 10 cos ωt = E0 cos ωt
Ey = −10 sin ωt = −E0 sin ωt = E0 cos ωt
ET  3p p
So, E = 2 cos  4 cos f + 8 
2 2
A  
So, E0 = 10 = E x + E y , for maximum radiation,
p p
f ==±p/2�
+� ,  3p p
2 2 cos  cos +  = 1
So the polarization is left polarization.  4 q

5. Two isotropic antennas A and B form an array as shown
3p p
in figure. The currents fed to the two antennas are I0∠0 ⇒ cos f + = 0 or ±π
and I0∠ respectively. What should be the value of a so 4 8
that the radiation pattern has a null at e = 30°. Find the fm = −9.6° and −170.4°
direction of the maximum radiation for that value of a
p / 2p
and draw the radiation pattern. (λ. is the wavelength of f = ± y�
+� , Et/Ea
2 2
operation).
0 1.85
7≠ π
or −
8 8
600 p/2 0
900 p/8 1.85
99.6 0 2
1800 0.77
−5π
8

[1996]
Solution:

2p 3l 6. A dipole antenna has a sin 9 radiation pattern, where


y=
y = b d cos f + a ;
,d= the angle 8 is measured from the axis of the dipole. The
l 4
dipole is vertically located above an ideal ground plane
2p 3l 3p
y= × cos f + a = cos f + a (Figure). What should be the height of the dipole, H in
l 4 2 terms of wave length so as to get a null in the radiation
pattern at an angle of 45° from the ground plane? Find
the direction of maximum radiation also.
9.78 | Electromagnetic Theory

for maximum radiation,


ψ = 0,
4p H p
cos f = 0, f =
2
l
7. The average power of an omnidirectional antenna var-
ies as the magnitude of cos(θ), where θ is is the azi-
muthal angle. Calculate the maximum directive gain of
the antenna' and the angles at which it occurs. [1999]
Solution:  Given that,
f (q , f ) = fm | cos f |

total radiated power,
[1997]
Solution: 

Wk f (q , f )d Ω
A dipole antenna having a sin θ radiation pattern can be
considered as an horizontal dipole,

Wr = fm cos q .sin q dq .df

p 2p
f
2 ∫
= m sin 2q .dq
0
∫ df = pf
0
m

Wr fm
favg = =
4p 4
Directive gain of the antenna,
(q ,f )
ga (q ,f ) = f = 4 cos q
favg

ga max = 4, this occur for θ = 0° or 180°
8. Consider a linear array of two half-wave dipoles A and
2p 2p B as shown in figure. The dipoles are λ/4 apart and are
b = , bd = 2H excited in such a way that the current on element B lags
l l
that on element A by 90° in phase.
y  y  (a) Obtain the expression for the radiation pattern for
2sin   cos  2 
ET
= 2    = 2 cos  y  E in the XY plane, i.e., (θ = 90°).
2
EA y    (b) Sketch the radiation pattern obtained in (a).
sin  
2
4p H
y = b d cos f + a = cos f
l
ET
= 2,
EA

p p
y = 0 or f ==±p / �
2+ � ,
2 2
E
T = 0, y = p
EA

for null
4p H p p
y =p = cos f , f ==±450�+ � ,
l 2 2
4p H
cos 45° = p , ⇒ H = l
l 2 2 [2002]

You might also like